You are on page 1of 540

22:20 ,27.6.

2023 �� C : ; 5 B

Downloaded from the site - online testing step

HTTP/1.0 200 OK Cache-Control: no-cache, private Date: Mon, 06 Dec 2021 07:37:45 GMT

TESTING.UKR test databases

Booklets 2005-2018 without repeating questions-test base 2019

1. A 36-year-old patient with severe meningeal syndrome, petechial


skin rashes, chills, body temperature of 39 ° C, inflammatory changes in peripheral blood and
neutrophil pleiocytosis in the cerebrospinal fluid was diagnosed with purulent meningitis. Which of the existing
syndromes in a patient is crucial for making a diagnosis of meningitis?
100% Neutrophilic pleiocytosis
0% Petechial skin rashes
0% Meningeal syndrome
0% Increased body temperature, chills
0% Inflammatory changes in the peripheral blood

2. in a 39-year-old man, 2 days after the calf was cut, a spot appeared on his arm, which
turned into a pustule with a black bottom, painless when touched, with a crown of daughter vesicles on
the periphery. Objectively: the temperature is 39oc, there is a painless swelling on the arm and shoulder. Ps-100 / min,
AT95/60 mmHg, BH-30 / min. What is the most likely diagnosis?
100% Anthrax
0% The plague
0% Tularemia
0% Brucellosis
0% Cold sores

3. construction of an industrial enterprise (hazard class 1A) is planned in the city, the emissions
of which will contain sulfur dioxide and nitrogen dioxide. At what distance from the emission source
of a given enterprise should the concentrations of these ingredients in atmospheric air
correspond to the Maximum Permissible concentration (MPC)?
100% 3000 m
0% 1000 m
0% 500 m
0% 100 m
0% 50 m

4.a 44-year-old man who works in the workshop


of a furniture factory, where furniture is covered with paints and varnish, turned to the general practitioner of the district polyclinic. The
patient's complaints and objective data
indicated acute occupational poisoning. The doctor temporarily released the victim from work,
prescribed
100% treatment andsanitary
District sent anand
"emergency message":
Epidemiological Station
0% To the company where the victim works
0% To the medical and sanitary part of the enterprise
0% To the Chief Medical Officer of the TMO
0% To a specialized clinic confirming the disease

https://translate.yandex.com/en/doc 1/540
22:20 ,27.6.2023 �� C : ; 5 B
Downloaded from the site - online testing step

5. A 39-year-old patient complains of shortness of breath, a moderate cough with a small amount
of sputum, heaviness in the left side of the chest, and an increase in body temperature up to 37.7 ° C.
These symptoms appeared and worsened during the week. Objectively: BH-26 / min. The left
side of the chest lags behind when breathing. Below the angle of the left scapula, the vocal tremor
is sharply weakened, the percussion tone is blunted, and vesicular
respiration is weakened auscultation. What is the preliminary diagnosis?
100% Left-sided exudative pleurisy
0% Left-sided pneumonia
0% Left-sided intercostal neuralgia
0% Bronchiectasis
0% Abscess of the lower lobe of the left lung

6. A 10-month-old girl has an acute respiratory infection with symptoms of toxicosis.


A hemorrhagic rash appeared on the skin of the abdomen, there was twice vomiting of "coffee grounds",
macrohematuria was noted. What laboratory tests should be performed to clarify the causes
of hemorrhagic syndrome?
100% Coagulogram
0% Analysis of feces for hidden blood
0% Blood test for the content of immunoglobulins
0% Urinalysis
0% Sternal puncture for bone marrow examination

7. A 23-year-old female patient went to a women's clinic with complaints of lower abdominal pain that
worsens during menstruation, spotting spotting before and after menstruation.
The disease is associated with an artificial abortion. In the mirrors: there are 5 dark red inclusions on the cervix
. What is the most likely diagnosis?
100% External endometriosis
0% Cervical polyposis
0% Cervical cancer
0% Cervical erosion
0% Cervical dysplasia

8. a 60-year-old man, according to his wife, drank a significant amount of alcohol, fell into a puddle
with his face, and remained lying down. In the study of the corpse, an increase and emphysema of the lungs,
an asphyxiated type of blood filling of the heart,the presence of plankton in the liver and internal organs,
and 2.91 o/oo of ethyl alcohol in the blood. Possible cause of death?
100% Drowning in water
0% Alcohol intoxication
0% Coronary heart disease
0% Poisoning with an unknown poison
0% The cause of death cannot be determined

9. in a 32-year-old woman in labor, manual removal of droppings and examination of the uterus revealed a rupture
of the cervix on the right with a transition to the lower segment. Blood loss reaches 1300 ml and
continues. What are the doctor's tactics?

Downloaded from the site - online testing step

100% Uterine extirpation


https://translate.yandex.com/en/doc 2/540
22:20 ,27.6.2023 �� C : ; 5 B

0% Superpichival uterine amputation


0% Uterine tamponade
0% Tampon with ether in the posterior vault
0% Clamps for Baksheev or Tikinadze

10. a 32-year-old woman complains of a feeling of unreality, a changed body. When


she stands in front of a mirror, she recognizes herself, but her hands, feet, and face don't seem
to belong to her. What is the most likely psychopathological disorder in the patient?
100% Depersonalization
0% Derealization
0% Hallucinations
0% Senestopathies
0% Illusions

11. A 13-year-old girl has been complaining of shortness of breath, swelling in the
lower legs and feet after physical activity for the last two weeks. After a night's sleep, the swelling is significantly
reduced. A clinical examination reveals an enlarged liver, a rough
systolic murmur over the heart area. The urine and blood tests were not changed. What is the most likely
origin of edema in this child?
100% Heart failure
0% Nephrotic syndrome
0% Acute pyelonephritis
0% Angioedema
0% Cirrhosis of the liver

12. A 24-year-old patient with peritonitis symptoms was urgently admitted to the hospital.
I had a criminal abortion 16 hours ago. Objectively: pale, dry tongue. The abdominal wall
does not take part in breathing, it is tense during palpation, and the Shchetkin-Blumberg symptom is positive in all
parts. In the blood: lake.-15 * 109 / l, stick-core 20%. What are the treatment tactics?
100% Urgent laparotomy
0% Conservative treatment, follow-up
0% Diagnostic laparocentesis
0% Diagnostic laparoscopy
0% Performing plasmapheresis

13. A 10-year-old child who was outside in windy and frosty weather developed moderate
pain and tingling in his fingers and toes. When the boy returned home, his parents found
the tips of his fingers and toes whitened, and he lost sensation. Warming of the affected areas was started
, soreness and tingling in the fingers again appeared. The pale color of the skin turned purple,
the tingling disappeared, there was a slight itching and a slight swelling of the fingers. Determine
the degree of frostbite in a child:
100% Frostbite of the first degree
0% Frostbite
0% Frostbite of the second degree

Downloaded from the site - online testing step

0% Frostbite of the third degree


0% Frostbite of the IV degree

https://translate.yandex.com/en/doc 3/540
22:20 ,27.6.2023 �� C : ; 5 B

14.On day 10, a 1-year-old child with bacterial pneumonia


was diagnosed with a subcortical abscess up to 4 cm in diameter with perifocal infiltration in the lower lobe of the right lung
. Choose a method of treating a lung abscess without draining it into
the bronchi:
100% Puncture and catheterization of the abscess cavity
0% Bronchoscopic lavage
0% Radical surgery
0% Conservative treatment
0% Partial bronchial occlusion

15. A 15-year-old girl was diagnosed with chronic cholecystocholangitis 2 years ago. I didn't
follow a diet. The condition has worsened in the last 3 months. There is an increase in body temperature.
Abdominal pain of paroxysmal nature after fatty, spicy food. Worried
about itchy skin. The tongue is covered with a white coating. Stomach is soft, liver + 3 cm, palpation is painful,
positive cystic symptoms. In the blood: lake.- 12•109/l, WSE-20 mm / year. Which drug should
be included in the treatment package?
100% Antibiotics
0% Hepatoprotectors
0% Prokinetics
0% Enzymes
0% Enterosorbents

16. the boy is 3 months old. It is on natural feeding. From the 4th month, the mother will
be forced to be absent for 8 hours a day. Choose the optimal feeding for the period of absence
of the mother:
100% Expressed mother's milk
0% Cow's milk
0% Adapted mix
0% Milk porridge
0% Apple juice

17. A 27-year-old repeat mother arrived at the maternity ward due to a 40


-week pregnancy and the onset of labor. 2 hours ago, the amniotic fluid left.
Fetal position is longitudinal, main presentation. Coolant - 100 cm, VDM-42 cm. Contractions
in 4-5 minutes, 25 seconds each. During internal obstetric examination: the cervix
is smoothed, the opening is 4 cm. There is no fetal bladder. The fetal head is pressed against the entrance to the small
pelvis. What complication occurred during childbirth?
100% Premature discharge of amniotic fluid
0% Primary weakness of labor activity
0% Secondary weakness of labor activity
0% Discoordinated labor activity
0% Clinically narrow pelvis

Downloaded from the site - online testing step

18. according to a 38-year-old patient, he hears voices that sound in his head. His language is controlled by someone,
he sometimes speaks out of his will. Sees events that happen outside the room where
it is located. His thoughts flow by themselves, without his desire, sometimes even two streams
of thoughts. He suspects that he is under the supervision of some scientific organization that is conducting experiments on him
. It can be:

https://translate.yandex.com/en/doc 4/540
22:20 ,27.6.2023 �� C : ; 5 B
100% Kandinsky-Clerambault syndrome
0% Auditory hallucinosis syndrome
0% Psychosensory disorder
0% Paraphrenic syndrome
0% Paranoid syndrome

19. A 50-year-old patient developed acute pain in the area of the central phalangeal joints of 1-2
toes of the right foot. On examination, the skin over the affected joints is purplish-cyanotic,
the joints are hot to the touch, the patient cannot stand on his foot due to sharp pain. For emergency
care, it is recommended to prescribe:
100% Colchicine
0% No-shpa
0% Diclofenac
0% Applications of dimexide
0% Antibiotics

20. A 14-year-old girl complains of tightness in the neck, coughing,


and thickening of the neck. Objectively: the thyroid gland is diffusely enlarged, dense during palpation,
not painful, the surface is smooth. Ultrasound shows that thyroid tissue is not uniform. Preliminary
diagnosis: autoimmune thyroiditis. To confirm the diagnosis, it should be determined in blood plasma:
100% Antibody titer to thyroglobulin
0% Calcitonin content
0% Growth Hormone Content
0% Parathyroid Hormone content
0% Iodine content in daily urine

21. A 37-year-old patient has been ill for more than 10 years. She says that her brain has long been possessed
by"criminal scientists-physicists" who are testing various types of psychotropic weapons on her. Feels the
effects of laser beams, constantly hears messages that are transmitted directly to her
brain. Emotionally monotonous, she spends most of her days in bed, making notes that she doesn
't show to anyone. Clear consciousness, no formal intellectual and mnestic disorders were detected.
What is the diagnosis of this patient?
100% Paranoid schizophrenia
0% Chronic delusional disorder
0% The involutional paranoid
0% Schizo-affective psychosis
0% Reactive Paranoid

22.A 38-year-old patient has shortness of breath, palpitations, sweating, fatigue,


limb tremor, and insomnia. I've lost 10 kg of weight in the last month. On the ECG-atrial fibrillation,

Downloaded from the site - online testing step

tachysystolic form (heart rate-105/min). What is the mechanism of heart damage in this case?
100% Toxic effect on the myocardium of excess thyroid hormones
0% Inflammatory damage to the myocardium
0% Postmyocarditis cardiosclerosis
0% Atherosclerotic cardiosclerosis
0% Tumor process (myxoma) in the left atrium

https://translate.yandex.com/en/doc 5/540
22:20 ,27.6.2023 �� C : ; 5 B

23. when studying the lesson schedule of students of the 8th grade of a general education school, it is established:
the number of lessons during the week is 30. On Monday, the first lesson is math, the second is a foreign
language, and the third is physics; there are 6 lessons in total. What is the violation in the lesson schedule?
100% Place of the math lesson in the schedule
0% Number of lessons per week
0% Number of lessons per day
0% Place of a foreign language lesson in the schedule
0% Place of the physics lesson in the schedule

24. An 18-year-old boy with no clinical symptoms was found


to have a II-tone accent and systolic murmur on the pulmonary artery during cardiac auscultation. The heart tones are sonorous and
rhythmic. What is the most
likely diagnosis?Functional noise
100%
0% Pulmonary artery valve stenosis
0% Pulmonary artery valve insufficiency
0% Botal duct nonunion
0% Atrial septal defect

25. A 33-year-old patient complained of neck pain. He's been ill for 2 weeks.
The city has a general malaise. Objectively: on the back surface of the neck there is a dense
inflammatory node the size of a walnut, with edema of adjacent tissues, on the surface of the node,
in the center there are 5 necrotic rods. What is the most likely diagnosis?
100% Carbuncle
0% A boil
0% Colliquative tuberculosis
0% Vulgar ectima
0% Leishmaniasis

26. A 23-year-old female kindergarten teacher complains of general weakness, decreased


appetite, nausea, epigastric discomfort, dark urine color, acholic feces, jaundice.
I got sick 7 days ago, when there was general weakness, decreased appetite, dull pain under
the right costal arch. Darkened urine, became acholic feces. In kindergarten, I had contact with
children who had jaundice. Objectively: the skin and mucous membranes are yellowish, the liver
is enlarged by 2-3 cm, the spleen is palpated. What is the most likely diagnosis?
100% Viral hepatitis A
0% Flu
0% Mechanical jaundice

Downloaded from the site - online testing step

0% Leptospirosis
0% Infectious mononucleosis

27. A 38-year-old woman has been suffering from hay fever for 7 years. For the last 2 years, in
August (ragweed flowering period), the patient notes 2-3 attacks of suffocation, which are eliminated by 1
dose of salbutamol. Objectively: temperature-36.5°C, BH-18 / min, Ps-78 / min, AT-1I5 / 70 mm
Hg above the lungs-vesicular respiration. The heart tones are sonorous, the rhythm is correct.
Which drug will be most effective in preventing suffocation attacks during the critical
season for the patient?

https://translate.yandex.com/en/doc 6/540
22:20 ,27.6.2023 �� C : ; 5 B
100% Intala inhalations
0% Berotek inhalations
0% Atrovent inhalations
0% Taking suprastin
0% Teopek's reception

28. A 36-year-old female patient complains of unbearable itching and burning in the vagina, which increases at
night. During the examination, a sharp hyperemia of the vulva, the vaginal mucosa and the vaginal part
of the cervix was found. Discharge is liquid, abundant, foamy with a greenish tinge. During the examination of the uterus and
appendages, no pathology was found. What kind of disease can you think about in this case?
100% Urogenital trichomoniasis
0% Urogenital herpes
0% Gonococcal infection
0% Bacterial vaginosis
0% Urogenital candidiasis

29. A 62-year-old patient complained of shortness of breath, pain in the right side, and palpitations
that gradually increased during the day. Cyanosis of the skin of the face and
hands, tachycardia, tympanitis and weakening of breathing over the right lung are objectively determined. On the radiograph:
enlightenment of the right pulmonary field with mediastinal displacement to the left. What pathology
is most likely in this patient?
100% Spontaneous pneumothorax
0% Spontaneous hydropneumothorax
0% External valvular pneumothorax
0% Right pulmonary artery thromboembolism
0% Obturation of the right main bronchus

30. A 7-month-old girl with measles was hospitalized on the first day of the rash. Her
brother, who is 4 years old, did not have measles and was not vaccinated against this infection due to absolute
contraindications for health reasons. which of these measures can be used
to prevent measles in this child?
100% Gamma-globulin prevention
0% Vaccination
0% Use of antibiotics
0% Use of chemotherapy drugs
0% Phagoprophylaxis

Downloaded from the site - online testing step

31. an 8 - year-old child became acutely ill with a rise in body temperature to 39.2°C, a single
vomiting. Complaints of cramping abdominal pain, tenesmus, frequent bowel movements - low-cal,
with a large amount of mucus, including pus and streaks of blood. Objectively: a thickened
sigmoid colon, painful during palpation. What is the most likely diagnosis?
100% Shigellez
0% Salmonellosis
0% Acute appendicitis
0% Cholera
0% Rotavirus infection

32. a 32-year-old man went to a cardiologist for palpitations, pain in the heart,

https://translate.yandex.com/en/doc 7/540
22:20 ,27.6.2023 �� C : ; 5 B
and dizziness. He has been ill for about 10 years. The examination revealed pallor of the skin,
the presence of symptoms of Mus and Quincke, as well as diastolic murmur over the aorta with spreading
to the left and down. What other symptom can correspond to this disease?
100% Increased carotid artery pulsation
0% Atrial fibrillation
0% Reduction of pulse blood pressure
0% Right ventricular hypertrophy
0% Small solid pulse

33. a 13-year-old patient complains of periodic (spring, autumn) attacks of suffocation,


cough, first dry, and later wet. Objectively: the face is pale, somewhat swollen,
the chest is raised and expanded in antero-posterior size, BH - 3/min. Dry wheezing sounds are heard. In the blood:
HB-120 g / l, lake -8•109/l, p-1, s-52, L-36, mon-3, SHOE-7 mm / year. What is the most likely
pathology that determines such a picture?
100% Bronchial asthma
0% Bronchitis
0% Acute respiratory viral infections
0% Pneumonia
0% Cystic fibrosis

34. A 43-year-old patient complains of severe dyspnea of an inspiratory nature.


In the anamnesis: hypertension, CHD. Objectively: acrocyanosis, bubbling breath. BH - 30/Hv,
AT-230/130 mm Hgduring auscultation of the heart, the accent of the second tone is placed over the pulmonary artery. Choose
a combination of medications for your treatment:
100% Morphine, furosemide, pentamine
0% Prednisone, eufillin
0% Salbutamol, atropine, papaverine
0% Strophanthin, Panangin, Platyphylline
0% Eufillin, furosemide

35. A 28-year-old patient complains of pain in the right hypochondrium and joints, jaundice
of the skin, weight loss by 10 kg per year, fever up to 38°C. The disease
started after giving birth six months ago. Objectively: icteric skin and sclera, on the eyelids-
xanthomas. Liver + 4 cm, dense, painful, pointed edge. Spleen +2 cm. In the blood: ASAT-

Downloaded from the site - online testing step

280 U/L, ALT-340 U/L, total bilirubin - 97.6 mmol/L, free-54.6 mmol/L,
HBsAg was not detected. Name the main mechanism of pathogenesis of the disease:
100% Autoimmune
0% Toxic damage to hepatocytes
0% Fatty liver disease
0% Violation of the outflow of bile
0% Viral infection

36. A 26-year-old wounded man was taken to the clinic with a wound in the right cheek penetrating into the skull
. There is a lot of dried blood around the wound,as well as small grayish-black spots,
some of which have penetrated through the upper layers of the skin. What causes this damage?
100% The injury is caused by a firearm because there are additional factors of the shot being fired
0% This wound was formed from the action of a sharp object, because there is a lot of blood around it

https://translate.yandex.com/en/doc 8/540
22:20 ,27.6.2023 �� C : ; 5 B
0% The damage is caused by a blunt object, because this is a typical place for such injuries
The wound occurred when falling from a height, because most often people in such cases
0%
injure the head
0% This is an electric shock, because the wound is penetrating

37. A 41-year-old woman complains of weakness, rapid fatigue, fever


up to 38°C, rash on the face, pain in the wrist and elbow joints.
He has been ill for 3 years. On examination: erythematous rashes in the form of a butterfly on the cheeks,
the wrist and elbow joints are affected symmetrically, swollen;
pleural friction is heard above the lungs. In the blood: anemia, leukopenia, lymphopenia. In the urine, proteinuria and cylindruria.
Which antibodies are most likely to form in the mechanism of disease development?
100% To native DNA
0% Myositis-specific
0% To endothelial cells
0% To myosin
0% Rheumatoid factor

38. A 24-year-old female patient went to the gynecologist with complaints about the appearance of outgrowths in
the genital area. After examining the patient, the doctor found on the large and small bashful lips papillary
growths resembling cauliflower, soft consistency, not painful, not eroded.
The patient is referred for consultation to a dermatologist. What is the most likely diagnosis?
100% Genital warts
0% Broad warts
0% Vegetative pemphigus
0% Granulomatous candidiasis
0% Papillomatosis

39. a 5-month-old child with catarrhal symptoms developed pallor, weakness,


breast failure, perioral cyanosis, tension in the wings of the nose,deep wet cough,
small cucumber wheezes over the left lung, hard breathing with prolonged exhalation. BH-68 / min
. Heart rate-168 / min. In the blood: Er -3.6•1012/l, Ht-0.38 g/l, leuc.- 22,0•109/ l, WSE-25 mm/hour. What
research should be conducted to verify the diagnosis?

Downloaded from the site - online testing step

100% Chest radiography


0% ECG
0% Tomography of the lungs
0% Bronchoscopy
0% Computed tomography of the chest

40. A 27-year-old kindergarten nurse was hospitalized with complaints of sharp pain around
the navel, cramps in the lower extremities, repeated vomiting with bile, frequent
green liquid bowel movements with an unpleasant smell in large quantities. At the same time, all
the kindergarten staff and children of the senior group fell ill. 2 days ago, all of them consumed cottage cheese with sour cream. General
condition of the patient
of moderate severity. Temperature-38.2°C. Heart tones are rhythmic, muted. Heart rate-95 / min. AT
-100/60 mm Hg. the abdomen is moderately swollen, painful. Liver + 2 cm. What is the most likely
diagnosis?
100% Salmonellosis
0% Dysentery

https://translate.yandex.com/en/doc 9/540
22:20 ,27.6.2023 �� C : ; 5 B
0% Food toxicoinfection
0% Cholera
0% Enterovirus infection

41. A 47-year-old female patient suffers from hypomotor dyskinesia of the large intestine. How
to upgrade your diet to improve bowel motility?
In a rational diet, increase the amount of plant fiber and fermented
100%
dairy products
In a rational diet, increase the amount of plant-based, fermented dairy products and
0%
magnesium
0% In a rational diet, increase the amount of vegetable proteins, animal fats, and potassium
In a hypoenergetic diet, increase the amount of vegetable proteins, dairy products,
0%
and calcium
0% In a reducing protein-vegetable diet, increase the amount of animal fats and magnesium

42. a 34-year-old woman complains of pain in the heart area ("pinching, drilling"), which occurs
mainly in the morning hours in the autumn-spring period, with pain radiating to the neck, back, stomach;
rapid heartbeat, as well as a decrease in overall vitality. The occurrence of this condition is not
related to physical activity. In the evening, the condition improves. Somatic, neurological
status and ECG-without pathology. What is the most likely pathology that caused this clinical picture?
100% Somatized depression
0% Resting angina pectoris
0% Neurosis-like schizophrenia
0% Neurocirculatory dystonia
0% Hypochondriac depression

43. A 19-year-old patient went to a doctor due to severe weakness, fever,


progressive weight loss, and dizziness. Objectively: height 165 cm, Weight 40 kg.
The skin is dry, flaky, the skin is very pale with a yellowish tinge. In the blood: Er. -1.8•1012/l,
NY85 g/l, leuc.- 500 • 109/l, myeloblasts-78%, neutrophils -15%, lymphocytes-7%. What is the most
likely diagnosis?

10

Downloaded from the site - online testing step

100% Acute myeloblastic leukemia


0% Acute lymphoblastic leukemia
0% Chronic lymphoblastic leukemia
0% Anemia
0% Leukemoid reaction

44. A 46-year-old patient was taken by ambulance to the emergency room complaining of
sharp, paroxysmal pain in the right lumbar region radiating to the groin area and
to the inner surface of the thigh. The pain came suddenly a few hours ago. The day before, the patient
had profuse pain-free hematuria with worm-shaped blood clots. I've
never been ill before. What disease should you think about first?
100% Cancer of the right kidney
0% Bladder tumor
0% Necrotizing papillitis
0% Acute glomerulonephritis
0% Urolithiasis, right kidney stone

https://translate.yandex.com/en/doc 10/540
22:20 ,27.6.2023 �� C : ; 5 B

45. A 29-year-old patient became acutely ill 3 weeks ago. The temperature rose sharply, there
were severe pains in the left shin. Radiographs of this area show multiple foci of destruction
of irregular geometric shape with uneven contours.
Dense sequester shadows are defined in the destruction areas. Around the affected parts of the bone, strips of calcification
of the exfoliating periosteum are determined. What is the most likely diagnosis?
100% Acute hematogenous osteomyelitis of the left shin bones
0% Tuberculous lesion of the left shin bones
0% Sarcoma of the left tibia
0% Fibrotic osteodystrophy of the left shin bones
0% Metastatic lesion of the left shin bones

46. A 50-year-old man suffered a broken humerus while staying at home and was
taken to the emergency room of a polyclinic. How to register a temporary disability
of a victim?
100% A certificate of the established form for 5 days, and from the 6th day a disability certificate
0% Disability certificate from day 1 for the entire period of illness
0% Any form of help
0% Disability certificate signed by the Chief Medical Officer
0% A 6-day disability certificate and a free-form certificate

47. a child of 4.5 months is on irrational artificial feeding with cow


's milk. When the yolk was introduced, the child developed a spotty-papular rash all over the body,
wet areas, excoriation. What are the most common symptoms in the clinic of this
disease?
100% Itchy skin
0% Vomiting
0% Delayed psychomotor development

11

Downloaded from the site - online testing step

0% Increasing IgM levels


0% Febrile body temperature

48. a 68 - year-old man complains of a cough with sputum discharge that has been bothering him
for several years, hoarseness of voice, and general weakness. Lives near
an asbestos processing plant. Objectively: in the lungs on the right - weakened breathing with an extended exhalation,
dry wheezing. On the radiograph: in the area of the root and basal zone, the case is heterogeneous, with
indistinct darkening contours, increased airiness of the lungs. ENT: paresis of the right vocal
cord. What is the most likely diagnosis?
100% Central cancer of the right lung
0% Right-sided basal pneumonia
0% Chronic dust bronchitis
0% Asbestosis
0% Tuberculosis of the lungs

49. a 49-year-old man complains of periodic headache, high blood


pressure, aching heart pain, pain and whiteness of the hands when immersed in
cold water, which disappear after 15 minutes after the usual work, paresthesia and numbness
of the upper extremities. Objectively: AT-160/100 mm Hg, accent of tone II over the aorta, systolic

https://translate.yandex.com/en/doc 11/540
22:20 ,27.6.2023 �� C : ; 5 B
murmur at the apex of the heart. Positive white spot symptom, pile, positive cold test.
Which diagnosis is most likely?
100% Vibration sickness
0% Hypertension
0% Raynaud's syndrome
0% Osteochondrosis of the spine with radicular syndrome
0% Coronary heart disease

50. A 55-year-old woman complains of gratuitous bruising, weakness, bleeding gums,


and dizziness. Objectively: the mucous membranes and skin are pale, with numerous
hemorrhages of various antiquities. The lymph nodes are not enlarged. Ps-100 / min., AT-110/70 mm Hg.
no changes were detected from the internal organs. In the blood: Er.- 3,0-1012/ l, HB-92 g / l, CP-0.9,
anisocytosis, poikilocytosis, leukemia.- 10-109/l, e-2%, p-12%, c-68%, l-11%, m-7%, WZ-12 mm / year.
Additional determination of which laboratory parameter is most appropriate for establishing
a diagnosis?
100% Platelets
0% Reticulocytes
0% Blood clotting time
0% Osmotic resistance of red blood cells
0% Fibrinogen

51. A 57-year-old man was taken to the neurological department complaining of weakness
in his left arm and leg, headache, drooping right eyelid, and double vision. I got sick gradually during
the day after a nervous overload. Objectively: stun, tongue preserved, divergent
strabismus of the left eye, ptosis of the right eyelid, diplopia. The strength of the muscles in the left extremities is reduced.
Tendon reflexes on the left are higher. Which of these diagnoses is most likely?

12

Downloaded from the site - online testing step

100% Brain infarction


0% Subarachnoid-parenchymal hemorrhage
0% Meningoencephalitis
0% Myasthenia gravis
0% Botulism

52. An outbreak of viral hepatitis


A, possibly of aquatic origin, has been reported in an urban-type settlement located on a river. An increase in what indicators of water
quality in a reservoir can
confirm
100% this assumption?
Number of coli-phages
0% E. Coli Index
0% Oxidizability
0% Presence of the causative agent of water fever
0% Index of fecal coli-forms

53. A 45-year-old patient, a tractor driver, was taken to the clinic by passing transport. It is known that 3
hours ago I worked in a warehouse with toxic chemicals. The general condition of the patient is severe.
There is a sharp narrowing of the pupils, tremor, convulsions of the muscles of the whole body, a disorder
of the sphincter function. There is a 40% decrease in cholinesterase activity in the blood. What
is the most likely diagnosis?
100% Acute organophosphate poisoning

https://translate.yandex.com/en/doc 12/540
22:20 ,27.6.2023 �� C : ; 5 B
0% Acute organochlorine poisoning
0% Acute organohercury poisoning
0% Acute poisoning with arsenic-containing compounds
0% Acute nitrophenol poisoning

54. A 56-year-old overweight patient complains of pain in the knee joints, which
increases when walking, especially on stairs, and when standing on her feet for a long time. He has been ill for 5 years.
Objectively: the knee joints are deformed, swollen, and painful during movement. On the radiograph:
the joint gap is narrowed, subchondral sclerosis, marginal osteophytes. What is the most
likely diagnosis?
100% Deforming osteoarthritis
0% Rheumatic arthritis
0% Reactive arthritis
0% Gout
0% Rheumatoid arthritis

55. A 30-year-old patient complains of obesity, weakness, drowsiness, headache, decreased


potency, which he notes 4 years after the acute respiratory viral infection. Objectively: height-171 cm, weight-101 kg
. Fat deposition mainly on the shoulders, torso. The skin is dry, purplish-marble. AT-160/102
mm Hg ACTH level is elevated. What is the most likely diagnosis?
100% Itsenko-Cushing's disease
0% Itsenko-Cushing Syndrome
0% Alimentary obesity

13

Downloaded from the site - online testing step

0% Hypertension
0% Pathological menopause

56.a full-term baby aged 6 days has erythema, flaccid


blisters, erosive surfaces, cracks, peeling of the epidermis, which look like after scalding
with boiling water. A positive symptom of Nikolsky was detected. The child's general condition is serious.
Severe anxiety, hyperesthesia, febrile fever. What is the most likely diagnosis in
this case?
100% Ritter's Exfoliative dermatitis
0% Phlegmon of the newborn
0% Figner's pseudofurunculosis
0% Newborn's pemphigus
0% Epidermolysis

57. A 32-year-old woman complains of rashes in the torso, limbs, and pronounced
itching of the skin. The disease started suddenly after mosquito bites. Concomitant diseases:
chronic tonsillitis, chronic gastritis, colitis. Objectively: on the skin of the trunk and limbs
there are a large number of swollen papules ranging in size from a pinhead to large
discharge foci with jagged edges. In some areas, the rash regresses. What is the most
likely diagnosis?
100% Urticaria
0% Allergic dermatitis
0% Toxicoderma

https://translate.yandex.com/en/doc 13/540
22:20 ,27.6.2023 �� C : ; 5 B
0% Microbial eczema
0% Eczema is real

58. A lead-smelter batch worker was exposed to lead in concentrations up


to 1 mg/m3 for 22 years. Which of the laboratory parameters is an early and probable sign of chronic
lead intoxication?
100% Porphyrinuria
0% Reticulocytosis, basophilic granularity of red blood cells
0% Increased lead concentrations in biological media
0% Leukocytosis
0% Anemia

59. A 10-year-old child suffering from diabetes mellitus for 3 years was taken to the
department in a state of hyperglycemic coma. The primary dose of insulin should be given
at the rate of:
100% 0.1-0.2 U/kg of body weight per hour
0% 0.05 U / kg of body weight per hour
0% 0.2-0.3 U/kg of body weight per hour
0% 0.3-0.4 U/kg of body weight per hour
0% 0.4-0.5 U/kg of body weight per hour

14

Downloaded from the site - online testing step

60. During a hospital examination, a 7-month-old girl was diagnosed with a high
ventricular septal defect. Which department of the cardiovascular system will be re-energized
first?
100% Right ventricle
0% Right atrium
0% Left ventricle
0% Left atrium
0% Large circle of blood circulation

61. An 11-year-old girl was registered with a gastroenterologist at a dispensary due to


duodenal ulcer and biliary dyskinesia. How
often should anti-relapse treatment courses be performed?
100% Twice a year
0% Every 2 months
0% Every 3 months
0% Once a year
0% Three times a year

62. A 60-year-old woman complains of general weakness, a sense of burden, embossment,


epigastric congestion, nausea, and belching after eating. He has been ill for about 15 years. Об’єктивно:
temperature-36.4°C, BH-20/xb, Ps-88/xb, AT-115/75 mm Hg. skin and mucous membranes are pale. In the blood:
Er.-2,0-1012/ l, HB-100 g/l. antibodies to gastric lining cells were detected. What is the most
likely cause of anemic syndrome in this patient?
100% Production of antibodies to internal factor
0% Violation of hemoglobin synthesis
0% Violation of erythropoietin synthesis
https://translate.yandex.com/en/doc 14/540
22:20 ,27.6.2023 �� C : ; 5 B

0% Iron malabsorption
0% Increased iron consumption

63. A 44-year-old patient was taken to the clinic for pain in the left half of the lumbar
region, which appeared after falling from a height of 2 m. after the injury, during twice
urination, he noticed the presence of macrohematuria. The condition is of moderate severity.
Blood pressure and pulse are normal. No pathological changes were found on the part of the thoracic and abdominal organs
. There is a slight painful swelling in the left lumbar
region. What is the preliminary diagnosis?
100% Closed kidney injury
0% Urolithiasis
0% Tuberculosis of the kidneys
0% Kidney tumor
0% Acute pyelonephritis

64. A 65-year-old female patient complained of periodic pain in the proximal


interphalangeal and wrist joints with periodic swelling and redness,
which had been bothering her for 4 years. On the radiograph, there are changes in the form of osteoporosis,
narrowing of the joint gaps and single Usures. What is the most likely diagnosis?

15

Downloaded from the site - online testing step

100% Rheumatoid arthritis


0% Osteoarthritis
0% Gout
0% Pseudogout
0% Myeloma disease

65. A 38-year-old patient complains of severe burning pain in the hand and
forearm. History: incised wound of the volar surface of the lower third of the forearm.
What nerve damage most often causes such pain?
100% Sredinnogo
0% Radiation
0% Elbow joint
0% Interosseous
0% Underarm

66. A 33-year-old patient was taken to the clinic with complaints of shortness of breath that suddenly appeared and
turned into suffocation, a feeling of "coma in the throat", tremor of the hands, and fear of death. The attack
developed for the first time due to strong excitement. I've never been ill before. Objectively:
BH-28 / min, Ps-104 / min, rhythmic, BP-150/85 mm Hg. respiration is vesicular,scratched,
shallow, with an extended exhalation. The borders of the heart are not changed by percussion. Heart sounds are loud
and rhythmic. What is the most likely diagnosis?
100% Neurocirculatory dystonia
0% Bronchial asthma
0% Hypertensive crisis
0% Cardiac asthma
0% Thyrotoxic crisis

https://translate.yandex.com/en/doc 15/540
22:20 ,27.6.2023 �� C : ; 5 B
67. A 48-year-old patient suffering from scleroderma for 5 years achieved stable
remission after using penicillamine for a year. What mechanism of action of the drug determines
the pathogenetic effect?
100% Inhibition of collagen synthesis
0% Effect on protein metabolism
0% Binding poisons
0% Effect on T-lymphocytes
0% Suppression of macrophage function

68. A 28-year-old patient complains of cutting abdominal pain. I fell ill abruptly 1.5
hours ago, when I felt a sharp pain under my chest. I've never been ill before. Objectively: the skin
is pale, the patient is lying on his back. When you change the position of the body, the pain worsens. Ps-70 / min,
rhythmic, blood pressure-100/60 mm Hg. there is no pathology on the part of the heart and lungs. The tongue is dry and clean.
The abdomen is retracted, does not participate in breathing, during palpation it is sharply tense and painful in
all parts. The Shchetkin-Blumberg symptom is positive. What is the most likely diagnosis?
100% Perforated stomach ulcer
0% Acute pancreatitis
0% Acute cholecystitis

16

Downloaded from the site - online testing step

0% Acute intestinal obstruction


0% Acute appendicitis

69. A 6 - year-old boy was diagnosed with acute glomerulonephritis with isolated
urinary syndrome, a period of incomplete clinical and laboratory remission, without impaired
renal function, and microhematuria was observed for 7 months. What course of the disease
should I think about in this case?
100% Lingering
0% Sharp
0% Subacute
0% Chronic
0% Recurrent

70. A 49-year-old patient was first diagnosed with open-angle glaucoma. Assign drops:
100% Pilocarpine hydrochloride 1%
0% Atropine Sulfate 1%
0% Dexamethasone 1%
0% Sulfacyl sodium 30%
0% Taufon

71. 23-year-old birthing child, II period of timely delivery. An eclampsia attack began. During
internal examination: the fetal head fills the entire sacral cavity, reaching the pelvic
floor, an arrow-shaped seam in a straight size, a small crown facing the pubis. What are the tactics
of labor management at this stage?
100% Applying obstetric forceps
0% Caesarean section
0% Conservative management of litter delivery - with episiotomy
0% Intensive care of gestosis with continued conservative management of labor
0% Vacuum extraction of the fetus

https://translate.yandex.com/en/doc 16/540
22:20 ,27.6.2023 �� C : ; 5 B

72. A 48-year-old female patient suffering from rheumatism, combined mitral heart disease,
sought medical help due to the sudden appearance of palpitations after physical exertion
. Objectively: AT-130/80 mm Hg, heart rate-112 / min, Ps-82 / min, the rhythm is incorrect. On
the ECG: P is absent, QRS is unchanged, RR = 0.98-0.54 s. the patient had a complication:
100% Atrial fibrillation paroxysm
0% Supraventricular extrasystole
0% Ventricular extrasystole
0% Paroxysmal supraventricular tachycardia
0% Paroxysmal ventricular tachycardia

73. A 59-year-old teacher complains of pad-like pains behind the sternum lasting up to 20
minutes. My condition worsened 2 days ago, and I had acute respiratory infections the day before. Objectively: heart tones
are muted, the accent is tone II above the aorta. AT - 180/100 mm Hg, Ps-80 / min. The abdomen is soft, the liver
is at the edge of the costal arch. On the ECG in dynamics-negative T Wave in V2-V5. What drug

17

Downloaded from the site - online testing step

what is considered mandatory for prehospital treatment?


100% Acetylsalicylic Acid
0% Thrombolytic Actilise (t RA)
0% Calcium Antagonist Nifedipine
0% Preduktal
0% Cardiac glycoside digoxin

74. A 47-year-old patient complains of recurrent pain in the left hypochondrium, which can be
triggered by eating spicy food. Pain radiates to the back. There is a weight loss of up to 3-5 kg during
the acute period. Bowel movements are unstable: pinning is replaced by gruel-like
bowel movements with an admixture of fat up to 3-5 times a day. The abdomen is painful in the epigastrium and left
hypochondrium. scar after cholecystectomy. Segments of the colon are sensitive during palpation.
Urine amylase on the day of hospitalization-1024 units. Pathogenetic treatment of the patient should include:
100% Anti-enzyme preparations (kontrikal)
0% Antibiotics for eradication of helicobacteriosis
0% Antispasmodics (no-shpa)
0% Prokinetics (metoclopramide-cerucal)
0% Anti-inflammatory drugs (aspirin)

75. A 25-year-old pregnant woman complained of aching pains in the lower


abdomen and lower back. The gestation period is 9-10 weeks. I have a history of 2 artificial abortions. During
vaginal examination: the cervix is 3 cm long, the cervical canal is closed, the uterus
is enlarged according to the gestational age, the discharge is mucous, scanty. What is the most
likely diagnosis?
100% Risk of spontaneous miscarriage
0% Spontaneous miscarriage that started
0% Undeveloped pregnancy
0% Bubble drift
0% Abortion is in progress

76. Upon admission to a pre-school institution, the doctor found that 20% of children

https://translate.yandex.com/en/doc 17/540
22:20 ,27.6.2023 �� C : ; 5 B
were100%
undergoingAppetite
prolonged adaptation. Which of these children's indicators is normalized first?

0% Sleep
0% Emotional state
0% Immunoglobulin secretion
0% Speech activity

77. A 49-year-old patient with a prolonged attack of bronchial asthma was hospitalized. In the lungs
, breathing is weakened, a lot of dry wheezing. PDR-32 / min, AT-140/90 mm Hg, Ps-90/min.
an intravenous injection of 1 ml of epinephrine was made. The condition has not improved: shortness of breath persists for 28 / min,
the number of dry wheezes in the lungs has decreased. Blood pressure rose to 170/110 mm Hg, tachycardia
130 / min. For emergency care, it is advisable to apply:
100% Verapamil IV
0% Intravenous beta blockers

18

Downloaded from the site - online testing step

0% Atrovent via nebulizer


0% Chew Corinthine
0% Cordaron internally

78.The area is home to 70,000 people, of whom 13 per cent are aged 0-14, 52 per cent are aged 15-49, and 35 per cent are aged 50
and over. What kind of graphical representation is the most appropriate way to represent this
data?
100% Pie Chart
0% Radial diagram
0% Line Chart
0% Bar chart
0% Cartogram

79. on the hospital's land plot, 25% of the area is occupied by buildings, 60% by green spaces, and
15% falls on the household yard and driveways. How should the layout
of the hospital site be improved?
100% Reduce the percentage of development
0% Reduce the percentage of landscaping
0% Increase the building percentage
0% Increase the percentage of landscaping
0% Reduce the percentage that falls on the household yard and driveways

80. A 14-year-old girl has a periodic increase


in body temperature up to 39°C for 2 months, fusiform swelling of the interphalangeal joints and pain in
the upper part of the sternum and neck, stiffness in the morning. What is the most likely diagnosis?
100% Juvenile rheumatoid arthritis
0% Rheumatism
0% Toxic Synovitis
0% Septic arthritis
0% Osteoarthritis

81. A 19-year-old patient went to the therapist complaining of poor health, chills,
runny nose, muscle and joint pain, nausea and diarrhea. He asks
https://translate.yandex.com/en/doc 18/540
22:20 ,27.6.2023 �� C : ; 5 B
for more painkillers and sedatives (tramadol or Solpadein, which help better, and
diazepam). Pharyngeal mucosa is pale pink and clear. In the lungs-vesicular respiration. Tachycardia.
The pupils are dilated, the reaction to light is flabby. On the skin of the forearms - traces of injections. When examined
, he is cheeky, irritated, rude, and deceitful. Make a diagnosis:
100% Opium addiction
0% Addiction to painkillers
0% Substance abuse due to the abuse of tranquilizers
0% Acute respiratory disease
0% Food toxicoinfection

82. A 25-year-old patient suddenly experienced intense headache, nausea, neck and lower back pain.

19

Downloaded from the site - online testing step

Objectively: there is a hemorrhagic rash on the body. Temperature 39.2° C. Severe meningeal
symptoms. Light, tactile, and pain hyperesthesia. In the blood: lake.- 25 • 109/L. which
survey method is most informative?
100% Lumbar puncture
0% Computed tomography
0% Electroencephalography
0% Transcranial Doppler imaging
0% Echoencephalography

83. A 57-year-old patient developed massive bleeding during cholecystectomy.


A blood transfusion was decided. The patient has a blood type AB (YY) Rh ( -). There is no such group at the blood transfusion station
. What group of donors can be called to donate blood?
100% Donors of rare blood types
0% Active Group
0% Related donors
0% Emergency donors
0% Reserve donors

84. a 5-year-old boy was vaccinated in violation of the vaccination schedule. Complaints of sore throat when
swallowing, headache, weakness, fever. Objectively: the child is pale,
enlarged anterior cervical lymph nodes, edema of the tonsils, their cyanotic hyperemia,
gray-white plaque on the tonsils, does not remove, if forcibly removed, the tonsils bleed. What is the most
likely diagnosis?
100% Oropharyngeal diphtheria
0% Lacunar sore throat
0% Simanovsky-Vincent's angina
0% Infectious mononucleosis
0% Follicular sore throat

85. An 18-year-old patient was taken to the hospital with significant edema that appeared two
weeks after suffering a sore throat. An increase in blood pressure to 160/110 mm Hg was detected.
suspected acute glomerulonephritis. What can manifest itself in the urinary sediment?
100% Significant proteinuria, erythrocyturia, and cylindruria
0% Minor proteinuria, leukocyturia
0% Microhematuria, crystalluria
0% Moderate proteinuria, macrohematuria, hyaline cylindruria
https://translate.yandex.com/en/doc 19/540
22:20 ,27.6.2023 �� C : ; 5 B

0% Macrohematuria, leukocyturia

86. A 44-year-old patient experienced chills every 48 hours for a week,


followed by fever. The body temperature rises to 40° C. A decrease in temperature after
3-4 hours is accompanied by sweating. My appetite worsened, and I felt generally weak.
The skin is pale. Enlarged liver and spleen. What is the most effective method for verifying
a diagnosis?
100% Microscopy of a blood smear and a thick drop
0% Hanging drop microscopy

20

Downloaded from the site - online testing step

0% General blood test


0% Bacteriological method
0% Enzyme-linked immunosorbent assay

87.A chemical plant is located on the northern outskirts of the village. During the year
, the following frequency of winds is observed: North -10%, east-20%, South-50%,
west-20%. What is the optimal location for placing a land plot for a hospital
in a medical and sanitary unit?:
100% South of the chemical plant
0% North of the chemical plant
0% East of the chemical plant
0% West of the chemical plant
0% On the territory of the chemical plant

88. a 54-year-old patient has splenomegaly without any particular subjective complaints. In the blood: Hb-142 g / l;
white blood cells - 32 • 109/l, e-5%, basal -2%, myeloblasts-6%, promyelocytes-5%, myelocytes-6%, juvenile
-8%, rod-4%, C-46%, L-12% , m-6%; WZ-19 mm / hour. For the treatment of the patient, it is advisable
to use:
100% Myelosan
0% Cyclophosphamide
0% Scheme 5+2
0% Prednisone
0% Treatment is not indicated

89. An acute group poisoning with nitrogene oxides occurred at a chemical plant with a fatal
outcome. A commission has been set up to investigate this case. Who should lead the commission of
inquiry?
100% Representative of the State supervision of labor Protection
0% Occupational pathology specialist of the health management body
0% Chief Physician of the Sanitary and Epidemiological station
0% Owner (manager) of the company
0% Representative of a trade union organization

90. The 300-bed city somatic hospital consists of the main building, which
houses the therapeutic and surgical departments. Several separate buildings house
maternity, children's, and radiology departments connected to the main building by underground and
above-ground covered walkways. Define the hospital development system:

https://translate.yandex.com/en/doc 20/540
22:20 ,27.6.2023 �� C : ; 5 B
100% Centralized-block system
0% Centralized
0% Decentralized
0% Vilna
0% Mixed version

91. The source of centralized domestic drinking water supply in a locality is

21

Downloaded from the site - online testing step

interplastic protected pressure aquifer. Water with high salinity


(up to 1.5 g / dm3, norm-1.0 g/dm3) and chloride content (700 mg/dm3, MPC-350 mg/dm3). What
pathology can develop in the population when consuming this water?
100% Hypertension
0% Nephroses
0% Enterocolitis
0% Gastritis
0% Cholelithiasis

92. A 36-year-old patient complains of shortness of breath, pain in the right hypochondrium, dry cough,
and swelling of the lower legs. Ill for 4 months, treated for rheumatism-no effect. Diffuse
cyanosis, leg edema, BH-28 / min, temperature-36.8°C. Above the lower parts of the lungs-small-
cucumber wheezes. The borders of the heart are expanded in all directions. Heart sounds are weakened,
systolic murmur at the 5th point. Ps-90 / min, AT-100/80 mm Hg. liver 4 cm below the costal
arch. What is the preliminary diagnosis?
100% Dilated cardiomyopathy
0% Rheumatic insufficiency of the mitral valve
0% Bacterial endocarditis of the aortic valve
0% Hypertrophic cardiomyopathy
0% Acute myocarditis

93. A 24-year-old patient has been suffering from rheumatism since the age of 15. I had 2 rheumatic attacks 4
years ago. During the last 6 months, paroxysms of atrial fibrillation occurred once every 2-3
months. What alternative antiarrhythmic therapy or tactics should be offered?
100% Preventive intake of Cordarone
0% Immediate hospitalization
0% Defibrillation
0% Taking lidocaine
0% Purpose of heparin

94. a 54-year-old patient has internal bleeding. An operation is underway. When determining the blood
type, agglutination occurred in the first and third drops of two series of standard hemagglutination
sera. What is the patient's blood type?
100% A (II)
0% 0 (I)
0% B (III)
0% AB (IV)
0% The definition should be repeated

https://translate.yandex.com/en/doc 21/540
22:20 ,27.6.2023 �� C : ; 5 B
95. during a blood transfusion during a biological test, the patient complained of chills,
was covered with cold sweat, notes the flickering of "panicles" in front of his eyes, a slight wobble in
the lumbar region. What does the patient have?
100% Blood transfusion reaction
0% Pyrogenic reaction
0% Bacterial-toxic shock

22

Downloaded from the site - online testing step

0% Citrate shock
0% Attack of renal colic

96. a 7-year-old child has a pronounced tilt of the head to the right, asymmetry of the facial skull, the right
sternoclavicular-nipple muscle is sharply tense, head movements are limited, and the right
shoulder is raised. What is the most likely diagnosis?
100% Congenital curvature
0% Scoliosis of the cervical spine
0% Klipel-Fein disease
0% Grizel's disease
0% Shereshevsky-Turner syndrome

97. On the second day of illness, a 24-year-old patient complains of high fever, headache
in the forehead, brow arches, when moving the eyeballs, in the muscles and joints. Objectively:
temperature-390C. the face is hyperemic, the sclera are injected. The mucous membrane of the soft
palate and posterior pharyngeal wall is brightly hyperemic with pinpoint hemorrhages. What
changes in the hemogram are typical for this disease?
100% Leukopenia
0% Leukocytosis
0% Neutrophilosis
0% Anemia
0% Accelerated ESR

98. a 30-year-old woman in labor. A full-term boy weighing 4,100 g and 54 cm long was born alive.
20 minutes after the birth of the fetus, a litter was born and 300 ml of blood was released. After
the litter is born, the bleeding continues. When examining the placenta, it was found that the vessels
pass through the edge of the placenta to the membranes and break off there. When examining the birth canal
with mirrors, no injuries to the cervix and vagina were found. The bleeding comes from
the uterine cavity. What interventions should be performed to stop the bleeding?
100% Perform a manual revision of the uterine body
0% Perform an external uterine massage
0% Enter uterotonics
0% Perform tamponade of the uterine body
0% Apply terminals to the parameters ' by the Baksheev method

99. A 46-year-old female patient was admitted to the clinic with complaints of diplopia, swallowing disorders, weakness
in the right arm, and unsteadiness during gait. The listed complaints occurred 12 days after
a sore throat, accompanied by a fever of up to 38°C, slight
sore throat and swelling of the subcutaneous tissue of the neck. What is the most likely diagnosis?
100% Diphtheria polyneuritis
0% Polio
0% Viral encephalitis
https://translate.yandex.com/en/doc 22/540
22:20 ,27.6.2023 �� C : ; 5 B
0% Botulism
0% Rheumatic damage to the nervous system
100. A 13-year-old teenager complains of diarrhea, weakness, and weight loss. My condition is getting worse

23

Downloaded from the site - online testing step

after consuming a large number of flour products. Such phenomena are observed from early
childhood. Objectively: the condition is satisfactory, low nutrition, lags behind in physical development.
The cause of the disease in this case is:
100% Gluten intolerance
0% Helminth infestation
0% Chronic pancreatitis, maldigestia syndrome
0% Intestinal dysbiosis
0% Lactase deficiency

101. A 37-year-old patient suffered from hepatitis A and B. She was delivered in a serious condition. If
you vomit liquid blood and blood clots with your mouth full. AT-80/40 mmHg Ps-108 / min. The abdomen
is enlarged and painless. From the mouth "liver" smell. Identify the most likely source
of bleeding:
100% Varicose veins of the esophagus and stomach
0% Acute ulcers of the duodenum 12
0% Rupture of the cardiac gastric mucosa
0% Chronic stomach ulcer
0% Pulmonary hemorrhage

102.when examining a 32-year-old victim of a car accident, the surgeon on duty found: the abdomen
is soft, there are no symptoms of peritoneal irritation, there is a painful pulsating
neoplasm to the left of the navel, auscultation above it is systolic noise, pulsation on the arteries of the lower
extremities is weakened. AT - 90/60 mm Hg, Ps-90 / min. What is the preliminary diagnosis?
100% Dissecting aortic aneurysm
0% Acute aortic bifurcation thrombosis
0% Traumatic injury to the left kidney
0% Colon rupture
0% Traumatic rupture of the spleen

103. an 11-year-old child with non-rheumatic carditis periodically has seizures, which
are manifested by a feeling of pain in the heart, shortness of breath, pallor, increased blood
pressure, a sharp increase in heart rate up to 180/min. Which of the medications is the most
effective in treatment?
100% Obzidan
0% Novocainamide
0% Lidocaine
0% Verapamil
0% Aimalin

104. A 46-year-old patient with an acute abscess of the left lung suddenly experienced severe
chest pain and suffocation when coughing, and tachycardia increased. The control Ro-gram revealed
a collaborated left lung, air in the left pleural cavity with a horizontal
fluid level. What is the mechanism of occurrence of this complication?
100% Breakthrough of the abscess into the pleural cavity

https://translate.yandex.com/en/doc 23/540
22:20 ,27.6.2023 �� C : ; 5 B
0% Ruptured Left Lung Bullae

24

Downloaded from the site - online testing step

0% Transfer of inflammation to the visceral pleura


0% Atelectasis of the left lung
0% Acute cardiopulmonary insufficiency

105.A 38-year-old woman went to the gynecological hospital complaining of significant


bleeding from the genital tract and a monthly delay in menstruation. In a bimanual study:
the cervix is barrel-shaped and soft in consistency. The uterus is of normal size, somewhat
softened. Appendages without features on both sides. In a mirror study:
the cervix is cyanotic, enlarged in size, the outer eye is opened up to 0.5 cm. A urine test for
HCG is positive. What is the most likely diagnosis?
100% Cervical pregnancy
0% Uterine pregnancy
0% Abortion is in progress
0% Threat of termination of pregnancy
0% Ectopic pregnancy

106. A 5-year-old child's general condition worsened after acute respiratory viral infection and developed rapid
fatigue. The skin is pale, the borders of the heart are extended to the left, and the tone at the apex is dull, a gentle
systolic murmur above the apex. ECG shows signs of left ventricular overload. What
is the most likely diagnosis?
100% Non-rheumatic myocarditis
0% Rheumatic carditis
0% Congestive cardiomyopathy
0% Congenital carditis
0% Acquired heart disease

107. A 46-year-old patient complains of headache and hallucinations. During the examination: 39oc,
agitated, face hyperemic, positive Govorov-Godelier symptom, petechial rash
on the trunk, limbs, tachycardia, hypotension, hepatosplenomegaly. Uses alcohol, injecting
drugs, and lives in a basement-type room. What is the preliminary diagnosis?
100% Typhus fever
0% hiv infection
0% Flu
0% Leptospirosis
0% Alcoholic psychosis

108. A 48-year-old patient has ulcerative colitis for 8 years and was treated with glucocorticoids.
Complains of cramping pain in the parotid and left iliac regions, which
has significantly increased over the past 2 weeks, diarrhea with mucus and blood 4-6 times a day,
fever up to 38-39 ° C, headache and pain in the knee joints. Objectively: the
patient's condition is moderate, Ps-108 / min, AT-90/60 mm Hg, heart and lungs without features, tongue
moist, abdominal muscle tone is significantly reduced, peristaltic noises are absent. What
complication did the patient develop?
100% Toxic dilatation of the colon
0% Perforation of the colon

https://translate.yandex.com/en/doc 24/540
22:20 ,27.6.2023 �� C : ; 5 B
25

Downloaded from the site - online testing step

0% Intestinal bleeding
0% Colon stricture
0% Colon cancer

109. a 36-year-old patient complains of frequent, painful, difficult urination, constant urge
to urinate, the appearance of several drops of blood in the urine at the end of urination. Got sick suddenly after
hypothermia. Ultrasound shows that there is little urine in the bladder, the walls of the bladder are swollen,
evenly thickened. In the urine: white blood cells-30-40 in p / S, protein-0.099 g / l, red blood cells unchanged-
5-7 in p / s. what is the most likely diagnosis?
100% Acute cystitis
0% Acute salpingoophoritis
0% Ureterocele
0% Bladder tumor
0% Bladder stone

110. during the examination of a child, it was found that he can watch a bright
toy for a long time, smiles. In the stomach position-raises and holds the head. It doesn't sit.
The child's age, based on his psychosomatic development, corresponds to:
100% 2 months
0% 4 months
0% 5 months
0% 6 months
0% 7 months

111. A 13-year-old boy complains of "hungry" nighttime abdominal pain, constipation, and heartburn. He has been ill
for more than two years. What is the most likely medical condition?
100% Duodenal ulcer disease 12
0% Acute gastritis
0% Chronic gastritis
0% Biliary dyskinesia
0% Peptic ulcer disease of the stomach

112. A 50-year-old man went to the trauma center with a lacerated shin
wound resulting from an unmotivated attack by a stray dog. What emergency
rabies prevention measures should be performed by a surgeon?
Washing the wound with a soapy solution, applying a bandage with virucidal
100%
drugs, introducing an anti-rabies vaccine
Washing the wound with a soapy solution, applying a bandage with virucidal
0%
drugs, do not vaccinate
Washing the wound with soapy water, applying a bandage with virucidal
0%
drugs, administration of rabies immunoglobulin
Surgical treatment of the wound, application of a bandage with virucidal drugs,
0%
administration of rabies immunoglobulin and rabies vaccine
Surgical treatment of the wound, applying a bandage, prescribing antibiotics with
0%
virucidal drugs

26

https://translate.yandex.com/en/doc 25/540
22:20 ,27.6.2023 �� C : ; 5 B

Downloaded from the site - online testing step

113. a child from a young smoker gave birth, the birth was complicated by the weakness of labor,
there were difficulties in vyvazhivanii head and shoulders. She was born with an Apgar score of 4
points. Which of these factors is a risk factor for spinal injury?
100% Difficulty in pulling out the head and shoulders
0% Young age of the mother
0% Bad habits
0% Weakness of labor activity
0% Chronic hypoxia

114. a full-term baby has cephalohematoma. On day 2, jaundice appeared, and on day 3
, the neurological condition changed (nystagmus, Grefe's symptom). The urine is yellow, the bowel
movements are yellow. Mother's blood type A (II)Rh+, child A (II) Rh+. On the third day, bilirubin in the blood is 58
mmol / l due to the indirect fraction. How to explain a child's jaundice?
100% Traumatic brain hemorrhage
0% Physiological jaundice
0% Hemolytic disease of newborns
0% Fetal hepatitis
0% Biliary tract atresia

115. at a woodworking plant, wood is dried using


electromagnetic fields of radio frequencies, the levels of which exceed the GDR. what material should
be used to make protective screens to prevent the negative impact of electromagnetic fields on
the body of workers?
100% Made of metal
0% Made of wood
0% Made of bricks
0% Made of plastic
0% Made of glass

116. A 10-year-old girl has scarlet fever. Received treatment: paracetamol, vitamins.
On the 14th day of the disease, lethargy, decreased appetite, pale skin, pasty and
puffy face, oliguria appeared. In the urine: specific gravity 1030, protein 1 g / l, red blood cells 60 v / s, hyaline
cylinders. What complication did the girl develop?
100% Glomerulonephritis
0% Pyelonephritis
0% Urolithiasis
0% Bladder tumor
0% Urethral polyps

117. A 34-year-old patient complains of throbbing pain in the depth of the right ear, which
increases with swallowing, coughing, hearing loss, poor health, and a runny nose.
Temperature 38.5-39°C. Ill for 2 days. Objectively: otoscopy shows hemorrhages under
the epidermis of the tympanic membrane and skin of the bony part of the right auditory canal.
Palpation of the mastoid process is painless. What is the possible diagnosis?
100% Acute right-sided otitis media

27

Downloaded from the site - online testing step

https://translate.yandex.com/en/doc 26/540
22:20 ,27.6.2023 �� C : ; 5 B

0% Boils of the external ear canal


0% Acute mastoiditis
0% Acute external diffuse otitis media
0% Exacerbation of chronic mesotimpanitis

118. on the 3rd day of life, a newborn who suffered asphyxia during childbirth developed a
bleeding from the umbilical wound. Laboratory-hypocoagulation, thrombocytopenia, hypothrombinemia.
What causes clinical and laboratory changes?
100% DIC syndrome
0% Hemorrhagic disease of newborns
0% Congenital angiopathy
0% Thrombocytopenic purpura
0% Injury to the umbilical vessel

119. a 40-year-old patient has a burn injury of 15% of the body surface of grade II-III. On the 20th day after
the injury, the patient has a sharp increase in body temperature, general weakness, frequent vesicular
respiration, exacerbation of facial features, blood pressure - 90/50 mm Hg, Ps-112 / min. What complication can
I think of?
100% Sepsis
0% Pneumonia
0% Acute intoxication
0% Purulent bronchitis
0% Anaerobic infection

120. during a visit to the Home of a 2-year-old child, a general practitioner diagnosed measles.
The child attends a nursery, has a 5-year-old sister. What document does the doctor need to fill out for
prompt anti-epidemic measures at the site?
100% Emergency notification of an infectious disease (form No. 058 / o)
0% Certificate of care for a sick child
0% !child development history (registration form No. 112 / o)
0% Doctor's home call book (Form # 031 / o)
0% Child care Disability certificate

121. an employee has been ill for 4 months, needs further treatment, and is unable to work. Who
is authorized to conduct an examination of the disability of this patient?
100% Medical and Social Expert Commission
0% Medical Advisory Commission
0% Doctor-curator with the head of the department
0% Chief physician of a medical and preventive institution
0% Deputy Chief Physician for Disability Assessment

122. in a 3-day-old newborn, feeding is almost always accompanied


by shortness of breath, paroxysmal cough, cyanosis, and foamy discharge from the mouth. After eating, during
auscultation, the number of moist, velvety wheezes in the lungs increases. Feeding

28

Downloaded from the site - online testing step

through the probe is not accompanied by coughing and increasing cyanosis. What is the most likely

https://translate.yandex.com/en/doc 27/540
22:20 ,27.6.2023 �� C : ; 5 B
diagnosis?
100% Esophageal-tracheal fistula
0% Esophageal atresia
0% Atresia hoan
0% Diaphragm keel
0% Nonunion of the hard palate

123. A 47-year-old female patient suffers from varicose veins of the right lower limb. Complaints of
pain, hyperemia along the subcutaneous veins, fever. When examining the great
saphenous vein on the lower leg and thigh, hyperemia is observed up to the middle third; during
palpation, painful, hard. What treatment does the patient need?
100% Operation Troyanov
0% Conservative therapy
0% Sclerosing treatment
0% Felder's Operation
0% Operation Palm

124. A 27-year-old patient suffered a fracture of the right femur in


the lower third due to a fall from a height, with the distal fragment protruding under the skin. The limb is warm,
the toes move. The ambulance doctor refused the Dieterichs splint and applied immobilization
with three ladder splints on the bent limb at the knee. What possible complication did he
prevent?
100% Damage to the popliteal neurovascular bundle
0% Development of traumatic shock
0% Development of fat embolism
0% Secondary displacement of bone fragments
0% Muscle interposition and skin perforation

125. A 62-year-old female patient came to the women's consultation complaining of watery
discharge, sometimes with admixtures of blood from the vagina. Menopause is 7 years old. Periodically, the patient
has minor pain in the lower abdomen, bloating of the intestines. The uterus during vaginal examination
is enlarged in size up to 10 weeks of pregnancy, limited mobility, sensitive. Applications are not
detected. What research should be done to clarify the diagnosis?
100% Separate diagnostic curettage of the cervical canal and uterine cavity
0% Ultrasound examination
0% Contrast radiography
0% Cytological examination of smears
0% Posterior arch puncture

126. A stout 73-year-old woman developed a clinical


picture of submassive pulmonary embolism on the 4th day after umbilical keel surgery. Duplex sonography of the veins of the lower
extremities revealed a floating blood clot in the femoral vein. What is the best way to prevent
recurrent embolism?
100% Implantation of a coffee filter

29

Downloaded from the site - online testing step

0% Continuous instantaneous infusion of heparin


0% Low molecular weight heparins
0% Thrombectomy

https://translate.yandex.com/en/doc 28/540
22:20 ,27.6.2023 �� C : ; 5 B
0% Plication of the inferior vena cava

127.A 19-year-old woman was admitted to the maternity hospital for the first time with labor.
Contractions in 3-4 minutes, 20-25 seconds each. The fetal head is fixed at the entrance to the small pelvis,
the fetal heartbeat is clear, rhythmic, 135 / min. During vaginal examination, the cervix is smoothed,
the opening of the external pharynx is 2.5 cm, and the fetal bladder is intact. What is the most likely diagnosis?
100% I labor period, latent phase
0% Primary weakness of generic forces
0% Secondary weakness of generic forces
0% Physiological preliminary period
0% I period of labor, active phase

128. a 26-year-old pregnant woman, second pregnancy, first delivery, urgent. Suddenly, during the second
stage of labor, she died. The section has an amniotic fluid embolism, acute
pulmonary heart failure. What diagnosis will be
recorded as the direct cause of death in the" medicinal death certificate"?
100% Acute pulmonary heart failure
0% Abnormal pregnancy
0% Amniotic fluid embolism
0% Second pregnancy, first delivery, urgent
0% Pulmonary embolism

129. A 37-year-old patient complains of pain in the left hypochondrium, which increases after
eating, bending the trunk. Two years ago, he underwent surgery for destructive
pancreatitis. Objectively: a body mass deficit of 15 kg. Ps-80 / min. The stomach is not deflated. In the left
subcostal area, the formation of a dense consistency is palpated, not mobile, slightly painful.
The dimensions are 15x20 cm, the contours are clear. There are no abnormalities in the blood. Suspected
pancreatic cyst. What is the instrumental method of examination that will allow you to establish
a diagnosis?:
100% Diagnostic laparoscopy and biopsy
0% Ultrasound examination
0% Overview radiography of the abdominal cavity
0% Computed tomography
0% Combination of ultrasound and cat

130.To characterize the health status of the population in the district, indicators
of fertility, mortality, natural growth, prevalence of diseases and primary
morbidity, general disability and disability of the population were calculated. What kind of statistics
do these indicators relate to?
100% Intensive
0% Extensive
0% Ratio

30

Downloaded from the site - online testing step

0% For visibility purposes


0% Standardized

131. a 69-year-old patient with symptoms of shortness of breath and cough was taken to the hospital. Pronounced
cyanosis of the lips. from the anamnesis: the patient ate cherries and choked on a bone. What is the most

https://translate.yandex.com/en/doc 29/540
22:20 ,27.6.2023 �� C : ; 5 B
likely diagnosis?
100% Foreign body in the respiratory tract
0% Bronchial asthma
0% Acute bronchitis
0% Pneumonia
0% Lung cancer

132. a 16-year-old girl has an irregular menstrual cycle, opsomenorrhea. Menarche from
the age of 14. Complaints of heavy spotting within 4 days after a 2-month delay. What
is the most likely diagnosis?
100% Juvenile uterine bleeding
0% Cancer of the uterine body
0% Malignant blood disease
0% Cervical polyp
0% Subepithelial endometriosis of the cervix

133. A 47-year-old woman complains of joint pain and muscle weakness, rapid
fatigue, sweating, palpitations, and fever. Objectively: dysphagia,
red rash around the eyes, on the cheekbones and over the interphalangeal joints, swelling
of the upper eyelids, tachycardia. In the blood: increased SSE and C-reactive protein. What disease
determines this picture?
100% Dermatomyositis
0% Myasthenia gravis
0% Rheumatoid arthritis
0% System red shepherd dog
0% Systemic scleroderma

134. A 49-year-old patient began to worry about constrictive pain behind the sternum, which occurs when
walking at 700-800 m. Drinks 2 liters of beer once a week. Arterial hypertension for
the last 7 years. Objectively: Ps-74 / min, AT-120/80 mm Hg. when conducting VEM at
a load of 75 W, ST segment depression was recorded 2 mm below the contour line in V4 — V6.
What is the most likely diagnosis?
100% Angina pectoris of tension, functional class II
0% Angina pectoris of tension, functional class III
0% Angina pectoris of tension, functional class IV
0% Hypertonic vegetative-vascular dystonia
0% Alcoholic cardiomyopathy

135. a 26-year-old patient had 4 generalized seizures per day, between

31

Downloaded from the site - online testing step

when the patient has not regained clear consciousness (is in a coma or sopor). Determine the state:
100% Epileptic status
0% Frequent generalized convulsive seizures
0% Frequent Jackson attacks
0% Hysterical attacks
0% Frequent complex partial seizures

https://translate.yandex.com/en/doc 30/540
22:20 ,27.6.2023 �� C : ; 5 B
136. A 44-year-old patient received a wound in the right shin during agricultural work
. I haven't received a preventive tetanus shot in the last 20 years.
What kind of tetanus prevention should be done?
Emergency, non-specific (primary surgical treatment of the wound), specific
100%
(active-passive immunization)
0% Routine, by introducing tetanus toxoid
0% Perform primary surgical treatment of the wound
0% To the victim to enter 3000 AO tetanus serum
0% Specific-active-passive immunization

137. A 41-year-old female patient punctured the index finger of her right hand with a fish bone. For 10
days, she did not seek medical help, but was treated independently. Complaints of throbbing pain,
redness, swelling of the injured finger, purulent discharge from the wound. The general condition
is satisfactory. Radiologically: osteoporosis and marginal destruction of the nail phalanx of the index
finger with the formation of cortical sequesters. What is the most likely diagnosis?
100% Bone panaritium
0% Syringomyelitic osteopathy
0% Osteogenic sarcoma
0% Deforming osteoarthritis
0% Bush Fracture

138. a 16-year-old patient's professional examination revealed an increase in blood pressure up to 150/90
mm Hg objectively: height-178 cm, body weight - 90 kg. The distribution of fat
is uniform. On the skin of the abdomen, inner thighs, shoulders, a small amount
of pink spots. The condition of the internal organs remains unchanged. Sexual development corresponds to age. What
is the most likely diagnosis?
100% Hypothalamic puberty syndrome
0% Hypertension
0% Hypertonic vegetative-vascular dystonia
0% The disease !cenca-Cushing
0% Corticosteroid treatment

139. the patient is 50 years old, has 18 years of experience as a welder, complains of suffocation and dry
cough. During X-ray examination of UCP: in the lungs, against the background of an enhanced, reticulated
pulmonary pattern, a large number of small, rounded, clearly defined shadows are detected in
the middle and lower parts on both sides, symmetrically. The roots of the lungs are compacted, expanded.
What is the most likely diagnosis?
100% Pneumoconiosis

32

Downloaded from the site - online testing step

0% Disseminated pulmonary tuberculosis


0% Miliary carcinomatosis
0% Cardiogenic pneumosclerosis
0% Sarcoidosis

140. A full-term newborn with a body weight of 4500 g was born in asphyxia with
an Apgar score of 4-6 points. During childbirth, vyvazhivanie of the shoulder girdle is difficult. In
the neurological status of general brain disorders, total upper flaccid paresis was found-the hand is atonic,

https://translate.yandex.com/en/doc 31/540
22:20 ,27.6.2023 �� C : ; 5 B
penetrated,
100% reflexes
CV-TEare not called - grasping, Babkin, Moro. Specify the segments
of the spinal cord lesion:
0% SI-SII
0% SIII-XIV
0% TI-TV
0% ТVI-ТVII

141. a newborn from the first pregnancy weighing 3500 g has jaundice, lethargy,
and decreased reflexes from the first day. Objectively: jaundice of the skin of the second degree with a saffron hue,
liver + 2 cm, spleen +1 cm. Urine and stool are yellow. In the blood: HB-100 g / l, ep.- 3,2 •
1012/ l, leuc. -18.7 • 109/l, mother's blood 0 (I) Rh ( + ), child's blood A (II) Rh ( - ), bilirubin-170
mmol/l, indirect fraction. ALT and AST levels are normal. What disease is suspected in the child?
100% Hemolytic disease of the newborn, AV0-conflict
0% Intrauterine hepatitis
0% Hemolytic disease of the newborn, Rh-conflict
0% Biliary tract atresia
0% Physiological jaundice

142. The Health Department needs to assess the effectiveness of implementing


maternity and child protection programs in the region. The level of which integral indicator
is most appropriate to use for such an analysis?
100% Infant mortality rate
0% Perinatal mortality
0% Morbidity of children
0% Children's disabilities
0% Physical development

143.a 58-year-old man experienced


acute pain in the heart area and severe shortness of breath at the 5th week after a myocardial infarction. Objectively: the patient's
condition is extremely severe,
marked cyanosis of the face, swelling and pulsation of the cervical veins, pulse in the peripheral
arteries is not detected, on the carotid artery - rhythmic, 130 / min, AT-60/20 mm Hg. with
auscultation of the heart - tones are sharply weakened, with percussion-expansion of the borders of the heart in both directions.
What100%
is the treatment strategy
Pericardial for this
puncture patient?
and immediate thoracotomy
0% Inhaling oxygen
0% Left pleural puncture

33

Downloaded from the site - online testing step

0% Conservative treatment, infusion of adrenomimetics


0% Drainage of the pleural cavity

144. A 12-year-old child is taken to the hospital after being stung by a bee. After the examination, the doctor
found the child to be in anaphylactic shock. What complex of urgent measures should
be prescribed in this case?
Stop receiving bee venom in the child's body, enter a subcutaneous 0.1%
100%
solution of epinephrine, in / in a jet of glucocorticosteroids
0% Immediately enter I / M Lasix
0% Immediately administer intravenous corticosteroids and perform tracheal intubation
Immediately administer an intravenous antihistamine and continue monitoring the
0%
child's condition
https://translate.yandex.com/en/doc 32/540
22:20 ,27.6.2023 �� C : ; 5 B

0% Immediately administer an intravenous antihistamine and intravenous Lasix

145. A 39-year-old female patient suffers from chronic idiopathic thrombocytopenic purpura.
Within two weeks complains of dizziness, sharp weakness, black color of feces.
Objectively: the condition is severe, the skin is pale, and there are many pinpoint hemorrhages on the face and limbs. In
the blood: Er.- 1,2 • 1012/ l, HB-50 g/l, CP-0.7, blood clot.- 2 • 109/l, lake.- 3,1 • 109/ l, ESR - 30
mm / h., anisocytosis, poikilocytosis. What is the nature of anemia in the patient?
100% Post-hemorrhagic
0% Hemolytic
0% Iron-deficient
0% Megaloblastnaya street
0% Aplastic

146. A newborn at the age of 32 weeks of gestation developed


progressive cyanosis, shortness of breath, inflated nose wings, retraction of intercostal space, and weakened breathing 2 hours after
delivery
. The Silverman score is 4 points. X-ray examination of the chest revealed
a nodal-reticular pattern of the lungs, reduced pneumatization. What is the most
likely diagnosis?Respiratory distress syndrome
100%
0% Intra-natal asphyxia
0% Congenital pneumonia
0% Pneumothorax
0% Congenital heart disease

147. A 48-year-old patient went to a doctor complaining of difficulty swallowing food,


voice loss, and weight loss. These symptoms have been bothering the patient for the last 3 months,
gradually progressing. Objectively: the patient is emaciated, there are enlarged supraclavicular
lymph nodes. During esophagoscopy, no esophageal pathology was detected. Which of these
studies is most appropriate in this case?
100% Computed tomography of the thoracic cavity and mediastinal organs
0% Lung radiography
0% Polypositional esophageal radioscopy
0% Radioisotope examination of the thoracic cavity and mediastinum

34

Downloaded from the site - online testing step

0% Ultrasound examination of the mediastinal organs

148. A 64-year-old patient was hospitalized with complaints of shortness of breath, heaviness in the right
hypochondrium, and abdominal enlargement. During the examination, the doctor noticed the absence of edema on
the legs with obvious ascites (ascitis praecox), swelling of the cervical veins, and a frequent small pulse. During
auscultation of the heart, the pericardial tone is determined. Echocardiography: pericardial leaf separation-0.5 cm,
right atrium diameter - 6 cm. On the X-ray: the heart is of normal size, along the edge
of the right parts of the heart, an X-ray contrast contour is determined. What disease should
be suspected in this case?
100% Constrictive pericarditis
0% Dilated cardiomyopathy
0% Myocardial infarction in the stage of scar formation
0% Exudative pericarditis
0% Mitral valve insufficiency

https://translate.yandex.com/en/doc 33/540
22:20 ,27.6.2023 �� C : ; 5 B

149. boarding school students complain of blood discharge when brushing their teeth, blurred vision
in the evening, bruising from minor injuries. What nutrients can
be considered insufficient in this case?
100% Ascorbic acid and retinol
0% Riboflavin and pyridoxine
0% Calcium and phosphorus
0% Tocopherol and calciferol
0% Selenium, copper, zinc

150.during an operation for autoimmune thyroiditis in combination with a multi-node goiter


, the right lobe was removed and the left lobe was subtotally resected. What should be prescribed
to the patient in order to prevent postoperative hypothyroidism?
100% L-thyroxine
0% Mercazolil
0% Yodomarin
0% Lithium preparations
0% Insulin

151. A 77-year-old patient complains for about six months of difficulty in passing food, sharp
weight loss. The last month, food passes freely, but there was a hoarseness of the voice, and then
a powerful cough during meals, especially liquid ones. My body temperature has increased.
He was hospitalized in serious condition. Chest X-ray revealed lower-lobe pneumonia.
What is the preliminary diagnosis?
100% Esophageal cancer, esophago-bronchial fistula
0% Hypostatic pneumonia
0% Lung cancer with mediastinal lymph node metastases
0% Cancer of the esophagus with spread to the reverse nerve
0% Abscessed lower kidney pneumonia

35

Downloaded from the site - online testing step

152.A 35-year-old employee, while on vacation in another city, fell ill and
was admitted to the city hospital for acute appendicitis. Will
a disability certificate be issued in this case?
100% The disability certificate is issued with the permission of the chief physician of the city hospital
0% A disability certificate is issued with the permission of the head of the hospital department
0% A disability certificate is not issued
0% An arbitrary form of certificate is issued for the entire period of hospitalization
0% A certificate of the established form is issued for the entire period of hospitalization

153. a newborn child has a sudden fever of up to 39oC 10 days after birth
. Objectively: on the erythematous skin in the navel, abdomen and chest
, multiple pea-sized blisters appeared, without infiltration at the base,
bright red wet erosions with fragments of the epidermis along the periphery are sometimes determined. Specify
a preliminary diagnosis:
100% Epidemic pemphigus of newborns
0% Syphilitic pemphigus

https://translate.yandex.com/en/doc 34/540
22:20 ,27.6.2023 �� C : ; 5 B
0% Streptococcal impetigo
0% Vulgar impetigo
0% Allergic dermatitis

154. an ambulance doctor responded to a call to a man who had been pulled out of a garrote by his relatives.
Objectively: there is no pulse in the carotid arteries, no consciousness, independent
breathing, corneal reflexes, and the presence of cadaveric spots on the back and back of the limbs.
What signs can be used to determine the onset of death?
100% Presence of cadaveric spots
0% Lack of independent breathing
0% Lack of corneal reflexes
0% No pulse rate
0% Lack of consciousness

155. 4 days after a gunshot wound to the soft tissues of the thigh, the patient's condition
began to progressively worsen. Complaints of bursting pain in the wound of the middle third of the thigh,
the pain increases in the last 12 hours. Swelling of the skin and subcutaneous tissue increases rapidly.
Body temperature 38.2°C. Ps-102 / min. The edges of the wound are gaping, dull in color, yesterday's viable
muscles explode into the wound, boiled-looking, dull, with a gray-dirty coating, when captured
with tweezers-disintegrate. What type of infection has developed in the hip wound?
100% Anaerobic clostridial wound infection
0% Wound diphtheria
0% Aerobic gram-negative wound infection
0% Putrefactive wound infection
0% Aerobic gram-positive infection

156. A 29-year-old patient went to the doctor for a sore throat, fever
up to 38.2°C. A week ago, I got sick with angina, performed my medical appointments in bad faith.
Objectively: forced head position, trismus of the masticatory muscles. Left peritonsillar

36

Downloaded from the site - online testing step

the area is sharply hyperemic, protruding. What is the preliminary diagnosis?


100% Left-sided peritonsillar abscess
0% Meningitis
0% Phlegmonous sore throat
0% Pharyngeal diphtheria
0% Tonsillar tumor

157. a 75-year-old patient complains of the inability to urinate, bursting pain over
the womb. I was acutely ill 12 hours ago. Objectively: a full bladder is palpated above the womb
. Rectally: the prostate is enlarged, tightly elastic, with clear contours, without nodes.
The interparticle groove is pronounced. In ultrasonography, the prostate volume is 120 cm3, it
is pushed into the bladder cavity, the parenchyma is uniform. Prostate-specific antigen-5
ng / ml. What is the most likely medical condition that caused acute urinary retention?
100% Prostatic hyperplasia
0% Prostate cancer
0% Prostate sclerosis
0% Tuberculosis of the prostate
0% Acute prostatitis

https://translate.yandex.com/en/doc 35/540
22:20 ,27.6.2023 �� C : ; 5 B

158. A 24-year-old patient complains of aching pains, a feeling of heaviness, bursting in the epigastrium
after eating, and early satiety. These complaints have been bothering you for 3 years (several
months a year), sometimes related to a stressful situation. A history of vegetative-vascular dystonia. On
FGDS and ultrasound of the abdominal organs, no organic pathology was detected. Leading in
the pathogenesis of this disease is:
100% Violation of motility of the stomach and duodenum 12
0% Autoimmune inflammation of the gastric mucosa
0% Infectious inflammation of the gastric mucosa
0% Low level of gastric secretion
0% High level of gastric secretion

159. a 43-year-old patient developed a rounded lump on the anterior surface of the right shoulder 3 months ago
, slightly painfully dense-elastic consistency, sedentary, measuring up to 8 cm in
diameter. What is the most likely diagnosis?
100% Fibrosarcoma
0% Lipoma
0% Fibroma
0% Atheroma
0% Hydroadenite

160. A 53-year-old woman complained to a dermatologist about a rash accompanied


by itching. Ill for one week. First, a large spot appeared on the torso. After washing in
the bath, the number of rashes increased dramatically, they became bright. Objectively: on the skin of the trunk and
limbs there are multiple edematous spots of pink-yellow color, with slight peeling in the center,
oval contours that are placed along the lines of skin tension. What is the most likely diagnosis?
100% Pink lichen

37

Downloaded from the site - online testing step

0% Secondary fresh syphilis


0% Psoriasis, progressive stage
0% Multicolored lichen
0% Seborrheic eczema

161. a 32-year-old woman complains about the absence of pregnancies for 5 years. A
complete clinical examination was performed: hormonal function was not impaired, urogenital infection
was absent, hysterosalpingography showed that the fallopian tubes were filled with contrast from both sides
to the isthmus, and the contrast was not visualized in the abdominal cavity. The man is healthy. Which
tactics are most appropriate?
100% In Vitro fertilization
0% Male sperm insemination
0% ICSI in the in vitro fertilization program
0% Hydrotubation
0% Laparoscopic tube plastic surgery

162. pregnant for 28 years, gestational age 37 weeks. A pregnant woman smokes a lot. The man is healthy.
No extragenital pathology was detected. Objectively: fetal heartbeat is 126 / min, muffled. According
to ultrasound data, the size of the fetus corresponds to the size of a 34-week pregnancy. What is the most
likely reason for the discrepancy in the duration of pregnancy?
https://translate.yandex.com/en/doc 36/540
22:20 ,27.6.2023 �� C : ; 5 B
100% Fetoplacental insufficiency
0% Fetal hypoxia
0% Age of the pregnant woman
0% Genetic pathology
0% Hormone deficiency

163. A 37-year-old female patient went to a antenatal clinic complaining of infertility for
7 years. The menstrual cycle is two-phase. According to hysterosalpingography, the fallopian tubes
are impassable in the ampullary sections, there is an adhesive process in the small pelvis. Which treatment method
is most appropriate in this case?
100% Laparoscopy
0% Laparotomy
0% Tubectomy
0% Adnexectomy
0% Hydrotubation

164. in the cold season, a patient taken from an open


reservoir was taken to the emergency room. There was no contact of the respiratory tract with water. Agitated, pale, complains of pain,
numbness of hands and feet, cold shivering. BH-22 / min, AT-120/90 mm Hg, Ps-110 / min, rectal
temperature 34.5°C. What kind of warming is indicated for the patient?
100% Passive heating
0% Effusion of solutions 37° C
0% Warming compresses
0% Warm bath

38

Downloaded from the site - online testing step

0% Hemodialysis with blood warming

165. A 63-year-old patient suffered an acute myocardial infarction a week ago. The general condition
worsened. Bothered by shortness of breath at rest, pronounced weakness. Objectively: edema of the lower
extremities, ascites. The borders of the heart are expanded, paradoxical pericardial pulsation is 2 cm
lateral to the apical push on the left. What is the most likely diagnosis?
100% Acute heart aneurysm
0% Repeated myocardial infarction
0% Acute pericarditis
0% Cardiosclerotic heart aneurysm
0% Pulmonary embolism

166. An electric welder with 15 years of work experience was found


to have dry wheezing in the lower parts of the lungs during a medical examination. Diffuse nodules
3-4 mm in size in the middle and lower parts of the lungs are observed on the X-ray. What disease can be suspected?
100% Metallokonioz
0% Silicosis
0% Silicatosis
0% Carboconiosis
0% Bronchitis

https://translate.yandex.com/en/doc 37/540
22:20 ,27.6.2023 �� C : ; 5 B
167. a 7-year-old girl complains of pain in the vaginal area, significant purulent
discharge, which worries for 5 days and gradually increases. On examination, the doctor found
significant swelling of the external genitalia, redness, purulent discharge from the vagina with
an unpleasant smell. Ultrasound scans show an echo-positive shadow in the vaginal area. What reason
can lead to this condition in children?
100% Presence of a foreign body in the vagina
0% Vaginal tumor
0% Cervical tumor
0% Purulent colpitis
0% Vulvovaginitis

168.According to the annual report of the city hospital's inpatient department, data were obtained on the actual
number of bed days and the number of patients treated during the year. Which of
the hospital performance indicators can be calculated based on these data?
100% Average length of a patient's hospital stay
0% Average bed occupancy
0% Bed turnover
0% Lethality rate
0% Efficiency of using the bed fund

169. when studying the sanitary and hygienic conditions in the 4-bed therapeutic ward, it was established:
the area of the ward is 30 m2, the height is 3.2 m, the air temperature is +20°C, the humidity is 55%,
the air velocity is 0.1 m / s, the light coefficient is 1: 5, the natural light coefficient is 0.6%, the content of

39

Downloaded from the site - online testing step

carbon dioxide in the air-0.1%. Which of the indicators does not meet the hygiene requirements?
100% Natural light ratio
0% Air velocity
0% Light coefficient
0% Carbon dioxide content in the air
0% Area of the Chamber

170. A 25-year-old patient is being treated by a hematologist for hemophilia A. After falling from
a horizontal bar, acute hemarthrosis of the knee joint has developed. Objectively: the right knee joint is sharply
enlarged in volume, the skin over it is hyperemic. Duke's bleeding time is 3 minutes,
and Lee-White's blood clotting time is 20 minutes. Which drug is most effective for treating this
patient?
100% Recombinant factor VIII
0% Recombinant X Factor
0% Tromboconcentrate
0% Amben
0% Aminocislaproic acid

171.An in-depth medical examination of a 10-year-old schoolboy revealed that he often


(almost monthly) suffers from acute respiratory diseases. Physical development is harmonious.
The student's biological age lags behind the calendar age in terms of the number of permanent teeth and annual
growth gain. Determine which health group the student belongs to:
100% 2
0% 1

https://translate.yandex.com/en/doc 38/540
22:20 ,27.6.2023 �� C : ; 5 B
0% 3
0% 4
0% 5

172.A 45-year-old patient complained to the district therapist about allergies,


acid belching, pain when swallowing food, and the inability to eat solid food. The patient
reported that he had experienced heartburn and acid belching for 8 years.
The deterioration of his condition marks the last 3 months. What examination should the patient undergo
first?
100% Fibroesophagogastroduodenoscopy
0% Proton Pump Inhibitor Test
0% Daily esophago-pH monitoring
0% Multi-hour esophago-pH monitoring
0% Computed tomography of the chest organs

173. on the tenth day after giving birth, a 23-year-old woman's general condition worsened:
chills appeared, pain in the right breast, and her body temperature rose to 39oc. Objectively: the right
mammary gland is slightly enlarged, a painful
dense infiltrate measuring 5x7 cm with a softening area in the center is palpable in the Upper-outer quadrant, the skin above it
is hyperemic with a bluish tinge. Axillary lymph nodes on the right are enlarged, painful
during palpation. What is the most likely diagnosis?

40

Downloaded from the site - online testing step

100% Purulent mastitis


0% Lactostasis
0% Serous mastitis
0% Breast fibroadenoma
0% Breast cancer

174. a 32-year-old patient complains of severe general weakness, abdominal pain, nausea,
vomiting, dizziness, and marked weight loss. Objectively: the skin is the color of an intense
tan. Ps-100 / min., rhythmic, weak filling, blood pressure-70/30 mm Hg. blood sugar-3.0
mmol / l. For the treatment of this condition is used:
100% Administration of saline sodium chloride and large doses of hydrocortisone
0% Intravenous administration of epinephrine and other adrenomimetics
0% Morphine injection for pain relief
0% Administration of phentolamine or tropafen
0% Introduction of nitroglycerin, diuretics

175.A 28-year-old patient developed severe weakness, dizziness,


darkening of the eyes, nausea and loss of consciousness without convulsions after stress. Objectively: the patient is unconscious, her skin
is pale, and her limbs are cold. Pupillary and tendon reflexes are preserved. AT-80/50 mm Hg,
Ps102 / min., low filling. What is the most likely diagnosis?
100% Syncopal state
0% Epileptic syndrome
0% A hysterical fit
0% Vegetative-vascular paroxysm
0% Transient cerebral circulatory disorders

https://translate.yandex.com/en/doc 39/540
22:20 ,27.6.2023 �� C : ; 5 B

176.as a result of the explosion of a benzene tank at a chemical plant, a


large number of people were killed and injured (more than 50 people) with burns, mechanical injuries and poisoning. Specify
the main elements that provide medical evacuation support for the population in this
situation:
100% Triage, medical care, evacuation
0% Sorting, evacuation, treatment
0% Medical assistance, evacuation, isolation
0% Isolation, rescue, recovery
0% Sorting, restoring, saving

177. A 64-year-old diabetic patient was treated with metformin. Due to high
blood pressure, I took diuretics. Gradually, the effectiveness of the diuretic decreased,
nausea and vomiting appeared. Contact with the patient is difficult. The skin is dry.
There is no smell of acetone. AT - 180/100 mm Hg. heart sounds are deaf. Ps-98 / min. Respiration is vesicular. The abdomen
is painful in the epigastrium. Liver + 4 cm. Blood glucose 48 mmol/ l, No. 156 mmol/l,k - 5,2 mmol/l,
urea - 15 mmol/l. what pharmacological approach should I start with?
100% Emergency hydration of the patient
0% Correction of Na + content in the blood
0% Purpose of sodium bicarbonate

41

Downloaded from the site - online testing step

0% Correction of blood pressure with sodium sulfate


0% Transfer the patient to protamine-zinc insulin treatment

178.A 29-year-old female patient applied to the district obstetrician-gynecologist with complaints of
irregular scanty menstruation, rapid fatigue, dizziness, and weight gain of
15 kg over the last year and a half after giving birth. Preeclampsia occurred during pregnancy. What
is the most likely pathological syndrome in the patient?
100% Postpartum neuroendocrine syndrome
0% Postpartum hypopituitarism
0% Premenstrual syndrome
0% Menopausal syndrome
0% Neurotransmitter-endocrine syndrome not related to pregnancy

179. A 59-year-old patient had an attack of chest pain that lasted more than 1.5 hours.
Objectively: the patient is sluggish, adynamic, skin pale, limbs cold, pulse
of weak filling, heart rate-120 / min., AT-70/40 mm Hg on the ECG: ST elevation in leads II, III,
aVF. What condition is characterized by these changes?
100% Cardiogenic shock
0% Arrhythmogenic shock
0% Breakthrough stomach ulcer
0% Acute pericarditis
0% Acute pancreatitis

180. Ibspi A 7-year-old child was hospitalized with complaints of fever up to 39.8°C,
lethargy, moderate headache, and vomiting. Examination revealed meningeal symptoms.
A lumbar puncture was performed. A liquid under high pressure, transparent, cytosis of 450
cells in 1 µl (mainly lymphocytes-90%), glucose content of 2.6 mmol/l was obtained. what pathogen can
cause the disease in a child?

https://translate.yandex.com/en/doc 40/540
22:20 ,27.6.2023 �� C : ; 5 B
100% Enterovirus
0% Meningococcus
0% Tuberculosis bacillus
0% Staphylococcus aureus
0% Pneumococcus

181. During self-examination, a 28-year-old female patient discovered a tumor in the upper outer
quadrant of the right breast. During palpation - painless, solid, mobile formation
of the breast with a diameter of 2 cm, peripheral lymph nodes are not changed. During
ultrasound examination of the mammary glands: in the upper outer quadrant of the right
mammary gland, a volume formation of increased echogenicity, measuring 21x18 mm. What is the most
likely diagnosis?
100% Fibroadenoma
0% Breast cyst
0% Diffuse mastopathy
0% Breast cancer
0% Mastitis

42

Downloaded from the site - online testing step

182. an 18-year-old patient has a high fast pulse. AT-120/70 mm Hg in the III intercostal space to the left
of the sternum margin - systolic tremor. The left border of the heart is shifted 1 cm outwards. In the II-III
intercostal spaces on the left, there is a systolic-diastolic murmur, which increases during systole, and an accent of the second tone.
What is the most likely diagnosis?
100% Open Ductus arteriosus (Botal duct)
0% Aortic coarctation
0% Ventricular septal defect
0% Atrial septal defect
0% None of the above

183. a 47-year-old female patient complains of itching, burning sensation in the external
genitals, and a scratched urinal. The symptoms have been bothering me for the last 7 months. Menstruation is irregular,
1 time in 3-4 months. Over the past 2 years, I have been worried about hot flashes, increased sweating,
and sleep disorders. After examination, no pathological changes on the part of the internal genitalia were
detected. General blood and urine tests without pathological changes. In a smear from the vagina-
20-25 white blood cells in the subcutaneous cavity, mixed flora. What is the most likely diagnosis?
100% Menopausal syndrome
0% Cystitis
0% Trichomonas colpitis
0% Vulvitis
0% Bacterial vaginosis

184. A 23-year-old patient developed anuria lasting for 5 days after consuming brake fluid
; the creatinine level increased to 0.569 mmol/l. what is the treatment strategy in this
case?
100% Hemodialysis
0% Detoxification therapy
0% Antidote therapy
0% Diuretics
0% Plasmapheresis

https://translate.yandex.com/en/doc 41/540
22:20 ,27.6.2023 �� C : ; 5 B

185. the majority of TV and radio station workers complain of headache, dizziness,
fatigue, decreased performance, adynamia, sleep disorders, and unpleasant sensations in
the heart. What is the effect of this factor in this case?
100% Electromagnetic radiation
0% Infrasound
0% Ultrasound
0% Noise
0% Microclimate

186. on the 7th day after delivery, a 28-year-old woman complained of pain in
her right breast, fever up to 39° C, chills, and deterioration of her general condition. On examination
, an infiltrate is determined in the upper-outer quadrant of the breast with
a softening area. What are the doctor's tactics?
100% Surgical treatment

43

Downloaded from the site - online testing step

0% Antibacterial therapy
0% Discontinuation of lactation
0% Dynamic monitoring
0% Physical therapy treatment

187. A 24-year-old woman with a pregnancy term of 16 weeks applied for an appointment due to
a history of reproductive losses (stillbirth, early infant death).
The study was not conducted due to a burdened medical history. In the phenotype of a pregnant woman, pay
attention: high growth, developed mammary glands. In cytogenetic studies, the karyotype
of a woman is 45, X / 46, XX/47, XXX. Ultrasound examination of the fetus revealed marker signs
of chromosomal pathology. What are the tactics of managing a pregnant woman?
100% Implementation of invasive methods of prenatal diagnostics (amniocentesis)
Ultrasound somatogenetic examination of the fetus with
0%
syndromological analysis
0% Conducting a cytogenetic study of men
0% Conducting a molecular cytogenetic study
0% Termination of pregnancy

188. A 46-year-old patient complains of intense sharp pain in the right side
radiating to the right thigh and perineum. Notes frequent urge to urinate, urine the color
of "meat slop". This condition is described for the first time. Positive Pasternatsky symptom on the right.
What is the most likely diagnosis?
100% Urolithiasis
0% Acute appendicitis
0% Acute pyelonephritis
0% Acute cholecystitis. Renal colic
0% Acute pancreatitis

189. A 34-year-old female patient complains of pain and enlargement of the right breast. Suffers
from infertility for 15 years. The right mammary gland is enlarged, its skin is pasty,
hyperemic, palpable infiltrate of a dough-like consistency without clear contours; a symptom
of "lemon peel". What is the most likely diagnosis?

https://translate.yandex.com/en/doc 42/540
22:20 ,27.6.2023 �� C : ; 5 B
100% Infiltrative cancer
0% Mastitis
0% Nodular mastopathy
0% Breast abscess
0% Breast fibroadenoma

190. A 73-year-old patient went to the doctor complaining of growing weakness, dizziness,
epigastric pain, and belching. In the blood: hyperchromia, moderate cytopenia.
After the examination, the diagnosis was made: B12-deficient anemia. What changes
are most likely detected by the doctor during the stomach examination?
100% Atrophic gastritis
0% Cardia insufficiency, erosive reflux-esophagitis
0% Multiple erosions of the stomach

44

Downloaded from the site - online testing step

0% Polyp of the antrum of the stomach


0% Hypertrophic gastritis

191. A 37-year-old logger patient developed fever, high


body temperature, and headache at the end of May. On the 6th day, hiccups, weakness of the hands, head sank on
the chest joined. In the neurological status, paresis of the hands was found with muscle atony, areflexia. My head
hangs down. What is the preliminary diagnosis?
100% Tick-borne encephalitis
0% Acute polio
0% Ischemic stroke
0% Spinal cord tumor
0% Shoulder plexopathy

192. A 58-year-old patient has severe headache, dizziness, and nausea. AT-200/110 mm
Hg focal neurological symptoms are absent. There are no meningeal signs. What
did the patient experience?
100% Hypertensive crisis
0% Acute hypertensive encephalopathy
0% Sympatho-adrenal crisis
0% Hemorrhagic stroke
0% Subarachnoid hemorrhage

193. a 29-year-old man complains of daily heartburn that occurs after eating, leaning
forward, lying down, belching sour, coughing. These complaints have been bothering
me for 4 years. Objective status and laboratory parameters are normal. With FGDS
, catarrhal esophagitis was detected. Leading in the occurrence of this disease is:
100% Insufficiency of the lower esophageal sphincter
0% Hypersecretion of hydrochloric acid
0% Duodenal-gastric reflux
0% Hypergastrinemia
0% Helicobacter pylori infection

194. A 22-year-old female patient went to the antenatal clinic complaining about the absence

https://translate.yandex.com/en/doc 43/540
22:20 ,27.6.2023 �� C : ; 5 B
of menstruation for 7 months. From the anamnesis: at an early age, she was ill with childhood infections and
sore throats, menarche from the age of 13, regular monthly periods, menstrual cycle of 28 days, menstruation
lasts 5-6 days, painless. 7 months ago I suffered stress. Gynecological examination
revealed no changes in the uterus and appendices. What is the most likely diagnosis?
100% Secondary amenorrhea
0% Primary amenorrhea
0% Algodismenorrhea
0% Oligomenorrhea
0% Fake amenorrhea

195. A 46-year-old woman was admitted to the gynecological department with complaints of lower abdominal pain

45

Downloaded from the site - online testing step

on the right side, in the lower back and sacrum, constipation. During bimanual examination: the uterus is enlarged
according to 10 weeks of pregnancy, with an uneven surface, motionless.
There are atypical cells in the uterine aspirate. What diagnosis can be made?
100% Cancer of the uterine body
0% Cervical cancer
0% Uterine fibromyoma
0% Colon cancer
0% Chorionic epithelioma

196. a 24-year-old woman in labor has a postpartum period, on the 4th day. The delivery was the first, timely,
without complications. The general condition of the woman in labor is satisfactory. Body temperature 36.6°C. Ps-78 / min,
rhythmic. Mammary glands in a state of engorgement. The bottom of the uterus is 2 cm below the navel. The uterus is in
normal condition, painless. Lochia are bloody, moderate. Stool, urination is normal. What
treatment is appropriate?
100% Uterotonic drugs
0% Analgesics
0% Antibiotics
0% Estrogenic hormones
0% Medications that stop lactation

197. A 14-year-old girl refused to eat food in order to lose weight, and sometimes
vomited or used laxatives after eating. Overly irritable, explosive, prone
to hysterical reactions. There are pronounced gastrointestinal disorders, amenorrhea. Identify
a mental disorder:
100% Mental anorexia
0% Hysterical neurosis
0% Somatic disorder
0% Dysphoric disorder
0% Psychogenic depression

198. A 32-year-old patient was admitted to an antitubercular dispensary due to changes detected
on fluorography: S1 of the right lung has a shadow up to 1 cm in diameter, of weak
intensity with indistinct contours. The tomogram shows destruction in the center of the shadow.
MBT was detected in sputum. The patient was diagnosed with focal tuberculosis. What
phases of the tuberculosis process correspond to the identified changes?
100% Infiltration and decay

https://translate.yandex.com/en/doc 44/540
22:20 ,27.6.2023 �� C : ; 5 B
0% Infiltrations and contamination
0% Resorption and scarring
0% Decay and contamination
0% Compaction and resorption

199. A 47-year-old patient is concerned about a tumor-like formation on the anterior surface of the neck, and notes
a progressive increase in the formation. Objectively: in the right lobe of the thyroid gland
there is a formation with a diameter of about 4 cm with a smooth surface, mobile, high
density. Slightly expressed symptoms of thyrotoxicosis. Which of the additional methods

46

Downloaded from the site - online testing step

is the examination the most informative way to clarify the diagnosis?


100% Puncture biopsy
0% Neck radiography
0% Ultrasound examination
0% Determination of thyroid hormones
0% Determination of protein-bound iodine

200. In the structure of the region's population, the proportion of people aged 0-14 years is 25%, and the proportion
of people aged 50 years and older is 30%. What is the most accurate definition of this demographic
situation?
100% Regressive type of population age structure
0% Progressive type of population age structure
0% Cohort type of population
0% Stationary type of population age structure
0% Population immigration

201. A 12-year-old boy complains of periodic headaches, palpitations, irritability,


and intolerance to public transport. Complaints have been bothering me for 2 months. During this time, three
attacks of rising blood pressure to 140/100 mm Hg were noted, this was accompanied by an increase
in temperature to 38oC, tachycardia, tremor, and a feeling of fear. The attack ended with significant
urination. What is the most likely diagnosis?
100% Autonomic dysfunction
0% Epilepsy
0% Congenital heart disease
0% Brain tumor
0% Myocarditis

202. A 6-year-old boy was given donor immunoglobulin due to


a family contact with viral hepatitis. according to the vaccination schedule, a revaccination against measles was planned for this time
. The child was examined by the district pediatrician and found healthy. What is the reason
why the immunologist did not give permission for vaccination?
100% Ineffectiveness of vaccination against the background of administered immunoglobulin
0% Possible anaphylactic reaction to the vaccine
0% High probability of getting measles after vaccination
0% It is possible to develop viral hepatitis against the background of vaccination against measles
0% -

https://translate.yandex.com/en/doc 45/540
22:20 ,27.6.2023 �� C : ; 5 B
203. A 45-year-old patient complains of hoarseness of voice, which continues for 7 years after
undergoing thyroid surgery. Objectively: the laryngeal mucosa is pink,
the vocal folds are light. During phonation, only the left vocal fold moves, the right vocal
fold 100%
is in the middle position.
Right-sided What is paresis
laryngeal the most likely diagnosis?

0% Neoplasms of the right vocal fold


0% Left-sided laryngeal paresis

47

Downloaded from the site - online testing step

0% Laryngeal stenosis in the compensation stage


0% Laryngeal stenosis in the stage of incomplete compensation

204. a 22-year-old patient 5 weeks after hypothermia is concerned


about fever, weakness, muscle pain, and inability to move independently. Objectively: soreness,
tightness of the muscles of the shoulders, shins; active movements are minimal; chest erythema in front.
Periorbital edema with heliotropic erythema. Gottron's positive symptom. What
research should be done to verify the diagnosis?
100% Muscle biopsy
0% Aminotransferase activity
0% Radiography of joints
0% ADL caption
0% Rheumatoid factor

205. a 32-year-old patient complains of epigastric pain that occurs at night and in the morning on an empty stomach and
2 hours after eating, vomiting "coffee grounds", heartburn, periodic dizziness,
general weakness. Objectively: pain in the gastroduodenal zone, positive
Mendel's symptom. In the blood: HB-90 g / l; leuc.- 8,0-109/ l; WSE-20 mm/hour. What has complicated the course
of the disease?
100% Gastrointestinal bleeding
0% Penetration
0% Perforation
0% Stenosis
0% Malignization

206. A 24-year-old patient went to the doctor complaining of enlarged submandibular


lymph nodes. Objectively: enlarged submandibular, axillary and inguinal lymph nodes.
Chest X-ray shows enlarged mediastinal lymph nodes. In the blood: Er -3.4 •
1012/l, hL-100 g/l, CP-0.88, tr.-190- 109/ l, lake.- 7,5- 109/l, e, - 8%, p, - 2%, c. - 67%, lymph.- 23%,
ESR-22 mm / h. What research is indicated to verify the cause of lymphadenopathy?
100% Open lymph node biopsy
0% Ultrasound examination of the abdominal organs
0% Tomography of the mediastinum
0% Puncture biopsy of lymph nodes
0% Sternal puncture

207. in a 35-year-old patient, on the 8th day after surgical treatment of the purulent focus, the wound
was cleared of purulent-necrotic contents, granulations appeared. Nevertheless, against the background
of antibacterial therapy, the body temperature is kept at 38.5-39.5 oC. Chills,

https://translate.yandex.com/en/doc 46/540
22:20 ,27.6.2023 �� C : ; 5 B
sweating,
100% euphoria, Ps-120 / min. What complication of the local purulent-inflammatory process
Sepsis
can you think about?
0% Purulent-resorptive fever
0% Thrombophlebitis
0% Meningitis

48

Downloaded from the site - online testing step

0% Pneumonia

208. a dyer of a car factory was diagnosed with acute poisoning


with moderate benzene amide compounds. After inpatient treatment, there is a significant
improvement in the condition. What expert decision should be made in this case?
100% Issue a professional bulletin for outpatient treatment
0% It can continue working in compliance with sanitary and hygienic standards.
0% Issue a disability certificate for outpatient treatment
0% Send it to the MSEC to determine the percentage of professional disability loss
Send it to the MSEC to determine the disability group due to an occupational
0%
disease

209. A 20-year-old female patient went to a women's clinic with complaints of itching,
burning and pain in the external genital area, fever,
irritability, sleep disturbance, headache. The patient is unmarried, has a promiscuous sex life.
When examining the external genitalia against the background of hyperemic, edematous mucosa
, vesicles are determined, a group of which occupies up to 2 cm of the affected surface. What is the most likely
cause that determines this clinical picture?
100% Genital herpes infection
0% Papillomavirus infection
0% Candylomatosis
0% Primary syphilis
0% Cytomegalovirus infection

210. A 51-year-old patient underwent successful cardiopulmonary resuscitation after a suicide


attempt by hanging. After 1.5 hours: unconscious, blood pressure-130/90 mm Hg, Ps-130 / min.
Respiratory failure increases: BH-28 / min, dyspnea of an inspiratory nature, retraction
of the supraclavicular pits on the inspiration, the wings of the nose participate in breathing. Auscultation:
rough whistling above the trachea, hard breathing above the lungs. What is the main cause of respiratory failure in a
patient?
100% Damage to the laryngeal cartilage
0% Aspiration syndrome
0% Neurogenic pulmonary edema
0% Bronchial obstruction syndrome
0% Respiratory center depression

211.A 40-year-old woman went to the women's clinic complaining of


menstrual irregularities such as hyperpolymenorrhea for six months, pulling pains in the lower
abdomen, and weakness. During gynecological examination, the body of the uterus is enlarged up to 12 weeks
of pregnancy, dense, mobile, painless. In the blood: Hb-90 g / l. what pathology can be suspected?
100% Uterine fibroids
0% Cancer of the uterine body
0% Pregnancy
https://translate.yandex.com/en/doc 47/540
22:20 ,27.6.2023 �� C : ; 5 B

0% Ovarian cysts
0% Dysfunctional uterine bleeding

49

Downloaded from the site - online testing step

212. A 25-year-old patient complains of a significant decrease in vision in the right eye, which occurred suddenly.
On examination by an optometrist: signs of retrobulbar neuritis. Objectively: abdominal reflexes
are absent, vibration sensitivity is reduced. Make a diagnosis:
100% Multiple sclerosis
0% Amyotrophic lateral sclerosis
0% Funicular myelosis
0% Brain tumor
0% Optic nerve atrophy

213. A 26-year-old repeat mother arrived at the maternity ward due to a 40


-week pregnancy and the onset of labor. 2 hours ago, the amniotic fluid left.
Fetal position is longitudinal, main presentation. Coolant - 100 cm, VDM-42 cm. Contractions
in 4-5 minutes, 25 seconds each. During internal obstetric examination: the cervix
is smoothed, the opening is 4 cm. There is no fetal bladder. The fetal head is pressed against the entrance to the small
pelvis. What complication occurred during childbirth?
100% Premature discharge of amniotic fluid
0% Primary weakness of labor activity
0% Secondary weakness of labor activity
0% Discoordinated labor activity
0% Clinically narrow pelvis

214. A 38-year-old patient was admitted to the narcological department with complaints of insomnia and anxiety.
In the evening, he became restless, jumped out of bed, shook off "spiders" from blankets and clothes, saw
"healthy rats and snakes" everywhere, and drove away non-existent flies. He believes that he is at home, hears "voices
of gypsies" outside the window, and wants to run away to "drive them out of the garden". Somatic condition: temperature-37.8 ° C,
severe hyperhidrosis, tremor of the whole body. What is the most likely diagnosis?
100% Alcoholic delirium
0% Pathological intoxication
0% Alcoholic hallucinosis
0% Acute alcoholic paranoid
0% Hysterical twilight state

215. a four-year-old boy spent the whole day at the beach. In the evening, the child appeared:
headache, weakness, vomiting. On objective examination: the face is hyperemic, body temperature
is 38.8 oC, BH-28 / min, heart rate-130/min. The most likely cause of this condition is:
100% Sunstroke
0% Anaphylactic shock
0% Syncopal state
0% Dizziness
0% Collapse

216. A 13-year-old girl with duodenal ulcer is registered at


a dispensary for a year. What should be the period of medical examination after
an exacerbation of peptic ulcer disease?
100% 5 years old
https://translate.yandex.com/en/doc 48/540
22:20 ,27.6.2023 �� C : ; 5 B

50

Downloaded from the site - online testing step

0% 4 years old
0% 3 years old
0% 2 years old
0% 1 year old

217. a patient with lymphogranulomatosis has enlarged cervical-supraclavicular lymph nodes on the right side.
General condition is satisfactory, no sweating, body temperature 36.7 oC. What treatment should
be prescribed?
100% Radiotherapy
0% Combined chemotherapy
0% Combined chemotherapy + radiation therapy
0% Monochemotherapy
0% Monochemotherapy + radiotherapy

218. a 35-year-old man complains of growing weakness, palpitations, "butterflies" in front


of his eyes, and dizziness. History: peptic ulcer of the stomach. Objectively: the skin is pale. In
the lungs-vesicular respiration. There is a systolic murmur above the apex of the heart. Ps-100 / min, AT-100/70
mm Hg Palpation-slight pain in the epigastrium. In the blood: Er.- 3,2 • 1012/ l, Hb-100 g / l,
CP-0.95. what is the nature of anemia in this case?
100% Post-hemorrhagic anemia
0% Sideroblastic anemia
0% Chronic iron deficiency anemia
0% Hemolytic anemia
0% Hypoplastic anemia

219. low-fat kefir was delivered to the hospital's food hall. The jar lid shows the date of its
storage until delivery. Specify the optimal storage time (hours) for kefir:
100% 36
0% 40
0% 20
0% 50
0% 60

220. Give an assessment of the physical development of a 12-year-old girl using the si-gm deviation method,
if the body weight indicator is +0.5 Sigma:
100% Average
0% Above average
0% High
0% Low
0% Below average

221. during the medical examination, the patient was found to have stunted growth, hypogonadism, sexual
infantilism, lack of secondary sexual characteristics, enlarged liver and spleen. This
indicates a lack of nutrition of such an element.:

51

https://translate.yandex.com/en/doc 49/540
22:20 ,27.6.2023 �� C : ; 5 B

Downloaded from the site - online testing step

100% Zinc
0% Calcium
0% Copper
0% Iodine
0% Vitamin D

222. A 30-year-old patient went to a dermatologist after being employed with complaints
of rashes all over her skin. Over the past 3 months, he has noticed a sharp weight loss,
general weakness, and constant subfebrility. For what disease is it advisable
to examine the patient?
100% AIDS
0% Syphilis
0% Tuberculosis
0% Cytomegalovirus infection
0% Malignant neoplasms of the skin

223. A 12-year-old girl lives in unsatisfactory social conditions. Complains


of periodic intermittent abdominal pain, constipation, rapid fatigue. The child is emotionally
labile. Palpation of the abdomen reveals soreness along the colon, spasmodic
sigmoid colon. Colonoscopy revealed no changes in the colon mucosa
. Coprological examination: fecal masses are fragmented, with admixtures of a small
amount of mucus, do not contain food residues or inflammatory products. What is the most likely
diagnosis?
100% Irritable bowel syndrome
0% Paralytic intestinal obstruction
0% Non-specific ulcerative colitis
0% Dysentery
0% Crohn's disease

224. the child is 7 years old. Complains of paroxysmal pain, which occurs after
mental stress, the use of cold drinks, ice cream. After
a clinical and instrumental examination, a diagnosis was made: gall bladder dyskinesia of the
hypertensive type. Which groups of drugs should be prescribed first for treatment?
100% Antispasmodics and choleretics
0% Choleretics and cholekinetics
0% Sedatives and cholekinetics
0% Antioxidants
0% Antibiotics

225. A 36-year-old woman has been suffering from hay fever for 7 years. For the last 2 years, in
August (ragweed flowering period), the patient notes 2-3 attacks of suffocation, which are eliminated by 1
dose of salbutamol objectively: temperature -36.5 oC, BH - 18/min., Ps-78 / min., AT-115/70 mm Hg
over the lungs-vesicular respiration. The heart tones are sonorous, the rhythm is correct. Which
drug will be most effective in preventing suffocation attacks during the critical
season for the patient?

52

https://translate.yandex.com/en/doc 50/540
22:20 ,27.6.2023 �� C : ; 5 B

Downloaded from the site - online testing step

100% Intala inhalations


0% Berotek inhalations
0% Atrovent inhalations
0% Taking suprastin
0% Teopek's reception

226. A 56-year-old patient has non-insulin-dependent diabetes. The disease is compensated by diet and
glucoform. The patient is waiting for surgery for panaritium. Tactics of hypoglycemic
therapy?
100% Don't change your therapy
0% Cancel the glyrenorm
0% Prescribe monocomponent insulin
0% Prescribe recombinant insulin
0% Assign a Manil

227. A 26-year-old man complains of thirst, increased urination, general weakness,


and weight loss. Objectively: dry skin, red cheeks, vesicular breathing. The heart tones
are sonorous. The tongue is dry. There are no symptoms of peritoneal irritation. Which study is the most
informative for clarifying the diagnosis?
100% Blood sugar test
0% General blood test
0% General urinalysis
0% Zimnitsky urine test
0% Blood test for liver tests

228. a 9-year-old child has developed pain in the upper third of the right shin, the temperature has risen
to 39oc, and cannot stand on his foot. He had a history of a shin injury and suffered a sore throat. What
is the most likely diagnosis?
100% Acute hematogenous osteomyelitis
0% Broken bone
0% Acute rheumatism
0% Tuberculous osteomyelitis
0% Malignant tumor

229. a 32-year-old woman complains of pain in the left hypochondrium, which appears 2 hours
after eating, nausea, bloating, and a tendency to diarrhea. Objectively: scleral subictericity.
The abdomen is painful on palpation at the Hubergritz-Skalsky point. Liver-at the edge of the costal
arch. In the blood: amylase-288 mmol / l, total bilirubin-20 mmol / l. what
is the most likely disease?
100% Chronic pancreatitis
0% Chronic hepatitis
0% Chronic enterocolitis
0% Chronic cholecystitis
0% Chronic gastritis

53

Downloaded from the site - online testing step

https://translate.yandex.com/en/doc 51/540
22:20 ,27.6.2023 �� C : ; 5 B
230. A 50-year-old patient complains of bursting pain in the left lower limb, which
increases with physical exertion, and swelling in the lower leg and foot. Objectively:
the left shin and foot are pasty, the skin in the lower part of the shin is induced, bronze
-colored, dilated subcutaneous veins, there is an ulcer with necrotic masses. What is the most
likely diagnosis?
100% Post-thrombophlebitic syndrome
0% Chronic arterial insufficiency
0% Acute arterial thrombosis
0% Deep vein thrombosis of the lower extremities
0% Gangrene of the lower extremity

231. a child from the age of 10 has rheumatism. Suffered 2 attacks. Specify measures
for secondary prevention of rheumatism in the child:
100% Year-round bicillin therapy for 5 years
0% Appointment of immunocorrective agents twice a year
0% Courses of nonsteroidal anti-inflammatory drugs in spring and autumn
0% Year-round bicillin therapy for 3 years
0% Cardiotrophics + vitamins in spring and autumn

232. A 43-year-old woman complains of pain in the wrist and interphalangeal


joints of the hands and feet, stiffness of movement in the morning. He has been ill for 7 years. The disease began with
a symmetrical lesion of the small joints of the hands and feet. There is deformity of the hands and
feet, atrophy of the interosseous muscles, edema and inflammation of the wrist joints;
their movements are limited and sharply painful. Internal organs without special features. Which indicator is most
informative for making a diagnosis?
100% Rheumatoid factor
0% Leukocytosis
0% Accelerated SSE
0% Dysproteinemia
0% C-reactive protein

233. a 35-year-old woman complains of pain in the heart area ("pinching, drilling"), which occurs
mainly in the morning hours in the autumn-spring period, with pain radiating to the neck, back, stomach;
rapid heartbeat, as well as a decrease in overall vitality. The occurrence of this condition is not
related to physical activity. In the evening, the condition improves. Somatic, neurological
status and ECG - without pathology. What is the most likely pathology that caused this clinical picture?
100% Somatized depression
0% Resting angina pectoris
0% Neurosis-like schizophrenia
0% Neurocirculatory dystonia
0% Hypochondriac depression

234. the parents of a 7-year-old boy went to the doctor complaining that the child
had been suffering from polyuria, thirst, and weight loss of up to 4 kg for the last 2-3 weeks. Objectively: the skin is pale,
dry, the cheeks are flushed, the lips and tongue are dry, the tissue turgor is reduced, the smell of acetone from the mouth.

54

Downloaded from the site - online testing step

What is the most likely diagnosis?


100% Diabetes mellitus

https://translate.yandex.com/en/doc 52/540
22:20 ,27.6.2023 �� C : ; 5 B
0% Diabetes insipidus
0% Acetonemic syndrome
0% Kidney diabetes
0% Alport Syndrome

235. A student went to the doctor with complaints of insomnia, irritability, and palpitations. My mother
has hypertension. During the examination: sweating of the palms,small tremor of the hands,
persistent red dermographism. In the lungs-vesicular respiration, heart boundaries are normal, heart
activity is rhythmic, 80/min., AT-150/65 mm Hg. what is the most likely diagnosis?
100% Hypertonic VSD
0% Thyrotoxicosis
0% Hypertension
0% Pheochromocytoma
0% Nodular periarteritis

236. In the area served by the children's polyclinic, 4,800 children and
adolescents were vaccinated, 4,800 children were examined, 4,320 people were given preventive vaccinations
, and temporary medical contraindications were identified in 480 children. What is the value of the indicator "full coverage
of preventive vaccinations of children and adolescents who are subject to vaccination"in this area?
100% 90%
0% 60%
0% 70%
0% 80%
0% 100%

237. A 20-year-old patient complains of severe headache, double vision, general


weakness, fever, irritability. Objectively: the body temperature is 38.1 oC, it comes into
contact reluctantly, reacts painfully to stimuli. Ptosis of the left eyelid, divergent strabismus,
anisocoria S > D. Severe meningeal syndrome. During lumbar puncture, the cerebrospinal fluid flowed out under
a pressure of 300 mm water. st., transparent, with a slight opalescence, a fibrinous film fell out after a day
. Protein -1.4 g / l, lymphocytes-600/3 in mm3, sugar-0.3 mmol/l. what preliminary diagnosis
should be made for the patient?
100% Tuberculosis meningitis
0% Meningococcal meningitis
0% Armstrong lymphocytic meningitis
0% Syphilitic meningitis
0% Mumps meningitis

238. a man experienced vomiting,weakness, shortness of breath, dry mouth, and dizziness 16 hours after eating dried fish and mushrooms
. Stool retention. The next
day, the patient was hospitalized in serious condition with a rapid pulse at normal
body temperature, with complaints of poor vision and double vision. What is the most likely diagnosis?
100% Botulism

55

Downloaded from the site - online testing step

0% Food toxicoinfection
0% Staphylococcal toxicosis
0% Opisthorchiasis
0% Mushroom poisoning
https://translate.yandex.com/en/doc 53/540
22:20 ,27.6.2023 �� C : ; 5 B

239. the head of a newborn has a dolichocephalic shape, elongated from front to back. During
the examination of the head on the occipital part, a generic tumor is determined, located in the middle
between the large and small crown. At what presentation of the fetal head did labor occur?
100% Posterior view of occipital presentation
0% Anterior view of occipital presentation
0% Anteroparietal presentation
0% Frontal presentation
0% Facial presentation

240. in a patient with suspected pheochromocytoma, blood pressure is within normal limits in the period between attacks
, there is a tendency to tachycardia. There is no pathology in the urine. It was decided to conduct
a provocative test with histamine. What drug should be prepared for emergency
treatment in case of a positive test result?
100% Phentolamine
0% Pipolfen
0% Nifedipine
0% Mezaton
0% Prednisone

241. a 54-year-old patient complains of frequent painful urination, chills, and an increase
in body temperature up to 38oC. In the urine: protein-0.33 g / l, white blood cells up to 5060 v / s, red blood cells-5-8
v / s, gram-negative rods. Which of the presented antibacterial drugs is best suited in
this case?
100% Ciprofloxacin
0% Oxacillin
0% Erythromycin
0% Tetracycline
0% Tseporin

242. therapists of the city hospital serve the population in 3 units during the day: reception,
site, hospital. Every 2 years, internists are sent to the hospital for 3-4 months
. What is the system of the site where doctors work?
100% On duty
0% Trilankova
0% Dvulankovaya street
0% Territorial section
0% Shop precinct

243. during the examination after birth, a child has a brachycephalic skull shape,

56

Downloaded from the site - online testing step

deformity of the auricles in the form of a protruding protivozavitki, epicanthus, oblique incision of the eyes,
shortening of the little finger, bilateral transverse fold on the palm. During auscultation of the heart
, a rough systolic murmur is heard with a maximum of 3-4 intercostal spaces along the left edge
of the sternum. What congenital heart disease most often accompanies this pathology?
100% Disadvantages of Falo
0% Aortic coarctation

https://translate.yandex.com/en/doc 54/540
22:20 ,27.6.2023 �� C : ; 5 B
0% Pulmonary artery stenosis
0% Open Ductus arteriosus
0% Fibroelastosis

244. A 40-year-old woman was hospitalized in a clinic due to an exacerbation of chronic


obstructive bronchitis. Complains of shortness of breath during physical exertion, cough with
sputum, swelling of the lower extremities. Objectively: temperature-37.2 oC, BH-24 / min,
Ps90 / min, rhythmic, blood pressure-110/70 mm Hg. Accent of the second tone over the pulmonary artery. Palpated
liver enlarged by 4 cm. What ECG changes are most likely to occur in the patient?
100% Right ventricular hypertrophy
0% Atrial fibrillation
0% Left bundle branch block
0% Left ventricular hypertrophy
0% Abnormal Q wave

245. the chief doctor of the polyclinic gave the task to the district doctor to determine
the pathological incidence of the population with the nth disease at the site. According to what document
is the population pathologically affected by diseases at the medical site determined?
100% Professional review Log
0% Statistical coupons ( + )
0% Statistical coupons ( -)
0% Statistical coupons (+) and ( -)
0% Doctor's appointment cards

246. The district doctor made a decision


to transfer the patient to a disability after a long dispensary observation. Who will refer the patient to the medical and social
expert commission?
100% Medical Advisory Commission
0% District doctor
0% Doctor of a narrow specialty
0% Head of the polyclinic Department
0% Head of the inpatient Department

247. A 44-year-old man was admitted to the infectious diseases department with a diagnosis of follicular angina.
Body temperature -38.6 oC, skin and mucous membranes of normal color. Ps-i02 / min., AT-130/70 mm Hg.
in the blood: Er.- 2,7-1012/ l, Hb-90 g / l, Cp-1,0; lake-38-109/l, blasti-68%, p-2%, c-14%, l-14%,
m-2%, WZ-46 mm/year. What disease should be suspected in the patient?
100% Acute leukemia
0% Leukemoid reaction

57

Downloaded from the site - online testing step

0% Chronic lymphocytic leukemia


0% Chronic myeloid leukemia
0% Acute agranulocytosis

248. A 55-year-old man was admitted to the clinic due to an attack of renal colic, which
recurs periodically throughout the year. Objectively: in the area of the auricles and the right
elbow joint, there are nodules covered with thin, shiny skin.
Ps88 / min AT-170/100 mmHg positive Pasternatsky symptom on both sides. The patient

https://translate.yandex.com/en/doc 55/540
22:20 ,27.6.2023 �� C : ; 5 B
is scheduled for an examination. The study of which laboratory parameter is most appropriate for
establishing a diagnosis?
100% Uric acid
0% Rheumatoid factor
0% SHZ
0% Urine sediment
0% Lactic acid

249. A 55-year-old man has been observed for 1.5 years for viral cirrhosis
of the liver with portal hypertension. Over the past month, weakness has increased
, and coffee grounds-colored vomiting has appeared. Fibrogastroduodenoscopy revealed bleeding from
dilated esophageal veins. What drug should be used to reduce the pressure in
the portal vein?
100% Vasopressin
0% Reserpine
0% Calcium Gluconate
0% Dicynonum
0% Furosemide

250. a 58 - year-old woman complains of gratuitous bruising, weakness, bleeding


gums, and dizziness. Objectively: the mucous membranes and skin are pale, with numerous
hemorrhages of various antiquities. The lymph nodes are not enlarged. Ps-100 / min., AT-110/70 mm Hg.
no changes were detected from the internal organs. In the blood: Er.-3,0-1012/ l, HB-92 g / l, CP-0.9,
anisocytosis, poikilocytosis, leuc.- 10-109/l, e-2%, p-12%, c-68%, l-11%, m-7%, WZ-12 mm / year.
Additional determination of which laboratory parameter is most appropriate for establishing
a diagnosis?
100% Platelets
0% Reticulocytes
0% Blood clotting time
0% Osmotic resistance of red blood cells
0% Phy6rinogen

251. A 25-year-old patient was admitted to the infectious diseases department on day 3 with
complaints of headache, lower back pain, calf muscles, high fever, and chills. The condition
is moderate to severe. Icteric sclera. The pharynx is hyperemic. The tongue is dry, covered with a dry
brown coating. My stomach is swollen. Liver + 2 cm. The spleen is not enlarged. Palpation of the muscles,
especially the calf muscles, is painful. The urine is dark. Normal-colored feces. What is the most likely
diagnosis?

58

Downloaded from the site - online testing step

100% Leptospirosis
0% Viral hepatitis A
0% Malaria
0% Infectious mononucleosis
0% Yersiniosis

252. a patient complains of a sudden onset of the disease: fever up to 39.0 oC, irregular
type, with acute chest pain. Sputum with a putrid smell and blood impurities, up to 400 ml
per day. With percussion: over the affected area, shortening of percussion sound, increased

https://translate.yandex.com/en/doc 56/540
22:20 ,27.6.2023 �� C : ; 5 B
vocal tremor. Anaerobic streptococcus was isolated in sputum. What disease can
100% in Gangrene
be suspected of the lung
the first place?
0% Lung abscess
0% Tuberculosis
0% Bronchiectasis
0% Abscessed pneumonia

253. a 50-year-old patient suddenly developed pain in the occipital region, vomiting. Objectively:
sopor, hyperemia of the facial skin, AT-210 / -20 mm Hg, Ps-60 / min, tense, body temperature
-37.8 oC. Horizontal nystagmus. Reflexes of oral automatism are expressed. Tendon
reflexes are uniform. Stiffness of the occipital muscles, bilateral Kernig's symptom. What
is the preliminary diagnosis?
100% Subarachnoid hemorrhage
0% Hemorrhagic parenchymal stroke
0% Subdural hematoma
0% Acute hypertensive encephalopathy
0% Meningococcal meningitis

254. a 40-year-old patient complains of fever up to 39 ° C, cough with


sputum and blood impurities, shortness of breath, general weakness, herpetic rash on the lips.
Objectively: BH-32 / min. On the right under the shoulder blade - increased vocal tremor, in the same place-
dulling of percussion sound. Auscultation: bronchial respiration. In the blood:
lake.14 * 109/l, WIDTH - 35 mm / hour. What is the preliminary diagnosis?
100% Croupous right-sided pneumonia
0% Focal right-sided pneumonia
0% Cavernous tuberculosis of the right lung
0% Leg cancer
0% Exudative pleurisy

255. a patient underwent gastric resection. During the operation, the patient's left upper limb
was removed and fixed to the operating table to perform anesthesia
support. In the postoperative period, the patient developed functional disorders of the upper
limb in the form of a"hanging hand". What nerve damage caused this
symptom to occur?
100% The radial nerve

59

Downloaded from the site - online testing step

0% Axillary nerve
0% Ulnar nerve
0% The median nerve
0% Musculocutaneous nerve

256. A 37-year-old woman went to the doctor for an exacerbation of chronic hepatitis. An
increase in the level of indirect bilirubin, AsAT, ALT and a decrease in the level
of albumin I prothrombin were found in the blood. Which of the pathological processes most likely caused these changes?
100% Cytolysis
0% Cholestasis
0% Portal hypertension
0% Hypersplenism
https://translate.yandex.com/en/doc 57/540
22:20 ,27.6.2023 �� C : ; 5 B
0% Hemostatic disorders

257. A 63-year-old woman complains of general weakness, a sense of burden, embossment,


epigastric congestion, nausea, and belching after eating. He has been ill for about 15 years. Об’єктивно:
temperature-36.4 oC, BH-20/xb, Ps-88/xb, AT-115/75 mm Hg. skin and mucous membranes are pale. In the blood:
Er.-2,0-1012/ l, HB-100 g/l. antibodies to gastric lining cells were detected. What is the most
likely cause of anemic syndrome in this patient?
100% Production of antibodies to internal factor
0% Violation of hemoglobin synthesis
0% Violation of erythropoietin synthesis
0% Iron malabsorption
0% Increased iron consumption

258. A case of familial food poisoning was characterized by a classic clinical picture
of botulism. The day before, all the patients ate fried eggs, borscht, cutlets, boiled sausage with
canned zucchini caviar, apricot jam, and ice cream. Which product is most
likely to cause poisoning?
100% Canned zucchini caviar
0% Chops
0% Ice cream
0% Boiled sausage
0% Jam

259. a 42-year-old woman suffers from micronodular cryptogenic cirrhosis of the liver. During
the last week, the condition has worsened: convulsions, confusion,
jaundice has increased. What kind of research can explain the reason for the deterioration of the condition?
100% Determination of serum ammonia
0% Determination of cholesterol esters
0% Determination of alpha-fetoprotein content
0% Definitions of AlAT and AsAT
0% Determination of alkaline phosphatase level

60

Downloaded from the site - online testing step

260. a 7-year-old girl complains of weakness, increased fatigue, increased


body temperature up to 38 ° C, a small amount of urine released per day, the color of "meat slop".
Objectively: pallor of the skin, swelling of the face, arms, legs, Lower back pain. In the blood: Er.-2,7-1012/ l,
HB-90 g / l, lake.- 17-109/l, E.-10%, P.-4%, S.-60%, L.-16%, M.-10%, SZE-30 mm / year. In the urine:
leukocyte count - 15 v / s, erythr.- 30 v / s, hyaline cylinders-8-10 v / s, protein-4 g / l. Blood cholesterol
-8 mmol/l, total protein-43 g/l. what is the leading mechanism of edema development?
100% Reduction of oncotic blood pressure
0% Cardiac disorders
0% Diselectrolyte violations
0% Hyperaldosteronism
0% Reduction of osmotic blood pressure

261. a 3-day full-term newborn child has an indirect


bilirubin level of 345 mmol / l, an hourly increase of 6.8 mmol / l. the child's condition is severe: decreased

https://translate.yandex.com/en/doc 58/540
22:20 ,27.6.2023 �� C : ; 5 B
reflexes, muscle hypotension, and limb tremor. The blood of the child and mother is incompatible with the Rh factor.
Which treatmentReplacement
100% method is most effective?
transfusion of single-group and Rh-compatible blood
0% Phototherapy
0% Taking phenobarbital
0% Hemosorption
0% Corticosteroid therapy

262. a 32-year-old patient complains of "heartache", bad mood, lack of appetite,


insomnia. These symptoms appeared gradually over 3 months for no apparent reason. In
mental status: answers the question in a low voice, the face is sad, the facial expressions are mournful,
the movements are slowed down, emotionally depressed, the pace of thinking is slowed down. The patient's condition improves
in the evening and worsens in the morning. What is the intended diagnosis?
100% Manic-depressive psychosis: the depressive phase
0% Reactive depression
0% Cyclothymia
Depressive syndrome due to organic damage to the central nervous
0%
system
0% Involutional melancholy

263. A 57-year-old man notes palpitations, sweating, sleep disturbances, increasing


weakness, and weight loss. He was treated for IHD, but had no effect. Objectively: temperature 36.8 oC,
heart rate-128 / min, Ps-112 / min, arrhythmic, blood pressure-160/70 mm Hg. skin is warm, moist. Tremor of the fingers
of outstretched hands. Heart sounds are amplified, systolic murmur above the apex. The thyroid gland is not
palpable. Which of these studies is most important for clarifying the diagnosis?
100% Study of the level of thyroid hormones in the blood
0% Blood lipid spectrum
0% Dosed exercise test
0% Ultrasound examination of the thyroid gland
0% Ultrasound examination of the heart

61

Downloaded from the site - online testing step

264. A 40-year-old woman suffering from a combined mitral defect with a predominance of stenosis
complains of shortness of breath, attacks of suffocation at night, and heart failure. Currently unable to
do light homework. What is the best patient management strategy?
100% Performing a mitral commissurotomy
0% Implantation of an artificial valve
0% Conducting antiarrhythmic therapy
0% Treatment of heart failure
0% Appointment of antirheumatic therapy

265. A 25-year-old patient complains of aching heart pain for 10


days, shortness of breath with little physical exertion, and palpitations. Got sick 2 weeks ago
after a respiratory infection. Objectively: acrocyanosis, blood pressure-90/75 mm Hg, Ps-96 / min. The borders of the heart
are shifted to the left and right. Heart sounds are weakened, three-term rhythm, systolic murmur at the apex.
ECG: sinus rhythm, complete blockage of the left bundle branch. What is the most likely diagnosis?
100% Infectious and allergic myocarditis
0% Exudative pericarditis

https://translate.yandex.com/en/doc 59/540
22:20 ,27.6.2023 �� C : ; 5 B
0% Infectious endocarditis
0% Myocarditis cardiosclerosis
0% Vegetative-vascular dystonia

266. A 41 — year-old patient went to the doctor on the second day of her illness with complaints of
general weakness, fever up to 38-39 ° C, loose stools up to 4-6
times a day of greenish color with mucus, repeated vomiting, pain in the epigastric region and
near the navel. Objectively: the condition is of moderate severity. The skin is pale. Ps-92 / min., AT-105-75 mm Hg.
the language is overlaid. The abdomen on palpation is painful in the epigastric and right iliac areas.
Which of the tests is most likely to confirm the diagnosis?
100% Bacteriological examination of the stool
0% Coprological examination of the stool
0% Rectoromanoscopy
0% Ultrasound of the abdominal cavity
0% Virological studies of the stool

267. a child is undergoing inpatient treatment for acute staphylococcal


destruction of the right lung. Suddenly there was a sharp pain in the right side of the chest,
shortness of breath, cyanosis. The right side of the chest lags behind in the act of breathing. Percussion
at the bottom right-dullness, in the upper sections - box sound. The boundaries of relative cardiac dullness are shifted
to the left. What complication is most likely to occur?
100% Pyopneumothorax on the right
0% Pleural empyema
0% Spontaneous pneumothorax
0% Exudative pleurisy
0% Right lung abscess

268. A 29-year-old patient underwent primary surgical treatment of a crushed wound in the upper
third of the left thigh. After 2 days, the patient's condition worsened: on examination, a wound with minor injuries was found.

62

Downloaded from the site - online testing step

the muscles are grayish in color, the limb is swollen, the skin is cold, covered with cyanotic
spots, crepitation is determined by palpation. My mind is dizzy. Body temperature
40.3 oC. What complication did the patient have?
100% Anaerobic gas gangrene
0% Hip phlegmon
0% Tetanus
0% Erysipelas
0% Arterial thrombosis

269. A photochronometric follow-up study of the admission of patients by general practitioners of polyclinic No. 1 in
1997 showed that 10.6% of the total time spent on preparation and familiarization with the medical record
, 15.1% on opit,35.9% on examination and examination, and 38.4% on other elements of work of the total time
spent on receiving one patient. What kind of diagrams can be used to visually illustrate
the results of the study?
100% Pie Chart
0% Cartodiagram
0% Line Chart
0% Radial diagram

https://translate.yandex.com/en/doc 60/540
22:20 ,27.6.2023 �� C : ; 5 B
0% Bar chart

270. An X-ray of the lungs shows a compaction and a sharp decrease in the upper lobe
of the right lung. The middle and lower lobes of the right lung are sharply pneumotized. The root of the right lung
is pulled up to the compacted lobe. There are multiple focal shadows in the upper and middle parts of the left pulmonary field
. In the basal area of the left pulmonary field
, ring-shaped shadows with sufficiently thick and uneven walls are clearly contoured. What
disease does the indicated X-ray picture correspond to?
100% Fibrous-cavernous tuberculosis of the lungs
0% Atelectasis of the upper lobe of the right lung
0% Pneumonia with abscess formation
0% Peripheral cancer
0% Pencost Cancer

271. An 11-year-old boy has had acute respiratory diseases five times in the past year
. What health group should your family doctor refer you to?
100% Second group
0% First group
0% Third group
0% Fourth group
0% Fifth group

272. A 50-year-old patient complains of pain in the right inguinal region. When examined in
a standing position, it was found that below the right skin inguinal fold there is a protrusion
measuring 3x3 cm, which in the supine position is exercised under the inguinal ligament,
and the femoral artery pulsation is determined along the outer edge. What is the most likely diagnosis?
100% Femoral hernia

63

Downloaded from the site - online testing step

0% Metastasis of a malignant tumor


0% Inguinal lymphadenitis
0% Inguinal hernia
0% A benign tumor

273. A 59-year-old man complains of heart pain, cough, fever


up to 38oC. I had a myocardial infarction 3 weeks ago. Objectively: Ps-86/min, rhythmic, BP-110/70
mm Hg Auscultation: pericardial friction noise, wet wheezing under the scapula. Radiologically
, there is no pathology. In the blood: lake.- 10 • 109/l, ESR - 35 mm/h. ECG-no dynamics.
Which pharmacological group would be most appropriate for prescribing drugs?
100% Glucocorticoids
0% Antibiotics
0% Direct anticoagulants
0% Nitrates and nitrites
0% Fibrinolytics

274. a patient suffering from diabetes mellitus suddenly had a drooping upper eyelid of the right
eye. The doctor found dilation of the right pupil and divergent strabismus on the right. There were no other changes in
the neurological status. What disease did the patient have?
100% Neuropathy of the right oculomotor nerve

https://translate.yandex.com/en/doc 61/540
22:20 ,27.6.2023 �� C : ; 5 B
0% Diabetic polyneuropathy
0% Acute cerebrovascular accident
0% Brain tumor
0% Secondary meningitis

275. a 22-year-old woman complains of nausea, vomiting once a day, drowsiness, delayed
menstruation for 2 months. Bimanual examination: the uterus is enlarged to the size
of a woman's fist, softened, especially in the isthmus, painless. Appendages are not
palpable. Discharge is slimy, milky in color. What is the most likely diagnosis?
100% Pregnancy 8 weeks
0% Uterine fibroids
0% Ectopic pregnancy
0% Menstrual disorders
0% Endometriosis of the uterus

276. A 17-year-old patient went to the therapist with complaints of poor health, chills,
runny nose, muscle and joint pain, nausea and diarrhea. He asks
for more painkillers and sedatives (tramadol or Solpadein, which help better, and
diazepam). Pharyngeal mucosa is pale pink and clear. In the lungs-vesicular respiration. Tachycardia.
The pupils are dilated, the reaction to light is flabby. On the skin of the forearms-traces of injections. When examined
, he is cheeky, irritated, rude, and deceitful. Make a diagnosis:
100% Opium addiction
0% Addiction to painkillers
0% Substance abuse due to the abuse of tranquilizers
0% Acute respiratory disease

64

Downloaded from the site - online testing step

0% Food toxicoinfection

277. The victim of a car accident was taken to a neurosurgery clinic because
there were signs of a traumatic brain injury. During the examination, symptoms of focal
brain damage were revealed, and a bruise was suspected. What research method should be used in this
case?
100% Computed tomography
0% X-ray of the skull bones
0% Angiography of brain vessels
0% Ultrasound examination of brain vessels
0% Fundus examination

278. A 13-year-old girl complains of an increase in body temperature up to 37.4 oC during


the last 2 months after acute respiratory viral infection. Objectively: lean, diffusely
enlarged thyroid gland (grade II), dense on palpation; exophthalmos, tachycardia. What pathological
syndrome does the patient have?
100% Thyrotoxicosis
0% Hypothyroidism
0% Hypoparathyroidism
0% Hyperparathyroidism
0% Thymomegaly

https://translate.yandex.com/en/doc 62/540
22:20 ,27.6.2023 �� C : ; 5 B
279. A 53-year-old patient has been under the supervision of a district doctor for 5 years for
high blood pressure. During the last month, blood pressure did not
decrease below 160/110 mm Hg Ps-60 / min. On the ECG: signs of hypertrophy of the left ventricle
of the heart. Which of the groups of antihypertensive agents should be prescribed to the patient as pathogenetically
justified?
100% Long-acting dihydroperidins
0% Beta-blockers
0% Drugs from the rauvolfii group
0% Calcium antagonists from the verapamil subgroup
0% Preparations of the clonidine group

280. A 26-year-old patient has been suffering from rheumatism since the age of 15. I had 2 rheumatic attacks 4
years ago. During the last 6 months, paroxysms of atrial fibrillation occurred once every 2-3
months. What alternative antiarrhythmic therapy or tactics should be offered?
100% Preventive intake of Cordarone
0% Immediate hospitalization
0% Defibrillation
0% Taking lidocaine
0% Purpose of heparin

281.a full-term boy developed moderate jaundice of the skin and mucous membranes on the 2nd day of life
. The child's general condition is not affected. In the blood: indirect hyperbilirubinemia -120

65

Downloaded from the site - online testing step

mmol / L. the child's blood type is A (II) Rh ( + ), the mother's blood type is B (III) Rh (+). What should be
the doctor's tactics?
100% Refrain from drug therapy
0% Assign enterosorbents
0% Prescribe prednisone
0% Prescribe cholekinetics
0% Perform a replacement blood transfusion

282. when determining blood groups, the isohemagglutination reaction was positive with
standard sera of groups A(II) and B(W), and negative with groups 0(1) and AB(IV). What
does this result indicate?
100% Unsuitability of standard serums
0% First blood type
0% Second blood type
0% Third blood type
0% Fourth blood type

283. when performing a complex of reactions necessary for blood transfusion, a patient with gastric
ulcer with blood group A (II) Rh+ was found to have blood in an ampoule of group A (II) Rh+ (upon
repeated examination). When reacting to individual compatibility by the Rh factor, there is agglutination
. What are the next tactics?
100% !individual blood collection at the blood transfusion station
0% Completely refuse to perform blood transfusion
0% Re-conduct an individual Rh compatibility test

https://translate.yandex.com/en/doc 63/540
22:20 ,27.6.2023 �� C : ; 5 B
0% Transfuse the patient's blood A (P) Rh+
0% Transfuse the indicated blood to the patient with prevention of blood transfusion shock

284. A 53-year-old patient was diagnosed with acute lung abscess. From the medical history, it was revealed that 2 weeks
ago she was discharged in a satisfactory condition from the therapeutic hospital, where she was treated for
acute lower lobar pneumonia. What is the most likely path of occurrence
of a lung abscess in a patient?:
100% Bronchial obstruction
0% Hematogenic pathway of infection
0% Embolic route of infection
0% Lymphogenic pathway of infection
0% Traumatic path of infection

285. a patient with a carbuncle of the upper lip applied to the polyclinic. Body temperature 39oc.
Pronounced swelling of the upper lip, eyelids. What should the surgeon do with this patient?
100% Be admitted to the surgical department
0% Prescribe an outpatient course of antibiotic therapy
0% Open the carbuncle, treat it on an outpatient basis
0% Prescribe physiotherapy procedures and warming compresses
0% Open the carbuncle and prescribe antibiotics

66

Downloaded from the site - online testing step

286. A 38-year-old patient was diagnosed with a pelvic bone fracture after a car accident. Within
24 hours, I am concerned about frequent, painful, small portions, with an admixture of blood urination. On
the ascending cystogram, which was performed in connection with a suspected rupture of the bladder, there is
a bladder in the form of a punching bag, a ray-like flow of contrast agent. What
treatment tactics should be used?
100% Urgent surgical intervention
0% Antibacterial, hemostatic therapy
0% Installing a permanent urethral catheter
0% Capillary puncture of the bladder
0% Cold on the suprapubic area, observation

287. on the second day of illness, a 22-year-old patient complains of high fever,
headache in the forehead, brow ridges, when moving the eyeballs, in the muscles and joints. Objectively:
temperature-390C. the face is hyperemic, the sclera are injected. The mucous membrane of the soft
palate and posterior pharyngeal wall is brightly hyperemic with pinpoint hemorrhages. What
changes in the hemogram are typical for this disease?
100% Leukopenia
0% Leukocytosis
0% Neutrophilosis
0% Anemia
0% Accelerated ESR

288. A 14-year-old boy with acute secondary obstructive pyelonephritis


was isolated from urine with Pseudomonas aeruginosa in the titer of 1,000,000 microbial bodies per 1 ml. Which antibacterial
drug is most appropriate to prescribe in this case?
100% Ciprofloxacin
0% Ampicillin
https://translate.yandex.com/en/doc 64/540
22:20 ,27.6.2023 �� C : ; 5 B
0% Cefazolin
0% Azithromycin
0% Levomycetin

289.A 45-year-old patient developed


weakness, hypotension, cyanosis of the upper half of the trunk, and swelling of the neck veins after a blunt chest injury with a sternum
fracture. With a pleural
puncture, the contents are absent. Ps-120 / min., rhythmic, weak filling. What is the most
likely diagnosis?Cardiac tamponade
100%
0% Pulmonary embolism
0% Concussion of the heart
0% Acute myocardial infarction
0% Collapsed hemopericardium

290. a patient has a grade 2a, 3a, and B flame burn on both arms, chest, and abdomen. The total area of burns
is approximately 35% of the body surface, including deep burns-28% of the body surface. I got a burn 4 weeks ago.
The patient's general condition is severe, body temperature 38.10 C, Ps-92 / min, arrhythmic, blood pressure-125/70
mm Hg. what is the most likely diagnosis?

67

Downloaded from the site - online testing step

100% Burn septicopyemia


0% Burn shock
0% Acute burn toxemia
0% Recovery
0% Chronic burn shock

291. A 32-year-old patient complains of low mood. She hears the "voices" of neighbors who
threaten her, comment on her actions. She thinks they're watching her through the walls, on the street, in
the store. Identify the syndrome:
100% Paranoid
0% Paranoid
0% Paraphrenic
0% Depressive
0% Hallucinosis

292. a 14-year-old boy with chronic tonsillitis and sinusitis developed feelings
of heart failure and additional pulse beats. Heart rate-83 / min. On the ECG: after every two
sinus contractions, impulses regularly occur in which the P Wave is absent, QRS is extended
for more than 0.11 seconds, the discordant T Wave is sharply deformed, after which a complete
compensatory pause is recorded. Specify the nature of rhythm disturbances:
100% Trigeminal extrasystole
0% Bigeminia-type extrasystole
0% Partial AV blockage
0% Complete AV blockage
0% Left bundle branch block

293. a 10-year-old child with non-rheumatic carditis periodically has seizures, which
are manifested by a feeling of pain in the heart, shortness of breath, pallor, increased blood

https://translate.yandex.com/en/doc 65/540
22:20 ,27.6.2023 �� C : ; 5 B
pressure, a sharp increase in heart rate up to 180/min. Which of the medications is the most
effective in treatment?
100% Obzidan
0% Novocainamide
0% Lidocaine
0% Verapamil
0% Aimalin

294. A 38-year-old patient


was found to have suffered an x-rib fracture on the left side with dislocated fragments and a parietal pneumothorax as a result of a blunt
object blow to the left side of the chest
. Notes pain in the left hypochondrium. objectively: pale, blood pressure-80/40 mm Hg,
Ps138 / min., weak filling and tension. Ultrasound revealed fluid in the left side of the abdomen.
A ruptured
100% spleen wasthe
Drain detected. Whatcavity
left pleural treatment tactics should
and perform I choose?
a laparotomy
Perform an upper-median laparotomy immediately and then drain the left
0%
pleural cavity
0% Immediately perform laparotomy and alcohol-novocaine blockade of Hm

68

Downloaded from the site - online testing step

Perform anti-shock measures and perform a laparotomy after increasing blood pressure
0%

0% Do a left-sided thoracotomy, and then immediately laparotomy

295. a 26-year-old patient operated on for diffuse toxic goiter of the third degree,
moderate thyrotoxicosis, developed convulsions of the hands, feet and face on the 2nd day after the operation
. Symptoms of Khvostek, Truso are positive. The patient complains of pain in the heart. On
the ECG-prolongation of the Q-T interval what complication did the patient have?
100% Hypoparathyroidism
0% Hyperparathyroidism
0% Thyrotoxic crisis
0% Paresis of the laryngeal nerves
0% Thyrotoxic myocardial dystrophy

296. A 36-year-old patient was admitted with complaints of cough with purulent sputum discharge up
to 150 ml per day, hemoptysis, periodic temperature increase up to 37.8 ° C, decreased
appetite, general weakness. Ill for 10 years, exacerbation of the disease in spring and
autumn. Objectively: pallor, slight acrocyanosis, thickening of the nail phalanges of the fingers.
Auscultation: wet wheezing under the left shoulder blade. On Ro-grams: expansion of the lung root,
severity and alkalinity of the lung pattern on the left. What is the most likely method to
clarify the diagnosis?
100% Bronchography
0% X-ray examination
0% Tomography
0% Bronchoscopy
0% Thoracoscopy

297. A 48-year-old woman has IHD for six months with progressive angina attacks.
Coronary ventriculography revealed: stenosis of > 70% of the anterior interventricular
artery (anterior ventricular artery) in the middle third for 0.7 cm. In other arteries - parietal
atherosclerotic changes, hemodynamically insignificant. Patient shown:

https://translate.yandex.com/en/doc 66/540
22:20 ,27.6.2023 �� C : ; 5 B
100% Balloon angioplasty of PMSA
0% Single artery CABG surgery
0% CABG surgery for 2-3 arteries
0% MCC to PMSA Operation
0% Drug therapy

298. when studying the class schedule of 3rd grade students, it is established that the number of lessons during
the week is 30; on Tuesday, the first lesson is Ukrainian, the second - music,the 3rd and 4th-physical education,
a total of 5 lessons. What is the leading place in the lesson schedule?
100% Number of physical education lessons
0% Number of lessons per week
0% Number of lessons per day
0% Place of the music lesson in the schedule
0% Place of the Ukrainian language lesson in the schedule

69

Downloaded from the site - online testing step

299. the company generates particularly toxic unutilized


industrial waste during the production process. Suggest a method of disposal and decontamination:
100% Burial in ditches of landfills in container containers
0% Heat treatment
0% Biothermal recycling in advanced landfills
Burial in ditches of landfills with insulation of the bottom and walls with a compacted layer
0%
of clay
0% Use as a raw material for recycling

300. A child of 1 month has regurgitation that occurs every time the
baby is placed in the crib after breastfeeding. During the examination, the pediatrician did not find any abnormalities in the child's
condition.
The doctor explained the occurrence of regurgitation by the peculiarities of the digestive system at this age and gave
advice on child Vertical
100% care. What are these
position tips?
of the baby immediately after feeding
0% Laying the baby on his stomach after feeding
0% Belly massage
0% Giving nipples immediately after breast-feeding
0% Thermal treatments on the stomach

301.The mother of a 4.5-month-old girl applied to the district pediatrician for advice on
the frequency of hygienic baths for the child. What frequency of hygienic baths
is most acceptable at this age?
100% 1 time per day
0% 1 time in 2 days
0% 1 time in 3 days
0% 1 time per week
0% 1 time in 10 days

302. a 55-year-old patient complains of aching epigastric pain, nausea, and heartburn.
These symptoms appeared after treatment with indomethacin. Objectively: the abdomen is soft,
painful in the epigastric region. The liver and spleen are not enlarged. The bowel
sections are painless. What should be done first in this case?
100% Cancel indomethacin
https://translate.yandex.com/en/doc 67/540
22:20 ,27.6.2023 �� C : ; 5 B

0% Set a fast for 2 days


0% Wash out the stomach
0% Prescribe antacids
0% Prescribe medications that affect Helicobacter pylori

303. A 27-year-old woman who is actively engaged in sexual activity complains of


numerous vesicles on the right labia, itching and heartburn. Rashes periodically
appear before menstruation and disappear after 8-10 days. What is the most likely diagnosis?
100% Herpes simplex virus
0% Bartholinitis
0% Primary syphilis
0% Cytomegalovirus infection

70

Downloaded from the site - online testing step

0% Genital warts

304. a 33-year-old patient developed an urticular rash on the skin


of the trunk and extremities and general weakness after penicillin administration. During transportation of the patient
by ambulance to a specialized department, he had a choking attack. Objectively: BH-28 / min,
Ps94 / min, AT-100/60 mm Hg. what is the most appropriate method of lung ventilation in this
case?
100% Tracheal intubation
0% Ambu Bag Application
0% Applying an oxygen mask
0% Duct Application
0% Applying a laryngeal mask

305. a 38-year-old patient suddenly developed pain in the left side of the chest, suffocation.
Objectively: the condition is moderate, Ps-100 / min, AT-90/60 mm Hg, breathing on the left is not
heard. Chest X-ray shows a collapse of the left leg to 1/2. what treatment
should be prescribed to the patient?
100% Passive drainage of the pleural cavity
0% Rest, resorption therapy
0% Pleural punctures
0% Surgical treatment
0% Active drainage of the pleural cavity

306. A 35-year-old female patient went to a gynecological hospital with complaints of periodic pain
in the lower abdomen, which worsened during menstruation, and dark brown smearing
discharge from the genital tract. During bimanual examination: the body of the uterus is slightly enlarged,
appendages are not detected, and a mirror examination of the cervix reveals cyanotic
"eyes". Which of the following diagnoses is most likely to occur?
100% Endometriosis of the cervix
0% Cervical erosion
0% Cervical polyp
0% Cervical cancer
0% Cervical fibroid

https://translate.yandex.com/en/doc 68/540
22:20 ,27.6.2023 �� C : ; 5 B
307. a 51-year-old patient complains of persistent spotting from the genital tract of a smearing
nature during the last 3 months, contact bleeding. During bimanual examination:
the cervix is enlarged, limited in mobility, dense to the touch. In the mirrors: a crater
-like ulcer in the center. The worm sample is positive. Which of the following diagnoses is most likely to occur?
100% Cervical cancer
0% Cervical erosion
0% Cervical polyp
0% Cervical pregnancy
0% Cervical leukoplakia

71

Downloaded from the site - online testing step

308.A 36-year-old woman applied to the gynecological hospital complaining of significant


bleeding from the genital tract and a monthly delay in menstruation. In a bimanual study:
the cervix is barrel-shaped and soft in consistency. The uterus is of normal size, somewhat
softened. Appendages without features on both sides. In a mirror study:
the cervix is cyanotic, enlarged in size, the outer eye is opened up to 0.5 cm. A urine test for
HCG is positive. What is the most likely diagnosis?
100% Cervical pregnancy
0% Uterine pregnancy
0% Abortion is in progress
0% Threat of termination of pregnancy
0% Ectopic pregnancy

309. A 64-year-old patient has been diagnosed with stomach cancer and is preparing for radical surgery.
There is a concomitant pathology: post-thrombophlebitic syndrome, edematous-painful form. A
history of pulmonary embolism 3 years ago. Please indicate the most effective method
of preventing the patient from developing repeated PE in the postoperative period after radical
gastric surgery:
100% Implantation of a coffee filter in the preoperative period
0% Heparin therapy in the postoperative period
0% Application of Unna zinc-gelatin dressing in the preoperative period
0% Administration of an indirect anticoagulant in the pre-and postoperative period
0% Application of elastic bandaging of limbs in the postoperative period

310. A 47-year-old patient has ulcerative colitis for 8 years and has been treated with glucocorticoids.
Complains of cramping pain in the parotid and left iliac regions, which
has significantly increased over the past 2 weeks, diarrhea with mucus and blood 4-6 times a day,
fever up to 38-39 ° C, headache and pain in the knee joints. Objectively: the
patient's condition is moderate, Ps-108 / min, AT-90/60 mm Hg, heart and lungs without features, tongue
moist, abdominal muscle tone is significantly reduced, peristaltic noises are absent. What
complication did the patient develop?
100% Toxic dilatation of the colon
0% Perforation of the colon
0% Intestinal bleeding
0% Colon stricture
0% Colon cancer

311. A 64-year-old patient went to a urologist with complaints of difficulty

https://translate.yandex.com/en/doc 69/540
22:20 ,27.6.2023 �� C : ; 5 B
urinating with a lifeless stream. Rectal examination revealed prostate changes that do not allow
differentiating between adenoma and cancer. What research should be done to clarify the diagnosis?
100% Blood test for prostate-specific antigen
0% General analysis of prostate secretions
0% Determination of the amount of residual urine
0% Determination of serum urea and creatinine levels
0% Urethrocystography

72

Downloaded from the site - online testing step

312. an aircraft factory processes materials using optical


quantum generators. It is established that the installation emits in the visible spectrum, the levels
of laser radiation in the workplace exceed the GDR which organ will be affected
first?
100% Oko
0% Skin
0% Liver
0% The spleen
0% Kidneys

313. a 72-year-old pensioner applied to the polyclinic. In the morning of the same day, at the dacha, I stepped on a nail
and injured my right foot. There are no data on previous vaccinations. Objectively: the condition
is satisfactory. The right foot is slightly swollen, and there is a stab wound on the sole. In order to prevent
the possible development of tetanus, first of all, you need to:
100% Intravenous administration of 1 ml of tetanus toxoid, 3000 IU of tetanus serum
0% Enter 3000 IU of tetanus serum intravenously
0% Treat the wound with soapy water
0% Enter 0.5 ml of tetanus toxoid intravenously
0% Prescribe a course of antibiotic therapy

314. A 27-year-old patient complains of pain in the right eye, which increases at night, decreased
vision, photophobia, and lacrimation. I had the flu a week ago. Objectively: the eye slit
is narrowed, the pupil is narrow, the color of the iris is changed, opalescence of the contents of the anterior
chamber. Cyclical soreness. What is the most likely diagnosis?
100% Iridocyclite
0% Irit
0% Keratitis
0% Con ' unctivit
0% Dacryodenite

315. A 3-year-old child is ill on day 3: fever 38-38. 5 oC, slight pain when swallowing,
enlarged anterolateral lymph nodes. On examination: the palatine tonsils are swollen, their surface
is covered with whitish-gray deposits with a smooth surface, which are tightly bound to the adjacent
tissues. Diphtheria was diagnosed. What process underlies the formation
of diphtheria plaques?
100% Fibrinous inflammation
0% Purulent inflammation
0% Catarrhal inflammation
0% Necrotic process

https://translate.yandex.com/en/doc 70/540
22:20 ,27.6.2023 �� C : ; 5 B
0% Dystrophic process

316. a newborn born as a result of rapid labor has paresis


of the hand muscles. There is no grasping reflex, and the palmar-oral reflex cannot be triggered.
There is no brush sensitivity. What is the most likely diagnosis?
100% Pareze Dejerin-Klumpke

73

Downloaded from the site - online testing step

0% Duchesne-Erba Pareze
0% Complete defeat of the brachial plexus
0% Diaphragm paresis
0% Horner-Bernard syndrome

317. a 30-year-old patient in a psychiatric ward is demonstrative, moody, talkative,


and constantly attracts the attention of others. Complains of pain all over the body. At the sight
of the doctor-moans, clutches his head, wails, demonstrates the inability to walk, holds
on to surrounding objects. In private, he walks freely around the ward, sings, applies makeup. Determine
the patient's condition:
100% Hysterical neurosis
0% Obsessive-compulsive disorder
0% Hypochondriac personality development
0% Schizophrenia
0% Manic-depressive psychosis

318. A 30-year-old patient was admitted to the emergency department after a car accident with
complaints of shortness of breath and pain in his left side. On the survey X-ray of the chest organs
, a large lumen of the left pulmonary field is determined with the absence of a pulmonary pattern,
and the mediastinal organs are shifted to the right. The left lung is pressed against the left root, the diaphragm
is shifted down by one intercostal space, and the sinuses are clearly contoured. What is the most likely
diagnosis?
100% Pneumothorax
0% Hemothorax
0% Bruised lung
0% Pneumohemothorax
0% Damage to the diaphragm

319. an 81-year-old patient complains of constant discharge of Sicha in drops, a feeling


of bursting in the lower abdomen. Objectively: there is a globular protrusion above the pubis, over which
a dullness is determined by percussion, and a positive suprapubic push. What
is the patient's symptom?
100% Paradoxical ischuria
0% Urinary incontinence
0% Dysuria
0% Enuresis
0% Polakiuria

320. A 50-year-old patient complained of an increase in body temperature up to 39 ° C,


throbbing pain and swelling of the right hand. The day before, I pricked my hand with a fish bone. On
examination, there is edema and sharp pain in the palm and in the area of raising the I finger of the hand. In addition
https://translate.yandex.com/en/doc 71/540
22:20 ,27.6.2023 �� C : ; 5 B
, there is swelling of the soft tissues of the back of the hand, redness of the skin and sharp pain when
moving the fingers. What is the patient's medical condition?
100% Phlegmon of the right hand
0% Carbuncle of the hand

74

Downloaded from the site - online testing step

0% Erysipelas of the hand


0% Tendon panaritium of the first finger of the hand
0% Foreign body of the hand

321. In the city, the effect of air emissions


from metallurgical waste on the incidence of obstructive bronchitis was studied on a sample basis. The calculated correlation coefficient
was +0.79. Evaluate the strength and direction of the connection:
100% Straight, strong
0% Reverse, strong
0% Straight, medium
0% Reverse, average
0% -

322. during a medical examination, a 43-year-old man was objectively found to have pale skin
and mucous membranes, smoothness of the papillae of the tongue, transverse alignment of the nails, cracks in
the corners of the mouth, and tachycardia. In the blood: Hb-90 g / l, anisocytosis, poikilocytosis. The most likely
causal factor for this condition is insufficient intake into the body:
100% The shackles
0% Midi
0% Of zinc
0% Magnesium content
0% Selena

323. a 29-year-old patient complains of the absence of menstruation for a year, rapid
fatigue, hypotension with syncopal states of memory loss, and dry skin. From
the anamnesis, it is known that 1.5 years ago in the second stage of labor there was significant bleeding,
uterine extirpation, hemotransfusion was performed, and she was on a ventilator for 3 days. What is the most likely
diagnosis?
100% Skien's syndrome
0% Chiari-Fromel syndrome
0% Farbs-Albright syndrome
0% Aronze del Castillo Syndrome
0% Adrenogenital syndrome

324. at the gestational age of 32 weeks, a pregnant woman went into preterm labor. Contractions in 10-15
minutes, 15-20 seconds each. Fetal heartbeat is clear, rhythmic-145 / min. During vaginal examination
, the cervix is shortened, the external eye is opened by 1.5 cm, the fetal bladder is intact,
the head is present, the discharge is mucous, the body temperature is 36.5°C. In the blood: no abnormalities. In a vaginal smear, there are
4-5
white blood cells
100% in the n /Prevention
Tocolysis. A. What is of
thefetal
obstetric tactic?
distress syndrome
0% Antispasmodics, analgesics
0% Give birth through the natural birth canal
0% Caesarean section operation
https://translate.yandex.com/en/doc 72/540
22:20 ,27.6.2023 �� C : ; 5 B

0% Give intravenous anesthesia to relieve labor


325. a citizen abuses alcoholic beverages, drinks up property and wages, and does not pay for it.

75

Downloaded from the site - online testing step

puts himself, his wife and two minor children in a difficult financial situation. I was
registered in a drug treatment dispensary at my place of residence. The wife turned to her
family doctor for advice on what application to the court she has the right to apply?
100% About restriction of civil legal capacity of a man
0% On declaring a man legally incompetent
0% About recognition of the man partially capable
0% On declaring a man unfit for work
0% On the recognition of a man as disabled

326. analysis of soil samples taken from the 0-20 cm layer on a plot of land reserved for
residential development showed that the number of Escherichia coli (cells per 1 g of soil) is 15,
enterococci-9, the titer of perfringens-0.1, the sanitary number-0.99, non-viable forms
of helminth eggs (specimens per 1 kg -5. which of the listed indicators does not meet
the hygienic standards and indicates the need for improving the soil on this land plot?
100% Number of Escherichia coli
0% The presence of non-viable forms of helminth eggs
0% Number of enterococci
0% Perfringens titer
0% -

327. the family lives in a district center that belongs to a radiation-contaminated zone.
A six-year-old child had ARVI for 19 days. She was treated on an outpatient basis. The child
was taken care of by his mother, a cafe worker. Determine the procedure for conducting an examination of disability:
100% A disability certificate is issued for the entire duration of the child's illness
A disability certificate is issued for only 14 days, followed by a certificate of
0%
care
0% A disability certificate is not issued, only a full-term care certificate
A disability certificate is issued for 14 days, after which no documents are
0%
issued
0% A disability certificate is issued for 7 days, followed by a certificate of care

328. the child is 12 years old. Complaints of dull aching pain in the epigastrium and right subcostal region,
which increases after eating fatty or fried food, headache, general weakness,
nausea, fever to subfebrile numbers. Palpation of the abdomen
shows resistance of the muscles in the right subcostal region, positive symptoms of Ker,
Ortner, Murphy. What is the most likely diagnosis?
100% Chronic cholecystitis
0% Acute appendicitis
0% Viral hepatitis
0% Acute gastritis
0% Acute pancreatitis

329. A 10-month-old girl suffering from a long-term cough with frequent relapses
was clinically suspected to have a pulmonary form of cystic fibrosis. What diagnostic method is appropriate
to use to confirm it?

https://translate.yandex.com/en/doc 73/540
22:20 ,27.6.2023 �� C : ; 5 B

76

Downloaded from the site - online testing step

100% Sweat Chlorides


0% Urease test
0% Coprogram
0% Chest radiography
0% Proteinogram

330. A 47 - year-old patient began to worry about constrictive pain behind the sternum, which occurs when
walking at 700-800 m. Drinks 2 liters of beer once a week. Arterial hypertension for
the last 7 years. Objectively: Ps-74 / min, AT-120/80 mm Hg. when conducting VEM at
a load of 75 W, ST segment depression was recorded 2 mm below the contour line in V4 — V6.
What is the most likely diagnosis?
100% Angina pectoris of tension, functional class II
0% Angina pectoris of tension, functional class III
0% Angina pectoris of tension, functional class IV
0% Hypertonic vegetative-vascular dystonia
0% Alcoholic cardiomyopathy

331. an 18-year-old patient has been ill since early childhood. In the blood: Hb-110 g / l, Er.- 3,9-1012/ l,
KP0, 8, lake.- 6,0-109/ l; WSE-30 mm / hour. Coagulogram: prothrombin index 95%,
blood coagulation retraction 50%, blood clotting time - after 40 minutes did not occur,
bleeding duration - 3 minutes. What mechanisms underlie the pathogenesis of this disease?
100% Lack of antihemophilic globulin a in the blood
0% Vitamin C deficiency
0% The presence of specific antibodies to the endothelial walls of blood vessels
0% !immune suppression of the bone marrow
0% Exogenous iron deficiency

332. A 60 - year-old woman has been experiencing weakness, dizziness,


and rapid fatigue over the past year. Recently-shortness of breath, paresthesia. Objectively: the skin and mucous
membranes are pale with an icteric tint. The papillae of the tongue are smoothed. Liver, spleen at the edge
of the costal arch. In the blood: Hb-70 g / l, Er.- 1,7•1012/ l, CP-1,2, macrocytes. Which
drug is pathogenetically justified for its use?
100% Vitamin B12
0% Vitamin B6
0% Ascorbic acid
0% Iron preparations
0% Vitamin B1

333. a 27-year-old man complains of shortness of breath,itching and swelling of the face
that occurred 25 minutes ago, ate crayfish and drank beer. The condition progressively worsens. Objectively:
excited. The face is puffy, the cheeks and lips are enlarged in size. BH-28 / min., wheezing wheezes are heard
on inhalation and exhalation, conducting noises during auscultation of the lungs, maximum above the sternum.
Heart sounds are loud, heart rate=P8 = 108 / min, AT-150/90 mm Hg. the most likely cause of shortness of breath is:
100% Laryngeal edema
0% Bronchial spasm

77

https://translate.yandex.com/en/doc 74/540
22:20 ,27.6.2023 �� C : ; 5 B

Downloaded from the site - online testing step

0% Hypertensive crisis
0% Neurocirculatory dystonia
0% Alcohol intoxication

334. An analysis of the organization of medical care in the regional center showed that every year about 12%
of patients receive inpatient care for diseases that do not require round-the-clock
supervision and intensive care. What are the most appropriate organizational changes to
address this problem?
100% Development of inpatient isolation care
0% Amendment of the charter of outpatient clinics
0% Development of primary health care
0% Structural adjustment of specialized care
0% Strengthening the material and technical base of hospitals

335. a 57-year-old woman complains of tightness in the esophagus, palpitations, difficulty


breathing when eating solid food, sometimes vomiting with her mouth full, and at night -
a"wet pillow"symptom. Ill for about 6 months. Objectively: temperature-39°C, height -168 cm,
Weight-72 kg, Ps-76/min, AT-120/80 mm Hg. Radiologically: the esophagus is significantly expanded, in
the cardiac part - narrowed. What pathology is most likely to cause dysphagia in the patient?
100% Achalasia of the cardia
0% Primary esophagospasm
0% Hiatal hernia
0% Esophageal cancer
0% Reflux esophagitis

336. A 62-year-old patient complains of losing 10 kg of weight in 2 months, the appearance of a "meat
slop" - colored sich, pain in the lumbar region, and fever up to 39oc. On palpation of the left
kidney, he complains of pain. In the blood: Er. -2.8 • 1012/l, Hb-90 g/l, leuc.- 8,8 • 109/l, ESR - 42
mm / hour. In the urine: specific gravity -1018, protein-0.66 g / l, red blood cells for the entire field of vision. What
is the preliminary diagnosis?
100% Kidney tumor
0% Kidney carbuncle
0% Tuberculosis of the kidney
0% Acute glomerulonephritis
0% Urolithiasis

337. a 54-year-old patient has mild primary arterial hypertension, CHD: tension angina II
FC, HF II B. Concomitant diagnosis: gastroesophageal reflux disease, erosive esophagitis
stage II. Constant use of which of the drugs can cause the patient to increase the manifestations
of gastroenterological pathology?
100% Lysorbide dinitrate
0% Metoprolol
0% Enalapril maleate
0% Omeprazole
0% Hydrochlorothiazide

78

https://translate.yandex.com/en/doc 75/540
22:20 ,27.6.2023 �� C : ; 5 B

Downloaded from the site - online testing step

338. A 78-year-old patient with prostate adenoma underwent hernia repair for a
direct inguinal hernia. There is no urination after surgery.
An enlarged bladder is detected above the womb. What needs to be done?
100% Bladder catheterization
0% Put cold on the bladder area
0% Assign UHF to the postoperative wound
0% Prescribe proserin intramuscularly
0% Prescribe subcutaneous antispasmodics

339. on the first day after surgery for a diffuse toxic goiter, a patient
complained of difficulty breathing, cold sweat, and weakness. Objectively: pale skin,
body temperature 38.5 oC, PDR-25 / min, Ps-110 / min, AT-90/60 mm Hg. what complication of the early
postoperative period did the patient develop?
100% Thyrotoxic crisis
0% Hypothyroid crisis
0% Postoperative tetany
0% Acute thyroiditis
0% Compression of the trachea with a hematoma

340. A first-time pregnant woman was hospitalized with complaints of headaches. I didn't
attend a women's consultation. The gestation period is 35-36 weeks. AT-180/120 mm Hg on the right, 140/90
mm Hg on the left, edema of the lower and upper extremities. In the urine: protein 3.97 g / l, hyaline and granular
cylinders. What is the most likely diagnosis?
100% Severe preeclampsia
0% AD-syndrome
0% Moderate preeclampsia
0% Mild preeclampsia
0% Combined NPG-preeclampsia

341. A 45-year-old female patient complains of headache, palpitation, and constrictive pain behind
the sternum. He has been suffering from hypertension for seven years. During treatment
, edema of the lower extremities appeared. The doctor suggested a side effect of the drug. Which of the listed drugs
could cause such a reaction?
100% Felodipine
0% Lisinopril
0% Metoprolol
0% fbesartan
0% Nebivolol

342. a female employee of a leather processing plant, who has been in contact with ursol for 12 years, complains
of severe itching of the skin. Objectively: on the hands and fingers, forearms, face and neck
there are symmetrical papular polymorphic rashes. The condition worsens after work, on
weekends and during holidays feels better. Which of the following should be applied in this
case?
100% Antihistamines

79

Downloaded from the site - online testing step

https://translate.yandex.com/en/doc 76/540
22:20 ,27.6.2023 �� C : ; 5 B
0% Disinfection solutions
0% Sulfur preparations
0% Radioactive isotopes
0% X-ray therapy

343. A 45-year-old patient was taken by ambulance with complaints of severe general
weakness, nausea, vomiting, and abdominal pain. Recently, he notes a decrease in appetite,
weight loss. Objective: hyperpigmentation of the skin, blood pressure-70/45 mm Hg, bradycardia. In additional
studies, the content of aldosterone and cortisol in the blood was reduced, the excretion of 17-CS and
17ox in the urine was reduced, hyponatremia, hypochloremia, hypokalemia. What treatment measures should be taken?
Appointment of glucocorticoids, mineralocorticoids, diets with a high content
100%
of Table salt
0% Appointment of diet therapy with a high content of Table salt
0% Prescribing prednisone
0% Appointment of aldosterone
0% Prescribing insulin

344. A 35-year-old patient complains of pain in the upper third of the shoulder, which increases at night.
Objectively: moderate swelling in the upper third of the shoulder,skin over it with elevated temperature,
soreness during palpation, limited movement in the shoulder joint. Radiographs show a site
of destruction of the humerus in the metadiaphyseal region with phenomena of tibial periostitis
(spicula) and detachment of the periosteum in the form of a "dasha". Make a preliminary diagnosis:
100% Osteogenic sarcoma
0% Osteoma
0% Hemangioma
0% Chondroblastoma
0% Chondroma

345. on the 8th day of life, a child born on time with a weight of 3500 g,
the body temperature rose to 37.5 oC, began to take the breast sluggishly. The baby was placed on the
mother's breast on day 3 due to postpartum endometritis in the mother. Objectively: the child
is active. On the skin of the chest, abdomen, and thighs,superficial flabby blisters of the number 10,
5-10 mm in diameter, with cloudy contents were found. Some of them collapsed, and a
bright surface appeared. There are no internal changes. Specify the most likely diagnosis:
100% Pemphigus of newborns
0% Vesiculopustulosis
0% Ritter's Exfoliative dermatitis
0% Lyell's syndrome
0% Infected diaper rash

346. during a preventive ultrasound examination of the abdominal organs in


a secondary school, a 5th-grade (11-year-old) student's left kidney is visualized 3 cm below
normal, of normal size, shape, and structure; a contralateral kidney is not found in a typical city
. Preliminary diagnosis: congenital malformation of the kidneys, dystopia of the left kidney,
absence of the right kidney or its pelvic dystopia. What method of radiation diagnostics is appropriate

80

Downloaded from the site - online testing step

apply to make a definitive diagnosis and determine the functional capacity


of both kidneys?
100% Dynamic renoscintigraphy
https://translate.yandex.com/en/doc 77/540
22:20 ,27.6.2023 �� C : ; 5 B

0% Radioimmune analysis
0% Radionuclide renography
0% Thermography
0% Excretory urography

347. in a 54-year-old female patient, a left paratracheal X-ray of the thoracic


organs revealed multiple formations ranging in size from 2 to 4 cm in diameter. Complaints of
coughing and shortness of breath during minor physical exertion. History of breast
cancer after radical therapy 5 years ago. Diagnosis: prolongation of the disease,
metastatic lung damage. What therapy tactics should I choose?
100% Palliative chemotherapy and radiation therapy
0% Surgical intervention
0% Radical chemotherapy
0% Anti-inflammatory antibacterial therapy
0% Radical radiation therapy

348. an ambulance doctor responded to a call to a person who had been pulled out of a garrote by relatives.
Objectively: there is no pulse in the carotid arteries, no consciousness, independent
breathing, corneal reflexes, and the presence of cadaveric spots on the back and back of the limbs.
What signs can be used to determine the onset of death?
100% Presence of cadaveric spots
0% Lack of independent breathing
0% Lack of corneal reflexes
0% No pulse rate
0% Lack of consciousness

349. A 48-year-old patient went to the doctor complaining of low back pain. Ill for 3 days after
hypothermia. After the examination, the doctor diagnosed acute lumbosacral
sciatica. What medications should be prescribed to the patient?
100% Nonsteroidal anti-inflammatory drugs
0% Corticosteroids
0% Vitamins
0% Antibacterial properties
0% Desensitizing agents

350. A 67-year-old patient has been undergoing inpatient treatment in an antitubercular


dispensary for two months due to recurrent tuberculosis (24.02.2005) S6 of the left lung
(infiltrative). The patient was prescribed the following treatment: isoniazid + rifampicin +
streptomycin + pyrazinamide + ethambutol. The patient complained of diplopia
and visual field limitations. Which of the above drugs caused this side effect?
100% Ethambutol
0% Isoniazid

81

Downloaded from the site - online testing step

0% Pyrazinamide
0% Streptomycin
0% Rifampicin

https://translate.yandex.com/en/doc 78/540
22:20 ,27.6.2023 �� C : ; 5 B
351. A 24-year-old repeat patient with Rh-negative blood type is under the supervision
of a perinatologist. In the anamnesis: in previous births, manual placental separation was performed for
bleeding in the third period. At 36 weeks of pregnancy, the antibody titer increased from 1: 16 to 1: 64.
Ultrasound revealed thickening of the placenta and slowing of fetal movements. How often
should blood tests for Rh antibodies be performed in the future?
100% Daily until delivery
0% 1 time per week
0% 1 time in 2 weeks
0% 1 time in 3 weeks
0% Before giving birth

352. A 49-year-old female patient was hospitalized with complaints of weakness, jaundice, and itchy skin.
I got sick 2.5 months ago. At hospitalization-significant jaundice. The liver is not palpable.
The gallbladder is enlarged and painless. Blood bilirubin -190 mmol / l, due to direct.
Feces are acholic. What is the most likely cause of jaundice?
100% Mechanical jaundice
0% Hemolytic jaundice
0% Gilbert's disease
0% Parenchymal jaundice
0% Karolyi syndrome

353. Microclimatic parameters were measured in the operating room of the regional clinical hospital
. The results of the conducted studies: the average air temperature is 22 ° C,
the relative humidity is 48%, the air velocity is 0.1 m / s. Give a hygienic assessment
of the operating room microclimate:
100% The microclimate is comfortable
0% The microclimate is uncomfortable
0% The microclimate is uncomfortable with high humidity
0% The microclimate is uncomfortable and cooling
0% The microclimate is uncomfortable with increased air velocity

354.A 12-year-old child periodically develops a fever of up to 38.5 oC,


cough, shortness of breath, and hemoptysis once during 6 months. BCG scar is absent. My grandfather
has tuberculosis. In the lungs-scattered dry and mixed wet wheezes. Radiologically:
uniform shallow-focal lung infiltration of the same type, thin-walled cavern in the upper lobes
with weakly expressed perifocal infiltration. What is the most
likely medical condition?
100% Tuberculosis of the lungs
0% Focal pneumonia
0% Chronic bronchitis
0% Bronchial asthma

82

Downloaded from the site - online testing step

0% Haman-Rich syndrome

355. A 37-year-old mining worker at a stope


face experienced intense shooting pain in
the lumbar spine, which spread down the left leg to the popliteal fossa,after being forced to bend his torso for a long time in the mine.
Spinal movements in the lumbar region are severely restricted. Positive Lasega symptom on the left. Palpatory

https://translate.yandex.com/en/doc 79/540
22:20 ,27.6.2023 �� C : ; 5 B
soreness of paravertebral points L5-S1. Tendon reflexes in the lower extremities-
reduced left Achilles reflex. Hypotension of the muscles of the left thigh and lower leg. Make a
preliminary clinical diagnosis:
100% Lumbosacral radiculopathy
0% Renal colic
0% Spinal stroke
0% Lumbar vertebra fracture
0% Transient ischemic attack

356. an 18-year-old patient complains of swelling of the right ankle joint,


limited mobility, and pain that increases during gait. In the anamnesis of "turning" of tuberculin
tests at the age of 7 years. I did not accept treatment because of the refusal of my parents. Radiologically: distal
epimetaphyseal focus with destruction of the tibial epiphysis, the articular gap is sharply
narrowed. The Mantoux reaction with 2 TO - 18 mm. What is the most likely diagnosis?
100% Tuberculin ostitis of the right ankle joint
0% Epiphyseal osteomyelitis
0% Right ankle sarcoma
0% Syphilis of the right ankle joint
0% Traumatic arthritis

357. A 5-year-old child was operated on for internal affairs six months ago. Complaints of fever
for 3 weeks, pain in the heart, muscles and bones. On examination - skin color "coffee with
milk", auscultation-systolic murmur in the heart with noise in the III-IV intercostal space. there are Januel spots on
the fingertips. What is the preliminary diagnosis?
100% Infectious endocarditis
0% Sepsis
0% Non-rheumatic carditis
0% Acute rheumatic fever
0% Typhoid fever

358. the third day after 1 urgent normal delivery. The child is cohabited,
naturally fed. Objectively: the general condition is satisfactory. Temperature 36,4 oC,
Ps-80/xv., AT-120/80 mm Hg. mammary glands are soft, painless. Lactation is moderate, the outflow
of milk is not difficult. The uterus is dense, the bottom of the uterus is 3 n / a below the navel. Lochia
are bloody-serous, in moderate quantity. Evaluate the dynamics of reverse uterine development:
100% Physiological involution
0% Subinvolution
0% Lochiometer
0% Pathological involution

83

Downloaded from the site - online testing step

0% And A Thermometer

359. the family doctor on call witnessed the death. On the basis of what medical document
is this case of death registered and in what maximum period should
death registration take place in the RAG-Si?
100% Medical death certificate. Duration-3 days
0% Medical death certificate. Duration-7 days
0% Medical death certificate. Term-1 month

https://translate.yandex.com/en/doc 80/540
22:20 ,27.6.2023 �� C : ; 5 B
0% Medical certificate of death. Duration-3 days
0% Medical certificate of death. Term-3 months

360. A 19-year-old patient complains of palpitations and suffocation during exercise.


Objectively: the borders of the heart are shifted to the right. During auscultation, a systolic murmur is heard in
the IIII intercostal space to the left of the sternum, an emphasis of the II tone on the pulmonary artery, and sometimes a Graham-Steele
murmur.
The ECG shows signs of right ventricular hypertrophy and blockage of the right bundle branch.
What100%
is the mostAtrial
likelyseptal
pathology
defectthat caused this picture?
0% Open Ductus arteriosus
0% Ventricular septal defect
0% Pulmonary artery stenosis
0% Fallot's Tetrad

361. A 35-year-old female patient went to the doctor of a women's clinic with the question: what can
be done to prevent pregnancy? Medical history: she underwent two operations for an ectopic
pregnancy, both fallopian tubes were removed during operations. What possible methods of treating
infertility in women should be used?
100% In Vitro fertilization
0% Ovulation induction
0% Operative laparoscopy
0% Surrogate motherhood
0% Male sperm insemination

362. a patient has been suffering from disseminated pulmonary tuberculosis for 5 years. Recently, there
have been pains in the lumbar region, frequent urination. In the urine: acid reaction, proteinuria; during
bacterioscopy of the urine smear-acid-resistant bacteria. What is the kidney damage of this patient?
100% Tuberculosis of the kidneys
0% Amyloidosis
0% Glomerulonephritis
0% Pyelonephritis
0% Hydronephrosis

363. A 30-year-old woman went to the doctor complaining of itchy skin, which worsens
in the evening and has been bothering her for a week. The pathological process spreads from the hands to the skin of the abdomen,
buttocks, pubis. Objective: polymorphic rash consisting of small paired vesicles,

84

Downloaded from the site - online testing step

mottled papules, excoriations, uncovers, inflammatory papules of red-brown color. What


is the most likely diagnosis?
100% Scabies
0% Pediculosis
0% Neurodermatitis
0% Dyshidrotic eczema
0% Contact-allergic dermatitis

364. A multi-specialty hospital with 500 beds is planned to be built in the locality. Where
should a polyclinic be located on a hospital site?

https://translate.yandex.com/en/doc 81/540
22:20 ,27.6.2023 �� C : ; 5 B
100% At the main entrance
0% In the garden and park area
0% In the center of the site near the medical buildings
0% Placement of a polyclinic on the territory of the site is not allowed
0% Near the economic zone

365. when studying the sanitary and hygienic conditions in a 4-bed therapeutic ward, it was established:
the area of the ward is 30 m2, height-3.2 m, air temperature + 20oC, humidity-55%,
air speed-0.1 m / s, light coefficient (SC) - 1: 5, natural light coefficient (KPO) -0.6
%, carbon dioxide content in the air - 0.1%. Which of the indicators does not meet the hygiene
requirements?
100% Natural Light Coefficient (KPO)
0% Air velocity
0% Light Coefficient (SC)
0% Carbon dioxide content in the air
0% Area of the Chamber

366. A 25-year - old 38-39-week-old pregnant woman living in rural areas


complained of cramping pains in the lower abdomen and
lumbar region. What kind of help should a health worker provide?
100% Hospitalize a woman in a district hospital, maternity ward
0% Observe during the day
0% Hospitalize a woman in a regional hospital, maternity ward
0% Enter an antispasmodic agent
0% Reassure a woman

367. a patient was hospitalized with suspected acute intestinal obstruction. Which of
the research methods will be the most informative to confirm the diagnosis?
100% Overview radiography of the abdominal organs
0% Laparocentesis
0% Laparoscopy
0% Ultrasound examination
0% FGDS

85

Downloaded from the site - online testing step

368. A 38-year-old female patient complains of "floating" heat sensations that recur up to 5 times
a day, headache in the occipital region with increased blood pressure, palpitation,
dizziness, rapid fatigue, irritability, memory impairment. 6 months ago
, an operation was performed to extirpate the uterus with appendages. What is the most likely diagnosis?
100% Post-castration syndrome
0% Premenstrual syndrome
0% Early pathological menopause
0% Secondary psychogenic amenorrhea
0% Physiological premenopause

369. 3 days ago, a boy had a foreign body removed from under his nail plate. After 2 days
, there was a sharp throbbing pain at the end of the nail phalanx, especially when pressing, hyperemia

https://translate.yandex.com/en/doc 82/540
22:20 ,27.6.2023 �� C : ; 5 B
of the nail roller, body temperature rose to 37.5 aboutC; the color of the nail plate did not
100%
change. What isSubungual panaritium
the most likely diagnosis?
0% Erysipelas
0% Paronychia
0% Erysipeloid
0% Abscess

370. when examining a 32-year-old overweight patient, the skin of the interdigital areas
of both feet is erythematous, macerated, dryly flaky, and there are isolated wet cracks in the folds between the V, IV, and III toes
. I'm worried about a slight itch. What is the most likely
diagnosis?
100% Epidermomycosis
0% Rubromycosis
0% Allergic dermatitis
0% Streptoderma
0% Microbial eczema

371. A 40-year-old woman from the age of 15 suffers from epilepsy with generalized epinapses
that occur 2-3 times a month at night and are accompanied by involuntary
urination and defecation. After the psychotrauma (the death of the father), the attacks became more frequent,
they occur with a frequency of 2-3 minutes, between attacks consciousness does not return,
the pupils ' reaction to light is absent. What is the most likely diagnosis?
100% Epileptic status
0% !steric attack
0% Increasing the number of large Epinapads
0% Absence
0% Epileptic psychosis

372. A 52-year-old patient was injured when an unknown explosive device went off in the entrance of the house
. In the area of the left surface of the neck, in the projection of m.
Strenoqleudomastoideus, three shrapnel wounds with a diameter of 2x3 mm are observed, from which dark blood constantly flows.
There is a pronounced edema of the left half of the neck, and a hematoma is immediately noted, which

86

Downloaded from the site - online testing step

constantly increasing, non-pulsating. Determine the scope of first aid:


100% Applying a compression bandage
0% Applying an occlusive dressing
0% Applying a tourniquet
0% Application of prothrombin sponge
0% !immobilization of the cervical spine with a Schantz collar

373. a 39-year-old patient complains of shortness of breath during exercise, general


weakness, and sweating. Objectively: barrel chest, BH-20 / min, Ps-84 / min, rhythmic.
AT-130/90 mm Hg Percussion: boxy shade of pulmonary sound, breathing is weakened,
heart tones are muffled, rhythmic. There is no peripheral edema. What kind of violation of the function
of external respiration will the patient have?
100% Reduced respiratory volumes
0% Increase in fast breath parameters per second
0% Paroxysmal deterioration of flow-volume parameters
https://translate.yandex.com/en/doc 83/540
22:20 ,27.6.2023 �� C : ; 5 B
0% Normal indicators
0% Reduction of peak inspiratory velocity while maintaining expiratory parameters

374. A 39-year-old patient complains of fever up to 41 ° C,headache,


and weakness. He became acutely ill, and on the 5th day of the disease, a roseolous-petechial rash appeared
on the lateral surface of the chest and back. RZK With 1: 640 Provacek rickettsii, lgM-89%. What
is the most likely diagnosis?
100% Typhus fever
0% Enterovirus infection
0% Lump Disease
0% Typhoid fever
0% Flu

375. a 55-year-old female patient, whose menstruation stopped 5 years ago, complains of a
feeling of dryness in the vagina, frequent and painful urination. The gynecologist during the examination
found signs of atrophic colpitis. In the urine: no features. What topical agents will give
the proper therapeutic effect?
100% Ovestin vaginal candles
0% Terzhinan vaginal tablets
0% Merotin-Combi vaginal cream
0% Metronidazole Vaginal Gel
0% Dalacin vaginal cream

376. A 63-year-old patient with persistent atrial fibrillation complains of moderate


shortness of breath. Objectively: there are no peripheral edema, vesicular respiration, heart rate-72 / min,
AT-140/90 mm Hg. what complex of medications is most appropriate in the secondary
prevention of heart failure?
100% b-blockers, ACE inhibitors
0% b-blockers, cardiac glycosides
0% Cardiac glycosides, diuretics

87

Downloaded from the site - online testing step

0% Cardiac glycosides, ACE inhibitors


0% Diuretics, b-blockers

377. A 40-year-old woman underwent bilateral adnexectomy 5 years ago. Complains of


weakness, fatigue, lack of menstruation, lower back pain, memory loss, does not
remember current events. During the examination, the presence of obesity, osteoporosis and hypercholesterolemia.
What syndrome does the patient have?
100% Post-castration syndrome
0% Adrenogenital syndrome
0% Adipozo-genital dystrophy
0% Antiphospholipid syndrome
0% Menopausal syndrome

378. An 8-year-old boy has been suffering from spotty rashes and itching
that occur after eating citrus fruits for 2 years. From the anamnesis: the rash first appeared at 6 months
after the introduction of juices into the diet. The father suffers from bronchial asthma, the mother
from allergic rhinitis. What diagnosis can be assumed?
https://translate.yandex.com/en/doc 84/540
22:20 ,27.6.2023 �� C : ; 5 B

100% Atopic dermatitis


0% Psoriasis
0% Pink lichen
0% Urticaria
0% Angioedema

379. on the 10th day of the postpartum period, a woman in labor complains of pain in the mammary glands.
Body temperature -38.2 oC, Ps-96 / min. In the mammary glands, significant and uniform edema, pain
during palpation. When pressed, milk droplets are released from the nipples. What are the tactics used in
relation to this patient?
100% Empty the breast by pumping or using a breast pump
0% Temporary restriction of the amount of liquid in the diet
0% Discontinuation of lactation
0% Prescribe antibiotic therapy and a diuretic
0% Compress on the mammary glands

380. An 8-year-old child was hospitalized with complaints of fever up to 39.8 ° C,


lethargy, moderate headache, and vomiting. Examination revealed meningeal symptoms.
A lumbar puncture was performed. A liquid under high pressure, transparent, cytosis of 450
cells in 1 µl (mainly lymphocytes-90%), glucose content of 2.6 mmol/l was obtained. what pathogen can
cause the disease in a child?
100% Enterovirus
0% Meningococcus
0% Tuberculosis bacillus
0% Staphylococcus aureus
0% Pneumococcus

88

Downloaded from the site - online testing step

381. A 25-year-old female patient discovered a tumor in the upper outer


quadrant of the right breast during self-treatment. Palpation revealed a painless, solid, mobile formation
of the breast with a diameter of 2 cm, the peripheral lymph nodes were not changed. During
ultrasound examination of the mammary glands: in the upper outer quadrant of the right
mammary gland, a volume formation of increased echogenicity, measuring 21x18 mm. What is the most
likely diagnosis?
100% Fibroadenoma
0% Breast cyst
0% Diffuse mastopathy
0% Breast cancer
0% Mastitis

382. A 23-year-old patient complains of a tumor in the lower outer quadrant


of the left breast during the 1st year, which becomes painful and increases
in size before menstruation. Palpation: fluid-filled mobile formation, up to 3 cm, with clear
contours, peripheral lymph nodes are not changed. During ultrasound examination
of the mammary glands: in the lower outer quadrant of the left mammary gland, a volume formation
of reduced echogenicity, measuring 31x29 mm. What is the preliminary diagnosis?
100% Breast cyst
0% Fibroadenoma
https://translate.yandex.com/en/doc 85/540
22:20 ,27.6.2023 �� C : ; 5 B

0% Fibrotic mastopathy
0% Breast cancer
0% Mastalgia

383. A 50-year-old patient worked at a chemical plant for 15 years. In the work
, I used a solvent-xylene. He was hospitalized with suspected chronic intoxication.
Established anemic syndrome. What is the primary measure of secondary prevention of anemia
?
100% Change of place of work
0% Appointment of glucocorticoids
0% Application of iron-containing preparations
0% Adding seafood to your diet
0% Inclusion of meat products in food

384. 2 weeks after giving birth, a woman in labor developed breast pain, which increased
within 3 days. Objectively: body temperature 39oC, chills, weakness, hyperemia of the skin,
enlargement, soreness and deformity of the breast. Palpation of the infiltrate reveals
the area of softening and fluctuation. What is the most likely diagnosis?
100% Infiltrative-purulent mastitis
0% Phlegmonous mastitis
0% Lactostasis
0% Serous mastitis
0% Mastopathy

385. a 38-year-old patient complains of difficulty swallowing solid food that has appeared near the mouth.

89

Downloaded from the site - online testing step

a month ago, and recently a semi-liquid one. It also notes weakness, deterioration of the general
condition, decreased appetite, weight loss, fever. Objectively: the skin is dry,
pale, the tongue is overlaid, no other abnormalities were found. In the blood: hypochromic anemia, increased
ESR. What is the preliminary diagnosis?
100% Esophageal cancer
0% Gastroesophageal reflux disease
0% Scleroderma
0% Stomach cancer
0% Esophagitis

386. A 70-year-old patient went to the doctor complaining of cardiac


arrhythmia and shortness of breath. Objective: AT-150/90 mm Hg, extrasystolic arrhythmia (10-12 extrasystoles per
minute), left ventricular systolic dysfunction (ejection fraction 42%). Which of
the antiarrhythmic drugs should be prescribed as the initial therapy in this case?
100% Amiodaronum
0% Flecainide
0% Encainide
0% Moracizin
0% Digoxin

387. A 45-year-old patient complains of intense sharp pain in the right side
https://translate.yandex.com/en/doc 86/540
22:20 ,27.6.2023 �� C : ; 5 B
radiating to the right thigh and perineum. Notes frequent urge to urinate, urine the color
of "meat slop". This condition is described for the first time. Positive Pasternatsky symptom on the right.
What is the most likely diagnosis?
100% Urolithiasis
0% Acute appendicitis
0% Acute pyelonephritis
0% Acute cholecystitis. Renal colic
0% Acute pancreatitis

388.during the investigation of a case of mass poisoning, car mechanics who were
testing diesel engines in the workshop box where the exhaust ventilation was damaged
showed symptoms at the end of the working day: headache, nausea,vomiting, tinnitus,
pulse lability. Objectively: the skin and mucous membranes are cherry-red in color. What
toxic factor caused mass poisoning of auto mechanics?
100% Carbon monoxide
0% Carbon dioxide
0% Sulfur dioxide
0% Nitric oxide
0% Carbon disulfide

389. A 56-year-old patient was delivered by an ambulance with nosebleeds. Suffers


from hypertension. At the time of examination: AT-200/130 mm Hg, antihypertensive agents were administered.
Objectively: blood is released from the nasal cavity, mainly flowing into the pharynx. Single
vomiting of blood clots. What kind of help should be given to the patient?

90

Downloaded from the site - online testing step

100% Posterior nasal tamponade


0% Anterior nasal tamponade
0% Ligation of the external carotid artery
0% Electrocoagulation of a bleeding vessel
0% Administration of hemostatic and antihypertensive agents

390. A patient with grade II obesity (BMI 36 kg / m2) went to an endocrinologist to


lose weight. What type of diet therapy should be offered to the patient?
100% Physiological subcaloric diet therapy
0% Protein-restricted diet
0% A carbohydrate-restricted diet
0% A fat-restricted diet
0% Fasting for 2 weeks

391. A 20-year-old female patient went to the antenatal clinic complaining about the absence
of menstruation for 7 months. From the anamnesis: at an early age, she was ill with childhood infections and
sore throats, menarche from the age of 13, regular monthly periods, menstrual cycle of 28 days, menstruation
lasts 5-6 days, painless. 7 months ago I suffered stress. Gynecological examination
revealed no changes in the uterus and appendices. What is the most likely diagnosis?
100% Secondary amenorrhea
0% Primary amenorrhea
0% Algodismenorrhea

https://translate.yandex.com/en/doc 87/540
22:20 ,27.6.2023 �� C : ; 5 B
0% Oligomenorrhea
0% Fake amenorrhea

392. A 4-year-old boy woke up at night due to an attack of coughing and shortness of breath. A history
of atopic dermatitis. The condition is disturbed: respiratory failure of the second degree. The child is pale,
frightened, the chest is swollen, distant wheezing is heard. Above the lungs - tympanitis,
exhalation is significantly prolonged, scattered dry and wet wheezes on both sides. What causes the identified
changes?
100% Bronchial obstructive syndrome
0% Stenotic laryngotracheitis
0% Foreign body aspiration
0% Stridor
0% Pneumonia

393. A 30-year-old patient was admitted to an antitubercular dispensary due to changes detected
on fluorography: S1 of the right lung has a shadow up to 1 cm in diameter, of weak
intensity with indistinct contours. The tomogram shows destruction in the center of the shadow.
MBT was detected in sputum. The patient was diagnosed with focal tuberculosis. What
phases of the tuberculosis process correspond to the identified changes?
100% Infiltration and decay
0% Infiltrations and contamination
0% Resorption and scarring

91

Downloaded from the site - online testing step

0% Decay and contamination


0% Compaction and resorption

394. in a victim with a stab wound to the chest on the right, cyanosis is detected,
AT70 / 40 mm Hg, Ps-120 / min, BH-34 / min, lagging of the right half of the chest during breathing.
The first priority for providing assistance will be:
100% Puncture of the right pleural cavity
0% Immediate transfer of the patient to a ventilator
0% Intercostal blockage
0% Introduction of inotropic pharmacological preparations
0% Administration of blood products

395. A 60-year-old patient had severe pain in his right arm for 2 days. On day 3
, vesicular rashes appeared in the form of a chain on the skin of the shoulder, forearm and hand. Sensitivity in
the area of rashes is reduced. What is the most likely medical condition?
100% Herpetic ganglionitis
0% Dermatitis
0% Cervical-thoracic sciatica
0% Psoriasis
0% Allergy

396. a 32-year-old patient caught a cold 4 days ago: sore throat, rapid fatigue.
In the morning of the next day, a dry cough appeared,the body temperature rose to 38.2 oC,
the state of health worsened, mucopurulent sputum appeared. Percussion: above the lungs-

https://translate.yandex.com/en/doc 88/540
22:20 ,27.6.2023 �� C : ; 5 B
a pulmonary tone, vesicular breathing, weakened below the angle of the scapula on the right side, there are also
heard small-scale sonorous and not sonorous wheezes. What is the most likely diagnosis?
100% Focal right-sided pneumonia
0% Bronchial asthma
0% Acute bronchitis
0% Lung cancer
0% Gangrene of the lung

397. A 30-year-old woman complains of general weakness, difficulty swallowing food, dry
skin and brittle hair. Objectively: to-36.6 oC, BH-16 / min, Ps-92 / min, AT-110/70 mm Hg.
the skin and visible mucous membranes are pale. In the blood: Hb-65 g / l, Er.- 3,2•1012/ l, CP-0,6,
ret, - 3%, lake.- 6,7•109/ l, E.-2%, P.-3%, S.-64%, L.-26%, M.- 5%, SHOE17 mm / year. Serum
iron -7.4 mmol / l, total protein-78 g / l. what factor caused the occurrence
of the disease?
100% Iron
0% Vitamin B6
0% Protein
0% Folic Acid
0% Glucose-6-phosphate dehydrogenase

92

Downloaded from the site - online testing step

398. a 36-year-old patient developed choking and coughing with


pink sputum discharge after physical exertion. He suffers from rheumatism and mitral heart disease. Auscultation over
the heart and lower parts of the lungs is characterized by small-and medium-puffy wet wheezing. What
is the leading mechanism of deterioration?
100% Increased hydrostatic blood pressure
0% Increased oncotic blood pressure
0% Reduced lymphatic outflow
0% Reduction of oncotic blood pressure
0% Increased aggregation of red blood cells and platelets

399. a 40-year-old woman was hospitalized with complaints of suffocation attacks, coughing with sputum.
He has been ill for 4 years. For the first time, a suffocation attack appeared in the patient while staying in the village. In the future, the
attacks
were disturbed when cleaning the premises. After 3 days of hospital stay, the patient's condition
improved
100% significantly.
HouseholdWhat is the most likely etiological factor?:
allergens
0% Plant pollen
0% Infectious
0% Chemicals
0% Psychogenic

400. A 65-year-old diabetic patient started taking antibiotics for pneumonia and
discontinued glibenclamide in the absence of appetite. Soon the thirst increased,
drowsiness appeared, and the muscles of the lower legs began to cramp. He lost consciousness and was hospitalized. Objectively:
there is no consciousness, the skin is dry. The tone of the eyeballs is reduced, breathing is shallow, rapid.
Blood glucose - 36 mmol / l. the reaction of urine to acetone is negative, to glucose-positive. Determine
the patient's condition:
100% Hyperosmolar coma

https://translate.yandex.com/en/doc 89/540
22:20 ,27.6.2023 �� C : ; 5 B
0% Ketoacidotic coma
0% Delayed-action allergic reaction
0% Cerebral coma
0% Lactic acidemic coma

401. a 57 - year-old woman with third-century obesity. Two months ago, I was diagnosed with diabetes
mellitus. The endocrinologist recommended a subcaloric diet and dosed physical activity.
The fasting blood glucose level is 9.2 mmol / l. what hypoglycemic drug can be prescribed to the patient?
100% Metformin
0% Glibenclamide
0% Gliquidone
0% Glipizid
0% Repaglinide

402. A 73-year-old female patient complains of weakness, drowsiness, chilliness, severe


memory loss, hair loss, constipation, and edema. Objectively: moderate nutrition. The skin is dry
and yellowish. Swelling of the face, limbs (after pressing on the skin of the lower leg, the fossa does not
remain). The borders of the heart are expanded, heart tones are muted, bradycardia. Thyroid gland

93

Downloaded from the site - online testing step

the gland is reduced in size. In the blood: NB-85 g / l, cholesterol-8.5 mmol/l; TSH-20.5 mmol / l
. what is the preliminary diagnosis?
100% Hypothyroidism
0% Cardiosclerosis, heart failure
0% Chronic hepatitis
0% Kidney failure
0% Severe atherosclerosis of the cerebral vessels

403. A patient with the bubonic form of plague has been identified in a natural plague center (Kazakhstan). Everyone
who communicated with the patient was placed in an isolation ward. What should be done immediately in isolation?
100% Antibiotic prevention
0% Phagoprophylaxis
0% Remantadine chemoprophylaxis
0% Introduction of interferon
0% Introduction of interferon inductors

404. An objective examination of a 9-month-old girl shows that her skin is pale
and cyanosis occurs during restlessness. Percussion: expansion of the heart boundaries in
the transverse direction. Auscultation: to the left of the sternum in the 3-4 intercostal
space, a prolonged systolic murmur is heard, conducted over the entire area of the heart and on the back. What
congenital heart disease can be suspected in a child?
100% Ventricular septal defect
0% Atrial septal defect
0% Aortic coarctation
0% Fallot's Tetrad
0% Pulmonary artery stenosis

405. A 32-year-old patient notes a tumor-like formation on the anterior surface of the neck that

https://translate.yandex.com/en/doc 90/540
22:20 ,27.6.2023 �� C : ; 5 B
appeared two years ago. For the last three months, the tumor has been rapidly growing, and there
are obstacles in swallowing, talking, and feeling pressure from the tumor. Objectively: skin of normal
humidity, Ps-80 / min., rhythmic, blood pressure-130/80 mm Hg.in the right lobe of the thyroid gland
, a node of 3. 0x3. 5 cm is determined, dense, bumpy, which shifts during swallowing. The
thyroid scan shows
100% a "cold
Thyroid knot". What is the preliminary diagnosis?
cancer
0% Thyroid adenoma
0% Thyroid cyst
0% Nodular goiter
0% Autoimmune thyroiditis

406. a 61-year-old man was eating pumpkin seeds, talking and coughing. Continuous coughing
changed to periodic coughing after 2 hours. On the exhalation during coughing
, a"clapping" is heard at a distance. During auscultation, pulmonary respiration is heard from both sides.
No pathological changes were found on the chest X-ray. Where is the balloting foreign
body located?
100% In the trachea

94

Downloaded from the site - online testing step

0% In the ventricles of the larynx


0% In the right bronchus
0% In the left bronchus
0% In the laryngopharynx

407. A 46-year-old woman was admitted to a psychiatric hospital for the first time in connection with
a suicide attempt. According to relatives: she became depressed, expressed the opinion that
she had lived her life in vain. During a conversation with a doctor, she is tense, her mood is low, and she is pessimistic
about her condition and future. She denies the suicide attempt, refuses to be hospitalized,
and insists that she be allowed to go home. What are the tactics for this patient?
100% Emergency admission to a hospital
0% Take a written refusal
0% Send them to a day hospital
0% Conduct a psychotherapy conversation
0% Send them for outpatient treatment

408. a dyer of a car factory was diagnosed with acute poisoning


with moderate benzene amide compounds. After inpatient treatment, there is a significant
improvement in the condition. What expert decision should be made in this case?
100% Issue a professional bulletin for outpatient treatment
0% It can continue working in compliance with sanitary and hygienic standards.
0% Issue a disability certificate for outpatient treatment
0% Send it to the MSEC to determine the percentage of professional disability loss
Send it to the MSEC to determine the disability group due to an occupational
0%
disease

409. during the year, 616 children were born alive in the maternity hospital,1 child was born dead,
and 1 child died on the 5th day of life. What indicator can be used to most accurately
assess this situation?
100% Perinatal mortality
0% Overall mortality rate

https://translate.yandex.com/en/doc 91/540
22:20 ,27.6.2023 �� C : ; 5 B
0% Birth rate
0% Infant mortality rate
0% Natural growth rate

410. a 38-year-old woman has the following blood levels: HB-84 g / l, ep. - 3, 5 * 1012/L, CP-0.72, leuc.- 4,1 • 109/ l;
retic.- 2%, ESR-26 mm / h. Serum iron-9.0 mmol / l. treatment was prescribed.
On the 6th day of treatment, a peripheral blood test was performed. What changes in the indicators
are most likely to confirm the adequacy of the prescribed therapy?
100% Increased reticulocyte count
0% Normalization of hemoglobin levels
0% Normalization of the number of red blood cells
0% Increasing the color indicator
0% Reduced red blood cell clotting rate
411. a 10-year-old child with mushroom poisoning developed a toxic hepatitis C clinic

95

Downloaded from the site - online testing step

signs of liver failure. Arousal attacks alternate with lethargy. There


was a"liver" smell in the exhaled air. What causes the development of the described clinic in the
patient?
Toxic effects on the central nervous system of products of nitrogenous
100%
compounds metabolism
0% Violation of venous outflow from the abdominal organs
0% Portal hypertension
0% Fatty liver infiltration
0% -

412. A 28-year-old maternity patient complains of headache,


visual impairment, and lethargy upon admission to the hospital. Objectively: AT-200/110 mm Hg, pronounced edema of the legs,
anterior abdominal wall. Fetal heartbeat is clear, rhythmic-190 / min. During internal
examination: the opening of the cervix is complete, the fetal head is in the pelvic cavity. What
are the further tactics of labor management?
100% Operation of applying obstetric forceps
0% Caesarean section
0% Fruit-destroying operation
0% Conservative delivery with episiotomy
0% Stimulation of labor activity

413. A 35-year-old patient complains of pain and morning stiffness of the joints of the hands and
temporomandibular joints lasting more than 30 minutes. He has been ill for 2 years. Objective: edema
of the proximal interphalangeal joints of the hands and limited movement in them. What research
needs to be done?
100% Radiography of the hands
0% General blood test
0% The Vaale-Rose reaction
0% Immunogram
0% Proteinogram

414. a patient who eats refined products for a long time complains of major emphases,
fatigue, depression, insomnia, irritability. Objectively: muscular asthenia, pain and cramps in

https://translate.yandex.com/en/doc 92/540
22:20 ,27.6.2023 �� C : ; 5 B
the tibia muscles, during which it steps on the heel, then on the outer edge of the foot. From the
cardiovascular system-tachycardia, hypoxia, dystrophic changes in the heart muscle. There
are violations of the gastrointestinal tract. What diagnosis can be made
100% Hypovitaminosis B1
0% Hypovitaminosis B2
0% Hypovitaminosis B12
0% Hypovitaminosis B6
0% Hypovitaminosis B5

415. on the 15th day after contact with a polio patient, a 2.5-year-old child developed moderate
catarrhal symptoms in the form of a runny nose and dry cough. On the 2nd day of the disease
, there was a decrease in appetite, lethargy, and sleep was disturbed. When examined, the child is moody, lethargic,

96

Downloaded from the site - online testing step

occipital muscles are rigid, hyperesthesia along the right sciatic nerve, to-38.2 oC. What is the period
of development of the paralytic form of polio in this child?
100% Preparolytic
0% Paralytic
0% Recovery mode
0% Residential
0% Incubation period

416. The district doctor of the polyclinic diagnosed the patient with dysentery. What accounting
document reflects this type of morbidity?
100% Emergency message
0% Statistical ticket
0% Message about the most important non-epidemic disease
0% Sick leave form
0% Control card of a dispensary patient

417. A child with jaundice, hepatosplenomegaly, and elevated liver tests


has a typical mild form of viral hepatitis A. The main treatment option is:
100% Compliance with the regime and diet
0% Appointment of hepatoprotectors
0% Prescribing antiviral drugs
0% Appointment of glucocorticoids
0% Conducting infusion therapy

418. a 47-year-old patient with decompensated pyloric gastric stenosis


of ulcerative origin developed convulsive syndrome. What is the most likely pathogenetic
justification for this syndrome?:
100% Hypochloremia
0% Iron deficiency anemia
0% Hypovolemia
0% Cachexia
0% Achlorhydria

419. The general practitioner is instructed to conduct an analysis of the health status of the adult population at

https://translate.yandex.com/en/doc 93/540
22:20 ,27.6.2023 �� C : ; 5 B
the service area. Which groups of im indicators will be involved in this analysis?
100% Demographic, morbidity, and disability factors
0% Demographic, morbidity, physical development
0% Morbidity, disability, and mortality rates
0% Birth rate, morbidity, disability
0% Morbidity, mortality, and physical development

420. a premature baby born at 34 weeks of gestation


has tachypnea, swing-like breathing, sternal sinking,
and expiratory noises 4 hours after birth. Respiratory rate 80 / min. Weakened breathing is heard above the lungs

97

Downloaded from the site - online testing step

with intermittent wheezes of various sizes. An X-ray of the lungs shows an air bronchogram
and a nodo-reticular mesh. What is the most likely diagnosis?
100% Hyaline membrane disease
0% Lung atelectasis
0% Massive meconial aspiration syndrome
0% Birth trauma
0% Neonatal pneumonia

421. A 40-year-old female patient was delivered with complaints of cramping pains in the lower abdomen and
excessive blood discharge from the genital tract. The last two years of menstruation last up to 16
days, excessive, with convolutions, painful. Bimanual examination: there is a fibromatous
node that is born. Choose the right tactics:
100% Removal of a fibromatous node through the vagina
0% Hormonal hemostasis
0% Vitamin Therapy
0% Superpichval amputation of the uterus without superstructures
0% Uterine extirpation without attachments

422. a 30-year-old patient suffering from headaches, when lifting a heavy object, felt
a strong headache in the form of a blow to the head; nausea, vomiting, and light
dizziness appeared. A day later-objectively expressed meningeal syndrome, body temperature -37.6
oC. The doctor suspected a subarachnoid hemorrhage. What additional examination should
be performed to confirm the diagnosis?
100% Lumbar puncture with examination of cerebrospinal fluid
0% X-ray of the skull
0% Computed tomography
0% Rheoencephalography
0% Cerebral angiography

423. the child is 4 months old. It develops normally. Objectively: the right border of relative cardiac
dullness is located along the right parasternal line, the upper one is the second rib, and the left one is 2 cm outward
from the midclavicular line. How do I interpret the results?
100% Age limit
0% Congenital heart disease
0% Cardiomyopathy
0% Hypertrophy, carditis
0% Pulmonary hypertension

https://translate.yandex.com/en/doc 94/540
22:20 ,27.6.2023 �� C : ; 5 B

424. the child is 7 months old. Body weight at birth-3450 g. is on natural


feeding. The bait was introduced in a timely manner. Determine the daily protein requirement of this child:
100% 3.0 g / kg
0% 2.0 g / kg
0% 2.5 g / kg
0% 3.5 g / kg

98

Downloaded from the site - online testing step

0% 4.0 g / kg

425. an 8-year-old boy developed swelling on his face and lower extremities 2 weeks after suffering from a sore
throat. Objectively: the condition is severe, blood pressure-120/80 mm Hg. urine is brown.
Oliguria. In the urine: relative density-1.015, protein -1.2 g / l, red blood cells-leached, cover the entire
field of view, granular cylinders-1-2 in p / S, salts-urates (a large amount). What is the most
likely diagnosis?
100% Acute glomerulonephritis with nephritic syndrome
0% Acute glomerulonephritis with nephrotic syndrome
0% Acute glomerulonephritis with nephrotic syndrome, hematuria and hypertension
0% Acute glomerulonephritis with isolated urinary syndrome
0% Kidney stone disease

426. A 60-year-old woman was hospitalized for an acute transmural infarction. An hour
ago, the patient's condition began to worsen: there was increasing shortness of breath, dry cough.
HR30 / min, HR-130 / min, AT-90/60 mm Hg. heart tones are deaf, emphasis of tone II on the pulmonary artery. In
the lower parts of the lungs on the right and left moist srednepukhyrchatye wheezing. The temperature is 36.4 oC.
Which drug should be used first?
100% Promedol
0% Euphyllinum
0% Dopamine
0% Heparin
0% Digoxin

427. As a result of long-term environmental pollution caused by the emissions


of a metallurgical plant, a biogeochemical province was formed in the adjacent residential area of the city
. What is the leading attribute that can be used to give it a hygienic assessment?
100% Accumulation of certain chemicals in the soil
0% Restriction of sanitary and domestic water use conditions
0% Air pollution
0% Deterioration of the quality of plant-based products
0% Increased morbidity rate in the population

428. workers work in conditions of high levels of dust. For the occurrence of occupational
dust diseases, the chemical (silicon dioxide content) and physical properties
of dust aerosols are important. What is the main physical property of dust aerosols?
100% Dispersion
0% Magnetization
0% Electric charge capacity

https://translate.yandex.com/en/doc 95/540
22:20 ,27.6.2023 �� C : ; 5 B
0% Solubility
0% !onizatsiya

429.Elevated levels of manganese in atmospheric air, drinking


water, and agricultural food products were observed in the city. What is the effect of these factors on

99

Downloaded from the site - online testing step

public health?
100% Comprehensive
0% Combined
0% Combined
0% Synergistic
0% Separate

430. a 42-year-old patient complains of palpitations, constant weakness, pain in the heart
area radiating to the left shoulder, increased during breathing, relieved when the patient sits down,
body temperature rises to 38.5 ° C. Objectively: AT-105/50 mm Hg, Ps-120 / min. In the zone
of absolute dullness of the heart, a two-phase noise is heard. In the lungs, breathing is weakened in
the lower parts. On the ECG-concordant rise of the ST segment in V2-V6. What is the most likely
diagnosis?
100% Pericarditis
0% Myocardial infarction
0% Lung infarction
0% Dressler's syndrome
0% Pneumonia

431. A 20-year-old patient complains of severe headache, double vision, general


weakness, fever, and irritability. Objectively: the body temperature is 38.1 oC, it comes into
contact reluctantly, reacts painfully to stimuli. Ptosis of the left eyelid, divergent strabismus,
anisocoria S > D. Severe meningeal syndrome. During lumbar puncture, the cerebrospinal fluid flowed out under
a pressure of 300 mm water. st., transparent, with a slight opalescence, a fibrinous film fell out after a day
. Protein -1.4 g / l, lymphocytes-600/3 in mm3, sugar-0.3 mmol/l. what preliminary diagnosis
should be made for the patient?
100% Tuberculosis meningitis
0% Meningococcal meningitis
0% Armstrong lymphocytic meningitis
0% Syphilitic meningitis
0% Mumps meningitis

432. a patient with eating disorders, steatorrhea,


has abdominal pain 4 hours after eating, especially above the navel and to the left. Diarrhea can be replaced by constipation
for up to 3-5 days. Palpation: moderate pain in the choledochopancreatic zone. The level of amylase in
the blood does not increase. Radiologically, calcifications located above the navel were detected.
What is the most likely diagnosis?
100% Chronic pancreatitis
0% Chronic gastroduodenitis
0% Duodenal ulcer 12
0% Zollinger-Ellison syndrome
0% Chronic calculous cholecystitis
https://translate.yandex.com/en/doc 96/540
22:20 ,27.6.2023 �� C : ; 5 B

433. a 56-year-old patient with diffuse toxic goiter has atrial fibrillation with
PE - 110/min, arterial hypertension. AT-165/90 mmHg what is the treatment next to mercazolil?

100

Downloaded from the site - online testing step

should I recommend it in this case?


100% Anaprilin
0% Radioactive iodine
0% Novocainamide
0% Verapamil
0% Corinfar

434. in the maternity hospital, on the 3rd day of life, a child developed a hemorrhagic rash,
vomiting with blood, and black emptying. The examination revealed anemia, prolongation
of blood clotting time, hypoprothrombinemia, and a normal platelet count. What is the best
therapeutic approach?
100% Vitamin K
0% Sodium Ethamsylate
0% Epsilon-Hydrocaproic acid
0% Fibrinogen
0% Calcium hyiuconate

435. a patient complains of frequent, copious,without pathological admixtures, foamy bowel movements,
cramping pain in the parotid region, rumbling in the abdomen, high fever.
The disease is associated with eating soft-boiled eggs. What is the most likely
causative agent of the disease?
100% Salmonella enteritidis
0% Salmonella typhi
0% Shigella sonnei
0% Escherichia coli
0% V Cholerae

436. a 48-year-old man complains of pain in the left side of the chest, cough, moderate
shortness of breath, which appeared after the injury. Objectively: there is painful swelling in the left 6-8th intercostal space,
the ribs are not damaged. Heart rate-85 / min. In the lungs to the left of the lower angle of the scapula, the pulmonary sound
is shortened, the breathing is weakened. X-ray of the lungs: homogeneous darkening from the angle
of the scapula. The puncture resulted in hemorrhagic fluid. Radiography after the puncture is homogeneous,
without clear contours, infiltration of the left root. What is the most likely diagnosis?
100% Left lung tumor
0% Traumatic pleurisy
0% Left-sided post-traumatic pneumonia
0% Pulmonary embolism
0% Infiltrative tuberculosis, tuberculous pleurisy

437.A 16-year-old boy was admitted to the hematology department complaining of pain in the right
shoulder joint that had occurred after a bruised joint. From the anamnesis, it is known that such a clinical
picture has been observed repeatedly since early childhood. Objectively: the joint is enlarged in
volume, sharply painful on palpation. In the blood: Er.- 3,7•1012/ l, Hb-110 g / l, blood clot.- 115 • 109/l,

https://translate.yandex.com/en/doc 97/540
22:20 ,27.6.2023 �� C : ; 5 B
lake. -6.9 * 109/l, WSE-25 mm / h. Prothrombin index-90%, recalcification time-280 min.,
blood clotting time-38 min., fibrinogen-3.5 g / l. what is the most likely diagnosis?
101

Downloaded from the site - online testing step

100% Hemophilia
0% Immune coagulopathy
0% Thrombocytopathy
0% Immune thrombocytopenia
0% Hemorrhagic vasculitis

438. a patient after falling from a height of the second floor, when pressing on the wings of the pelvis
, shows pathological mobility, a symptom of a "stuck" heel on the left, external rotation and
shortening of the left lower limb. What is the most likely diagnosis?
100% Vertical pelvic fracture (Melgen type)
0% Marginal fracture of the iliac wing
0% "Butterfly" pelvic bone fracture
0% Acetabular fracture with hip dislocation
0% Left hip neck fracture

439. a 20-year-old woman complains of pain, unpleasant heavy sensation in the heart, rapid
heartbeat. ECG - no pathology. Sedatives eased the pain,but there was still anxiety,
the certainty of a serious heart condition, the fear of death, the expectation of another attack and the fear
of it. What is the leading psychopathological syndrome?
100% Cardiophobic
0% Hypochiondrial
0% Obsessive
0% Hysterical
0% Depressive

440. a 36-year-old man passes a professional examination before employment, and does not file any complaints.
General analysis of urine: light yellow, transparent, specific gravity-1008, pH-6.5; protein-0.6 g / l,
ep. 18-20 v / s, changed; leuc.- 2-3 v / s; cylinders-2-3 v/s, granular. What is the most likely
diagnosis?
100% Chronic glomerulonephritis
0% Acute glomerulonephritis
0% Chronic pyelonephritis
0% Amyloidosis of the kidneys
0% Urolithiasis

441. A 24-year-old patient went to the polyclinic complaining of epigastric pain 1-1. 5
hours after eating, night pain, and frequent vomiting, which gives relief. Smokes a lot
and drinks alcohol. Objectively: the language is clean. Superficial palpation of the abdomen revealed
muscle defanse, pain on the right above the navel. Laboratory: reaction to occult blood in the stool
is positive. What is the most likely diagnosis?
100% Peptic ulcer disease
0% Chronic gastritis
0% Chronic cholecystitis
0% Chronic enteritis

https://translate.yandex.com/en/doc 98/540
22:20 ,27.6.2023 �� C : ; 5 B
102

Downloaded from the site - online testing step

0% Chronic colitis

442. A 25-year-old patient was admitted to the infectious diseases department on day 3 with
complaints of headache, lower back pain, calf muscles, high fever, and chills. The condition
is moderate to severe. Icteric sclera. The pharynx is hyperemic. The tongue is dry, covered with a dry
brown coating. My stomach is swollen. Liver + 2 cm. The spleen is not enlarged. Palpation of the muscles,
especially the calf muscles, is painful. The urine is dark. Normal-colored feces. What is the most likely
diagnosis?
100% Leptospirosis
0% Viral hepatitis A
0% Malaria
0% Infectious mononucleosis
0% Yersiniosis

443. A 60-year-old patient was admitted to the clinic complaining of shortness of breath, heaviness in the right
hypochondrium, and abdominal enlargement. The phenomena grew during the year. During auscultation of the heart, a
presystolic gallop rhythm is observed. Objectively: swelling of the cervical veins, ascites, palpable liver and
spleen. What disease needs to be differentially diagnosed?
100% Constrictive pericarditis
0% Cirrhosis of the liver
0% Lung cancer with pleural growth
0% Chronic pulmonary heart disease
0% Pulmonary embolism6oem6

444. A 49-year-old polygraph worker went to the district therapist with complaints of
low-grade fever, general weakness, dizziness and loss of appetite. Objectively:
gray skin tone, gray-lilac border on the gums. From the gastrointestinal tract:
constant colic in the epigastrium, periodic constipation. In the blood: Er.- 3,8•1012/ l, Hb-105 g / l,
CP-0.9, basophilic granularity of red blood cells. Increased amount of hematoporphyrin in the urine. What
is the most likely medical condition?
100% Lead poisoning
0% Chronic gastritis
0% Chronic enterocolitis
0% Hypochromic anemia
0% Vegetative-vascular dystonia

445. A 29-year-old female patient was admitted to the gynecological department with complaints of lower
abdominal pain and spotting before and after menstruation for 5 days.
The disease is associated with an abortion 2 years ago. Anti-inflammatory treatment had no effect
. During bimanual examination: the uterus is enlarged, dense, painful, smooth. During
hysteroscopy, dark red holes are visible in the area of the uterine floor, from which dark blood is released.
What is the most likely diagnosis?
100% Internal endometriosis
0% Polymenorrhea
0% Hypermenorrhea

103

https://translate.yandex.com/en/doc 99/540
22:20 ,27.6.2023 �� C : ; 5 B

Downloaded from the site - online testing step

0% Submucous fibromatous nodule


0% Dysfunctional uterine bleeding

446. a farmer who injured his right foot while working in the field went to the emergency room.
He doesn't remember when he was vaccinated, and he didn't serve in the army. Examination of the right foot revealed a contaminated
wound 5-6 cm long with uneven edges. What is the doctor's next strategy?
100% Enter tetanus toxoid and tetanus serum
0% Enter tetanus toxoid
0% Enter tetanus serum
0% Only surgical treatment of the wound
0% Prescribe an antibiotic

447. a 53-year-old patient complains of weakness, lack of appetite, weight


loss, fever. Objectively: polylymphadenopathy, dough-like nodules, hepatomegaly. In
the blood: Er.- 4,0•1012/ l, Hb-110 g / l, lake -100 * 109/l, B.-1%, E.-1%, P.-4%, S. -42%, M.-25%,
L. 50%, Botkin-Gumprecht shadows. What is the most likely diagnosis?
100% Chronic lymphocytic leukemia
0% Acute leukemia
0% Chronic myeloid leukemia
0% Lymphogranulomatosis
0% Myeloma disease

448. a 27-year-old patient complains of bleeding from the gums, nosebleeds, multiple bruises on the skin
of the anterior surface of the trunk and limbs, and severe general weakness. In the blood: Hb-64 g / l,
Er. -2.5•1012/l, reticulocytes-16%, platelets - 30•109/l, WSE-25 mm/hour. What is the most
effective way to treat this pathology?
100% Splenectomy
0% Calcium Dobesylate
0% Thrombomass transfusion
0% Cytostatics
0% B vitamins

449. a 50-year-old patient suddenly developed pain in the occipital region, vomiting. Objectively:
sopor, hyperemia of the facial skin, blood pressure-210/120 mm Hg, Ps-60 / min, tense, body temperature
-37.8 oC. Horizontal nystagmus. Reflexes of oral automatism are expressed. Tendon
reflexes are uniform. Stiffness of the occipital muscles, bilateral Kernig's symptom. What
is the preliminary diagnosis?
100% Subarachnoid hemorrhage
0% Hemorrhagic parenchymal stroke
0% Subdural hematoma
0% Acute hypertensive encephalopathy
0% Meningococcal meningitis

450. in a 58-year-old patient undergoing chemotherapy for cancer

104

https://translate.yandex.com/en/doc 100/540
22:20 ,27.6.2023 �� C : ; 5 B
Downloaded from the site - online testing step

the disease appeared sore throat. On examination, areas of necrosis were found in the pharynx on the tonsils and mucous membrane
. A lot of carious teeth. Neutrophilic granulocytes are almost completely absent in the blood against the background of leukopenia
. White blood cells are mainly represented by lymphocytes and
monocytes. Which of these diseases should you think about?
100% Agranulocytic angina
0% Lacunar angina
0% Vincent-Simanovsky's angina
0% Syphilitic sore throat
0% Diphtheria

451. A 47-year-old male complains of periodic headache, palpitations, and


cardiac arrhythmias. Objectively: heart sounds are muted, heart rate-108 / min, AT-170/100 mm Hg on
the ECG: left ventricular hypertrophy, ventricular extrasystoles 10-15 / min. What drugs are most
appropriate to prescribe for the treatment of the patient?
100% b-blockers
0% ACE inhibitors
0% Diuretics
0% Sympatholytics
0% Nitrates

452. A 66-year-old patient suddenly developed chest pain


, shortness of breath, palpitation, and weakness a week after an adenomectomy. Subsequently, there was pain in the right hypochondrium.
objectively:
the condition is severe, diffuse cyanosis, shortness of breath at rest. BH-38 / min. The neck veins are swollen. The borders of the heart are
not
changed,
100%the tones are clear,embolism
Pulmonary tachycardia. Ps-124 / min., small filling, rhythmic. AT-80/40 mm Hg on
the ECG: signs of blockage of the right bundle branch. What is the most likely diagnosis?
0% Myocardial infarction
0% Dissection aneurysm
0% Cardiac asthma
0% Pneumonia

453. A 38-year-old patient has been working at a reinforced concrete products factory for 13 years. The work
is connected with the action of general and local vibration. Complains of general weakness,
headache, numbness of the limbs, dizziness, periodic pain in the heart. Objectively: Ps-84/min,
rhythmic, blood pressure-150/100 mm Hg. heart tones are muted. The skin of the limbs is cold to the touch.
Cyanotic on the hands, when pressed, white spots remain. Palpation of the lower leg muscles
shows pain. Pain, tactile, and temperature sensitivity are reduced. ECG: signs
of myocardial hypoxia. On the radiograph of the tibia-metatarsal joints, moderate
osteoporosis is noted. What is the most likely diagnosis?
100% Vibration sickness
0% Polyneuritis
0% Myopathy
0% Neurocirculatory dystonia
0% Funicular myelosis

105

Downloaded from the site - online testing step

454. A 27-year-old patient was admitted to the hospital with complaints of sharp pain in the right half
https://translate.yandex.com/en/doc 101/540
22:20 ,27.6.2023 �� C : ; 5 B
of the scrotum, its enlargement, body temperature rising to 39 ° C, sweating, and general
weakness. I got sick after hypothermia, when the pain in the gate appeared and gradually
increased, the temperature rose, and the right half of the gate increased. There were no injuries. Objectively:
the right testicle with the appendix is enlarged, dense, sharply painful during palpation, the skin
of the scrotum is hyperemic. In the blood: leukocytosis with a shift of the formula to the left. What disease should
I think about?
100% Acute orchiepididymitis
0% Acute venous thrombosis of the spermatic cord
0% Acute funicular railway
0% Acute orchitis
0% Acute colliculitis

455. a 36-year-old patient complains of a skin rash that appeared a week ago and
does not subjectively bother him. Objectively: on the skin of the palms and soles - multiple lenticular
disseminated papules that do not rise above the skin level, stagnant red color,
covered with a cluster of horny scales, dense during palpation. What is the preliminary diagnosis?
100% Secondary syphilis
0% Multiple warts
0% Palmar-plantar psoriasis
0% Rubrophytosis of the palms and soles
0% Frozen palms and soles

456. a student of the 10th grade is not vaccinated, suddenly fell ill: the temperature rose to 38oC,
headache, cough, runny nose, general weakness and lacrimation are bothering. After 4 days
, a spotty-papular rash appeared on the skin of the face. On the soft palate-enanthemum.
Slightly enlarged cervical lymph nodes. Tachycardia, hard breathing, liver and spleen are not
enlarged. What is the preliminary diagnosis?
100% Measles
0% Enterovirus infection
0% Scarlet fever
0% Rubella
0% Infectious mononucleosis

457. A 32-year-old patient has been experiencing suffocation attacks for 3 years, which are almost not stopped
by berotec. In recent months, he has noticed joint pain and sensitivity disorders of the skin of the shins and
feet. Ps-80 / min, AT-210/100 mm Hg in the blood: eosinophilia 15%. What kind of disease
can you think about?
100% Nodular periarteritis
0% System red shepherd dog
0% Systemic scleroderma
0% Dermatomyositis
0% Wegener's disease

458. a patient complains of rashes all over the body, itching, and a feeling of tightness in the skin,

106

Downloaded from the site - online testing step

which a month after the psychotrauma appeared on the scalp, then on


the extensor surface of the upper and lower extremities, on the back. Objectively: the rash is represented
by a monomorphic papular rash, bright red in color, rounded in shape, measuring

https://translate.yandex.com/en/doc 102/540
22:20 ,27.6.2023 �� C : ; 5 B
from 0.5 to 3 cm in diameter. The surface of the papules in the center is covered with silvery-white scales, along
the periphery - a bright red rim. What is the most likely diagnosis?
100% Psoriasis
0% Lichen planus erythematosus
0% Secondary papular syphilis
0% Pink lichen
0% Seborrhea

459.An unconscious 11-year-old girl was taken to the sanitarium. Objectively: Kusmaul's breathing
, reduced eyeball tone, heart tones are deaf, arrhythmia. In the blood: urea-6.8
mmol/ l, residual nitrogen-17.3 mmol/ l, creatinine-0.049 mmol/ l, total bilirubin-18.5
mmol/l due to indirect, AsAT-0.32 mmol/l, ALT-0.26 mmol/l, Sugar-20 mmol / l
. what is the cause of the development of these symptoms?
100% Ketoacidotic coma
0% Acute renal failure
0% Acute vascular insufficiency
0% Hypoglycemic coma
0% Hepatic coma

460. a full-term child developed hemorrhages on the skin on the 3rd day, bloody vomiting, ground blood.
Hemorrhagic disease of newborns was diagnosed. What medication does your child need as
an emergency?
100% Fresh frozen plasma
0% Red blood cell mass
0% Platelet mass
0% Ethamzylate
0% Calcium Chloride

461. An 8-year-old girl was hospitalized in the cardiology department. Objectively:


there is a skin lesion over the extensor surfaces of the joints with atrophic
scars, depigmentation, symmetrical damage to skeletal muscles (weakness, pain, swelling,
hypotrophy). Which disease is most characterized by these changes?
100% Dermatomyositis
0% Systemic scleroderma
0% Nodular periarteritis
0% System red shepherd dog
0% Reiter's disease

462. A 43-year-old patient was diagnosed with stage I silicosis 2 years ago. Complains of
increased shortness of breath, pain in the subscapular areas. The radiograph shows
a diffuse strengthening and deformation of the pulmonary pattern, against which numerous nodular shadows
with a diameter of 2-4 mm are observed. Compaction of the interparticle pleura on the right. The roots of the lungs are compacted,

107

Downloaded from the site - online testing step

advanced ones. What is the radiological form of pneumoconiosis in this case?


100% Nodular
0% Interstitial
0% Interstitial nodular
0% Nodal

https://translate.yandex.com/en/doc 103/540
22:20 ,27.6.2023 �� C : ; 5 B
0% Tumorozna

463. A 55-year-old man complains of constant shortness of breath, which increases with physical
exertion, and a morning cough with a small amount of mucopurulent sputum. He has been ill
for about 10 years. Smokes for 40 years. Objectively: BH - 20 / min, height-175 cm, weight -95 kg.
The chest is barrel-shaped. Above the lungs on the background of weakened vesicular respiration-
a small number of scattered dry wheezes. What mechanism of alveolar ventilation disturbance is
conductive?
100% Bronchial obstruction
0% Impaired chest mobility
0% Neuromuscular dysfunction
0% Violation of the function of the respiratory center
0% Restrictive insufficiency of external respiration

464.A two-month-old child was admitted to the surgical department with complaints of a temperature
of 38 ° C, edema, hyperemia and lack of movement in the left shoulder joint. In the anamnesis:
omphalitis, pseudofurunculosis. What is the most likely diagnosis?
100% Metaepiphyseal osteomyelitis
0% Erb's palsy
0% Plexit
0% Phlegmon of the newborn
0% Shoulder injury

465.Ultrasound examination of a 3-year-old child with periodic abdominal pain and pyuria revealed an enlarged
kidney and rounded cavities connected to an enlarged bowl. The right kidney is not
changed. What is the most likely diagnosis?
100% Hydronephrosis
0% Polycystic kidney disease
0% Multicystic fibrosis
0% Echinococcus
0% Coralloid kidney stone

466. a patient, a liquidator of an accident at a nuclear power plant, has a suspected lung pathology. There
is a need for an X-ray examination. What method of examination is
optimal for this patient?
100% Radiography
0% Fluorography
0% X-ray examination
0% Computed tomography

108

Downloaded from the site - online testing step

0% Nuclear magnetic resonance

467. A 24-year-old man complains of loose stools up to 20 times a day with the presence of blood
and mucus, tenesmus, general weakness, weight loss, pain in the knee joints. He has been ill for 2 years,
the disease proceeds with periodic exacerbations. Objectively: body temperature 38.4 oC,
BH - 20/min, heart rate-82 / min. Dry skin, aphthous stomatitis. Liver + 2 cm. Segments

https://translate.yandex.com/en/doc 104/540
22:20 ,27.6.2023 �� C : ; 5 B
of the large intestine are cramped and painful. What is the most informative diagnostic method
100%
for this disease?Colonoscopy
0% Bacterial culture of feces
0% Coprogram
0% Rectoromanoscopy
0% Irrigoscopy

468. a 51-year-old patient complains of a feeling of heaviness in the epigastrium after eating,
belching with a rotten smell, vomiting after overeating. For a long time
, he suffers from a pyloric stomach ulcer with frequent exacerbations. Objectively: body weight is reduced, the lower
border of the stomach is 4-5 cm below the navel, splashing noise on an empty stomach. What complication of peptic
ulcer disease has the patient developed?
100% Pylorostenosis
0% Ulcer perforation
0% Perivisceritis
0% Penetration
0% Malignancy of the ulcer

469. A 59-year-old patient complains of pain in the left eye and the left side of the head, a significant
decrease in vision in the left eye, nausea and vomiting. Visual acuity of the right eye is 1.0. Visual acuity
of the left eye-0.03, with correction does not improve. Intraocular pressure of the right eye -21 mm
Hg, left eye-65 mm Hg in the left eye on the sclera stagnant injection. The cornea is swollen
and thickened. The front camera is shallow, moist, and transparent. The pupil is dilated, does not react to light,
the fundus is not visible. What is the most likely diagnosis?
100% Acute attack of glaucoma in the left eye
0% Acute iridocyclitis of the left eye
0% Intraocular tumor of the second century of the left eye
0% Endophthalmitis of the left eye
0% Panophthalmitis of the left eye

470. a patient has cirrhosis of the liver. Shortness of breath, swelling of both legs, ascites have appeared in the last two months.
The patient takes hepatoprotectors, glucocorticoids. What combination of medications is most appropriate
to add to your treatment?
100% Aldactone + furosemide
0% Lipocaine + Hypothiazide
0% Phenol + furosemide
0% Aldactone + ascorutin
0% Albumin + ascorutin

109

Downloaded from the site - online testing step

471. A 45-year-old patient suffered a large-sided myocardial infarction 3 months ago, complains
of periodic palpitations and shortness of breath with rapid gait. Objectively: the heart tones are somewhat
muted. AT-170/90 mm Hg liver at the edge of the costal arch,
no peripheral edema. On the ECG: heart rate-94 / min, cicatricial changes in the posterior wall of the left ventricle.
There is no concomitant pathology. What combination of medications can be prescribed for the most effective
prevention of recurrent myocardial infarction?
100% Aspirin + obzidane
0% Ticlopidine + nitrong
0% Curantil + Nifedipine
0% Theonicol + nitrosorbite
https://translate.yandex.com/en/doc 105/540
22:20 ,27.6.2023 �� C : ; 5 B

0% Aspirin + Nitrates

472. A 64-year-old patient was admitted to the department with uterine bleeding and anemia. After a 12-year
absence of menstruation 7-8 months ago, serous-watery vaginal discharge first appeared
, then serous-bloody, such as "meat slop", pain in the lower abdomen appeared. What
pathology is most likely?
100% Uterine cancer
0% Incomplete abortion
0% Bubble drift
0% Chorionic epithelioma
0% Internal genital endometriosis

473. a 42-year-old patient was diagnosed with gastrointestinal bleeding, which was confirmed
by the Gregersen reaction and a decrease in the hemoglobin level to 90 g / l and the number of red blood cells to
2.7 * 1012/l. The general condition of the patient is satisfactory. What additional survey methods should
be performed?
100% Fibrogastroduodenoscopy
0% Laparoscopy
0% Determination of gastric juice acidity
0% Hollander's Test
0% Gastric X-ray examination

474. a 73-year-old patient suffers from atherosclerosis of the aorta and large arteries. Complains of pain
in the right shin. Gradually, the toes of the right foot turned black, and the foot was cold. The skin on the fingers
and in the distal part of the foot is dry, wrinkled, dark in color. Pulsation on the popliteal
artery is sharply weakened, and on the posterior tibial and dorsal arteries of the foot is absent. What
complication of atherosclerosis did the patient have?
100% Obliteration of the lower leg arteries and dry gangrene of the foot
0% Deep vein thrombosis of the right tibia
0% Wet gangrene of the right foot
0% Right popliteal artery embolism
0% Necrotic erysipelas of the right foot

475. A 40-year-old female patient reports heavy menstruation throughout the year, which is accompanied
by cramping pains in the lower abdomen. Bimanual examination during menstruation:

110

Downloaded from the site - online testing step

in the cervical canal, a formation with a diameter of up to 5 cm and a dense consistency is determined. The uterus
is enlarged up to 5-6 weeks of pregnancy, normal consistency, mobile, painful. Appendages are not
defined. The discharge is bloody and copious. What diagnosis can be assumed?
100% Submucous fibromatous nodule that is born
0% Abortion is in progress
0% Cervical cancer
0% Cervical fibroids
0% Algodismenorrhea

476. An employee who is under dispensary observation for duodenal


ulcer disease was granted a 24-day reduced-price permit for sanatorium

https://translate.yandex.com/en/doc 106/540
22:20 ,27.6.2023 �� C : ; 5 B
treatment at the enterprise. The duration of the employee's annual basic leave is 24
calendar days, and 4 days must be spent on travel to and from the sanatorium. What is the procedure
for registering temporary disability in this case?
100% A 4-day disability certificate is issued through the LCC
0% A 28-day disability certificate is issued through the LCC
0% The attending physician issues a disability certificate for 4 days
0% The attending physician issues a certificate and a "Sanatorium card" for 28 days
0% A 4-day certificate is issued through the LCC

477. when measuring the pelvis of a pregnant woman with a height of 160 cm and a weight of 58 kg, the following pelvic dimensions
were established:
26-29-31-20
100% cm.PelvicHow dimensions
do I interpret
arethe results?
normal
0% First degree of narrowing
0% Second degree of narrowing
0% Third degree of narrowing
0% Fourth degree of narrowing

478. a 6-year-old boy has a fever of up to 39 ° C, abdominal pain, cough, severe


shortness of breath of mixed type, lagging of the right half of the chest when breathing,
a dull sound on the lower right above the lungs, breathing is not listened to. The belly is soft. What is the most
likely diagnosis?
100% Pneumonia
0% Congenital lung disease
0% Right-sided atelectasis
0% Right-sided exudative pleurisy
0% Right-sided pneumothorax

479. a 25-year-old woman complains of fever up to 39oc, purulent discharge


from the vagina, which occurred on the 3rd day after an artificial abortion. Ps-100 / min, AT-120/70 mm Hg. the abdomen
during palpation is painful in the lower parts. In a bimanual study, the uterus is enlarged
up to 6 weeks of pregnancy, softened and painful. Uterine appendages are not detected due
to pain during the study. The discharge is pus-like, with an unpleasant smell. What is the most likely
diagnosis?
100% Acute endomyometritis

111

Downloaded from the site - online testing step

0% Acute adnexitis
0% Acute appendicitis
0% Torsion of the ovarian cyst
0% Ovarian apoplexy

480. the patient has a sad expression on her face, slow thinking. Notes a depressed mood,
decreased performance, and decreased appetite. Thoughts about one's own inferiority,
hopelessness of one's life situation and pessimistic forecasts for the future prevail. Identify
a mental disorder:
100% Depression
0% Apathy
0% Asthenia
0% Dysphoria

https://translate.yandex.com/en/doc 107/540
22:20 ,27.6.2023 �� C : ; 5 B
0% Amnesia

481. a 37-year-old patient was taken to the department in an unconscious state. Got a closed
injury in the abdominal area. Suspected intra-abdominal bleeding, damage
to internal organs. They are preparing for an urgent surgical intervention. The doctor determined the
blood type using standard sera and obtained the following result: 0(i) - no
agglutination, A (II) - no agglutination, b (III) - no agglutination. What is the
victim's blood type?
100% 0 (s)
0% A(II)
0% B(III)
0% AB (IV)
0% -

482. a 26-year-old man complains of prickly spines when breathing, coughing, and shortness of breath.
Objectively: to-37.3 oC, HR-19 / min, HR = Ps-92 / min; AT-120/80 mm Hg. vesicular respiration.
On the left side,in the upper-lateral parts of the chest, during the inhalation and exhalation phase, a noise is heard,
which increases with pressure with a phonendoscope and persists after coughing. ECG - no
pathological changes. What is the most likely diagnosis?
100% Acute pleurisy
0% Intercostal neuralgia
0% Subcutaneous emphysema
0% Spontaneous pneumothorax
0% Dry pericarditis

483. a 55-year-old patient, a smoker, complains of coughing, sputum production (sometimes


streaked with blood), shortness of breath during physical activity. Objectively:
the right side of the chest has a lag in breathing, the supraclavicular fossa is retracted. Breathing on the right
is weak, thin, dry wheezing. Radiologically-reduced pneumatization of the upper lobe,
compaction associated with the root of the lungs. To verify the diagnosis, first of all, you should
conduct:
100% Trial treatment of tuberculosis

112

Downloaded from the site - online testing step

0% Single detection of mycobacteria


0% Transthoracic biopsy
0% Computer tomography
0% Radioisotope lung scan

484. A 13-year-old girl complains of an increase in body temperature up to 37.4 oC during


the last 2 months after acute respiratory viral infection. Objectively: thin, diffuse enlargement
of the thyroid gland of the second degree, its density on palpation, exophthalmos, tachycardia. What
pathological syndrome does the patient have?
100% Thyrotoxicosis
0% Hypothyroidism
0% Hypoparathyroidism
0% Hyperparathyroidism
0% Thymomegaly

https://translate.yandex.com/en/doc 108/540
22:20 ,27.6.2023 �� C : ; 5 B

485.a full-term boy developed moderate jaundice of the skin and mucous membranes on the 2nd day of life
. The child's general condition is not affected. In the blood: indirect hyperbilirubinemia -120
mmol / l. the child's blood type is A (II) Rh (+), the mother's blood type is B (III) Rh (+). What should be
the doctor's tactics?
100% Refrain from drug therapy
0% Assign enterosorbents
0% Prescribe prednisone
0% Prescribe cholekinetics
0% Perform a replacement blood transfusion

486. a 47-year-old female patient complains of a cough with purulent sputum discharge, pain in the lower
part of the left half of the chest, and a periodic increase in body temperature. He has been ill
for about 10 years. Objectively: distal phalanges of the fingers in the form of "drumsticks".
Which examination will be most informative for making a diagnosis?
100% Bronchography
0% Bronchoscopy
0% Overview X-ray of the lungs
0% Puncture of the pleural cavity
0% Bacteriological examination of sputum

487. when determining blood groups, the isohemagglutination reaction was positive with
standard sera of groups A(II) and b(III), and negative with groups 0(I) and AB(II). What
does this result indicate?
100% Unsuitability of standard serums
0% First blood type
0% Second blood type
0% Third blood type
0% Fourth blood type

113

Downloaded from the site - online testing step

488. a man with a stab wound in his right foot turned to the surgeon. I stepped on
a board with a nail two hours ago. The outpatient patient's card indicates that he received a full
course of tetanus vaccination 3 years ago. How is emergency tetanus prevention performed in this case
?
100% Not being held
0% Enter 0.5 ml of tetanus toxoid
0% Enter 1.0 ml of tetanus toxoid
0% Enter 3000 IU tetanus serum
0% Enter 1 ml of toxoid and 3000 IU of serum

489. A 32-year-old woman went to the doctor complaining of skin lesions on her torso, face and
limbs after oral Biseptol administration. A diagnosis of drug-induced
toxicoderma was established. What treatment methods are most effective for the patient?
100% Extracorporeal therapy
0% Anti-inflammatory therapy
0% Antihistamine therapy
0% Detoxification therapy

https://translate.yandex.com/en/doc 109/540
22:20 ,27.6.2023 �� C : ; 5 B
0% Immunotherapy

490. A 2-year-old child suddenly developed a cough, stridorotic breathing,


a call to vomit, and cyanosis after playing with a mosaic. What will your doctor suspect first?
100% Foreign body aspiration
0% Acute laryngitis
0% Acute laryngotracheitis
0% Pneumonia
0% Whooping cough

491. a patient complains of swelling of the lower extremities, brittle nails, dry skin,
memory loss, and weakness. Monthly periods are absent from the moment of delivery. Objective: hypotrophy
of the genitals and mammary glands. From the anamnesis: 2 years ago, during childbirth, there was a massive
bleeding of more than 2000 ml. What is the most likely diagnosis?
100% Sheehan's syndrome
0% Hypothyroidism
0% Infantilism
0% Cardiovascular insufficiency
0% Postpartum obesity

492. A 27-year-old patient is undergoing inpatient treatment for obliterating


endarteritis. He has been ill for 2 years. The effect of conservative treatment is not noted. What
surgical intervention can be most effective?
100% Lumbar sympathectomy
0% Periarterial sympathectomy
0% Removal of II-III thoracic sympathetic nodes
0% Adrenal gland resection

114

Downloaded from the site - online testing step

0% Removal of IV-V lumbar sympathetic nodes

493. as a result of the impact of the truck in the middle third of the left thigh, the victim
felt a sharp pain, which is significantly increased when trying to move. I couldn't move around on my own
. During clinical examination, hemorrhage, hip deformity of
the"breeches" type, sharp pain on palpation in the middle third of the left thigh, shortening of the lower
limb are determined. What is the most likely diagnosis?
100% Fracture of the femoral diaphysis
0% Soft tissue bruise and hematoma
0% Damage to the quadriceps femoris
0% Anterior hip dislocation
0% Central hip dislocation

494. residents of a locality located in the third climate zone have a massive
dental fluorosis lesion. At what concentration of F-Ion in drinking water can
fluorosis be observed?
100% 1.5 mg / l
0% 0.7 mg / l
0% 1.0 mg / l

https://translate.yandex.com/en/doc 110/540
22:20 ,27.6.2023 �� C : ; 5 B
0% 0.3 mg / l
0% 0.1 mg / l

495. a 27-year-old man looks older than his age, is irritable," sniffs
powder " for a long time, after which a state of euphoria occurs with a feeling of increased strength, activity, acceleration
of the flow of thoughts, talkativeness, desire for activity, motor excitement, with a feeling
of lightness in the whole body. What is the most likely diagnosis in this case?
100% Cocaine addiction
0% Morphinism
0% Hashish addiction
0% Barbituromania
0% Nicotinomania

496. a patient with acute leukemia developed jaundice


and dyspeptic disorders during cytostatic treatment. Moderate increase and soreness of the liver, increased levels
of at least 40 mmol were detected./l) and indirect (10 mmol/l) bilirubin, ALT - 2.1 mmol/g-l,
AsAT2, 2 mmol/g-l. the reaction to hepatitis B and C antigens is negative. What complication did
the patient have?
100% Toxic hepatitis
0% Viral hepatitis
0% Acute cholecystitis
0% Acute pancreatitis
0% Hemolytic anemia

497. An electric gas welder welds large-sized structures in an uncomfortable working position,

115

Downloaded from the site - online testing step

bent over, in an unfavorable acoustic environment eq. = 83.2 dBA (GDR - 80 dBA).
The concentration of harmful substances in the air of the working area exceeds the maximum permissible concentration by 3-4 times.
Identify
priority
100% actions Local
to improve working
exhaust conditions:
ventilation
0% Aeration system
0% Interior cladding with sound-and-clay materials
0% Rationalization of the work and rest regime
0% Use of personal protective equipment

498. A 39-year-old patient underwent surgery for acute appendicitis three days ago.
Postoperative period without complications, noted some irritability, restless sleep.
In the evening, on the third day after the operation, I became restless, fussy, and heard girls singing in the street
. He kept the" white flies" away from him, shouting that the room was full of rats. He thinks he's at
home, disoriented in time. Somatically: hyperhidrosis, blood pressure-160/100 mm Hg,
body temperature-37.4 oC, tachycardia. According to relatives, he has been abusing alcohol for many years. Diagnose
the patient's mental disorder:
100% Alcoholic delirium
0% Toxic-infectious psychosis
0% Hypertoxic schizophrenia
0% Acute schizo-affective psychosis
0% Ganzer syndrome

https://translate.yandex.com/en/doc 111/540
22:20 ,27.6.2023 �� C : ; 5 B

499. a 14-year-old boy with chronic tonsillitis and sinusitis developed feelings
of heart failure and additional pulse beats. Heart rate-83 / min. On the ECG: after every two
sinus contractions, impulses regularly occur in which the P Wave is absent, QRS is extended
for more than 0.11 seconds, the discordant T Wave is sharply deformed, after which a complete
compensatory pause is recorded. Specify the nature of rhythm disturbances:
100% Trigeminal extrasystole
0% Bigeminia-type extrasystole
0% Partial AV blockage
0% Complete AV blockade
0% Left bundle branch block

500. A 38-year-old patient suddenly developed pain in the left side of the chest, suffocation.
Objectively: the condition is moderate, Ps-100 / min, AT-90/60 mm Hg, breathing on the left is not
heard. Chest X-ray shows a collapse of the left lung to 1/2. what treatment
should be prescribed to the patient?
100% Passive drainage of the pleural cavity
0% Rest, resorption therapy
0% Pleural punctures
0% Surgical treatment
0% Active drainage of the pleural cavity

501. A 49-year-old patient is undergoing surgery for thyroid cancer. During the revision of the tumor and
rapid diagnostics, it was found that the degree of spread of the process is T3NaM0,

116

Downloaded from the site - online testing step

morphologically-papillary form of carcinoma. What should be the scope of radical


surgical intervention?
100% Thyroidectomy
0% Subtotal resection of the gland
0% Resection of the affected lobe
0% Enucleation of the tumor
0% Resection of the affected lobe with isthmus

502. after a conversation with the mother of a seven-month-old boy who is on natural
feeding, the pediatrician found out that the child is fed 7 times a day. What number of feedings
is set for a child of this age?
100% 5 times
0% 3 times
0% 4 times
0% 6 times
0% 7 times

503. A 40-year-old patient underwent surgery for lumbar phlegmon. His


body temperature rose sharply to 38oC again, intoxication symptoms appeared
, and the number of white blood cells in the blood increased. In the wound, which was almost cleared of necrotic tissues and
filled with granulations, putrefactive discharge appeared, the granulations became pale. What
complication did this patient have?
100% Sepsis

https://translate.yandex.com/en/doc 112/540
22:20 ,27.6.2023 �� C : ; 5 B
0% Putrid phlegmon
0% Erysipelas
0% Allergic reaction
0% Erysipeloid

504. a 40-year-old patient suffers from rheumatism. Complains of severe pain in the left eye, especially
at night, blurred vision, photophobia, lacrimation. The disease does not connect with anything.
Objectively: weak pericorneal injection, smoothness of the iris relief, change in its color.
What is the most likely diagnosis?
100% Iridocyclite
0% Irit
0% Keratitis
0% Choroiditis
0% Acute glaucoma attack

505. a 63-year-old patient complains of pain in the right groin area, bloating, weakness,
persistent constipation for more than 7 months. Objectively: the skin is pale and dry. In the right iliac region
, a 5-7 cm formation is palpated, sedentary, painless. Auscultation: increased intestinal
noise. In the blood: Er -2.9*1012/l, Hb-80 g / l, WZ-32 mm / year. Blood in the stool. What is the most likely
diagnosis?
100% Colorectal cancer
0% Crohn's disease

117

Downloaded from the site - online testing step

0% Polyp of the cecum


0% Cancer of the right kidney
0% Spastic colitis

506. the patient has severe pain in the right ear, fever - 38oC, suppuration from the ear and
hearing loss. He has been ill for 13 days. Objectively: the right auricle is protruding, swelling and
soreness of the soft tissues of the mastoid process on the right. In the external auditory canal, pus,
hyperemia and perforation of the tympanic membrane, protrusion of the posterior-upper wall of the auditory canal in the
bone department. What is the most likely diagnosis?
100% Right-sided mastoiditis
0% Right-sided acute purulent otitis media
0% Right acute diffuse external otitis media
0% Furuncle of the external auditory canal on the right
0% Right ear lymphadenitis

507. in a patient with a widespread post-injection abscess of the right buttock, a surgeon performed
first a puncture, and then a surgical opening of the abscess with emptying the purulent contents and
setting up a constant washing of the cavity with antiseptic solutions, administration
of proteolytic enzymes. He prescribed broad-spectrum antibiotics and immunocorrective
therapy. Which of the performed and prescribed actions of the surgeon is the leading component
of sepsis prevention?
100% Surgical opening of the abscess
0% Abscess puncture
0% Topical application of proteolytic enzymes
0% Prescribing antibiotic therapy
0% Appointment of medical immunocorrection
https://translate.yandex.com/en/doc 113/540
22:20 ,27.6.2023 �� C : ; 5 B

508. A 25-year-old patient complains of pain and pain during urination, purulent
discharge from the urethra that appeared 2 days ago. Objectively: the urethral sponges are swollen, hyperemic,
and there is a profuse purulent discharge from the urethra. Preliminary diagnosis: fresh acute gonorrheal urethritis. What
additional data from clinical examinations can help clarify the diagnosis?
100% Microscopy of urethral discharge
0% Serological blood testing
0% General blood test
0% Detection of coccoid cells by microscopy of secretions
0% Detection of yeast cells by microscopy of secretions

509. a 54-year-old patient with ulcerative bleeding, blood type 0 (I), Rh-negative,
received 200 ml of red blood cell mass transfused. A few hours after the transfusion, the patient experienced the following
symptoms: a weakened pulse, a decrease in blood pressure to 80/50 mm Hg, pallor
of the facial skin, dizziness, headache, pain throughout the body, especially in the lumbar
region. A year ago, the patient already underwent a blood transfusion for bleeding. What
complication are we talking about?
100% Rh-factor incompatibility of blood
0% Blood incompatibility by group factors

118

Downloaded from the site - online testing step

0% Poor quality of donated blood


0% Anaphylactic shock
0% Citrate intoxication

510.Trends in changes
in the overall mortality rates of various population groups living in different administrative territories have been studied for a long time.
Which of the statistical methods can be used for this purpose?
100% Dynamic series analysis
0% Analysis of standardized indicators
0% Correlation and regression analysis
0% Estimation of the probability of a difference in indicators
0% Analysis of relative value levels

511. The following air quality indicators were recorded in the residential area:
the air temperature is 35°C, the air velocity is 0.1 m / s, the relative humidity is 95 %,
the surface temperature of the walls and ceiling is 33°C. Which of the means of heat transfer by the body will be the main one in
these conditions?
100% Radiation
0% Evaporation
0% Conducting system
0% Convection
0% Sweating

512. A 39-year-old pregnant woman was admitted with a 12-week pregnancy and complaints of spotting
from the genital tract. When viewed in mirrors: on the cervix around an external eye
the size of a 2-kopeck coin, uneven lumpy growths in the form of cauliflower with
foci of necrosis and hemorrhages that bleed easily. Discharge is bloody, liquid, with

https://translate.yandex.com/en/doc 114/540
22:20 ,27.6.2023 �� C : ; 5 B
an ichorous smell. The body of the uterus is enlarged up to 12 weeks of pregnancy, softened, mobile.
Applications are not defined, and the parameters are free. What is the most likely diagnosis?
100% Pregnancy is 12 weeks. Cervical cancer
0% Ectopic pregnancy
0% Pregnancy is 12 weeks. Initial spontaneous miscarriage
0% Cervical pregnancy
0% Bubble Toe Cap

513. examination of a 25-year-old woman who applied to a antenatal clinic for


lack of pregnancy during 3 years of regular sexual activity revealed: increased
body weight, male-type pubic hair growth, increased hip
hair, dense, enlarged ovaries, monophasic basal temperature. What is the most likely diagnosis?
100% Sclerocystic ovaries
0% Inflammation of the uterine appendices
0% Adrenogenital syndrome
0% Premenstrual syndrome
0% Gonadal dysgenesis
514. a 50-year-old patient complains of acute epigastric pain and frequent vomiting,

119

Downloaded from the site - online testing step

sharp general weakness. Gets sick for 2 days. Objectively: sharp pallor of the skin,
dry tongue. Ps-100 / min., weak filling, blood pressure-110/70 mm Hg. the abdomen on palpation is soft,
moderately painful in the epigastrium and mesogastrium, there are no symptoms of irritation of the peritoneum.
Peristalsis is significantly enhanced in the left mesogastrium. In the X-ray survey, there are Cloiber bowls
on the left. What pathology should I think about?
100% Intestinal obstruction
0% Acute pancreatitis
0% Breakthrough duodenal ulcer
0% Acute cholecystitis
0% Food poisoning

515. a laboratory study of drinking water found that the dry residue is 1200
mg / l, the total hardness is 5.5 mg-eq/l, the total alkalinity is 4.5 mg-eq/ L, the magnesium content is 20 mg/l,
the fluorine content is 0.8 mg/l. does the drinking water meet the physiological adequacy indicators? Water
requirements of Dsanpi-Well?
100% Not responsible for overall mineralization
0% Not vvdpovvdat on indicators of hardness
0% Not vvdpovvdat on indicators of alkalinity
0% Not vvdpovvdat on the content of magnesium
0% Not responsible for the fluoride content

516. in the mine, sinking is performed by drilling and blasting operations and manual loading
of the destroyed rock. A team of 6 miners works
in the face, and hand tools are used when drilling holes. What degree of mechanization do these
working conditions correspond to?
100% The first
0% Another
0% Third
0% Fourth
0% Fifth

https://translate.yandex.com/en/doc 115/540
22:20 ,27.6.2023 �� C : ; 5 B

517. a worker who is under medical supervision for gastric ulcer disease
applied this year with an exacerbation of this disease in February, May and September.
How do I statistically register this case?
100% One article with a ( - ) sign when first accessed in the current year
0% Three statalons for each use of the ( -)sign
0% Three outpatient patient coupons with code 2
0% One outpatient card with code 2 and two tickets with code 3
0% One article with the first use of the ( + ) sign and two articles with the ( -)sign

518. the child is 12 years old. Complaints of dull aching pain in the epigastrium and right subcostal region,
which increases after eating fatty or fried food, headache, general weakness,
nausea, fever to subfebrile numbers. Palpation of the abdomen
shows resistance of the muscles in the right subcostal region, positive symptoms of Ker,
Ortner, Murphy. What is the most likely diagnosis?

120

Downloaded from the site - online testing step

100% Chronic cholecystitis


0% Acute appendicitis
0% Viral hepatitis
0% Acute gastritis
0% Acute pancreatitis

519. a doctor was assigned to conduct an analysis of the infectious morbidity of the population
subordinate to polyclinic No. 1. What documents will be used for the current analysis?
100% Confirmed f. 058 / O in SES
0% Journal of accounting for infectious diseases
0% Morbidity Report
0% Medicinal "notebooks of contact persons"
0% Referral to the Infectious diseases department

520. at the scene of an accident, a 45-year-old man is unconscious, the skin is cyanotic, the pulse on the carotid
arteries is not detected, independently stopped breathing a few minutes before the arrival of doctors,
the pupils are wide, there is no reaction to light. According to outsiders, the accident occurred 15-20 minutes ago,
all the time the patient had uneven breathing. What is the prognosis for
irreversible cortical changes in the patient?
100% Less than 4-5 minutes
0% After 4-5 minutes
0% After 5-6 minutes
0% After 6-7 minutes
0% More than 7 minutes later

521. A 32-year-old patient was admitted to the hospital 3 hours after being uniformly
exposed to ionizing radiation from a closed caesium-137 source for 10 minutes. Complaints
occurred 1.5 hours after exposure. Objectively: slight hyperemia of the sclera. Ps-82 / min.
AT-110/70 mm Hg in the blood at the end of the second day of hospital stay: ep.- 4•1012/ l, Hb-135
g/l, blood clot.- 230*109/l, lake.- 10*109/l, E.-2%, P.-5%, S.-80%, L.-7%, M.-6%, SZE-15 mm / year.
Blood amylase-200 mg / ml / hour. What group of drugs should be prescribed to the victim during
the primary reaction?
https://translate.yandex.com/en/doc 116/540
22:20 ,27.6.2023 �� C : ; 5 B

100% Antiemetics
0% Iodine preparations
0% Complexon antidotes
0% Antibiotics
0% Anticoagulants

522. A 5-year-old boy with acute lymphoblastic leukemia is completing the phase
of remission consolidation according to the polychemotherapy protocol. In the peripheral blood of leukemia.- 2,9•109/l,
blood clot.-120 * 109/l, normochromic anemia of the first century. in the bone marrow, 5% of blasts. What
kind of chemotherapy will be used for maintenance therapy?
100% 6-mercaptopurine
0% Cyclophosphamide
0% With methotrexate

121

Downloaded from the site - online testing step

0% Prednisone
0% Daunorubicin

523. A 24-year-old patient went to the clinic complaining of weight gain and increased
appetite. Objectively: hypersthenic build, body mass index 33.2 kg / m2, waist circumference 100
cm. The ratio of waist circumference to hip circumference is 0.95. what is the preliminary diagnosis?
100% Alimentary-constitutional obesity, first century, abdominal type
0% Hypothalamic obesity according to the Itsenko-Cushing type, II century, genoid type
0% Alimentary-constitutional obesity, III century, genoid type
0% Alimentary-constitutional obesity, II century, abdominal type
0% Hypothalamic obesity according to the Schenko-Cushing type, I century, abdominal type

524. a 27-year-old patient complains of periodic pain in the heart, palpitations, a feeling
of lack of air, and dizziness. A history of frequent acute respiratory infections. Objectively: Ps-98 / min, BP-100/70 mm
Hg. results of laboratory tests without special features. The level of T3, T4 is within the normal range.
The VEM test is negative, the reaction of the cardiovascular system is dystonic. What is the most
likely diagnosis?
100% Neurocirculatory dystonia
0% Stable angina pectoris of tension
0% Unstable angina pectoris
0% Myocarditis
0% Thyrotoxicosis

525. A 67-year-old woman suffering from hypertension suddenly developed a


headache and shortness of breath at night, which quickly turned into suffocation. Objectively: pale, perspiration drops on the forehead,
blood pressure-210/140 mm Hg, heart rate-120 / min., isolated dry wheezing above the lungs
, wet wheezing in the lower parts. The lower legs are pasty. What kind of emergency care is most appropriate in this
case?
100% Nitroglycerin and furosemide iv
0% Enalapril and furosemide IV
0% Digoxin and nitroglycerin IV
0% Labetalol and furosemide iv
0% Nitroglycerin iv and kapoten inside

https://translate.yandex.com/en/doc 117/540
22:20 ,27.6.2023 �� C : ; 5 B

526. a 25-year-old woman in labor, on the second day of the postpartum period. The first delivery, urgent,
passed without complications. What should be the character of lochias?
100% Blood cells
0% Blood-serous diseases
0% Mucous membranes
0% Purulent
0% Serous ones

527. a 59-year-old man suddenly developed lower back pain on the right side and macrohematuria,
a similar phenomenon that occurred several weeks ago. Objectively: to-37.7 oC, Ps-88 / min, AT-140/80 mm Hg.

122

Downloaded from the site - online testing step

No changes were found in the lungs and heart. The belly is soft, painless. Liver +1 cm. There is no swelling
of the lower extremities. Varicose veins of the spermatic cord on the right. In the blood: anemia,
acceleration of ESR, hypoproteinemia. In the urine: proteinuria, erythrocyturia. What is the most
likely diagnosis?
100% Kidney tumor
0% Chronic glomerulonephritis
0% Urolithiasis
0% Tuberculosis of the kidneys
0% Chronic hepatitis

528. A 1-year-old child born prematurely with signs of intrauterine infection


was diagnosed with chorioretinitis and hydrocephalus. Specify the most likely cause of the disease:
100% Toxoplasmosis
0% hiv infection
0% Viral hepatitis B
0% Mycoplasmosis
0% Chlamydia

529. An analysis of the organization of medical care in the regional center showed that every year 6.12%
of patients receive inpatient care for diseases that do not require round-the-clock
supervision and intensive care. What are the most appropriate organizational changes to
address this problem?
100% Development of inpatient isolation care
0% Amendment of the charter of outpatient clinics
0% Development of primary health care
0% Structural adjustment of specialized care
0% Strengthening the material and technical base of hospitals

530.A 20-year-old youth went to the hospital complaining of pain that occurs in the lower third
of the femur during exercise and at rest. Denies injury. Objectively:
normal-colored skin, pasty appearance and pain during deep palpation, reduced movement in the knee
joint. The radiograph of the distal femoral metaepiphysis shows a zone of destruction and spicules. In the blood:
immature cell forms, no signs of inflammation. What is the most likely diagnosis?
100% Osteogenic sarcoma
0% Hyperparathyroid dystrophy
0% Chronic osteomyelitis
https://translate.yandex.com/en/doc 118/540
22:20 ,27.6.2023 �� C : ; 5 B
0% Myeloma disease
0% Marble Disease

531. The district doctor found that the incidence


of influenza increased by 30% compared to the previous year. What relative indicator did he use in this case?
100% For visibility purposes
0% Intensive
0% Extensive

123

Downloaded from the site - online testing step

0% Ratio
0% Relative intensity

532. a 13-year-old girl complains of prolonged bleeding from the vagina for 3 weeks,
weakness, and dizziness. My last period was 4 months ago. In the blood: Hb-74 g / l,
Er. 2.8 * 109/l, blood clot.- 280•109/L. vaginally: the uterus is not enlarged, the appendages are unchanged. What
is the likely diagnosis?
100% Juvenile uterine bleeding
0% Abortion is in progress
0% Willibrandt's disease
0% Thrombocytopenic purpura
0% Werlhof's disease

533. bimanual examination of a 32-year-old female patient to the left of the uterus revealed a tumor
formation of a tightly elastic consistency, up to 10 cm in diameter, mobile and painless. Uterus and
appendages on the right without features. Menstrual and reproductive functions are not impaired. Somatically
healthy. What is the most likely diagnosis?
100% Tumor of the left ovary
0% Pyosalpinx
0% Hydrosalpinx
0% Ovarian cancer
0% Ectopic pregnancy

534. a 47-year-old employee had a heart attack. The diagnosis was made: myocardial infarction.
After completing the course of treatment, the patient developed a heart aneurysm, heart
failure of the second degree. Taking into account the nature of the disease, it was decided to send the patient for
recognition regarding the establishment of a stable disability. An
appropriate referral was issued for the patient. Who has the right to issue it?
100% Medical Consultation Commission
0% Attending physician independently
0% The attending physician together with the head of the department
0% Deputy Chief Physician for Health Expertise
0% Medical and Social Expert Commission

535. the head of the polyclinic department wants to conduct an expert assessment of the implementation
of medical and technological standards of patient care by doctors. What documentation should
it check for this purpose?

https://translate.yandex.com/en/doc 119/540
22:20 ,27.6.2023 �� C : ; 5 B
100% Medical records of outpatient patients
0% Control cards of the dispensary SPO strazhe
0% Statistical coupons for final (updated) diagnoses
0% Clinic doctor's work diaries
0% Outpatient patient vouchers

536. a 40-year-old man was taken by ambulance after a traffic accident

124

Downloaded from the site - online testing step

adventures with complaints of severe shortness of breath,pain in the right side of the chest and sternum,
cough with a small amount of bright foamy blood. Objectively: the patient's condition is extremely severe,
facial cyanosis, severe subcutaneous emphysema of the chest wall, neck and face, pulse
is rhythmic, 110 / min, AT-90/60 mm Hg. during auscultation of the lungs - on the right, breathing is sharply
weakened throughout, on the left-satisfactory. What is the most likely diagnosis?
100% Rupture of the right main bronchus
0% Traumatic pulmonitis
0% Right-sided pneumothorax
0% Right-sided total hemothorax
0% Aperture Gap

537. A 13-year-old boy suddenly experienced loss of consciousness and generalized clonic-tonic
convulsions. Objectively: pupils dilated, no photoreaction, facial cyanosis,
tongue biting, white foam coming out of the mouth, involuntary urination and defecation. From
the anamnesis, it is known that these attacks occur from the age of three several times a year, the child
constantly takes sedatives and anticonvulsants. The emergency medical service team
diagnosed an epileptic status. Choose a medication to stop an attack:
100% Diazepam
0% Korglikon
0% Atropine
0% Aminazine
0% Adrenaline rush

538. a 1-month-old boy with symptoms of agitation has a head circumference of 37 cm


and a large crown of 2x2 cm. The child regurgitates after feeding small portions of milk;
the stool is normal in composition and volume. Muscle tone is normal. What
is the most likely diagnosis?
100% Pylorospasm
0% Meningitis
0% Pylorostenosis
0% Microcephaly
0% Craniostenosis

539. on the first day after surgery for a diffuse toxic goiter, a patient
complained of difficulty breathing, cold sweat, and weakness. Objectively: pale skin,
body temperature 38.5 oC, PDR-25 / min, Ps-110 / min, AT-90/60 mm Hg. what complication of the early
postoperative period did the patient develop?
100% Thyrotoxic crisis
0% Hypothyroid crisis
0% Postoperative tetany

https://translate.yandex.com/en/doc 120/540
22:20 ,27.6.2023 �� C : ; 5 B
0% Acute thyroiditis
0% Compression of the trachea with a hematoma

540. A 45-year-old patient underwent conservative therapy for


gastroesophageal reflux disease. Which of these groups of drugs is the most effective?

125

Downloaded from the site - online testing step

is it advisable to use it for relapse prevention?


100% H2 blockers
0% Holinolytics
0% Reparants
0% Anti-Campylobacter agents
0% Antacids

541.During the year, 6,500 women were treated in the hospital of the gynaecological department and spent
102,000 bed days there. What indicator of the activity of the gynecological department can
be calculated from this data?
100% The average length of the patient's stay in the hospital
0% Average number of bed jobs per year
0% Number of beds by department
0% Turnover of beds
0% Planned number of bed operations per year

542.a patient with hoarse breathing (more


difficult breathing), skin cyanosis, tachycardia and arterial hypertension was admitted to the emergency department of the hospital. A
history
of bronchial asthma. An hour ago, I inhaled salbutamol and forgot to remove the cap, which
I aspirated
100% with Perform
a deep breath. Whattrick
a Heimlich are the doctor's actions?
0% Immediately perform a conicotomy
0% Call an anesthesiologist and wait for him to show up
0% Make an inhalation of beta-2-adrenomimetics
0% Enter dexamethasone subcutaneously

543. an ambulance doctor, when providing medical care to a person removed from the garrote
by relatives, noted: the absence of a pulse in the carotid arteries, lack of consciousness,
independent breathing, corneal reflexes, and the presence of cadaveric spots on the back and back
of the limbs. What signs can be used to determine the onset of death?
100% Presence of cadaveric spots
0% Lack of independent breathing
0% Lack of corneal reflexes
0% No pulse rate
0% Lack of consciousness

544.two people were killed in a road accident. A full-time forensic expert


was busy examining another scene of the incident, and therefore the investigator requested that the place and
corpses be examined in accordance with the current Code of Criminal Procedure of Ukraine:
100% Nearest doctor
0% A paramedic

https://translate.yandex.com/en/doc 121/540
22:20 ,27.6.2023 �� C : ; 5 B
0% Nurse of the surgical department
0% Pharmacist of the nearest pharmacy
0% Dental technician of the nearest dental hospital

126

Downloaded from the site - online testing step

545. a 3-year-old child with a well-tolerated acute respiratory viral infection, for which she received ampicillin,
developed rashes on the limbs and buttocks, swelling and soreness of the wrist
and knee joints, abdominal pain, and single vomiting. The rash is symmetrically located,
papular-hemorrhagic. History: food allergy. What research is necessary for
the appointment of pathogenetic therapy?
100% Lee-White blood coagulation
0% Determination of serum fibrinogen
0% Acute-phase indicators
0% Prothrombin index
0% Circulating immune complexes

546.on the 6th day of life, a child developed vesicles


filled with serous-purulent contents in the occiput,neck and buttocks, which tightly cover the skin. The child's general condition is not
affected. What disease should I think about?
100% Vesiculopustulosis
0% Pemphigus of newborns
0% Sweaty skin
0% Impetigo
0% Epidermolysis bullosa

547. a patient is preparing for surgery for varicose veins of the lower extremities.
Examination of the soles revealed flaky flaking along the skin folds. All the nails of the feet
are grayish-yellow, thickened and partially destroyed. What kind of dermatosis should I think about in
this case?
100% Rubromycosis
0% Pityriasis versicolor
0% Candidiasis
0% Microsporia
0% Microbial eczema

548.a 32-year-old patient complained to the dermatologist about the appearance of rashes in the area
of the corners of the lips, accompanied by minor itching. He's been ill for about three days. Objectively:
against the background of minor hyperemia, single small flictenes and surface erosions covered
with honey-yellow crusts. What is the most likely diagnosis?
100% Streptoderma
0% Cold sores
0% Atopic cheilitis
0% Dermatitis
0% Eczema

549.A first-time mother was admitted to the maternity hospital complaining of irregular, painful
contractions that last for 36 hours. Tired, I didn't sleep all night. Objectively: the fetal position
is longitudinal, the main presentation. Fetal heartbeat is clear, rhythmic-145 / min. Vaginal

https://translate.yandex.com/en/doc 122/540
22:20 ,27.6.2023 �� C : ; 5 B
examination: the cervix of the uterus is up to 3 cm long, dense, located sideways, the external peephole
is closed. Mucosal discharge. What is the most likely diagnosis?
127

Downloaded from the site - online testing step

100% Pathological preliminary period


0% Cervical dystocia
0% Primary weakness of labor activity
0% Physiological preliminary period
0% Secondary weakness of labor activity

550. A 35-year-old female patient reports weakness, sweating, low-grade


fever, pain and restricted movement in the left knee joint throughout the year. 2 years ago I was treated for
disseminated pulmonary tuberculosis. Objectively: the position of the left limb is forced,
the knee joint is enlarged, painful on palpation, movement in it is limited. Mantoux test Z 2
TO - papule 23 mm. In the lungs, focal shadows of increased intensity. What is the preliminary diagnosis?
100% Tuberculosis drives
0% Post-traumatic arthritis
0% Chronic osteomyelitis
0% Knee joint tumor
0% Rheumatoid arthritis

551. Rodilya in labor 8 hours. Labor activity is weak, contractions for 25 seconds, 2 contractions
in 10 minutes. During vaginal examination: the opening of the cervix is up to 6 cm, the head is presented.
During the examination, amniotic fluid with meconium impurities was spilled out. Fetal heartbeat 108 / min.,
dull. What are the tactics of labor management?
100% Emergency delivery by caesarean section
Continue conservative management of labor under constant monitoring
0%
of the fetus ' condition
0% Prescribe a gentle pacing with oxytocin solution
0% Apply obstetric forceps
0% Observation. Treatment of fetal distress

552. examination of a 77-year-old patient reveals jaundice of the sclera, symmetrical


paresthesias, and enlarged spleen. In the blood-anemia, macrocytosis, leukopenia, thrombocytopenia.
FGDS-atrophic gastritis with achlorhydria. What research should be conducted to verify
the disease?
100% Sternal puncture
0% Blood Vitamin B12 content
0% Blood folic acid content
0% Biopsy of the gastric mucosa
0% Blood bilirubin

553. during self-examination, a 39-year-old female patient discovered a tumor in the Lower inner
quadrant of the left breast. Palpation shows a mobile formation of up to 2 cm, with
clear contours, peripheral lymph nodes are not changed. What are the next tactics?
Breast ultrasound, mammography,
100%
fine needle aspiration biopsy
0% Anti-inflammatory therapy, physical therapy
0% Radical mastectomy

https://translate.yandex.com/en/doc 123/540
22:20 ,27.6.2023 �� C : ; 5 B
128

Downloaded from the site - online testing step

Ultrasound monitoring of the genitals during the entire course


0%
of antiestrogen therapy, systemic enzyme therapy, and herbal medicine
0% Dynamic monitoring

554.as a result of the accident at the nuclear power plant, a large area was contaminated with radionuclides, which
led to an increase in the radiation background. What diseases should
be expected to increase in the population of these territories?
100% Malignant neoplasms
0% ENT disease
0% Diseases of the gastrointestinal tract
0% Cardiovascular diseases
0% Eye diseases

555. a 24-year-old woman in labor. First delivery. The weight of the newborn is 5400 g. during childbirth
, a rupture of the pubic joint was diagnosed. What are the tactics in the postpartum period?
100% Immobilization in the "hammock" bandage on the pelvic area
0% Surgical treatment
0% Plaster immobilization
0% Fixing with an external locking device
0% Bed rest

556. A pregnant woman went to the antenatal clinic with complaints about a decrease
in fetal motor activity at the age of 34-35 weeks. Ultrasound revealed: the placenta is thickened, 52 mm,
granular, with petrifications, the head and trunk of the fetus have a double contour, the diameter of the abdomen
is increased, the fetal motor activity is sluggish, the fetal heart rate is 160-170/min. The pregnant woman's blood type is 0 (I)
Rh (). The titer of antiresus antibodies is 1: 128. what is the strategy?
100% Early delivery by caesarean section
0% Enter 1 dose (300 mcg) of anti-RHo (D) immunoglobulin
0% Perform a skin flap transplant from a man
0% Conduct infusion antihypoxic therapy for the fetus
0% Repeat ultrasound after 4 days

557. a patient suddenly developed acute pain behind the sternum radiating to the left arm. Objectively:
agitated, pale skin, PDR-38 / min., AT-180/110 mm Hg. in the future, the patient
loses consciousness, falls. The pulse on the main vessels is not detected, the pupils are evenly
dilated. What is the most likely diagnosis?
100% Clinical death
0% Agonal state
0% Coma
0% Heart attack
0% Impaired cerebral circulation

558. A 62-year-old female patient was hospitalized with complaints of enlarged cervical, supraclavicular, and
axillary lymph nodes, general weakness, increased sweating, and low-grade fever

129

https://translate.yandex.com/en/doc 124/540
22:20 ,27.6.2023 �� C : ; 5 B

Downloaded from the site - online testing step

within the last 3 months. In the blood: lake.- 64 • 109/l, in the Formula lymphocytes -72%. What
research method should be used to clarify the diagnosis?
100% Myelogram
0% Lymphography
0% Lymphoscintigraphy
0% X-ray examination
0% Thermography

559. the child is 2 years old. Complains of hoarseness of voice, shortness of breath with difficulty breathing. Gets sick
for 3 days. The disease started with a dry cough and nasal congestion. Objectively:
the general condition is disturbed, stridorotic respiration is noted. The skin is pale. Body temperature
37.3 ° C. Palatine arches are hyperemic, no plaque. The heart sounds are rhythmic. In the lungs
, hard breathing, no wheezing. A parainfluenza virus was detected in the nasopharyngeal flush. What
is the most likely diagnosis?
100% Acute stenosing laryngotracheitis
0% Epiglottitis
0% Foreign body
0% Diphtheria
0% Laryngospasm

560. A comprehensive school teacher was diagnosed with tuberculosis


of the lungs after an examination. What is the maximum period for which a person can be issued a disability certificate?
100% For 10 months
0% 5 months
0% 4 months
0% 2 months
0% 1 month

561. A patient with schizophrenia considers himself"an outstanding scientist, a brilliant composer and
an unsurpassed artist." He complains that " his family and friends are constantly jealous of him and
want to poison him." Identify the psychopathological syndrome:
100% Paranoid
0% Paranoid
0% Manic
0% Paraphrenic
0% Hebephrenic

562. A 42-year-old patient complains of lower back pain, darkening of urine, general weakness,
dizziness that occurred after treating a cold with aspirin and ampicillin. Objectively:
pale, subicteric sclera. Heart rate-98 / min. Liver + 2 cm, spleen + 3 cm. In the blood: Er. -2.6 •
1012/l, Hb-60 g/l, CP-0.9, leuc. -9.4 • 109/l, b. - 0.5%, E.-3%, P.-6%, S.-58%, L.-25%, M.-7%,
SSE - 38 mm / year, reticulocyte - 24%. Total bilirubin - 38 mmol / l
. what complication did the patient have?
100% Acquired hemolytic anemia
0% Toxic hepatitis

130

Downloaded from the site - online testing step

https://translate.yandex.com/en/doc 125/540
22:20 ,27.6.2023 �� C : ; 5 B

0% Cholelithiasis
0% Agranulocytosis
0% Paroxysmal nocturnal hemoglobulinuria

563. a 52-year-old patient suffering from disseminated


pulmonary tuberculosis was admitted to the hospital with complaints of sharp pain in the right side of the chest, which appeared
during the ascent to the 3rd floor, cough, shortness of breath at rest, and pronounced cyanosis. What
is the first complication to consider?
100% Spontaneous pneumothorax
0% Heart failure
0% Pulmonary insufficiency
0% Pleurisy
0% Acute myocardial infarction

564. A 44-year-old patient was admitted with complaints of dull, aching pain in the left lumbar region,
pus impurities in the urine. During the examination, a coralloid stone of II c was diagnosed on the left. what
treatment method is indicated?
100% Surgical treatment
0% Contact lithotripsy
0% Remote lithotripsy
0% Conservative therapy
0% Ascending litholysis

565. On the 21st day after the appearance of vesicular rash in chickenpox, a 7-year-old child
developed ataxia, nystagmus, intention tremor, and muscle hypotension. In the cerebrospinal fluid, low
lymphocytic pleocytosis, slightly increased protein level. What complication did
this child develop?
100% Encephalitis
0% Purulent meningitis
0% Pneumonitis
0% Acute neuritis
0% Postherpetic neuralgia

566. a 38-year-old man has been drinking alcohol for 3 years. 3 days after the binge, I began
to experience anxiety and fear. I saw spiders and worms all around me, and I was harassed
by"voices"of a judgmental nature. Behaved aggressively. In his own person, he is correctly oriented, in
place and time - disoriented. What is the most likely diagnosis?
100% Alcoholic delirium
0% Alcoholic paranoid
0% Alcoholic hallucinosis
0% Alcoholic encephalopathy
0% Pathological intoxication

567. during transfusion of erythrocyte mass in an 8-year-old boy with hemophilia A,

131

Downloaded from the site - online testing step

suddenly there was pain behind the sternum and in the lower back, shortness of breath, cold sweat. Objectively:

https://translate.yandex.com/en/doc 126/540
22:20 ,27.6.2023 �� C : ; 5 B
pale skin, heart rate-100 / min, AT-60/40 mm Hg. oliguria, brown urine. In the treatment of this
complication,
100% the first priority is the appointment of:
Prednisone
0% Lasixu
0% Adrenaline rush
0% Euphyllina
0% Analgin

568. A 16-year-old patient who suffered from severe bleeding from small cuts
and wounds had a question about the need for tooth root extraction. Examination revealed
an increase in the volume of the right knee joint, limited mobility. There are no other changes. In
the blood test, there is a tendency to anemia (Hb-120 g / l). What should be done to prevent
bleeding before the dentist's intervention?
100% Cryoprecipitate
0% Epsilon-Hydrocaproic acid
0% Fibrinogen
0% Dry blood plasma
0% Infusion of calcium chloride

569. A 3-year-old child has been diagnosed with type I diabetes mellitus and hyperosmolar coma. The diagnosis
was confirmed in the laboratory. What laboratory data are typical for this condition?
100% High hyperglycemia in the absence of ketonemia
0% Hyperglycemia + ketonemia
0% Hyperglycemia + glucosuria
0% Hyperglycemia + ketonuria
0% Hyperglycemia + elevated CBS scores

570. A 58-year-old female patient went to a antenatal clinic with complaints of bloody
discharge from the genital tract of the "meat slop" type. Postmenopausal 12 years. During
gynecological examination: external genitalia and vagina with signs of age-related involution;
the cervix is not changed, there are continuous spotting from the cervical canal; the uterus is of normal
size; appendages are not determined; parameters are free. What is the most likely diagnosis?
100% Cancer of the uterine body
0% Atrophic colpitis
0% Menstrual disorders of a climacteric nature
0% Cervical cancer
0% Granulocellular tumor of the ovary

571. A 3-year-old child suddenly developed a paroxysmal cough and


shortness of breath while playing Maniju. Objectively: dry cough, mixed dyspnea. A small
amount of dry wheezing is auscultated in the lungs. The breathing matter is weakened. The children's group
does not attend. Vaccinated according to age. What pathological condition can be suspected?
100% Foreign body of the respiratory tract
0% Pneumonia

132

Downloaded from the site - online testing step

0% Acute respiratory viral infection


0% Whooping cough
0% Bronchial asthma

https://translate.yandex.com/en/doc 127/540
22:20 ,27.6.2023 �� C : ; 5 B

572. A 44-year-old patient complained of difficulty urinating and a feeling of incomplete


emptying of the bladder. During the sonographic examination of the bladder at the entrance
to the urethra,an oval hyperechoic formation with equal contours of 2x3 cm in size is determined
, which is moved during the study. What conclusion can be drawn?
100% Concretion
0% Malignant tumor of the bladder
0% Polyp of the bladder
0% Prostate adenoma
0% Tumor of the initial part of the ureter

573. A 10-year-old child is being treated for dilated cardiomyopathy.


Shortness of breath, cardialgia are noted. On the lower extremities and sacrum, dense, sedentary edema. Ps-120 / min.
The borders of the heart are expanded in diameter. The heart sounds are muffled, there is a systolic murmur at
the apex and above the xiphoid process. Liver + 3 cm. Reduced diuresis. In the blood, total
protein is 58.6 g / l. in the urine: protein-0.025 g/l, leuc. - 2-4 v / s, ep. - 2-3 v / s. specify the main
mechanism of development of edematous syndrome:
100% Venous congestion in the large circulatory system
0% Venous congestion in the small circulatory system
0% Peripheral circulatory disorders
0% Development of secondary nephropathy
0% Consequence of hypoproteinemia

574. a 47-year-old patient complains of insomnia, heaviness in the whole body, and a constantly depressed
mood. He considers himself insignificant and untenable. He wonders what is a burden to his family, prefers
to die. Depressed, sedentary, face hypomimic, mournful. The language is quiet, monotonous,
and the answers are short. What is the most likely diagnosis?
100% Major depressive Disorder
0% Atherosclerotic depression
0% The initial stage of Alzheimer's disease
0% Late-onset schizophrenia
0% Neurotic depression

575. A bacterial study of residential premises in winter using


the Krotov apparatus showed that the total number of microorganisms in 1 m3 of air is 7200. What
is the permissible amount of micro-organisms that characterizes the air quality as "clean"?
100% Up to 4500
0% Up to 2500
0% Up to 3500
0% Up to 5500
0% Up to 7,500
576. A 48-year-old patient injured a varicose subcutaneous vein while working on a construction site.

133

Downloaded from the site - online testing step

a vein in the lower leg, accompanied by intense venous bleeding. Specify


the best first aid option:
100% Compression bandage and elastic bandaging of the limb
0% Applying the Esmarch tourniquet above the injury site
0% Applying the Esmarch tourniquet below the injury site
0% Compression of the femoral artery in a typical location
https://translate.yandex.com/en/doc 128/540
22:20 ,27.6.2023 �� C : ; 5 B

0% Maximum flexion of the limb at the knee joint

577. an ambulance delivered a patient who had eaten mushrooms in the morning to the infectious diseases hospital at night.
The disease started suddenly-a sharp pain in the abdomen, frequent diarrhea, waking vomiting,
unbearable thirst, headache, dizziness. On the third day, the disease ended in death.
Poisoning with what mushrooms took place?
100% Pale toadstool
0% Morels
0% Fly Agarics
0% Fake honey mushrooms
0% Syroezhki

578. A 28-year-old patient complains of periodic compressive pain in the heart. My brother died at
the age of 34 from a heart condition with similar symptoms. Objectively: the skin is pale. The boundaries of the heart
are not significantly shifted. The heart sounds are sonorous, and a systolic murmur is heard above all points, with
a maximum above the aorta. On Echocardiography-thickening of the interventricular septum in the basal
sections, a decrease in the left ventricular cavity. A means of preventing the progression
of the disease is:
100% Metoprolol
0% Digoxin
0% Nitroglycerin
0% Captopril
0% Furosemide

579. A 10-year-old boy was treated in the cardiology department for a diagnosis of rheumatism,
attack I, active phase, activity II B. he was discharged in a satisfactory condition. What drug is most
appropriate to prescribe for the prevention of secondary rheumatism?
100% Bicillin-5
0% Bicillin-1
0% Erythromycin
0% Ampicillin
0% Oxacillin

580. The following data were obtained for a 13 - year-old boy: Body length-147 cm (+2), body weight-38 kg
(+1.5), chest circumference-72 cm (+0.2). Evaluate the harmony
of the child's physical development:
100% Disharmonious
0% Harmonious
0% Above average

134

Downloaded from the site - online testing step

0% Sharply disharmonious
0% Excessive

581. canned food was delivered to the children's recreation camp. During an external inspection of the cans, it was noted
that the cans have deep dents, bend inward when pressed and do not immediately
return to their original position, there is no rust, the cans are smeared with technical grease.
Determine the bombardment:

https://translate.yandex.com/en/doc 129/540
22:20 ,27.6.2023 �� C : ; 5 B
100% Physical address
0% Chemical
0% Biological
0% Mixed
0% Physical and chemical analysis

582. A 29-year-old patient noticed facial edema,


weakness, and decreased performance 2 weeks after suffering a sore throat. Gradually,shortness of breath,swelling of the lower
extremities,
lumbar spine appeared. Objectively: pallor of the skin, weakening of heart tones,
anasarca. AT - 160/100 mm Hg in the urine: relative density-1021, protein-5 g / l, red blood cells-20-30
v / s,100%
hyaline cylinders-4-6 v / s. what is the most likely diagnosis?
Acute glomerulonephritis
0% Hypertension
0% Acute pyelonephritis
0% Infectious and allergic myocarditis
0% Myxedema

583. a 56-year-old patient, a researcher, has a compressive pain behind the sternum
several times a day while walking at a distance of 100-150 m, lasting up to 10 minutes. The pain is eliminated
by nitroglycerin. Objectively: increased nutrition, heart borders are normal, rhythmic tones,
Ps78 / min, AT-130/80 mm Hg on the ECG: reduced amplitude of the T wave in V4-5. what disease
can I think about?
100% Stable angina pectoris of FC III
0% Unstable angina pectoris
0% Stable angina pectoris of FC I
0% Stable angina II FC
0% Stable angina pectoris IV FC

584. A 25-year-old patient developed epigastric pain in autumn, which occurs 1.5-2 hours
after eating and at night. Complains of heartburn, constipation. The pain increases when taking
spicy, salty and acidic food, decreases-after applying soda and a hot water bottle. He has been ill
for a year. Objectively: the tongue is not covered, moist. On palpation of the abdomen-pain in
the epigastrium on the right, in the same area - resistance of the abdominal muscles. What
is the most likely medical condition?
100% Duodenal ulcer disease 12
0% Chronic cholecystitis
0% Phrenic hernia
0% Peptic ulcer disease of the stomach

135

Downloaded from the site - online testing step

0% Chronic pancreatitis

585. in a full-term child of 3 days of life,erythema,


erosive surfaces, cracks, and peeling of the epidermis are found on various skin areas. The baby looks like it's been scalded with boiling
water.
A positive symptom of Nikolsky was detected. The general condition of the child is severe: severe
anxiety,
100% hyperesthesia, febrile
Exfoliative fever. What is the most likely diagnosis?
dermatitis
0% Phlegmon of the newborn
0% Figner's pseudofurunculosis

https://translate.yandex.com/en/doc 130/540
22:20 ,27.6.2023 �� C : ; 5 B
0% Newborn's pemphigus
0% Mycotic erythema

586. A 7-year-old boy has been undergoing medical treatment for a month. At admission
, severe edema was observed, proteinuria-7.1 g / l, Protein in the daily urine-4.2 g.in
the biochemical analysis of blood, hypoproteinemia (43.2 g/l), hypercholesterolemia (9.2
mmol/l) were retained. Which of the above variants of glomerulonephritis is most likely to occur in the patient?
100% Nephrotic
0% Nephritic
0% Isolated urinary tract
0% Hematuric
0% Mixed

587. a repeat mother with a history of regular labor has had a cesarean section 3
years ago due to acute intrauterine fetal hypoxia. During labor, I felt
a lot of pain in the area of the postoperative scar. Fetal heartbeat is rhythmic, 140 / min. During
vaginal examination, the opening of the cervix is 5 cm. The prolific bubble is intact. What are the doctor's tactics?
100% Caesarean section
0% Stimulation of labor activity
0% Obstetric forceps
0% Waiting delivery tactics
0% Birth through the natural genital tract

588. A 54-year-old woman complained of bleeding from the genital tract after 2 years
of amenorrhea. During ultrasound and bimanual examination, no pathology from the genitals was
detected. What are the doctor's tactics?
100% Fractional curettage of the cervix and uterine cavity
0% Hemostatic drugs
0% Reducing drugs
0% Estrogenic hemostasis
0% Hysterectomy

589. A 32-year-old man has been bothered by pain in the sacrum and hip
joints, soreness and stiffness when moving in the lumbar spine for the past year. ESR is 56 mm / hour.
Radiologically revealed signs of bilateral sacroiliitis. The patient is a carrier of the HLA B27 antigen.

136

Downloaded from the site - online testing step

What is the most likely diagnosis?


100% Ankylosing spondylitis
0% Coxarthrosis
0% Rheumatoid arthritis
0% Reiter's disease
0% Spondylosis

590. A 58-year-old female patient complains of periodic headache, dizziness, and tinnitus.
He has been suffering from diabetes for 15 years. Objectively: heart sounds are rhythmic, heart rate-76 / min,
emphasis on the tone above the aorta, AT-180/110 mm Hg in the urine: relative density -1.014. daily
protein loss in the urine is up to 1.5 g. The drug of choice for the treatment of arterial hypertension in this
patient will be:
https://translate.yandex.com/en/doc 131/540
22:20 ,27.6.2023 �� C : ; 5 B
100% ACE Inhibitor
0% beta-blocker
0% Calcium channel antagonist
0% Thiazide diuretic
0% alpha-blocker

591. A 62-year-old patient was taken to the surgical department with complaints of sudden
pain in the umbilical region, which was accompanied by fainting, radiating to the back
and groin area. Objectively: the condition is severe, pain during palpation in the umbilical region,
intestinal murmurs are weakened. There is a decrease in blood pressure. On ultrasound: free fluid
in the abdominal cavity, thickening of the abdominal aortic wall. What is the most likely diagnosis?
100% Ruptured abdominal aortic aneurysm
0% Gastric ulcer perforation
0% Acute pancreatitis
0% Peritonitis
0% Acute appendicitis

592. A 54-year-old female patient has been suffering from femoral osteomyelitis for more than 20 years. Over the past month
, edema of the lower extremities has appeared and gradually increased. In the urine, proteinuria is 6.6 g / l, in the blood
, diss-proteinemia in the form of hypoalbuminemia, an increase in α2-and γ-globulins, SSE-50 mm / h .
What is the most likely diagnosis?
100% Secondary amyloidosis of the kidneys
0% Acute glomerulonephritis
0% Myeloma disease
0% Chronic glomerulonephritis
0% System red shepherd dog

593. An ambulance delivered an 83-year-old female patient complaining of impaired


support on her right leg after falling on her right side. Objectively: the patient is lying on
a gurney, the right lower limb is rotated outward, the outside of the foot touches
the bed. Positive symptom of "stuck heel". What is the preliminary diagnosis?
100% Femoral neck fracture
0% Hip diaphysis fracture

137

Downloaded from the site - online testing step

0% Hip dislocation
0% Hip contusion
0% Acetabular fracture

594. A 29-year-old patient developed a throbbing pain in the


rectum, fever, and general weakness 3 days ago. Objectively: local soreness in
the anal area for 6 hours. In the digital examination of the rectum, a painful infiltrate
reaches the comb line. What is the most likely diagnosis?
100% Acute paraproctitis
0% Acute anal fissure
0% Acute hemorrhoids
0% Rectal tumor
0% Acute prostatitis

https://translate.yandex.com/en/doc 132/540
22:20 ,27.6.2023 �� C : ; 5 B

595. A 62-year-old man was hospitalized in the intensive care unit with a long-term attack
of chest pain, which does not disappear when taking nitroglycerin. Objectively: AT - 80/60 mm
Hg, heart rate-106/min, PDR-22 / min. Heart tones are muffled, the rhythm of a gallop. How can I explain a decrease
in blood pressure?
100% Reduced cardiac output
0% Reduced peripheral resistance
0% Depositing blood in the abdominal cavity
0% Adrenergic receptor blockade
0% Internal bleeding

596. a patient is on sick leave for 4 months continuously due to an injury. The treatment
will still last 1-2 months. Who has the right to extend the disability certificate for this patient?
100% Medical Advisory Commission after the ICEC decision
0% Chief Physician of the medical institution
0% Medical advisory board after treatment of a patient in a hospital
0% District doctor after consultation with the head of the department
0% Medical and Social Expert Commission

597. a 9-month-old child has a body temperature of 36.7 ° C, pale, moist skin, and severe pain in
the leg muscles. There are no movements in the lower extremities, sensitivity is preserved. Polio was diagnosed.
What genus does the causative agent of this disease belong to?
100% Picornaviruses
0% Paramyxoviruses
0% Togoviruses
0% Adenoviruses
0% Rotaviruses

598. the working conditions of a construction worker are characterized by a cooling


microclimate, dust containing silicon dioxide, caustic alkali (quicklime), and noise. Which
specialist doctor should be in charge of the commission that conducts periodic medical examinations

138

Downloaded from the site - online testing step

workers of the specified category?


100% Therapist
0% Ophthalmologist
0% Neurologist
0% Dermatologist
0% Otolaryngologist

599. a full-term child from the first uncomplicated pregnancy, burdened by childbirth, had a
cephalohematoma. On the 2nd day jaundice appeared, on the 3rd - changes in the neurological condition:
nystagmus, Grefe's syndrome. Urine is yellow, feces are golden yellow. Mother's blood type A (II)
Rh_, child - A (II) Rh+. On day 3, the child had Hb-200 g / l, ep. -6.1•1012/l, blood bilirubin-58
mmol / l due to the unbound fraction, Ht-0.57 what caused jaundice in the child?
100% Craniocerebral birth trauma
0% Physiological jaundice
0% Hemolytic disease of newborns
0% Biliary tract atresia
https://translate.yandex.com/en/doc 133/540
22:20 ,27.6.2023 �� C : ; 5 B
0% Fetal hepatitis

600. After birth, the child is pale, has arrhythmic breathing, which does not improve against the background
of oxygen therapy. Her pulse is weak and fast, and her blood pressure is difficult to measure. There is no swelling.
What is the most likely cause of these symptoms?
100% Asphyxia
0% Congestive heart failure
0% Intracranial hemorrhage
0% Intrauterine sepsis
0% Intrauterine pneumonia

601. of the 350 workers of the metallurgical plant who were subject to preventive examinations in
the current year, 325 people were examined in the territorial polyclinic. As a result, one
employee was temporarily suspended from work, 15 underwent further rehabilitation in
sanatoriums, and 10 were provided with dietary food. What indicator that characterizes the
preventive work of a polyclinic is appropriate to use in this case?
100% Completeness of coverage of periodic medical examinations
0% Frequency of detection of diseases during examinations
0% Specific weight of people who are rehabilitated in the dispensary
0% Specific weight of persons who are provided with dietary nutrition
0% Proportion of people temporarily suspended from work

602. A 14-year-old girl developed irritability and tearfulness about a year ago. At the same
time, a diffusely enlarged thyroid gland of the second degree was determined. The condition was regarded as a manifestation
of puberty, and no treatment was performed. Irritability gradually gave way to complete
apathy. Puffiness of the face, pasty soft tissues, bradycardia, constipation appeared.
Overgrown pallor of the skin, it has acquired a waxy hue. The gland has become more dense. What
disease should be assumed?
100% Autoimmune thyroiditis

139

Downloaded from the site - online testing step

0% Diffuse toxic goiter


0% Thyroid cancer
0% Subacute thyroiditis
0% Puberty juvenile basophilism

603. a patient with eating disorders, steatorrhea, has abdominal pain 4 hours after eating
, especially above the navel and to the left. Diarrhea can be replaced by constipation for up to 3-5 days.
Palpation: moderate pain in the choledochopancreatic zone. The level of amylase in the blood does not
increase. Radiologically, calcifications located above the navel were detected. What
is the most likely diagnosis?
100% Chronic pancreatitis
0% Chronic gastroduodenitis
0% Duodenal ulcer 12
0% Zollinger-Ellison syndrome
0% Chronic calculous cholecystitis

604. a 56-year-old patient with diffuse toxic goiter has atrial fibrillation with

https://translate.yandex.com/en/doc 134/540
22:20 ,27.6.2023 �� C : ; 5 B
an atrial rate of 110/min, arterial hypertension. AT-165/90 mmHg. what treatment along
with mercazolil should be recommended in this case?
100% Anaprilin
0% Radioactive iodine
0% Novocainamide
0% Verapamil
0% Corinfar

605. on the 14th day after delivery, a woman in labor went to the doctor complaining of
sudden pain, hyperemia and tightness in the left breast, fever up
to 39 ° C, headache, malaise. Objectively: a crack in the nipple area, an increase in the volume
of the left breast, increased pain during palpation. What pathology can be considered in
this case?
100% Lactation mastitis
0% Cyst of the left breast with suppuration
0% Fibroadenoma of the left breast
0% Breast cancer
0% Phlegmon of the breast

606. on day 5, a 24-year-old man with a high fever


developed a severe headache, systemic dizziness, double vision, paresis of the facial
muscles on the right side, and choking when swallowing. Antiviral encephalitis was diagnosed.
Determine the main direction of emergency therapy:
100% Zovirax
0% Glucocorticoids
0% Ceftriaxone
0% Lasix

140

Downloaded from the site - online testing step

0% Hemodesis

607. a 54-year-old man complains of aching pain in the lumbar region, which increases
after being in an upright position, physical exertion, hypothermia;
weakness in the afternoon. Pain in the lumbar region is noted for about 10 years.
Objectively: pallor of the skin, to-37.2 oC, AT-180/100 mm Hg. weakly positive
Pasternatsky symptom. In the blood: Er.- 3,5 • 1012/ l, lake.-6,5- 109/ l, WSE-22 mm / hour. In the urine:
relative density -1010, leu.- 12-15 v p/s, ep.- 2-3 V p/s. the microbial number of urine is 100000 in 1 ml.
What is the most likely diagnosis?
100% Chronic pyelonephritis
0% Kidney stone disease
0% Polycystic kidney disease
0% Chronic glomerulonephritis
0% Amyloidosis

608. A 49-year-old man complains of mixed dyspnea, cough, swelling


of the lower legs, abdominal enlargement due to ascites. He has been ill with chronic bronchitis for more than 20 years.
For the last 3 years, he has been a disabled person of group II due to changes in the heart. Objectively: cyanosis of a mixed
nature, edema. Ps-92 / min, rhythmic, blood pressure-120/70 mm Hg, PDR-24/min. Accent of tone II over

https://translate.yandex.com/en/doc 135/540
22:20 ,27.6.2023 �� C : ; 5 B
the pulmonary artery. Above the lungs - a box sound. Dry wheezing over the entire surface of the lungs.
100% The Euler-Liljestrand reflex
What is the mechanism of development of changes in the heart in this patient?
0% Kitaev's Reflex
0% The Bainbridge Reflex
0% Cardiovascular reflex
0% Respiratory reflex

609. A 43-year-old woman complains of unstable bowel movements with a predominance of constipation,
bloating and spastic pain in the lower abdomen, as well as headache
and sleep disorders. Body weight is not changed. What disease is most likely to determine such a clinical
picture?
100% Irritable bowel syndrome
0% Chronic enteritis
0% Chronic pancreatitis
0% Chronic atrophic gastritis
0% Bowel cancer

610. A 43-year-old man who had contact with leaded gasoline was admitted to the clinic
complaining of general weakness, dizziness, memory impairment, daytime drowsiness and
insomnia at night, a feeling of "hair" in his mouth, colic-like pain in the right hypochondrium. what
is the most likely diagnosis?
100% Chronic tetraethyl lead intoxication
0% Alcoholic delirium
0% Chronic mercury intoxication
0% Khroshchna shtoksikashcha manganese

141

Downloaded from the site - online testing step

0% Hroshchna shtoksikashcha lead

611. A 35-year-old patient is on the fourth day in the intensive care unit due to
acute renal failure due to prolonged compression syndrome. Objectively:
inadequate. PDR-32 / min, single wet wheezes are detected in the lungs for the last 3 hours. On
the ECG: high T waves, right ventricular extrasystoles. CVT-159 mm of water in the blood: residual
nitrogen-62 mmol / l, K + - 7.1 mmol/ L, Cl-78 mmol/L, Na+ - 120 mmol/L, Ht-0.32, Hb - 100 g/l,
blood creatinine - 0.9 mmol/l. the most appropriate treatment method would be:
100% Hemodialysis
0% Plasmosorption
0% Hemosorption
0% Plasma filtration
0% Ultrafiltration

612. A 47-year-old woman underwent thyroid resection for a nodular


euthyroid goiter. What treatment is most likely to help prevent
a relapse of the disease?
100% Thyroid Hormones
0% Mercazolil
0% Thyrotropin
0% Antistrumin
0% Radioactive iodine
https://translate.yandex.com/en/doc 136/540
22:20 ,27.6.2023 �� C : ; 5 B

613. A 30-year-old male fire victim has thermal burns of grade III-A and III-B
that account for 20% of the total skin. AT - 110/70 mm Hg, heart rate-120/min. Which
transfusion agents are most appropriate to use for blind infusion correction during
transportation?
100% Saline solutions
0% Polyglukin
0% 10% glucose solution
0% Fresh frozen plasma
0% Albumin

614. a patient has been suffering from rheumatoid arthritis for more than a year, and the course of the disease
progresses rapidly. X-ray examination confirmed the presence of marginal usurs. Treatment with what
basic drug is most appropriate?
100% Methotrexate
0% Delagil
0% Prednisone
0% Diclofenac-sodium
0% Aspirin

615. a 3-year-old child who is taken to the hospital has a soporotic condition, a sharp
decrease in muscle tone, suppression of tendon and periosteal reflexes, and miosis is detected,

142

Downloaded from the site - online testing step

reduced pupil response to light. Corneal reflexes are preserved. Pulse is fast
and weak. AT-80/50 mmHg. parents suspect that the child swallowed pills.
Poisoning with what tablet medications does this clinical picture correspond to?
100% Tranquilizers
0% Atropine-like drugs
0% Antihypertensive drugs
0% Barbiturates
0% a2-adrenomimetics

616. A 23-year-old patient complains of dull pain, a feeling of heaviness and fullness in the epigastrium
immediately after eating, rotten belching, dry mouth, nausea on an empty stomach, diarrhea. Objectively:
slim build, pale skin. The abdomen during palpation is soft, there is pain in the epigastrium.
The liver does not protrude from under the edge of the costal arch. In the blood: Hb-110 g / l, Er.- 3,4 • 1012/ l,
the leukocyte formula is unchanged. SSE-16 mm / hour. What is the most informative study
to help establish a diagnosis?
100% Esophagogastroduodenoscopy
0% X-ray of the digestive system
0% Gastric juice testing
0% pH-metric
0% Duodenal probing

617. a 49-year-old patient complains of impaired swallowing, especially solid food, hiccups,
hoarseness of voice, nausea, regurgitation, significant weight loss (15 kg in 2.5 months). Objectively:
the body weight is reduced. The skin is pale and dry. In the lungs-vesicular respiration, heart tones

https://translate.yandex.com/en/doc 137/540
22:20 ,27.6.2023 �� C : ; 5 B
of sufficient sonority, rhythmic activity. The abdomen is soft and painless on palpation. The liver
is not enlarged. What research is most necessary to make a diagnosis?
100% Esophagoduodenoscopy with biopsy
0% Clinical blood test
0% X-ray of the digestive tract
0% Trendelenburg radiography
0% Study of gastric secretion

618. A 60-year-old patient was admitted to the clinic complaining of shortness of breath, heaviness in the right
hypochondrium, and abdominal enlargement. The phenomena grew during the year. During auscultation of the heart, a
presystolic gallop rhythm is observed. Objectively: swelling of the cervical veins, ascites, palpable liver
and spleen. What disease needs to be differentially diagnosed?
100% Constrictive pericarditis
0% Cirrhosis of the liver
0% Lung cancer with pleural growth
0% Chronic pulmonary heart disease
0% Pulmonary embolism

619. A 40-year-old patient, a forest ranger, complains of severe headache, body temperature up
to 39.5 ° C, trembling of the limbs. From the medical history, it is known that he recently severely cut his hand during
the autopsy of a dead fox. Objectively: the mood is depressed. Asks not to turn on the light, not to

143

Downloaded from the site - online testing step

open the door. It responds to knocking with a sharp motor excitement. When I saw the carafe of water,
I developed a convulsive spasm in my throat. What are the emergency doctor's tactics?
100% Admission to an infectious diseases hospital
0% Hospitalization in the intensive care unit
0% Hospitalization in the neurological department
0% Admission to a psychiatric hospital
0% Leave it at home and then consult a psychiatrist

620. A 5-year-old child became acutely ill with fever up to 38.5 oC, runny nose, cough, and
conjunctivitis. On the 4th day of the disease, a spotty-papular rash appeared on the face, a new
temperature rise to 39.2 oC. In the following days, the rash spread to the torso and limbs.
The palate mucosa is hyperemovated, and there are remnants of small whitish layers on the cheek mucosa near the lower angular teeth
. What is the most likely preliminary diagnosis?
100% Measles
0% Acute respiratory viral infections
0% Yersiniosis
0% Enterovirus infection
0% Rubella

621. A 28-year-old woman has been suffering from chronic glomerulonephritis for 12 years, which had
a latent course. Over the past six months, there has been general weakness, decreased appetite,
working capacity, and nausea. Complains of headache, joint pain. On examination: anemia,
blood urea - 34.5 mmol / l, blood creatinine-0.766 mmol/l, hyperkalemia. What has complicated
the course of the disease?
100% Chronic renal failure
0% Acute renal failure

https://translate.yandex.com/en/doc 138/540
22:20 ,27.6.2023 �� C : ; 5 B
0% Nephrotic syndrome
0% Amyloidosis of the kidneys
0% Pyelonephritis

622. the child is 1 year old. After the introduction of complementary foods in recent months, there is a loss
of appetite, diarrhea with the release of large amounts of feces, sometimes vomiting, body temperature
is normal. Objectively: body weight 7 kg, very pale, swollen legs, very enlarged abdomen
. The coprogram contains a lot of fatty acids and soaps. Celiac disease was diagnosed
and a gluten-free diet was prescribed. What is excluded from the diet according to this diet?
100% Cereals-wheat, oats
0% Milk and dairy products
0% Fruit
0% Animal protein
0% Easily digestible carbohydrates

623. the development of chronic venous insufficiency of the lower extremities depends on the functional
state of the so-called "muscular-venous pump". What muscle group does this term apply
to?
100% Lower legs

144

Downloaded from the site - online testing step

0% Abdominal wall
0% Gluteal region
0% Loins
0% Feet

624. after treating the field with toxic chemicals, the machine operator developed severe weakness,
headache, nausea, vomiting, diarrhea, visual impairment, and lacrimation. Objectively: the patient
is agitated, hypersalivation, hyperhidrosis, fibrillation of the muscles of the tongue, eyelids are noted. The pupils
are narrowed, tachycardia, and the lungs are filled with moist small-and medium-cirrhotic wheezes. In
the blood - a change in the level of cholinesterase activity. What is the most likely diagnosis?
100% Intoxication with organophosphate pesticides
0% Intoxication with organochlorine pesticides
0% Intoxication with organohercury pesticides
0% Intoxication with arsenic-containing pesticides
0% Intoxication with carbamic acid derivatives

625. A 7-year-old child applied to the polyclinic for a follow-up examination and recommendations
. Suffers from bronchial asthma for 4 years, suffocation attacks occur mainly in
the spring and summer period. According to the results of allergy tests: increased sensitivity to poplar fluff,
field grasses. What is the most likely doctor's recommendation?
100% Performing specific hyposensitization
0% Performing physical therapy
0% Conducting spa treatment
0% Conducting herbal medicine
0% Performing acupuncture

626. A 35-year-old patient was admitted to the hospital with complaints of pain in the left sternococcal and
knee joints, and in the lower back. He became acutely ill, with a fever of up to 38oC.
https://translate.yandex.com/en/doc 139/540
22:20 ,27.6.2023 �� C : ; 5 B
Objectively: the left sternoclavicular and knee joints are swollen and painful. In the blood: lake.- 9,5 •
109/ l, SSE - 40 mm/h, CRP - 1.5 mmol/l, fibrinogen - 4.8 g/l, uric acid - 0.28 mmol/l. in
scraping from the urethra-chlamydia. What is the most likely diagnosis?
100% Reiter's Syndrome
0% Rheumatic arthritis
0% Gout
0% Ankylosing spondylitis
0% Rheumatoid arthritis

627. topographical percussion of the lungs in a patient who had received a significant
barotrauma at the enterprise revealed that the lower borders of the lungs were one rib lower than normal,
the height of the apices of both lungs and the Krenig field were significantly increased. What disease should
a doctor think about first?
100% Emphysema of the lungs
0% Exudative pleurisy
0% Chronic bronchitis
0% Bronchial asthma

145

Downloaded from the site - online testing step

0% Pneumothorax

628. A 32-year-old patient lives in an area endemic for echinococcosis. For the last 6 months, he
has been worried about pain in the right hypochondrium, fever. Suspected echinococcal
liver damage. What kind of research is most informative in this case?
100% Ultrasound examination
0% Overview X-ray examination of the abdominal cavity
0% Biochemical laboratory research
0% Angiography
0% Liver scan

629. A 30 - year-old pregnant woman who was 32 weeks pregnant went to a antenatal clinic complaining
of bright-colored vaginal spotting.
She was admitted to a hospital with suspected placenta previa. Under what conditions is it advisable to conduct an internal
study to establish a diagnosis?
100% In the operating room when it is ready for surgery
0% In the women's review clinic
0% In the emergency department of the maternity hospital
0% In the delivery room in compliance with all the rules of asepsis
0% Do not carry out due to the risk of profuse bleeding

630. A 35-year-old female patient developed areas of redness and swelling on the back of her hands after severe nervous tension
, followed by the formation of small
inflammatory nodules, blisters, and then erosions with significant serous fluid release. The process
is accompanied by severe itching. What is the most likely diagnosis?
100% Real eczema
0% Allergic dermatitis
0% Microbial eczema
0% Simple contact dermatitis
0% Toxicoderma
https://translate.yandex.com/en/doc 140/540
22:20 ,27.6.2023 �� C : ; 5 B

631. a 36-year-old patient complains of a skin rash that appeared a week ago and
does not subjectively bother him. Objectively: on the skin of the palms and soles - multiple lenticular
disseminated papules that do not rise above the skin level, stagnant red color,
covered with a cluster of horny scales, dense during palpation. What is the preliminary diagnosis?
100% Secondary syphilis
0% Multiple warts
0% Palmar-plantar psoriasis
0% Rubrophytosis of the palms and soles
0% Icing of the palms and soles

632. a 30-year-old patient complains of paroxysmal pain in the lower abdomen, frequent
liquid bowel movements up to 10 times a day. In the first 3 days there was a high fever, from the 2nd day
of the disease-liquid unexplained bowel movements with mucus admixtures. During palpation: soreness of all

146

Downloaded from the site - online testing step

segments of the large intestine. Sigma is spasmodic. What is the preliminary diagnosis?
100% Acute dysentery
0% Intestinal amoebiasis
0% Salmonellosis
0% Cholera
0% Balantidiasis

633. a 38-year-old woman has a paroxysmal increase in blood pressure up to 240/120 mm Hg,
with nausea, vomiting, tachycardia,increased sweating,
and hyperglycemia. After an attack, there is a profuse discharge of urine. During sonography of the kidneys
, an additional formation was found adjacent to the upper pole of the right kidney, possibly
related to the adrenal gland. What laboratory test can help clarify the diagnosis?
100% Determination of urinary excretion of catecholamines and vanillylmigdalic acid
0% Determination of insulin and C-peptide in the blood
0% Determination of glomerular filtration rate by endogenous creatinine
0% Determination of thyroxine and thyroid-stimulating hormone in the blood
0% Determination of blood renin level

634. A 32-year-old patient has been experiencing suffocation attacks for 3 years, which are almost not stopped
by berotec. In recent months, he has noticed joint pain and sensitivity disorders of the skin of the shins and
feet. Ps-80 / min, AT-210/100 mm Hg in the blood: eosinophilia 15%. What kind of disease
can you think about?
100% Nodular periarteritis
0% System red shepherd dog
0% Systemic scleroderma
0% Dermatomyositis
0% Wegener's disease

635. A 46-year-old patient complains of a sudden palpitation, which is accompanied


by pulsation in the neck and head, fear, and nausea. The heartbeat lasts 15-20 minutes,
passes when you hold your breath with straining. What kind of heart
failure can you think about?

https://translate.yandex.com/en/doc 141/540
22:20 ,27.6.2023 �� C : ; 5 B
100% Supraventricular paroxysmal tachycardia attack
0% An attack of ventricular paroxysmal tachycardia
0% Atrial flutter attack
0% Atrial fibrillation attack
0% An attack of extrasystolic arrhythmia

636. a patient complains of excruciating swelling in the chin area, general malaise,
and headache. In this area, an acute inflammatory cone-shaped dense node was found.
The skin above it is tense and red. In the center of the node there is an ulcer with steep edges and
a necrotic rod of dirty green color. Submandibular lymph nodes on the right
are enlarged and painful. What is the most likely diagnosis?
100% A boil
0% Tuberculosis

147

Downloaded from the site - online testing step

0% Carbuncle
0% Tertiary syphilis (resinous)
0% Deep trichophytosis

637. a welder at work received a first-degree burn to the middle third of his right shin.
After 5 days, skin swelling and itching appeared around the burn. Objectively: on the background of erythema
with clear borders, a polymorphic rash represented by papules, vesicles, pustules,
erosions with serous discharge. What is the most likely diagnosis?
100% Microbial eczema
0% True eczema
0% Toxicoderma
0% Professional eczema
0% Streptoderma

638. A 58-year-old patient has been suffering from type II diabetes mellitus for 3 years. I followed
a diet and regularly took Glibenclamide. Delivered urgently with an acute
abdominal clinic. Objectively: increased nutrition, dry skin. In the lungs-vesicular respiration.
Heart tones are rhythmic, 90 / min. AT-130/70 mm Hg belly "plank-shaped". Blood sugar-9.8
mmol / l. the patient is indicated for laparotomy. How is it advisable to continue treatment of diabetes mellitus?
100% Transfer the patient to simple insulin
0% Continue taking glibenclamide
0% Semelong-in the morning, and in the afternoon and evening-insulin
0% Ilurenorm on 1 t. 3 times a day
0% Maninil on 1 t. 3 times a day

639. a 43-year-old patient complains of a rash on the skin of the right leg, pain, weakness,
fever up to 38oC. She became acutely ill. Objectively: on the skin of the right leg in the foot
area, edema, sharply outlined bright red spot, hot to the touch,
irregular contour, in the form of flames. there are single bubbles in the cell. What
is the preliminary diagnosis?
100% Erysipelas
0% Microbial eczema
0% Contact dermatitis
0% Toxicoderma

https://translate.yandex.com/en/doc 142/540
22:20 ,27.6.2023 �� C : ; 5 B
0% Hemorrhagic vasculitis

640. a 45-year-old patient complains of the appearance of nodular painless elements on the skin of the lower back and limbs
, which are prone to peripheral growth and fusion. He has been ill for 2 years. Exacerbation
mainly in spring. In the anamnesis: the patient's father had a similar skin lesion. Objectively:
pathological elements are represented by teardrop-shaped and coin-like nodules,
plaques covered with white scales. What is the preliminary diagnosis?
100% Psoriasis
0% Lichen planus erythematosus
0% Neurodermatitis
0% Pink lichen

148

Downloaded from the site - online testing step

0% Seborrheic eczema

641. A 47-year-old patient visited a doctor on the 7th day of his illness. He became acutely ill: after a chill
, his body temperature rose to 40 ° C, lasted up to 7 hours, after which it dropped sharply
, followed by profuse sweat. Similar attacks were repeated three times every other day.
I arrived from Africa two weeks ago. Objectively: the skin is pale. The sclera is subicteric. Significantly enlarged liver and
spleen. What causes the development of a fever attack in this disease?
100% Erythrocytic schizogonia
0% Tissue schizogonia
0% Action of the exotoxin of the pathogen
0% Action of the pathogen endotoxin
0% Gametocytes

642. A 27-year-old patient complains of unbearable headache and repeated vomiting on the 2nd day of illness
. Objectively: the condition is serious. Conscious, adynamic. In bed, he takes
a forced position with his head thrown back. Skin without rash. Stiffness
of the occipital muscles, symptoms of Kernig, Brudzinsky are expressed. Temperature 39.5 oC, Ps-120/Xb, AT-130/80 mm
Hg. what is the cause of the leading syndrome of the disease?
100% CSF hypertension
0% CSF hypotension
0% Damage to the cranial nerve nuclei
0% Adrenal hemorrhage
0% Hyperthermia

643. a girl born from 2 urgent deliveries with a weight of 3500 g,


an Apgar score of 8 points, developed jaundice on the 1st day of life. Indirect bilirubin in the blood-80 mmol / l,
after 6 hours -160 mmol / l. what is the most optimal method of treatment?
100% Blood transfusion replacement surgery
0% Phototherapy
0% Infusion therapy
0% Purpose of phenobarbital
0% Enterosorbents

644. a 30-year-old patient was hospitalized with a diagnosis of intestinal obstruction. During the operation
, it was discovered that the obstruction of the small intestine is caused by a ball of worms. What type of helminth
caused the obstruction?

https://translate.yandex.com/en/doc 143/540
22:20 ,27.6.2023 �� C : ; 5 B
100% Roundworms
0% Guinea worm
0% Filarii
0% Cysticerci
0% Pinworms

645. on the 5th day after surgery, a patient complains of


bursting pain in the postoperative wound, weakness, drowsiness, headache, and fever due to damage to the colon.

149

Downloaded from the site - online testing step

temperatures up to 40oC. There is swelling around the wound, crepitation of gas. Discharge from the wound is scanty
dark gray with a putrid smell. What is the most likely diagnosis?
100% Anaerobic clostridial wound infection
0% Abscess
0% Postoperative wound suppuration
0% Erysipelas
0% Phlegmon

646. a child was born at the gestational age of 34 weeks in a serious condition. The leading
symptoms of respiratory disorders were: sonorous prolonged exhalation, the participation of additional muscles in breathing,
the presence of crepitating wheezes against the background of hard breathing. Silverman scale score at
birth-0 points, after 3 hours-6 points with clinical data. What
diagnostic method will allow you to determine the type of pneumopathy in a child?
100% X-ray examination of the chest organs
0% Clinical blood test
0% Determination of blood gas composition
0% Proteinogram
0% Immunological research

647. A 1-month-old boy was admitted to the children's surgical department


and prenatally diagnosed with left-sided pyeloectasia. Based on the analysis of data
from infusion-drip urography, cystography, ultrasound, the child was diagnosed with hydronephrosis of the initial
stage. There are no data on secondary pyelonephritis. What
is the most appropriate management strategy for this patient?
100% Follow-up for 6 months
0% Urgent nephrostomy
0% The Andersen-Hines Operation
0% There is no need for supervision and treatment
0% Antibacterial therapy

648. A 24-year-old man has chronic glomerulonephritis. In urine: relative density -1010,
protein -1.65 g / l, ep. - 5-7 v p/ s, leuc.- 2-3 v / s. blood creatinine-0.350 mmol/l. serum sodium
-148 mmol / l. what is the main cause of hyperazotemia in a patient?
100% Reduced glomerular filtration rate
0% Reduced tubular reabsorption
0% Enhanced proteinuria
0% Reduced renal blood flow
0% Sodium retention in the body

https://translate.yandex.com/en/doc 144/540
22:20 ,27.6.2023 �� C : ; 5 B

649. a patient complains of hip pain after an accident. The leg in the position of flexion,
adduction and internal rotation is significantly reduced. Elastic resistance when attempting passive
adduction or abduction of a limb. The big swivel is located high above
the Rosernelaton line. Significant lordosis is detected. What is the preliminary diagnosis?
100% Iliac hip dislocation
0% Dislocated femoral neck fracture

150

Downloaded from the site - online testing step

0% Acetabular fracture with central hip dislocation


0% Gluteal hip dislocation
0% Hip hypertrol fracture

650. A 20-year-old patient developed a low-grade


fever, pain and swelling in the knee and ankle joints, erythema on the face and
bridge of the nose, leukopenia, and accelerated shock after prolonged sun exposure. A preliminary diagnosis of systemic red
shepherd dog has been established. What pathognomonic laboratory data confirm this diagnosis?
100% Presence of an antinuclear factor
0% Accelerated SSE
0% Presence of C-reactive protein
0% Anemia
0% Lymphocytosis

651. when examining the corpse of a person who died from hanging, it was found: cadaveric spots
disappear under pressure, recover after 50 seconds, rigor mortis is moderately pronounced
only in the masticatory muscles, in the muscles of the neck and fingers of the cyst, body temperature 31.0 oC. How long
has it been since death?
100% 6-7 hours
0% 1-2 hours
0% 16-24 hours
0% 8-10 hours
0% 10-18 hours

652. after concussion of the right eye, the patient complains of sudden loss of vision before light perception.
Objectively: the eye is calm, the cornea is clear, the pupil's response to light is lively. The pupil area
is black. There is no reflex from the fundus. What is the most likely cause of vision loss?
100% Hemophthalmos
0% Retinal detachment
0% Traumatic cataract
0% Acute retinal vascular occlusion
0% Optic nerve detachment

653. A 68-year-old female patient went to the doctor complaining of a tumor in


her left breast. Objectively: in the upper inner quadrant of the left breast there
is a formation with a diameter of up to 2.5 cm, dense, lumpy, painless during palpation. Regional
lymph nodes are not enlarged. What is the likely diagnosis?
100% Cancer
0% Cyst
0% Fibroadenoma

https://translate.yandex.com/en/doc 145/540
22:20 ,27.6.2023 �� C : ; 5 B
0% Mastopathy
0% Lipoma

654. A 65-year-old patient complains of pain in the lumbar spine, moderate dysuria.

151

Downloaded from the site - online testing step

He's been ill for about six months. Prostate volume - 45 cm3 (there are hypoechoic nodes in both lobes,
capsule invasion). Prostate-specific antigen-60 ng / ml. According to the prostate biopsy-
adenocarcinoma. Which of the additional research methods will allow us to determine the stage
of the tumor process in this patient?
100% Computed tomography of the pelvic organs
0% Radiography of the lumbar spine
0% Excretory urography
0% Chipping bones
0% Chest radiography

655. a 19-year-old girl, emotionally labile, developed severe


weakness, dizziness, darkening of the eyes, nausea and loss of consciousness without convulsions in a stuffy room. Objectively:
unconscious, skin pale, limbs cold. AT-90/60 mm Hg, Ps-96 / min., low
filling, shallow breathing. Pupillary and tendon reflexes are preserved.
There are no pathological signs. What is the most likely diagnosis?
100% Syncopal state
0% Vegetative-vascular paroxysm
0% An epileptic seizure
0% Hysterical neurosis
0% Transient ischemic attack

656. in the air of the caster's working area, there is a condensation aerosol with a dust
particle size of 2 nm (90%), 2-5 nm (2%), more than 5 nm (6%), up to 2 nm (about 2%). Describe
the dust dispersion:
100% Fine-grained
0% Medium-dispersed
0% Magnodisperse
0% Ultra-fine
0% Fog

657. A 40-year-old patient complains of a major spasm, dizziness, muscle weakness, and sometimes
convulsions in the extremities. He has been taking antihypertensive medications for 10 years. AT - 180/100
mm Hg in the blood: potassium-1.8 mmol/l, sodium - 4.8 mmol/l. in the urine: the reaction is alkaline, relative
density-1012, protein and sugar are not detected, leuc. - 3-4 in p/ s, ep. - 1-2 in p/s. suspected
Cohn's disease. What drug should I choose to treat hypertension?
100% Spironolactone
0% Anaprilin
0% Enalapril
0% Hypothyazide
0% Clonidine

658. a 30-year-old patient, a carpenter. Six months ago, changes in behavior began to manifest themselves: he developed

https://translate.yandex.com/en/doc 146/540
22:20 ,27.6.2023 �� C : ; 5 B
an interest in philosophy, began writing a treatise on the purpose of his human existence, left his
main job, stopped caring for children, went out untidily dressed, and heard "
voices in his head" controlling his behavior. I am sure that he is an ambassador from the God of the Earth and

152

Downloaded from the site - online testing step

constantly feels its influence. It is not critical to the disease. What diagnosis can be assumed
in the patient?
100% Schizophrenia
0% Alcoholic psychosis
0% Reactive psychosis
0% Somatogenic psychosis
0% Organic psychosis

659. a 27-year-old patient complains of nosebleeds, multiple bruises on the skin of the anterior
surface of the trunk and limbs, and severe general weakness. In the blood: HB-74 g / l, reticulocytes-16%,
red blood cells-2.5 * 1012/l, platelets - 30- 109/l, ESR-25 mm / hour. What is the most effective way to
treat thrombocytopenia?
100% Splenectomy
0% Iron preparations
0% Blood transfusion
0% Cytostatics
0% Vitamin B12

660. a patient 2 days ago developed a sharp pain in the left side of the chest, general
weakness, fever and headache. Objectively: on the left, in the 45th intercostal
space, there are multiple grouped vesicles filled with transparent contents, 2-4 mm in size, on an erythematous background. What
disease is characterized by the above symptoms?
100% Herpes zoster
0% Pemphigus
0% Herpes simplex
0% Streptococcal impetigo
0% Duhring's herpetiform dermatosis

661. a patient experienced unbearable low back pain after lifting a load. The local therapist
diagnosed acute lumbosacral sciatica. Which of the following
is contraindicated for the patient?
100% Warming procedures
0% Dehydrating agents
0% Analgesics
0% B vitamins
0% Intravenous eufillin

662. institutions involved in conducting medical examinations may


include medical and preventive institutions, medical commissions of the Ministries of Defense and Internal Affairs,
medical and social expert commissions, bureaus of forensic medical expertise, and the like. Who
conducts a medical and social examination of temporary disability?
100% Medical and preventive institutions
0% Health care facilities
0% Medical and social expert commissions

https://translate.yandex.com/en/doc 147/540
22:20 ,27.6.2023 �� C : ; 5 B

153

Downloaded from the site - online testing step

0% Medical commissions of the Ministry of Defense


0% Medical commissions of the Ministry of Internal Affairs

663. 10 minutes after the birth of a child, a litter with


a 5x6 cm tissue defect was released. Discharge from the genital tract is significant, bloody. The tone of the uterus is reduced,
its bottom is located above the navel. When examining the genital tract: the cervix, vaginal walls,
perineum are intact. Clotting blood flows out of the uterus. What should be done first to
stop the bleeding?
100% Perform a manual revision of the uterine cavity
0% Apply hemostatic clamps to the cervix
0% Insert a swab with ether into the posterior arch
0% Put an ice bubble on the bottom of the belly
0% Assign uterotonics

664. a 39-year-old patient, while lifting a load, suddenly felt a strong headache, pain in
the interscapular region; vomiting occurred. Objectively: pulse is rhythmic, 60 / min, AT-180/100 mm
Hg. the patient is excited. Photophobia, hyperacusis. Positive symptoms of Kernig,
Brudzinski on both sides. In the blood: lake.- 10 • 109/L. bloody CSF, cytosis 240/3. which
of the diagnoses is most likely?
100% Subarachnoid hemorrhage
0% Sympatho-adrenal crisis
0% Acute hypertensive encephalopathy
0% Meningococcal meningitis
0% Ischemic stroke

665. A 42-year-old patient was examined by a surgeon. Diagnosis: furuncle of the right forearm,
putrefactive necrotic stage. An autopsy of the boil was performed. What kind of dressing should I put
in the hydration phase?
100% Hypertonic solution
0% Vishnevsky Ointment
0% Ichthyol ointment
0% Chloramine
0% Dimexide

666. A 10-year-old boy was taken to the clinic after being stung by a bee with complaints
of swelling of the lip, face, neck, feeling hot and lack of air. Shortness of breath, noisy,
foamy discharge from the mouth, cough. The skin is pale and cold. Bradypnea. Heart sounds are dull,
arrhythmic. Pulse is threadlike. What diagnosis will the resuscitator make?
100% Anaphylactic shock
0% Angioedema
0% Bronchial asthma
0% Acute cardiovascular insufficiency
0% Cerebral coma

154

https://translate.yandex.com/en/doc 148/540
22:20 ,27.6.2023 �� C : ; 5 B

Downloaded from the site - online testing step

667. a 30-year-old woman developed the disease suddenly when, half an hour after eating
a cake with cream, cutting abdominal pain, nausea, and vomiting appeared. Objectively: the temperature
is 36.0 oC, the skin is pale, BDR-20 / min, Ps-100 / min, AT-95/65 mm Hg, heart tones are sonorous. The tongue is dry.
The abdomen is painful in the epigastrium, there are no symptoms of irritation of the peritoneum. How should
I start treatment?
100% Gastric lavage
0% Purpose of enterosorbent
0% Introduction of Cerucal
0% Performing intravenous rehydration
0% Antibiotic Therapy

668. A 45-year-old sailor patient was hospitalized on the 2nd day of his illness.
I got back from India a week ago. Complaints of a temperature of 41 ° C, severe headache, shortness of breath, cough with
the release of foamy rusty sputum. Objectively: pale, mucosal cyanosis, PDR-24 / min,
tachycardia. In the lungs, breathing is weakened,wet wheezing is heard over both lungs,
crepitation. What is the most likely diagnosis?
100% Plague, a pulmonary form
0% Miliary tuberculosis
0% Flu
0% Ornithosis
0% Sepsis

669. to which blood cells does the human immunodeficiency virus have the greatest tropicity?
100% T-helpers
0% T-suppressors
0% T-killers
0% Platelets
0% Red blood cells

670. various lighting fixtures are used to illuminate educational premises.


What type of lighting fixture creates the most acceptable, from a hygienic point of view,
lighting?
100% Reflected light fittings
0% Direct light fittings
0% Semi-broken light fittings
0% Stray light fittings
0% Combined light fittings

671. a 24-year-old patient complains of sharp pain in the lower abdomen, which occurred suddenly after
physical exertion. Notes nausea, vomiting, dry mouth, to-36.6 oC. A history
of right ovarian cyst. Bimanually: uterus of dense consistency, painless, of normal size.
The left arch is deep, applications are not defined, and the right arch is shortened. To the right of
the uterus, a sharply painful formation is determined, rounded in shape, elastic in consistency,
limited in movement, 7x8 cm. In the blood: leucocytosis with a shift to the left. What is the most likely
diagnosis?

155

https://translate.yandex.com/en/doc 149/540
22:20 ,27.6.2023 �� C : ; 5 B

Downloaded from the site - online testing step

100% Ovarian cyst with twisted pedicle


0% Pyosalpinx on the right
0% Subserous uterine fibromyoma
0% Acute inflammation of the uterine appendices
0% Ectopic pregnancy

672. A 39-year-old female patient complains of a tumor on the anterior surface of the neck. I got sick 2
years ago. The tumor is sedentary, has increased in size, the timbre of the voice has changed, there
is a feeling of pressure. Objectively: in the left lobe of the thyroid gland,a node of 3 cm in diameter,
increased density, lumpy, non-painful is palpated. Enlarged lymph nodes of the neck.
The functional state of the thyroid gland is not changed. What is the most likely diagnosis?
100% Thyroid cancer
0% Nodular euthyroid goiter
0% Nodular hyperthyroid goiter
0% Hashimoto's Chronic Lymphomatous Thyroiditis
0% Riedel's chronic fibrous thyroiditis

673. a 22-year-old sick office worker. Open all day in an air-conditioned room.
In the summer, he became acutely ill: high fever, shortness of breath, dry cough,pleural pain,myalgia,
arthralgia. Objectively: wet wheezing on the right, pleural friction noise. Radiologically
, infiltration of the lower lobe persists for a long time. In the blood: lake.-11 * 109/l, p. - 6%, c. - 70%, lymph. - 8%,
ESR-42 mm / h. What is the etiological factor of pneumonia?
100% Legionella
0% Mycoplasma
0% Streptococcus
0% Staphylococcus aureus
0% Pneumococcus

674. a 53-year-old woman complains of pain and heart failure. She has been ill since childhood, and her father
suffered from arrhythmia. Objectively: the condition is severe, Ps-220 / min, AT-80/60 mm Hg ECG:
Heart rate-215 / min, expansion and deformation of the QRS complex with the presence of atrioventricular
dissociation; The P-wave is positional. In the future, the heart rate decreased to
45 / min, complete separation of the P wave and the QRST complex. Which of the above is most
effective in treating the patient?
100% Artificial pacemaker implantation
0% beta-blockers
0% Holinolytics
0% Calcium antagonists
0% Cardiac Glycosides

675. a 49-year-old patient complains of choking and coughing. Sputum does not excrete. Repeatedly
applied salbutamol, Intal, but without effect. Objectively: sitting, leaning on the table.
Cyanosis of the face, acrocyanosis. Breathing shallow, labored, sometimes not audible;
scattered wheezing, significantly prolonged exhalation. Heart-muted tones, tachycardia. Ps-112 / min,
AT110 / 70 mm Hg liver at the edge of the costal arch. There is no peripheral edema. Which

156

Downloaded from the site - online testing step

https://translate.yandex.com/en/doc 150/540
22:20 ,27.6.2023 �� C : ; 5 B
preliminary diagnosis of the patient?
100% Asthmatic status
0% Chronic obstructive bronchitis
0% Moderate bronchial asthma
0% Foreign body aspiration
0% Cardiac asthma

676. a 3-year-old girl has a whooping cough with


a thick sputum discharge. Persistent changes in the lungs from 6 months when acute pneumonia was first diagnosed.
Chlorides in sweat-112 meq/l. Cystic fibrosis was diagnosed. What is the basis of this disease?
100% Violation of cellular transport of chlorine and sodium ions
0% A1-antitrypsin deficiency
0% Alveolar deposits of calcium triphosphates and carbotates
0% Cysts of the lungs
0% Hypoplasia of the pulmonary arteries

677. Rodilya is 23 years old. During internal obstetric examination, the opening of the cervix
is complete. There is no fertile bubble. The head lies in the plane of the exit from the small pelvis.
Arrow-shaped suture in the direct size of the exit from the pelvis, small crown closer to the womb. What
size of the head will the fetus be born with this presentation option?
100% Small oblique
0% Direct
0% Crosswise
0% Medium oblique
0% Big oblique

678. A 42-year-old woman has been menstruating in the form of


hyperpolymenorrhea and progressive algodismenorrhea for the last 10 years. Gynecological examination: the cervix is not
changed, the discharge is moderate, "chocolate" color, the uterus is slightly enlarged, painful, the appendices are not
palpable, the arches are deep, painless. What is the most likely diagnosis?
100% Endometriosis of the uterus
0% Uterine cancer
0% Subserous uterine fibromyoma
0% Endomyometritis
0% Endometriosis of the appendages

679. A 38-year-old patient


was found to have suffered an x-rib fracture on the left side with dislocated fragments and a parietal pneumothorax as a result of a blunt
object blow to the left side of the chest
. Notes pain in the left hypochondrium. objectively: pale, blood pressure-80/40 mm Hg
Ps138 / min., weak filling and tension. Ultrasound revealed fluid in the left side of the abdomen.
A ruptured
100% spleen wasthe
Drain detected. Whatcavity
left pleural treatment tactics should
and perform I choose?
a laparotomy
Perform an upper-median laparotomy immediately and then drain the left
0%
pleural cavity
0% Immediately perform laparotomy and alcohol-novocaine blockade of Hm

157

Downloaded from the site - online testing step

Perform anti-shock measures and perform a laparotomy after increasing blood pressure
0%

https://translate.yandex.com/en/doc 151/540
22:20 ,27.6.2023 �� C : ; 5 B
0% Do a left-sided thoracotomy, and then immediately laparotomy

680. A 4-month-old boy showed signs of Quincke's edema 15 minutes after the second DPT vaccination
. What medication should I use to provide
emergency care to my child?
100% Prednisone
0% Heparin
0% Adrenaline rush
0% Furosemide
0% Seduxenum

681. A 35-year-old patient complains of heart pain on the 2nd day after surgery for mixed toxic goiter IV
. On the ECG: prolongation of the Q-T interval. Symptoms of tailedema,
Truso are not clearly defined. A preliminary diagnosis of latent tetany was made. What research
should be done to confirm the diagnosis?
100% Determine the content of calcium and phosphorus in the blood
0% Determine the content of thyroid-stimulating hormone
0% Determine the potassium content
0% Determine the sodium content
0% Determine the content of thyroid hormones in the blood

682. A patient with bilateral hydrothorax underwent multiple pleural punctures on both
sides. After the next puncture, the condition worsened-fever, chest pain. The next
day, the attending physician received pus during a pleural puncture on the right side. What is the mechanism
of occurrence of right-sided acute empyema?
100% Contact-aspiration pump
0% Lymphogenic
0% Hematogenic
0% Implantation
0% Air

683. A student evaluates the noise level in a cold forming shop. What
device does he use to perform this hygiene test?
100% noise and vibration meter
0% Noise Spectrum Analyzer
0% audio test
0% actinometer
0% pyranometer

684. Mechanical separation


of non-toxic solid household waste is used to prevent environmental pollution. What is the method that can be
used for mechanical disposal of this waste?:

158

Downloaded from the site - online testing step

100% Compacting waste into building blocks


0% Hydrolysis
0% Incineration as an energy fuel
0% Landfill disposal of waste
https://translate.yandex.com/en/doc 152/540
22:20 ,27.6.2023 �� C : ; 5 B
0% Decontamination in biothermal chambers

685. a 26-year-old woman complains of spotting from the genital tract during
the last 14 days, pain in the lower abdomen, general fatigue, weakness, weight
loss, fever, chest pain, difficulty breathing. 5 weeks ago, I underwent
an artificial termination of pregnancy at a period of 6-7 weeks. Objectively: pale, lethargic. In
a bimanual study: the uterus is enlarged to 8-9 weeks of pregnancy. In the blood: Hb-72 g/l
. the urine test for HCG is sharply positive. What is the most likely diagnosis?
100% Chorionic epithelioma
0% Metroendometritis
0% Uterine perforation
0% Uterine fibromyoma
0% Cancer of the uterine body

686. A 28-year-old woman complains of nausea and vomiting up to 10 times a day.


Notes weight loss, dry skin. Heart rate up to 100 / min. Body temperature 37.2 ° C.
Reduced diuresis. After an ultrasound examination
, a pregnancy of 5-6 weeks was detected. What is the most likely diagnosis?
100% Moderate vomiting of pregnant women
0% Mild vomiting of pregnant women
0% Grade 1 preeclampsia
0% Premature termination of pregnancy
0% Food poisoning

687. a full-term baby was born with a body weight of 3200 g, a body length of 50 cm, with
an Apgar score of 8-10 points. What is the optimal time for the first application of it to the breast?
100% In the first 30 minutes
0% In the first 6 hours
0% In the first 24 hours
0% In the first 48 hours
0% After 48 hours

688. a 26-year-old patient with left-sided mandibular pneumonia developed a


sharp pain in the left chest during coughing. Objectively: diffuse cyanosis, dilatation of the left half
of the chest. Percussion-high tympanite. Auscultation-absence of respiratory murmurs
over the left half of the chest. Displacement of the right border of the heart to the midclavicular line.
Which survey will be the most informative?
100% Radiography
0% Bronchoscopy
0% Bronchography

159

Downloaded from the site - online testing step

0% Pneumotachometry
0% Spirography

689. A 3-year-old child was brought to the pediatrician's office. I haven't been ill lately. During
an objective examination, no pathology from the internal organs was detected. What
disease should this child be immunized against on a planned basis?
https://translate.yandex.com/en/doc 153/540
22:20 ,27.6.2023 �� C : ; 5 B
100% Polio
0% Diphtheria and tetanus
0% Measles, rubella, and mumps
0% Whooping cough
0% Hepatitis B infection

690. A 37-year-old patient has severe pain and bleeding in


the middle third of the right thigh after a car accident. Objective: wound on the anterior surface of the right thigh with
massive bleeding, abnormal mobility at the level of the middle third of the thigh. How should
I start providing assistance?
100% Perform finger compression of the femoral artery
0% Enter painkillers
0% Apply a tourniquet
0% Immobilize the limb with a transport tire
0% Perform venipuncture and start intravenous infusion of polyglucin

691. a 9-year-old boy fell from a tree, hit the occipital region,
and fainted for a short time. Objectively: the child's condition is satisfactory, bothered by headache,
dizziness. X-rays of the skull revealed a depressed fragmentary fracture
of the occipital bone in the occipital protuberance. What treatment is indicated for the patient?
100% Surgical intervention
0% Anti-inflammatory therapy
0% Hemostatic therapy
0% Unloading lumbar punctures
0% Comprehensive conservative treatment

692. A 49-year-old female patient underwent surgery 14 days ago for perforated appendicitis,
a common fibrinous-purulent peritonitis. The postoperative period
was uneventful. On day 9, the patient developed a low-grade fever, abdominal pain, and frequent
liquid bowel movements. Ultrasound of the abdominal cavity in the left mesogastric region
determines a fluid formation of 9x10 cm. In the blood: leukocytosis with a shift of the leukocyte formula
to the left. What is the preliminary diagnosis?
100% Inter-loop abscess
0% Abdominal tumor
0% Liver abscess
0% Cyst of the left kidney
0% Spleen abscess

160

Downloaded from the site - online testing step

693. The Carpathian region is characterized by constant high (more than 80%)
atmospheric humidity. During the cold period of the year, with moderately low air temperatures
, the population of this region experiences severe cold. This is due to an increase in heat output
by:
100% Convections
0% Radiation
0% Evaporation
0% Conduits

https://translate.yandex.com/en/doc 154/540
22:20 ,27.6.2023 �� C : ; 5 B
0% Of radiation

694. A 22-year-old patient complains of aching pain in the right iliac region that has been bothering
her for a week, morning sickness, and changes in taste. From the medical history: delayed menstruation-3 weeks.
Objectively: AT-110/70 mm Hg, Ps-78 / min, to-37.0 oC. Bimanually: the uterus is slightly enlarged,
softened, mobile, painless. Appendices: on the right,a painful formation of 3x4 cm is palpated,
tightly elastic consistency, moderately mobile. What is the most likely diagnosis?
100% Progressive tubal pregnancy
0% Aborted tubal pregnancy
0% Right ovarian cyst
0% Uterine pregnancy
0% Acute appendicitis

695. a 65-year-old female patient 3 hours ago felt a sharp pain in the abdomen radiating to the right shoulder blade;
she had a single vomiting. He is being treated for rheumatoid arthritis. Objectively:
skin pallor, blood pressure-100/60 mm Hg, Ps-60 / min. The abdomen is significantly painful and tense in the epigastrium and
right hypochondrium, positive symptoms of irritation of the parietal peritoneum, over the right
costal arch-tympanitis. What are the emergency doctor's tactics?
100% Deliver the patient to a surgical hospital
0% Enter painkillers, dynamic monitoring
0% Wash out the stomach
0% Administer antispasmodic medications
0% Take the patient to the rheumatology department

696. A 72-year-old patient complains of pain and blood discharge during defecation.
Rectal finger examination revealed a tumor of the anal canal of the rectum.
The resulting verification is squamous cell carcinoma. Where is the first place to look for a secondary
(metastatic) tumor?
100% Lungs
0% Liver
0% Pelvic bones
0% The mediastinum
0% The brain

697. on the fourth day after receiving a stab wound to the right foot, the patient
's body temperature rose to 38oC, the inguinal lymph nodes became enlarged, painful, and the skin above
them turned red. What complication of the wound can you think about?

161

Downloaded from the site - online testing step

100% Lymphadenitis
0% Lymphangitis
0% Phlegmon
0% Tetanus
0% Erysipelas

698. a 25-year-old female patient complains about the absence of menstruation for 3 years, which is associated with
a difficult birth, which was complicated by massive bleeding, weight loss, brittleness and
hair loss, lack of appetite, depression. Objective examination: uterus and
appendages without pathological changes. What is the pathogenesis of the disease?

https://translate.yandex.com/en/doc 155/540
22:20 ,27.6.2023 �� C : ; 5 B
100% With reduced gonadotropin production
0% With hyperproduction of estrogens
0% With androgen overproduction
0% With reduced progesterone production
0% With hyperproduction of prolactin

699. during surgical intervention, a 30-year-old patient was found


to have a dark-colored small-intestinal conglomerate, in which the intestine was recognized as nonviable. The adductor colon
is dilated to 7-8 cm, swollen, overflowing with intestinal contents and gas. What pathology
led to surgical intervention?
100% Intussusceptible (mixed) obstruction
0% Strangulation obstruction
0% Obturation obstruction
0% Paralytic obstruction
0% Spastic obstruction

700.In the current year


, 11 patients with coronary heart disease were referred to the hospital by general practitioners of the city polyclinic. In 3 cases, the
diagnosis was not confirmed. What
is the100%
best management
Analysisdecision to make
of each case in this case?
of divergence of diagnosis
0% Analysis of the quality of dispensary follow-up
0% Analysis of the quality of diagnostic tests
0% Analysis of the level of qualification of polyclinic doctors
0% Analysis of the polyclinic's material and technical base

701. in the ward of the therapeutic department, it is necessary to analyze the level of natural
light. What device is used to determine the level of natural light?
100% Lux Meter
0% Anemometer
0% Catathermometer
0% Actinometer
0% Psychrometer

702. A 43-year-old patient was admitted to the clinic of ischiorectal paraproctitis. On the 12th day

162

Downloaded from the site - online testing step

treatment the patient's condition worsened dramatically: the level of intoxication and liver
failure began to increase, the body temperature was hectic, blood pressure was 100/60 mm Hg. ultrasound revealed
hydrophilic formation in the liver. In the blood: lake.- 19,6 • 109/ l, Er.- 3,0 • 1012/ l, HB-98 g / l. what
is the complication of the course of the disease?
100% Liver abscess
0% Pylephlebitis
0% Liver cyst
0% Liver necrosis
0% Budd-Chiari syndrome

703. A 32-year-old pregnant woman complains of episodes of loss of consciousness, spontaneous syncope
that quickly disappears when the body position changes. Syncope may be accompanied

https://translate.yandex.com/en/doc 156/540
22:20 ,27.6.2023 �� C : ; 5 B
by rapidly passing bradycardia. Otherwise, pregnancy proceeds without complications. What
is the most possible reason for this condition?:
100% Compression of the inferior vena cava by a pregnant uterus
0% Increased pressure in the veins of the upper extremities
0% Reduction of pressure in the veins of the lower extremities
0% Cardiac-type vegetative-vascular dystonia
0% Psychosomatic disorders

704. A 54-year-old woman applied to the gynecologist with complaints of vaginal bleeding
for 1 month. Last menstruation 5 years ago. Gynecological examination
revealed no pathology. What are the doctor's actions?
100% Fractional diagnostic curettage of the uterine cavity walls
0% Colposcopy
0% Ultrasound examination
0% Take a smear for cytological examination
0% Prescribe symptomatic therapy

705. a patient, an employee of an enterprise, has acute respiratory infections complicated by acute bronchitis. It is treated
on an outpatient basis. The attending physician gave him a disability certificate for 5 days and extended it for another
5 days. The patient's state of health does not allow him to start working. Who, together with the attending
physician, should continue the disability certificate for this patient?
100% Department Head
0% Chief Medical Officer
0% Deputy Chief Physician for Health Expertise
0% Deputy Chief Medical Officer
0% Medical Advisory Commission

706.A 21-year-old woman was taken by ambulance to the gynaecological department


complaining of cramping pain in the lower abdomen and spotting from the genital tract.
Bimanually: the uterus is enlarged up to 6 weeks of pregnancy, soft consistency,
a fetal egg is palpated in the cervical canal. Appendages are not palpable. The arches are free, deep,
and painless. Discharge from the genital tract is bloody, abundant. What is the most likely diagnosis?
100% Miscarriage on the go

163

Downloaded from the site - online testing step

0% Cervical pregnancy
0% Risk of miscarriage
0% Miscarriage that started
0% Disrupted tubal pregnancy

707. according to the laboratory quality control of drinking water from the water supply system, the following
results were obtained: turbidity -1.5 mg / m3, smell-3 points, taste-metallic-2 points, color-
light yellow, chroma-20°, temperature-12oC. Which of the indicators does not meet the hygiene
requirements?
100% Smell
0% Turbidity
0% Chroma value
0% Temperature
0% Aftertaste

https://translate.yandex.com/en/doc 157/540
22:20 ,27.6.2023 �� C : ; 5 B

708. A 25-year-old female patient


developed purulent discharge from the genital tract and itching after an accidental sexual encounter five days ago. During vaginal
examination: the vaginal
part of the cervix is hyperemic, edematous, there is an erosion site around the outer eye
, abundant mucopurulent discharge flows from the cervical canal; the body of the uterus and appendages are without
features. Bacterioscopic examination revealed bean-shaped diplococci,
while Gram-colored diplococci turn red. What is the most likely
diagnosis?
100% Acute gonorrheal endocervicitis
0% Trichomonas colpitis
0% Candida vulvovaginitis
0% Chlamydial endocervicitis
0% Bacterial vaginosis

709. The relationship


between work experience and blood eosinophil content was studied in female workers working in textile factory paint shops. What is the
most appropriate indicator to use
to analyze
100% this data?
Correlation coefficient
0% Student's Criterion
0% Standardized indicator
0% Compliance criteria
0% Indicator of characters

710. A 30-year-old female patient was admitted to the gynecological department with complaints of sharp pain in
the lower abdomen and a temperature of 38.8°C. In the anamnesis-sexual life outside of marriage, 2 artificial abortions.
During gynecological examination: the uterus is not changed. Appendages-enlarged, painful on both sides.
Vaginal discharge purulent, significant. What research should be done to confirm
the diagnosis?
100% Bacteriological and bacterioscopic examination
0% Hysteroscopy
0% Scraping the walls of the uterine cavity

164

Downloaded from the site - online testing step

0% Colposcopy
0% Laparoscopy

711. A 38-year-old patient complains of weakness, subfebrility, enlarged lymph


nodes, nosebleeds, and bone pain. Objectively: pallor of the skin and mucous membranes, palpable
enlarged painless lymph nodes, sternalgia, liver +2 cm, spleen +5 cm, painless. In
the blood: Er-2.7 • 1012/l, Hb-84 g/l, leuc.- 58 • 109/l, E. - 1%, P.-2%, C.-12%, lymphocytes.- 83%,
lymphoblastic cells -2%, Botkin-Iumprecht cells; ESR-57 mm / h. What is the most likely diagnosis?
100% Chronic lymphocytic leukemia
0% Chronic myeloid leukemia
0% Acute lymphocytic leukemia
0% Acute myeloid leukemia
0% Lymphogranulomatosis

712. A 25-year-old road accident victim is concerned about chest pain and shortness of breath. Objectively:
the condition is severe, Ps-120 / min, AT-90/70 mm Hg. pathological mobility of fragments of III-V ribs on the right.

https://translate.yandex.com/en/doc 158/540
22:20 ,27.6.2023 �� C : ; 5 B
Percussion over the lung on the right-box sound, breathing is not even listened to. What
kind of examination should be prescribed first?
100% Chest radiography
0% Bronchoscopy
0% Puncture of the pleural cavity
0% Ultrasound of the chest organs
0% Thoracoscopy

713. a 58-year-old patient complains of a feeling of numbness, sharp paleness of the II-IV fingers
of the hands, a feeling of stiffness in the muscles, interruptions in the work of the heart. In addition,polyarthralgia,
manifestations of dysphagia, constipation were detected. The patient's face is "masklike", dense swelling of the hands. The size
of the heart is increased, dry wheezing is heard in the lungs. In the blood: ESR-20 mm / h, total
protein -85 g / l, y-globulins-25%. What is the most likely diagnosis in this case?
100% Systemic scleroderma
0% Dermatomyositis
0% Rheumatoid arthritis
0% System red shepherd dog
0% Raynaud's disease

714. during the last two weeks, a 45-year-old man became very active, became
multilingual, euphoric, slept little, claimed that he could "save humanity", was able to solve
the problem of cancer and AIDS, and distributed money to strangers. What is the most likely diagnosis?
100% A manic attack
0% Panic disorder
0% Agitated depression
0% Schizoaffective disorder
0% Catatonic arousal

165

Downloaded from the site - online testing step

715. a 25-year-old woman in labor, on the second day of the postpartum period. The first delivery, urgent,
passed without complications. What should be the character of lochias?
100% Blood cells
0% Blood-serous diseases
0% Mucous membranes
0% Purulent
0% Serous ones

716. A 32-year-old female patient went to the doctor complaining that she had not been pregnant for
4 years. 5 years ago, my first pregnancy ended with an artificial abortion. According to the vaginal
examination and ultrasound, the diagnosis was made: endometrioid cyst of the right ovary. What
is the best treatment method?
100% Operative laparoscopy
0% Anti-inflammatory therapy
0% Conservative therapy with estrogen-progestogenic drugs
0% Hormone therapy with male sex hormones
0% Spa treatment

https://translate.yandex.com/en/doc 159/540
22:20 ,27.6.2023 �� C : ; 5 B
717. a 6 - year-old girl developed a feeling of pressure in her throat after drinking carbonated colored water
. After 30 minutes, there was swelling of the lips, gradually developed swelling of the face and
difficulty in laryngeal breathing. The child is excited. Ps-120 / min., PDR-28 / min., breathing is noisy,
retraction of intercostal spaces. What basic help is most appropriate for restoring
laryngeal breathing?
100% Corticosteroid medications
0% Sedatives
0% Tracheostomy
0% Antibacterial drugs
0% Conicotomy

718. A 60-year-old patient suddenly developed pain in the right


hypochondrium, nausea, vomiting of bile, and a sharp feeling of bitterness in the mouth after excessive consumption of fatty foods. Two
days
later, jaundice appeared, the urine darkened. Objectively: the sclera and skin are icteric, the belly is swollen,
the liver is enlarged by 3 cm, soft, painful during palpation, with-mi Ortner, Murphy,Ker,
Zakharyin, Mayo-Robson positive. What method should be used for diagnosis
first?100% Ultrasound of the gallbladder and bile duct
0% Fibrogastroduodenoscopy
0% Radiography of the abdominal organs
0% Radionuclide scan of the liver and gallbladder
0% Diagnostic laparoscopy

719. A 20-year-old patient complains of nosebleeds and numbness of the lower extremities. Objectively:
hyperemia of the face. In the upper extremities, blood pressure is 160/90 mm Hg, in the lower extremities-80/50 mm Hg. on
the popliteal arteries and arteries of the feet, the pulse is weak, filling and tension, above the carotid
arteries there is systolic noise. What is the most likely diagnosis?

166

Downloaded from the site - online testing step

100% Aortic coarctation


0% Dissecting aortic aneurysm
0% Non-occlusion of the ductus arteriosus
0% Ventricular septal defect
0% Atrial septal defect

720. An 8-month-old infant experiences a week-long deterioration of breathing through the nose and
mucopurulent discharge from it. Examination revealed swelling
of the nasal mucosa,mucopurulent discharge from the middle nasal passage, as well as along the back wall of the pharynx.
Which of these diseases is most characterized by these symptoms?
100% Etmoidite
0% Sphenoiditis
0% Sinusitis
0% Frontit
0% Gemisinuit

721. the head of the hospital department wants to conduct an expert assessment of the implementation
of medical and technological standards of patient care by doctors ' coordinators. What documentation
should it check for this purpose?
100% Medical records of inpatient patients

https://translate.yandex.com/en/doc 160/540
22:20 ,27.6.2023 �� C : ; 5 B
0% Statistical maps of patients who left the hospital
0% Medical appointment cards
0% Surgical intervention Logbook
0% Annual report of a health care facility

722. While assessing the health status of graduates of a general secondary school, a doctor found
that one of the students had grade III hypertrophy of the tonsils, chronic rhinitis, and vegetative-vascular dystonia.
The functional capabilities of the body are reduced. For health reasons, this student belongs to:
100% Group III
0% Groups II
0% I groups
0% Groups IV
0% V groups

723. a 17-year-old patient developed pain in the area of the left knee joint. The soft tissues of the hip in
the area of pain are infiltrated, and the joint function is limited. Radiologically-in
the distal metaepiphyseal part of the left femur, a lesion with periosteal detachment and
the formation of a defect in the cortical layer of the triangle bone ("dasha")at the border Codman.
X-ray examination of the chest revealed multiple small-focal
metastases. What treatment is indicated?
100% Palliative chemotherapy course
0% Radioiodine Therapy
0% Remote gamatherapy
0% Exarticulation of the lower limb

167

Downloaded from the site - online testing step

0% Lower limb amputation

724. A 62 - year-old woman, after lifting a load, felt acute pain in the lumbar region, buttock,
posterolateral surface of the right thigh, outer surface of the right shin and back
of the foot. Objectively: weakness of the anterior tibial muscle, long extensor
of the thumb, short extensor of the fingers of the right foot. Reduced Achilles reflex on the right.
Positive Lasega symptom. What is the most informative method of investigation to clarify
the diagnosis of discogenic compression of the L5 root?
100% Magnetic resonance imaging
0% Radiography of the spine
0% Electromyography
0% Angiography
0% Lumbar puncture

725. at the first appointment of a pregnant woman, the obstetrician-gynecologist gives her a referral for consultation with
other specialist doctors. Which specialists must have it examined?
100% Therapist, dentist
0% Therapist, endocrinologist
0% Dentist, phthisiatrist
0% ENT specialist, optometrist
0% Dentist, cardiologist

https://translate.yandex.com/en/doc 161/540
22:20 ,27.6.2023 �� C : ; 5 B
726. A 19-year-old female student was urgently hospitalized due to severe shortness
of breath and pain in the left side of her chest. I got sick 3 days ago. Objectively: body temperature -38.8
oC. PDR-42 / min., shallow breathing. Percussion to the right of the middle of the scapula
sound is dulled, breathing is not listened to. The left border of the heart is shifted outward by 3 cm
. Embryocardia, heart rate-110 / min. Palpation of the right hypochondrium is painful. What are the urgent medical
measures in this situation?
100% Urgent puncture of the pleural cavity
0% Prescribing penicillin antibiotics
0% Introduction of Lasix
0% Administration of cardiac glycosides
0% Transfer of the patient to the thoracic surgery department

727. a pregnant woman was admitted to the gynecological department with complaints of pain in the lower abdomen,
minor spotting from the genital tract for 3 hours. My last period was 3
months ago. A vaginal examination revealed: the body of the uterus corresponds to 10 weeks
of pregnancy, the external eye passes the tip of the finger, the discharge is bloody insignificant. On ultrasound:
small blisters in the uterine cavity. What is the most likely diagnosis?
100% Bubble drift
0% Miscarriage on the go
0% Miscarriage that started
0% Risk of spontaneous miscarriage
0% Incomplete miscarriage
728. A 30-year-old patient went to the family doctor 2 months after the operation for

168

Downloaded from the site - online testing step

open fracture of the humerus. Objectively: the patient's condition is satisfactory,


there is a fistula with a slight Purulent discharge, redness, and fluctuation in the area of the surgical wound. The
radiograph shows destruction of the humerus with sequestrations. What complication did the patient
have in the postoperative period?
100% Post-traumatic osteomyelitis
0% Hematogenous osteomyelitis
0% Suppuration of the wound
0% Post-traumatic phlegmon
0% Ligature fistula

729.A 50-year-old male resident of the village of punctovo did not recover after receiving outpatient
care in a rural outpatient clinic for pneumonia, the disease
was complicated by exudative pleurisy. What institution should the patient be referred
to by the family doctor to continue providing care?
100% Central District Hospital
0% Regional Hospital
0% Phthisio-pulmonological dispensary
0% City Hospital
0% TB dispensary

730. a patient is preparing for surgery for varicose veins of the lower extremities.
Examination of the soles revealed flaky flaking along the skin folds. All the nails of the feet
are grayish-yellow, thickened and partially destroyed. What kind of dermatosis should I think about in
this case?
100% Rubromycosis
https://translate.yandex.com/en/doc 162/540
22:20 ,27.6.2023 �� C : ; 5 B

0% Pityriasis versicolor
0% Candidiasis
0% Microsporia
0% Microbial eczema

731. a 58-year-old patient complains of headache in the back of the head, nausea, suffocation,
flickering of "butterflies" in front of the eyes, which occurred after physical exertion. Objectively:
agitated, flushed face, pale skin. The heart tones are rhythmic, with an accent of tone II over
the aorta. AT - 240/120 mm Hg HEART rate-92 / min. In the lower parts of the lungs, there is a small amount
of small-bubbly wet wheezing. The liver is not enlarged. ECG shows signs of
left ventricular hypertrophy and overload. What is the most likely diagnosis?
100% Complicated hypertensive crisis, pulmonary edema
0% Acute myocardial infarction, pulmonary edema
0% Exacerbation of bronchial asthma
0% Uncomplicated hypertensive crisis
0% Non-hospital-acquired pneumonia

732. A 43-year-old female patient was hospitalized in serious condition. He has Addison's disease. Constantly
took prednisone. I stopped taking it within a week. Objectively: soporotic state,
skin and visible mucous membranes are pigmented, skin and muscle turgor is reduced. Heart Tones

169

Downloaded from the site - online testing step

muted, accelerated, blood pressure-60/40 mm Hg, heart rate-96/min. In the blood: Na-120 mmol / l, K-5,8
mmol/l. what hormone deficiency plays a leading role in the development of this complication?
100% Cortisol
0% Corticotropin (ACTH)
0% Adrenaline rush
0% Norepinephrine
0% Androstenedione

733. during the forensic examination of the newborn's corpse, it was revealed: weight 3500 g,
body length 50 cm, the umbilical cord is smooth, moist, shiny, without signs of drying out. Swimming
lung tests are positive. What do the results of swimming tests indicate in this case?
100% The baby was born alive
0% The baby was stillborn
0% Primary atelectasis
0% Secondary atelectasis
0% Hyaline membrane disease

734. A 22-year-old pervovagite with Rh ( -) negative blood type has a Rh(+)


positive male. No Rh antibodies were detected before 32 weeks. At 35 weeks of gestation
, no Rh antibodies were detected during re-determination. What is the frequency of further
antibody detection?
100% Once a week
0% Once every two weeks
0% Once every three weeks
0% Once a month
0% Further definition is impractical

https://translate.yandex.com/en/doc 163/540
22:20 ,27.6.2023 �� C : ; 5 B

735. A 50-year-old patient, a construction worker with 20 years of work experience, was admitted to the clinic with complaints of
chest pain, dry cough, and slight shortness of breath. Objectively: sallow skin,
acrocyanosis, asbestos warts on the hands. Auscultation: hard breathing, scattered dry
wheezing. On the X-ray-the pulmonary pattern is enhanced, signs of emphysema of the lungs. What
is the most likely diagnosis?
100% Asbestosis
0% Lung cancer
0% Pneumonia
0% Chronic obstructive bronchitis
0% Tuberculosis

736. a 14-year-old girl complains of pain in the vaginal area and lower abdomen, which lasts 3-4
days, has been bothering her for the last 3 months at approximately the same time and
is getting worse each time. Objectively: the mammary glands are developed, the hairiness corresponds to the age. Hymen
without opening, cyanotic, protruding. There is no menstruation and never was. The doctor diagnosed
primary amenorrhea. What is the cause of amenorrhea?
100% Hymen atresia
0% Shereshevsky-Turner syndrome

170

Downloaded from the site - online testing step

0% Pehrantz-Babinski-Fröhlich syndrome
0% Pregnancy
0% Delayed sexual development

737. a 22-year-old female patient of reduced nutrition, vegetarian, applied to the polyclinic with
complaints of perversion of the sense of smell, taste, "jamming" in the corners of the mouth. Objectively: pronounced blueness
of the sclera. The diagnosis was iron deficiency anemia. Which clinical syndrome has
the advantage?
100% Sideropenic
0% Anemic
0% Gemological Center
0% Hemolytic
0% Myelodysplastic

738. a patient with purulent otitis media experienced a sharp deterioration in his condition: headache,vomiting,
febrile fever, and general hyperesthesia. Meningeal signs, stagnant
discs of the optic nerves are detected. There are no focal symptoms. The cerebrospinal fluid is cloudy, high blood pressure,
cell-protein dissociation due to neutrophils. What disease should I think about?
100% Secondary purulent meningitis
0% Meningoencephalitis
0% Serous meningitis
0% Primary purulent meningitis
0% Subarachnoid hemorrhage

739. According to the inpatient report of the city hospital, information was obtained on the number
of patients operated on, including the number of deaths after the operation. What indicator
of hospital performance can be calculated based on these data?
100% Postoperative lethality

https://translate.yandex.com/en/doc 164/540
22:20 ,27.6.2023 �� C : ; 5 B
0% Overall mortality rate
0% Late hospitalization rate from the onset of the disease
0% Postoperative mortality
0% Standardized mortality rate

740. a 10-year-old boy periodically experiences short-term states (up to 10-15 seconds) in
the form of" sudden disconnection", which are accompanied by stopping and fixing the gaze in
a straight position, absent meaningless facial expression, stopping movements and subsequent
amnesia. What is the most likely condition described?
100% Absence
0% Obnubilation
0% Trans
0% Fugue
0% Sperrung

741. a 20-year-old patient who was delivered from the street in the summer with brachial artery bleeding, when

171

Downloaded from the site - online testing step

a tourniquet was applied to provide first aid to temporarily stop the bleeding
. Specify the maximum exposure of the harness:
100% 120 minutes
0% 15 minutes
0% 30 minutes
0% 60 minutes
0% 180 minutes

742. A 3-year-old child was admitted to the department with complaints of osalgia, fever
up to 39oc. Objectively: the child's condition is severe, cannot stand due to osalgia, severe
intoxication, lymph nodes up to 1.5 cm. The liver protrudes 3 cm and the spleen 2 cm from under
the edge of the costal arch. In the blood: Er.- 3.0 • 1012/ l, Hb-87 g / l, CP-0.87, blood clot.- 190-104, lake.- 3,2
-109/ l, E.-1%, P.-1%, S.-0, L.-87%, M.-2%, SHOE-36 mm / year. What research should be done to
determine the diagnosis?
100% Sternal puncture
0% Ultrasound examination
0% Lymph node puncture
0% Lymph node biopsy
0% Computed tomography

743. A 22-year-old girl complains of an itchy rash on her face for 2 days
. The disease is associated with the use of cosmetic cream. Objectively:
pronounced redness and swelling of the skin in the cheeks, chin, forehead; the presence of a small
papulo-vesicular rash. What is the most likely diagnosis?
100% Allergic dermatitis
0% Simple dermatitis
0% Eczema
0% Erysipelas
0% Neurodermatitis

https://translate.yandex.com/en/doc 165/540
22:20 ,27.6.2023 �� C : ; 5 B
744. A girl was born with an Apgar score of 7-8 points at 1-5 minutes. During
labor, there was a short-term difficulty in pulling out the shoulder girdle. After
birth, the child has a violation of the function of the proximal part and a forced
position of the right handle. The shoulder is turned inwards, the elbow is unbent,
pronation of the forearm is noted, the hand is bent in the form of a"doll's hand". What is the clinical diagnosis of this
child?
100% Duchesne-Erba Pareze
0% Thoracic spine injury
0% Osteomyelitis of the right hand
0% Intracranial hemorrhage
0% Right hand soft tissue injury

745. A 27-year-old patient with a history of bronchial asthma was stung by a bee. There
were sensations of pressure in the chest, lack of air, difficulty exhaling, heat sensations in the upper half
of the body, dizziness, pronounced itching, convulsions. Objectively: breathing is noisy, wheezing, AT-

172

Downloaded from the site - online testing step

90/60 mmHg, Ps-110 / min. Auscultation: heart sounds are rhythmic, weakened; over the lungs-
hard breathing, dry wheezing. Which group of drugs should be used first?
100% Glucocorticoids
0% Methylxanthines
0% Cardiac Glycosides
0% Anticonvulsants
0% Analgesics

746. A 24-year-old patient went to the doctor complaining of enlarged submandibular


lymph nodes. Objectively: enlarged submandibular, axillary and inguinal lymph nodes.
Chest X-ray shows enlarged mediastinal lymph nodes. In the blood: Er -3.4 •
1012/l, hL-100 g/l, CP-0.88, tr.-190- 109/ l, lake.- 7,5- 109/l, e, - 8%, p, - 2%, c. - 67%, lymph.- 23%,
ESR-22 mm / h. What research is indicated to verify the cause of lymphadenopathy?
100% Open lymph node biopsy
0% Ultrasound examination of the abdominal organs
0% Tomography of the mediastinum
0% Puncture biopsy of lymph nodes
0% Sternal puncture

747. A 32-year-old patient complained of heart failure, dizziness,


and shortness of breath during exercise. I haven't been ill yet. Objectively: Ps-74 / min,
rhythmic. AT-130/80 mm Hgon auscultation: first tone of normal sonority,
systolic murmur over the aorta. On the ECG: left ventricular hypertrophy, signs
of repolarization disorders in the I, V5, V6 leads. Echocardiography: interventricular membrane 2 cm. What is the most
likely diagnosis?
100% Hypertrophic cardiomyopathy
0% Aortic stenosis
0% Hypertension
0% Myocardial infarction
0% Aortic coarctation

748. The clinic of the Research Institute of Occupational Diseases has established that a worker working at
a processing plant has an occupational disease - chronic dust bronchitis. The investigation
https://translate.yandex.com/en/doc 166/540
22:20 ,27.6.2023 �� C : ; 5 B
of the case is carried out by a commission consisting of representatives of: the enterprise, the medical and sanitary unit,
the territorial SES, the branch of the Social Insurance Fund, and the trade union organization.
A representative of which institution should lead the work of the commission in accordance with the current " regulations on
investigation..."?
100% Territorial SES
0% Businesses
0% Social Insurance Fund
0% Trade union organization
0% Medical and sanitary unit

749. according to laboratory monitoring of the sanitary condition of the soil on the territory of the hospital, according
to the indicators of the sanitary number, the soil is slightly polluted, according to the titer of Escherichia coli -

173

Downloaded from the site - online testing step

polluted, according to the titer of anaerobes (Cl. perfringens) - slightly polluted. This indicates that:
100% Arrival of fresh fecal contamination
0% Insufficient intensity of humification processes in the soil
0% Ancient fecal contamination
0% Constant supply of organic protein contaminants
0% Insufficient insolation and aeration of the soil

750. A 28-year-old female patient underwent uterine curettage due to an incomplete


abortion. Blood loss of 900 ml. Blood transfusion started. After administration of 60 ml
of erythromass, pain in the lumbar region, fever appeared, as a result of which the blood transfusion was stopped.
After 20 minutes, the condition worsened dramatically: adynamia, sharp pallor of the skin, acrocia-noz, copious sweat,
prolonged fever, to-38.5 oC, Ps-110 / min, AT-70/40 mm Hg. what is the most likely diagnosis?
100% Blood transfusion shock
0% Hemorrhagic shock
0% Septic shock
0% Anaphylactic shock
0% DIC-syndrome

751. A 58-year-old patient complains of difficulty passing solid food through the esophagus,
"coughing while eating". Considers himself ill for four months. Objectively: in
the supraclavicular areas, one dense lymph node is up to 0.7 cm in diameter. What
is the most likely diagnosis in this case?
100% Esophageal cancer
0% Lymphogranulomatosis
0% Tracheitis
0% Esophagitis
0% Mediastinal tumor

752. an ambulance team delivered an unconscious patient picked up on


the street in winter. Objectively: pale, shallow breathing, bradycardia with heart rate - 54/min, to-35.0 oC.
AT-100/60 mm Hg during palpation of the abdomen and chest, there are no peritoneal signs. The smell
of alcohol from the mouth. What is the most likely diagnosis?
100% General cooling
0% Acute cardiovascular insufficiency
0% Clinical death
https://translate.yandex.com/en/doc 167/540
22:20 ,27.6.2023 �� C : ; 5 B

0% Frostbite of the trunk, limbs


0% -

753. A 68-year-old patient complains of fever up to 38.3 ° C, hematuria. ESR is 55


mm / hour. Antibacterial therapy is ineffective. What diagnosis is possible to assume?
100% Kidney cancer
0% Polycystic kidney disease
0% Amyloidosis of the kidneys
0% Urolithiasis

174

Downloaded from the site - online testing step

0% Chronic glomerulonephritis

754. A 52-year-old chronic smoker, while lifting a bag of potatoes, suddenly experienced
severe stabbing pain in the left side of the chest, coughing, severe shortness of breath, tachycardia,
and decreased blood pressure. On the ECG: signs of overload of the right heart. On
the chest X-ray, there is no pulmonary pattern in the left half of the chest
; the mediastinal organs are displaced to the right. What condition does this clinical picture correspond to?
100% Spontaneous pneumothorax
0% Acute myocardial infarction
0% Pulmonary embolism
0% Acute left-sided pneumonia
0% Exudative pleurisy

755. a patient who eats refined products for a long time complains of headache,
fatigue, depression, insomnia, irritability. Objectively: muscular asthenia, pain and cramps in
the tibia muscles, during which it steps on the heel, then on the outer edge of the foot. From the
cardiovascular system-tachycardia, hypoxia, dystrophic changes in the heart muscle. There
are violations of the gastrointestinal tract. What diagnosis can be made?
100% Hypovitaminosis B1
0% Hypovitaminosis B2
0% Hypovitaminosis B12
0% Hypovitaminosis B6
0% Hypovitaminosis B15

756. A 9-year-old boy has been suffering from bronchiectasis since the age of 3. Exacerbations
occur frequently, 3-4 times a year. After conservative therapy - short-term periods of remission.
The disease progresses, the child lags behind in physical development. The skin is pale, acrocyanosis, the nails
are deformed in the form of "watch glasses". Bronchography revealed
sac-like bronchiectasis of the lower lobe of the right lung. What should be the further treatment strategy?
100% Surgical treatment
0% Continue conservative therapy
0% Physical therapy treatment
0% Spa treatment
0% Tempering the child

757. A 46-year-old patient participated in the elimination of the consequences of an accident at a nuclear power plant. He is currently

https://translate.yandex.com/en/doc 168/540
22:20 ,27.6.2023 �� C : ; 5 B
undergoing inpatient treatment. Diagnosis: progressive vegetative insufficiency.
What100%
group of effects of ionizing radiation
Somato-stochastic methods can this disease be attributed to?
0% Somatic diseases
0% Genetic resources
0% Hormesis
0% Heterosis

175

Downloaded from the site - online testing step

758. A 3-year-old boy was discharged from the cardiology department, where he was treated for frequent
suffocating cyanotic attacks due to Fallot's tetrad. Which drug is most appropriate
to use for further prevention of seizures?
100% Obzidan
0% Kurantil
0% Relanium
0% Digoxin
0% Cordarone

759. the child is 4 years old, on the 5th day of illness. Complaints of cough, skin rash, to-38.2 oC, puffy
face, photophobia, conjunctivitis. Objectively: on the face, neck, upper half
of the chest there is a bright spotty-papular, sometimes discharge rash. The pharynx is hyperemic. From the nose-
serous-purulent discharge. In the lungs-dry wheezing. What is the most likely diagnosis?
100% Measles
0% Adenovirus infection
0% Scarlet fever
0% Rubella
0% Enterovirus exanthema

760. a patient has been worried for a month about pain in the right hypochondrium, bitterness in the mouth, weakness,
weight loss by 12 kg, periodic vomiting with bile admixture. Body temperature
in the evening 37.6 oC. In sonography: the gallbladder is 5. 5x2. 7 cm, its wall is 0.4 cm, the common bile
duct is 0.8 cm in diameter. In the anterior segment of the liver, there is a rounded hypoechoic
formation up to 5 cm in diameter, next to two more similar ones-1.5 cm each; the walls of the formations are up to 0.3 cm
thick. What is the most likely diagnosis?
100% Alveolar echinococcus of the liver
0% Liver cancer
0% Liver abscess
0% Cystic liver cancer
0% Paravesical liver abscesses

761. A 27-year-old woman complains of irregular periods with delays of up to 2-3 months,
significant weight gain, and hirsutism. Married 5 years, no pregnancies. During
the vaginal examination, the uterus is slightly smaller than normal, and dense, mobile
ovaries up to 4-5 cm in diameter are detected on both sides. What pathology can be considered in this case?
100% Sclerocystic ovary syndrome
0% Bilateral ovarian cysts
0% Chronic bilateral salpingitis
0% Tuberculosis of the uterine appendages

https://translate.yandex.com/en/doc 169/540
22:20 ,27.6.2023 �� C : ; 5 B
0% Hypomenstrual syndrome

762. A 46-year-old patient who is preparing for surgery for stomach cancer is undergoing
preoperative infusion therapy. Up to 3.0 liters of solutions were injected into the right ulnar vein. On
the next day, he felt a pulling pain in the area of his right shoulder. On examination: on the inner
surface of the shoulder there is a longitudinal band of hyperemia, swelling of the skin, painful weight. Which

176

Downloaded from the site - online testing step

did the patient develop a complication?


100% Acute thrombophlebitis
0% Venous puncture and paravenous edema
0% Necrosis of paravenous tissue
0% Acute lymphangoitis
0% Phlegmon of the paravenous fiber

763. a 35-year-old man worked as a bulldozer operator for 13 years. Complains of dizziness,
headache at the end of the working day, sore fingers and sore leg muscles at night. The
examination revealed a violation of pain and tactile sensitivity by peripheral type,
pain in the calf muscles. Pulsation on A. dorsalis pedis is preserved. What
is the most likely medical condition?
100% Vibration sickness
0% Raynaud's disease
0% Syringomyelia
0% Vegetative polyneuritis
0% Obliterating atherosclerosis

764. on the 4th day after suturing a perforated gastric ulcer, a patient suffering
from pulmonary emphysema developed spontaneous pneumothorax. The most advantageous place for drainage
of the pleural cavity to eliminate pneumothorax is:
100% Second intercostal space along the mid-key line
0% Eighth intercostal space along the posterior axial line
0% Seventh intercostal space along the anterior axial line
0% Sixth intercostal space along the anterior axillary line
0% Five intercostal spaces along the mid-beak line

765. The diagnosis of typhoid fever was made to a man in his 50s who works
as a locksmith and lives in an isolated apartment with all amenities. There are two adults in the family besides him
. What activities are carried out in relation to persons who communicated with the patient?
100% Bacteriological research
0% Antibiotic prophylaxis
0% Isolation
0% Dispensary supervision
0% Vaccination

766. a child was hospitalized with focal changes in the skin folds. On examination, the child
is restless, the skin is dry, with separate papular elements and areas of lichenification with
indistinct edges. A rash on the skin is accompanied by severe itching. Improvement of
the child's condition is noted in the summer months, deterioration - in winter. A child from 2 months on artificial

https://translate.yandex.com/en/doc 170/540
22:20 ,27.6.2023 �� C : ; 5 B
feeding, underwent exudative diathesis. My maternal grandmother suffers
from100%
bronchial asthma. What is the most likely diagnosis?
Atopic dermatitis
0% Contact dermatitis
0% Seborrheic eczema

177

Downloaded from the site - online testing step

0% Strofulus
0% Urticaria

767. in the mental status of a 32-year-old patient, there is a persistent pathological decline in mood. Objectively:
contact, briefly, but essentially answers questions. The speech output is extremely concise, in
slow motion. Movement is constrained and inhibited. Expresses ideas of self-blame and
self-deprecation. Which group of drugs are primarily indicated for the patient?
100% Antidepressants
0% Neuroleptics
0% Hippotics
0% Tranquilizers
0% Normotimics

768. a 52-year-old man complains of suffocation attacks, pain in the right side during breathing.
Got sick suddenly. From the medical history, it is known that for the last month he was treated for thrombophlebitis
of the right lower limb. In the emergency department, he suddenly lost consciousness, developed
suffocation and pain in his side. Objectively: HR-102 / min., HR-28 / min., AT-90/70 mm Hg. Auscultation:
accent of the second tone over the pulmonary artery, gallop rhythm, over the lungs on the right side under the scapula
fine bubbly wheezing, pleural friction noise. Which study is most informative for
making a diagnosis?
100% Angiography of the lung vessels
0% Echocardioscopy
0% Studying the function of external respiration
0% ECG
0% Coagulogram

769. A first-time patient at 20 weeks complains of pain in the lower abdomen, spotting
spotting from the genital tract. The uterus is in high tone, feels the movement of the fetus. In
a bimanual study: the uterus is enlarged according to the gestational age, the cervix
is reduced to 0.5 cm, the external peephole is opened by 2 cm. The discharge is bloody, smearing. What
is the most likely diagnosis?
100% Abortion that started
0% Threat of termination of pregnancy
0% Abortion is in progress
0% Incomplete abortion
0% Frozen pregnancy

770. A 41-year-old woman has been suffering from chronic cholecystitis for 8 years. Worries about almost
constant monotonous aching pain or a feeling of heaviness in the right hypochondrium, bitterness in the mouth
in the morning, constipation. During palpation of the abdomen, there is a slight soreness at the point of projection
of the gallbladder. The volume of the bladder after a choleretic breakfast decreased only by 15% (
according to ultrasound). What medications are most appropriate to prescribe?
100% Cholekinetics

https://translate.yandex.com/en/doc 171/540
22:20 ,27.6.2023 �� C : ; 5 B
0% Peripheral m-cholinolytics
0% Myotropic antispasmodics

178

Downloaded from the site - online testing step

0% Non-narcotic analgesics
0% Choleretics

771. A 39-year-old train conductor was hospitalized on the 4th day of his illness with complaints of
headache, weakness, dizziness, sweating, insomnia, and fever. Objectively: the face
is hyperemic, edematous, conjunctivitis. On the transitional fold of the conjunctiva-single petechiae.
On the skin of the trunk, chest, abdomen, and extremities, there is a profuse roseolous-petechial rash.
Tachycardia. AT-100/60 mm Hg, tongue tremor is noted. The liver and spleen are palpated.
The chair is detained. What is the most likely diagnosis?
100% Typhus fever
0% Typhoid fever
0% Flu
0% Meningococcemia
0% Leptospirosis

772. on the 4th day after suffering a cold, a patient was hospitalized with complaints of coughing with
isolated spitting of mucosal sputum. On day 2
, about 250 ml of purulent sputum with streaks of blood was released once. Objectively: the condition is of moderate severity.
PDR-28-30 / min., Ps-96/Xb, AT-110/70 mm Hg. respiration over the left lung is vesicular, over
the right - weakened, wet wheezing of different calibres over the lower lobe and amphoreal breathing
near the angle of the scapula. What is the most likely diagnosis?
100% Acute lung abscess
0% Exudative pleurisy
0% Acute focal pneumonia
0% Pleural empyema
0% Pyopneumothorax

773. in the current year, 10% of the employees of the institution were not ill once, 30%
once, 15% twice, 5% four times, and all others 5 or more times. How many employees will
be assigned to the I-th health group?
100% 55%
0% 10%
0% 40%
0% 60%
0% 22%

774.A patient working as a geologist in


South-East Asia was taken to the department with suspected meningitis. The disease started very acutely, developed rapidly and was
accompanied
by hyperthermia, headache, vomiting. Objectively: on the right side of the groin area
, a tambourine was found enlarged to 3x5 cm, very painful, with indistinct contours. The skin above it
is purple,
100%glistening. Meningeal signs are negative. What is the most likely diagnosis?
The plague
0% Acute leukemia
0% Tularemia
0% Anthrax

https://translate.yandex.com/en/doc 172/540
22:20 ,27.6.2023 �� C : ; 5 B

179

Downloaded from the site - online testing step

0% Benign lymphoreticulosis (felinosis)

775. A 16-year - old boy was hospitalized with complaints of uncut nosebleeds and
unbearable pain in his right elbow joint. Objectively: the affected joint
is enlarged, defigured, hyperemia of the skin above it. There are manifestations of arthropathy of other
joints. Ps-90 / min. In the blood: Er. -3.9 • 1012/l, HB-130 g/l, CP-1.0, leuc. -5.6 • 109/l, tr.- 220•
109/ l, ESR - 6 mm / hour. Lee-White blood clotting time: Start-24 minutes, end-27 minutes
and 10 seconds. Which drug is most effective in treating this patient?
100% Cryoprecipitate
0% Calcium Chloride
0% Erythromasa
0% Aminocislaproic acid
0% Vikasol

776. A 42-year-old woman complains of bruising on her legs and prolonged menstruation, general
weakness, and noise in her head. Objectively: a large number of spotty hemorrhages on the legs and torso.
Tachypnea, tachycardia, systolic murmur at all points. AT-75/50 mm Hg in the blood: Er.-1,9-1012/ l,
HL-60 g / l, CP-0,9, lek. - 6, 5 -109 / l, tr. - 20 -109/l, SHOE-12 mm / year. The duration of
Duka bleeding is 12 minutes. In the bone marrow-a large number of young immature forms
of megakaryoblasts without signs of platelet exfoliation. What is the most likely diagnosis?
100% Ideopathic thrombocytopenic purpura
0% Hemophilia A
0% Willebrandt's disease
0% Acute megakaryoblastic leukemia
0% Hemophilia B

777. A 43-year-old man who had contact with leaded gasoline was admitted to the clinic
complaining of general weakness, dizziness, memory impairment, daytime drowsiness and
insomnia at night, a feeling of "hair" in his mouth, and colic pain in the right hypochondrium. what is the most
likely diagnosis?
100% Chronic tetraethyl lead intoxication
0% Alcoholic delirium
0% Chronic mercury intoxication
0% Khroshchna shtoksikashcha manganese
0% Hroshchna shtoksikashcha lead

778. a 30-year - old woman with influenza had fasting blood glucose of 11.3 mmol/l
and glucosuria of 25 g/l. Height 168 cm, Weight 67 kg. Which study is most informative for
clarifying the diagnosis?
100% Fasting insulinemia
0% Daily fluctuations in glycemia
0% Daily fluctuations in glucosuria
0% Glycemia an hour after eating
0% Glucose tolerance Test
779. a 52-year-old man has been suffering from diabetes mellitus for 18 years. I had cystitis a year ago.

180

https://translate.yandex.com/en/doc 173/540
22:20 ,27.6.2023 �� C : ; 5 B

Downloaded from the site - online testing step

Takes Maninil 0.005 -3 times a day. Objectively: height-176 cm, Weight-82 kg. Fluctuations
in fasting blood glucose are 10.3-12.4 mmol/l. detected proteinuria - 0.033 g/l. To prevent
the progression of diabetic nephropathy, it is most appropriate:
100% Replace Maninil with insulin
0% Increase the dose of maninil
0% Reduce your daily food intake
0% Supplement your insulin therapy
0% Prescribe antibacterial therapy

780. a 34-year-old woman complains of weakness, weight loss of 12 kg in six months, sweating,
palpitations, irritability. Objectively: the thyroid gland of the third degree, elastic, against the background
of diffuse enlargement in the right lobe node. The cervical lymph nodes are not enlarged. What
is the most appropriate treatment strategy?
100% Surgery after antithyroid therapy
0% Administration of radioactive iodine
0% Immediate surgical intervention
0% Conservative antithyroid therapy
0% Immediate telegamatherapy

781.A 46-year-old woman developed an increased headache,


dizziness, and nausea one day after the onset of the flu. Objectively: consciousness is preserved, psychomotor agitation, general
hyperesthesia, moderate meningeal syndrome, nystagmus. Tendon reflexes on the right are higher,
reduced muscle strength in the right extremities, on the right is the pathological Babinsky reflex. CSF:
transparent, pressure 220 mmHg, moderate cytoz-46.3 * 106/l, mainly lymphocytes. Which
diagnosis is most likely?
100% Influenza meningoencephalitis
0% Bacterial meningoencephalitis
0% Subarachnoid hemorrhage
0% Parenchymal-subarachnoid hemorrhage
0% Ischemic stroke

782. A 19-year-old woman complains of abdominal and joint pain and demands more injections
of painkillers and sleeping pills. The examination revealed no pathology of the gynecological and
urological spheres. Objectively: there are traces of old punctures along the superficial veins of the extremities,
the origin of which cannot be explained. The tendon reflexes of the upper and lower extremities are the same,
lively. Photoreaction of the pupils is weak. The tongue is covered with a gray coating. In conversation - affectively
unbalanced. Diarrhea without pathological inclusions is noted. What is the next strategy for
the patient?
100% Consultation with a narcologist
0% Prescribe the medicine that the patient requires
0% Additional surgical advice
0% Prescribe antibiotic therapy
0% Consultation with an infectious disease specialist

783. A 46-year-old female patient was diagnosed with T2N2M0 left breast cancer,

181

https://translate.yandex.com/en/doc 174/540
22:20 ,27.6.2023 �� C : ; 5 B

Downloaded from the site - online testing step

cl. gr. II-A. what is the treatment plan for this patient?
100% Radiation therapy + Surgery + Chemotherapy
0% Operation only
0% Surgery + radiation therapy
0% Radiation therapy only
0% Chemo only

784. A 34-year-old man was taken to the neurological department with complaints of intense
headache, double vision when looking straight, unbearable light and noise. He became acutely ill,
and the attack occurred while lifting the cargo. Objectively: stun, moderate divergent strabismus,
diplopia. Kernig symptoms on both sides. There are no paresis. The cerebrospinal fluid is bloody. Which drug
should be prescribed first?
100% Epsilon-Hydrocaproic acid
0% Acetylsalicylic Acid
0% Heparin
0% Nicotinic acid
0% Glutamic Acid

785. a mother complains of shortness of breath and cough in a child aged 2 years. During the newborn period, the
girl has a rough systolic murmur in the heart. Objectively: the skin is pale,
the heart borders shift to the left side and up, rough systolic-diastolic murmur in the II intercostal space on the left side, which
is conducted to the top of the heart, to the cervical vessels, aorta, interscapular space, strengthening of the II
tone on the pulmonary artery. Above the lungs, hard breathing, various wet wheezes. Liver
+ 3 cm. What is the preliminary diagnosis?
100% Open Ductus arteriosus
0% Pulmonary artery stenosis
0% Atrial septal defect
0% Pulmonary hypertension
0% Bilateral bronchopneumonia

786. a patient with post-traumatic acute pain became significantly better after morphine administration.
Which of the listed mechanisms of action provided the anti-shock effect of morphine in this patient?
100% Opiate receptor stimulation
0% Central cholinergic receptor blockade
0% Activation of benzodiazepine receptors
0% Suppression of dopamine mediation
0% Increased GABA-ergic responses

787.A 38-year-old woman developed muscle pain,


fever up to 39 ° C, headache, dysuria, and a positive Pasternatsky symptom after hypothermia. In the urine:
leukocyturia, bacteriuria. In the blood: a decrease in hemoglobin to 103 g / l, leukocytosis with a shift
to the left, an acceleration of ESR to 32 mm / h. Blood urea - 6.0 mmol / l. what is the most likely
diagnosis?
100% Acute pyelonephritis
0% Tuberculosis of the kidneys

182

Downloaded from the site - online testing step

https://translate.yandex.com/en/doc 175/540
22:20 ,27.6.2023 �� C : ; 5 B
0% Acute glomerulonephritis
0% Urolithiasis
0% Acute cystitis

788. a patient with duodenal ulcer disease complained of weakness, shortness


of breath with little physical exertion, and a desire to eat chalk. Objectively: pale, trophic
skin changes. In the blood: Hb-90 g / l, Er -3.6 * 1012/l, CP-0.75, retic.- 2%, blood iron - 5.6
mmol/l. what complication did the patient develop?
100% Iron deficiency anemia
0% Hemolytic anemia
0% Erythremia
0% Aplastic anemia
0% B12-folate deficiency anemia

789. A 50-year-old female patient complains of right hypochondrium pain attacks that
occur mainly after eating fatty foods for a year. For the last week, the attacks were repeated
daily and became more painful. On the 3rd day of hospital stay, jaundice
of the sclera and skin appeared, feces discolored, and urine was dark in color. In the blood:
neutrophilic leukocytosis -13.1 * 109/l, ESR-28 mm/h. What is the most likely diagnosis?
100% Chronic calculous cholecystitis
0% Chronic pancreatitis, recurrent form
0% Fatty liver disease
0% Acute chronic cholangitis
0% Hypertensive dyskinesia of the gallbladder

790. a 72-year-old patient complains of itching of the skin of the left shin, especially around the trophic
ulcer, where the skin is reddened and swollen, sometimes there are wet spots, isolated yellowish
crusts. The boundaries of the lesion are clear. What is the most likely diagnosis?
100% Microbial eczema
0% Allergic dermatitis
0% Seborrheic eczema
0% Tuberculosis of the skin
0% Streptoderma

791. a 36-year-old patient complains of a skin rash that appeared a week ago and
does not subjectively bother him. Objectively: on the skin of the palms and soles - multiple lenticular
disseminated papules that do not rise above the skin level, stagnant red color,
covered with a cluster of horny scales, dense during palpation. What is the preliminary diagnosis?
100% Secondary syphilis
0% Multiple warts
0% Palmar-plantar psoriasis
0% Rubrophytosis of the palms and soles
0% Frozen palms and soles

183

Downloaded from the site - online testing step

792. A 30-year-old female patient had a body temperature of up to 38.5 ° C, pain in small
joints, as well as edema and erythema of the face. In the blood: Er. -2.6 • 1012/l; Hb-98 g/l; leukemia.- 2 •

https://translate.yandex.com/en/doc 176/540
22:20 ,27.6.2023 �� C : ; 5 B
109/l; ESR-58 mm / hour. In the urine: protein-3.1 g / l; Er. - 10-15 in p / S. what kind of disease can
you 100%
think aboutSystem
in this case?
red shepherd dog
0% Sepsis
0% Systemic scleroderma
0% Nodular periarteritis
0% Acute glomerulonephritis

793. A 5th grade student complained about a common skin rash


accompanied by severe itching, especially at night. Objectively: in the area
of the interdigital folds of both hands, the flexor surface of the wrist joints, on
the skin of the abdomen and buttocks,and on the inner surface of the thighs, small red papules
were found that tend to be arranged in pairs. In the center of individual papules, vesicles or
serous-hemorrhagic crusts are visible. Multiple excoriations are noted. What is the most likely
diagnosis?
100% Scabies
0% Dermatitis
0% Mycosis of smooth skin
0% Toxicoderma
0% Eczema

794. A 2-year-old child has frequent, long-term respiratory diseases and


a pancreatogenic form of malabsorption. Cystic fibrosis was suspected. What research
should be done to confirm this diagnosis?
100% Chloride content in sweat fluid
0% Bronchoscopy
0% Immunogram
0% Chest X-ray
0% Karyotyping

795. for a clinically healthy child of 5 months who is on natural feeding,


it is necessary to prescribe the first bait. Which dish is most appropriate to recommend in
this case?
100% Vegetable puree
0% 5% manna porridge
0% Cookie
0% Barley porridge
0% Grated apple

796. A 37-year-old patient with a large crushed wound in his left thigh was admitted to the surgical department 4 hours after the injury
. What is the main condition for successful prevention of gas
gangrene in the victim?
100% Removal of non-viable tissues and timely surgical treatment of the wound

184

Downloaded from the site - online testing step

0% Introduction of specific serum 3,000 UNITS


0% Introduction of specific serum 30 000 UNITS
0% Infiltration of soft tissues around the wound with an antibiotic solution
0% Washing the wound with a solution of hydrogen peroxide 6%

https://translate.yandex.com/en/doc 177/540
22:20 ,27.6.2023 �� C : ; 5 B

797. A 35-year-old patient was admitted to a psychiatric hospital for the first time. The diagnosis was established:
schizophrenia, paranoid form. What treatment method is appropriate to apply?
100% Neuroleptic therapy
0% Vitamin Therapy
0% Physical therapy
0% Psychotherapy
0% Reflexology

798. a patient with acute large-sided myocardial infarction was complicated by cardiogenic shock.
The latter occurs with low total peripheral resistance and reduced cardiac
output. What antihypertensive medication should be administered to the patient first?
100% Dopamine
0% Norepinephrine
0% Adrenaline rush
0% Mezaton
0% Prednisone

799. a 3-month-old child with signs of rickets has symptoms of Khvostek, Trusso, Maslov.
A day ago, the parents observed an attack of cyanosis, evisceration, the child was not breathing, covered
with sticky sweat. After one minute, there was a loud breath, after which the condition returned to normal. What
is the basis for the occurrence of the above symptoms?
100% Reduced blood calcium levels
0% Increased blood calcium levels
0% Reduced blood phosphorus levels
0% Increased blood phosphorus levels
0% Metabolic acidosis

800. A patient with an injury to the lower third of the volar surface
of the forearm came to the emergency room. Objectively: the flexion of the IV and V fingers is disturbed, the sensitivity of the inner
back and palmar surfaces of the hand and IV finger is reduced. What nerve is injured?
100% Ulnar
0% Radiation therapy
0% Sredinny
0% Musculoskeletal
0% Axillary

801. a patient was taken to the emergency department of the hospital 40 minutes after a minor
injury with complaints of pain in the thoracic spine. In the anamnesis: pulmonary tuberculosis.
Radiography: suspected compression fracture of the VIII and their thoracic vertebrae. Which

185

Downloaded from the site - online testing step

a pathological process in the spine could contribute to a pathological fracture of the vertebral bodies?
100% Tuberculosis spondylitis
0% Syphilis
0% Metastasis of a malignant tumor
0% Osteomyelitis
0% Traumatic fracture

https://translate.yandex.com/en/doc 178/540
22:20 ,27.6.2023 �� C : ; 5 B

802. A 44-year-old patient developed emphysema


in both supraclavicular areas, pain during swallowing and behind the sternum, and cyanosis the next day after esophagoscopy and biopsy.
The temperature is 39oc.
In the blood: lake.-16 * 109 / l, shift of the leukocyte formula to the left. The radiograph shows a
limited darkening of the posterior mediastinum at the level of tracheal bifurcation. The following diagnosis was made:
rupture of the esophagus.
100% Operative:What treatment
mediastinal strategygastrostomy
drainage, is most appropriate to choose?
application
0% Observation
0% Conservative antibacterial treatment
0% Operative radical with suturing of the esophageal wound
0% Operative: gastrostomy insertion

803. a 30-year-old woman who has been suffering from chronic pyelonephritis for a long time complains of significant
weakness, drowsiness, and a decrease in diuresis to 100 ml / day. AT-200/120 mm Hgin the blood: creatinine
-0.62 mmol / l, hypoproteinemia, albumin-32 g / l, potassium-6.8 mmol/l, hypochromic anemia,
elevated ESR. What is the first priority in the patient's treatment?
100% Hemodialysis
0% Antibacterial therapy
0% Enterosorbtion
0% Hemosorption
0% Blood transfusion

804. A 10-year-old girl went to the doctor complaining of thirst, frequent urination,
and weight loss. Considers himself ill for about a month. Objectively: no pathology of internal organs was
detected. What laboratory examination should be performed first?
100% Fasting blood sugar test
0% Urinalysis for sugar from daily diuresis
0% Urine test for acetone
0% Glucose tolerance test
0% Glucosuric profile

805. the child is 10 months old. Complaints of to-39.6 oC, loose stools. On day 5: to-38.7 oC, lethargic,
pale. The tongue is moist, covered with a white coating. Heart tones are muted. The abdomen is moderately
swollen, rumbling in the navel area, segments of the small and large intestines are sensitive during
palpation. Bowel movements are liquid, 8 times, dark green in color, smelly, with an admixture of mucus.
What is the preliminary diagnosis?
100% Salmonellosis
0% Dysentery
0% Yersiniosis

186

Downloaded from the site - online testing step

0% Escherichiosis
0% Rotavirus infection

806. A 19-year-old patient complains of a skin rash that appeared 2 days ago after
eating smoked fish. The rash disappears after 4-6 hours, but then reappears.
Accompanied by itching. Objectively: the torso and upper limbs have numerous
blisters ranging in size from peas to beans, pink in color. What is the most likely
diagnosis?
100% Acute urticaria
https://translate.yandex.com/en/doc 179/540
22:20 ,27.6.2023 �� C : ; 5 B

0% Allergic dermatitis
0% Angioedema
0% Toxicoderma
0% Pochesukha

807. A 40-year-old female patient reports heavy menstruation throughout the year, which is accompanied
by cramping pain in the lower abdomen. During bimanual examination during menstruation: in
the cervical canal, a formation with a diameter of up to 5 cm and a dense consistency is determined. The uterus
is enlarged up to 5-6 weeks of pregnancy, normal consistency, mobile, painful. Appendages are not
defined. The discharge is bloody and copious. What diagnosis can be assumed?
100% Submucous fibromatous nodule that is born
0% Abortion is in progress
0% Cervical cancer
0% Cervical fibroids
0% Algodismenorrhea

808. A 65-year-old patient complains of pain in the lumbar spine, moderate dysuria.
He's been ill for about six months. Prostate volume - 45 cm3 (there are hypoechoic nodes in both lobes,
capsule invasion). Prostate-specific antigen-60 ng / ml. According to the prostate biopsy-
adenocarcinoma. Which of the additional research methods will allow us to determine the stage
of the tumor process in this patient?
100% Computed tomography of the pelvic organs
0% Radiography of the lumbar spine
0% Excretory urography
0% Chipping bones
0% Chest radiography

809. A 30-year-old female patient who has been suffering from infertility for 10 years complains of
heavy, prolonged menstruation accompanied by cramping pain. In a bimanual
study: the uterus is enlarged according to 8 weeks of pregnancy, appendices are not determined. When
probing the uterine cavity, its deformation is determined. What is the most likely diagnosis?
100% Submucous uterine fibroids
0% Chronic endometritis
0% Metrorrhagia
0% Algodismenorrhea
0% Uterine pregnancy

187

Downloaded from the site - online testing step

810. A 44-year-old worker who has been working in coke production for more than 16 years
has a dust concentration 5-10 times higher than the maximum permissible concentration in the workplace. X-ray
examination of the lungs revealed changes characteristic of pneumoconiosis. What type of pneumoconiosis
is most likely in this case?
100% Anthracosis
0% Anthracosilicosis
0% Silicatosis
0% Asbestosis
0% Siderose

https://translate.yandex.com/en/doc 180/540
22:20 ,27.6.2023 �� C : ; 5 B
811. When studying the morbidity rate in city N, it was found that the population of different
administrative districts differs in their age composition. Which of the statistical methods
allows you to exclude
100% the influence of this factor on morbidity rates:
Standardization
0% Wilcoxon's T-test
0% Correlation and regression analysis
0% Dynamic series analysis
0% Calculating average values

812. the driver repaired the car in a closed garage, after which he complained of
headache, dizziness, nausea, muscle weakness, drowsiness. Objectively: increased
heart rate, respiration, agitation, hypertension, delusions of persecution. What is the most likely
diagnosis?
100% Carbon monoxide intoxication
0% Leaded gasoline intoxication
0% Post-traumatic encephalopathy
0% Hypertensive crisis
0% Astheno-vegetative syndrome

813. the total area of the therapeutic ward is 28 m2. What is the maximum
number of beds available in this ward?
100% 4
0% 1
0% 2
0% 3
0% 5

814. A 9-year-old child has been treated for chronic glomerulonephritis for 4 years.
He is registered in a dispensary. What examination should be performed to study
glomerular filtration?
100% Endogenous creatinine clearance
0% Zimnitsky's Test
0% Daily proteinuria
0% General urinalysis

188

Downloaded from the site - online testing step

0% Residual nitrogen and urea levels in the blood

815. A 47-year-old patient complains of fever, general weakness, diffuse


dull abdominal pain, which increases before defecation and subsides after it. Bowel movements up
to 10 times a day, contain blood. Has been ill for 2 months. Objectively: the skin is pale and dry.
Palpation of the colon is painful. The cecum is palpated in the form of a string. Which examination
is most informative for making a diagnosis?
100% Colonoscopy
0% Rectoromanoscopy
0% Irrigoscopy
0% Fibrogastroscopy
0% Overview X-ray examination of the abdominal organs

https://translate.yandex.com/en/doc 181/540
22:20 ,27.6.2023 �� C : ; 5 B

816. A 27-year-old serviceman was admitted to the Ministry of Emergency Situations 1.5 hours after being wounded in
the stomach by an aerial bomb fragment. Objectively: the condition is severe, pale, Ps-132 / min, AT-80/60 mm Hg. in
the area of the hypochondrium, a PPI bandage is applied, which is soaked in blood.my stomach is tight.
Percussion: dullness is detected in sloping areas, which is moved when
the body position changes. Symptoms of peritoneal irritation are determined. In which department should
assistance be provided at the Ministry of Emergency Situations?
100% In the dressing room
0% In the reception and sorting tent
0% In the isolation ward
0% In the evacuation room
0% Through Self-Help

817. The sergeant was wounded by a shell fragment in the area of the left hypochondrium. on the battlefield, in
the order of mutual assistance, he was put on a bandage with an individual package. Delivered to the Ministry of Internal Affairs.
Complains of dizziness, weakness, thirst, abdominal pain. General condition
is severe, pale, Ps-120 / min. The abdomen is soft and painful on palpation. The bandage is well fixed,
but slightly soaked in blood. what type of transport and in what turn should
the victim be evacuated to OmedB?
100% By ambulance in the 1st stage
0% By passing vehicles in the 1st stage
0% By ambulance to the 2nd stage
0% By passing vehicles in the 2nd stage
0% By passing vehicles in the 3rd stage

818. a 24-year-old woman in labor has a sudden increase in body temperature on the 5th day after giving birth.
Complains of weakness, headache, lower abdominal pain, irritability. Objectively:
AT120 / 70 mmHg, Ps-92 / min, to-38.7 oC. Bimanually: the uterus is enlarged up to 12 weeks of pregnancy,
dense, somewhat painful on palpation, the cervical canal passes 2 transverse fingers,
discharge is moderate, cloudy, with an unpleasant smell. In the blood: left-shifted leukocytosis,
lymphopenia, ESR - 30 mm / h. What is the most likely diagnosis?
100% Endometritis
0% Parametrite

189

Downloaded from the site - online testing step

0% Pelvioperitonitis
0% Metroflebitis
0% Lochiometer

819. an 11-year-old girl became acutely ill: there was pain in the lumbar region, nausea,
vomiting, frequent urination, temperature 39oc. Objectively: the abdomen is soft, painful
on palpation in the lumbar region. In the general analysis of urine, significant leukocyturia, bacteriuria.
E. coli is sown from the urine. What is the most likely diagnosis?
100% Acute pyelonephritis
0% Acute appendicitis
0% Chronic glomerulonephritis
0% Acute vulvovaginitis
0% Acute glomerulonephritis

https://translate.yandex.com/en/doc 182/540
22:20 ,27.6.2023 �� C : ; 5 B
820. the patient is active, mobile, and fussy. Counteracts the survey. He speaks quickly
and loudly, and his statements are spontaneous and inconsistent. Determine the psychopathological state:
100% Psychomotor agitation
0% Catatonic arousal
0% Delirium
0% Behavioral disorder
0% Paranoid syndrome

821.An 18-year-old patient developed pain and swelling in


the ankle joints and diffuse abdominal pain that worsens after eating,
periodic liquid hemorrhagic diarrhea,and papular-hemorrhagic rash on the lower extremities 2 weeks after acute respiratory
infections. In the blood: Er.- 3,0 • 1012/ l, Hb-95 g / l, ESR-40 mm / h, platelets are normal. What
is the most likely diagnosis?
100% Hemorrhagic vasculitis
0% Werlhof's disease
0% System red shepherd dog
0% Infectious endocarditis
0% Rheumatoid arthritis

822. a 37-year-old patient was taken to the department in an unconscious state. Got a closed
injury in the abdominal area. Suspected intra-abdominal bleeding, damage
to internal organs. They are preparing for an urgent surgical intervention. The doctor determined the
blood type using standard sera and obtained the following result: 0 (1) - no
agglutination, A(II) - no agglutination, B (III) - no agglutination. What is the
victim's blood type?
100% 0 (s)
0% A(II)
0% B (III)
0% LV (1 V)
0% -
823. A 36-year-old patient was hospitalized in the surgical department after an accident some time ago. Condition

190

Downloaded from the site - online testing step

progressively worsens: respiratory failure increases, cardiac disorders are noted


. On the basis of clinical and radiological data, displacement of mediastinal organs was revealed.
What process caused this complication?
100% Valvular pneumothorax
0% Open pneumothorax
0% Closed pneumothorax
0% Subcutaneous emphysema
0% Mediastinit

824. A pervovagit 18-year-old woman underwent surgery for acute


phlegmonous appendicitis at 27-28 weeks of pregnancy. What complications should be prevented in
the postoperative period?
100% Miscarriage of pregnancy
0% Intestinal obstruction
0% Fetal hypotrophy
0% Premature placental abruption
0% Late preeclampsia
https://translate.yandex.com/en/doc 183/540
22:20 ,27.6.2023 �� C : ; 5 B

825. a 30-year-old patient has sudden syncope, tonic-clonic convulsions, blood pressure-150/100 mm
Hg, heart rate-100/min. Before the attack, he experienced dizziness,a sense of unreality of reality,
an inexplicable feeling of fear, and does not remember the attack of convulsions. What is the most likely diagnosis?
100% Epilepsy
0% Morgagni-Adams-Stokes syndrome
0% Vestibular thinking
0% Transient ischemic attack
0% Hyperventilation ischemic attack

826. a 26-year-old man complains of stabbing pain when breathing, coughing, shortness of breath.
Objectively: to-37.3 oC, HR-19 / min, HR = Ps-92 / min; AT-120/80 mm Hg. vesicular respiration.
On the left,in the lower-lateral parts of the chest, during the inhalation and exhalation phase, a noise is heard,
which increases with pressure with a phonendoscope and persists after coughing. ECG - no
pathological changes. What is the most likely diagnosis?
100% Acute pleurisy
0% Intercostal neuralgia
0% Subcutaneous emphysema
0% Spontaneous pneumothorax
0% Dry pericarditis

827. The adult diet contains 30


g of protein, 37 g of fat, 137 g of carbohydrates, 15 mg of vitamin C, and 0.6 mg of thiamine (vitamin B) in terms of megacalories (1000
kcal = 4184 kJ). The diet is not
balanced
100%in content:
Vitamin C
0% Proteins
0% Fat content

191

Downloaded from the site - online testing step

0% Carbohydrates
0% Thiamine

828. a 22-year-old patient is an office worker. Open all day in an air-conditioned room.
In the summer, he became acutely ill: high fever, shortness of breath, dry cough,pleural pain,myalgia,
arthralgia. Objectively: wet wheezing on the right, pleural friction noise. Radiologically
, infiltration of the lower lobe persists for a long time. In the blood: lake.-11 * 109/l, p. - 6%, c. - 70%, lymph. - 8%,
ESR-42 mm / h. What is the etiological factor of pneumonia?
100% Legionella
0% Mycoplasma
0% Streptococcus
0% Staphylococcus aureus
0% Pneumococcus

829. a 53-year-old woman complains of pain and heart failure. She has been ill since childhood, and her father
suffered from an arrhythmia of the heart. Objectively: the condition is severe, Ps-220 / min, AT-80/60 mm Hg. ECG:
HR215 / min, expansion and deformation of the QRS complex with the presence of atrioventricular dissociation;
P wave is positive. In the future, the heart rate decreased to 45 / min, complete

https://translate.yandex.com/en/doc 184/540
22:20 ,27.6.2023 �� C : ; 5 B
separation of the P wave and the QRST complex. Which of the above is most effective in
100%
treating Artificial pacemaker implantation
the patient?
0% beta-blockers
0% Holinolytics
0% Calcium antagonists
0% Cardiac Glycosides

830. During the foggy, overcast December days


, the number of diseases with respiratory tract damage and signs of general intoxication increased in the area of the solid-fuel CHPP
. There were deaths among older residents. What is the most
likely factor that caused the toxic effect?
100% Suspended substances
0% High air humidity
0% Calm
0% Reduced air temperature
0% Temperature gradient

831. On the tenth day after discharge from the maternity ward, a 20-year
-old female patient complained of an increase in body temperature up to 39 ° C and pain in the right breast. Objectively:
the mammary gland is enlarged,a site of hyperemia is noted in the upper outer quadrant
,a seal with indistinct contours is palpated there, lactostasis, and there is no fluctuation.
The lymph nodes in the right axillary fossa are enlarged and painful. What is the most likely diagnosis?
100% Lactation mastitis
0% Abscess
0% Erysipelas
0% Dermatitis

192

Downloaded from the site - online testing step

0% A tumor

832. A 47-year-old patient suffered a flame burn to his upper limbs and torso. During
hospitalization, the condition was severe, consciousness became dark, fever, blood pressure-80/50 mm Hg, Ps-118 / min.
Locally identified burns of III B degree with an area of 20%. What therapeutic actions should be taken?
100% Introduce narcotic analgesics and anti-shock blood substitutes
0% Perform primary surgical treatment of the burn wound
0% Prescribe detoxifying blood substitutes
0% Burn surface necrotomy, blood transfusion
0% Antibacterial and detoxification therapy

833. a 54-year-old female patient was hospitalized with severe acrocyanosis, swollen
cervical veins, enlarged liver, and ascites. The borders of the heart are expanded. Heart tones are not
heard, and the apical push is not detected. AT-100/50 mm Hg
chest X-ray revealed an enlarged shadow of the heart in the form of a trapezoid. What pathological
condition can explain these symptoms in the patient?
100% Cardiac tamponade
0% Exudative pleurisy
0% Combined heart disease
0% Acute heart failure
0% Hiatal hernia
https://translate.yandex.com/en/doc 185/540
22:20 ,27.6.2023 �� C : ; 5 B

834. A 16-year-old patient underwent surgery 12 years ago for diffuse toxic goiter of iIl-IV
degree. Currently, the patient has experienced a relapse of thyrotoxicosis. The patient
is offered surgery, but before that it is necessary to find out where the functioning gland tissue is located. What
research method should I use?
100% Breast scan
0% Ultrasound examination of the gland
0% Puncture aspiration biopsy of the gland
0% X-ray examination of the neck
0% X-ray examination of the esophagus

835. a 1.5-year-old child became acutely ill: the temperature rose to 39 ° C, vomiting occurred
up to 5 times. The study of the nervous system revealed positive symptoms of Kernig, Brudzinsky.
These symptoms include::
100% Meningeal signs
0% Discoordination syndrome
0% Movement disorders syndrome
0% Encephalic syndrome
0% Signs of infectious toxicosis

836. after a conversation with the mother of a seven-month-old boy who is on natural
feeding, the pediatrician found out that the child is fed 7 times a day. What number of feedings
is set for a child of this age?

193

Downloaded from the site - online testing step

100% 5 times
0% 3 times
0% 4 times
0% 6 times
0% 7 times

837. a 19-year-old patient was injured a day ago due to a fall from a height of 1.5 meters - "sat
astride" on a wooden beam. Urinary excretion is very difficult, macrohematuria, urethrorrhagia. On
the perineum and scrotum - a hematoma. During urethrography, the contrast
agent was found to leak in the membranous part of the urethra. What should be the treatment strategy?
100% Operation epicystostomy
0% Catheterization of the bladder to remove urine
0% Bubbling of the urethra
0% Cold on the perineum, hemostatic therapy
0% Performing a primary urethral suture

838. A 10-month-old child was hospitalized in serious condition with expiratory dyspnea and
dry cough. Ill for a week-had a runny nose, temperature 38oC. Objectively:
there is percussive tympanitis above the lungs, auscultation-a lot of dry whistling and single wet wheezes on
both sides. What is the most likely diagnosis?
100% Acute obstructive bronchitis
0% Pneumonia
0% Bronchial asthma
https://translate.yandex.com/en/doc 186/540
22:20 ,27.6.2023 �� C : ; 5 B

0% Whooping cough
0% Acute bronchitis

839. the child is 5 months old, was born full-term, but in the first days there were symptoms
of hemolytic disease of newborns. I didn't get any vaccinations in the hospital.
Which vaccine should she be immunized with first?
100% BCG
0% Koreva
0% AKDP
0% ADP
0% ADP-M

840. Three days ago, a boy had a foreign body removed from under his nail plate. After 2 days
, there was a sharp throbbing pain at the end of the nail phalanx, especially when pressing, hyperemia
of the nail roller, body temperature rose to 37.5 oC, the color of the nail
plate changed. What is the most likely diagnosis?
100% Subungual panaritium
0% Erysipelas
0% Paronychia
0% Erysipeloid
0% Abscess

194

Downloaded from the site - online testing step

841. A 38-year-old man, falling from a height of two meters, hit his head on a metal
object and briefly lost consciousness. Complains of headache, nausea, dizziness,
discharge of drops of light liquid from the right nasal passage. Objectively: a symptom of "glasses",
retrograde amnesia. There are no focal neurological symptoms. What is the most likely
cause of nasal liquorrhea?
100% Skull base fracture
0% CSF hypertension
0% Violation of CSF outflow
0% Inflammation of the nasal mucosa
0% Brain contusion

842. The
number of elderly and senile patients in general medical practice has steadily increased in recent years. What type of pathology
should be expected to prevail in the morbidity structure of the population served?
100% Chronic pathology
0% Industrial pathology
0% Non-epidemic pathology
0% Acute pathology
0% Infectious pathology

843. A 30-year-old patient went to the doctor with complaints about the absence of menstruation for
2 years after giving birth, hair loss, and weight loss. The delivery was complicated by bleeding due to
uterine hypotension. Objectively: the patient is asthenic, the external genitalia are hypoplastic,
the uterine body is small, painless. Uterine appendages are not detected. What is the most
likely diagnosis?

https://translate.yandex.com/en/doc 187/540
22:20 ,27.6.2023 �� C : ; 5 B
100% Sheehan's syndrome
0% Ovarian amenorrhea
0% Shereshevsky-Turner syndrome
0% Ovarian depletion syndrome
0% Galactorrhea-amenorrhea syndrome

844. A 28-year-old female patient was taken to the gynecological department three days after an accidental sexual act
. Worries about pain in the lower abdomen and during urination, significant
purulent discharge from the vagina, an increase in body temperature up to 37.8 oC. The diagnosis was established: acute
two-stage adnexitis. An additional examination revealed: 4 degree of purity of the vaginal contents,
white blood cells cover the entire field of vision, diplococcus bacteria located intra-and
extracellularly. What is the etiology of acute adnexitis in the patient?
100% Gonorrhea
0% Coli-bacillary
0% Chlamydia
0% Trichomonas
0% Staphylococcal infection

845. the boy is 8 years old. The state of physical development corresponds to the age. From birth
, a noise is heard above the heart. Objectively: the skin and visible mucous membranes are normal in color. Ad - 100/70 mm

195

Downloaded from the site - online testing step

above the pulmonary artery, systolic-diastolic murmur and accent of the second tone are heard. On
the ECG-signs of overload of the left heart. During X-ray examination, there is an increase
in the pulmonary pattern, darkening of the heart of the usual shape. What is the most likely diagnosis?
100% Atrial septal defect
0% Pulmonary artery stenosis
0% Aortic coarctation
0% Fallot's Tetrad
0% Open Ductus arteriosus

846. A 45-year-old patient with urolithiasis has had an attack of renal colic.
What is the mechanism of an attack?
100% Violation of the outflow of urine from the kidney
0% Increased relative density of urine
0% Ureteral kinks
0% Damage to the renal glomeruli
0% Renal artery spasm

847. examination of a 25-year-old woman who applied to a antenatal clinic for


lack of pregnancy during 3 years of regular sexual activity revealed: increased
body weight, male-type pubic hair growth, increased hip
hair, dense, enlarged ovaries, monophasic basal temperature. What is the most likely diagnosis?
100% Sclerocystic ovaries
0% Inflammation of the uterine appendices
0% Adrenogenital syndrome
0% Premenstrual syndrome
0% Gonadal dysgenesis

https://translate.yandex.com/en/doc 188/540
22:20 ,27.6.2023 �� C : ; 5 B

848. a 65-year-old patient who has been suffering from deep vein thrombophlebitis of the lower
extremities for a long time suddenly developed shortness of breath, cough with sputum and blood impurities, pain
behind the sternum radiating to the right side. Ps-100 / Hv, AT-90/50 mm Hg, CVT-300 mm water.
what complication did the patient have?
100% Pulmonary embolism
0% Acute pericarditis
0% Acute pneumonia
0% Myocardial infarction
0% Acute cardiovascular insufficiency

849. a 49-year-old woman complains of headache, "hot flashes" to the head and neck,
increased sweating, palpitations, increased blood pressure up to 170/100 mm Hg,
irritability, insomnia, tearfulness, memory loss, rare scanty menstruation,
weight gain of 5 kg in the last six months. What is the most likely diagnosis?
100% Menopausal syndrome
0% Premenstrual syndrome
0% Vegetative vascular dystonia

196

Downloaded from the site - online testing step

0% Arterial hypertension
0% Post-castration syndrome

850. A 34-year-old patient complained of general weakness, swelling of the face and hands,
rapid fatigue when walking, difficulty swallowing, and heart failure. These symptoms
appeared 11 days after the sea holiday. Objectively: erythema of the face, swelling
of the lower leg muscles. Heart tones are muted, blood pressure is 100/70 mm Hg: activity of AsAT - 0.95
mmol / dl, ALT-1.3 mmol / h-l, aldolase-9.2 mmol/h-l, serum creatine phosphokinase-2.5
mmol P/h-l. which method of examination is the most specific?
100% Muscle biopsy
0% ECG
0% ECHO-KG
0% Electromyography
0% Determination of cortisol levels in blood and urine

851. a 33-year-old patient has acute blood loss (ep.- 2,2*1012/ Hb-55 g / l), blood type A (II) Rh+.
He was mistakenly transfused with donor erythromas AB (IV) Rh+. After an hour
, there were feelings of anxiety, pain in the lower back, stomach. Ps-134 / min, AT-100/65 mm Hg,
body temperature-38.6 oC. During catheterization of the bladder, 12 ml/h
of dark brown urine was obtained. What complication did the patient have?
100% Acute renal failure
0% Cardiac shock
0% Allergic reaction to donor erythromas
0% Citrate intoxication
0% Infectious and toxic shock

852. during the last two weeks, a 45-year-old man became very active, became
multilingual, euphoric, slept little, claimed that he could "save humanity", was able to solve
the problem of cancer and AIDS, and distributed money to strangers. What is the most likely diagnosis?

https://translate.yandex.com/en/doc 189/540
22:20 ,27.6.2023 �� C : ; 5 B
100% A manic attack
0% Panic disorder
0% Agitated depression
0% Schizoaffective disorder
0% Catatonic arousal

853. A patient with gastroesophageal reflux disease has been taking the drug "reducing acidity"periodically for 5 years, on the
recommendation of a pharmacist. The following
side effects occurred: osteoporosis, Muscle weakness, malaise. What drug has this side
effect?
100% Aluminum-containing antacid
0% Proton Pump Inhibitor
0% H2 blocker
0% Metoclopramide
0% Gastrotsepin
854. a 26-year-old patient complains of severe muscle weakness, dizziness, and excessive pain in the back.

197

Downloaded from the site - online testing step

abdominal pain, nausea, vomiting, which does not improve the patient's condition. The disease started gradually 6
months ago. General weakness increased, the skin darkened. The condition worsened sharply after
acute respiratory infections; abdominal pain and frequent vomiting occurred. Objectively: the skin is dry, diffusely pigmented.
Heart sounds are sharply weakened, heart rate-60 / min, AT-80/40 mm Hg. the abdomen is somewhat painful in
the epigastric region. In the blood: lake.- 8,1 * 109/ l, glucose-3.0 mmol / l. what is the most
likely diagnosis?
100% Chronic adrenal insufficiency. Addisonian Crisis
0% Acute pancreatitis
0% Infectious and toxic shock
0% Acute food poisoning
0% Acute cholecystitis

855. the patient complains of a strong pressing pain in the throat, more than a matter of fact, the inability
to swallow even liquid food. He's been ill for 5 days. Objectively: the condition is severe, to-38.9 oC, speech is difficult,
voice is compressed, mouth opens with difficulty. Submandibular lymph nodes on the right are painful, enlarged. What
is the most likely diagnosis?
100% Paratonsillar abscess
0% Diphtheria
0% Pharyngeal tumor
0% Simanovsky-Vincent's angina
0% Phlegmonous sore throat

856. A 10-year-old girl was admitted to the hospital with symptoms of carditis. In the anamnesis: two weeks ago
there was an exacerbation of chronic tonsillitis. What is the most likely etiological factor in this
case?
100% Streptococcus
0% Staphylococcus aureus
0% Pneumococcus
0% Klebsiela
0% Proteus

https://translate.yandex.com/en/doc 190/540
22:20 ,27.6.2023 �� C : ; 5 B
857. employees of the laboratory for the manufacture of measuring instruments (pressure gauges, thermometers
, etc.) complain of a metallic taste in the mouth, stomatitis, dyspeptic symptoms,sleep disorders,
unsteady walking, and a sharp decrease in heart activity. Poisoning with what substance caused
these symptoms?
100% Mercury
0% Lead
0% Manganese
0% Toluene
0% Tetraethyl lead

858. a full-term newborn has the following symptoms:: PDR-26 / min., heart rate-90 / min., skin is blue,
muscle tone is low, the child reacts with a grimace to the suction of mucus and amniotic fluid from the nose and mouth with a catheter
. Reflexes are reduced. Auscultation:
vesicular respiration is weakened over the lungs. The heart tones are sonorous. After 5 minutes - rhythmic breathing, 38/min, heart rate-

198

Downloaded from the site - online testing step

120 / min. What is the most likely diagnosis?


100% Asphyxia
0% Congenital pneumonia
0% Birth trauma
0% Bronchopulmonary dysplasia
0% Respiratory disease syndrome

859. examination of the placenta revealed a defect. A manual examination of the uterine cavity
and a fist massage of the uterus were performed. How to prevent endometritis in the postpartum period?
100% Antibacterial therapy
0% Instrumental revision of the uterine cavity
0% Hemostatic therapy
0% Reducing agents
0% Intrauterine instillation of Dioxidin

860. a 35-year-old man became ill for the first time. Complains of intense lower back pain
radiating to the groin area, external genitalia, thigh, frequent urination,
chills, nausea, vomiting. Objectively: a positive symptom of Pasternatsky. In the urine: red blood cells
and white blood cells cover the entire field of vision, increased protein content. What is the most likely pathology
that caused this clinical picture?
100% Urolithiasis, renal colic
0% Cholelithiasis, biliary colic
0% Kidney infarction
0% Intestinal obstruction
0% Osteochondrosis, acute radicular syndrome

861. a patient complained to a venereologist about painful urination, redness


of the external opening of the urethra, and profuse purulent discharge from the urethra. Considers himself ill for 3 days.
The disease is associated with casual sexual contact, about a week ago. If
the preliminary diagnosis of acute gonorrheal urethritis is confirmed, then a bacterioscopic
examination in the discharge from the urethra will be detected:
100% Gram-negative diplocs
0% Gram-positive diploid cells

https://translate.yandex.com/en/doc 191/540
22:20 ,27.6.2023 �� C : ; 5 B
0% Spirochete
0% Vulgar proteus
0% Mycoplasma

862. A 55-year-old man with stomach cancer underwent surgery 1.5 years ago and underwent
chemotherapy. Within 4 months, he notes heaviness in the right hypochondrium, nausea, bitter
taste in the mouth, sometimes vomiting with bile admixtures, subfibrility in the evening. Ultrasound revealed
multiple bulky formations in the liver. The gallbladder and pancreas are unchanged.
Preliminary diagnosis: metastatic liver damage. Determination of what substance
by radioimmunological analysis can confirm this diagnosis?
100% CA-19-9
0% β2 microglobulin

199

Downloaded from the site - online testing step

0% Prolactin
0% Gastrin
0% Insulin

863. A 28-year-old patient was taken by an ambulance team to the emergency department
in an unconscious state with generalized epileptic seizures that recur in
15-20 minutes. During transportation, the patient was twice injected with sibazone
and magnesia sulphate, but the level of consciousness was not restored. What department should I provide
emergency care in?
100% Intensive care unit
0% Neurological
0% Surgical treatment
0% Therapeutic use cases
0% Psychiatric services

864. on the second day after the preventive vaccination, a 2-year-old boy developed a
fever of up to 38 ° C and abdominal pain without a specific localization. On day 3, the child
developed a red papular-hemorrhagic rash on the extensor surfaces of the limbs and
around the joints. There is swelling and slight soreness of the knee joints. From
other organs and systems-without pathological changes. What is the most likely diagnosis?
100% Hemorrhagic vasculitis
0% Thrombocytopenic purpura
0% Meningococcemia
0% Urticaria
0% DIC-syndrome

865. A 60-year-old patient complains of suffocation, palpitations, and rapid fatigue.


He has been suffering from hypertension for 8 years. Objectively: the left border of the heart is shifted by 2 cm to the left of
the midclavicular line, the heart tones are rhythmic, weakened; above the aorta-the accent of tone II.
AT-170/100 mm Hg liver + 2 cm, pasty legs. On the ECG: deviation of the heart axis to the left,
hypertrophy of the left ventricle. EF-63%. Which variant of heart failure is observed in
this case?
100% Diastolic
0% Systolic
0% Mixed

https://translate.yandex.com/en/doc 192/540
22:20 ,27.6.2023 �� C : ; 5 B
0% Standard
0% Undefined

866. To assess the health status of the population,a report on diseases


registered in the population of the service area was compiled and analyzed (f. 12). What indicator is calculated
based on this report?
100% Overall morbidity rate
0% Indicator of pathological lesion
0% Morbidity rate with temporary disability
0% Rate of hospitalized morbidity

200

Downloaded from the site - online testing step

0% Indicator of the main non-epidemic morbidity

867. A 49-year-old female patient is under dispensary observation for uterine fibromyoma
. Over the past year, the uterine tumor has increased to 20 weeks of pregnancy. What is the rational
method of treatment?
100% Surgical treatment
0% Hormone treatment
0% Follow-up
0% Uterine artery embolization
0% Treatment with prostaglandin inhibitors

868. A 46-year-old female patient was issued a 10-day disability certificate for acute
chronic cholecystitis. The patient's general condition has improved, but clinical manifestations of the disease still
occur. Who has the right to extend their sick leave?
100% LCC
0% Family Doctor
0% Deputy Chief Physician for Medical Work
0% Deputy Chief Physician for Labor Expertise
0% Chief Medical Officer

869. a 43-year-old female patient was hospitalized in serious condition. He has Addison's disease. Constantly
took prednisone. I stopped taking it within a week. Objectively: soporotic state,
skin and visible mucous membranes are pigmented, skin and muscle turgor is reduced. Heart tones
are muted, scratched, blood pressure-60/40 mm Hg, heart rate-96/min. In the blood: Na-120 mmol / l, K-5,8 mmol / l
. what hormone deficiency plays a leading role in the development of this complication?
100% Cortisol
0% Corticotropin (ACTH)
0% Adrenaline rush
0% Norepinephrine
0% Androstenedione

870. 52-year-old patient, secretary-typist, 30 years of work experience. Complains of cramps in


the right hand while working and inability to type or write. The load on
the brush is up to 80% of the working time. He has been ill for 2 years. Objectively: the right hand is tense,
the muscle tone is increased, and convulsions occur when trying to write. During the examination, no pathological
manifestations of the central nervous system were found. What is the most likely diagnosis?
100% Convulsive form of coordination neurosis

https://translate.yandex.com/en/doc 193/540
22:20 ,27.6.2023 �� C : ; 5 B
0% Neuralgic form of coordination neurosis
0% Paretic form of coordination neurosis
0% Hysterical neurosis
0% Chronic manganese intoxication

871. a 15-year-old teenager's examination at the military registration and enlistment office revealed an interval systolic
murmur at the apex of the heart, an accent of the second tone over the pulmonary artery, and tachycardia. Which of the additional ones

201

Downloaded from the site - online testing step

what are the most informative methods of examination for establishing a diagnosis?
100% Echocardiography
0% Electrocardiography
0% Radiography
0% Phonocardiography
0% Rheography

872. A 67 - year-old female patient complains of swelling of the face and legs, lower back pain that increases
during movement, severe weakness, sometimes nosebleeds, and an increase in body temperature up to 38.4 ° C.
Objectively: pain on palpation of the spine and ribs. During a laboratory examination:
proteinuria - 4.2 g / day, ESR-52 mm/hour. What changes in laboratory parameters are most
expected?
100% Total serum protein -101 g / l
0% White blood cells -15.3 g / l
0% Hemoglobin -165 g / l
0% Albumins-65%
0% y-globulins -14%

873. A 24-year-old" injectable " drug addict who has been suffering from infectious endocarditis for 4 months
is undergoing inpatient treatment in the cardiology department. The patient
was treated three times with different antibiotic regimens. Currently, the patient complains of an
increase in body temperature to 37.5 oC, there are signs of left ventricular
failure. Clinical examination revealed grade III aortic valve
insufficiency. What is the most appropriate treatment strategy?
100% Surgical treatment (aortic valve replacement)
0% Next Antibiotic Replacement
0% Treatment with nonsteroidal anti-inflammatory drugs
0% Immunomodulator therapy
0% Introduction to the probiotic treatment regimen

874. the number of births of viable fetuses per 1000 women aged 15 to 44 years in the population
determines:
100% Childbearing index
0% Reproductive level
0% Birth rate
0% Perinatal level
0% Midwifery level

875. A 9 - year-old girl with a history of intermittent wheezing was examined by a pediatrician
https://translate.yandex.com/en/doc 194/540
22:20 ,27.6.2023 �� C : ; 5 B
. I haven't taken any medications lately. Objectively: anxiety and perioral cyanosis.
There is an expansion of intercostal spaces. On auscultation: breathing is weakened, dry
, scattered wheezes that can be heard at a distance. The child was hospitalized. Which of the following is not
indicated for providing emergency care to a child?
100% Cromoline Inhalations
0% Eufillin IV

202

Downloaded from the site - online testing step

0% Oxygen Therapy
0% Intravenous Corticosteroids
0% Salbutamol inhalations

876. in which of the following rhythm disorders is it advisable to use ATP to


restore sinus rhythm?
100% Paroxysmal supraventricular tachycardia
0% Paroxysmal ventricular tachycardia
0% Atrial fibrillation
0% Atrial flutter
0% Ventricular fibrillation

877. 5 days after hip joint transplantation, a 72-year-old woman suddenly


developed an attack of suffocation, hypotension, and sweating. Increased filling and
pulsation of the neck veins is noted. No slight changes were detected during auscultation and percussion. Your heart sounds
normal. On the ECG: sinus tachycardia, first-time block of the right bundle
branch, non-specific minor changes in the S-T interval and T wave. What is the most likely diagnosis?
100% Pulmonary embolism
0% Acute myocardial infarction
0% Dissecting aortic aneurysm
0% Pericarditis
0% Aspiration

878. A 38-year-old man, previously healthy, suddenly developed an attack of acute


abdominal pain radiating to the left lumbar and groin areas, accompanied
by nausea, sweating and frequent urination. The patient is restless, fidgeting in bed.
During the examination, no pathological changes were detected. What is the most likely diagnosis?
100% Left-sided renal colic
0% Herpes zoster
0% Sigmoid diverticulitis
0% Torsion of the left testicle
0% Retroperitoneal hemorrhage

879. a patient who was on parenteral nutrition after a serious injury


developed diarrhea, depression, alopecia, and dermatitis around the mouth and eyes. Which of
the following elements is necessary to correct the changes that have occurred?
100% Zinc
0% Iodine
0% Selenium
0% Silicon
0% Copper
https://translate.yandex.com/en/doc 195/540
22:20 ,27.6.2023 �� C : ; 5 B

880. a 4-year-old girl suddenly developed a coughing fit and shortness of breath while playing with toys.
Objectively: HR-45 / min, HR-130 / min. Percussion: shortening the percussion sound from the right

203

Downloaded from the site - online testing step

sides in the lower sections. Auscultation: on the right side, breathing is weakened with a bronchial
tinge. On the radiograph: darkening of the lower lobe of the lungs on the right. No inflammatory
changes were found in the blood. The diagnosis was made: a foreign body of the right bronchus. Please indicate what complication
caused this clinical picture:
100% Atelectasis
0% Emphysema
0% Pneumothorax
0% Bronchitis
0% Pneumonia

881. A 47-year-old woman complains of spotting from the vagina for 2 weeks, which
appeared after a 3-month delay in menstruation. Menarche from the age of 13. Menstruation
has been irregular for the last year. In the blood: Hb-90 g / l, ep.- 2,0•1012/ l, lake.- 5,6•109/L. on vaginal
examination: the uterus is of normal size, the appendages are not palpable. What
is the most likely diagnosis?
100% Climacteric bleeding
0% Endometrial polyp
0% Blood clotting disorders
0% Endometrial cancer
0% Incomplete abortion

882. a 35-year-old man is agitated, constantly trying to look behind the door and into the next
room, because he is convinced that his friends are there. Claims to hear
friends and strangers talking, even though no one is around at that time. Tries to convince
the doctor that there is an argument going on behind the wall about "his punishment", loudly repeats
phrases that he allegedly heard from behind the wall. Determine the patient's condition:
100% Acute hallucinosis
0% Verbal illusions
0% Nonsense
0% Confabulations
0% Obsessions

883. in a newborn at the 1st minute after birth: HR-26 / min., HR-90 / min., muscle
tone is low. During the suction of mucus with a catheter from the nose and mouth-the child reacts
with a grimace; the skin is cyanotic. Auscultation: vesicular respiration is weakened over the lungs.
The heart tones are sonorous. In 5 minutes: PDR-40 / min., rhythmic, heart rate-120 / min., acrocyanosis, muscle
tone is reduced. What is the most likely diagnosis?
100% Neonatal asphyxia
0% Birth trauma of a newborn
0% Hemolytic disease of the newborn
0% Hemorrhagic disease of the newborn
0% Neonatal sepsis

https://translate.yandex.com/en/doc 196/540
22:20 ,27.6.2023 �� C : ; 5 B
884. a 39-year-old patient has been suffering from diabetes mellitus for 10 years. The last year marks
cold toes, pain and numbness. Objectively: the skin of the lower extremities is dry,
204

Downloaded from the site - online testing step

elegant, cool to the touch; pulsation in the femoral and popliteal arteries is preserved. What
is the most likely diagnosis?
100% Diabetic vascular microangiopathy of the lower extremities
0% Diabetic macroangiopathy of the vessels of the lower extremities
0% Raynaud's disease
0% Obliterating atherosclerosis of the vessels of the lower extremities
0% Obliterating endarteritis of the vessels of the lower extremities

885. A 7-year-old child was admitted to the clinic with a broken upper jaw after
a car accident. The unconscious child was vomiting. After 2 hours I woke up, answered
questions, but after 4 hours I developed sopor. Objectively: Ps-180 / min, convulsive contractions
of the left extremities, anisocoria. What is the main diagnosis?
100% Epidural hematoma
0% Concussion of the brain
0% Brain contusion
0% Skull base fracture
0% Dislocation syndrome

886. A 68-year-old patient has been suffering from chronic pancreatitis for 35 years. In the last 5
years, he has noted a decrease in pain, bloating, frequent bowel
movements (grayish in color, shiny, with impurities of undigested food) up to 3-4 times a day,
progressive weight loss. The change in symptoms in the patient is due to the addition of:
100% External secretory pancreatic insufficiency
0% Intracretory pancreatic insufficiency
0% Lactase deficiency syndrome
0% Irritable bowel syndrome
0% Chronic enterocolitis

887. A 60-year-old woman has been suffering from arterial hypertension for about 15 years. After a repeated
stroke, he complains of an unmotivated decrease in mood,difficulty concentrating,
forgets to close the front door, and does not remember the events of the past day. CT scans show areas of post-infarction
changes in the cortical posterolabial areas. What is the most likely diagnosis for the patient?
100% Vascular dementia
0% Alzheimer's disease
0% Huntington's Disease
0% Pick's disease
0% Dissociative amnesia

888. a mother with a 6-year-old girl complained to a gynecologist about the appearance
of purulent whites in the girl, irritating the skin of the genitals and thighs. During
the examination: the external genitals are hyperemic, swollen. Severe hyperemia of the presenka of the vagina and
hymen. Discharge from the genital fissure is abundant, mucopurulent. What kind of disease can
you think about?
100% Vulvovaginitis
0% Diabetes mellitus

https://translate.yandex.com/en/doc 197/540
22:20 ,27.6.2023 �� C : ; 5 B
205

Downloaded from the site - online testing step

0% Vaginal diphtheria
0% Trichomonas colpitis
0% Cystitis

889. A typhoid patient suddenly developed abdominal pain on the 14th day of her illness. The patient
is conscious. The pain is acute. The abdomen is involved in the act of breathing, not deflated. Palpation: moderate
muscle tension of the anterior abdominal wall, more in the right iliac region. Hepatic
dullness is not detected. Peristalsis is audible, flabby. The Shchetkin-Blumberg symptom
is positive. Body temperature 38.9 oC. Ps-104 / min. What is the most likely diagnosis?
100% Perforation of a typhoid ulcer
0% Adhesive intestinal obstruction
0% Acute perforated appendicitis
0% Perforation of acute gastric ulcer
0% Acute pancreatitis

890. at a doctor's appointment, after an objective clinical examination, a 12-year-old child


was diagnosed with mitral valve prolapse. Which of the additional instrumental
examination methods should be used to confirm the diagnosis?
100% Echocardiography
0% Chest radiography
0% FCG
0% ECG
0% Veloergometry

891. A 48-year-old patient complains of contact bleeding. Examination in mirrors revealed


hypertrophy of the cervix. The latter has the appearance of "cauliflower", easily injured, dense.
During bimanual examination: the arches are shortened, the body of the uterus is sedentary. What is the most
likely diagnosis?
100% Cervical cancer
0% Uterine fibromyoma
0% Endometriosis
0% Cervical pregnancy
0% Papillomatosis of the cervix

892. A 34-year-old patient has been suffering from anxiety disorders with panic attacks for about two years.
I was treated by a psychotherapist. Incomplete remission was achieved. In connection with moving to another
city interrupted psychotherapy. Soon the condition worsened significantly, and he feels almost constant
anxiety. Panic attacks up to 5-6 times a day, accompanied by a strong heartbeat,
shortness of breath, cold sweat, fear of death. Which group of drugs is most appropriate for
drug therapy?
100% Antidepressants
0% Neuroleptics of sedative action
0% Antipsychotic neuroleptics
0% Lithium preparations
0% Cardiotonics, respiratory analeptics

206

https://translate.yandex.com/en/doc 198/540
22:20 ,27.6.2023 �� C : ; 5 B

Downloaded from the site - online testing step

893. the maximum


permissible concentration of carbon dioxide in the air is a sanitary indicator of the cleanliness of the air in the study room. What is the
maximum permissible concentration of carbon dioxide in the
air (in%)?
100% 0,1%
0% 0,05%
0% 0,15%
0% 0,2%
0% 0,3%

894. A 48-year-old patient experienced a sharp abdominal pain during esophageal augmentation due to cicatricial stricture
. Previously, he was ill with duodenal ulcer of the 12th duodenum. On examination: the abdomen is sharply
tense, painful; Ps-110 / min. Painful swallowing of saliva. Pallor of the skin. What
is the most likely diagnosis?
100% Perforation of the abdominal esophagus
0% Acute myocardial infarction
0% Pinched diaphragmatic hernia
0% Perforation of duodenal ulcer 12
0% Mesenteric vascular thrombosis

895. A 38-year-old patient was admitted with complaints of headache, dizziness, and rapid
heartbeat during exercise. An increase in blood pressure is noted
for 3 years, the maximum figures are 180/110 mm Hg. objectively: the state of moderate severity,
blood pressure-160/100 mm Hg on the ECG: sinus rhythm with a heart rate of 98 / min., horizontal position
of the electrical axis of the heart, liquid supraventricular extrasystole. What drug should
I start treatment with?
100% Atenolol
0% Nifedipine
0% Clonidine
0% Papazolum
0% Captopril

896. a 3-year-old boy suddenly fell ill. His voice became hoarse and hoarse, sometimes Aphonic.
A rough, loud, dry, unproductive cough appeared. Body temperature 37.4 oC. Objectively:
the child is calm, the position in bed is forced (with a fixed shoulder girdle). Breathing
is stenotic, clearly audible at a distance. The constant work of the respiratory muscles is clearly visible.
Auscultation: breathing is hard, evenly weakened, especially in the posterior and lower parts
of the lungs. Please indicate which mechanism dominates the pathogenesis of croup?
100% Swelling of the laryngeal and tracheal mucosa
0% Spasm of the smooth muscles of the larynx, trachea and bronchi
0% Hypersecretion of the laryngeal, tracheal and bronchial mucosa glands
0% Obstruction
0% Laryngospasm

897. A 52-year-old patient was admitted to the hospital with complaints of bleeding of the mucous membranes,
significant skin hemorrhages in the form of ecchymoses, spots, nasal and gastric bleeding. After

207

https://translate.yandex.com/en/doc 199/540
22:20 ,27.6.2023 �� C : ; 5 B
Downloaded from the site - online testing step

A diagnosis of thrombocytopenic purpura was established after clinical examinations. What is the most
likely cause of this disease?
100% Formation of antiplatelet autoantibodies
0% Hemostatic disorders
0% Blood coagulation factor VIII deficiency
0% Hereditary insufficiency of plasma coagulation factors
0% Iron deficiency in serum, bone marrow and depot

898.A 37-year-old patient


who was temporarily disabled for 117 days during a year due to an exacerbation of chronic obstructive bronchitis applied to the district
therapist.
It will
100%determineThe
thepatient
legallyiscorrect
subjecttactics of thetodoctor
to referral in this case:
the MSEC
0% Extension of the disability certificate by the therapist
0% Referral to the LCC for extension of the disability certificate
0% Issue of a labor (extra-paid) disability certificate
0% Referral of the patient for sanatorium treatment

899. a 2-year-old child who is being treated in the children's department for acute
pneumonia has a sharply worsened general condition: the child is restless, shortness of breath is manifested,
BDR50/min, Ps-130 / min. During percussion: displacement of the inter-whistle to the left, on the right-dulling
of the percussion sound to the 5th rib, above-with a boxy tint; during auscultation - breathing
on the right is sharply weakened. What is the most likely diagnosis?
100% Strained right-sided pyopneumothorax
0% Atelectasis of the left lung
0% Acute interstitial emphysema
0% Right-sided pneumonia
0% Acute pericarditis

900. A 54-year-old patient complains of constant dull pain in the mesogastric region,
weight loss, impurities of dark blood in the stool, constipation. During the year, weight loss up to 10 kg. In the blood:
Er. -3,5•1012/l, Hb-87 g/l, lake.- 12,6•109/ l, rod-shaped shift, ESR-43 mm / h. What
is the most likely diagnosis?
100% Cancer of the transverse colon
0% Peptic ulcer disease of the stomach
0% Chronic colitis
0% Chronic pancreatitis
0% Stomach cancer

901. a 48-year-old female patient complains of an aching spire in the lateral parts of the abdomen, which
decreases after the act of defecation and gas discharge; alternation of diarrhea and constipation.
I had dysentery two years ago. During palpation of the abdomen, soreness is detected, alternating
spasmodic and atonic, and burbling parts of the colon. The most informative method for
making a diagnosis is:
100% Colonoscopy
0% Digital examination of the rectum

208

Downloaded from the site - online testing step

0% Rectoromanoscopy
https://translate.yandex.com/en/doc 200/540
22:20 ,27.6.2023 �� C : ; 5 B
0% Coprocytogram in dynamics
0% Ultrasound of the abdominal cavity

902. a child of 10 months after drinking unboiled milk is ill on the 5th day: temperature
38-39 ° C, vomiting, loose stools. Flabby, pale skin. The tongue is overlaid with white
layers. Heart tones are muted. The stomach is swollen, there is a grunt in the navel area, the liver is + 3
cm. Bowel movements are liquid, up to 5 times a day, dark green in color, with an admixture of mucus. What
is the most likely diagnosis?
100% Salmonellosis
0% Staphylococcal intestinal infection
0% Escherichiosis
0% Acute shigellosis
0% Rotavirus infection

903. A 22-year-old man with polyarthralgia and high fever was found to have right-sided
exudative pleurisy. Radiologically, there is a homogeneous darkening on the right side of the IV rib downwards
. On the left, in segment II, there are single dense focal shadows. Mantoux test with 2TE -16 mm
papule. In the pleural fluid, an increased protein content, a positive Rivalt reaction, an increased
content of white blood cells with a predominance of lymphocytes. Determine the most likely etiology of pleurisy:
100% Tuberculosis
0% Rakova
0% Staphylococcal infection
0% Viral
0% Autoimmune

904. In district N, the prevalence of diseases among the population was 1,156 cases per 1,000
population. Which of the following indicators characterizes the prevalence of diseases?
100% Intensive
0% Extensive
0% Ratio
0% For visibility purposes
0% Standardized

905. A 49-year-old patient with stable angina pectoris of FC II suddenly developed angina
pain, which was eliminated only by 6 tablets of nitroglycerin. Objectively: AT-120/80 mm Hg,
Ps-104 / min, single extrasystoles. On the ECG: liquid left ventricular extrasystoles,
S-T segment depression in I, aVL and high, pointed T waves in V2, V3, V4 leads. What is the most
likely diagnosis?
100% Unstable angina pectoris
0% Stable angina, FC II
0% Transmural myocardial infarction
0% Stable angina pectoris, IV FC
0% Variant angina pectoris
906. A 58-year-old female patient complains of periodic headache, dizziness and tinnitus.

209

Downloaded from the site - online testing step

He has been suffering from diabetes for 15 years. Objectively: heart sounds are rhythmic, heart rate-76 / min,
emphasis on the tone above the aorta, AT-180/110 mm Hg in the urine: relative density -1.014. daily

https://translate.yandex.com/en/doc 201/540
22:20 ,27.6.2023 �� C : ; 5 B
protein loss in the urine is up to 1.5 g. The drug of choice for the treatment of arterial hypertension in this
100% !ACE INHIBITOR
patient will be:
0% b-blocker
0% Calcium channel antagonist
0% Thiazide diuretic
0% a-blocker

907. A 25-year-old patient 2 weeks after pharyngitis started complaining of


fever up to 38 ° C, general weakness, suffocation during walking, swelling and pain in the joints
of a volatile nature. Objectively: cyanosis of the lips, Ps-100 / min., weak filling, rhythmic. The left
border of the heart is shifted outside from the medioclavicular line by 1 cm. I the tone at the apex is weakened,
a gentle systolic murmur is heard. What is the most likely etiological factor that caused this
pathological process?
100% b-hemolytic streptococcus
0% Staphylococcus aureus
0% Pneumococcus
0% Virus
0% Mushrooms

908. a patient received radiation exposure. Complains of weakness, frequent nosebleeds,


"bruises"on the body, palpitations, shortness of breath. Often suffers from respiratory diseases. Blood test:
Er.- 1,2*1012/ l, Hb-54 g / l, lake.-1.7*109/l, E.-0%, P.-0%, S.-32%, L.-62%, M.-6%; SCHOE - 52
mm / year; blood clot.-30 * 109/l. what treatment is most appropriate in this case?
100% Bone Marrow Transplant
0% Platelet mass transfusion
0% Red blood cell mass transfusion
0% Whole blood transfusion
0% Anti-lymphocytic immunoglobulin

909. A 50-year-old patient complains of dull pain in the heart, suffocation,


and fever up to 38 ° C. I had the flu a week ago. Objectively: Ps-100 / min, disappears on inspiration.
AT-100/70 mm Hg, heart tones are deaf. ECG: low voltage, ST segment raised above
the isoline in all leads. On the X-ray: the shadow of the heart is extended in all directions.
Low-amplitude heart pulsation. What is the most likely diagnosis?
100% Exudative pericarditis
0% Myocardial infarction
0% Dilated cardiomyopathy
0% Myocarditis
0% Angina pectoris

910. A 39-year-old patient went to the doctor complaining of morning headache, decreased
appetite, nausea, morning vomiting, and recurrent nosebleeds. At the age of 15, he suffered

210

Downloaded from the site - online testing step

acute glomerulonephritis. During the examination, there was an increase in blood pressure to
220/130 mm Hg, hemorrhages on the skin of the hands and feet, pallor of the skin and mucous membranes. Which of
the above blood biochemical parameters is of diagnostic significance in this case?
100% Creatinine
0% Bilirubin

https://translate.yandex.com/en/doc 202/540
22:20 ,27.6.2023 �� C : ; 5 B
0% Sodium
0% Uric acid
0% Fibrinogen

911. A 43-year-old patient was admitted to the infectious diseases hospital with complaints of high
body temperature and severe headache. Ill on day 2. On examination, a carbuncle was found on the forearm
, around which edema and minor soreness were expressed. Regional lymphadenitis.
Hepatolienal syndrome. From the medical history, it is known that the patient works on a livestock farm.
What disease should you think about first?
100% Anthrax
0% Erysipelas
0% Erysipeloid
0% Skin cancer
0% Eczema

912. in the study of morbidity with temporary disability


of machine-building plant workers, the average duration of one case was 20 days. What diseases
affected the indicator value?
100% Chronic diseases
0% Sharp
0% With subacute flow
0% Pre-chords
0% Hard to determine

913. A 49-year-old patient developed pain in the right side of her abdomen 4 days ago, which initially
increased and gradually subsided by the end of the second day, increasing only with sudden movements
and coughing. The body temperature was initially 38oC and gradually decreased to 37.5 oC. Condition
satisfactory, Ps-84 / min. The tongue is wet. In the right iliac region, there is a weakly expressed
muscle tension, due to which a dense painful tumor-like formation with an uneven
contour and limited mobility is palpated. What is the most likely diagnosis?
100% Appendicular infiltrate
0% Appendicular abscess
0% Cecal tumor
0% Pylephlebitis
0% Right-sided adnexitis

914. A 4-month-old child is acutely ill: the temperature is 38.5 ° C. Single vomiting, flabbiness.
After 10 hours, a rash appeared on the buttocks and lower extremities in the form of petechiae, spots and
papules. Some hemorrhagic elements with necrosis in the center. What is the most likely medical condition?
100% Meningococcemia

211

Downloaded from the site - online testing step

0% Rubella
0% Flu
0% Hemorrhagic vasculitis
0% Scarlet fever

915. A 13-year-old girl was hospitalized in the cardiology department with complaints of
muscle and joint pain. When examining the face in the area of the bridge of the nose and cheeks, erythema was found with edema
https://translate.yandex.com/en/doc 203/540
22:20 ,27.6.2023 �� C : ; 5 B
in the form of a butterfly. What is the most likely disease in this case?
100% System red shepherd dog
0% Rheumatism
0% Dermatomyositis
0% Rheumatoid arthritis
0% Nodular periarteritis

916. A 22-year-old woman was hospitalized in the gynecological department with complaints of pain in the lower
abdomen, fever up to 39.5 oC. Objectively: heart rate-108 / min, AT-120/80 mm Hg,
the abdomen is moderately swollen, sharply painful in the hypogastric region. Shchetkin's symptom is positive in
the hypogastric region. Vaginal examination: the uterus and appendices are not palpable
due to the tension of the anterior abdominal wall, the posterior vault of the vagina hangs, sharply painful. What
is the most likely diagnosis?
100% Pelvioperitonitis
0% Acute adnexitis
0% Acute endometritis
0% Ectopic pregnancy
0% Ovarian apoplexy

917. A 15-year-old girl suddenly developed arthralgia, headache,nausea, vomiting, pain and
muscle tension in the lower back; her body temperature rose to 38-39 ° C. Pasternatsky's symptom
is sharply positive on the right side. In the urine: bacteriuria, pyuria. What is the most
likely diagnosis?
100% Acute pyelonephritis
0% Renal colic
0% Acute glomerulonephritis
0% Paranephral abscess
0% Cystitis

918. the district pediatrician examined a 4-year-old child. Complaints of pain when chewing and
opening the mouth, headache, fever up to 38.9 oC. General: in the areas
of the parotid salivary glands, a swelling is contoured, which is moderately painful on palpation, and the skin above
it is not changed. On examination of the oropharynx, the opening of the Stenon duct is hyperemic. What
is the most likely diagnosis?
100% Mumps infection
0% Oropharyngeal diphtheria
0% Sialoadenitis
0% Infectious mononucleosis

212

Downloaded from the site - online testing step

0% Cervical lymphadenitis

919. A 59-year-old female patient went to a antenatal clinic complaining of spotting


from the genital tract. Postmenopausal 12 years. During vaginal examination: external
genitalia with signs of age-related involution, the cervix is not eroded,
and there is a slight spotting from the cervical canal. The uterus is of normal size, the appendages are not palpable.
The arches are deep, not painful. What additional research methods should be used to
clarify the diagnosis?
100% Separate diagnostic curettage

https://translate.yandex.com/en/doc 204/540
22:20 ,27.6.2023 �� C : ; 5 B
0% Laparoscopy
0% Puncture of the abdominal cavity through the posterior arch
0% Advanced colposcopy
0% Culdoscopy

920. after previously suffering from purulent otitis media, a 1-year-old boy began to worry about pain in
the upper third of his left thigh, and his body temperature rose to 39oc. Objective: swelling
of the thigh in the upper third and smoothness of the inguinal fold. The limb is in a half-bent state.
Active and passive movements are impossible due to sharp pain. What is the most likely diagnosis?
100% Acute hematogenous osteomyelitis
0% Acute coxit
0% Intermuscular phlegmon
0% Osteosarcoma
0% Brody's abscess

921. in the current year, 10% of the employees of the institution were not ill once, 30%
once, 15% twice, 5% four times, and all the others 5 or more times. What is the specific weight of employees
assigned to the I-th health group?
100% 55%
0% 10%
0% 40%
0% 60%
0% 22%

922. a 16-day-old child, due to the mother's hypogalactia, needs to be prescribed additional food. Which of
the following should be assigned to your child?
100% "Malyutka "
0% "Baby"
0% Kefir
0% Whole cow's milk
0% Acidophilic milk

923. a 32-year-old patient complains of pain in the small joints of the hands, paresthesia in
the fingertips, weakness, difficulty swallowing. He has been ill for 13 years. Objectively:
facial expression, shortening of the nail phalanges. Compaction of the skin in the shoulder area

213

Downloaded from the site - online testing step

belts. In the lungs radiologically-basal pneumosclerosis. With FGS-narrowing of the esophagus


in the cardiac department. In the blood: lake.- 9,8•109/ l, ESR-22 mm / h, y-globulin-22%. What
is the most likely diagnosis?
100% Systemic scleroderma
0% System red shepherd dog
0% Rheumatoid arthritis
0% Dermatomyositis
0% Myxedema

924. a 32-year-old patient with chronic viral hepatitis complains of dull aching
pain in the right hypochondrium, nausea, and dry mouth. Ob-no: liver size according to Kurlov data -13-12-11

https://translate.yandex.com/en/doc 205/540
22:20 ,27.6.2023 �� C : ; 5 B
see, spleen +2. AsAT - 3.2 mmol/l-h, AlAT-4.8 mmol/l-h. Serological
testing revealed HBeAg, a high concentration of HBV DNA. Which of the following
medications is the drug of choice in the treatment of this patient?
100% a-interferon
0% Acyclovir
0% Remantadine
0% Arabinoside Monophosphate
0% Essentiale forte

925. A 6-month-old child develops jaundice with a greenish tinge from birth.
There are phenomena of hemorrhagic diathesis and pruritus of the skin. What is the most likely
pathology in a child?
100% Biliary tract atresia
0% Erythroblastosis
0% Gilbert's syndrome
0% Krigler-Nayyar syndrome
0% Dubin-Johnson syndrome

926. A 5-month-old boy was born prematurely


and did not have any further illnesses during the neonatal period. When examined in the clinic,pallor of the skin,
drowsiness is noted. In the blood: Hb-95 g / l, Er.- 3,5•1012/ l, reticular -9%, CP - 0.7, osmotic resistance
of red blood cells - 0.44-0.33%, serum iron - 4.9 mmol/l. what is the most
likely cause of anemia?
100% Iron deficiency
0% Immaturity of hematopoiesis
0% Infectious process
0% Hemolysis of red blood cells
0% Vitamin B12 Deficiency

927. A 45-year-old woman complains of paroxysmal unbearable pain in the left side
of her face lasting 1-2 minutes. Seizures are triggered by chewing. I got sick two months ago
after hypothermia. Objectively: pain at the exit points of the trigeminal nerve on the left. A touch at
the left wing of the nose causes another attack with a tonic spasm of the facial muscles. Which of
these diagnoses is most likely?

214

Downloaded from the site - online testing step

100% Trigeminal neuralgia


0% Neuralgia of the glossopharyngeal nerve
0% Arthritis of the mandibular joint
0% Facial migraine
0% Sinusitis

928. an 8 - year-old girl complains of joint pain, fever up to 38oC,


shortness of breath. Objectively: the left border of the heart is shifted to the left by 2.5 cm, tachycardia, systolic murmur at
the apex and at the V point. In the blood: lake.- 20,0•109/ l, SHOE-18 mm / year. What is the most likely sign
that justifies the diagnosis of rheumatism?
100% Cardit
0% Arthralgias
0% Leukocytosis

https://translate.yandex.com/en/doc 206/540
22:20 ,27.6.2023 �� C : ; 5 B
0% Delirium tremens
0% Accelerated ESR

929. A 38-year-old man was admitted to the hospital in an unconscious state. Got sick yesterday:
headache, nausea, vomiting, to - 38.5 oC, dizziness, delirium. During
the last 4 days, he complained of pain and hearing loss in his left ear. Objectively: the condition
is soporotic, rigidity of the occipital muscles, Kernig's symptom on both sides, general hyperesthesia,
suppuration from the left ear. Which of these diagnoses is most likely?
100% Secondary purulent meningitis
0% Primary purulent meningitis
0% Tuberculosis meningitis
0% Subarachnoid hemorrhage
0% Parenchymal-subarachnoid hemorrhage

930. a 22-year-old female patient complains of delayed menstruation for 2 months.


Your taste preferences have changed. Childbirth-0, abortions-0. Vaginal examination: the mucous membrane of the vagina and
cervix is cyanotic, the uterus is bullet-shaped, enlarged to 7-8 weeks of pregnancy,
of a soft consistency. Appendages without features. The isthmus of the uterus is softened.
The vaginal arches are free. What is the most likely diagnosis?
100% Uterine pregnancy
0% Uterine fibroids
0% Ovarian-menstrual disorders
0% Bubble drift
0% Chorionic epithelioma

931. a 23-year-old woman complains of periodic chills with an increase in body temperature up
to 40oC, a feeling of heat, which is replaced by torrential sweat. The patient had already suffered 3 such attacks,
they occurred every other day and lasted 12 hours. I've been living in Africa for the last three months
. Liver and spleen are enlarged. In the blood: Er.- 2,5•1012/ L. what is the most
likely diagnosis?
100% Malaria
0% Typhus fever

215

Downloaded from the site - online testing step

0% Sepsis
0% Hemolytic anemia
0% Leptospirosis

932. a two-year-old girl has an elevated body temperature of up to 37.2 ° C and a skin rash. Objectively:
the child's general condition is satisfactory. On the skin of the back, extensor surfaces of the extremities-
a pink macular rash. The mucosa of the palatine arches is fine-toothed, moderately
hyperemic. Slight mucosal discharge from the nose. Enlarged occipital and posterior cervical
lymph nodes. What is the most likely diagnosis?
100% Rubella
0% Scarlet fever
0% Chickenpox
0% Enterovirus exanthema
0% Measles

https://translate.yandex.com/en/doc 207/540
22:20 ,27.6.2023 �� C : ; 5 B
933. A 40-year-old woman with mitral stenosis complains of shortness of breath and weakness, which gradually
progress. Today, he can only do light homework. What
therapy is most appropriate for this patient?
100% Surgical intervention
0% Bicillin Therapy
0% Vasodilators
0% Cardiac Glycosides
0% Diuretics

934. A 39-year-old patient suffering from hypertension suddenly experienced intense


headache in the back of the head, nausea and repeated vomiting lasting for the fifth hour. Objectively:
Ps-88 / min, BP-205/100 mm Hg, pain of occipital points, rigidity of occipital muscles.
Kernig's symptom is positive on both sides. Subarachnoid hemorrhage is suspected. Which of
the following research methods is crucial for confirming
a previous diagnosis?
100% Lumbar puncture
0% Fundus examination
0% Ultrasound Doppler imaging
0% EEG
0% Computed tomography

935. immediately after the birth of the child, the parent started bleeding, the blood loss exceeded
the physiological one and continues. There are no signs of placental separation. What are the doctor's tactics?
100% Perform manual placental separation and litter removal
0% Perform uterine tamponade
0% Make an instrumental revision of the uterine cavity walls
0% Remove droppings using the Crede-Lazarevich method
0% Enter intravenous Methylergometrine with glucose

216

Downloaded from the site - online testing step

936. A 33-year-old patient was admitted to the infectious diseases hospital on the 7th day of the disease with
complaints of severe weakness, high fever, pain in the leg and lower back muscles, jaundice,
dark urine color, headache. I became acutely ill from chills, fever up to 40oC,
headache, pain in the calf muscles and lower back. On the 4th day of the disease
, jaundice appeared, on the 5th - nosebleed, hemorrhage in the sclera. The duration of fever is 6 days. Daily
diuresis-200 ml. What is the most likely diagnosis?
100% Leptospirosis
0% Typhoid fever
0% Viral hepatitis A
0% Sepsis
0% Yersiniosis

937. A 25-year-old man developed facial edema and moderate sacral pain 2 weeks after having a sore
throat. Objectively: temperature 37.5 oC, blood pressure-100/80 mm Hg in the urine: Er. - fresh up to 100
v / S, protein-2.20 g / l, hyaline cylinders - up to 10 v / S, relative density-1.020. what is the most
likely diagnosis?
100% Acute glomerulonephritis
0% Kidney cancer

https://translate.yandex.com/en/doc 208/540
22:20 ,27.6.2023 �� C : ; 5 B
0% Acute pyelonephritis
0% Urolithiasis
0% Chronic glomerulonephritis

938. A 22-year-old female patient suffered from acute enterocolitis of yersiniasis origin. She was treated in
an infectious diseases hospital. After discharge, there was acute pain in the right knee joint, edema,
and limited mobility. In the blood: leukocytosis, ESR-30 mm / h, CRP-2 mm, fibrinogen-5.2 g / l,
a2globulins-12%, RF ( - ). What is the most likely diagnosis?
100% Reactive arthritis
0% Purulent arthritis
0% Deforming osteoarthritis
0% Rheumatoid arthritis
0% Joint chondromatosis

939. A 60-year-old patient complains of weakness, dizziness, heaviness in


the upper abdomen, paresthesia of the fingers and toes. Objectively: jaundice of the skin, tongue of crimson
color, smooth. Hepatomegaly. In the blood: Hb-90 g / l, Er.- 2,3•1012/ l, reticule.- 0,2%;
KP1, 2, macrocytosis; Gelli corpuscles, Cabot's rings. Which drug is most
appropriate for treatment?
100% Vitamin B12
0% Ferroplex
0% Red blood cell mass
0% Prednisone
0% Dysferol

940.a child with massive edema on the face, torso,


limbs, and ascites was admitted to the hospital. AT-90/50 mm Hg in urine: protein-3.1 g / l, leuc.- 2-3 v p/ s, ep. - 5-7 v p / s. What

217

Downloaded from the site - online testing step

will it allow the most reliable assessment of kidney function?


100% Endogenous creatinine clearance
0% Nechiporenko urine test
0% Proteinogram
0% Daily proteinuria
0% General urinalysis

941. A 29-year-old woman complains of periodic pain in the right side of her head, which
is provoked by sharp smells and excitement. The patient's mother suffers from a similar disease.
Internal organs-without pathology. During the attack, there is general hyperesthesia, nausea,
and at the end - polyuria. When palpating the right temporal artery during an attack, its
tension and soreness are noted. General blood and urine tests without signs. What is the most
likely diagnosis?
100% Migraines
0% Neuritis of the facial nerve
0% Trigeminal neuralgia
0% Meniere's syndrome
0% Epilepsy

https://translate.yandex.com/en/doc 209/540
22:20 ,27.6.2023 �� C : ; 5 B
942. a 52-year-old woman complains of a skewed face. I got sick 2 days ago after
hypothermia. Objectively: the temperature is 38.2°C. There is an asymmetry of the face. The frontal
folds on the left side are smoothed out. The left eye is wider than the right and does not close. Smoothed left
nasolabial fold, drooping corner of the mouth. No other pathology was detected. In the blood: lake.- 10•109/l,
WSE-20 mm/hour. What is the most likely diagnosis?
100% Neuritis of the facial nerve
0% Trigeminal neuralgia
0% Hemicrania (migraine)
0% Ischemic stroke
0% Brain tumor

943. a patient went to the doctor complaining of general weakness, fever, and a painful rash
on the torso. Ill for 3 days. Objectively: on the lateral surface of the trunk on the left side, against the background
of hyperemia and edema, there are grouped blisters with serous-cloudy and hemorrhagic
contents. What is the most likely diagnosis?
100% Herpes zoster
0% Simple contact dermatitis
0% Allergic contact dermatitis
0% Microbial eczema
0% Duhring's herpetiform dermatosis

944. A 60-year-old man complains of heart failure, shortness of breath at rest, and pronounced
general weakness. 8 months ago I had a myocardial infarction. Objectively: AT - 80/60 mm Hg,
heart rate-32 / min, PDR-26 / min, cyanosis. ECG: complete atrioventricular block. What tactics of
patient management are most appropriate?
100% Pacemaker implantation

218

Downloaded from the site - online testing step

0% Use of antiarrhythmic drugs


0% Peripheral vasodilators in combination with dopamine
0% Intra-aortic balloon counterpulsation
0% Intravenous administration of prednisone

945. A 10-year-old boy has chronic viral hepatitis B with maximum activity. Which of
the laboratory tests most clearly characterizes the degree of cytolysis in a patient?
100% Transaminases
0% The Veltman Test
0% Takata-Ara Test
0% Prothrombin
0% Total Protein

946. during an internal obstetric examination of a parent, the sacral cavity is completely
filled with the fetal head, and the sciatic spine is not detected. Arrow-shaped seam in straight
size, small crown facing the symphysis. In what plane of the small pelvis is
the pre-lung part of the fetus?
100% Plane of exit from the small pelvis
0% Plane of the broad part of the pelvic cavity
0% Plane of the narrow part of the pelvic cavity
0% Plane of entrance to the small pelvis

https://translate.yandex.com/en/doc 210/540
22:20 ,27.6.2023 �� C : ; 5 B
0% Above the entrance to the pelvis

947. A 22-year-old patient complains 3 days after ARVI of a sore throat behind the sternum, cough with
mucosal sputum. The condition is satisfactory. Percussion: pulmonary sound, auscultation:
hard breathing over the lungs. X-ray of the lungs - no changes. What is the patient's diagnosis?
100% Acute bronchitis
0% Flu
0% Acute laryngitis
0% Basal pneumonia
0% Acute pharyngitis

948.A 12-year-old child was found to have a diffuse enlargement


of the thyroid gland of the second degree during a preventive examination. Auscultation of the heart revealed muffled heart
tones, heart rate-64/min. Has a tendency to constipation. Anemia was detected. Increased levels of
thyroglobulin antibodies. What is the probable cause of these symptoms?
100% Autoimmune thyroiditis
0% Diffuse toxic goiter
0% Thyroid cancer
0% Hyperplasia of the thyroid gland
0% Endemic goiter

949. A 13-year-old patient became acutely ill: thirst, polyuria, and weakness appeared. In 2 weeks I lost
4 kg of weight. Objectively: the general condition is satisfactory, there is no smell of acetone from the mouth. Blood glucose level

219

Downloaded from the site - online testing step

fasting blood-32 mmol / l, urine -6%, acetone +. What are the further treatment tactics?
100% Short-acting insulin
0% Long-acting insulins
0% Biguanides
0% Preparations of sulfonylureas
0% Diet

950. A 24-year-old patient complains of general weakness, dizziness,


fever up to 37.5 ° C, sore throat, neck edema, enlarged submandibular lymph nodes.
Objectively: the oropharyngeal mucosa is swollen and cyanotic, the tonsils are enlarged, covered
with films extending beyond them, and are difficult to remove. What is the main mechanism of development
of this disease?
100% Action of bacterial exotoxin
0% Action of bacterial endotoxin
0% Allergic
0% Accumulation of under-oxidized foods
0% Bacteremia

951. A 40-year-old patient was bitten by a stray dog an hour ago. On the left shin there is a bite mark-
a wound measuring 4x2x0. 5 cm. What surgical treatment is most appropriate in this case?
100% Toilet the wound with soapy water, provision stitches on the wound
0% Aseptic dressing
0% Ointment dressing

https://translate.yandex.com/en/doc 211/540
22:20 ,27.6.2023 �� C : ; 5 B
0% Blind seam
0% Provision stitches for the wound

952.It was observed that a 7-year-old student became inattentive several times during lessons.
The teacher drew attention to the "absent" look of the child, smacking his lips. No falls or
seizures were observed. During his brief "absence", he did not respond to his name. the mother
noticed the same phenomena before, but did not attach any importance to them, considering that the child
was thinking. What is the most likely type of epileptic seizure according to the accepted classification
?
100% Absence
0% Generalized tonic-clonic attack
0% A simple partial seizure
0% Complex partial seizure
0% Jackson's partial seizure

953. a 38-year-old patient has morning stiffness in the whole body, especially in the joints
of the upper and lower extremities, which disappears after active movements in 30-60 minutes, arthritis
of the metacarpophalangeal and proximal phalangeal joints, subfebrile temperature. ESR is 45
mm / hour. Radiologically: osteoporosis and usuration of the articular surface of small
joints of the hand and foot are noted. What is the most likely diagnosis?
100% Rheumatoid arthritis
0% Psoriatic arthropathy

220

Downloaded from the site - online testing step

0% Deforming osteoarthritis
0% Systemic lupus erythematosus
0% Reactive polyarthritis

954. 3 weeks after acute angina, the patient remains weak,frail,


low-grade fever, enlarged extra-maxillary lymph nodes. The tonsils are loose, soldered
to the arches, and there are purulent plugs in the lacunae. What is the most likely diagnosis?
100% Chronic tonsillitis
0% Chronic pharyngitis
0% Acute lacunar sore throat
0% Paratonsillitis
0% Tumor of the tonsils

955. A 40-year-old patient became ill a month ago due to fever


and intoxication, which was followed by shortness of breath and cough with sputum ingress. On
X-ray examination, foci of various sizes were found in the upper parts of the lungs
, sometimes merging. What is the most likely diagnosis?
100% Disseminated pulmonary tuberculosis
0% Bilateral focal pneumonia
0% Lung carcinomatosis
0% Idiopathic fibrosing alveolitis
0% Congestive lung

956. A 7-year-old girl, about 2 months old, complains of poor appetite, upper
abdominal pain, and nausea. The pain is aching, prolonged, without a clear localization, passes at rest. It is not

https://translate.yandex.com/en/doc 212/540
22:20 ,27.6.2023 �� C : ; 5 B
related to food consumption, but occurs more often in the morning. Attempts by the mother to give no-shpu did not
improve the child's condition. What research method will allow you to establish the most likely
diagnosis?
100% Esophagogastroduodenoscopy
0% Fractional study of gastric juice
0% pH-metric of gastric juice
0% Fractional study of bile
0% Ultrasound examination of the abdominal organs

957. a 28-year-old woman experiences abdominal pain of a bursting


nature during menstruation; "chocolate" vaginal discharge. A history of chronic adnexitis. During a bimanual
examination: to the left of the uterus, a tumor-like formation with dimensions of 7x7 cm of uneven
consistency is determined, painful when displaced, and mobile is limited. What is the most likely diagnosis?
100% Endometrioid cyst of the left ovary
0% Follicular cyst of the left ovary
0% Fibromatous nodule
0% Exacerbation of chronic adnexitis
0% Sigmoid colon tumor

221

Downloaded from the site - online testing step

958. a patient complains of pain in the epigastric region of the shingles, nausea,
repeated vomiting, bloating, and increasing weakness. The condition is severe, marked pallor
of the skin, Ps-100 / min., AT-90/50 mm Hg. tongue is dry, overlaid. The abdomen is swollen, but
soft on palpation. There are no symptoms of peritoneal irritation. In the blood: lake.- 26•109/l,
amylase-44 g (h-l). What is the most likely diagnosis?
100% Acute pancreatitis
0% Penetration and bleeding from duodenal ulcer 12
0% Acute cholecystitis
0% Acute mesenteric vascular thrombosis
0% Acute appendicitis

959. a 54-year-old patient suddenly developed severe pain in the epigastric region. Objectively:
forced body position. Pain on palpation of the abdomen over the entire surface, tension
of the abdominal wall muscles. Hepatic dullness is absent. Which diagnostic method should I use to
make a diagnosis?
100% Overview radiography of the abdominal organs
0% Pneumoperitoneography
0% Fibrogastroduodenoscopy
0% Irrigoscopy
0% Contrast radiography of the stomach and duodenum 12

960. the baby was born a day ago. In childbirth, it is difficult to remove the shoulders. Body weight 4300.0.
Objectively: the right arm hangs down along the torso, the hand is pierced, there are no movements in the hand.
Positive scarf symptom. Specify the most likely diagnosis:
100% Total right-sided obstetric paralysis
0% Proximal type of right-sided obstetric paralysis
0% Distal type of right-sided obstetric paralysis
0% Hemiparesis
https://translate.yandex.com/en/doc 213/540
22:20 ,27.6.2023 �� C : ; 5 B
0% Tetraparesis

961. A 42-year-old man came to the hospital for help 10 minutes after being
stung by a bee, complaining of facial swelling and difficulty breathing. Ob-no: Ps-98 / xb.,
AT-130/80 mm Hg. the duty doctor intramuscularly injected the patient with 1 ml of 1%
diphenhydramine solution and recommended that the next day to contact the local therapist. What actions
of the doctor in this situation would be correct?
100% Intravenous administration of prednisone and hospitalization
0% Intravenously introduce a solution of calcium chloride
0% There is no need to provide further assistance to the patient
0% Take them to a hospital for observation
0% Intravenous administration of calcium chloride solution and hospitalization

962. A 44-year-old worker working in the coke industry (16 years of experience)
has a dust concentration 5-10 times higher than the Maximum Permissible concentration (MPC). X-ray examination of the
lungs revealed changes characteristic of pneumoconiosis. What type of pneumoconiosis is most
likely in this case?

222

Downloaded from the site - online testing step

100% Anthracosis
0% Anthracosilicosis
0% Silicatosis
0% Asbestosis
0% Siderose

963. in the classroom of a school, the concentration of CO2 in the air is 0.2%, the light
coefficient is -1: 5, the temperature is 20oC, and the humidity is 55%. General lighting
with incandescent lamps-200 lux. what functional or pathological abnormalities are most likely to occur in students?
100% Reduced working capacity, headache
0% Colds
0% Visual disturbances
0% Scoliosis
0% Vegetative-vascular dystonia

964. A 72-year-old female patient was treated in the urology department for
urolithiasis. After the injection of atropine, severe pain appeared in the left eye and
vision deteriorated sharply. Objectively: visual acuity of the left eye is -0.01, the eye is dense, but painless during
palpation, stagnant injection of the eyeball vessels, the cornea is cloudy. What is the most likely
diagnosis of the left eye?
100% Acute attack of primary glaucoma
0% Acute iridocyclitis
0% Secondary glaucoma
0% Acute keratitis
0% Corneal degeneration

965. A nurse was registered in a antenatal clinic for pregnancy. The delivery was
timely and normal. Twins were born. What is the duration of prenatal and
postnatal leave?

https://translate.yandex.com/en/doc 214/540
22:20 ,27.6.2023 �� C : ; 5 B
100% 70 days before delivery and 70 days after delivery
0% 70 days before delivery and 56 days after delivery
0% 56 days before delivery and 70 days after delivery
0% 56 days before delivery and 56 days after delivery
0% 90 days before delivery and 90 days after delivery

966. at the enterprise where lead meerkat is produced, a team


of specialist doctors is formed to conduct periodic medical examinations. Which doctor should be
included in the team?
100% Neurologist
0% Gynecologist
0% Psychiatrist
0% Dermatologist
0% Otolaryngologist

223

Downloaded from the site - online testing step

967. a 42-year-old male employee of a boiler house. Complaints of shingles of the head, repeated
vomiting. There was a temporary loss of consciousness. Objectively: there is an increase
in tendon reflexes, unexpected myofibrillation. AT - 150/80 mm Hg, Ps-104 / min. Visible
mucous membranes and skin with a crimson tint. Which diagnosis is most likely?
100% Carbon monoxide poisoning
0% Hydrocyanic acid poisoning
0% Poisoning with aniline dyes
0% Methane poisoning
0% Gasoline poisoning

968. The value of the patient's UV radiation bio-dose is measured in minutes. What
device was used to determine the amount of biodose?
100% The Gorbachev biodosimeter
0% UV Meter
0% With an actinometer
0% With a radiometer
0% With a catathermometer

969. A 63-year-old patient complained of lower back pain.


A course of physiotherapy was performed for sciatica. However, the patient's condition did not improve. The patient
underwent Ro-graphics of the bones of the spine and pelvis, on which osteoporosis and significant bone
defects were found. There is moderate normochromic anemia in the blood, and proteinuria in the urine. Total blood protein - 10.7
g/l. what disease should you think about first?
100% Myeloma disease
0% Urolithiasis
0% Acute sciatica
0% Bone metastases
0% Systemic osteoporosis

970.A 20-year-old patient


has a diffuse papular rash of bright red color on the skin of the extensor surface of the limbs, back and scalp
,ranging in size from millet grain to lentils, the surface of the elements of which is covered with whitish
https://translate.yandex.com/en/doc 215/540
22:20 ,27.6.2023 �� C : ; 5 B
scales in the center. Using the scraping method, you can identify the symptom of "stearin stain",
"terminal film" and "blood dew". What disease should I think about?
100% Psoriasis
0% Lichen planus erythematosus
0% Secondary syphilis
0% Papular necrotic tuberculosis of the skin
0% Atopic dermatitis

971. a 35-year-old patient received a laceration of the thigh 3 days ago due to a domestic injury.
I asked for help. She underwent additional skin dissection,excision of dead tissues,
opening of purulent swims, and wound drainage. What kind of operation was performed by the patient?
100% Secondary surgical treatment of the wound
0% Early primary surgical treatment of the wound

224

Downloaded from the site - online testing step

0% Delayed surgical treatment of the wound


0% Late primary surgical treatment of the wound
0% Repeated surgical treatment of the wound

972. A 30-year-old patient is being treated for Verlgoff's disease. Objectively:


pale, has petechial hemorrhages on the extensor surfaces of the forearms. Ps-92 / xb, AT-100/60
mm Hg. lower edge of the spleen at the level of the navel. In the blood: Er.- 2,8•1012/ l, Hb-90 g / l, Ht-38%,
blood clot.- 30•109/L. the patient is preparing for a splenectomy operation. Which transfusion medium should
be chosen first for preoperative preparation?
100% Platelet mass
0% Canned blood
0% Native red blood cell mass
0% Red blood cell curl
0% Washed red blood cells

973. A pervovagit 18-year-old woman underwent surgery for acute


phlegmonous appendicitis at 27-28 weeks of pregnancy. What pregnancy complications should be prevented in
the postoperative period?
100% Miscarriage of pregnancy
0% Intestinal obstruction
0% Fetal hypotrophy
0% Premature placental abruption
0% Late preeclampsia

974. An adult's diet contains 30


g of protein, 37 g of fat, 137 g of carbohydrates, 15 mg of vitamin C, and 0.6 mg of thiamine (vitamin B1) in terms of megacalories
(1000 kcal = 4184 kJ). The diet is not
balanced
100%in content:
Vitamin C
0% Proteins
0% Fat content
0% Carbohydrates
0% Thiamine

https://translate.yandex.com/en/doc 216/540
22:20 ,27.6.2023 �� C : ; 5 B
975. A 39-year-old female patient complains of a tumor on the anterior surface of the neck. I got sick 2
years ago. The tumor is sedentary, has increased in size, the timbre of the voice has changed, there
is a feeling of pressure. Objectively: in the left lobe of the thyroid gland, a node of 3 cm in
diameter, increased density, lumpy, non-painful is palpated. Enlarged lymph nodes of the neck.
The functional state of the thyroid gland is not changed. What is the most likely diagnosis?
100% Thyroid cancer
0% Nodular euthyroid goiter
0% Nodular hyperthyroid goiter
0% Hashimoto's Chronic Lymphatic Thyroiditis
0% Riedel's chronic fibrous thyroiditis

225

Downloaded from the site - online testing step

976. a 3-year-old boy developed a petechial rash. No other pathological changes were
detected. Platelet count- 20•109/l; the content of hemoglobin and white blood cells is normal. What
is the most likely diagnosis?
100% Immune thrombocytopenic purpura
0% Schonlein-Henoch disease
0% Disseminated intravascular coagulopathy
0% Acute lymphoblastic leukemia
0% System red shepherd dog

977. Population health indicators include demographic


indicators. Which of the following is the environment for calculating these metrics?
100% Population size
0% Number of employees
0% Number of people hospitalized
0% Number of patients
0% Number of the population subject to professional inspections

978. the baby was born on time. Birth is abnormal, buttocks forward. When examining a child:
restriction of the right lower limb lead to 50°, positive symptom scales on the right
side. Asymmetry of the inguinal folds. What is the most likely diagnosis?
100% Congenital dislocation of the right hip
0% Congenital hip dislocation
0% Varus deformity of the necks of the thighs
0% Femoral neck fracture
0% Right hip dysplasia

979. A 55-year-old patient was diagnosed with diabetes mellitus during a professional examination. Not treated.
Objectively: height-170 cm, body weight-106 kg, skin of normal humidity. Ps-76 / min, rhythmic,
the left border of relative dullness of the heart is shifted 1 cm to the left of the midclavicular line,
heart tones are weakened, blood pressure is 140/80 mm Hg. fasting blood glucose is 6.9 mmol / l. the glucose content in
daily urine is 0.5% with a diuresis of 2.5 l. what is the main treatment strategy?
100% Assign the patient only diet therapy
0% Prescribe metformin
0% Prescribe Glibenclamide
0% Prescribe repaglinide

https://translate.yandex.com/en/doc 217/540
22:20 ,27.6.2023 �� C : ; 5 B
0% Prescribe insulin

980. medical examination of male drivers revealed an increase in body weight in 10% of individuals
(BMI= 39.3 kg / m2, body weight is 65% higher than normal). What is the energy status of
the Quetelet index?:
100% Grade 3 obesity
0% Energy status is normal (adequate nutrition)
0% 1st degree obesity
0% Grade 2 obesity

226

Downloaded from the site - online testing step

0% Grade 4 obesity

981. A 37-year-old miner experienced lower back pain radiating to his left leg after lifting the load
. The gait is gentle. Flattened lumbar lordosis. Scoliosis by protruding to the left. Tension
of para-vertebral muscles. Positive symptoms of Neri, Dejerina, Lasega's symptom to the left of
the 350-degree angle. Specify the necessary study to clarify the diagnosis:
100% CT scan of the lumbosacral spine
0% Lumbar puncture
0% Kidney sonography
0% Descending myelography
0% Electromyography

982. The incidence of cervical cancer is on the rise in rural medical areas.
It was decided to conduct a survey of women. What is the type of medical examination?
100% Target
0% Previous
0% Current
0% Comprehensive
0% Screening process

983. a patient complains of pain in the lower abdomen, which increases during menstruation and sexual
intercourse, and radiates into the vagina. From the anamnesis: 2 years ago there was a suspicion of endometriosis. During
a vaginal examination, dense, nodular, painful formations are found behind the uterus. What
is the most likely diagnosis?
100% Retrocervical endometriosis
0% Adenomyosis
0% Perimeter
0% Chronic inflammation of the uterine appendages
0% Parametrite

984. A 16-year-old patient underwent surgery 12 years ago for diffuse toxic goiter of iIl-IV
degree. Currently, the patient has experienced a relapse of thyrotoxicosis. The patient
is offered surgery, but before that it is necessary to find out where the functioning gland tissue is located. What
kind of research method should be performed?
100% Breast scan
0% Ultrasound examination of the gland
0% Puncture aspiration biopsy of the gland

https://translate.yandex.com/en/doc 218/540
22:20 ,27.6.2023 �� C : ; 5 B
0% X-ray examination of the neck
0% X-ray examination of the esophagus

985.a 26-year-old patient has a node in the right lobe of the thyroid gland. The node appeared
within the last 3 months. The patient associates its appearance with stress.
The patient does not notice an increase in the node and pain. Ultrasound revealed a 2x2. 5 cm node in the lower right lobe
of the thyroid gland. What treatment should be prescribed?

227

Downloaded from the site - online testing step

100% Surgical treatment


0% Conservative therapy
0% Dynamic supervision
0% Does not require treatment
0% -

986. a newborn with a gestation period of 31 weeks develops lethargy, muscle hypotension, and
depression of consciousness. Analysis of cerebrospinal fluid: increased number of red blood cells, protein and increased
glucose content. What is the most likely diagnosis?
100% Intracranial hemorrhage
0% Meningitis
0% Sepsis
0% Anemia
0% Intrauterine infection

987. a patient complains about the presence of protrusions in the anus, which appear during
defecation and require correction. When viewed with an anoscope above the comb line
, protrusions of the mucosa with a size of 1x1 cm are determined. What is the most likely diagnosis?
100% Internal hemorrhoids
0% Acute paraproctitis
0% External hemorrhoids
0% Anal fissure
0% -

988. A student complains of coughing with the discharge of mucopurulent sputum, sometimes with
blood streaks, fever up to 37.6 ° C, weakness, sweating. Since childhood
, he often suffered from colds. In recent years
, exacerbations of chronic bronchitis have been observed twice a year. The doctor suspected that the patient had a bronchoectal disease.
What
diagnostic
100% method will confirm this assumption?
Bronchography
0% Spirography
0% Physical examination of the lungs
0% Lung radiography
0% Tomography of the lungs

989. a 34-year-old patient was hospitalized for follicular angina with high
fever. Abusing alcoholic beverages for the last 12 years. In the evening on the day of hospitalization
, an alarm appeared, he could not lie down in bed, repeatedly left the ward, addressed the
staff on duty with various complaints. I saw a lot of "spiders on the walls" in the ward, and a large

https://translate.yandex.com/en/doc 219/540
22:20 ,27.6.2023 �� C : ; 5 B
number of "flies". I heard "threats"against myself from the corridor. I was surprised and indignant that other patients
did not
100%hear this.Delusional
I lost my bearings in my surroundings. What is the patient's psychopathological syndrome?

0% Oneiroid
0% Amenity

228

Downloaded from the site - online testing step

0% Twilight disorder of consciousness


0% Asthenic confusion

990. on the 3rd day of life, a child's skin has acquired an icteric color. The baby was born with
a weight of 3,200 kg, body length 52 cm. Active. Puerile breathing over the lungs. PDR-36 / min.
The heart sounds are rhythmic. Heart rate-130 / min. The belly is soft. The liver protrudes from under the edge of the costal arch by
2 cm, the spleen is not palpable. Bowel movements in the form of meconium. What is the most likely
diagnosis?
100% Physiological jaundice
0% Hemolytic disease of newborns
0% Neonatal sepsis
0% Minkowski-Shafar anemia
0% Biliary tract atresia

991. A 40 - year-old patient, 2 weeks after eating lightly fried pork,


which was purchased at a "spontaneous" market, suddenly had a body temperature of up to 40oC,
developed intense muscle pain, facial edema, papular rash on the limbs and trunk,
and a dry cough. I was taking an aspirin. In the blood: lake.- 12,2•109/ l, E. -40%. What disease should
I think about?
100% Trichinosis
0% Ascariasis, migration phase
0% Leptospirosis
0% Teniosis
0% Drug-induced illness

992. A 28-year-old patient complains of enlarged lymph nodes during the last
six months, progressive weakness, sweating, and periodic temperature increases up to 38oC.
He has been using intravenous drugs for several years. Reduced power supply.
Seborrheic dermatitis is widespread, enlarged, painless cervical, axillary and inguinal lymph
nodes up to 2-2. 5 cm are palpated. Which study should be prescribed first?
100% Blood testing for HIV antibodies
0% Bacteriological blood testing
0% Lymph node biopsy
0% Sternal puncture
0% Blood testing for Epstein-Barr virus antibodies

993.in order to study the influence of the microclimate on the human body, it is necessary to organize
systematic monitoring of the air temperature for 3 days. Select the device that
will allow you to record the temperature most accurately:
100% Thermograph
0% Alcohol Thermometer
0% Mercury thermometer
https://translate.yandex.com/en/doc 220/540
22:20 ,27.6.2023 �� C : ; 5 B
0% August Psychrometer
0% Assman Psychrometer
994. a 52-year-old female patient complains of pain in the right side of the chest, shortness of breath, and cough.

229

Downloaded from the site - online testing step

a large amount of foamy sputum in the form of" meat slop", with an unpleasant smell.
Objectively: the condition is severe, cyanosis, PDR-31/min, with percussion, a shortened percussion sound over
the right lung, auscultation-different-sized wet wheezes. What is the most likely
diagnosis?
100% Gangrene of the lung
0% Lung abscess
0% Pleural empyema
0% Bronchiectasis
0% Chronic pneumonia

995. an 8-month-old girl was born prematurely. Suffocation,tachycardia,


hepatosplenomegaly, lag in physical development, cyanosis of the extremities are noted.
Parasternal heart hump is determined,systolic diastolic murmur is heard in the II intercostal space on the left
, blood pressure is 90/0 mm Hg. what disease should I think about?
100% Open Ductus arteriosus
0% Aortic coarctation
0% Aortic valve stenosis
0% Pulmonary artery stenosis
0% Non-closure of the interventricular septum

996. a patient with frostbite on both feet was taken to the emergency department. What
needs to be done?
100% Apply a bandage, enter vasodilators
0% Prescribe heart medications
0% Put your feet in hot water
0% Rub snow on your feet
0% Apply an alcohol compress

997. A 52-year-old patient complains of headache and weakness of the left upper limb. With
physical exertion of the left limb, neurological symptoms increase. Pulsation on
the arteries of the left upper limb is sharply weakened, on the carotid arteries-preserved. What
is the most likely diagnosis?
100% Occlusion of the left subclavian artery,"robbing" syndrome
0% Thoracic outlet syndrome
0% Raynaud's syndrome
0% Takayasu's disease
0% Occlusion of the brachiocephalic trunk

998. A 28-year-old patient complained of an increase in the interval between menstruation to


2 months, hirsutism. During the gynecological examination, it was revealed: the ovaries are enlarged in
size, painless, dense consistency, the uterus is without features. Ultrasound of the pelvic organs:
ovaries measuring 4-5 cm in diameter, with multiple enlarged follicles on the periphery.
Radiography of the base of the skull: the area of the Turkish saddle is expanded. What is the most likely
diagnosis?

https://translate.yandex.com/en/doc 221/540
22:20 ,27.6.2023 �� C : ; 5 B
100% Stein-Leventhal syndrome

230

Downloaded from the site - online testing step

0% Algodismenorrhea
0% Sheehan's syndrome
0% Premenstrual syndrome
0% Morgagni-Stewart syndrome

999. A 40-year-old patient complained of throbbing pain and


swelling of the first finger of the right hand. Examination revealed: uniform edema of the finger, smoothed
interphalangeal furrows. Examination with a button probe is sharply painful along the
flexor tendons. The affected finger is in a state of flexion. What is the patient's diagnosis?
100% Tendon panaritium
0% Subcutaneous panaritium
0% Pandactyl
0% Articular panaritium
0% Bone panaritium

1000. A 35-year-old patient with cholelithiasis experienced an acute


attack of pain in the right hypochondrium after a violation of the diet, which stopped on the 3rd day, but jaundice appeared, which
increases every day. What non-invasive method of diagnosing the disease should be used in the
patient?
100% Endoscopic retrograde cholangiopancreatography
0% Infusion cholecystocholangiography
0% Determination of the level of bilirubin in the blood
0% Duodenal probing
0% Overview radiography of the abdominal organs

1001. A plot of land with a total area of 2.0 hectares has been allocated for the construction of a hospital in the district center
. What is the maximum capacity hospital that can be built on this
land plot?
100% Per 100 beds
0% For 200 beds
0% For 400 beds
0% For 800 beds
0% More than 1000 beds

1002. a 26-year-old woman complains of body edema, swelling and soreness of the mammary glands,
headache, tearfulness, irritability that occur 5 days before menstruation and disappear with its
onset. What clinical syndrome does a woman have?
100% Premenstrual syndrome
0% Post-castration syndrome
0% Adrenogenital syndrome
0% Menopausal syndrome
0% Stein-Leventhal syndrome

1003. A 2-year-old child had upper respiratory tract ARVI for a year in February,

https://translate.yandex.com/en/doc 222/540
22:20 ,27.6.2023 �� C : ; 5 B
231

Downloaded from the site - online testing step

April and October. Last year, the same diagnosis was made in September and December. How
do I register these cases of diseases in a given year?
100% Fill in three statesalons with a ( + )sign
0% Fill in three statesalons with a ( -)sign
0% Fill in one article with a ( + )sign
0% Fill in one article list with a ( + ) sign and two with a ( -)sign
0% Fill in one article with a ( -)sign

1004. in a hospital corridor, a 55-year-old patient suddenly became ill; he was immediately examined by a doctor.
Objectively: the skin is pale, there is no independent breathing, there is no pulse in the carotid arteries
, the pupils are not dilated. What measures should be taken to start resuscitation in order to restore
cardiac activity?
100% Precardial stroke
0% Mouth-to-mouth breathing
0% External heart massage
0% Improvement of airway patency
0% Defibrillation

1005. a 27-year-old patient suffers from hemophilia. Hospitalized with molota, pale skin.
Objectively: Ps-110 / min., AT-100/60 mm Hg in the blood: Hb-80 g / l; Er. -2.8 * 1012/l
. which of the following medications is the priority in this case?
100% Cryoprecipitate
0% Canned blood
0% Red blood cell mass
0% Dicynonum
0% Epsilon-Hydrocaproic acid

1006. A 27-year-old female patient complained about the absence of menstruation for 7
months after the mental trauma caused by the death of her father. From the medical history, it is known that menstruation
began at the age of 13, regular, after 28 days, 5-6 days, painless.
Gynecological examination revealed no changes in the uterus and appendages. What is the most
likely diagnosis?
100% Secondary amenorrhea
0% Primary amenorrhea
0% Algodismenorrhea
0% Oligomenorrhea
0% Fake amenorrhea

1007. A 38-year-old patient complains of frailty, low-grade fever, enlarged lymph


nodes, nosebleeds, and bone pain. Objectively: pallor of the skin and mucous membranes, palpable
enlarged painless lymph nodes, sternalgia, liver +2 cm, spleen +5 cm, not painful. In
the blood: Er.- 2,7•1012/ l, Hb-84 g / l, lake.- 58•109/l, e. - 1%, p. - 2%, p. - 12%, limfoc.- 83%,
lymphoblastic 2%, Botkin-Iumprecht cells; ESR - 57 mm / h. What is the most likely diagnosis?
100% Chronic lymphocytic leukemia
0% Chronic myeloid leukemia

232

https://translate.yandex.com/en/doc 223/540
22:20 ,27.6.2023 �� C : ; 5 B

Downloaded from the site - online testing step

0% Acute lymphocytic leukemia


0% Acute myeloid leukemia
0% Lymphogranulomatosis

1008. A 50-year-old patient has been suffering from chronic obstructive bronchitis for 15 years.
Smokes since the age of 20. Spirographic examination revealed: VEL < 40% of the prescribed value,
Fev < 40%, Tifno index < 40%, MVL <30%. What degree of ventilation disorders
does the patient have?
100% IV
0% II
0% I
0% III
0% 0

1009. a 45-year-old man has shown increased activity over the past two weeks,
has become multilingual, euphoric, has not slept much, has claimed that he can "save humanity", is able
to solve the problem of cancer and acne, and has given money to strangers. What is the most likely
diagnosis?
100% A manic attack
0% Panic disorder
0% Agitated depression
0% Schizoaffective disorder
0% Catatonic arousal

1010. 27-year-old woman in labor, delivery II, urgent, normal. Day 3 of the postpartum period.
Body temperature 36.8 oC, Ps-72 / min, blood pressure-120/80 mm Hg. mammary glands are moderately rough,
nipples are clean. The belly is soft, painless. The bottom of the uterus is 3 transverse fingers below the navel. Lochia
are bloody, moderate. What is the most likely diagnosis?
100% Physiological course of the postpartum period
0% Subinvolution of the uterus
0% Postpartum metroendometritis
0% Remnants of placental tissue after delivery
0% Lactostasis

1011. a 42-year-old patient was hospitalized with complaints of headache and body rash. On
examination: to-40oC, agitated, delirious, face hyperemic, tongue enlarged,
jerky, petechial rash on the trunk and limbs, tachycardia, hypotension,
hepatosplenomegaly. No permanent place of residence, unemployed. What is the most
likely diagnosis?
100% Typhus fever
0% LICE infection
0% Flu
0% Leptospirosis
0% Typhoid fever
1012. a woman in labor, the first day after giving birth. General condition is satisfactory, to-36.6 oC, Ps-80 / min. S

233

Downloaded from the site - online testing step

https://translate.yandex.com/en/doc 224/540
22:20 ,27.6.2023 �� C : ; 5 B

no pathological changes were detected on the part of the internal organs. The mammary glands are slightly enlarged,
there is no hyperemia, and when pressed, a thick, yellow liquid is released from the nipple. What is the nature
of breast secretions?
100% Colostrum
0% Milk
0% Frost
0% Ichor
0% -

1013. a 34-year-old patient complains of profuse night sweats, 9 kg of weight loss in the last 3 months
, and itchy skin. On examination: reduced nutrition, pale skin. On the neck and in the axillary
areas, tightly elastic lymph nodes about 1 cm in diameter are palpated, not soldered to
the skin, sedentary. What is the most likely diagnosis?
100% Lymphogranulomatosis
0% Chronic lymphadenitis
0% Lymphosarcoma
0% Burkit's lymphoma
0% Cancer metastases

1014. on the ECG of a 10-year-old child, a sharp increase in the heart rate - 180/min,
the P wave is layered on the T wave, deforms it, a moderate lengthening of the PQ interval,
the QRS complex is not changed. What is the pathology of the child?
100% Paroxysmal supraventricular tachycardia
0% Paroxysmal ventricular tachycardia
0% Atrial flutter
0% Atrial fibrillation
0% Extrasystole

1015. 26-year-old pregnant woman, pregnancy II, 14-15 weeks. The first pregnancy ended with an abortion at
11-12 weeks. A woman has 1 (0) Rh -, a man has 1 (0) Rh+ blood type. What examinations should
a woman conduct?
100% Determination of antiresus antibodies
0% Determination of group antibodies
0% Coagulogram
0% Biochemical blood test
0% Cordocentesis

1016. Rodilya 34 years old, tenth emergency delivery. From the medical history, it is known that labor began 11
hours ago, labor activity was active, after the discharge of water, painful
attempts began, which became continuous. Suddenly, the woman felt a dagger pain in the lower
abdomen, labor stopped. Positive signs of peritoneal irritation were found,
the contours of the uterus are not clear, the fetus is palpable easily, mobile. The fetal heartbeat is not heard.
What is the most likely diagnosis?
100% Uterine rupture that occurred
0% Weakness of labor activity

234

Downloaded from the site - online testing step

0% Discoordinated labor activity

https://translate.yandex.com/en/doc 225/540
22:20 ,27.6.2023 �� C : ; 5 B
0% Threat of uterine rupture
0% II period of labor

1017. a 22-year-old patient complains of frequent and painful urination, calls to urinate at
night, urinary incontinence, pain in the suprapubic region and in the lower back. Often the urine is the color of beer.
I got married a month ago. Objectively: the general condition is satisfactory. In the lungs-vesicular
respiration. Heart sounds, rhythmic, heart rate-78 / min, AT-128/68 mm Hg. the abdomen is soft, painful in
the suprapubic region. In the urine: Er. - 12-18 in the field of vision, bacteria-12-15 in the field of vision. What is the most
likely diagnosis?
100% Lower urinary tract infection-cystitis
0% Urolithiasis
0% Upper urinary tract infection-pyelonephritis
0% Gonorrhea
0% Primary syphilis

1018. a 46-year-old woman who has been suffering from hypertension for 5 years has developed
a hypertensive crisis. Complaints of palpitation, throbbing sensation in the head, heart rate-100 / min, AT-190/100
mm Hg. which drug should be preferred?
100% beta-blocker
0% ACE Inhibitor
0% Diuretic
0% alpha-adrenoblocker
0% Dihydropyridine calcium antagonist

1019. a 9-year-old boy with acute respiratory viral infection developed polydipsia, polyuria, weakness,
and nausea. On examination: consciousness is confused, the skin is dry, the eyeballs are soft,
Kussmaul's breath, the smell of acetone from the mouth, heart sounds are deaf, the stomach is soft, painless. Blood sugar-19
mmol / l. what is the acute condition that has occurred in the child?
100% Ketoacidotic coma
0% Hyperosmolar coma
0% Cerebral coma
0% Hepatic coma
0% Acute renal failure

1020. A 30-year-old patient complains of intense itching of the skin, mainly at night.
Objectively: he has been ill for 1 week. On the skin of the trunk and abdomen, small, paired
papules, vesicles, combs, crusts are observed. What is the most likely diagnosis?
100% Scabies
0% Urticaria
0% Eczema
0% Allergic dermatitis
0% Neurodermatitis

235

Downloaded from the site - online testing step

1021. a 4-year-old child choked on a walnut a few days ago. Currently


, I am concerned about a cough, fever up to 37.5 oC. On auscultation - multiple dry and wet
wheezes over the right lung. What is the most likely diagnosis?
100% Foreign body of the right bronchus
https://translate.yandex.com/en/doc 226/540
22:20 ,27.6.2023 �� C : ; 5 B
0% Foreign body of the left bronchus
0% Foreign body of the larynx
0% Foreign body of the trachea
0% Foreign body of the pharynx

1022. to assess the child's physical development, the child's body length and weight, annual
increase in body length, chest circumference, number of permanent teeth, secondary sexual
characteristics, vital capacity of the lungs (VEL) were determined, and dynamometry was performed. Which of these
indicators is considered to be physiometric?
100% ZHEL, dynamometry
0% Body length and weight, OGC
0% Secondary sexual characteristics
0% Number of permanent teeth
0% Annual increase in body length

1023. during a professional examination, a 16-year-old patient does not complain. Objectively: low
nutrition, asthenic, blood pressure-110/70 mm Hg, Ps-80 / min, heart borders are normal,
three tones are heard above the apex of the heart during auscultation, no noises are present. ECG-no pathological changes were detected.
PCG-the third tone is detected above the tip 0.15 seconds after the second tone. How can
these changes be interpreted?
100% III-th physiological tone
0% Rhythm of"quail"
0% Proto-diastolic rhythm of "gallop"
0% Presystolic rhythm of the "gallop"
0% ivth physiological tone

1024. A 37-year-old patient was hospitalized in the intensive care unit due to
recurrent convulsive tonic-clonic seizures every half hour.
He doesn't regain consciousness between seizures. AT-120/90 mm Hg, Ps-100 / min. I was at a wedding last night, drinking
alcohol. 5 years ago, he suffered a closed craniocerebral injury, a brain contusion, after
which there were isolated convulsive attacks with loss of consciousness, but the patient
did not undergo antiepileptic treatment. What drug should be administered first for
emergency care?
100% Sibazonum
0% Magnesium Sulfate
0% Sodium oxybutyrate
0% Aminazine
0% Sodium thiopental

1025. during the forensic examination of the corpse, the forensic medical expert described
a linear wound in the temyanoskronny area on the right, measuring 6.4 cm in size with the edges brought together; the edges

236

Downloaded from the site - online testing step

uneven, precipitated, in the depth of the wound tissue membranes are visible. Give a name to the described wound:
100% Clogged
0% Dissected
0% Chopped
0% Knees

https://translate.yandex.com/en/doc 227/540
22:20 ,27.6.2023 �� C : ; 5 B
0% Sliced

1026. A 30-year-old female patient complained of infertility for


5 years at a women's clinic. History of gonorrhea. During the examination: development of the genitals without deviations from
the norm. The basal temperature is biphasic for three cycles. What is the most likely
cause of infertility?
100% Violation of the patency of the fallopian tubes
0% Anomaly of the structure of the genitals
0% Immunological infertility
0% Endometriosis
0% Endocrine factor

1027. A 30-year-old patient complains of infertility for 3 years. In the anamnesis-ectopic


pregnancy (surgical treatment-salpingectomy on the left 1 year ago) and a cyst of the right ovary
(surgical treatment-adnexectomy on the right side 2 years ago). The man was examined,
no pathology was detected. What infertility treatment tactics are indicated for this married couple?
100% In Vitro fertilization
0% Laparoscopy
0% Hysteroscopy
0% Hydrotubation
0% Artificial insemination with donor sperm

1028. A 50-year-old patient has been suffering from hypertension for 20 years. Within 2 days, he notes
deterioration of the condition: headache, nausea, dizziness, facial edema, shortness of breath with
minor loads, which he associates with excessive consumption of salty food and interruption
of the course of prescribed antihypertensive therapy. Volume: AT-180/120 mm Hg, heart rate-88 / min,
HR-24 / min, weakening of respiration in the posterobasal parts of the lungs. Patient care in
the hospital should begin with:
100% Intravenous administration of 80 mg of furosemide
0% Intravenous administration of 2 ml of Seduxen
0% I / O drip input of an isoket
0% Sublingual administration of 10 mg of Corinfar
0% Intravenous administration of labetalol

1029. a patient went to the clinic complaining of weight gain, chilliness, swelling,
dry skin, drowsiness, difficulty concentrating. Objectively: height 165 cm, body weight 90 kg,
female body proportions, to-35.8 oC, heart rate-58 / min, AT-105/60 mm Hg. heart tones are weakened,
bradycardia. Other internal organs are unchanged. The thyroid gland is not palpable.
There is a release of drops of milk from the mammary glands. Hormone testing
revealed an increase in TSH and pro-Lactin levels, and a decrease in T4. Which for various reasons led to

237

Downloaded from the site - online testing step

formation of obesity?
100% Primary hypothyroidism
0% Secondary hypothyroidism
0% Prolactinoma
0% Hypopituitarism
0% Adipozo-genital dystrophy

https://translate.yandex.com/en/doc 228/540
22:20 ,27.6.2023 �� C : ; 5 B
1030. a patient was admitted to the surgical department from the scene of an accident with a closed
chest injury and broken ribs on the right side. The patient was diagnosed with right-sided
pneumothorax. The patient is urgently shown drainage of the pleural cavity. Specify the location
of the pleural puncture:
100% In the 2nd intercostal space along the midclavicular line
0% In the 6th intercostal space along the posterior cingulate line
0% In the 7th intercostal space along the scapular line
0% In the projection of the pleural sinus
0% At the point of greatest dullness determined during percussion

1031. a 65-year-old man developed acute pain, paresthesia, and pallor of the left lower limb.
Objectively: there is no pulse on A. dorsalis pedis on the left side. There is a cooling of the skin
and pallor, which gradually spread up the limb. These symptoms
are most likely to indicate that:
100% Arterial occlusion
0% Superficial vein thrombophlebitis
0% Herniated lumbar vertebral disc
0% Deep vein thrombophlebitis
0% -

1032. A 23-year-old woman suffering from insulin-dependent diabetes mellitus was taken to the
emergency department with confused consciousness, inappropriate fussy
behavior, sweating, increased salivation, and tachycardia. What kind of research
should be done first?
100% Blood sugar test
0% General blood test
0% Plasma electrolytes
0% Gas composition of arterial blood
0% Blood urea and creatinine

1033. you can think about the diagnosis of hepatitis in a pregnant woman with an increase in the level of:
100% ALT
0% SHZ
0% White blood cells
0% Alkaline Phosphatase
0% Urea Nitrogen

238

Downloaded from the site - online testing step

1034. A 20-year-old patient who was delivered from the street in winter with brachial
artery bleeding was put on a tourniquet during first aid to temporarily stop
the bleeding. Specify the maximum exposure of the harness:
100% 60 minutes
0% 15 minutes
0% 30 minutes
0% 120 minutes
0% 180 minutes

https://translate.yandex.com/en/doc 229/540
22:20 ,27.6.2023 �� C : ; 5 B
1035. the occurrence of preeclampsia at 16 weeks of pregnancy may be due to:
100% Hydatid drift
0% Anencephaly
0% Twins
0% Kidney disease in the mother
0% The presence of an interventricular defect in the fetus

1036. Endometrial adenocarcinoma that has grown into the serous membrane of the bladder should be
classified as:
100% IIIA
0% IC
0% IIA
0% IIB
0% IVAB

1037.during a routine examination of a child suffering from bronchial asthma, an


increase in blood pressure up to 140/90 mm Hg was detected. what is the possible cause of arterial
hypertension in a child?
100% Kidney diseases
0% Overdose of theophylline
0% Chronological lung disease
0% Aortic coarctation
0% Fatness

1038. A 6-week-old child was taken to the hospital for shortness of breath. The delivery was
uneventful, although on day 3 the baby developed conjunctivitis, which lasted 2 weeks.
Examination revealed shortness of breath, bilateral crepitating wheezes in the lungs, and isolated dry wheezes.
Radiologically confirmed bilateral pneumonia. The body temperature is normal and previously its
increase was not noted. In the blood: lake.- 15•109/l, E. -28%. What causes
the resulting symptom complex?
100% Clamydia trachomatis
0% Pneumocystis carinii
0% Mycoplasma pneumoniae
0% Parasitic mea3in
0% Vtryanaya vkpa

239

Downloaded from the site - online testing step

1039. a child with a Fallot notebook has a place:


100% Increased pressure in the right ventricle
0% Increased blood circulation in the small circulatory system
0% Increased pulse pressure
0% Normal pressure gradient on the pulmonary valve
0% Normal oxygen saturation (PaO2) in the left ventricle

1040. a 48 - year-old man complains of weakness and shortness of breath. When examining the blood-
Ht32%, Hb-103 g / l, macrocytes are detected in the smear. The level of vitamin B12 in blood plasma is 90 pg
(normal - 170-940), the level of folic acid in serum is 6 ngr/ml (normal - 2-14). Possible
reasons for the development of these changes can be all of the following, except:
100% Diverticulosis of the colon
https://translate.yandex.com/en/doc 230/540
22:20 ,27.6.2023 �� C : ; 5 B

0% Vegetarianism
0% Regional enteritis
0% Pancreatitis
0% Infection with worms

1041. a 56-year-old male complains of weakness, shortness of breath during


exercise, and palpitations. Since childhood, a heart murmur is heard. Objectively:
expansion of the right border of the heart, splitting of the II tone, systolic murmur in the II-III intercostal space to the left
of the sternum edge and a short systolic murmur above the xiphoid process. On
the X-ray: enlargement of the right ventricle and the arch of the pulmonary artery. ECG: atrial
fibrillation and right bundle branch block. What is the most likely diagnosis?
100% Atrial septal defect
0% Aortic coarctation
0% Open Ductus arteriosus
0% Fallot's Tetrad
0% Ventricular septal defect

1042. a decrease in the voltage of the QRS complex in all leads can be caused by all
of the listed reasons, except:
100% Hyperthyroidism
0% Presence of fluid in the pericardial cavity
0% Cardiotransplant rejection
0% Amyloidosis
0% IXC

1043. 5 days after hip joint transplantation, a 72-year-old woman suddenly


developed an attack of suffocation, hypotension, and sweating. Increased filling and
pulsation of the neck veins is noted. No slight changes were detected during auscultation and percussion. Your heart sounds
normal. On the ECG: sinus tachycardia, first-time block of the right bundle
branch, non-specific changes in the S-T interval and T wave. What is the most likely diagnosis?
100% Pulmonary embolism
0% Acute myocardial infarction
0% Dissecting aortic aneurysm

240

Downloaded from the site - online testing step

0% Pericarditis
0% Aspiration

1044. which of the listed causes of acute renal failure is


considered post-renal?
100% Urolithiasis
0% Heart failure
0% Septicemia
0% Rhabdomyolysis
0% Acute glomerulonephritis

1045. which of the following is not TYPICAL for Adison's disease?


100% Hypernatremia

https://translate.yandex.com/en/doc 231/540
22:20 ,27.6.2023 �� C : ; 5 B
0% Hyperkalemia
0% Increased urea nitrogen levels
0% Low osmolarity of urine
0% Increased hematocrit

1046. a 40-year-old woman has a dense formation in the breast area. Which of
the listed diagnostic methods will be the most informative in establishing the diagnosis?
100% Histological examination of the biopsy sample
0% Mammography
0% Thermography
0% Ultrasound examination
0% Aspiration biopsy with cytology

1047. a patient who was on parenteral nutrition after a serious injury


developed diarrhea, depression, alopecia, and dermatitis around the mouth and eyes. Which of
the following elements is necessary to correct the changes that have occurred?
100% Zinc
0% Iodine
0% Selenium
0% Silicon
0% Copper

1048. a 40-year-old male abuses alcohol. Expresses complaints of persistent pain in


the epigastric region. During the examination, changes in the pancreatic duct were found-
multiple areas of narrowing followed by expansion. Which of these operations
is most suitable for the patient?
100% Lateral pancreatojejunostomy
0% Distal pancreatojejunostomy
0% Sphincterotomy
0% Distal pancreatectomy

241

Downloaded from the site - online testing step

0% Total pancreatectomy

1049. treatment of choice for duodenal obstruction caused by a secondary duodenal


hematoma that developed a few days after blunt abdominal trauma:
100% Nasogastric decompression and parenteral nutrition
0% Retro-intestinal gastrojunostomy
0% Duodenoejunostomy
0% Immediate revision
0% Duodenostomy formation

1050. a 25-year-old man suddenly developed acute pain in the right side of his chest and
shortness of breath. Objectively: the trachea is deviated to the left. All of the above can take place, except:
100% Left pleural friction noise
0% No dry wheezing
0% No wet wheezing

https://translate.yandex.com/en/doc 232/540
22:20 ,27.6.2023 �� C : ; 5 B
0% Increased vocal tremor on the right side
0% Remote dry wheezes

1051. a patient complains of severe shortness of breath, which increases with physical exertion.
Complaints suddenly appeared 2 hours ago at work: sharp pain in the left chest, cough. Over
time, the pain decreased, but shortness of breath, dizziness, pallor, cold sweat, and cyanosis increased.
There is no vesicular respiration. Radiologically-darkening on the left side. What pathology
can be suspected?
100% Spontaneous left-sided pneumothorax
0% Lung infarction
0% Pleurisy
0% Left-sided pneumonia
0% Lung abscess

1052. in a 62-year-old patient who smokes and often suffers from "pneumonia", a survey X
-ray of the chest revealed a triangular darkening in the right lung with the apex
directed towards the lung root, as well as a displacement of the shadow of the heart and mediastinum towards the lesion. What
is the most likely diagnosis?
100% Central lung cancer
0% Lung abscess
0% Lung cyst
0% Peripheral lung cancer
0% Atelectasis of the lung

1053. the patient complains of a feeling of heaviness behind the sternum and a periodic sensation
of food stopping, dysphagia. During X-ray examination, barium contrast reveals a single
sac-like protrusion of the right anterior wall of the esophagus with smooth contours and a clearly
defined neck. What is the most likely diagnosis?
100% Esophageal diverticulum

242

Downloaded from the site - online testing step

0% Esophageal cancer
0% Hiatal hernia
0% Esophageal varicose veins
0% Esophageal polyp

1054. an 8 - year-old child complains of headache and vomiting. Ill for 1 day. Objectively:
t039, 30C, clear skin, hyperemic pharynx, overlaid tongue, conjunctivitis, scleritis. Weakly
expressed rigidity of the muscles of the occiput. Positive Kernig symptom on the right side. Li-kvor:
cytosis-340, lymph.- 87%, neutrophil -13%, protein -140 mg / l. what is the most likely diagnosis?
100% Serous meningitis
0% Arachnoiditis
0% Neurotoxicosis
0% Brain tumor
0% Purulent meningitis

1055. A typhoid patient suddenly developed abdominal pain on the 14th day of her illness. The patient
is conscious. The pain is acute. The abdomen is involved in the act of breathing, not deflated. Palpation: moderate

https://translate.yandex.com/en/doc 233/540
22:20 ,27.6.2023 �� C : ; 5 B
muscle tension of the anterior abdominal wall, more in the right iliac region. Hepatic
dullness is not detected. Peristalsis is audible, flabby. The Shchetkin-Blumberg symptom
is positive. Body temperature 38.90 C. Ps-104 / min. What is the most likely diagnosis?
100% Perforation of a typhoid ulcer
0% Adhesive intestinal obstruction
0% Acute perforated appendicitis
0% Perforation of acute gastric ulcer
0% Acute pancreatitis

1056. a patient came to the hospital complaining of a violation of pain and tactile sensitivity,
pain in the end phalanges of the fingers that occurs after work. At the enterprise, he works with
mechanical devices. What pathology should be suspected in this case?
100% Vibration sickness
0% Caisson disease
0% Noise sickness
0% Signs of overwork
0% Vitamin B1 hypovitaminosis

1057. A 25-year-old female patient complains of weakness, dizziness, and hemorrhagic skin rashes.
He has been ill for a month. In the blood: Er.- 1,0•1012/ l, Hb-37 g / l, CP-0.9, lake.- 1,2•109/l,
tr. 42 * 109/l. which research method will be most informative for making a diagnosis?
100% Sternal puncture
0% Spleen biopsy
0% Liver biopsy
0% Coagulogram
0% Ultrasound of the abdominal cavity

243

Downloaded from the site - online testing step

1058. a 28-year-old woman at the second birth had a girl weighing 3,400 g,
52 cm long with manifestations of anemia and increasing jaundice. Blood type B (III) Rh-in the woman
, A (III) Rh+ in the father of the newborn, and B (III) Rh+in the newborn. What is the cause of anemia?
100% Rhesus conflict
0% Conflict by antigen a
0% Conflict by antigen in
0% AB antigen conflict
0% Intrauterine infection

1059. a 68-year-old patient, an employee. About two years ago, memory disorders began:
he became forgetful at work and in everyday life, and could not cope with his official duties. Gradually, he began
to forget the names of relatives, the names of objects, became helpless, speech was disrupted. I lost
my writing, reading, and self-care skills. Computed tomography revealed atrophy
of the cerebral cortex. What is the most likely diagnosis?
100% Alzheimer's disease
0% Pick's disease
0% Atherosclerotic dementia
0% Senile dementia
0% Progressive paralysis

https://translate.yandex.com/en/doc 234/540
22:20 ,27.6.2023 �� C : ; 5 B

1060. a 64-year-old patient developed compressive pain behind the sternum 2 hours ago, radiating to the
left shoulder, and pronounced weakness. Objectively: the skin is pale, cold sweat. Ps-108 / min., AT-70/50 mm
Hg. heart tones are deaf. Respiration is vesicular. The belly is soft, painless.
Varicose veins on the left shin. On the ECG: sinus rhythm, heart rate-100/min, sharp rise
of the ST segment above the isoline in leads II, III, aVF. What pathology did the patient have?
100% Cardiogenic shock
0% Cardiac asthma
0% Pulmonary embolism
0% Dissecting aortic aneurysm
0% Cardiac tamponade

1061. when studying the actual nutrition of an adult, it was found that the share
of energy value of the daily diet due to protein is 16%, fat is 25%, and hydrocarbons are 59%. Evaluate
the compliance of protein, fat, and hydrocarbon content with their recommended particles in
the energy value of the diet:
100% The proportion of carbohydrates in the diet is insufficient, an excess of proteins
0% The proportion of fat in the diet is insufficient
0% The proportion of hydrocarbons in the diet is insufficient
0% The proportion of hydrocarbons in the diet is excessive
0% The content of nutrients corresponds to the recommended proportions of energy value

1062. A worker of a chemical plant was found


to have a malignant neoplasm of the bladder during a periodic medical examination. Contact with what industrial poison is most
likely to have caused this occupational disease?
100% Benzidine

244

Downloaded from the site - online testing step

0% Vinyl Chloride
0% Nickel carbonyl
0% Asbestos
0% Arsenic

1063. A 52-year-old patient was admitted to the hospital with complaints of increased bleeding
of the mucous membranes, significant skin hemorrhages in the form of ecchymoses, spots, nasal and gastric
bleeding. After clinical examinations, the diagnosis was established: thrombocytopenic purpura.
What is the most likely cause of this disease?
100% Formation of antiplatelet autoantibodies
0% Hemostatic disorders
0% Blood coagulation factor VIII deficiency
0% Hereditary insufficiency of plasma coagulation factors
0% Iron deficiency in serum, bone marrow and depot

1064. a man in serious condition was taken to the hospital's emergency department on the second day of his illness
. During the examination, the body temperature is 36.10 C, the facial features are pointed, the skin is dry,
gathers in a fold, aphonia, convulsive twitching of individual muscle groups. Acrocyanosis.
Heart sounds are muffled, Ps-102 / min, BP-50/20 mm Hg. the abdomen is soft, retracted, painless. Anuria. Liquid
bowel movements in the form of "rice broth". What is the most likely diagnosis?
100% Cholera
0% Acute dysentery
https://translate.yandex.com/en/doc 235/540
22:20 ,27.6.2023 �� C : ; 5 B
0% Salmonellosis
0% Escherichiosis
0% Intestinal amoebiasis

1065. in a full-term child of 3 days of life,erythema,


erosive surfaces, cracks, and peeling of the epidermis are found on various skin areas. The baby looks like it's been scalded with boiling
water.
A positive symptom of Nikolsky was detected. The child's general condition is serious. Severe
anxiety,
100% hyperesthesia, febrile
Exfoliative fever. What is the most likely diagnosis?
dermatitis
0% Phlegmon of the newborn
0% Figner's pseudofurunculosis
0% Newborn's pemphigus
0% Mycotic erythema

1066.During the year, 11,600 cases were registered in the polyclinic. Among them: flu and
acute respiratory infections - 5800, circulatory system diseases-3480, digestive diseases-1300,
other diseases-1020. what relative metric can be calculated from this data?
100% Extensive
0% Intensive
0% For visibility purposes
0% Ratio
0% -

245

Downloaded from the site - online testing step

1067. a patient with left lung mandibular pneumonia complained of pain in the left
side of the chest. Objectively: a wide area of bluntness on the left side
of the chest (Sokolov-Alison-Damoiseau line) is determined. What method of lung examination is advisable
to start an examination to clarify the diagnosis?
100% Radiography
0% Bronchoscopy with biopsy
0% Bronchography
0% Spirography
0% Thoracoscopy

1068. a 35-year-old woman went to the doctor with a lesion of the skin of the hands and lower third
of the forearm in the form of significant edema, hyperemia, vesiculation, and weeping. The disease
developed after using washing powder for washing clothes, which the patient
uses for a month. Previously, there were no dermatological diseases. What is the most
likely diagnosis?
100% Allergic dermatitis
0% Simple dermatitis
0% Toxic allergic dermatitis
0% Microbial eczema
0% Limited neurodermatitis

1069. a 64-year-old patient complains of constant dull pain in the right swollen area,
losing 20 kg of weight in 6 months, persistent constipation, and lack of appetite. Objectively: the skin
is earthy in color, its turgor is lowered. In the right iliac region, an infiltrate

https://translate.yandex.com/en/doc 236/540
22:20 ,27.6.2023 �� C : ; 5 B
of 6x8 cm is palpated, dense, little mobile, not painful. Free fluid in the abdominal cavity is not
detected. During auscultation-increased intestinal noise. In the blood: Hb-80 g / l
. the Gregersen reaction is positive. What is the most likely diagnosis?
100% Colorectal cancer
0% Cancer of the right kidney
0% Appendicular infiltrate
0% Retroperitoneal tumor
0% Malignant tumor of the small intestine

1070. a 32-year-old man complains of heartburn and aching pain in the epigastric body 2-3 hours
after eating. Exacerbation occurs in spring and autumn. Food intolerance to eggs and fish.
Objectively: palpation of the abdomen shows pain in the gastroduodenal region. EFGDS: 5
mm ulcer on the anterior wall of the duodenum. Positive urease test. What is the most
likely leading mechanism for the development of the disease?
100% Helicobacter pylori infection
0% Food allergies
0% Autoantibody production
0% Reduced prostaglandin synthesis
0% Violation of gastric motility

1071. a 48-year-old woman complains of weakness, weight loss, decreased appetite, and headache.

246

Downloaded from the site - online testing step

In her youth, she suffered from acute glomerulonephritis. He has been suffering from arterial hypertension since the age of 25
. She was not treated systematically, and rarely went to the doctor. After the
study, signs of chronic renal failure of the first stage were revealed. (creatinine - 0.43 mmol / l).
What dietary recommendations are most appropriate for this patient?
100% Protein Restriction
0% Fat restriction
0% Restriction of hydrocarbons
0% Eating food with a high content of "alkaline" dishes
0% Increasing the volume of liquid

1072. A 50-year-old patient complains of dull pain in the heart, suffocation,


and fever up to 380C. I had the flu a week ago. Objectively: Ps-100 / min, disappears on inspiration.
AT-100/70 mm Hg, heart tones are deaf. ECG: low voltage, ST segment raised above
the isoline in all leads. On the X-ray: the shadow of the heart is extended in all directions.
Low-amplitude heart pulsation. What is the most likely diagnosis?
100% Exudative pericarditis
0% Myocardial infarction
0% Dilated cardiomyopathy
0% Myocarditis
0% Angina pectoris

1073. when examining a child aged 4 months


, lemon-yellow scales with greasy crusts were found on the scalp. What is the most likely diagnosis?
100% Gneiss
0% Milk scab
0% Strofulus

https://translate.yandex.com/en/doc 237/540
22:20 ,27.6.2023 �� C : ; 5 B
0% Pseudofurunculosis
0% Children's eczema

1074. A 39-year-old patient went to the doctor complaining of morning headache, decreased
appetite, nausea, morning vomiting, and recurrent nosebleeds. At the age of 15, he suffered
from acute glomerulonephritis. During the examination, there was an increase in blood pressure to
220/130 mm Hg, hemorrhages on the skin of the hands and feet, pallor of the skin and mucous membranes. Which of
the above biochemical parameters is of diagnostic significance in this case?
100% Blood Creatinine
0% Blood bilirubin
0% Blood Sodium
0% Uric acid
0% Fibrinogen

1075. a 42-year-old patient has fever attacks every 48 hours for a week,
followed by fever. Body temperature rises to 400C. A decrease in temperature after 3-4
hours is accompanied by excessive sweating. My appetite worsened, and I felt generally
weak. The skin is pale, with an earthy tinge. Enlarged liver and spleen,
palpation-dense. What is the most effective method for verifying a diagnosis?

247

Downloaded from the site - online testing step

100% Microscopy of a blood smear and a thick drop


0% General blood test
0% Bacteriological method
0% Enzyme-linked immunosorbent assay
0% Microscopy of a hanging drop of blood

1076. a 21-year-old patient, without a permanent place of residence, complains of loose stools
for 2 months, weight loss of 13 kg, weakness, constant subfebrile temperature,
recurrent herpes. Objectively: herpetic rashes on the lips, generalized
lymphadenopathy, liver enlargement up to 2 cm. In the blood: Er.- 4,4•1012/ l, Hb-115 g / l, WSE-15
mm / year, lake.- 10,0•109/l, E.-2%, P.-6%, S.-61%, L.-17%, M.-3%. Atypical mononuclears-6%.
What is the most likely diagnosis?
100% AIDS
0% Prolonged dysentery
0% Infectious mononucleosis
0% Lymphogranulomatosis
0% Amoebiasis

1077. A 58-year-old patient complains of weakness, leg swelling, shortness of breath, and anorexia.
He has been suffering from chronic bronchitis for many years. For the last 5 years, he has been noticing increased
sputum discharge, often of a purulent nature. Objectively: heart rate-80 / min, AT-120/80 mm Hg,
widespread edema, pale, dry skin, reduced turgor. In the urine: significant proteinuria,
cylindruria. Specify the most likely pathological process in the kidneys:
100% Amyloidosis of the kidneys
0% Chronic glomerulonephritis
0% Chronic pyelonephritis
0% Interstitial nephritis
0% Acute glomerulonephritis

https://translate.yandex.com/en/doc 238/540
22:20 ,27.6.2023 �� C : ; 5 B

1078. A 23-year-old male complains of facial edema, headache,


dizziness, decreased urine output, and a change in the color of urine (dark red). These
complaints appeared after pharyngitis. Objectively: edema on the face,
pale skin, temperature-37.40 C; heart rate-86 / min, blood pressure-170/110 mm Hg. heart tones are muted,
accent II tone over the aorta. What etiological factor is possible in this disease?
100% beta-hemolytic streptococcus
0% Staphylococcus aureus
0% Greenish streptococcus
0% Pyogenic streptococcus
0% Saprophytic staphylococcus

1079. A 3-month-old child


developed jaundice, hepatosplenomegaly, dark yellow urine, and fecal discoloration after several days of anxiety, anorexia, and
subfebrility. At
the age
100%of 1 month,
Viralthere wereBblood transfusions. What is the most likely diagnosis?
hepatitis
0% Hemolytic anemia

248

Downloaded from the site - online testing step

0% Viral hepatitis A
0% Biliary tract atresia
0% Conjugation jaundice

1080. a 4-year-old boy was admitted to the hospital with complaints of shortness of breath, rapid
fatigue. A history of frequent respiratory diseases. Percussion: the heart borders
are extended to the left side and up. Auscultation: amplification of the II tone above the pulmonary artery, in
the III intercostal space to the left of the sternum, a rough systolic
"machine"noise is heard, conducted to all other points and to the back. AT-100/20 mmHg. what
is the most likely diagnosis?
100% Open Ductus arteriosus
0% Ventricular septal defect
0% Isolated stenosis of the pulmonary artery opening
0% Atrial septal defect
0% Valvular aortic stenosis

1081. a 22-year-old woman was hospitalized in the gynecological department with complaints of pain in the lower
abdomen, fever up to 39.50 C. Objectively: heart rate-108 / min, AT-120/80 mm Hg,
the abdomen is moderately swollen, sharply painful in the hypogastric region. Shchetkin's symptom is positive in
the hypogastric region. Vaginal examination: the uterus and appendices are not palpable
due to the tension of the anterior abdominal wall, the posterior vault of the vagina hangs, sharply painful. What
is the most likely diagnosis?
100% Pelvioperitonitis
0% Acute adnexitis
0% Acute endometritis
0% Ectopic pregnancy
0% Ovarian apoplexy

1082. it was found that for every 100 births in women with risk factors, 30 were premature,
and in women without risk factors,5 were premature. What method of statistical

https://translate.yandex.com/en/doc 239/540
22:20 ,27.6.2023 �� C : ; 5 B
data processing is optimal for a doctor to use to estimate the probability of discrepancies in
the compared groups?
100% Calculation of Student's criterion
0% Calculation of average values
0% Calculation of relative values
0% Standardization method
0% Correlation analysis

1083. a 12-year-old girl complains of severe weakness, nausea, dizziness,


and visual impairment. The day before, I ate dried fish and home-made beef. On examination: pallor
of the skin, scratch of the left knee, dryness of the mucous membranes of the oropharynx, bilateral ptosis, pupils
dilated, unable to read simple text ("grid", " fog " in front of the eyes). What therapy will
be most appropriate in this case?
100% Parenteral administration of polyvalent anti-botulinum serum
0% Parenteral detoxification

249

Downloaded from the site - online testing step

0% Parenteral administration of antibiotics


0% Gastric lavage
0% Parenteral administration of tetanus serum

1084. a 32-year-old patient was hit by a car. Complaints of pain in the lower abdomen above
the pubis with radiation to the perineum, rectum, frequent, difficult and painful urination in small
portions, blood impurities in the urine. Objectively: tension over the pubis is determined, percussion
-bluntness extending to the iliac region. What is the most likely diagnosis?
100% Extraperitoneal rupture of the bladder
0% Kidney rupture
0% Urethral rupture
0% Ureteral rupture
0% Kidney injury

1085. A 26-year-old woman who gave birth 7 months ago


has been experiencing nausea, morning vomiting, and drowsiness for the past two weeks. Breast-feeding, no menstruation.
I wasn't warned against pregnancy. Which of the methods should be used to clarify the diagnosis?
100% Ultrasound examination
0% Ro-graph of pelvic organs
0% Palpation of mammary glands and milk extraction
0% Two-handed vaginal examination
0% Research using mirrors

1086. a 25-year-old worker found himself in the zone of spontaneous combustion of coal in bunkers, with the release
of large concentrations of carbon monoxide. Delivered unconscious. The contact
is unavailable. It does not respond to a needle prick. The skin is pink in color. Ps-110 / xb,
AT110 / 60 mm Hg right-sided pyramidal insufficiency is registered. What is the most
appropriate treatment?
100% Hyperbaric oxygenation
0% Hormone Therapy
0% Lumbar puncture
0% Blood transfusion

https://translate.yandex.com/en/doc 240/540
22:20 ,27.6.2023 �� C : ; 5 B
0% Hemosorption

1087. a 2-year-old girl has a history of recurrent pneumonia with obstructive symptoms.
In the lungs, various wet and dry wheezes are heard, breathing is weakened. With
difficulty, a viscous secret is expectorated. The presence of "drumsticks",
a lag in physical development was noted. What is the most likely diagnosis for this child?
100% Cystic fibrosis, a pulmonary form
0% Recurrent bronchitis
0% Bronchial asthma
0% Congenital polycystic lung disease
0% Tuberculosis of the lungs

250

Downloaded from the site - online testing step

1088. a patient was admitted to the cardiology department with complaints of aching pain in
the atrial region, moderate shortness of breath, swelling of the legs, and chilliness. On the ECG: negative
T waves V2-V6 (up to 34 mm). Objectively: the skin is dry, the voice is hoarse, the face is puffy, with
an icteric tinge, heart tones are deaf, Ps-60 / min, blood pressure-160/90 mm Hg, the lower limbs
are swollen, when pressing on the skin of the shins, a fossa is formed with difficulty. In the blood:
Hb76 g / l, WSE-17 mm / h. What is the most likely diagnosis?
100% Hypothyroidism, myocardial dystrophy
0% Chronic glomerulonephritis, CRF
0% IHD: small-focal myocardial infarction, HF II a st
0% Hypertension II century, HF II a st
0% B12 - deficiency anemia

1089. a 16-day-old child, due to the mother's hypogalactia, needs to be prescribed do-feed. What formula
should be prescribed to your child?
100% "Malyutka "
0% "Baby"
0% Kefir
0% Whole cow's milk
0% Acidophilic milk

1090. a 46-year-old patient complains of paroxysmal pain in the right lumbar region
radiating to the lower abdomen, nausea. Previously, such pain was not observed. On
the X-ray survey of the abdominal organs, no pathological shadows are detected. On
the ultrasound sonogram, in the enlarged right renal pelvis, a hyperechoic
formation with a diameter of about 1.5 cm is determined, from which the "ultrasound track" departs. What
is the most likely diagnosis?
100% Kidney stone
0% Kidney tumor is benign
0% Cyst in the kidney
0% Tuberculosis of the kidney
0% Kidney tumor is malignant

1091. a milkmaid has a 3x3 cm ulcer on her right hand, covered with a dense black scab.
The brush is sharply swollen, painless. The skin color in the area of edema is not changed. Enlarged

https://translate.yandex.com/en/doc 241/540
22:20 ,27.6.2023 �� C : ; 5 B
axillary lymph node on the right side. Palpation of the lymph node is painless. Symptoms of intoxication
are moderate.
100% What is the most
Cutaneous formlikely diagnosis?
of anthrax
0% Bubonic skin form of plague
0% Ulcerative-bubonic form of tularemia
0% Banal carbuncle
0% Erysipelas

1092. a 32-year-old patient complains of pain in the small joints of the hands, paresthesia in
the fingertips, weakness, difficulty swallowing. He has been ill for 13 years. Objectively:
facial expression, shortening of the nail phalanges. Compaction of the skin in the shoulder area

251

Downloaded from the site - online testing step

belts. In the lungs radiologically basal pneumosclerosis. With FGS-narrowing of the esophagus in
the cardiac department. In the blood: lake.- 9,8•109/ l, SSE-22 mm / h, y-globulin-22%. What
is the most likely diagnosis?
100% Systemic scleroderma
0% System red shepherd dog
0% Rheumatoid arthritis
0% Dermatomyositis
0% Myxedema

1093. The information and analytical department of the CRH should analyze the demographic situation in
the district. The corresponding tasks were distributed among the employees of the department. In particular
, one of them was assigned to study the birth rate of the population. What accounting documents
should serve as a source of information for them?
100% Medical birth certificate
0% History of newborn development
0% Exchange card: information about the newborn
0% Birth history
0% Birth certificate

1094. a 30-year-old woman with flu developed shortness of breath during moderate physical
exertion, palpitations, and aching pain in the heart area. Objectively: Ps-96 / min, BP-100/60 mm
Hg. Above the tip of the I tone is a weak, soft systolic murmur. What complication
does this clinical picture indicate?
100% Acute viral myocarditis
0% Acute infectious and allergic myocarditis
0% Idiopathic myocarditis
0% Myocardiopathy
0% Neurocirculatory dystonia

1095. a 6-month-old child develops jaundice with a greenish tinge from birth.
There are phenomena of hemorrhagic diathesis and pruritus. What is the most likely pathology
in a child?
100% Biliary tract atresia
0% Erythroblastosis
0% Gilbert's syndrome
0% Krigler-Nayyar syndrome

https://translate.yandex.com/en/doc 242/540
22:20 ,27.6.2023 �� C : ; 5 B
0% Dubin-Johnson syndrome

1096. A 5-month-old boy was born prematurely


and did not have any further illnesses during the neonatal period. When examined in the clinic,pallor of the skin,
drowsiness is noted. In the blood: Hb-95 g / l, Er.- 3,5•1012/ l, retic.-90 / 00, CP-0.7, osmotic resistance
of red blood cells-0.44-0.33%, serum iron-4.9 mmol/l. what is the most
likely cause of anemia?
100% Iron deficiency
0% Immaturity of hematopoiesis

252

Downloaded from the site - online testing step

0% Infectious process
0% Hemolysis of red blood cells
0% Vitamin B12 Deficiency

1097. Rodilla is 28 years old and is in the maternity ward. Labor activity is active.
The size of the pelvis is 26-29-3120 cm. Fetal position is longitudinal, mixed sciatic presentation.
Amniotic fluid with meconium has left, fetal heartbeat is deaf, 100 / min. Vaginal examination:
the opening of the cervix is complete, the buttocks are in the pelvic cavity. What are the tactics of labor management?
100% Fetal extraction by the pelvic end
0% Caesarean section
0% Conservative and curative care
0% Pologostimulation with oxytocin
0% Introduction of partusisten and antispasmodics

1098. an 8 - year-old girl complains of joint pain, fever up


to 380C, shortness of breath. Objectively: the left border of the heart is shifted to the left by 2.5 cm, tachycardia, systolic
murmur at the apex and at the V point. In the blood: lake.- 20,0•109/ l, WSE-18 mm / hour. What is the most
likely sign that justifies the diagnosis of rheumatism?
100% Cardit
0% Arthralgias
0% Leukocytosis
0% Delirium tremens
0% Accelerated SSE

1099. A 26-year-old pregnant woman was transferred from the nephrology department to the maternity hospital with
a diagnosis of 24-25 weeks ' pregnancy, chronic glomerulonephritis, mixed
edematous-hypertonic form. What is the strategy of an obstetrician-gynecologist?
100% Termination of pregnancy at 24-25 weeks
0% Terminating a pregnancy before 28 weeks and terminating it
0% Treatment of glomerulonephritis
0% Treatment of glomerulonephritis and gestation up to 38 weeks
0% Full-term pregnancy up to 40 weeks

1100. An 18-year-old patient was admitted to the gynecological department with complaints of aching pain
in the lower abdomen. Last menstruation 2 months ago. During bimanual examination
, the cervix is up to 2 cm long, cyanotic, and the external eye is closed. The body of the uterus is spherical in shape,
of a soft consistency, enlarged accordingly to 7-8 weeks of pregnancy. What is the most
likely diagnosis?

https://translate.yandex.com/en/doc 243/540
22:20 ,27.6.2023 �� C : ; 5 B
100% Threatening abortion
0% Abortion that started
0% Abortion is in progress
0% Fibromyoma of the uterine body
0% Incomplete abortion

253

Downloaded from the site - online testing step

1101. a 37-year-old man, a farmer, complains of general weakness, spastic pain in


the lower abdomen, mainly in the left iliac region, frequent liquid bowel movements up to 18
times a day with admixtures of mucus and blood. The disease started acutely, three days ago with
chills, a feeling of heat, and a headache. General condition of moderate severity, temperature
37.8°C. Sigmoid colon is spasmodic and painful. What is the most likely diagnosis?
100% Dysentery
0% Amoebiasis
0% Non-specific ulcerative colitis
0% Yersiniosis
0% Salmonellosis

1102. a two-year-old girl has an elevated body temperature of up to 37.2°C and a skin rash.
The child's general condition is satisfactory. On the skin of the back, extensor surfaces of the extremities-
a pink macular rash. The mucosa of the palatine arches is fine-toothed, moderately hyperemic.
Slight mucus discharge from the nose. Enlarged occipital and posterior cervical lymph nodes. What
is the most likely diagnosis?
100% Rubella
0% Scarlet fever
0% Chickenpox
0% Enterovirus exanthema
0% Measles

1103. a 2-month-old child born with a weight of 5100 g has jaundice, hoarse
crying, an umbilical hernia, and a lag in physical development. Liver + 2 cm, spleen is not
enlarged. Normal-colored feces and urine. In the anamnesis-delay in the fall of the umbilical
cord residue. In the blood: Hb-120 g / l, Er.- 4,5•1012/ l, WSE-3 mm / hour. Total serum bilirubin-28
mmol/l, indirect - 20 mmol/l, direct-8 mmol / l. what disease should you think
about first?
100% Congenital hypothyroidism
0% Congenital hepatitis
0% Hemolytic anemia
0% Conjugation jaundice
0% Cytomegalovirus infection

1104. A 40-year-old patient, a forester, called the SMD team. Complaints of severe headache,
body temperature up to 39.5°C, trembling of the limbs. From the medical history, it is known that he severely cut
his hand during the autopsy of the killed fox. The patient's mood is depressed. Asks not to turn on the light, not
to open the door. It responds to knocking with a sharp motor excitement. When the patient saw the carafe of
water, he had convulsive spasms in his throat. What are the tactics of the SMD doctor?
100% Admission to an infectious diseases hospital
0% Hospitalization in the intensive care unit

https://translate.yandex.com/en/doc 244/540
22:20 ,27.6.2023 �� C : ; 5 B
0% Hospitalization in the neurological department
0% Admission to a psychiatric hospital
0% Leave it at home and then consult a psychiatrist

254

Downloaded from the site - online testing step

1105. a 3-year-old child became acutely ill, body temperature rose to 39.5°C, became
sluggish, repeated vomiting, headache. Examination revealed positive meningeal
symptoms, and a lumbar puncture was performed. Cerebrospinal fluid is turbid, flows out under
pressure, protein content -1.8 g / l; Pandey + + + reaction, sugar-2.2 mmol/l, chlorides -123 mmol/l,
cytosis-2350 * 106 (80% neutrophils, 20% lymphocytes). What is the most likely diagnosis?
100% Purulent meningitis
0% Serous meningitis viral
0% Serous meningitis tuberculosis
0% Subarachnoid hemorrhage
0% Brain tumor

1106. A 13-year-old girl complains of an increase in body temperature to febrile numbers


within a month, joint pain, and periodic skin rashes. During the examination
, a persistent increase in SSE and LE cells was found in the blood. What is the most likely diagnosis?
100% System red shepherd dog
0% Juvenile rheumatoid arthritis
0% Systemic scleroderma
0% Acute lymphoblastic leukemia
0% Rheumatism

1107. for 8 years, a patient has been worried about coughing in the morning with a small amount
of sputum, shortness of breath. Smokes for more than 10 years. Objectively: cyanosis, increased duration
of exhalation, dry wheezing. What is the most likely diagnosis?
100% Chronic obstructive bronchitis
0% Chronic non-obstructive bronchitis
0% Idiopathic fibrosing alveolitis
0% Bronchiectasis
0% Bronchial asthma

1108. A 10-year-old girl was admitted to the hospital for another examination and treatment.
He has been ill for two years. A week ago, after ARVI, there was weakness,headache,
swelling on the legs and lower back, on the face, t°- 37.6°C, blood pressure - 105/65 mm Hg, heart rate-86/min.
Protein loss per day - 6 g, WSE-44 mm / h, total protein-45 g / l, A2-globulins-27%. What
is the main pathogenetic mechanism of the disease?
100% Glucocorticoids
0% Antibiotics
0% Nonsteroidal anti-inflammatory drugs
0% Diuretics
0% Anticoagulants

1109. A 70-year-old patient complains of weakness, dizziness, short periods


of fainting, and pain in the heart. Objectively: heart rate-40 / min, rhythmic tones, I-th tone is muffled,

https://translate.yandex.com/en/doc 245/540
22:20 ,27.6.2023 �� C : ; 5 B
periodically significantly amplified. AT - 180/90 mm Hg. what is the most likely cause
of hemodynamic disorders?
100% AV block of the third degree

255

Downloaded from the site - online testing step

0% AV block of the first degree


0% Bradysystolic form of atrial fibrillation
0% Sinus bradycardia
0% Complete blockage of the left bundle branch of Gis

1110. a 40-year-old man has autoimmune hepatitis. In the blood: a/G coefficient-0.8, bilirubin -42
mmol/l, transaminases - ALT - 2.3 mmol/l, AST - 1.8 mmol / l. Which of the following is
most effective in treatment?
100% Glucocorticoids, cytostatics
0% Antibacterial agents
0% Hepatoprotectors
0% Antiviral drugs
0% Hemosorption, vitamin therapy

1111. a 10-year-old girl complains of abdominal pain that occurs and increases after
eating rough or spicy food, acid belching, heartburn, frequent constipation, headache,
irritability. He's been ill for 12 months. Meals are irregular, "in dry water". Objectively: a girl
of satisfactory nutrition. The tongue is moist with a white coating at the root. The abdomen is soft, painful in
the epigastrium. Which of the research methods is most likely to help diagnose
the disease?
100% Esophagogastroduodenofibroscopy
0% Intragastric pH measurement
0% Fractional study of gastric juice
0% Contrast radioscopy
0% Biochemical blood test

1112. A 35-year-old patient was admitted to the hospital with complaints of pain in the left
sternoclavicular, knee joints, and lower back. He became acutely ill, with a fever of up
to 38°C. objectively: the left thoracoclavicular and knee joints are swollen and painful during
palpation. In the blood: lake.- 9,5•109/ l, SSE-40 mm / h, CrP-1.5 mm, fibrinogen-4.8 g / l, uric
acid-0.28 mmol/l. in scraping from the urethra-chlamydia. What is the most likely diagnosis?
100% Reiter's Syndrome
0% Rheumatic arthritis
0% Gout
0% Ankylosing spondylitis
0% Rheumatoid arthritis

1113. A 60-year-old patient complains of weakness, dizziness, heaviness in


the upper abdomen, paresthesia of the fingers and toes. Objectively: jaundice of the skin, tongue of crimson
color, smooth. Iepathomegaly. In the blood: Hb-90 g / l, Er.- 2,3•1012/ l, reticulocytes-0.2%;
CP-1.2, macrocytosis; Gelli corpuscles, Cabot's rings. Which drug is most
appropriate for treatment?
100% Vitamin B12
0% Ferroplex

https://translate.yandex.com/en/doc 246/540
22:20 ,27.6.2023 �� C : ; 5 B
0% Red blood cell mass
256

Downloaded from the site - online testing step

0% Prednisone
0% Dysferol

1114. A 53-year-old patient went to a neurologist with complaints of low back pain.
A course of physiotherapy was performed for sciatica, but the patient's condition did not improve.
The patient underwent an R-graph of the spine and pelvic bones, found osteoporosis and significant bone
"holes". In the blood-moderate normochromic anemia, in the urine - proteinuria. Total blood protein
- 10.7 g / l. what disease should you think about first?
100% Myeloma disease
0% Urolithiasis
0% Acute sciatica
0% Bone metastases
0% Lymphogranulomatosis

1115. a 34-year-old coal miner with a professional experience of 10 years was discharged from the clinic
of occupational diseases after examination and treatment with a diagnosis of anthracosilicosis, stage 1,
peribronchitis, DN 0 V. what expert decision should be made regarding his performance?
100% Send it to the MSEC to determine the disability group for the period of retraining
Can continue to work in their profession in compliance with sanitary and hygienic
0%
working conditions
0% Issue a disability certificate for continuing outpatient treatment
0% Issue a professional bulletin to consolidate the results of treatment
0% Send it to the MSEC to determine the percentage of professional disability loss

1116. A 43-year-old patient complains of periodic bouts of pain in the right side
of her face. During an attack, there is a spasm of the facial muscles of the right half of the face,
the skin on this side of the face turns red. In the blood-no pathology. He was diagnosed
with trigeminal neuralgia on the right side. Which drug is most appropriate to prescribe?
100% Finlepsinum
0% Prednisone
0% Actovegin
0% Analgin
0% !ndomethacin

1117. a patient went to the hospital complaining of general weakness, fever, and a painful
rash on the torso. Ill for 3 days. Objectively: on the lateral surface of the trunk on the left side, against the
background of hyperemia and edema, there are grouped blisters with serous-cloudy and
hemorrhagic contents. What is the most likely diagnosis?
100% Herpes zoster
0% Simple contact dermatitis
0% Allergic contact dermatitis
0% Microbial eczema
0% Duhring's herpetiform dermatosis

257

https://translate.yandex.com/en/doc 247/540
22:20 ,27.6.2023 �� C : ; 5 B

Downloaded from the site - online testing step

1118. during an internal obstetric examination of a parent, the sacral cavity is completely
filled with the fetal head, and the sciatic spine is not detected. Arrow-shaped seam in straight
size, small crown facing the symphysis. In what plane of the small pelvis is
the pre-lung part of the fetus?
100% Plane of exit from the small pelvis
0% Plane of the broad part of the pelvic cavity
0% Plane of the narrow part of the pelvic cavity
0% Plane of entrance to the small pelvis
0% Above the entrance to the pelvis

1119. A 56-year-old patient complains of compressive pain that occurs


at the same time at night. The pain radiates to the left shoulder blade, does not go away after taking nitroglycerin. On
the ECG during the attack: ST is 5 mm above the contour line in V5, V6; after the attack, it returns to the contour line.
What is the most likely pathology that determines such a picture?
100% Variant angina pectoris
0% Myocardial infarction
0% Osteochondrosis of the cervical region
0% Dry pericarditis
0% Resting angina pectoris

1120. a 75-year-old man with a diagnosis of IHD: diffuse cardiosclerosis, atrial fibrillation,
stage 2B HF, and chronic pyelonephritis was prescribed digoxin. For the first 6 days, the digoxin dose
was 0.25 mg twice daily, which reduced shortness of breath, edema, and cyanosis. However
, on the 7th day, the patient developed nausea and bradycardia. What is the most likely cause
of digoxin intoxication?
100% Impaired elimination of the drug by the kidneys
0% The saturating dose is too long
0% Excess of the daily saturating dose
0% Impaired liver digoxin metabolism
0% There is no unitiol in the treatment package

1121. the 30-year-old man was always locked up by nature. I've never
been treated by psychiatrists. Complains of headache, feeling "as if something is bursting under the scalp,
moving, boiling." Objectively: no pathology was detected. What is the most likely
psychopathological symptom in this case?
100% Senestopathy
0% Paresthesia
0% Hallucination
0% Hyperesthesia
0% Dysmorphopsia

1122. a 45-year-old man complains of intense epigastric pain 1.5-2 hours after
eating. He has been suffering from peptic ulcer disease for 11 years. Ob'ektivno: t° - 36,5°C,
PDR-16/xv, Ps-70/xv, AT-120/80 mm Hg Palpation: local soreness in the right
epigastric region. What are the indicators of intragastric pH-metry in the area of the stomach body?

258

https://translate.yandex.com/en/doc 248/540
22:20 ,27.6.2023 �� C : ; 5 B

Downloaded from the site - online testing step

will they be the most characteristic of this patient's disease?


100% pH = 1.0-2.0
0% pH = 3.0-4.0
0% pH = 4.0 - 5.0
0% pH = 5.0-6.0
0% pH = 6.0-7.0

1123. A 20-year-old patient complains of redness of the eyes, lacrimation, moderate purulent
discharge from the conjunctival cavity, and a foreign body sensation in the eyes. Objectively:
hyperemia of the conjunctiva of the eyelids. Conjunctival injection on the eyeballs. The cornea is transparent.
The pupil is 3 mm in diameter, reacts vividly to light. The lens and vitreous body are transparent. The fundus is
normal. What is the most likely diagnosis?
100% Acute bacterial conjunctivitis
0% Gonoblenorrhea
0% Adenoviral keratoconjunctivitis
0% Allergic conjunctivitis
0% Acute iridocyclitis

1124. A 24-year-old patient complains of general weakness, dizziness,


fever up to 37.5°C, sore throat, neck edema, enlarged submandibular lymph nodes.
Objectively: the oropharyngeal mucosa is swollen and cyanotic, the tonsils are enlarged, covered
with films extending beyond them, and are difficult to remove. What is the main mechanism of development
of this disease?
100% Action of bacterial exotoxin
0% Action of bacterial endotoxin
0% Allergic component
0% Dysbiotic changes
0% Metabolic disorders

1125. a 13-year-old teenager suffering from hemophilia a was hospitalized after a fight at school
. Right-sided hemarthrosis of the knee joint, extraperitoneal hematoma was diagnosed.
What should be prescribed to the patient first?
100% Fresh frozen plasma
0% Aminocislaproic acid
0% Washed platelets
0% Placental albumin
0% Dry plasma

1126. A 27-year-old girl has been suffering from rheumatoid arthritis for 7 months. Objectively:
swelling of the elbow,lumbar, knee and ankle joints,
rheumatoid nodules in the area of the elbow joints. In the blood: SSE-56 mm/h, C-reactive
protein (+++). Radiographs of the joints reveal pronounced osteoporosis. What is the basic
therapy in this situation?
100% Methotrexate
0% Indomethacin

259

Downloaded from the site - online testing step

https://translate.yandex.com/en/doc 249/540
22:20 ,27.6.2023 �� C : ; 5 B
0% Diclofenac Sodium
0% Prednisone
0% Meloxicam

1127. a 38-year-old man complains of low-intensity wobbling and growing


weakness in the muscles of the shoulder and pelvic girdle, back, significant difficulties when getting out of bed,
shaving, and moving up the stairs in the last 3 weeks. Suspected dermatomyositis. In the blood: Hb-114 g / l,
leuc. 10.8 * 109/l, eosin.- 9%, SSE-22 mm / h, C-reactive protein (++). Changes in which laboratory
parameter will be of crucial diagnostic importance?
100% Creatine Phosphokinase
0% Ceruloplasmin
0% Sialic acids
0% Antibodies to nDNA
0% y-globulins

1128. A 30-year-old patient was hospitalized with a diagnosis of intestinal obstruction. During the operation
, it turned out that the obstruction of the small intestine is caused by a ball of worms. What kind of worms are they?
100% Roundworms
0% Guinea worm
0% Filarii
0% Cysticerci
0% Pinworms

1129. the mother of a newborn child suffers from chronic pyelonephritis. Before giving birth
, she suffered acute respiratory viral infections. Delivery is urgent, a long waterless period. On day 2, the child developed
an erythematous rash, followed by blisters about 1 cm in size filled
with serous contents. Ni-Kolsky's symptom is positive. After opening the blisters
, erosions appear. The child is sluggish, the body temperature is subfebrile. What is the most likely diagnosis?
100% Newborn's pemphigus
0% Vesiculopustulosis
0% Pseudofurunculosis
0% Sepsis
0% Ritter's dermatitis

1130. a 9-year-old boy has been suffering from diabetes for the first year. Receives insulin (Humulin R,
NPH) at the rate of 0.4 U/kg of body weight per day. Insulin is injected under the skin of the shoulder with a syringe pen. What
measures should be taken to prevent lipodystrophy?
100% Change the place of administration of insulin
0% Limit the amount of fat in your child's diet
0% Reduce your insulin dose
0% Periodically switch to a different type of insulin
0% Prescribe antioxidants

1131. a 25-year-old pediatrician fell ill a week ago: the temperature rose to 37.6°C,

260

Downloaded from the site - online testing step

there was a swelling on the neck. The diagnosis was determined: acute respiratory infections, cervical lymphadenitis. Prescribed
treatment: erythromycin, a warming compress on the neck. During treatment

https://translate.yandex.com/en/doc 250/540
22:20 ,27.6.2023 �� C : ; 5 B
, the body temperature increased to 39°C, headache, repeated vomiting, and meningeal
syndrome appeared. What tests should be performed to determine the final diagnosis?
100% Spinal puncture
0% Puncture of the cervical lymph node
0% General blood test
0% Sputum on secondary flora
0% X-ray examination of the lungs

1132. A 65-year-old patient, who is in the intensive care unit of the pulmonology
department due to an asthmatic condition, has started constant oxygenation through a face mask.
However, after 4.5 minutes, the patient developed "oxygen apnea". To get out of this state
, it is best to implement:
100% Artificial ventilation of lungs
0% Intravenous administration of analeptics
0% Additional CO2 inhalation
0% Stopping oxygen inhalation
0% Infusion of alkaline solutions

1133. a 34-year-old woman became acutely ill after acute respiratory viral infection, when pain appeared in the interphalangeal
joints, and then after 2 weeks in the knee joints, morning stiffness,
body temperature rose to 38°C. Objectively: the interphalangeal, metacarpophalangeal, and knee joints
are swollen, swollen, and hot to the touch, with limited movement. In the blood: SSE-45 mm / h, CRP +++,
Waaler-Rose reaction 1:128. what mechanism of the disease can we think about?
100% Autoimmune
0% Allergic
0% Exchange Rate
0% Degenerate
0% Dystrophic

1134. a patient with cirrhosis of the liver after drinking alcohol developed headache,
vomiting, aversion to food, insomnia, jaundice," hepatic " bad breath, bloating.
What complication of cirrhosis of the liver can you think about?
100% Hepatic-cellular insufficiency
0% Bleeding from varicose veins of the esophagus
0% Portal hypertension
0% Acute stomach ulcer
0% Mesenteric vascular thrombosis

1135. in a 60-year-old patient, on the 4th day after the injection, pain and
tissue compaction appeared in the left buttock. Objectively: in the Upper-outer quadrant of the left buttock, the skin
is red, hot to the touch, during palpation, an infiltrate of 6x6 cm is determined, painful, in the center
of softening. Body temperature-37.9°C. What should be done to determine the presence of an abscess?
100% A puncture

261

Downloaded from the site - online testing step

0% A biopsy
0% Ultrasound examination
0% Radiography
0% General blood test

https://translate.yandex.com/en/doc 251/540
22:20 ,27.6.2023 �� C : ; 5 B

1136. A 40-year-old patient became ill a month ago due to fever


and intoxication, which was later joined by shortness of breath, cough with sputum discharge. On
X-ray examination, foci of various sizes were found in the upper parts of the lungs
, sometimes merging. What is the most likely diagnosis?
100% Disseminated pulmonary tuberculosis
0% Bilateral focal pneumonia
0% Lung carcinomatosis
0% Idiopathic fibrosing alveolitis
0% Congestive lung

1137. a 1-year-old boy developed a pinched inguinal hernia. Objective: hyperemia and
pastyness of the hernial lunge tissues. Which treatment method is optimal?
100% Immediate surgical intervention
0% A set of conservative measures designed for self-correction of a hernia
0% Manual hernia repair
0% Immediate hernia repair under short-term anesthesia
0% Appointment of barbiturates

1138. in the patient, varicose veins of the lower limb were complicated by acute
thrombophlebitis. The infiltrate has increased in volume, acquired a sharp soreness, the skin is hyperemic.
Hectic temperature. In the hospital at the place of residence, she was treated according to the usual
scheme. 2 days before hospitalization, a second infiltrate was formed under the pupart ligament.
There was a chill. What is the most likely diagnosis?
100% Ascending thrombophlebitis of superficial veins
0% Acute deep vein thrombosis
0% Erysipelas
0% Burger's disease
0% Post-thrombophlebitic syndrome

1139. A 43-year-old patient suffered deep vein thrombosis on the right side of the iliofemoral segment 3 years ago
. To date, I am concerned about the severity, swelling of the right lower
limb. Objectively: moderate edema of the lower leg,brown induration of the skin in the lower third of the lower leg,
varicose superficial veins of the lower leg. What is the most likely diagnosis?
100% Post-thrombophlebitic syndrome, varicose veins
0% Acute venous thrombosis on the right side
0% Lymphedema of the right lower limb
0% Parks-Weber syndrome
0% Acute superficial vein thrombophlebitis

262

Downloaded from the site - online testing step

1140. a 23-year-old patient became ill 3 weeks ago when a very painful hardening of the armpit appeared
. After 4-5 days, it opened with the release of a large amount of pus. Then
new infiltrates appeared all around. Prior to this case, I did not suffer from skin diseases. What
is the most likely diagnosis?
100% Hydradenite
0% A boil

https://translate.yandex.com/en/doc 252/540
22:20 ,27.6.2023 �� C : ; 5 B
0% Mycosis
0% Herpes zoster
0% Streptoderma

1141. a 39-year-old patient complains of shortness of breath during physical exertion, swelling of the
lower legs, palpitation, and heart failure. Objectively: heart rate-150 / min, atrial fibrillation. The borders
of the heart are extended in both directions. Heart tones are muted. The liver is 6 cm below the edge of the costal
arch. Echocardiography-dilation of the heart chambers (CDR of the left ventricle 6,8 cm) EF-29%, valvular apparatus
without changes. What is the most likely diagnosis?
100% Dilated cardiomyopathy
0% Exudative pericarditis
0% Restrictive cardiomyopathy
0% Hypertrophic cardiomyopathy
0% Thyrotoxic cardiomyopathy

1142. a 25-year-old pregnant woman with a gestation period of 40 weeks. Dimensions of the pelvis 26-26-3118 cm. Coolant-100
cm. VDM - 42 cm. The fetal position is longitudinal, the head is pressed against the entrance to the small pelvis.
Fetal heartbeat is muted-160 / min. The contractions that started 6 hours ago are painful and plentiful.
The amniotic fluid has receded. The traction ring is located at the level of the navel and
is located obliquely. What complication occurred during childbirth?
100% Threat of uterine rupture
0% Amniotic fluid embolism
0% Premature detachment of the normally located placenta
0% Complete uterine rupture
0% End and period of labor

1143. the child was born a day ago. In childbirth, vyvazhivanie shoulders is difficult. Body weight
4300.0. Objectively: the right arm hangs down along the torso, the hand is pierced, there are no movements in the hand.
Positive scarf symptom. Specify the most likely diagnosis:
100% Total right-sided obstetric paralysis
0% Proximal type of right-sided obstetric paralysis
0% Distal type of right-sided obstetric paralysis
0% Hemiparesis
0% Tetraparesis

1144. An outbreak of food poisoning has been registered in an urban-type settlement. The diagnosis of botulism
is established on the basis of the clinical picture of the disease. Which of the listed products
should be first selected for analysis to confirm the diagnosis?
100% Preserves

263

Downloaded from the site - online testing step

0% Potato
0% Pasteurized milk
0% Boiled meat
0% Cabbage

1145. The 42-year-old locksmith has been working for five years in a workshop near a lathe for
the production of heavy large-sized parts, using manual and pedal levers
that require significant physical effort. What measures should be recommended to prevent osteoarthritis?
https://translate.yandex.com/en/doc 253/540
22:20 ,27.6.2023 �� C : ; 5 B

100% Limit heavy physical labor


0% Protein-carbohydrate diet
0% Protein-vitamin diet
0% Wellness on the Black Sea coast
0% Weightlifting classes

1146. a patient was taken to the emergency department after falling from a height, with clinical
signs of multiple fractures of both lower limbs. Objectively: the patient's condition is severe,
consciousness is preserved, but mentally retarded, the skin is pale gray,
cold sweat is on his forehead. Breathing is shallow, up to 30 / min, blood pressure-80/60 mm Hg, Ps-120 / min, weak
filling. What complication does the victim have?
100% Traumatic shock 2 st
0% Traumatic shock 1 st
0% Traumatic shock 3 st
0% Traumatic shock 4 st
0% Fainting

1147. a 20-year-old patient


has a diffuse papular rash of bright red color on the skin of the extensor surface of the limbs, back and scalp
,ranging in size from millet grain to lentils, the surface of the elements of which is covered with whitish
scales in the center. Using the scraping method, you can identify the symptom of "stearin stain",
"terminal film" and "blood dew". What disease should I think about?
100% Psoriasis
0% Lichen planus erythematosus
0% Secondary syphilis
0% Papular necrotic tuberculosis of the skin
0% Atopic dermatitis

1148. a mother who is on partially paid leave to care for a child under 3 years
of age became ill and was hospitalized. What document will be issued in this case to the working
father who will take care of the child during the mother's illness?
100% Sick leave form
0% Information about the mother's illness
0% Certificate of the need for child care
0% Extract from the medical record of an outpatient or inpatient patient

264

Downloaded from the site - online testing step

0% Any form of help

1149. A 56-year-old female patient developed massive bleeding during cholecystectomy.


A blood transfusion was decided. In the patient, blood type AB (IV) Rh-. There is no such group at the blood transfusion station.
What group of donors can be called to donate blood?
100% Donors of rare blood types
0% Active Group
0% Related donors
0% Emergency donors
0% Reserve donors

https://translate.yandex.com/en/doc 254/540
22:20 ,27.6.2023 �� C : ; 5 B

1150. A 30-year-old patient is being treated for Verlhof's disease. Objectively:


pale, has petechial hemorrhages on the extensor surfaces of the forearms. Ps-92 / xb, AT-100/60
mm Hg. lower edge of the spleen at the level of the navel. In the blood: Er.- 2,8•1012/ l, Hb-90 g / l, Ht-0.38,
blood clot.- 30•109/L. the patient is preparing for a splenectomy operation. Which transfusion medium should
be chosen first for preoperative preparation?
100% Platelet mass
0% Canned blood
0% Native red blood cell mass
0% Red blood cell curl
0% Washed red blood cells

1151. A clinical and statistical study of the effect of a new pharmacological drug on
patients with coronary heart disease was conducted. What parametric criterion (coefficient) can be
used to estimate the probability of results?
100% Student Coefficient (t)
0% Character criteria
0% Compliance rate
0% Wilcoxon's T-test
0% Kolmogorov-Smirnov criterion

1152. A 45-year-old sailor patient was hospitalized on the 2nd day of his illness.
I got back from India a week ago. Complaints of a temperature of 41°C, severe headache, shortness of breath, cough with
foamy, rusty sputum. Objectively: pale, mucosal cyanosis, PDR-24 / min,
tachycardia. In the lungs, breathing is weakened,wet wheezing is heard over both lungs,
crepitation. What is the most likely diagnosis?
100% Plague, a pulmonary form
0% Miliary tuberculosis
0% Flu
0% Ornithosis
0% Sepsis

1153. for the prevention of nosocomial infections that are transmitted


by airborne droplets, full boxes are planned in the infectious diseases department. They consist of

265

Downloaded from the site - online testing step

vestibule, ward, airlock. What other structural part should be included in a complete box?
100% Bathroom
0% Manipulative
0% Doctor's office
0% Overview page
0% Nurse's post

1154. A 40-year-old patient with an injury to both thighs was taken to the emergency department from
the scene of the accident by an accompanying vehicle. Objectively: cyanosis, purpura in the lower half
of the neck, tachypnea, blood pressure-60/40 mm Hg, Ps-120 / min. In the lungs - wet wheezing. Diuresis of 20 ml / hour. In the blood:
HY-i0o g/l, ep. -3,6 * 1012/l. what is the most likely diagnosis?
100% Fat embolism

https://translate.yandex.com/en/doc 255/540
22:20 ,27.6.2023 �� C : ; 5 B
0% Traumatic shock
0% Hemorrhagic shock
0% Pain shock
0% Thromboembolism

1155. a patient complains of severe headache in the frontal region, purulent runny nose,
nasal congestion, decreased sense of smell. He has been ill for a week after suffering from acute respiratory viral infections. Objectively:
the general condition is satisfactory, the skin is clean, t° - 37.5°C. palpation of the exit points of the first branch
of the trigeminal nerve is painful, breathing through the nose is moderately difficult, after instilling 0.1% naphthyzine solution into the
nose, a
lot of mucus and pus blows up, after which breathing improves. What
is the100%
most likely diagnosis?
Acute purulent sinusitis
0% Acute chronic rhinitis
0% Chronic purulent sinusitis in the acute stage
0% Acute respiratory viral infection complicated by trigeminal neuralgia
0% Acute viral infection

1156. a 10-year-old boy was diagnosed with diabetes mellitus. During the examination
, the smell of acetone from the mouth is noted. In the blood: sugar - 20.5 mmol/l. in the urine: sugar-20 g / l, acetone -
(+++). What can explain the appearance of acetone in the exhaled air and urine?
100% Enhanced breakdown of ketogenic amino acids and lipids
0% Violation of the water-electrolyte balance
0% Violation of the acid-base balance
0% Violation of glucose phosphorylation processes
0% Weakening of glycolysis processes

1157. the newborn has purulent discharge from the umbilical wound, the skin around the navel is swollen.
Objectively: the skin is pale, with a yellow-gray tint, generalized hemorrhagic rash.
Body temperature is hectic in nature. What is the most likely diagnosis?
100% Sepsis
0% Hemorrhagic disease of newborns
0% Hemolytic disease of newborns

266

Downloaded from the site - online testing step

0% Thrombocytopathy
0% Omphalitis

1158. Based on the results of five-year monitoring, the degree of influence


of environmental factors on public health indicators was assessed. What statistical method should
I use for this purpose?
100% Calculating the correlation coefficient
0% Calculating the matching coefficient
0% Calculating the difference probability coefficient
0% Calculating the regression coefficient
0% Calculating dynamic series indicators

1159. A doctor has developed a program to improve the physical fitness of boarding school students
. This program is based on the following principles: gradualness,
https://translate.yandex.com/en/doc 256/540
22:20 ,27.6.2023 �� C : ; 5 B
consistency, individuality, complexity. Which of the basic principles of tempering is not
taken into account?
100% Systematic approach
0% Increased resistance
0% Building up the power of action
0% Increasing the intensity of the action
0% Increasing resilience

1160. the patient complains of heartburn and constant pain behind the sternum. After eating, when
the body is tilted, regurgitation occurs. X-ray examination revealed: a slippery
cardiofunctional keel of the esophageal opening of the diaphragm, with the placement of the car-action on the bottom
of the stomach 8 cm above the diaphragm. During esophagoscopy, there are signs of reflux esophagitis. What
is the treatment strategy for this patient?
100% Operation in a surgical hospital
0% Conservative treatment in a polyclinic
0% Conservative treatment in a therapeutic hospital
0% Conservative treatment in a surgical hospital
0% Spa treatment

1161. a 54-year-old patient, who has been suffering from duodenal ulcer for 10 years
, experienced acute "dagger" pain in the epigastrium after eating. Objectively: pale,
bradycardia, "board-shaped belly", sharply positive Shchetkin-Blumberg symptom. What
pathology will the ambulance doctor suspect?
100% Breakthrough duodenal ulcer of the 12th duodenum
0% Intestinal thrombosis
0% Acute intestinal obstruction
0% Pinched keel
0% Acute pancreatitis

1162. a 40-year-old woman notes gradually increasing pain in her right arm for 3 days

267

Downloaded from the site - online testing step

the iliac region starting from the epigastric region. Temperature up to 37.5-37.8°C.
During examination: the tongue is moist, Ps-80 / min, the abdomen is soft on palpation, moderately painful in the right
iliac region, where a small formation is indistinctly palpated. In the blood: lake.-12 * 109/l. what
pathology should I think about?
100% Appendicular infiltrate
0% Cecal tumor
0% Twisted ovarian cyst
0% Ectopic pregnancy
0% Appendicular abscess

1163. at the time of examination, the patient is unconscious, his skin is dry, hot, and his face is flushed. Kusmaul's breath
, the smell of acetone in the air. Positive symptoms of peritoneal irritation. Blood sugar -33
mmol / l. what is the first emergency measure to take?
100% Short-acting intravenous insulin infusion
0% Intravenous glucose infusion with insulin
0% Administration of long-acting insulin

https://translate.yandex.com/en/doc 257/540
22:20 ,27.6.2023 �� C : ; 5 B
0% Intravenous infusion of neohemodesis with glutamic acid
0% Intravenous infusion of sodium chloride saline solution

1164. the child is 8 months old. Physical and neuropsychic development is age-appropriate. I haven't
been ill yet. What is the most likely number of baby teeth a child can have?
100% 4
0% 1
0% 2
0% 3
0% 5

1165. An 18-year-old student complains for 7 days of weakness, body temperature up


to 37.8°C, mucous discharge from the nose, sore throat during swallowing, a feeling of" sand " in the eyes.
Objectively: an increase in the posterior cervical, angulomandibular lymph nodes, edema and injection
of conjunctival vessels, hyperemia of the oropharyngeal mucosa, hypertrophy of the tonsils, no plaque.
What is the most likely diagnosis?
100% Adenovirus infection
0% Flu
0% Infectious mononucleosis
0% Rhinovirus infection
0% Parainfluenza

1166. A 23-year-old patient underwent primary surgical treatment of a gunshot wound


in the lower third of the right lower leg 4 days ago. The patient's condition is relatively satisfactory.
No stitches were applied to the wound after treatment. During the dressing, it was found that the wound with untidy bloody-purulent
discharge, edema of the tissues around it is moderately pronounced. When you touch the wound
with a tampon, the muscles near the wound twitch. Regional lymph nodes are not enlarged. What
is the most likely diagnosis?
100% Tetanus

268

Downloaded from the site - online testing step

0% Anthrax
0% Gas gangrene
0% Putrefactive infection
0% Staphylococcal infection

1167. A 35-year-old patient was admitted to the district hospital a week after an accident with
a collapsed hemothorax clinic. What is the appropriate treatment strategy in terms of preventing the development
of acute pleural empyema in a patient?
100% Surgical removal of coagulated hemothorax
0% Treatment with pleural puncture
0% Complex conservative therapy
0% Passive drainage of the pleural cavity
0% Active drainage of the pleural cavity

1168. in order to study the influence of the microclimate on the human body, it is necessary to organize
systematic monitoring of the air temperature for 3 days. Select the device that
will allow you to record the temperature most accurately:
100% Thermograph

https://translate.yandex.com/en/doc 258/540
22:20 ,27.6.2023 �� C : ; 5 B
0% Alcohol Thermometer
0% Mercury thermometer
0% August Psychrometer
0% Assman Psychrometer

1169. a child aged 3 months developed baldness of the back of the head, restless sleep, increased
sweating. What kind of illness can you think about?
100% Rickets
0% Spasmophilia
0% Anemia
0% Phosphate-diabetes
0% Chondrodystrophy

1170. the worker is registered for work, has passed a medical preventive examination. It is recognized
as suitable for work in the conditions of this production. What type of medical
check-up did the worker undergo?
100% Previous
0% Planned
0% Periodic
0% Target
0% Comprehensive

1171. a 7-year-old boy has "blackouts" for 10-15 seconds during the day
, during which he freezes in one position, does not respond to questions, does not respond to his name.
he doesn't remember anything about the attacks afterwards. Neurological examination revealed no changes. What
research method is most appropriate to use to clarify the diagnosis?

269

Downloaded from the site - online testing step

100% Electroencephalography
0% X-ray of the skull
0% Computed tomography (CT) scan
0% Echoencephaloscopy
0% Research of evoked potentials

1172. insufficiently treated industrial waste is discharged into the river, the water from which
is used for domestic drinking water supply. This causes the death of some
microorganisms, disruption of water self-purification processes and deterioration of its quality, which can
have a negative impact on people's health. What is the name of such an action
of environmental factors?
100% Indirect
0% Direct line
0% Combined
0% Comprehensive
0% Combined

1173. the cold period of the year was characterized by low air temperature, constant
cloud cover, and snowfall. In this regard, the stay of preschool children in the open
air was limited. During the medical examination, children complained of drowsiness, weakness,

https://translate.yandex.com/en/doc 259/540
22:20 ,27.6.2023 �� C : ; 5 B
poor appetite, and low activity in physical education classes. A
laboratory blood test revealed a violation of phosphorus-calcium metabolism.
The reason for this condition of children can be:
100% Ultraviolet insufficiency
0% Heating microclimate in the room
0% Cooling microclimate in the room
0% Uncomfortable indoor climate
0% Excessive physical activity

1174. a 22-year-old patient complains of aching pain in the right swollen area for a week,
morning sickness, and changes in taste. Delayed menstruation for 3 weeks. Objectively: AT-110/70 mm Hg,
Ps-78 / min, t° - 37.0°C. Bimanually: the uterus is slightly enlarged, soft, mobile, painless. Palpation
of the appendages: on the right side there is a painful formation of 3x4 cm, tightly elastic consistency, moderately
mobile. What is the most likely diagnosis?
100% Progressive tubal pregnancy
0% Aborted tubal pregnancy
0% Right ovarian cyst
0% Uterine pregnancy
0% Acute appendicitis

1175. a 46-year-old patient went to the residential complex with complaints of moderate spotting from the vagina,
which appeared after the delay of the next menstruation for 1.5 months. During vaginal examination
, the cervix is clean, the uterus is not enlarged, mobile, non-painful, and the appendages are not changed. What
is the most likely diagnosis?
100% Dysfunctional uterine bleeding

270

Downloaded from the site - online testing step

0% Adenomyosis
0% Ectopic pregnancy
0% Submucosal uterine fibroids
0% Cancer of the uterine body

1176. a 15-year-old patient has a headache, nosebleeds, and a cold sensation in the lower
extremities. Objectively: the muscles of the shoulder girdle are well developed, the lower limbs are hypotrophied.
Pulsation on the arteries of the foot and femoral artery is sharply weakened. AT - 150/90 mm Hg-on
the hands, 90/60 mm Hg-on the feet. Above the carotid arteries - systolic murmur. What is the most
likely diagnosis?
100% Aortic coarctation
0% Aortic aneurysm
0% Aortic stenosis
0% Aortic insufficiency
0% Coarctation of the pulmonary artery

1177. A 15-year-old patient complained of periodic pain in the lower abdomen, more
on the right side. Menstrual function is not impaired. He doesn't have a sexual life. Rectoabdominal
examination: the uterus is not changed, the appendages are not detected on the left side
, the ovoid formation is palpated on the right side, the size is 10x11 cm with a smooth surface,
a tight-elastic consistency, mobile, painless. What is the most likely diagnosis?
100% Tumor of the left ovary

https://translate.yandex.com/en/doc 260/540
22:20 ,27.6.2023 �� C : ; 5 B
0% Chronic salpingoophoritis
0% Piovar on the left
0% Ovarian cancer
0% Appendicitis

1178. A 30-year-old maternity patient was taken to the hospital with complaints of sharply painful,
regular contractions for 25-30 seconds, every 1.5-2 minutes. Labor started 6
hours ago. The uterus is in high tone. The fetal head is located above the entrance to the small pelvis.
Fetal heart rate-136 / min. PV: opening of the cervix 4 cm, uterine eye spasms at the height of contractions.
The head is in the plane of the entrance to the small pelvis, repelled. What is the most likely diagnosis?
100% Discoordinated labor activity
0% Secondary weakness of labor activity
0% Pathological preliminary period
0% Primary weakness of labor activity
0% Normal labor activity

1179. on the fourth day after receiving a stab wound to the right foot, the patient
's body temperature rose to 38°C, the inguinal lymph nodes increased, became painful, and the skin above
them turned red. What complication of the wound can you think about?
100% Lymphadenitis
0% Lymphangitis
0% Phlegmon
0% Tetanus

271

Downloaded from the site - online testing step

0% Erysipelas

1180. A 22-year-old woman went to a women's clinic on the 15th day after cesarean
section complaining of pain in the right breast, fever up to 39.00 C,
and chills. Objectively: the mammary gland is enlarged, hyperemic, compacted, painful during
palpation. The child is breastfed from 8 days, the milk is expressed irregularly. What is the prevention
of this condition?
Regular breast pumping after feeding, prenatal preparation
100%
of nipples and mammary glands
0% Regulation of the menstrual cycle
0% Regular medical examinations, use of the IUD
0% Expressing both mammary glands after each feeding
0% Conducting professional examinations

1181. A 29-year-old female patient complaining of infertility and menstrual disorders of the
oligomenorrhea type was found to be 160 cm tall, 91 kg in body weight, and to have
hair growth on her face and thighs. A bimanual examination revealed enlarged
ovaries of a dense consistency,measuring 5x6 cm, on both sides. The same data is confirmed by ultrasound. What is the reason
for a woman's complaints?
100% Sclerocystic ovary syndrome (Stein-Leventhal)
0% Ovarian androblastoma
0% Premenstrual syndrome
0% Chronic bilateral adnexitis
0% Adrenogenital syndrome

https://translate.yandex.com/en/doc 261/540
22:20 ,27.6.2023 �� C : ; 5 B

1182. a 52-year-old patient complains of headache and weakness of the left upper limb. With
physical exertion of the left limb, neurological symptoms increase. Pulsation on
the arteries of the left limb is sharply weakened, on the carotid arteries-preserved. What is the most
likely diagnosis?
100% Occlusion of the left subclavian artery,"robbing" syndrome
0% Thoracic outlet syndrome
0% Raynaud's syndrome
0% Takayasu's disease
0% Occlusion of the brachiocephalic trunk

1183. a 57-year-old patient suddenly lost sight in one eye. On examination: vision is 0.02
eccentric, on the fundus of the eye hemorrhages of different sizes and shapes (a symptom of "crushed
tomato"). The optic disc is edematous and hyperemic. In the anamnesis: general vascular
pathology. Direct-acting anticoagulants were urgently prescribed. What is the most likely
diagnosis?
100% Central retinal vein thrombosis
0% Hypertensive angiopathy
0% Hypertensive angioneuropathy
0% Central retinal artery embolism
0% Diabetic retinopathy

272

Downloaded from the site - online testing step

1184. on the 20th day of the postpartum period, a woman went to the gynecologist complaining of pain
in the left breast, purulent discharge from the nipple. Objectively: Ps-120 / min, body temperature
39°C. The left mammary gland is painful, larger than the right, its skin is hyperemic; in the upper
quadrant there is an infiltrate of 10x5 cm with softening inside. In the blood: SHZ-50 mm / h,
lake.15.0 * 109/l. what will be the doctor's tactics?
100% Be admitted to the surgical department for surgical treatment
0% Send it to the gynecological department
0% Send to the postpartum department
0% Refer to a polyclinic surgeon for conservative treatment
0% Open a breast abscess in a women's clinic

1185. a woman went to the therapist complaining of fatigue, significant weight loss,
weakness, loss of appetite. Amenorrhea for 8 months. A year ago, I gave birth to a live full-term baby.
Blood loss in childbirth up to 2 liters. blood and blood substitutes were transfused. What is the most likely
diagnosis?
100% Sheehan's syndrome
0% Stein-Leventhal syndrome
0% Shereshevsky-Turner syndrome
0% Homologous blood syndrome
0% Vegetative-vascular dystonia

1186. a 68-year-old patient complains of sharp pain in the right foot, swelling of the fingers and
darkening of the skin of the fourth finger. He has been suffering from diabetes for 15 years and is treated on an irregular basis. What
complication of diabetes mellitus did the patient have?
100% Gangrene of the fourth toe of the right foot
0% Panaritius
0% Hematoma
https://translate.yandex.com/en/doc 262/540
22:20 ,27.6.2023 �� C : ; 5 B
0% Erysipelas
0% Fracture of the fourth toe of the right foot

1187. An 11-year-old boy's posture was determined during a routine medical examination
. It was found that the child's shoulders are tilted and brought forward,the head is tilted forward,
the chest is flattened, the abdomen is convex. In the spine, there is an increase in the depth
of the cervical and lumbar curvatures. What kind of posture is found in the child?
100% Kyphosis
0% Lordoz
0% Stooped
0% Fixed
0% Normal

1188. during the hygienic examination of the hospital, it was established that the area per bed in wards
with two beds or more is: in the infectious diseases department for children - 7 m2, in the infectious
diseases department for adults-8 m2, in the burn wards-9 m2, in the radiology department-10 m2, in
the intensive care wards-13 m2. Which wards do not have enough space for one bed to meet
the hygiene requirements?

273

Downloaded from the site - online testing step

100% In burn areas


0% In infectious diseases for children
0% In infectious diseases for adults
0% For radiological purposes
0% In intensive care units

1189. We studied the incidence of influenza and acute respiratory viral infections over the past 5 years. What kind of graphic
image is most appropriate to use to visually display this data?
100% Line Chart
0% Pie Chart
0% Bar chart
0% Histogram
0% Radial diagram

1190. A 46-year-old taxi driver suffered a stroke, which left him with right-sided hemiparesis.
The driver is recognized as a disabled person of group I. Determine which health group the specified person belongs to?
100% 5
0% 4
0% 3
0% 2
0% 1

1191. in a hospital corridor, a 55-year-old patient suddenly became ill and was immediately examined by a doctor.
On examination: the skin is pale, there is no independent breathing, there is no pulse in the carotid arteries
, the pupils are not dilated. What measures should be taken to start resuscitation in order to restore
cardiac activity?
100% Precardial stroke
0% Mouth-to-mouth breathing
https://translate.yandex.com/en/doc 263/540
22:20 ,27.6.2023 �� C : ; 5 B
0% External heart massage
0% Improvement of airway patency
0% Defibrillation

1192.at the material sterilization facility (a radioactive closed source of cobalt-60),


due to a gross violation of the operating rules, external acute ionizing
radiation occurred to two operators. The victims were taken to the hospital 3 hours after the accident.
What are the priority measures needed in the hospital?
100% Symptomatic therapy
0% Iodine prevention
0% Antidote therapy
0% Decontamination
0% Iemotransfusion

1193. A 24-year-old female patient complained of weight gain and increased appetite.
Objectively: hyperstyle, body mass index 33.2 kg / m2, waist circumference 100 cm.

274

Downloaded from the site - online testing step

The ratio of waist circumference to hip circumference is 0.95. what is the most likely
diagnosis?
100% Alimentary-constitutional obesity, etc., abdominal type
0% Hypothalamic obesity according to the Itsenko-Cushing type, II century, genoid type
0% Alimentary-constitutional obesity, III century, genoid type
0% Alimentary-constitutional obesity, II century, abdominal type
0% Hypothalamic obesity according to the Itsenko-Cushing type, I century, abdominal type

1194. a 27-year-old woman in labor, giving birth II, urgent, normal. Day 3 of the postpartum period.
Body temperature 36.8°C, Ps-72 / min, blood pressure-120/80 mm Hg. mammary glands are moderately rough, nipples
are clean. The belly is soft, painless. The bottom of the uterus is 3 transverse fingers below the navel. Lochia
are bloody, moderate. What is the most likely diagnosis?
100% Physiological course of the postpartum period
0% Subinvolution of the uterus
0% Postpartum metroendometritis
0% Remnants of placental tissue after delivery
0% Lactostasis

1195. a 19-year-old patient, after a superficial skin injury on the palmar surface
of the nail phalanx of the second finger, developed minor pain and a blister filled with pus. There are signs of lymphangitis on the back
of the hand. What is the most likely diagnosis?
100% Skin panaritium
0% Erysipelas
0% Lymphangitis
0% A boil
0% Bone panaritium

1196. a 46-year-old patient has diarrhea with the release of a large amount of cough-like feces
of an unpleasant smell, without blood admixture and without tenesmus, bloating, and weight loss.
Objective examination revealed moderate pain in the mesogastrium and left side of the abdomen. In

https://translate.yandex.com/en/doc 264/540
22:20 ,27.6.2023 �� C : ; 5 B
a coprological study-steatorrhea due to neutral fat, creatorrhea. In this
case, it is most rational to assign:
100% Polyenzyme preparations
0% Holinolytics
0% Metronidazole and Loperamide
0% Antacids and antispasmodics
0% Anticholinergics and antibacterial drugs

1197. A 72-year-old patient with severe concomitant pathology suffered from esophageal wall damage during urgent
fibroesophagogastroscopy.
The symptoms of acute respiratory failure began to progressively increase and a collapse of the left lung occurred. What
kind of help should the patient receive?
Bullau pleural drainage, mediastinal drainage,
100%
antibacterial therapy
0% Drainage of the pleural cavity according to Buellau, antibacterial therapy

275

Downloaded from the site - online testing step

0% Left-sided thoracotomy, suturing of the mediastinal and esophageal wounds


0% Left-sided thoracotomy with esophageal wound suturing
0% Endoscopic suturing of esophageal wound, drainage

1198. a 46-year-old woman who had been suffering from hypertension for 5 years
developed a hypertensive crisis. Complaints of palpitation, throbbing sensation in the head, heart rate-100 / min,
AT-190/100 mm Hg. which drug should be preferred?
100% beta-blocker
0% ACE Inhibitor
0% Diuretic
0% alpha-adrenoblocker
0% Dihydropyridine calcium antagonist

1199. a 32-year-old patient complains of "heartache", bad mood, lack of appetite,


insomnia. These symptoms appeared gradually over the course of 3 months, for no apparent reason.
In mental status: answers the question in a low voice, the face is sad, the facial expressions are mournful,
the movements are slow, the mood background is reduced, emotionally depressed, the pace of thinking is slowed down.
The patient's condition improves in the evening and worsens early in the morning. Which specialist should
the patient be referred to?
100% Psychiatrist
0% Neurologist
0% Endocrinologist
0% Gastroenterologist
0% Medical psychologist

1200. A 30-year-old patient complains of not being pregnant for 3 years after
marriage. Objectively: increased nutrition, along the midline of the abdomen, on the inner
surface of the thighs and in the periosal area, hair growth is noted. Menstruation from
the age of 16, fluid and unexplained. Ultrasound: uterus of normal size, ovaries 4x5x5 cm, with a large number
of cystic inclusions. What is the most likely diagnosis?
100% Polycystic ovary disease
0% Ovarian cysts
0% Chronic oophoritis
https://translate.yandex.com/en/doc 265/540
22:20 ,27.6.2023 �� C : ; 5 B

0% Menstrual disorders
0% Bilateral ovarian tumors

1201. a 4-month-old child was admitted to the surgical department after 8 hours with
anxiety attacks for 2-3 minutes, at intervals of 10 minutes. There was a single vomiting event.
Objectively: the child's condition is serious. During palpation, the abdomen is soft, and a tumor-like formation is detected in the right
iliac region
. Rectal examination shows blood on the finger. What
is the100%
most likely diagnosis?
Ileocecal invagination
0% Gastrointestinal bleeding
0% Wilms ' tumor
0% Silt infestation

276

Downloaded from the site - online testing step

0% Pylorostenosis

1202. during a professional examination, a 16-year-old patient does not complain. Objectively: low
nutrition, asthenic, blood pressure-110/70 mm Hg, Ps-80 / min, heart borders are normal,
three tones are heard above the apex of the heart during auscultation, no noises are present. ECG-no pathological changes were detected.
PCG-the third tone is detected above the tip after 0.15 seconds. after the second tone. How can
these changes be interpreted?
100% III-th physiological tone
0% Rhythm of"quail"
0% Proto-diastolic rhythm of "gallop"
0% Presystolic rhythm of the "gallop"
0% ivth physiological tone

1203. a 45-year-old man, a loader, complains of pain in his lower back and right leg. This
symptom persists for 2 months and did not decrease after a course of conservative therapy.
Objectively: the dynamics of the spine in the lumbar region is limited, the Lasega symptom is positive
on the right. Knee reflexes D= S, live. Achilles: S > D, sharply reduced on the right. What
examination should the patient perform to clarify the diagnosis?
100% Magnetic resonance imaging
0% Radiography of the spine
0% Electromyography of leg muscles
0% Leg reovasography
0% -

1204. A 28-year-old patient, 2 hours after eating unknown mushrooms, felt a decrease
in mobility and concentration, which later changed to agitation and aggression. Upon inspection:
disorientation, incomprehensible speech. After 4 hours, hepatic bad breath,
fainting. What syndrome does the patient have?
100% Acute liver failure
0% Hepatolienal syndrome
0% Portal hypertension
0% Cholestatic syndrome
0% Icteric syndrome

https://translate.yandex.com/en/doc 266/540
22:20 ,27.6.2023 �� C : ; 5 B
1205. A 52-year-old patient with widespread spinal osteochondrosis
developed pain in the lower back and along the left sciatic nerve after lifting weights. Objectively: on the left side
there is a positive Lasega symptom and a reduced Achilles reflex. Which drug
is most pathogenetically justified?
100% Diclofenac
0% Aspirin
0% Analgin
0% Novocaine
0% Spasmalgon

277

Downloaded from the site - online testing step

1206. an 8 - year-old child developed a sub-febrile fever, arthritis, transient


colic-like abdominal pain, and a pur-pur-like rash on the lower extremities. In the urine:
erythrocyturia and protea-Nuria. In the blood: the number of platelets is normal. The most likely
diagnosis is:
100% Schonlein-Henoch disease
0% SLE
0% Rocky Mountain Fever
0% Werlhof's disease
0% Post-streptococcal glomerulonephritis

1207. A 52-year-old woman was admitted to the emergency department with an epileptic
seizure. The first step in providing assistance in this case is to:
100% Make sure that the patient's airway is clear and sufficiently oxygenated
0% Fix the language
0% Enter a 50% dextrose solution in an intravenous jet
0% Enter 5 mg of diazepam followed by phenytoin
0% Cause a medically induced pentobarbital coma

1208. when is it possible to determine the gestational age with the greatest accuracy based on the size of the uterus?
100% Up to 12 weeks
0% Between 12 and 20 weeks
0% Between 21 and 30 weeks
0% Between 31 and 40 weeks
0% After 40 weeks

1209. a 45-year-old woman complains of progressive weakness in the last 6 weeks,


discomfort and bloating. I didn't lose any weight, but I became apathetic. Constantly uses
alcohol. Objectively: the abdomen is enlarged, tense. Defecation is not disturbed. Ultrasound
of the abdominal organs-a small amount of ascitic fluid; liver, kidneys, spleen
are unchanged, there are several cysts in the left ovary, the right one is not visualized due to the formation associated with
the omentum. In the urine-the norm. What is the most likely diagnosis?
100% Ovarian carcinoma
0% Colon lymphoma
0% Sigmoid colon cancer
0% Alcoholic liver disease
0% Crohn's disease

https://translate.yandex.com/en/doc 267/540
22:20 ,27.6.2023 �� C : ; 5 B

1210. a 21-year-old man complains of back pain, which occurs in the last 3 months, in the morning
. Pain decreases throughout the day and after exercise. Objectively: limited
mobility in the lumbar spine, increased muscle tone in the lumbar region and
hunchback during movement. On the X-ray of the spine: bilateral sclerotic changes in
the lumbar-lumbar region. What test would be most appropriate to confirm
the diagnosis?
100% HLA-B27
0% SHZ

278

Downloaded from the site - online testing step

0% Rheumatoid factor
0% Uric acid in blood plasma
0% Antinuclear antibodies

1211. a 44-year-old man developed dysarthria, Horner's syndrome on the right side,hiccups,
right-sided ataxia, loss of pain sensitivity of the face on the right and trunk on the left. Consciousness
is preserved. CT scan of the brain is within the normal range. The most rational action would be:
100% Administration of direct anticoagulants and follow-up
0% Endarterectomy of the right carotid artery
0% Endarterectomy of the left carotid artery
0% Outpatient monitoring
0% Surgical decompression of the brain

1212. a 65-year-old man with difficulty urinating due to benign


prostatic adenoma developed fever with chills, hypotension, and sinus tachycardia.
The skin is warm and dry. There is absolute neuropenia in the blood. What causes such hemodynamic
changes?
100% Endotoxemia with complement system activation
0% Secondary reflex vasodilation-this, as a result of a decrease in cardiac output
Secondary hemo6igu insufficiency with preserved systolic function, as a
0%
result of peripheral vasoconstriction
0% Reflex vagal stimulation with reduced cardiac output
0% Secondary endothelial changes resulting from bacterial damage

1213. a 32-year-old woman, 39 weeks pregnant, second delivery. Regular labor


activity began. Uterine contractions every 3 minutes. Which of these criteria most accurately
determine the beginning of the second stage of labor?
100% Opening of the cervix at least 4 cm
0% Smoothing of the cervix more than 90%
0% Duration of uterine contractions more than 30 seconds
0% Presenting part in the lower part of the pelvis
0% Rupture of the amniotic sac

1214. a 20-year-old male has a stab wound in the left side of the chest
medial to the nipple. Objectively: AT - 90/60 mm Hg, Ps-130/min, PDR-32 / min. During inspiration
, there is an increase in the pulse wave in the area of the jugular vein, a weakening
of the peripheral arterial pulse and a decrease in blood pressure. Breathing sounds are not
changed. Chest X-ray without special features. After the introduction of two liters

https://translate.yandex.com/en/doc 268/540
22:20 ,27.6.2023 �� C : ; 5 B
of isotonic solution, blood pressure remains low, the CVT has risen to 32 cm of water.
the first
100% step in further patient
Performing an management
ECG is:
Catheterization of the left pleural cavity with the outer end
0%
of the catheter placed under water
Continuation of parenteral fluid administration until blood
0%
pressure increases

279

Downloaded from the site - online testing step

0% Introduction of peripheral vasodilators to reduce CVT


0% Introduction of loop diuretics to reduce CVT

1215. in a young man, painful seals are noted in the periarticular areas of both mammary glands
. These seals should be:
100% Leave it alone
0% Remove
0% Open and drain
0% Take an aspirate for backseeding and cytology
0% Prescribe topically steroids

1216. a 34-year-old woman, 29 weeks pregnant, giving birth on the 4th day. She was admitted to the obstetric department
with complaints of sudden painful vaginal discharge that appeared 2 hours ago. Selections are plentiful,
with the presence of convolutions. Fetal heart activity is rhythmic-150 / min, uterine tone is normal. What
is the most likely diagnosis?
100% Placenta previa
0% Detachment of the normally located placenta
0% Presentation of the vessel
0% Spotting
0% DIC-syndrome

1217. a 65-year-old man has been smoking for 40 years. I've lost 10 kg of weight in recent months.
Complains of epigastric pain after eating, diarrhea, jaundice.
Objectively: enlarged, painless gallbladder. Feces discolored, clay-like. In
the blood: increased levels of total and direct bilirubin, alkaline phosphatase and
glutamine pyruvate transferase. In the urine: positive reaction to bilirubin and negative reaction to urobilinogen.
Where is the primary process that led to these changes located?
100% The pancreas
0% Common bile duct
0% Liver
0% The duodenum
0% The gallbladder

1218. A 75-year-old man experiences acute


pain in the parotid-ciliary region approximately 30 minutes after eating, accompanied by vomiting and a feeling of bloating. Over
the past few months, he has lost 10 kg of weight because he doesn't eat, preventing pain. There are no changes between the pain
attacks during the examination of the abdomen. Noise is heard above the right femoral artery
, and peripheral pulsation in the lower extremities is weakened. X-ray examination
of the stomach and colonoscopy revealed no changes. What is the leading factor in the pathogenesis of these changes?
100% Ischemia
0% Psychogenic changes
https://translate.yandex.com/en/doc 269/540
22:20 ,27.6.2023 �� C : ; 5 B
0% Neoplastic process
0% Inflammation
0% Temporary obstruction
1219. A 75-year-old woman with CHD is constantly taking warfarin. Hospitalized in the department

280

Downloaded from the site - online testing step

emergency treatment with complaints of sudden weakness in the left side of the body and deviation
of the eyeball to the right side. What primary research should the patient conduct?
100% Computed tomography of the brain
0% MRI of the brain
0% Electroencephalogram
0% Ultrasound examination of the carotid arteries
0% Spinal puncture

1220. a 65-year-old man developed acute pain, paresthesia, and pallor of the left lower limb.
Objectively: there is no pulse on A. dorsalis pedis on the left. There is a cooling of the skin and
pallor, which gradually spread up the limb. These symptoms
are most likely to indicate that:
100% Arterial occlusion
0% Superficial vein thrombophlebitis
0% Herniated lumbar vertebral disc
0% Deep vein thrombophlebitis
0% -

1221. A 58-year-old man developed an acute myocardial infarction 4 hours ago and
is currently in the emergency department. Short paroxysms
of ventricular tachycardia are recorded on the ECG. Which drug is most appropriate to administer?
100% Lidocaine
0% Flecainide
0% Amiodaronum
0% Propafenone
0% Verapamil

1222. A 42-year-old woman was found to have changes in the mammary gland. What are the most common symptoms
that precede malignancy?
100% Tightening of the skin with a retracted nipple
0% Painful moving seal
0% Painless movable seal
0% Bloody discharge from the nipple
0% Clear discharge from the nipple

1223. a patient taking diuretics developed arrhythmia as a result of an overdose


of cardiac glycosides. What is the treatment strategy in this case?
100% Increase in the concentration of potassium in the blood
0% Increased blood sodium concentration
0% Reducing the concentration of magnesium in the blood
0% Increased blood calcium levels
0% -

https://translate.yandex.com/en/doc 270/540
22:20 ,27.6.2023 �� C : ; 5 B

1224. the victim is in a state of traumatic shock. Objectively: AT-70 mm Hg, heart rate-140/min.

281

Downloaded from the site - online testing step

Specify the approximate amount of blood loss based on the shock index:
100% 30%
0% 20%
0% 40%
0% 50%
0% 60%

1225. A 27-year-old patient notes fatigue, sweating, and heaviness in the left hypochondrium for about a year,
especially after eating. Objectively: enlargement of the spleen, liver. In the blood: Er -3.2 •
1012/l, Hb-100 g/l, CP-0.87, Leuc. -100 * 109/l, B.-7%, E.-5%, M.-15%, Y. -16%, P.-10%, S.-45%,
lymph. -2%, mon. -0%, reticule.- 0.3%, blood clot.- 400 • 109/l, WSE-25 mm/hour. What is the most
likely diagnosis?
100% Chronic myeloid leukemia
0% Chronic lymphocytic leukemia
0% Acute leukemia
0% Erythremia
0% Cirrhosis of the liver

1226. A 34-year-old patient has been suffering from pulmonary tuberculosis for 7 years; complains of
muscle weakness, weight loss, diarrhea, and frequent urination. Objectively:
hyperpigmentation of the skin, gums, and inner cheek surfaces. AT-90/58 mm Hg in the blood: Er -3.1 • 1012/l, Hb-95
g/l, CP-0.92; leuc.-9.4 • 109/l, E.-7, S.-45, P.-1, L.-40, M. -7, Na+ - 115 mmol / l, k+ - 7.3 mmol/l.
what is the most likely diagnosis?
100% Primary insufficiency of the cranial cortex
0% Pheochromocytoma
0% Primary hyperaldosteronism
0% Congenital hyperplasia of the adrenal cortex
0% Diabetes insipidus

1227. A 43-year-old patient complains of head pain, mainly in


the morning, and vomiting for five months. Recently, the pain was worse when changing the position of the head.
Objectively: anomaly, stasis of the papillae of the optic nerves, ataxia, Yanishevsky's symptom, decreased
memory and criticism. Craniography: signs of hypertensive and cerebrospinal fluid syndrome. On the Eheg:
displacement of the median structures of the brain to the right side by 5 mm. What is the most likely diagnosis?
100% Tumor of the frontal lobe of the brain
0% Occipital lobe abscess of the brain
0% Chronic cerebrovascular insufficiency
0% Tumor of the left hemisphere of the cerebellum
0% Alzheimer's disease

1228. a full-term child suffered from ante-and intra-natal hypoxia and was born in asphyxia
(Apgar score 2-5 points). After birth, the child
's agitation progresses,vomiting, nystagmus, convulsions, strabismus, spontaneous Moreau and
Babinsky reflexes are noted. What localization of intracranial hemorrhage is most likely?
100% Subarachnoid hemorrhage

https://translate.yandex.com/en/doc 271/540
22:20 ,27.6.2023 �� C : ; 5 B

282

Downloaded from the site - online testing step

0% Minor brain tissue hemorrhages


0% Subdural hemorrhage
0% Periventricular hemorrhage
0% Hemorrhages in the ventricles of the brain

1229. a 64-year-old patient 2 hours ago developed compressive pain behind the sternum radiating
to the left shoulder, pronounced weakness. Objectively: the skin is pale, cold sweat. Ps-108 / min., AT-70/50
mm Hg. heart tones are deaf. Respiration is vesicular. The belly is soft, painless.
Varicose veins on the left shin. On the ECG: sinus rhythm, heart rate-100/min, sharp rise
of the ST segment above the isoline in leads II, III, aVF. What pathology did the patient have?
100% Cardiogenic shock
0% Cardiac asthma
0% Pulmonary embolism
0% Dissecting aortic aneurysm
0% Cardiac tamponade

1230. a 39-year-old patient with clinical signs of peptic ulcer disease in combination with severe
diarrhea. In the anamnesis-three duodenal ulcers, which often recur. One of
the ulcers is located close to the small intestine. Gastrin blood serum 200 pg / ml. What
would be the most informative study in this case?
100% Secretin Injection Test
0% Colonoscopy
0% Endoscopic retrograde cholangiography
0% CT scan of the abdominal cavity
0% Radiography of the OCP

1231. in a healthy 75-year-old woman who leads a moderately active lifestyle, a


preventive examination revealed a serum concentration of total cholesterol at the level
of 5.1 mmol/l (208 mg / dl) and HDL cholesterol-70 mg / dl. ECG-without pathology. Which of
these dietary recommendations is most acceptable?
100% No dietary changes
0% Reducing cholesterol intake
0% Reduced saturated fat intake
0% Reduce your intake of simple carbohydrates
0% Increase your fiber intake

1232. A 26-year-old patient suffered a closed chest injury 2 days ago as a result of a car accident
. Complaints of pain in the affected area, severe shortness of breath, tachycardia, general
weakness. Above the lungs on the right side, below the VI rib, blunted percussion sound,
sharply weakened breathing. During the puncture of the pleural cavity, hemorrhagic fluid was obtained.
What post-traumatic complication are we talking about?
100% Right-sided hemothorax
0% Closed pneumothorax
0% Exudative pleurisy
0% Right-sided hydrothorax

283

https://translate.yandex.com/en/doc 272/540
22:20 ,27.6.2023 �� C : ; 5 B

Downloaded from the site - online testing step

0% Post-traumatic chylothorax

1233. A 54 - year-old woman underwent laparotomy for a large mass in her pelvis, which
turned out to be a unilateral ovarian tumor with significant metastases to the omentum. The most
appropriate intraoperative tactic involves::
100% Removal of the omentum, uterus and both ovaries with tubes
0% Omentum biopsy
0% Ovarian biopsy
0% Removal of ovary and omentum metastases
0% Removal of the omentum and both ovaries with tubes

1234. a maternity patient was admitted to the maternity hospital with regular labor,
and amniotic fluid was discharged. The fetal head is tightly pressed against the entrance to the small pelvis. Fetal heartbeat
is clear, rhythmic-136 / min. During vaginal examination: opening of the neck 7 cm, arrow-shaped seam
in the right oblique size, small crown in front and below the large one, located on the left.
Determine the position and type of fetus:
100% First position, foreground view
0% Second position, front view
0% First position, rear view
0% Second position, rear view
0% Occipital presentation, anterior view

1235. in a 16-year-old boy with no clinical symptoms, cardiac auscultation revealed


an accent of tone II and systolic murmur over the pulmonary artery. The heart tones are sonorous and rhythmic. What
is the most likely diagnosis?
100% Functional noise
0% Pulmonary artery valve stenosis
0% Pulmonary artery valve insufficiency
0% Non-infection of the Botal duct
0% Atrial septal defect

1236. A 74-year-old patient has been suffering from hypertension for about 20 years. Complains of
frequent headache, dizziness; takes Enalapril. Objectively: the accent of the second tone over
the aorta, Ps-84/min, rhythmic, blood pressure-180/120 mm Hg. what group of antihypertensive drugs
should be prescribed additionally, taking into account age?
100% Tazide diuretics
0% Loop diuretics
0% b-blockers
0% "- adrenoblockers
0% Central sympatholytics

1237. A 10-year-old boy with hemophilia has symptoms of acute


respiratory viral infection with fever. Which of the above drugs, administered
for antipyretic purposes, is contraindicated in this patient?

284

https://translate.yandex.com/en/doc 273/540
22:20 ,27.6.2023 �� C : ; 5 B

Downloaded from the site - online testing step

100% Acetylsalicylic Acid


0% Analgin
0% Pipolfen
0% Paracetamol
0% Panadol extra

1238. The Deputy Chief Physician for Medical Work conducted a study of the level
of morbidity of the population that was served in the polyclinic for the last 5 years.
What statistics can it use to calculate the prevalence of diseases?
100% Relative values
0% Standardized values
0% Average values
0% Absolute values
0% Dynamic range

1239. a 58-year-old patient, accountant, has been suffering from knee osteoarthritis for 2 years.
She was treated in a hospital for 2 weeks. She was discharged in satisfactory condition with complaints of minor
pain after prolonged static exercise. Local hyperthermia and exudative
phenomena in the joints are absent. What is the most appropriate further management strategy for the patient?
100% Treatment in a sanatorium
0% Repeated inpatient treatment
0% Perform an arthroscopy
0% Send it to MSEC
0% Orthopedic consultation

1240. a 52-year-old patient complains of joint and muscle pain, muscle weakness. He's been ill
for about a year. Treatment with nonsteroidal anti-inflammatory drugs was ineffective.
Objectively: independent movements of the trunk and limbs are difficult, with dark erythema in the paraorbital region
. Palpation of the shoulder girdle and thigh muscles is painful. Heart sounds are weakened,
systolic murmur above the apex. What is the most likely underlying pathogenetic mechanism
of this disease?
100% Synthesis of myosin-specific antibodies
0% Hyperproduction of collagen
0% Formation of antibodies to RNA
0% Formation of antibodies to native DNA
0% Formation of circulating immune complexes

1241. a patient complains of periodic accumulation of red blood impurities in the stool after the act
of defecation. He has been ill for more than 5 years. The disease is associated with heavy physical
exertion. Bleeding increases after physical exertion or drinking
alcoholic beverages. Bowel movements 1 time in 2-3 days, painless, each time
accompanied by loss of hemorrhoids. What is the most likely diagnosis?
100% Internal bleeding hemorrhoids
0% Rectal cancer
0% Acute proctosigmoiditis

285

Downloaded from the site - online testing step

https://translate.yandex.com/en/doc 274/540
22:20 ,27.6.2023 �� C : ; 5 B
0% External bleeding hemorrhoids
0% Diverticulosis of the small intestine

1242. A 43-year-old patient underwent cholecystectomy for chronic


calculous cholecystitis 6 years ago. For the last six months, I started noticing pain in the right hypochondrium and
transient jaundice. The last 2 weeks of jaundice does not go away. During the examination
, stenosing papillitis with a length of up to 0.5 cm was detected. Choose the best treatment option for the patient:
100% Perform endoscopic papilosphincterotomy
Conduct conservative treatment: antispasmodics, antibiotics, anti-inflammatory
0%
drugs
0% Perform external drainage of khole-dokha
0% Perform transduodenal papilo-sphincterotomy
0% Perform choledochoduodenostomy

1243. A 30-year-old Pakistani citizen has become acutely ill: frequent watery
bowel movements that resemble "rice broth" have appeared. Objectively: t0-35.40 S, cold limbs,
acrocyanosis, dry mouth, sharply reduced tissue turgor. What should be done to assess the degree
of dehydration?
100% Determine the density of blood plasma
0% Measure central venous pressure
0% Determine the level of urea and creatinine in the blood
0% Measure your blood pressure
0% Determine the level of hemoglobin

1244.
Cases of chronic and asthmatic bronchitis and bronchial asthma have increased among preschool-aged children in an industrial district of
an industrial city
. Which
100%of the persistent air pollutants could have caused them?
Sulfur dioxide
0% Carbon monoxide
0% Lead
0% Nitrogen oxides
0% Products of photochemical reactions

1245. In region H, the prevalence of diseases among the population was 1,156 cases per 1000
population. Which of the following indicators characterizes the prevalence of diseases?
100% Intensive
0% Extensive
0% Ratio
0% For visibility purposes
0% Standardized

1246. A 29-year-old female patient went to the gynecologist with complaints of irritability, tearfulness,
headache, nausea, sometimes vomiting, heart pain, tachycardia attacks,
memory loss, flatulence. These complaints occur 6 days before menstruation and disappear the day before or after menstruation.

286

Downloaded from the site - online testing step

in the first two days. Vaginally: uterus and appendages unchanged. What is the most likely diagnosis?
100% Premenstrual syndrome

https://translate.yandex.com/en/doc 275/540
22:20 ,27.6.2023 �� C : ; 5 B
0% Algodismenorrhea
0% Ovarian apoplexy
0% Genital endometriosis
0% Neurosis

1247. A 46-year-old patient was diagnosed with functional class II angina pectoris
with stage II hypertension. Blood pressure in the range of 160/105 mm Hg. which
antihypertensive drug should be preferred?
100% Metoprolol
0% Enalapril
0% Doxozazine
0% Clonidine
0% Adelphanum

1248. a patient with left lung mandibular pneumonia complained of pain in the left
side of the chest. Objectively: a wide area of bluntness on the left side
of the chest (Sokolov-Alison-Damoiseau line) is determined. What method of lung examination is advisable
to start an examination to clarify the diagnosis?
100% Radiography
0% Bronchoscopy with biopsy
0% Bronchography
0% Spirography
0% Thoracoscopy

1249. a 3-year-old girl has an increase in body temperature up to 380C, which lasts
for the second day, a runny nose, a dry superficial cough, weakness, and a decrease in appetite. There were no palpatory
changes over the lungs. Percussive sound with a boxy tinge, auscultative-
puerile breathing, no wheezing. In the blood - leukopenia, lymphocytosis, accelerated SSE. What
is the most likely diagnosis?
100% Acute simple tracheitis
0% Acute obstructive bronchitis
0% Recurrent bronchitis, acute phase
0% Acute simple bronchitis
0% Bilateral small-focal pneumonia

1250. a patient suffers from chronic recurrent pancreatitis with a pronounced violation
of the external secretory function. After eating fatty spicy food, alcohol,
"oily"feces appear. Reduced production of what factor is the most likely cause of steatorrhea?
100% Lipase
0% Trypsin
0% Acidity of gastric juice
0% Amylase

287

Downloaded from the site - online testing step

0% Alkaline Phosphatase

1251. A 54-year-old female patient has been suffering from femoral osteomyelitis for more than 20 years. Over the past
month, edema of the lower extremities has appeared and gradually increased. In the urine: proteinuria - 6.6 g / l

https://translate.yandex.com/en/doc 276/540
22:20 ,27.6.2023 �� C : ; 5 B
. in the blood: dysproteinemia in the form of hypoalbuminemia, an increase in A2-and y-globulins, SSE-50
mm/h . What is the most likely diagnosis?
100% Secondary amyloidosis of the kidneys
0% Acute glomerulonephritis
0% Myeloma disease
0% Chronic glomerulonephritis
0% Systemic lupus erythematosus

1252. a 28-year-old patient is a drug addict. He has been ill for a year, when he noted general weakness,
increased sweating, and weight loss. He was often ill with respiratory diseases. For the last 2 days
there was an intermittent fever with profuse night sweats, general weakness increased,
diarrhea with admixtures of mucus and blood joined. On examination: poly-lymphadenopathy,
herpetic rashes in the oral cavity, during palpation of the abdomen - enlarged liver and spleen. What
is the most likely diagnosis?
100% AIDS
0% Tuberculosis
0% Chronic lymphocytic leukemia
0% Colon cancer
0% Chroniosepsis

1253. A 62-year-old patient complains of a protrusion in the left groin area, which
gradually increased. Objectively: in the left inguinal region, the protrusion is 5-6 cm in size,
dense, elastic, painful, does not set in the abdominal cavity, the skin above it is not changed,
the outer inguinal ring passes the tip of the finger. What is the most likely diagnosis?
100% Inguinal lymphadenitis
0% Fixable left-sided inguinal hernia
0% Left-sided pinched inguinal hernia
0% Nonrecoverable left-sided inguinal hernia
0% Tumor formation

1254. after a long period of subfebrality, the patient noticed increased shortness of breath, pain in
the right hypochondrium, and swelling of the legs. Objectively: the cervical veins are swollen. Ps-120 / min, sometimes disappears
during inhalation. Heart tones are dramatically weakened. On the ECG: reduced voltage of the ventricular
complex teeth. A month ago, there was a rise in the ST V1-V4 segment. The shadow of the heart is broad and rounded.
What is the most likely diagnosis?
100% Exudative pericarditis
0% Small-focal myocardial infarction
0% Post-infarction cardiosclerosis
0% Metabolic postinfectious Myocardiopathy
0% Primary rheumocarditis
1255. in a 14-year-old child suffering from vegetative-vascular dystonia of puberty,

288

Downloaded from the site - online testing step

a sympatho-drenal crisis developed. Which drug is indicated for crisis management?


100% Obzidan
0% No-shpa
0% Amizil
0% Euphyllinum
0% Korglikon

https://translate.yandex.com/en/doc 277/540
22:20 ,27.6.2023 �� C : ; 5 B

1256. A patient with acute respiratory viral infections went to see a doctor. The patient was declared disabled.
The attending physician issued a disability certificate for 5 days. The patient continues to be ill. What
should the attending physician do to further register the patient's disability?
Extend the disability certificate independently, but not for more than 10 days
100%
in total
Extend the disability certificate independently, but not for more than 6 days
0%
in total
0% Extend the disability certificate together with the head of the department
0% Refer the patient to a medical advisory board
0% Refer the patient to the medical and social expert commission

1257. a 64-year-old patient complains of general weakness, noise in the head, hoarseness of voice.
Objectively: the skin is pale with an icteric tinge, the tongue is red with smooth papillae,
asymmetry of tactile and pain sensitivity, Ps-120 / min, blood pressure-80/50 mm Hg
, the spleen is palpated. In the blood: HB-58 g / l, ep. -1, 2-1012/l, leuc.- 2,8-109/l, tr.- 140- 109/ l, SSE-17 mm/h,
anisocytosis, poikylocytosis-pronounced (++). What research will be crucial to elucidate
the genesis of anemia?
100% Sternal puncture
0% Indirect Coombs test
0% Direct Coombs test
0% Fibrogastroscopy
0% Endolumbal puncture

1258. a 27-year-old patient, 10 hours after eating canned mushrooms, developed


diplopia, bilateral ptosis, swallowing disorders, shallow breathing with a frequency of 40 / min, Muscle
weakness, intestinal paresis. What medical event should be held first?
100% Tracheal intubation for artificial respiration
0% Gastric and intestinal lavage
0% Introduction of anti-botulinum serum
0% Introduction of glucocorticosteroids
0% Intravenous detoxification therapy

1259. a 4-year-old boy was admitted to the hospital with complaints of shortness of breath, rapid
fatigue. A history of frequent respiratory diseases. Per-cutorno: the heart borders
are extended to the left and up. Auscultation: amplification of the II tone above the pulmonary artery,in the II - III
intercostal space to the left of the chest,a rough systolic "machine"noise is heard,
produced in all other points and on the back. AT - 100/20 mmHg. what is the most likely
diagnosis?

289

Downloaded from the site - online testing step

100% Open Ductus arteriosus


0% Ventricular septal defect
0% Nolated stenosis of the pulmonary artery opening
0% Atrial septal defect
0% Valvular aortic stenosis

1260. the patient was hospitalized with complaints of periodic pain in the lower abdomen, which increases
during menstruation, weakness, malaise, nervousness, smearing dark blood discharge from

https://translate.yandex.com/en/doc 278/540
22:20 ,27.6.2023 �� C : ; 5 B
the vagina on the eve and after menstruation. During bimanual examination: the body of the uterus is enlarged,
appendages are not defined, in the posterior arch - a bumpy surface. During laparoscopy: on
the ovaries, peritoneum of the utero-rectal depression and pararectal tissue -
"blue eyes". What is the most likely diagnosis?
100% Common form of endometriosis
0% Polycystic ovary disease
0% Chronic salpingitis
0% Tuberculosis of the genitals
0% Ovarian cysts

1261. a 28-year-old patient without a permanent place of residence, who was hospitalized with a preliminary
diagnosis of "flu", on the 5th day of the disease developed a roseolous-petechial rash on the trunk and
internal surfaces of the limbs. Temperature 410c, euphoria, flushing of the face, redness
of the sclera, tremor of the tongue, tachycardia, splenomegaly, agitation. What is the most likely diagnosis?
100% Typhus fever
0% Alcoholic delirium
0% Leptospirosis
0% Measles
0% Typhoid fever

1262. a 9-month-old child with acute respiratory viral infections suddenly developed seizures, general cyanosis,
and loss of consciousness. Objectively: clonic-tonic convulsions, cyanotic skin, foam on the lips,
signs of rickets. No pathological changes were detected on the part of the internal organs. What medications
should be administered to the child first?
100% Anticonvulsants
0% Calcium supplements
0% Glucocorticoids
0% Antirachitic drugs
0% Sedatives

1263. the child is 2 days old. She was born full-term with signs of intrauterine infection,
and therefore the child was prescribed antibiotics. Please indicate why the interval between administration
of antibiotics in newborns is longer compared to older children and adults, and
the dose is lower?
100% Newborns have lower glomerular filtration rate
0% Newborns have a lower concentration of protein and albumins in the blood
0% Newborns have reduced glucuronyltransferase activity

290

Downloaded from the site - online testing step

0% Newborns have low blood pH


0% Newborns have a higher hematocrit

1264. A woman tested positive for pregnancy during a doctor's visit on 10 July 1997.
She has a regular 28-day cycle. The last menstrual period was from May 1 to May 4, 1997. What
is the expected delivery time?
100% February 8, 1998
0% February 1, 1998
0% February 15, 1998
0% February 22, 1998

https://translate.yandex.com/en/doc 279/540
22:20 ,27.6.2023 �� C : ; 5 B
0% February 28, 1998

1265. A 20-year-old patient underwent VAMP polychemotherapy for acute


lymphoblastic leukemia. What morphological picture of blood can indicate the onset of remission?
100% Blast cell count up to 5%
0% Blast cell count up to 15%
0% Blast cell count up to 10%
0% Blast cell count up to 1%
0% Absence of blast cells

1266. a patient was admitted to the cardiology department with complaints of aching pain in
the atrial region, moderate shortness of breath, swelling of the legs, and chilliness. On the ECG: negative
T waves V2-V6 (up to 34 mm). Objectively: the skin is dry, the voice is hoarse, the face is puffy, with
an icteric tinge, heart tones are deaf, Ps-60 / min, blood pressure-160/90 mm Hg, the lower limbs
are swollen, when pressing on the skin of the shins, a fossa is formed with difficulty. In the blood:
HY-76 g / l, SSE-17 mm / h. What is the most likely diagnosis?
100% Hypothyroidism, myocardial dystrophy
0% Chronic glomerulonephritis, CRF
0% 1hs: small-focal myocardial infarction, HF 11 V.
0% Hypertension of the 11th century, HF of the 11th century.
0% B12 - deficiency anemia

1267.on the dispensary account of the district therapist there are convalescents after
infectious diseases, often and long-term ill, people with chronic pathology. Which of
these patients should be assigned to the 111 health group?
100% People suffering from chronic diseases
0% Often and long-term ill
0% Chronic pathology and PTH
0% Recovering from infectious diseases and people with chronic pathology
0% All categories of patients listed in the terms and conditions

1268. A 2-month-old baby born at term with a body weight of 3,500 g was on
mixed feeding. The actual body weight is 4900 g. Give an estimate of the child's body weight:
100% Age-appropriate

291

Downloaded from the site - online testing step

0% 150 g less than expected


0% Hypotrophy of the first degree
0% Grade II hypotrophy
0% Paratrophy of the first degree

1269. A 46-year-old female patient complains of pressure-like pain in the right lumbar region,
radiating to the lower abdomen, and nausea. Previously, such pain was not observed. On
the X-ray survey of the abdominal organs, no pathological shadows are detected. On
the ultrasound sonogram, in the enlarged right renal pelvis, a hyperechoic
formation with a diameter of about 1.5 cm is determined, from which the "ultrasound track" departs. What
is the most likely diagnosis?
100% Kidney stone
0% Kidney tumor is benign
https://translate.yandex.com/en/doc 280/540
22:20 ,27.6.2023 �� C : ; 5 B
0% Cyst in the kidney
0% Tuberculosis of the kidney
0% Kidney tumor is malignant

1270. A 47-year-old patient is being observed for a long time for infectious asthma
. Recently, seizures have become more frequent and are not stopped by astmopent and
berotek injections. With the appointment of what drug is it advisable to start intensive treatment?
100% Glucocorticoids
0% Bronchodilators
0% Oxygen Therapy
0% Infusion therapy
0% Cardiac Glycosides

1271. a 62-year-old woman has bronchial asthma. Complains of pain behind the sternum
of an angina pectoris nature, interruptions in the work of the heart. Objectively: t0 - 36.60 C, Ps - 78/min,
extrasystolic arrhythmia, BP - 160/95 mm Hg, PDR - 18/min. In the lungs, hard
breathing is heard with an extended exhalation, scattered dry wheezes. Which drug is contraindicated in
this situation?
100% Obzidan
0% Corinfar
0% Nitrosorbidum
0% Sustak
0% Ritmilen

1272. a 42-year-old man complains of weakness, palpitations, nosebleeds, and the appearance
of hemorrhages on the skin. The condition progressively worsens over the course of a month. Objectively: the condition
is severe, there are petechial and spotty hemorrhages on the skin of the limbs and trunk, lymph nodes are not
palpable, Ps-116 / min, liver +2 cm, spleen is not palpable. In the blood: severe
pancytopenia. What is the first disease to think about?
100% Hypoplastic anemia
0% Acute leukemia
0% Werlhof's disease

292

Downloaded from the site - online testing step

0% Hemorrhagic vasculitis
0% Acute agranulocytosis

1273. A 30-year-old patient developed an unexplained roseoglio-like rash on the skin of the abdomen on the 9th day of the disease, which
began gradually with a slow
increase in fever and intoxication.
Objectively: pale, t0-400C, Ps-80 / min, BP-100/65 mm Hg. tongue covered, abdomen swollen,
spleen palpated and liver enlarged. What disease should
be tested
100%for first?
Typhoid fever
0% Typhus fever
0% Measles
0% Scarlet fever
0% Sepsis

https://translate.yandex.com/en/doc 281/540
22:20 ,27.6.2023 �� C : ; 5 B
1274. A 63-year-old female patient underwent surgery for a large multi-nodular euthyroid goiter
. Due to technical difficulties, subtotal resection of both
thyroid lobes was performed. On the 4th day after the operation, there were convulsions of the muscles of the face and
upper extremities, abdominal pain. Positive symptoms of Khvostek and Cowardice. What
is the most likely reason for this condition of the patient?
100% Parathyroid gland failure
0% Postoperative hypothyroidism
0% Thyrotoxic crisis
0% Reverse nerve damage
0% Tracheomalacia

1275. A 50-year-old woman has been experiencing dull, intermittent acute pain in the right
hypochondrium associated with eating fatty foods, bitterness in the mouth in the morning, constipation, and flatulence for 2 years.
Objectively: excessive nutrition, t0-36.90 S, tongue overlaid at the root, abdomen moderately
swollen, painful at the point of projection of the gallbladder. What is the most appropriate
study for making a diagnosis?
100% Ultrasound examination
0% Duodenal probing
0% Cholecystography
0% Duodenoscopy
0% Liver scan

1276. a 47-year-old man suffers from end-stage lung cancer. He says that
during the last 2-3 weeks he noticed changes in his mental state: he sees
his late mother in the evening, who calls to her, feels the smell of damp earth, rotten leaves, sees
strangers carrying a coffin. At the same time, he feels fear. What are the symptoms of mental disorders in the patient?
100% Real hallucinations
0% Illusory disorders
0% Psychosensory disorders
0% Crazy ideas
0% Depressive disorders

293

Downloaded from the site - online testing step

1277. An 18-year-old patient was admitted to the gynecological department with complaints of aching pain
in the lower abdomen. Last menstruation 2 months ago. During bimanual examination
, the cervix is up to 2 cm long, cyanotic, and the external pharynx is closed. The body of the uterus is spherical in shape,
of a soft consistency, enlarged accordingly to 7-8 weeks of pregnancy. What is the most
likely diagnosis?
100% Threatening abortion
0% Abortion that started
0% Abortion is in progress
0% Fibromyoma of the uterine body
0% Incomplete abortion

1278. a pregnant woman was registered in a antenatal clinic with a gestation period of 11 weeks and
was monitored for the entire period during the normal course of pregnancy. What
document must be issued by the doctor of a pregnant woman for hospitalization in the maternity hospital?
100% Exchange card
0% Referral for hospitalization
0% Individual card for pregnant women

https://translate.yandex.com/en/doc 282/540
22:20 ,27.6.2023 �� C : ; 5 B
0% Sick leave form
0% Help from the sanitary and epidemiological station

1279. a 1.5-year-old child became acutely ill, body temperature-38.50 C, head pain, weakness. On
the fifth day of the illness, the temperature dropped. In the morning, the right leg developed muscle pain,
lack of movement and tendon reflexes; sensitivity was preserved. What is the most likely
diagnosis?
100% Polio
0% Viral encephalitis
0% Polyatropathy
0% Osteomyelitis
0% Hip joint arthritis

1280. during the survey of hygienic conditions of training in a technical university, it became necessary
to assess the visual regime of students studying from 9.00 to 15.00. What
is the most informative indicator of natural light?
100% Natural light ratio
0% World Coefficient
0% Depth of foundation of the study room
0% Room insolation time
0% Availability of compatible (Top-side) lighting

1281. A 25-year-old man who has been ill with multiple sclerosis for 4 years complains of increased
unsteadiness, weakness of the lower extremities, and urinary retention. Objectively: central tetraparesis.
Cerebellar ataxia. Pelvic organ dysfunction. What therapy is most appropriate in this
case?
100% Glucocorticoids
0% Antibiotics

294

Downloaded from the site - online testing step

0% Nootropics
0% Desensitizing agents
0% Vitamins

1282. a 70-year-old man has coronary heart disease. The mood is noticeably low,
anxious. Against the background of prolonged insomnia, there were fears, unwillingness to live, thoughts
of suicide. He sits for a long time without changing his position, does not answer immediately, quietly, in a monotonous voice.
An expression of suffering, pain, and fear. What is the leading psychopathological syndrome?
100% Depressive
0% Paranoid
0% Asthenic
0% Phobic
0% Obsessive

1283. A 65-year-old female patient fell and clogged the right side of her body. He lies
on his back, the right lower limb is rotated outward, there is no obvious shortening
, and can sit up in bed with outside help. What is the most likely diagnosis?
100% Fracture of the proximal femur
0% Bruised right hip joint
https://translate.yandex.com/en/doc 283/540
22:20 ,27.6.2023 �� C : ; 5 B

0% Pelvic and thoracic contusion


0% Pelvic bone fracture
0% Fracture of the femoral diaphysis

1284. a patient with rheumatism is characterized by diastolic tremor of the chest wall ("cat
purr"), increased I tone at the apex, diastolic murmur with pre-systolic amplification,
tone of opening of the mitral valve flaps, accent of the II tone over the pulmonary artery. What
is the patient's heart defect?
100% Stenosis of the left atrioventicular opening
0% Aortic valve insufficiency
0% Pulmonary artery stenosis
0% Mitral valve insufficiency
0% Open Ductus arteriosus

1285. a 3-year-old child who was taken to the hospital has a soporotic state, a sharp
decrease in muscle tone, suppression of tendon and periosteal reflexes;
miosis and a decrease in pupil response to light are detected. Corneal reflexes are preserved. Pulse is fast
and weak. AT-80/50 mmHg. parents suspect that the child swallowed the tablet medicine.
Poisoning with what blethered drugs corresponds to such a clinical picture?
100% Tranquilizers
0% Atropine-like drugs
0% Antihypertensive drugs
0% Barbiturates
0% ^ - adrenomimetics

295

Downloaded from the site - online testing step

1286. A 32-year-old patient has been suffering from systemic scleroderma for 14 years. She was repeatedly
treated in hospitals. Complains of periodic dull pain in the heart, palpitations,
shortness of breath, headache, swelling of the eyelids, weight loss, pain and deformity of the joints of the extremities.
What organ damage worsens the prognosis of the disease?
100% Kidneys
0% Heart
0% Lungs
0% Gastrointestinal tract
0% Skin and joints

1287. a 2-month-old child born with a weight of 5100 g has jaundice, hoarse
crying, an umbilical hernia, and a lag in physical development. Liver + 2 cm, spleen is not
enlarged. In the anamnesis, there is a delay in the drop-off of the umbilical cord residue. In the blood: Hb-120 g / l, Er.- 4,5
* 1012/ l, WSE-3 mm / hour. Total serum bilirubin-28 mmol/l, indirect - 20
mmol/l, direct-8 mmol / l. what disease should you think about first?
100% Congenital hypothyroidism
0% Congenital hepatitis
0% Hemolytic anemia
0% Conjugation jaundice
0% Cytomegalovirus infection

https://translate.yandex.com/en/doc 284/540
22:20 ,27.6.2023 �� C : ; 5 B
1288. A 13-year-old girl complains of an increase in body temperature to febrile figures
within a month, joint pain, and a periodic rash on her pelvis. During the examination
, a persistent increase in SSE and LE cells was found in the blood. What is the most likely diagnosis?
100% Systemic lupus erythematosus
0% Juvenile rheumatoid arthritis
0% Systemic scleroderma
0% Acute lymphoblastic leukemia
0% Rheumatism

1289.A 50-year - old woman suffering from chronic pyelonephritis was


prescribed a combination of antibacterial agents-gentamicin (80 mg 3 times a day) and
Biseptol (960 mg 2 times a day). What are the consequences of prescribing such
a combination of antibiotics?
100% Acute renal failure
0% Glomerulosclerosis
0% Chronic renal failure
0% The combination of antibiotics is optimal and quite safe
0% Acute adrenal insufficiency

1290. A 60-year-old patient had a short-term decrease in strength in the left


extremities for a month. Later in the morning after sleep, persistent weakness in the limbs appeared. Objectively:
sane, blood pressure-140/90 mm Hg, central paresis of VII and XII pairs of cranial nerves on the left,
central hemiparesis and hemigiperesthesia on the same side. What are the drugs of choice for
differential treatment of this patient?

296

Downloaded from the site - online testing step

100% Anticoagulants
0% Hemostatics
0% Antihypertensive drugs
0% Diuretics
0% Corticosteroids

1291. the child is 1 year old. After the introduction of complementary foods, during the last months, loss
of appetite, diarrhea with the release of large amounts of feces, and sometimes vomiting are noted. Objectively:
body temperature is normal, body weight 7 kg., pronounced pallor of the skin, swelling on the legs, the abdomen is enlarged in
volume. Coprogram contains a lot of fatty acids and soaps. The diagnosis was established: celiac disease. A
gluten-free diet is prescribed. What is excluded from the diet with this diet?
100% Cereals-wheat, oats
0% Milk and dairy products
0% Fruit
0% Animal protein
0% Easily digestible carbohydrates

1292. during a fluorographic examination of a 45-year-old man, for the first


time, small foci of low intensity with indistinct contours were found on the top of the right lung.
The patient's state of health is not disturbed. Smokes for many years. Objectively:
there is a percutaneous-pulmonary sound above the lungs, vesicular respiration, and wheezing is not audible. The blood test is
unchanged. What
is the100%
most likely diagnosis?
Focal tuberculosis of the lungs

https://translate.yandex.com/en/doc 285/540
22:20 ,27.6.2023 �� C : ; 5 B
0% Peripheral lung cancer
0% Eosinophilic pneumonia
0% Bronchopneumonia
0% Disseminated pulmonary tuberculosis

1293. an 8 - year-old boy complains of a persistent cough with greenish


sputum, shortness of breath during physical exertion. For the first time, at the age of 1 year and 8 months, he fell ill
with bilateral pneumonia, which had a prolonged course. In the future, relapses
of the disease were observed 5-6 times a year, during periods of remission, a wet cough was persistently retained. The results of which
examination will be most significant for establishing a definitive diagnosis?
100% Bronchography
0% Chest radiography
0% Bakposev mokotinnya
0% Bronchoscopy
0% Spirography

1294. after manual reposition and application of a plaster splint, a patient with fractures
of the forearm bones developed swelling of the hand and fingers, pain, and impaired sensitivity. What should be
the doctor's tactics?
100% Cut the bandage that is used to fix the longet
0% Prescribe analgesics and diuretics
0% Remove the plaster cast

297

Downloaded from the site - online testing step

0% This is a natural phenomenon, the edema itself will decrease in a day


0% Repeat reposition

1295. a 35-year-old patient suffering from chronic glomerulonephritis and


who has been on hemodialysis for the last 3 years has experienced heart failure, hypotension, increasing
weakness, and shortness of breath. On the ECG: bradycardia, atrioventricular block of the first century, high pointed
teeth of the pelvis - a gross violation of drinking and dietary regimes. What is the most likely
reason for these changes?
100% Hyperkalemia
0% Hyperhydration
0% Hypokalemia
0% Hypernatremia
0% Hypocalcemia

1296. A 60-year-old female patient complains of pain in the interphalangeal joints of the hands, which
increases during work. Objectively: the distal and proximal joints of the fingers II-IV
are defigured, with Goeberden and Bouchard nodes, painful, with limited mobility. X-ray
of joints: narrowed joint spaces, marginal osteophytes, subchondral sclerosis. What
is the most likely diagnosis?
100% Deforming osteoarthritis, nodular form
0% Reiter's disease
0% Ankylosing spondylitis
0% Rheumatic arthritis
0% Psoriatic arthritis

https://translate.yandex.com/en/doc 286/540
22:20 ,27.6.2023 �� C : ; 5 B

1297. A 35-year-old patient was admitted to the hospital with complaints of pain in the left
sternoclavicular and knee joints, as well as in the lower back. He became acutely ill, with the temperature rising to
380C. objectively: the left thoracoclavicular and knee joints are swollen and painful during
palpation. In the blood: lake.- 9,5 • 109/ l, SSE-40 mm / h, CRP-1.5 mm, fibrinogen-4.8 g / l, uric
acid-0.28 mmol/l. in scraping from the urethra-chlamydia. What is the most likely diagnosis?
100% Reiter's Syndrome
0% Rheumatic arthritis
0% Gout
0% Ankylosing spondylitis
0% Rheumatoid arthritis

1298. A 32-year-old patient lives in an area endemic with echinococcosis. For the last 6 months, he
has been worried about pain in the right hypochondrium, fever. Suspected echinococcal
liver damage. What kind of research is most informative in this case?
100% Ultrasound examination
0% Overview radiography of the abdominal cavity
0% Biochemical laboratory research
0% Angiography
0% Liver scan
1299. A 5-year-old patient was admitted to the clinic with complaints of edema, oliguria, and hypertension.

298

Downloaded from the site - online testing step

The child's general condition is serious. After a clinical and laboratory examination, the following diagnosis was established:
acute glomerulonephritis. Which of the clinical and laboratory parameters in this disease is
an absolute indication for inclusion in the therapy of glucocorticoid hormones?
100% Daily proteinuria of more than 3 g
0% Daily proteinuria up to 3 g
0% Increased creatinine
0% Macrohematuria
0% Anemia

1300. In a 30-year-old child giving birth for the first time, intense attempts began with an interval of 1-2
minutes, lasting 50 seconds. Eruption of the fetal head occurs. The crotch, which
is 4 cm high, turned pale. What should be done in this situation?
100% Episiotomy
0% Perineal protection
0% Perineotomy
0% Vacuum extraction of the fetus
0% Stick to wait-and-see tactics

1301. among the causes of death of the population at the general practitioner's site, in the last
year, the first place is occupied by cardiovascular diseases (60%), the second - neoplasms (18%),
then - injuries (8.3%) and others. Which diagrams most informatively illustrate the structure
of the phenomena under study?
100% Sector information
0% Cartogram
0% Linear
0% Radial
0% Stolbikovaya street

https://translate.yandex.com/en/doc 287/540
22:20 ,27.6.2023 �� C : ; 5 B

1302. the patient has complaints of nocturia, constant aching pain in the perineum and suprapubic region,
flabby urine stream, rapid, difficult and painful urination. He has been ill for
several months, when urination gradually began to become difficult, and pain in
the perineum appeared. During rectal examination, the prostate is enlarged (more
due to the right lobe), dense, asymmetric, the central sulcus is smoothed, the right lobe
is stony, non-painful, bumpy. What disease should I think about?
100% Prostate cancer
0% Prostate sclerosis
0% Urolithiasis, a stone of the right lobe of the prostate
0% Tuberculosis of the prostate
0% Chronic congestive prostatitis

1303. 8 people were hospitalized in the infectious diseases department with complaints of head pain, soreness in
the calf muscles, swelling of the face and eyelids, chest pain during breathing,
and fever up to 40 ° C. all the patients were hunting and eating wild boar meat 10 days ago.
What is the most likely diagnosis?
100% Trichinosis

299

Downloaded from the site - online testing step

0% Brucellosis
0% Typhoid fever
0% Leptospirosis
0% Pseudotuberculosis

1304. a 35-year-old patient, after severe nervous tension,


developed areas of redness and edema on the back of her hands, followed by the formation of small
inflammatory nodules, blisters, and then erosions with a significant release of serous fluid. The process
is accompanied by severe itching. What is the most likely diagnosis?
100% Real eczema
0% Allergic dermatitis
0% Microbial eczema
0% Simple contact dermatitis
0% Toxicoderma

1305. a 37-year-old patient complains of damage to the nail plates for 2 years.
Objectively: nail plates of the I, II, III fingers of both feet and II, III fingers of the hands are thickened,
dull, with signs of subungual hyperkeratosis; the nails are broken off along the free edge. On the soles
and right palm-hyperkeratosis, in the folds of the skin - flaky peeling. !nshi skin areas
without rash. What additional research methods should be used to clarify the diagnosis?
100% Microscopic and bacteriological examination of scales
0% General blood test
0% Serological blood testing
0% Blood sugar test
0% Skin biopsy

1306. an employee of a pig farm, on the background of full health, developed a sharp chill, an increase
in body temperature up to 39.9 aboutC, intense headache, nausea. The next day, there
was pain in the muscles of the lower extremities, nosebleeds. Objectively: on the 3rd day of the disease, the condition is severe,

https://translate.yandex.com/en/doc 288/540
22:20 ,27.6.2023 �� C : ; 5 B
the face is hyperemic, scleritis, subicteric sclera. Liver + 3 cm. Daily diuresis of 700 ml.
What is the most likely diagnosis?
100% Leptospirosis
0% Viral hepatitis
0% !yersiniosis
0% Flu
0% Hemorrhagic fever with renal syndrome

1307. after lifting a heavy object, a 42-year-old patient noted sharp pain in the right
side of the chest, shortness of breath sharply increased. The patient's condition is severe: cyanosis of the lips and
mucous membranes, PDR-28/min, Ps-122 / min. Over the right half of the chest percussion-tympanitis,
auscultation-sharply weakened breathing; accent of the second tone over the pulmonary artery. AT - 80/60 mm
Hg. what is the main emergency measure at the prehospital stage?
100% Aspiration of air from the pleural cavity
0% Introduction of epinephrine
0% Introduction of eufillin

300

Downloaded from the site - online testing step

0% Call a cardiology team


0% !ngalia of oxygen

1308. a 46-year-old patient complains of sudden attacks of palpitations, which


are accompanied by pulsation in the neck and head, fear, and nausea. Attacks
last 15-20 minutes, pass after holding your breath with straining. What
disorder of cardiac activity occurs in the patient?
100% Supraventricular paroxysmal tachycardia attack
0% An attack of ventricular paroxysmal tachycardia
0% Atrial flutter attack
0% Atrial fibrillation attack
0% An attack of extrasystolic arrhythmia

1309. in a newborn after a pathological birth from the first day of life, there are no active
movements in the right hand. The state is broken. The Moreau reflex on the right is not triggered. Sharply reduced
tendon-periosteal reflexes on the affected arm. What is the most likely diagnosis?
100% Traumatic plexitis, total type
0% Traumatic plexitis, distal type
0% Osteomyelitis of the right humerus
0% Traumatic fracture of the right humerus
0% Intracranial birth trauma

1310. A 62-year-old woman complained of vulvar pruritus at the gynecologist's office.


Suffers from chronic pancreatitis for 8 years. High nutrition, body weight 102 kg, Height
158 cm. Palpable inguinal lymph nodes up to 0.8 cm. On the skin of the perineum there are traces of combs.
Blood sugar is 7.8 mmol / l. what is the most likely diagnosis?
100% Diabetes mellitus
0% Alimentary obesity
0% Vulvitis
0% Lymphogranulomatosis
0% Allergic dermatitis
https://translate.yandex.com/en/doc 289/540
22:20 ,27.6.2023 �� C : ; 5 B

1311. a 74-year-old patient complains of pain and bloating, nausea. He suffers


from ischemic heart disease, post-infarction and diffuse cardiosclerosis. Objectively: the condition is severe, the abdomen
is swollen, the abdominal wall is poorly involved in the act of breathing. During laparoscopy:
there is a small amount of cloudy effusion in the abdominal cavity, one of the loops of the small intestine is dark bluish
in color. What is the most likely diagnosis?
100% Mesenteric vascular thrombosis
0% Inversion of the bowel
0% Acute intestinal obstruction
0% Ischemic abdominal syndrome
0% Erysipelas

1312. in a 10-year-old child who is in the oligoanuric stage of acute renal failure

301

Downloaded from the site - online testing step

insufficiency, there were tingling sensations in the mucous membrane of the mouth,tongue,
numbness of the extremities, decreased reflexes, respiratory distress, arrhythmia. What causes these
symptoms?
100% Hyperkalemia
0% Hyponatremia
0% Hyperazotemia
0% Acidosis
0% Alkalosis

1313. a 30-year-old woman has repeated labor lasting 14 hours. Fetal heartbeat
is muffled, arrhythmic, 100 / min. Vaginal examination: the opening of the cervix is complete,
the fetal head is in the plane of the exit from the small pelvis. Sagittal suture in straight size, small crown near
the womb. What are the further tactics of childbirth?
100% Using the original obstetric forceps
0% Oxytocin stimulation of labor
0% Caesarean section
0% Skin-main forceps by Ivanov
0% Use of abdominal obstetric forceps

1314. A 75-year-old man with a diagnosis of CHD: difuse cardiosclerosis, atrial fibrillation, stage 2B HF
, chronic pyelonephritis, was prescribed digoxin. For the first 6 days, the digoxin dose was
0.25 mg twice daily, which reduced shortness of breath, edema, and cyanosis. However, on day 7, the
patient developed nausea and bradycardia. What is the most likely cause
of digoxin intoxication?
100% Impaired elimination of the drug by the kidneys
0% The saturating dose is too long
0% Excess of the daily saturating dose
0% Violation of digoxin metabolism in the liver
0% There is no unitiol in the treatment package

1315. a patient who has been exposed to benzene for 6 years


has mild leukopenia, moderate reticulocytosis, bleeding gums,dizziness,

https://translate.yandex.com/en/doc 290/540
22:20 ,27.6.2023 �� C : ; 5 B
and asthenovegetative syndrome in her blood. What degree of severity of chronic benzene intoxication
100% Easy
corresponds to this picture?
0% Average
0% Heavy
0% The disease is not related to working conditions
0% -

1316. A 45-year-old man complains of intense epigastric pain 1.5-2 hours after
eating. He has been suffering from peptic ulcer disease for 11 years. Ob'ektivno: t0-36,50 S,
PDR-16/xv, Ps-70/xv, AT-120/80 mm Hg Palpation: local soreness in the right
epigastric region. What indicators of intragastric pH-metry in the area of the stomach body
will be most characteristic of this patient's disease?

302

Downloaded from the site - online testing step

100% pH = 1.0-2.0
0% pH = 3.0-4.0
0% pH = 4.0 - 5.0
0% pH = 5.0-6.0
0% pH = 6.0-7.0

1317. a 3-month-old child became acutely ill with fever up to 37.80 C, coughing.
On day 3, the cough worsened, and shortness of breath joined in. Percussion: tympanic
sound above the lungs, auscultation-a large amount of small bubbles on both sides-the subsiding of wet and whistling
wheezes during exhalation. What is the most likely diagnosis?
100% ARVI, bronchiolitis
0% Acute respiratory viral infections, bronchopneumonia
0% Acute respiratory viral infections, bronchitis
0% ARVI, bronchitis with an asthmatic component
0% ARVI, focal pneumonia

1318. A 37-year-old porcelain factory worker complains of a cough,


shortness of breath, and chest pain after 10 years of service. What occupational diseases are the most
common complaints for?
100% Silicosis
0% Bronchiectasis
0% Chronic dust bronchitis
0% Occupational bronchial asthma
0% Chronic pulmonary heart disease

1319. A 46-year-old patient has been suffering from chronic pyelonephritis for a long time. There were edemas,
increased body weight. Blood pressure stabilized at -160 / 110 mmHg. What is the predominant
protein fraction to be expected in the urine electrophoregram?
100% Albumin
0% od-and A2-globulin
0% b-globulin
0% y-globulin
0% Fibrinogen

https://translate.yandex.com/en/doc 291/540
22:20 ,27.6.2023 �� C : ; 5 B
1320.during a medical examination, the man was found to have a "geographical language". What vitamins are lacking in the diet?This
micro-symptom indicates a lack
100% B
of vitamins in the diet.
0% A
0% C
0% D
0% PP

1321. A 15-year - old patient was diagnosed with AT-170/85 mm Hg during the medical examination
. Objectively: the muscles of the lower extremities are poorly developed, pulsation on the arteries

303

Downloaded from the site - online testing step

the stage is weakened. The borders of the heart are extended to the left, with auscultation-accent of the second tone above
the aorta, systolic murmur. In the blood-no changes. In urine: specific gravity-1020, lake.- 2-4 v p/ s, ep. - 0-1
v p/ s, uric acid crystals. What is the most likely diagnosis?
100% Aortic coarctation
0% Vasorenal hypertension
0% Essential hypertension
0% Vegetative vascular dystonia
0% Renoparenchymal hypertension

1322. A 60-year-old man developed dysphagia, which progresses rapidly over several
weeks. Weight loss and anemia were noted. What is the most likely diagnosis?
100% Esophageal cancer
0% Foreign body of the esophagus
0% Achalasia of the cardia
0% Esophageal diverticulum
0% Hiatal hernia

1323. the child has a sudden cessation of breathing,blueness of the skin,


disappearance of the pulse on the main vessels and narrowing of the pupils. What are the best priority
measures?
100% Artificial ventilation, closed heart massage
0% Intravenous administration of eufillin
0% Gastric lavage
0% Oxygen Therapy
0% Intracardiac administration of epinephrine

1324. a child was born with a weight of 3250 g and a body length of 52 cm. At the age of 1.5 months, the actual
weight is sufficient (4350 g), psychophysical development corresponds to the age. The child is on
natural feeding, regurgitation is periodically noted. What causes regurgitation in a
child?
100% Aerophagy
0% Pylorostenosis
0% Pylorospasm
0% Acute gastroenteritis
0% Esophageal atresia

https://translate.yandex.com/en/doc 292/540
22:20 ,27.6.2023 �� C : ; 5 B
1325. in a 60-year-old patient, on the 4th day after the injection, pain and
tissue compaction appeared in the left buttock. Objectively: in the upper-outer quadrant of the left buttock, the skin
is red, hot to the touch, during palpation, an infiltrate of 6x6 cm is detected, painful, in the center
of softening.
100% Body temperature-37.90 C. What should be done to determine the presence of an abscess?
A puncture
0% A biopsy
0% Ultrasound examination
0% Radiography

304

Downloaded from the site - online testing step

0% General blood test

1326. A 34-year-old patient suffered a TBI 3 years ago. During his studies at the university, he occasionally used
hashish. During the last 2 months, there have been "embedding1^ "taking away"thoughts,their transfer,
delusions of external influence that relate to his thoughts. Neurological
microsymptomatics are noted. Somatic condition without pathology. What mental illness
criteria does this condition meet?
100% Schizophrenia
0% Intoxication psychosis
0% Post-traumatic psychosis
0% Manic-depressive psychosis
0% Epileptic personality

1327. a 39-year-old patient complains of shortness of breath during physical exertion, swelling of the
lower legs, palpitation, and heart failure. Objectively: 4cc-150 / min, atrial fibrillation. The borders
of the heart are extended in both directions. Heart tones are muted. The liver is 6 cm below the edge of the costal
arch. Echocardiography-dilation of the heart chambers (CDR of the left ventricle 6,8 cm) EF-29%, valvular apparatus
without changes. What is the most likely diagnosis?
100% Dilated cardiomyopathy
0% Exudative pericarditis
0% Restrictive cardiomyopathy
0% Hypertrophic cardiomyopathy
0% Thyrotoxic cardiomyopathy

1328. a 30-year-old woman gave birth to a child with an anemic-jaundice


form of hemolytic disease during the second delivery. Female blood type A (II)Rh -, blood type in the newborn
B (III) Rh+, in the father of the newborn also B (III) Rh+. What is the most likely cause
of an immune conflict?
100% Rhesus conflict
0% Conflict by antigen a
0% Conflict by antigen in
0% AB antigen conflict
0% Conflict over AB0

1329. during the major renovation of the hospital, it was planned to update the color design
of the hospital premises, because the latter is of great psychological and aesthetic importance; at
the same time, the walls of the wards for patients decided to paint taking into account:
100% Window orientations
0% Hospital Profile

https://translate.yandex.com/en/doc 293/540
22:20 ,27.6.2023 �� C : ; 5 B
0% Diseases of patients who were admitted to these wards
0% Wall reflection coefficient
0% Creating a sense of comfort

1330. a 10-year-old girl has an average body length and chest circumference

305

Downloaded from the site - online testing step

above average values, the body weight index is increased due to fat deposition.
Functional signs of physical development are below average. The physical development of this child
can be comprehensively assessed as:
100% Disharmonious
0% Average
0% Below average
0% Harmonic
0% Sharply disharmonious

1331. the atmospheric air of an industrial center receives emissions from metallurgical
enterprises: oxides of sulfur, nitrogen, metals, and carbon that negatively affect the health
of the population. The effect of these harmful factors is characterized as:
100% Combined
0% Comprehensive
0% Binder
0% Adjacent area
0% Mixed version

1332. on the survey X-ray of the lungs,an intense uniform darkening is determined,
corresponding to the position of the lower lobe of the left lung. On the side radiograph
, it is particularly clear that the entire lobe is damaged, and it is reduced, and its anterior contour is sharp and
straightened. The heart is slightly shifted to the left and down. What disease causes
this X-ray picture?
100% Atelectasis of the lower lobe of the left lung
0% Left-sided exudative pleurisy
0% Croup pneumonia
0% Peripheral cancer of the left lung
0% Echinococcus of the left lung

1333. an X-ray of the chest organs on the right side,at the level of the 4th rib,
shows an intense non-homogenous shadow of large size with indistinct contours. In the center
of the specified shadow, the horizontal level and the illumination of the lung tissue above it are determined.
What disease does this X-ray picture correspond to?
100% Right lung abscess
0% Peripheral cancer
0% Tuberculosis of the right lung
0% Bullous disease
0% Right-sided pneumothorax

1334. a 29-year-old patient complains of purulent discharge from the urethra, pain during urination.
These symptoms appeared 5 days after accidental sexual intercourse. A Gram-stained smear of

https://translate.yandex.com/en/doc 294/540
22:20 ,27.6.2023 �� C : ; 5 B
urethral secretions revealed paired red-violet cocci. What
is the most likely diagnosis?
100% Gonorrheal urethritis
0% Trichomonas urethritis

306

Downloaded from the site - online testing step

0% Yeast urethritis
0% Bacterial urethritis
0% Chlamydial urethritis

1335. A 28-year-old patient, who 2 days ago injured the distal phalanx of the right index
finger with a needle, complains of sharp pain of a pulsating nature and swelling in the area of this
phalanx, a pronounced violation of finger mobility. Because of the pain, I couldn't sleep last night.
Objectively: the index finger is slightly bent, its distal phalanx is hyperemic, significantly
increased in volume. The probe clearly defines the point of maximum pain.
Body temperature 36,90 C. What should be the local treatment?
100% Opening and drainage of the purulent process of the finger
0% Opening and drainage of the abscess after local infiltrative anesthesia
0% Pricking the affected area with antibiotics with novocaine
0% Application of semi-alcohol compresses
0% Resection of the distal phalanx of the finger

1336. a gas station worker with 15 years of work experience who has contact with leaded
gasoline has memory loss, bradycardia, hair sensation in the mouth, and skin
paresthesias. Intoxication with what substance should be suspected in this case?
100% Tetraethyl lead
0% Lead Chloride
0% Organophosphate compounds
0% Benzene
0% Nitrobenzene

1337. an ambulance delivered a patient after a fall from a height, with clinical
signs of multiple fractures of both lower limbs. Objectively: the patient's condition is severe,
consciousness is preserved, but mentally retarded, the skin is pale gray,
cold sweat is on his forehead. Breathing is shallow, up to 30 / min, blood pressure-80/60 mm Hg, Ps-120 / min, weak
filling. What complication does the victim have?
100% Traumatic shock 2 st
0% Traumatic shock 1 st
0% Traumatic shock 3 st
0% Traumatic shock 4 st
0% Fainting

1338. the patient experienced unbearable low back pain after lifting the load. The local
therapist diagnosed acute lumbosacral sciatica. Which of the following
prescriptions is contraindicated for the patient?
100% Warming procedures
0% Dehydrating agents
0% Analgesics
0% B vitamins

https://translate.yandex.com/en/doc 295/540
22:20 ,27.6.2023 �� C : ; 5 B
0% Intravenous eufillin
1339. a 30-year-old man developed a large developing rash on the skin of his torso

307

Downloaded from the site - online testing step

It consists of small paired elements that are scattered over the skin irregularly and mainly focusally and
are accompanied by itching. The rash appeared a few days after he visited
the sports and wellness center and sauna. What is the most likely diagnosis?
100% Scabies
0% Eczema
0% Contact dermatitis
0% Allergic dermatitis
0% Neurodermatitis

1340. a 38-year-old patient underwent a glucose tolerance test: fasting


capillary blood glucose - 5.9 mmol/l, after 2 hours-8.9 mmol/ l. evaluate the test results:
100% Impaired glucose tolerance
0% Normal Test
0% Violation of fasting blood glucose
0% Questionable test
0% Overt diabetes mellitus

1341. Pervovagitnaya 23 years with a period of 37-38 weeks. My condition is serious.


I had an eclampsia attack at home. Objectively: AT-180/100 mm Hg, Ps-98 / min, generalized edema, consciousness
is clouded. Determine the tactics of managing a pregnant woman:
100% Urgent delivery by caesarean section on the background of intensive care
0% Prolongation of pregnancy during intensive care
0% Intensive care for 2-3 days followed by delivery
0% Applying obstetric forceps
0% Early amniotomy

1342. A 33-year-old patient with stopped recurrent ulcerative


bleeding was admitted to the hospital. During the examination, emaciated, pale. In the blood: HB-77 g / l, ni-0.25. due to
the presence of anemia, a single-group A(II)transfusion was attempted twiceRh+ of blood. Both times
the transfusion was stopped due to the development of an anaphylactic reaction. Transfusion of what
transfusion medium is desirable in this case?
100% Washed red blood cells
0% Fresh Blood Citrate
0% Red blood cell mass (native)
0% Red blood cell curl
0% Red blood cell mass, which is poor in white blood cells and platelets

1343. a 19-year-old boy who received a closed abdominal injury was admitted to a surgical hospital
. Numerous ruptures of the spleen and small intestine were found during the operation. Blood pressure gradually decreases.
There was a need for blood transfusion. Who can
determine the victim's blood type and Rh status?
100% Doctor of any specialty
0% Laboratory assistant
0% Surgeon

https://translate.yandex.com/en/doc 296/540
22:20 ,27.6.2023 �� C : ; 5 B
308

Downloaded from the site - online testing step

0% Traumatologist
0% Doctor-anesthesiologist

1344. A 14-year-old girl received an unsatisfactory grade and a reprimand


from the teacher during a math lesson, which caused her to cry for a long time. After the lesson, she suddenly lost consciousness and fell
down. Objectively:
skin of pale pink color, Ps-100 / min, satisfactory properties, blood pressure-110/70 mm Hg.
the eyelids
100% are tightly closed.
Hysterical No visible damage was noted. What is the most likely diagnosis?
collapse
0% Epilepsy
0% Vagotonic coupling
0% Extended Q-T interval syndrome
0% Sympathicotonic collapse

1345. the patient complains of severe headache in the frontal region, Purulent runny nose,
nasal congestion, decreased sense of smell. Ill for a week after suffering acute respiratory viral infection. Objectively:
the general condition is satisfactory, the skin is clean, t0-37.50 s. palpation of the exit points of the first branch
of the trigeminal nerve is painful, breathing through the nose is moderately difficult, after instilling 0.1% naphthyzine solution into the
nose
, a lot of mucus and pus blows out, after which breathing improves. What
is the100%
most likely diagnosis?
Acute purulent sinusitis
0% Acute chronic rhinitis
0% Chronic purulent sinusitis in the acute stage
0% Acute respiratory viral infection complicated by trigeminal neuralgia
0% Acute viral infection

1346. a 32-year-old woman complains of pain in the lower abdomen, fever up


to 38.00 C, purulent discharge from the genital tract. Got sick 5 days ago after an accidental sexual
encounter. Objectively: hyperemia is observed in the area of the excretory ducts of the bartholinium glands.
Bimanually: the uterus is not enlarged, mobile, sensitive during palpation. Appendages on both sides
are enlarged, pasty, sharply painful. The discharge is pus-like, greenish in color, with an unpleasant
smell. What is the most likely diagnosis?
100% Acute gonorrheal adnexitis
0% Parametrite
0% Metroendometritis
0% Colpitis
0% Pelvioperitonitis

1347. in older people, the incidence of tumors increases. One of the main reasons for this is:
100% Decreased activity of cellular immunity
0% Reduced antibody formation rate
0% Growth of mitotic disorders
0% Increased activity of cellular immunity
0% Increasing the intensity of antibody formation

309

https://translate.yandex.com/en/doc 297/540
22:20 ,27.6.2023 �� C : ; 5 B

Downloaded from the site - online testing step

1348. A patient applied to the doctor of a rural outpatient clinic twice during a calendar year (in March and
in November). In both cases, he was diagnosed with the same
acute respiratory viral infection. How do I fill out statistical coupons for registering final (updated) diagnoses in order
to record these cases of diseases?
100% For each case separately marked ( + )
0% For each case separately marked ( -)
0% In the first case - marked (+), In the second-marked ( -)
0% In the first case, it is marked ( + ). In the second case, it is not filled in
0% Not filled in for any of the cases

1349. a 7-year-old boy was taken to the bakery by ambulance. Complaints of unpleasant
sensations in the heart, Pain in the epigastrium, dizziness, vomiting. Objectively: pronounced
validity of the skin, shortness of breath, pulsation of the jugular veins. The boundaries of the heart are within the normal range.
Heart sounds are clear, heart rate-170/min, pulse of small filling. AT-90/50 mm Hg on the ECG: paroxysm
of ventricular tachycardia. To relieve paroxysm, the pediatrician prescribed:
100% Lidocaine
0% Morphine
0% Enalapril
0% Nifedipine
0% Strophanthin

1350. the child is 2 months old. Acute-combustible foci with


clear borders in the form of spots that slightly protrude above the surrounding areas due to skin edema were found in the inguinal-femoral
folds
. The rash appeared within a week. Vesiculation and wetting are not
observed.
100% WhatDiaper
is the most likely diagnosis?
dermatitis
0% Children's eczema
0% Mycosis of smooth skin
0% Psoriasis
0% Complicated course of scabies

1351. a 23-year-old pregnant woman has increased body temperature, bitterness in the mouth, pain in
the right hypochondrium, and positive symptoms of Murphy and Ker. Which diagnostic method is
most appropriate in this case?
100% Ultrasound scanning
0% Thermography
0% Oral cholecystography
0% Intravenous cholecystography
0% Retrograde pancreatocholangiography

1352. the child is ill on the 5th day. Worried about increased to 39-40oC body temperature,
paroxysmal, frequent, deep unproductive cough, photophobia, nasal congestion.
Objectively: puffiness of the face; loose oral mucosa, hemorrhagic enanthema on the soft
palate, remnants of Filatov-Koplik spots. What is the most likely diagnosis?
100% Measles

310

https://translate.yandex.com/en/doc 298/540
22:20 ,27.6.2023 �� C : ; 5 B
Downloaded from the site - online testing step

0% Scarlet fever
0% Rubella
0% Adenovirus infection
0% Flu

1353. A 38-year-old patient developed pain in the lumbar spine after lifting the load
, radiating along the back of the left leg. The pain increases when changing the position of the body and in
an upright position. During the examination, positive symptoms of tension were revealed. What
is the most likely diagnosis?
100% Pathology of intervertebral discs
0% Spinal cord tumor
0% Arachnomyelitis
0% Polyneuritis
0% Myelopathy

1354. An 18-year-old student complains for 7 days of weakness, body temperature up


to 37.8 aboutC, mucosal discharge from the nose, sore throat during swallowing, a feeling of"sand" in the eyes.
Objectively: an increase in the posterior cervical, angulomandibular lymph nodes, edema and injection
of conjunctival vessels, hyperemia of the oropharyngeal mucosa, hypertrophy of the tonsils, no plaque.
What is the most likely diagnosis?
100% Adenovirus infection
0% Flu
0% Infectious mononucleosis
0% Rhinovirus infection
0% Parainfluenza

1355. an aircraft factory processes materials using optical


quantum generators. It is established that the installation emits in the visible spectrum and the levels
of laser radiation in the workplace exceed the maximum permissible level.
Please indicate which organs will be affected first?
100% Oko
0% Skin
0% Liver
0% The spleen
0% Kidneys

1356. at a machine-building plant, castings are cleaned using grinding


machines, which are a source of local vibration. What are the most effective preventive measures
that should be taken to prevent the harmful effects of vibration on the body of employees?
100% Using vibration-reducing gloves
0% Conducting preliminary and periodic medical examinations
0% Hand massage
0% Taking warm hand baths
0% Carrying out sanitary and pro-vet work among workers

311

Downloaded from the site - online testing step

1357. A 25-year-old patient complains of an increase in body temperature up to 380C, pain in the lower

https://translate.yandex.com/en/doc 299/540
22:20 ,27.6.2023 �� C : ; 5 B
abdomen and vaginal discharge. Three days ago, an artificial abortion was performed at 10 weeks
of pregnancy. PV: the cervix is clear, the uterus is slightly enlarged, painful. Uterine appendages
are not detected. The arches are deep and painless. Vaginal discharge is purulent-bloody. What
is the most likely diagnosis?
100% Post-abortion metroendometritis
0% Hematometer
0% Pelvioperitonitis
0% Uterine perforation after abortion
0% Parametrite

1358. A 67 - year-old patient complains of shortness of breath, chest pain, and general weakness. He's been ill for 5
months. Objectively: t0 - 37, 30C, Ps-96/min. Above the right lung, the vocal tremor is not
detected, the percussion sound is dull, and breathing is not listened to. In sputum-impurities
of blood, diffusely mixed with mucus. What is the most likely diagnosis?
100% Lung cancer
0% Bolshegnishchevaya pneumonia
0% Bronchiectasis
0% Focal tuberculosis of the lungs
0% Exudative pleurisy

1359. for the population living in a radiation-contaminated area,


it is recommended to include pectins in the diet in order to remove radionuclides from the body. Which of
the following foods are the main source of pectins?
100% Vegetables and fruits
0% Bread
0% Milk
0% Meat
0% Pasta

1360. the cold period of the year was characterized by low air temperature, constant
cloud cover, and snowfall. In this regard, the stay of preschool children in the open
air was limited. During the medical examination, children complained of drowsiness, weakness,
poor appetite, and low activity in physical education classes. A
laboratory blood test revealed a violation of phosphorus-calcium metabolism.
The reason for this condition of children can be:
100% Ultraviolet insufficiency
0% Heating microclimate in the room
0% Cooling microclimate in the room
0% Uncomfortable indoor climate
0% Excessive physical activity

1361. A 65-year-old patient suffered an acute myocardial infarction a week ago. The general condition
has worsened, suffocation at rest, pronounced weakness, the presence of edema and ascites are concerned. The borders
of the heart are expanded, there is a paradoxical pericardial pulsation laterally from the heart.

312

Downloaded from the site - online testing step

apical push on the left. What is the most likely diagnosis?


100% Acute heart aneurysm
0% Chronic heart aneurysm

https://translate.yandex.com/en/doc 300/540
22:20 ,27.6.2023 �� C : ; 5 B
0% Acute pericarditis
0% Cardiosclerotic heart aneurysm
0% Rheumatic heart aneurysm

1362. A 22-year-old patient complains of aching pain in the right swollen area for a week,
morning sickness, and changes in taste. Delayed menstruation for 3 weeks. Objectively: AT-110/70 mm Hg,
Ps-78 / min, t0-37.00 s. Bimanually: the uterus is slightly enlarged, soft, mobile, painless. Palpation
of the appendages: on the right side there is a painful formation of 3x4 cm, tightly elastic consistency, moderately
mobile. What is the most likely diagnosis?
100% Progressive tubal pregnancy
0% Aborted tubal pregnancy
0% Right ovarian cyst
0% Uterine pregnancy
0% Acute appendicitis

1363. a 15-year-old patient lags behind in physical development, notes periodic yellowing of the skin.
Objectively: Selesina 16x12x10 cm, cholecystolithiasis, skin ulcer on the lower third of the left
shin. In the blood: Er.- 3,0 • 1012/ l, Hb-90 g/l, CP-1.0, microspherocytosis, reticulocytosis.
Blood bilirubin - 56 mmol/l, indirect-38 mmol/l. choose a treatment method:
100% Splenectomy
0% Spleen transplant
0% Portocaval anastomosis
0% Omentosplenopexy
0% Omentohepatopexy

1364. Pervovagitnaya went to the women's consultation on 22.03.03 with complaints of pulling pain
in the lower abdomen. From the medical history-the last menstruation was on 4.01.03 of the year. During bimanual
examination: the cervix is preserved, the external eye is closed, the uterus is enlarged up to 9 weeks
of pregnancy, mobile, painless. What complication should be suspected?
100% Risk of miscarriage at 9 weeks of pregnancy
0% Abortion that started at 9 weeks of pregnancy
0% Uterine fibroids
0% Frozen pregnancy
0% Bubble drift

1365. on the 15th day after a minor injury to the right foot, the patient felt malaise,
fatigue, irritability, headache, increased body temperature, a feeling of compression,
tension and twitching of the muscles of the right shin. What kind of disease can you think about?
100% Tetanus
0% Anaerobic gas gangrene
0% Erysipelas

313

Downloaded from the site - online testing step

0% Acute thrombophlebitis
0% Popliteal artery throm6oem6olism

1366. the patient had a fracture of the middle third of the left femur 10 years ago, after which
, over the past 7 years, acute inflammation periodically appears in the area of the previous fracture
https://translate.yandex.com/en/doc 301/540
22:20 ,27.6.2023 �� C : ; 5 B
with the formation of a fistula, due to which purulent contents with small fragments
of bone tissue are released. After a while, the fistula closes. What complication of a fracture
can you think about?
100% Chronic osteomyelitis
0% Bone tuberculosis
0% Soft tissue phlegmon
0% False joint
0% Trophic ulcer

1367. A 36-year-old female patient was found to have a deformity


of the cervix caused by old postpartum ruptures during a routine examination in mirrors. Colposcopic examination
revealed areas of dysplasia on the posterior lip. What should be done to clarify the diagnosis?
100% Cervical biopsy
0% Diagnostic curettage
0% Cystoscopy, irrigoscopy
0% Bacteriological examination of secretions
0% Ultrasound of the pelvic organs

1368. A 22-year-old woman went to the women's clinic on the 15th day after cesarean
section complaining of pain in the right breast, fever up to 39.0°C*,
and chills. Objectively: the mammary gland is enlarged, hyperemic, compacted, painful during
palpation. The child is breastfed from 8 days, the milk is expressed irregularly. What is the prevention
of this condition?
Regular breast pumping after feeding, prenatal preparation
100%
of nipples and mammary glands
0% Regulation of the menstrual cycle
0% Regular medical examinations, use of the IUD
0% Expressing both mammary glands after each feeding
0% Conducting professional examinations

1369. a 43-year-old man is undergoing a medical examination. Objectively: pallor of the skin and mucous
membranes, recollection of the papillae of the tongue, transverse immutability of the nails, cracks in the corners of the mouth,
tachycardia. Hemoglobin content in the blood -90 g / l; anisocytosis, poikilocytosis. The most likely
causal factor of this condition is insufficient intake of the following
trace element in the body:
100% Iron
0% Copper
0% Zinc
0% Magnesium
0% Selenium

314

Downloaded from the site - online testing step

1370.in the ward of the therapeutic department, an analysis of the level of natural lighting should be carried
out. Please indicate which device is used to determine the level of natural
light:
100% Lux Meter
0% Anemometer
0% Catathermometer
0% Actinometer

https://translate.yandex.com/en/doc 302/540
22:20 ,27.6.2023 �� C : ; 5 B
0% Psychrometer

1371. a 37-year-old woman complains of a sore throat. Body temperature-36.9°C*, general condition
is satisfactory. A patient with another disease takes prednisone for a long time.
Oropharyngoscopia: the pharyngeal mucosa is somewhat hyperemic, the palatine tonsils and
anterior arches of the soft palate show whitish-gray layers that are easily
removed; the mucosal surface under them is somewhat eroded. What is the most likely diagnosis?
100% Pharyngomycosis
0% Ziva's diphtheria
0% Ulcerative film sore throat of Plaut-Vincent
0% Lacunar sore throat
0% Pharyngeal tuberculosis

1372. a 54-year-old woman applied to the gynecologist with complaints of blood discharge from the vagina
for 1 month. Last menstruation 5 years ago. During the gynecological examination, no pathology
was detected. Your actions:
100% Fractional diagnostic curettage of the uterine cavity walls
0% Colposcopy
0% Ultrasound examination
0% Take a smear for cytological examination
0% Prescribe symptomatic therapy

1373. A plot of land with a total area of 2.0 hectares has been allocated for the construction of a hospital in the district center
. What is the maximum capacity hospital that can be built on this
land plot?
100% Per 100 beds
0% For 200 beds
0% For 400 beds
0% For 800 beds
0% More than 1000 beds

1374. A 33-year-old woman has had two ectopic pregnancy surgeries in the past,
and both fallopian tubes have been removed. I asked about the possibility of having a child. What should
be recommended for pregnancy in this case?
100% In Vitro fertilization
0% Male sperm insemination
0% Surrogate motherhood

315

Downloaded from the site - online testing step

0% Artificial insemination with donor sperm


0% Ovulation induction

1375. A female employee with a diagnosis of acute dysentery was referred by a health center doctor to the
infectious diseases department of the hospital. What document should I use to record this
disease?
100% Emergency notification of an infectious disease
0% Statistical coupon for registration of final (updated) diagnoses
0% Medical card of an outpatient patient

https://translate.yandex.com/en/doc 303/540
22:20 ,27.6.2023 �� C : ; 5 B
0% Medical card of an inpatient patient
0% Statistical map of a patient who left the hospital

1376. A 47-year-old woman has been complaining of a pada-like headache for the past 5 years.
The pain is unilateral, intense, localized in the frontal region,
accompanied by nausea and abdominal discomfort, and begins suddenly. Before seizures
, blurred vision is noted. In the anamnesis-episodes of high blood pressure, but
at the moment constantly does not take any medications. Between attacks of headache-
the condition is satisfactory. Objectively: increased nutrition (body mass index - 29), blood pressure-170/95 mm
Hg.st. neurological status without special features. What is the most likely diagnosis?
100% Migraines
0% Chronic subdural hematoma
0% Epilepsy
0% Benign intracranial hypertension
0% Hypertensive encephalopathy

1377. a 7-year-old boy suddenly developed pain and swelling in his right knee. The day before
, I took part in a cross-country race at school. There is no family history of hemophilia or
increased bleeding. Objectively: the body temperature is 37.50 C. the knee is painful to
touch, hot to the touch, swollen with local tension of the tissues above it. In the blood:
NY123 g / l, leuc. - 5, 6 * 109/l, blood clot.- 354 * 109/ l, prothrombin time-12 sec (normal-10-15 sec),
partially activated thromboplastin time-72 sec (normal-3545 sec). Bleeding time
is normal, factor VlII: C-5% of normal. What is the most likely diagnosis?
100% Hemophilia A
0% Hemophilia B
0% Schonlein-Yenoch disease
0% Lack of vitamin K
100% Thrombocytopenia 3 * * * *

1378. A 54-year-old man was taken to Lee Carney complaining of acute, sudden chest pain
that occurred while lifting a heavy load. Pain localized in the central
part of the chest without radiating, does not increase with changes in body position,
is accompanied by nausea, without vomiting. There are no respiratory symptoms. He has a history
of hypertension and is taking ACE inhibitors. Objectively: the skin is pale and moist. Ps-115/min, PDR-20 / min.
No significant changes were detected in the internal organs. On the ECG-sinus tachycardia.
Chest X-ray shows darkening in the upper left and lower right areas. The level of cardiac
enzymes is normal. What is the most likely diagnosis?

316

Downloaded from the site - online testing step

100% Dissecting aortic aneurysm


0% Myocarditis
0% Acute pericarditis
0% Pinched esophageal hernia
0% Acute myocardial infarction

1379. a 55-year-old man on the 3rd day after acute anterior myocardial infarction complains of
dull pain behind the sternum, which decreases when leaning forward, shortness of breath. Objectively:
AT140 / 80 mm Hg, heart sounds are deaf. On the ECG: atrial fibrillation with
a ventricular contraction rate of 110 / min, abnormal Q wave and S-T segment elevation in the right thoracic leads.
Tromboli-ZISu refused. What is the most likely diagnosis?

https://translate.yandex.com/en/doc 304/540
22:20 ,27.6.2023 �� C : ; 5 B
100% Acute pericarditis
0% Pulmonary embolism
0% Tietze syndrome
0% Dissecting aortic aneurysm
0% Dressler's syndrome

1380. a 54-year-old man was admitted to the hospital complaining of a sudden severe headache in
the occipital region and vomiting. He has a history of moderate arterial hypertension and is taking
hypothiazide. Three days ago, I went to the therapist about a severe headache, which was
stopped by taking an analgesic. Objectively: consciousness is confused, the left pupil
is dilated. Severe photophobia and neck muscle tension. Left-sided hemiparesis with
increased muscle tone and reflexes. The temperature is reduced, there is no rash.
AT-230/130 mm Hg, Ps-50/min, PDR-12 / min. What is the most likely diagnosis?
100% Acute subdural hematoma
0% Myasthenia gravis
0% Multiple sclerosis
0% Migraines
0% Acute bacterial meningitis

1381. A 78-year-old woman complains of spontaneous pain in the lumbar region. The pain increases
during movement and decreases at rest, does not radiate. Temperature is normal, body weight has not
changed. Objectively: peripheral joints without special features, moderate obesity,
limited mobility in the lumbar spine. Local soft tissue tension over the 2nd
lumbar vertebra. In the blood: HB-147 g / l, leuc.- 8,8 * 109/l, blood clot.- 222* 109/l, SSE-5 mm / h,
Na+-140 mmol/L, K+ - 4.2 mmol/L, Ca2+ - 2.35 mmol / l. what is the most likely diagnosis?
100% Osteoporosis with a pathological fracture
0% Amyloidosis
0% Secondary hyperparathyroidism
0% Myeloma disease
0% Sciatica

1382. a 58-year-old man who abuses alcohol and tobacco smoking was admitted to the hospital with
complaints of a persistent cough, shortness of breath. I've lost weight recently. Objectively: the cervical lymph
nodes are enlarged, dense, and the tissues above them are not tense. On the chest X-ray

317

Downloaded from the site - online testing step

cells-fibrosis of the upper lobe of the lung, left-sided pleurisy. The pleural cavity is straw
-colored with a protein content of 52 g / l and a large number of lymphocytes. No malignant cells were detected.
When seeding pleural fluid in a week,there is no growth. What is the most likely diagnosis?
100% Tuberculosis of the lungs
0% Atypical pneumonia
0% Systemic lupus erythematosus
0% Bronchiectasis
0% Sarcoidosis

1383. a 35-year-old man complains of chest pain for several months,


and occasionally has a bitter taste in his mouth. The pain is localized behind the sternum, occurs at rest
and sometimes radiates to the neck, does not increase with physical exertion, and can increase
after drinking alcohol and eating a large amount of food. The condition worsens at night. Swallowing is not

https://translate.yandex.com/en/doc 305/540
22:20 ,27.6.2023 �� C : ; 5 B
disturbed, body weight is increased. No changes were found during the inspection. What is the most likely
diagnosis?
100% Gastro-esophageal reflux disease
0% Achalasia of the esophagus
0% Esophageal tumor
0% Phrenic hernia
0% Globus hystericus

1384. A 76-year-old male complains of epigastric pain with intermittent vomiting


during the last 2 months. Dysphagia and vomiting of "coffee grounds" were not observed. During this
period, I lost 5 kg of weight, my appetite is reduced. Does not smoke, does not abuse alcohol. Previously, in connection with
gastric dyspepsia, I took antacids and proton pump inhibitors, but recently these
drugs do not bring relief. Objectively: low nutrition,
a lymph node is palpated in the left supraclavicular fossa. The liver is not enlarged. What is the most likely diagnosis?
100% Gastric carcinoma
0% Stomach ulcer
0% Esophageal tumor
0% Pylorostenosis
0% Phrenic hernia

1385. A 76-year-old woman complains of progressive swallowing disorders, mainly of solid food,
during the last 6 weeks. Sometimes it notes regurgitation of solid masses. Pain during
swallowing is not noted. I lost 6 kg. 10 years ago I suffered a myocardial infarction, constantly
takes aspirin and prolonged nitrates. Uses alcohol in moderation, smokes. Objectively: the skin
is icteric, the neck is without features, the lymph nodes are not enlarged. The chest is not changed;
there are no pronounced changes in the cardiovascular system. Liver + 3 cm. What is the most
likely diagnosis?
100% Esophageal cancer
0% Phrenic hernia
0% Diffuse narrowing of the esophagus
0% Myasthenia gravis
0% Achalasia of the esophagus
1386. a 38-year-old man complains of periodic difficulty swallowing both solid and liquid food.

318

Downloaded from the site - online testing step

food for many months. Sometimes there is severe pain behind the sternum, especially after
a hot drink. There are attacks of suffocation at night. I didn't lose any weight. Objectively:
the general condition is satisfactory, the skin is of normal color. There were no changes in the
gastrointestinal tract during the examination. On the chest X-ray
,the esophagus is dilated with the level of fluid in it. What is the most likely diagnosis?
100% Achalasia of the esophagus
0% Myasthenia gravis
0% Esophageal cancer
0% Esophageal candidiasis
0% Gastro-esophageal reflux disease

1387. A 45-year-old woman complains of progressive weakness,


discomfort and bloating during the last 6 weeks. I didn't lose any weight, but I became apathetic. Constantly uses
alcohol. Objectively: the abdomen is enlarged, tense. Defecation is not disturbed. Ultrasound
of the abdominal organs-a small amount of ascitic fluid; liver, kidneys, spleen

https://translate.yandex.com/en/doc 306/540
22:20 ,27.6.2023 �� C : ; 5 B
are unchanged, there are several cysts in the left ovary, the right one is not visualized due to the formation associated with
100%
the omentum. InOvarian carcinoma
the urine-the norm. What is the most likely diagnosis?
0% Colon lymphoma
0% Sigmoid colon cancer
0% Alcoholic liver disease
0% Crohn's disease

1388. A 35-year-old woman complained of periodic pain in the parotid and left
iliac regions, which decreased after defecation or gas discharge. Bowel movements up to 6
times a day, unformed, with admixtures of mucus. My appetite is normal, I haven't lost any weight. Similar
symptoms were observed for 1.5 years, but according to coloscopy, no organic changes were
found. Objectively: the abdomen is soft, somewhat painful on palpation in the left iliac region.
Blood and urine levels are normal. What is the most likely diagnosis?
100% Irritable bowel syndrome
0% Celiac disease
0% Crohn's disease
0% Pseudomembranous colitis
0% Dyspancreatism

1389. A 46-year-old woman woke up at 3 a.m. with a sharp pain in her right hypochondrium
radiating to her right shoulder. Restless, vomiting twice, notes fever and sweating.
The temperature is 39.0 oC. objectively: tension of the abdominal muscles in the right hypochondrium. what
is the most likely diagnosis?
100% Acute cholecystitis
0% Peptic ulcer disease
0% Unstable angina pectoris
0% Cholelithiasis
0% Benign stricture of the common bile duct

319

Downloaded from the site - online testing step

1390. a 76-year-old woman complains of insomnia at night and early awakening in the morning,
memory loss. Worries about the possible development of dementia; feels unnecessary to his
family. CT scans of the brain showed no changes. What is the most likely diagnosis?
100% Depression
0% Alzheimer's disease
0% Vascular dementia
0% Age-related memory loss
0% Dementia due to frontal lobe involvement

1391. a 58-year-old man complains of the appearance of a tumor in the groin area, which increases
during straining and coughing. Objectively, there is a pulsating tumor formation
below and laterally to the puparto ligament, which does not decrease during palpation. What
is the most likely diagnosis?
100% Femoral artery aneurysm
0% Femoral hernia
0% Ectopic testicle
0% Inguinal hernia

https://translate.yandex.com/en/doc 307/540
22:20 ,27.6.2023 �� C : ; 5 B
0% Neurinoma of the femoral nerve

1392. a 60-year-old man complains of a feeling of heaviness in the scrotum. Objectively:


left scrotum edema. The testicle is of normal size, but a soft swelling is palpable above it,
limited to the scrotum, which can be compressed and disappears when the patient lies down. What is the most
likely diagnosis?
100% Varicocele
0% Inguinal lymphadenopathy
0% Ectopic testicle
0% Inguinal hernia
0% Varicose veins of the subcutaneous veins

1393. an 8-year-old girl complained of general weakness and pain during swallowing in the morning.
In the afternoon, the parents called the doctor because the girl had advanced weakness and developed
a bilateral swelling slightly below and in front of her ears. What is the most likely diagnosis?
100% Mumps infection
0% Lymphadenopathy
0% Dermoid cysts
0% Salivary gland tumor
0% Carotid body tumor

1394. A 35-year-old woman complains of progressive pain in her left arm for 4 months,
which increases after physical exertion on this limb. Also notes a feeling of cold and
"tingling needles" in the hand. The pain is somewhat reduced at night with the arm hanging down
from the bed. In addition, she notes a deterioration in vision, lost 4 kg of weight. He doesn't smoke. Objectively:
low power, no pulse detected on the left arm. There is no pulsation on the left carotid artery
, but a noise is heard above the right carotid artery. What is the most likely diagnosis?
100% Obliterating aortoarteritis

320

Downloaded from the site - online testing step

0% Dermatomyositis
0% Systemic lupus erythematosus
0% Neurological amyotrophy
0% Arterial thrombosis of the left upper limb

1395. a 25-year-old man complains of weakness, progressive shortness of breath and swelling of the lower legs. Before
that, he was healthy, but recently he took ibuprofen for a sprained ligament
of the small-step joint. Objectively: Ps-90 / min, blood pressure-180/100 mm Hg. heart tones are sonorous. In
the lungs, the percussion sound is dulled in the lower right part. Liver + 3 cm. In the
blood: HB-103 g/l, leuc. - 6.7 * 109/l, thrombus. -236 * 109/l, urea-24.6 mmol/l, creatinine -0.254
mmol/l, Na+ - 135 mmol/l, K+- 5.6 mmol/ll, albumin -27 g/l. on radiography of OGK-right-
sided pleurisy, heart is normal. What is the most likely diagnosis?
100% Nephrotic syndrome
0% Acute renal failure
0% Tuberculosis of the kidneys
0% Acute pyelonephritis
0% Nephritic syndrome

1396.a sample of milk was sent to the laboratory for research. The following

https://translate.yandex.com/en/doc 308/540
22:20 ,27.6.2023 �� C : ; 5 B
data were found: color-whitish, smell - without features, taste-characteristic of milk, density -1.038
, acidity-350 Turner, fat content -3.2%. What is the degree of milk quality?
100% Milk of poor quality
0% Milk of good quality
0% Reduced quality milk
0% Adulterated milk
0% Milk is conditionally suitable

1397. A 74-year-old patient has been suffering from benign prostatic hyperplasia for the last 5 years
. 4 days ago, after drinking alcohol, there was an acute urinary retention. At
the pre-hospital stage, the bladder was catheterized twice a day with a metal catheter. On
examination: the appendage of the right testicle is enlarged, compacted, painful; there is purulent discharge from
the urethra. What type of emergency care should I choose?
100% Trocar or open epicystomy
0% Transurethral resection or prostatectomy
0% Installing a permanent urethral catheter
0% Microwave Prostate Thermotherapy
0% Installing an intraprostatic stent

1398. a 28-year-old man became acutely ill when chills, a feeling of heat,
fever up to 38.50 C, napal-like pain in the left iliac region, frequent liquid
bowel movements in the form of a bloody-mucous mass appeared. During palpation of the abdomen
, soreness is noted in its left half, the sigmoid colon is spasmodic. What is the most likely
diagnosis?
100% Acute dysentery
0% Amoebiasis

321

Downloaded from the site - online testing step

0% Escherichiosis
0% Non-specific ulcerative colitis
0% Malignant tumor of the colon

1399. a 30-year-old man complains of severe pain, redness of the skin, swelling in the
ankle joint, fever up to 390C. got sick suddenly. In the past
, there were similar attacks lasting up to 56 days without any residual changes in the joint. The skin above the joint
is hyperemic, without clear contours and an infiltrative shaft on the periphery. What is the most
likely diagnosis?
100% Gout
0% Infectious arthritis
0% Rheumatoid arthritis
0% Erysipelas
0% Osteoarthritis

1400. A 27-year-old patient has been experiencing fatigue, sweating,and heaviness in the left hypochondrium for about a year,
especially after eating. Objectively: enlargement of the spleen, liver. In the blood: Er-3.2 *
1012/l, Hb-100 g/l, CP-0.87, leuc.- 100 * 109/l, b. -7%, e. - 5%, m. - 15%, s. - 16%, s. - 10%, s. - 45%,
l. - 2%, m. - 0%, reticule -0.3%, thrombus.- 400 * 109/l, WSE-25 mm/hour. What is the most
likely diagnosis?
100% Chronic myeloid leukemia

https://translate.yandex.com/en/doc 309/540
22:20 ,27.6.2023 �� C : ; 5 B
0% Chronic lymphocytic leukemia
0% Acute leukemia
0% Erythremia
0% Cirrhosis of the liver

1401. a boy was admitted to the surgical department on the first day after birth with
foamy discharge from the nose and mouth, attacks of cyanosis. Radiologically: blind end
of the esophagus at the level of the II thoracic vertebra, gas bubble of the stomach under the left dome of the diaphragm.
What is the most likely diagnosis?
100% Esophageal atresia, tracheo-esophageal fistula
0% Total esophageal atresia
0% Paraesophageal hernia of the esophageal orifice of the diaphragm
0% Esophageal atresia without fistulas
0% Bronchoesophageal fistula

1402. When studying the incidence of pulmonary tuberculosis, data were obtained on
the socio-economic living conditions and harmful habits of patients. Which of these methods
allows us to assess the degree of influence of these factors on the incidence of tuberculosis?
100% Calculating the correlation coefficient
0% Calculating the compliance indicator
0% Calculating the regression coefficient
0% Calculating standardized metrics
0% Calculating the confidence factor
1403. within 2 hours after the birth of the fetus, the state of the child is good: the uterus is dense, spherical,

322

Downloaded from the site - online testing step

its bottom is at the level of the navel, there is no bleeding. The clamp applied to the umbilical cord segment
is at a preliminary level, with a deep breath and when pressing the edge of the palm
over the symphysis, the umbilical cord is pulled into the vagina. There is no bloody discharge from the genital tract.
What will be the doctor's next strategy?
100% Perform manual placental separation
0% Apply the Abuladze method
0% Apply the Kredit-Lazarevich method
0% Perform curettage of the uterine cavity
0% Intravenous administration of oxytocin

1404. a 34-year-old patient has been suffering from pulmonary tuberculosis for 7 years; he is suffering
from muscle weakness, weight loss, diarrhea, and frequent urination. Objectively:
hyperpigmentation of the skin, gums, and inner cheek surfaces. AT-90/58 mm Hg in the blood: Er.- 3,1 *
1012/ l, Hb-95 g / l, CP-0.92; lake.- 9,4 * 109/ l, E.-7, S.-45, P.-1, L.-40, M.-7, Na + - 115 mmol / l,
k+ - 7.3 mmol / l. what is the preliminary diagnosis?
100% Primary insufficiency of the cranial cortex
0% Pheochromocytoma
0% Primary hyperaldosteronism
0% Congenital hyperplasia of the adrenal cortex
0% Diabetes insipidus

1405. A child was born with an Apgar score of 8-9 points. When should I apply it to
my chest?
100% After treatment of the umbilical cord and prevention of gonoblenorrhea
https://translate.yandex.com/en/doc 310/540
22:20 ,27.6.2023 �� C : ; 5 B

0% Immediately after birth


0% After umbilical cord treatment
0% 30 minutes after birth
0% 2 hours after birth

1406. after surgery for a breakthrough stomach ulcer, end-stage


diffuse peritonitis, and endotoxic shock, the patient is
given artificial ventilation with 60% oxygen inhalation in the postoperative period. Blood gases: Pa02-70-78 mmHg,
hypoxemia does not decrease, CVT-150-180 mmHg, AT-90/60 mmHg (against the background of
high doses of dopamine). On the R-gram-diffuse infiltration of the lungs. What is the cause of persistent
arterial hypoxemia?
100% Respiratory distress syndrome
0% Bilateral pneumonia
0% Pneumothorax
0% Mendelssohn's syndrome
0% Pulmonary edema

1407. based on the results of hospital-pedagogical observation of a physical education lesson in the 9th
grade, a physiological curve is constructed, characterized by a gradual increase
in the pulse rate in the introductory part, an increase in the pulse rate by 80% during the main part; the curve has
a 4-toothed form. How can I evaluate the organization of a physical education lesson?

323

Downloaded from the site - online testing step

100% Long intervals between exercises


0% The lesson is built correctly
0% Physical activity is insufficient
0% Excessive physical activity
0% Physical activity is sufficient

1408. a 16-year-old patient. Over the past year, my behavior has gradually changed: I began
to withdraw, lost interest in my friends and studies. He became indifferent to his family, was
unreasonably rude, talked to himself or laughed. The question is answered formally
correctly, with little justification. He considers himself quite healthy, but somewhat tired, says that
he is considering writing a book “the projection of humanity on the plane of the Universe " and carries a notebook
with its pages filled with many identical crosses. What is the most likely diagnosis?
100% Schizophrenia
0% Depressive disorder
0% Schizoid personality disorder
0% Autistic Personality Disorder
0% Pick's disease

1409. the employee was temporarily disabled for 16 days due to illness.
He was treated on an outpatient basis. The doctor-curator issued a disability certificate first for 5 days and
extended it to 10 days. Who can continue the disability certificate for this
employee?
100% Doctor-curator together with the head of the department
0% Medical Advisory Commission
0% Curator doctor with the permission of the chief Physician

https://translate.yandex.com/en/doc 311/540
22:20 ,27.6.2023 �� C : ; 5 B
0% Deputy Chief Physician for Disability Assessment
0% Department Head

1410. in a 37-year-old pervovagit woman, labor continues for 10 hours. Contractions for
20-25 seconds. after 6-7 min. The fetal position is longitudinal, with the head pressed against
the entrance to the small pelvis. During vaginal examination: the cervix is up to 1 cm long, passes 2
transverse fingers. There is no fertile bubble. What is the most likely diagnosis?
100% Primary weakness of labor activity
0% Secondary weakness of labor activity
0% Normal labor activity
0% Discoordination of labor activity
0% Pathological preliminary period

1411. at the reception, the patient complains of fever up to 38, 20C, the presence of edema
in the upper lip. Objectively: the upper lip is sharply swollen, with a cone-
shaped swelling in the center of the edema. The skin and mucous membrane above it are dark red. Diagnosis:
upper lip furuncle. The surgeon performed an autopsy of the boil, treated the wound with a solution
of hydrogen peroxide and applied a bandage with a hypertonic solution. What treatment regimen should
be recommended to the patient?
100% Inpatient treatment with a general regimen

324

Downloaded from the site - online testing step

0% Outpatient treatment, followed by inpatient treatment


0% Outpatient treatment
0% Treatment is inpatient with a bed regime
0% -

1412. A 15-year-old boy suffered two bee attacks, resulting in severe anaphylactic
shock. Which of the following is the most effective method of prevention?
100% Desensitization with bee venom extract
0% Prescribing corticosteroids during the summer
0% Long-term preventive treatment with antihistamines
0% Restriction of outdoor use during the summer months
0% Protective cloth

1413. in a healthy 75-year-old woman who leads a moderately active lifestyle, a preventive
examination revealed a serum concentration of total cholesterol at the level of 5.1 mmol/l (208
mg / dl) and HDL cholesterol at 70 mg / dl. ECG without pathology. Which of the listed dietary
recommendations is most acceptable?
100% No dietary changes
0% Reducing cholesterol intake
0% Reduced saturated fat intake
0% Reduce your intake of simple carbohydrates
0% Increase your fiber intake

1414. A 6-year-old patient complains of significant weakness. He became acutely ill with
fever, malaise, pain in the joints and in the course of the leg muscles. Objective:
purple-blue erythema around the eyes and above the knee joints. Heart rate-120 / min., heart sounds
are weakened. In the blood: lake.-12 * 109/l, WSE-40 mm / h. What is the most likely diagnosis?

https://translate.yandex.com/en/doc 312/540
22:20 ,27.6.2023 �� C : ; 5 B
100% Dermatomyositis
0% Systemic lupus erythematosus
0% Rheumatoid arthritis
0% Atopic dermatitis
0% Reactive polyarthritis

1415. the district doctor was instructed to prepare a plan for carrying out a complex
of therapeutic and preventive measures among the population in the subordinate territory. What
secondary disease prevention measures should it include in this plan?
100% Prevention of disease complications
0% Prevention of diseases
0% Elimination of disease-causing factors
0% Improving the living conditions of the population
0% Implementation of rehabilitation measures

1416. A 20-year-old patient was admitted to the clinic on the 7th day of illness with complaints of bloodied
skin and sclera, dark urine, single vomiting, decreased appetite, fever

325

Downloaded from the site - online testing step

body temperature is up to 380C for 2 days. Three weeks ago on a fishing trip I used the same dishes with
my friends. Objectively: flaccid, t0-36.80 S, the skin and sclera are jaundiced, the liver protrudes 3 cm from the
edge of the costal arch, is sensitive during palpation; the spleen is not palpated.
The urine is dark, and the feces are partially acholic. What is the most likely diagnosis?
100% Viral hepatitis A
0% Leptospirosis
0% Infectious mononucleosis
0% Hemolytic anemia
0% Intestinal yersiniosis

1417. A 7-year-old boy got sick 2 weeks ago when he developed a runny nose. I took drops in the nose.
I went to an ENT doctor for purulent-bloody discharge from the nose, maceration of the wings of the nose and
upper lip. Rhinoscopy shows whitish-gray islets on the nasal membrane. The oropharyngeal mucosa
is not changed. What is the most likely medical condition?
100% Diphtheria of the nose
0% Adenovirus infection
0% Rhinovirus infection
0% Allergic rhinitis
0% Sinusitis

1418. a full-term newborn child was diagnosed with Rh factor-related hemolytic disease of newborns
. Indicators of bilirubin content are critical. The child's blood type is B(III), the mother's
is A (II). Replacement blood transfusion is indicated. What kind of blood donation selection is necessary for this?
100% Blood type B (III), Rh ( -)
0% Blood type A (II), Rh ( -)
0% Blood type B (III), Rh (+)
0% Blood type A (II), Rh (+)
0% Blood type O (i), Rh ( -)

https://translate.yandex.com/en/doc 313/540
22:20 ,27.6.2023 �� C : ; 5 B
1419. a 33-year-old patient with newly diagnosed diabetes mellitus uses a diet to maintain
glycemia after eating less than 10.0 mmol / l. he refrains from insulin therapy. What
is the most important study to differentiate between type 1 (insulin-dependent) and type 2
(insulin-dependent) diabetes?
100% Determination of antibodies to islet cells
0% Glucose tolerance test
0% Fasting blood glucose test
0% Determination of glycosylated hemoglobin in the blood
0% Determination of fructosamine in the blood

1420. a 55-year-old man complains of general weakness, decreased fluid excretion,


and aching heart pain. He has been suffering from chronic pyelonephritis for 15 years
and has been treated in hospitals. Objectively: the skin is dry, with a yellowish tinge. Ps-80 / min,
rhythmic, BP-100/70 mm Hg during auscultation of the heart-muffled tones, pericardial friction noise.
After additional studies: creatinine -1.1 mmol / l, glomerular filtration 5 ml / min. What
treatment is indicated for the patient?

326

Downloaded from the site - online testing step

100% Hemodialysis
0% Xylitol, sorbitol
0% Rheopolyglukin
0% Antibiotics
0% Diuretics

1421. A 37-year-old patient has been experiencing frequent nosebleeds, severe


metrorrhagia, and periodic bruising of the skin for the past 6 years. 10 days ago, after
a significant nosebleed, weakness increased, dizziness and palpitations appeared. Objectively:
the skin is pale, petechial hemorrhages and isolated ecchymoses are common on the anterior surface of the trunk, legs and arms
. In the blood: HB-80 g / l, ep.- 4,0 * 1012/ l, CP-0.7; leuc. - 5, 3 * 109/l; p-2%,
c65%, e-2%, l-24%, m-5%, blood clot.- 10 * 109/l, WSE-15 mm / year. What is the most likely diagnosis?
100% Werlhof's disease
0% Hemophilia
0% Hemorrhagic vasculitis
0% Iron deficiency anemia
0% Aplastic anemia

1422. A 7-year-old boy has been undergoing medical treatment for a month. During hospitalization
, severe edema was observed, proteinuria-7.1 g / l, Protein in the daily urine-4.2 g.in
the biochemical analysis of blood, hypoproteinemia (43.2 g/l), hypercholesterolemia (9.2
mmol/l) were retained. Which of these variants of glomerulonephritis is most likely to occur in the patient?
100% Nephrotic
0% Nephritic
0% Isolated urinary tract
0% Hematuric
0% Mixed

1423. A 32-year-old patient who underwent an appendectomy for gangrenous appendicitis


was diagnosed with a pelvic abscess on the 6th day after the operation. Specify the best way to open
the pelvic abscess in this patient:
100% Through the rectum
https://translate.yandex.com/en/doc 314/540
22:20 ,27.6.2023 �� C : ; 5 B

0% Through the postoperative wound


0% Sacred access
0% Through the anterior abdominal wall
0% Through the zatulnoe hole

1424. A 58-year-old patient underwent urgent cholecystectomy


and abdominal drainage. In the postoperative period, heparin was prescribed, and hemorrhagic
syndrome was noted. What drug should be used to eliminate the side effect of heparin?
100% 1% protamine sulfate solution
0% 10% calcium chloride solution
0% 1% calcium chloride solution
0% 1% vikasol solution

327

Downloaded from the site - online testing step

0% 5% hydrocaproic acid solution

1425. a 3-year-old girl has an increase in body temperature up


to 380C, a runny nose, a dry superficial cough, weakness, and a decrease in appetite for 2 days. There were no palpatory changes over
the lungs. Percussion sound with a boxy tinge, auscultative puerile
breathing, no wheezing. In the blood-leukopenia, lymphocytosis, accelerated SSE. What is the most
likely diagnosis?
100% Acute simple tracheitis
0% Acute obstructive bronchitis
0% Recurrent bronchitis, acute phase
0% Acute simple bronchitis
0% Bilateral small-focal pneumonia

1426. after taking two aspirin tablets, a 12-year — old girl


's body temperature rose to 39-400C in 4 hours.complains of general malaise, dizziness,
sudden appearance of red spots and blisters on the skin. Objectively: the lesions on the skin
resemble second-degree burns, sometimes with surface erosion or peeling
of the epidermis. Nikolsky's symptom is positive. What is the most likely diagnosis?
100% Acute epidermal necrolysis
0% Pemphigus vulgarus
0% Polymorphic exudative erythema
0% Bullous dermatitis
0% Duhring's herpetiform dermatitis

1427. a newborn with suspected intracranial birth trauma underwent


a lumbar puncture. A bloody cerebrospinal fluid was obtained. What kind of hemorrhage occurs in this
attack?
100% Subarachnoid
0% Cephalohematoma
0% Epidural
0% Supratentorial
0% Subtentorial

https://translate.yandex.com/en/doc 315/540
22:20 ,27.6.2023 �� C : ; 5 B
1428. a 54-year-old female patient has had femoral osteomyelitis for more than 20 years. Over the past month
, edema of the lower extremities has appeared and gradually increased. In the urine: proteinuria - 6.6 g / l. in
the blood: dysproteinemia in the form of hypoalbuminemia, increased A2-and y-globulins, SSE-50
mm/h. What is the most likely diagnosis?
100% Secondary amyloidosis of the kidneys
0% Acute glomerulonephritis
0% Myeloma disease
0% Chronic glomerulonephritis
0% Systemic lupus erythematosus

1429. the mine well is located on the territory of the household plot at a distance of 20 m from the

328

Downloaded from the site - online testing step

residential building, 10 m - from the toilet, 15 m - from the neighbor's house. What is the smallest distance,
according to sanitary standards, should be between the well and the source of possible
water contamination?
100% 30 m
0% 25 m
0% 20 m
0% 15 m
0% 10 m

1430. on the 4th day after surgery for right ovarian cysts, a patient
suddenly developed pain in the right side of the chest with pink sputum
discharge, and an increase in body temperature to 37.70 C. During the examination of the lungs
, a dulled pulmonary sound was detected and isolated wet wheezing in the lower parts on the right side. What
is the most likely complication?
100% Lung infarction
0% Pneumonia
0% Lung abscess
0% Exudative pleurisy
0% Pneumothorax

1431. in a 40-year-old woman, palpation of the thyroid gland revealed a knot in the left lobe,
compacted, moderately painful. During ultrasound examination, it is of increased density,
"cold" in scintigraphy with 131i. what is the most appropriate solution to clarify
the diagnosis?
100% Aspiration fine needle biopsy
0% Determination of TSH levels in the blood
0% Reflexometry
0% Determination of urinary iodine excretion
0% Thermography

1432. a 14-year-old child suffering from vegetative-vascular dystonia of puberty


developed a sympathoadrenal crisis. Which drug is indicated for crisis management?
100% Obzidan
0% No-shpa
0% Amizil
0% Euphyllinum
https://translate.yandex.com/en/doc 316/540
22:20 ,27.6.2023 �� C : ; 5 B

0% Korglikon

1433. a 16-year-old teenager complains of weakness, dizziness, and heaviness in the left
hypochondrium. objective: skin and visible mucous jaundice. Tower skull. Liver + 2 cm,
lower pole of the spleen at the level of the navel. In the blood: Er-2.7 * 1012/l, Hb-88 g/l, leuc. -5.6 * 109/l,
SSE-15 mm / h. Indicate the most likely change in the patient's bilirubin level:
100% Increase in free bilirubin
0% Increased bound bilirubin
0% Increased free and bound bilirubin

329

Downloaded from the site - online testing step

0% Reduced bound bilirubin


0% Reduced free bilirubin

1434. a 28-year-old woman went to the doctor with complaints of facial puffiness, moderate
swelling of the legs, and periodically notes urine the color of "meat slop". As a teenager, she often
had sore throats. Objectively: the skin is pale, t0-36.80 s, Ps-68 / min, rhythmic. AT-170/110
mmHg. what changes in urine are most likely?
100% Proteinuria, hematuria, and cylindruria
0% Increased relative density, hematuria, bacteriuria
0% Reduced relative density, proteinuria, slight urinary sediment
0% Erythrocyturia and urinosuria
0% Reduced relative density, proteinuria, myoglobinuria

1435. a 58-year-old woman suffered heavy bleeding from a torn varicose node on
her left shin. Provide first aid:
100% Elevated position of the limb, compressive sterile dressing
0% Applying a tourniquet distal to the source of bleeding
0% Applying a tourniquet proximal to the source of bleeding
0% Troyanov-Trendelenburg Operation
0% Z-shaped suture on a torn varicose node

1436. a patient is on sick leave for 4 months continuously due to an injury. The treatment
will still last 1-2 months. Who has the right to extend the disability certificate for this
patient?
100% Medical Advisory Commission after the ICEC decision
0% Chief Physician of the medical institution
0% Medical advisory board after treatment of a patient in a hospital
0% District doctor after consultation with the head of the department
0% Medical and Social Expert Commission

1437. A 39-year-old man was treated in the surgical department for acute
cholecystitis. He was transferred to the therapeutic department due to an increase
in blood pressure up to 180/120 mm Hg, the appearance of protein in the urine, and persistent fever. Resistant
subfebrility was resistant to antibiotic therapy. 10 days after treatment, the
patient experienced an attack of suffocation with difficulty exhaling; subsequently, arthralgias and
erythematous skin lesions appeared. In the blood: eosinophils-18%. What disease
did the patient have?
100% Nodular periarteritis
https://translate.yandex.com/en/doc 317/540
22:20 ,27.6.2023 �� C : ; 5 B

0% Non-specific aortoarteritis
0% Acute glomerulonephritis
0% Systemic lupus erythematosus
0% Hemorrhagic vasculitis

1438. A 27-year-old electrician was electrocuted after touching a naked woman with his hand.

330

Downloaded from the site - online testing step

electrical wiring, after which circulatory and respiratory arrest were observed. Resuscitation
measures provided restoration of cardiac activity in 5 minutes. Which of the complications is possible
a few hours or even days after an electric shock?
100% Circulatory arrest
0% Respiratory arrest
0% Acute liver failure
0% Acute renal failure
0% Pulmonary edema

1439. a patient was hospitalized with complaints of periodic pain in the lower abdomen, which increases
during menstruation, weakness, malaise, nervousness, smearing dark blood discharge from
the vagina on the eve and after menstruation. During bimanual examination: the body of the uterus is enlarged,
appendages are not defined, in the posterior arch - a bumpy surface. During laparoscopy: on
the ovaries, peritoneum of the utero-rectal depression and pararectal tissue -
"blue eyes". What is the most likely diagnosis?
100% Common form of endometriosis
0% Polycystic ovary disease
0% Chronic salpingitis
0% Tuberculosis of the genitals
0% Ovarian cysts

1440. a 7-week pregnant woman is referred for an artificial abortion. During the operation, while
dilating the cervical canal with the Geghar No. 8 dilator, the doctor suspected
uterine perforation. What are the main tactics of the doctor to confirm the diagnosis?
100% Probing the uterine cavity
0% Bimanual research
0% Ultrasound examination
0% Laparoscopy
0% Metrosalpingography

1441. an 18-year-old patient complains of pain in the knee and ankle joints,
an increase in body temperature up to 39.50 C. A week and a half before that, he suffered a respiratory
illness. Objectively: body temperature 38.50 C, swelling of the knee and shin
joints. Ps-106 / min, rhythmic. AT-90/60 mm Hg. heart borders are not changed, tones are weakened,
soft systolic murmur at the apex. What indicator is most associated with the possible
etiology of the process?
100% Antistreptolysin-0
0% 1-antitrypsin
0% Creatine Kinase
0% Rheumatoid factor

https://translate.yandex.com/en/doc 318/540
22:20 ,27.6.2023 �� C : ; 5 B
0% Serumocoid

1442. A 26-year-old woman was taken to the emergency department with complaints of sudden spasms in
the lower abdomen, weakness, and loss of consciousness. There was no previous menstruation.
Objectively: Ps-120 / min, AT-80/50 mm Hg, pain and symptoms of peritoneal irritation

331

Downloaded from the site - online testing step

at the bottom on the right side. In the blood: Hb-106 g / l. what is the most likely diagnosis?
100% Disrupted tubal pregnancy
0% Acute appendicitis
0% Right-sided acute adnexitis
0% Twist the legs of the ovary
0% Ovarian apoplexy

1443. a 28-year-old patient without a permanent place of residence, who was hospitalized with a preliminary
diagnosis of “flu" on the 5th day of the disease, developed a roseolous-petechial rash on the trunk and
internal surfaces of the limbs. Temperature-410c, euphoria, flushing of the face, redness
of the sclera, tremor of the tongue, tachycardia, splenomegaly, agitation. What is the most likely diagnosis?
100% Typhus fever
0% Alcoholic delirium
0% Leptospirosis
0% Measles
0% Typhoid fever

1444. a 45-year-old man with a normal body weight was first diagnosed with diabetes mellitus.
It was not possible to adjust blood glucose with the help of a diet, glycemia during the day is from 10 to 15
mmol/l. which drug is optimal in this case?
100% Sulfonamide hypoglycemic drugs
0% Insulin
0% Biguanides
0% Biguanides in combination with sulfonamide hypoglycemic drugs
0% Insulin in combination with sulfonamide hypoglycemic drugs

1445. a 28-year-old patient complains of pain in the legs during gait, chilly feet,
fingers. He has been ill for a year. Objectively: the skin of the legs is pale, cool; turgor is low,
hypotrichosis. Pulsation on the femoral and popliteal arteries is weakened, on the arteries of the feet
palpated after a nitroglycerin test. Rheographic index <1. what is the most likely
diagnosis?
100% Obliterating endarteritis
0% Chronic thrombophlebitis
0% Obliterating atherosclerosis
0% Raynaud's disease
0% Burger's disease

1446. a child of preschool age has a harmonious physical development and correspondence
of biological age to the calendar age. There are no chronic diseases. During the year, he was ill
with acute diseases 5 times. What health group should this child belong to?
100% II
0% And
https://translate.yandex.com/en/doc 319/540
22:20 ,27.6.2023 �� C : ; 5 B

0% III
0% V

332

Downloaded from the site - online testing step

0% IV

1447.during the medical examination of the population, chronic


diseases, various pathological conditions, and abnormalities were identified and registered. What type of morbidity was studied in this
case?
100% Pathological lesion
0% Prevalence rate
0% General morbidity rate
0% Primary morbidity
0% Morbidity with temporary disability

1448. a 2-month-old girl is transferred to artificial feeding. Born with a body weight
of 3500 g. currently, the body weight is 3900 g. what is the daily volume of nutrition for this child?
100% 650 ml
0% 600 ml
0% 690 ml
0% 730 ml
0% 750 ml

1449. In April, during a medical examination of population groups, 27% of people complained of reduced
working capacity and increased fatigue. During the examination,edematous loose gums were found,
pronounced bleeding when pressing on them; on the skin - follicular hyperkeratosis, against the
background of normal skin moisture. What pathology is most likely associated with such
manifestations?
100% Hypovitaminosis With
0% Paradontosis
0% Hypovitaminosis A
0% Hypovitaminosis In and
0% Polyhypovitaminosis

1450. delivery on time ended with the birth of a full-term live girl, without asphyxia.
Objectively: the child is sluggish, the skin is pale, with an icteric tinge, there is no edema. The abdomen
is soft, the liver and spleen are enlarged. Studies have shown that the mother has a blood type-
A (II)Rh -, in a child-A (II) Rh+. What is the most likely diagnosis?
100% Hemolytic disease of the newborn
0% Impaired cerebral circulation
0% Abnormal development of parenchymal organs
0% Physiological jaundice
0% Intracranial birth trauma

1451.a woman in labor went to the doctor on the 14th day after giving birth complaining of
sudden pain, hyperemia and tightness in the left breast, fever up

https://translate.yandex.com/en/doc 320/540
22:20 ,27.6.2023 �� C : ; 5 B
to 390C, headache, malaise. Objectively: a crack in the nipple area, an increase in the volume
of the left breast, increased pain during palpation. What pathology should I think about in the future?
333

Downloaded from the site - online testing step

in this case?
100% Lactation mastitis
0% Cyst of the left breast with suppuration
0% Fibroadenoma of the left breast
0% Breast cancer
0% Phlegmon of the breast

1452. A medical commission consisting of:


therapist, dermatologist, traumatologist, optometrist, neurologist, immunologist, surgeon, otolaryngologist,
hematologist. Determine the minimum composition of the medical commission, taking into account the specifics
of the products produced by the plant:
100% Otolaryngologist, therapist, dermatologist
0% Therapist, dermatologist, traumatologist
0% Therapist, optometrist, neurologist
0% Therapist, immunologist, and surgeon
0% General practitioner, hematologist, optometrist

1453. A 60-year-old woman complains of unbearable pain in her right hypochondrium. a history of acute
pancreatitis. Temperature-38.20 C. objectively: zhovtyannost sclera.
There are no symptoms of peritoneal irritation. Positive symptoms of Ortner, Hubergritz-Skulpsky. Urine diastasis - 320
g / hour. What is the most likely diagnosis?
100% Chronic pancreatitis
0% Acute cholangitis
0% Chronic cholecystitis
0% Acute cholecystitis
0% Pancreatic cancer

1454. a 37-year-old woman complains of suffocation, constricting pain behind the sternum.
I had the flu a week ago. Objectively: acrocyanosis, heart rate-98 / min, AT-90/75 mm Hg, PDR-26/min. The borders
of the heart are shifted to the left and right by 3 cm. Heart sounds are muffled, there is a protodiastolic gallop rhythm above the apex, and
a systolic murmur. Hb - 100 g / l, WSE-25 mm / year. What is the most likely diagnosis?
100% Infectious and allergic myocarditis
0% Dilated cardiomyopathy
0% Exudative pericarditis
0% Myocardial dystrophy
0% IHD, angina pectoris

1455. a 36-year-old woman complains of pain in the epigastric region that occurs after eating,
nausea, belching, and loosening of bowel movements. The disease gradually progresses for about 2 years.
Objectively: the skin is pale and dry, the tongue is covered, moist, with teeth imprints on the edges.
During palpation of the abdomen-common pain in the epigastric region. What is the most
informative research method to use?
100% Fibrogastroscopy with gastric mucosa biopsy
0% Detailed clinical blood test

https://translate.yandex.com/en/doc 321/540
22:20 ,27.6.2023 �� C : ; 5 B
334

Downloaded from the site - online testing step

0% X-ray examination of the stomach and intestines


0% Fractional study of gastric secretion
0% Computed tomography of the abdominal cavity

1456. A 63-year-old female patient was operated on with a large multi-node euthyroid goiter
. With technical difficulties, subtotal resection of both lobes
of the thyroid gland was forced. On the 4th day after the operation, there were convulsions of the muscles of the face and upper
extremities, abdominal pain. Positive symptoms of Khvostek and Trousseau. What is most likely
caused by this condition of the patient?
100% Parathyroid gland failure
0% Postoperative hypothyroidism
0% Thyrotoxic crisis
0% Reverse nerve damage
0% Tracheomalacia

1457. a 70-year-old patient injured his eye with a tree branch 3 days ago. Complains of pain, lacrimation,
decreased visual acuity in the right eye. Objectively: the eye slit is narrowed, photophobia, mixed
injection. Corneal opacity with indistinct contours, progressive margin and
epithelial defect. Corneal sensitivity is normal. Hypopion. Chronic dacryocystitis. What is the most
likely diagnosis?
100% Creeping corneal ulcer
0% Acute conjunctivitis
0% Viral keratitis
0% Acute iridocyclitis
0% Penetrating wound of the cornea of the right eye

1458. A 48-year-old patient spent the second day in the intensive care unit due to acute
anteroposteretinous myocardial infarction. During the examination, he suddenly "snored". There
was a single tonic contraction of skeletal muscles; pupils dilated. There is no pulse on a.carotis
. What are the main tactics?
100% Electrical defibrillation
0% Triple reception for Safar
0% ECG recording
0% Administration of epinephrine with atropine intracardiac
0% Precardial stroke

1459. A 31-year-old woman has been complaining for 3 years of pain and swelling of the radial-carpal
and lumbar-phalangeal joints, morning stiffness for up to 1.5 hours. Two weeks ago, there was pain,
swelling and redness of the knee joints, an increase in body temperature to 37.50 C. during
the examination of internal organs, no pathological changes were found. The patient was diagnosed
with rheumatoid arthritis. What changes on radiographs of joints are most likely?
100% Narrowing of the joint gap, Usura
0% Narrowing of the articular fissure, subchondral osteosclerosis
0% Cysts in the subchondral bone
0% Multiple marginal osteophytes

335

https://translate.yandex.com/en/doc 322/540
22:20 ,27.6.2023 �� C : ; 5 B

Downloaded from the site - online testing step

0% Epiphysis osteolysis

1460. A 51-year-old woman has been experiencing dull, intermittent acute pain in the right
hypochondrium associated with eating fatty foods, bitterness in the mouth in the morning, constipation, and flatulence for 2 years.
Objectively: excessive nutrition, t0-36.90 S, tongue overlaid at the root, abdomen moderately
swollen, painful at the point of projection of the gallbladder. What is the most appropriate
study for making a diagnosis?
100% Ultrasound examination
0% Duodenal probing
0% Cholecystography
0% Duodenoscopy
0% Liver scan

1461. a victim with a closed craniocerebral injury, unconscious, was taken to the department 8 hours after the accident
. Objectively: anisocoria, wound in the parietal region-3. 0x1. 0 cm,
neck muscle rigidity, Ps-58/min, tense. Convulsive syndrome. What is the most important
indicator for immediate surgical intervention?
100% Intracranial bleeding
0% Anisocoria
0% Fainting
0% Presence of wounds
0% Increased seizures

1462. in the population structure of the N-region, the proportion of people aged 0-14 years was
31%, and the proportion of people aged 50 years and older was 20%. Which population composition best
describes this demographic situation?
100% Progressive composition of the population
0% Regressive composition of the population
0% Stationary population composition
0% Population migration
0% Population emigration

1463.a 13-year-old girl went to the school doctor. Moderate


spotting from the genital tract first appeared 2 days ago. You will develop secondary sexual characteristics. What is the most
likely cause of spotting?
100% Menarche
0% Juvenile bleeding
0% Hemophilia
0% Endometrial cancer
0% Werlhof's disease

1464. A pregnant woman was registered in a antenatal clinic at 11 weeks ' gestation and
was monitored during the entire period during the normal course of pregnancy. What
document must be issued by the doctor of a pregnant woman for hospitalization in the maternity hospital?

336

Downloaded from the site - online testing step

https://translate.yandex.com/en/doc 323/540
22:20 ,27.6.2023 �� C : ; 5 B

100% Exchange card


0% Referral for hospitalization
0% Individual card for pregnant women
0% Sick leave form
0% Help from the sanitary and epidemiological station

1465.as a result of an accident, the victim received multiple fractures of the limbs and pelvic bones. In
the anamnesis: hemophilia A. objectively: hematomas appear on the damaged areas. The condition
is getting worse. AT-90/50 mm Hg. what is the most appropriate combination of infusion agents for
the treatment of a patient after the use of polyglucin and saline solutions?
100% Cryoprecipitate, red blood cell mass
0% Fresh frozen plasma, albumin
0% Red blood cell mass, freshly frozen plasma
0% Cryoprecipitate, glucose
0% Red blood cell mass

1466.during the survey of hygienic conditions of training at a technical university, it became necessary
to assess the visual mode of students studying from 9.00 to 15.00. What
is the most informative indicator of natural light?
100% Natural light ratio
0% World Coefficient
0% Depth of foundation of the study room
0% Room insolation time
0% Availability of compatible (Top-side) lighting

1467. a 70-year-old man has coronary heart disease. The mood is noticeably low,
anxious. Against the background of prolonged insomnia, there were fears, unwillingness to live, thoughts
of suicide. He sits for a long time without changing his position, does not answer immediately, quietly, in a monotone
voice. The expression on his face is one of suffering, pain, and fear. What is the leading psychopathological
syndrome?
100% Depressive syndrome
0% Paranoid syndrome
0% Asthenic syndrome
0% Phobic syndrome
0% Obsessive-compulsive disorder

1468. A 20-year-old man sat down abruptly on his right leg while playing football, turning to the left,
and there was acute pain in the right knee joint. The victim was taken to the
emergency room. Objectively: the volume of movement in the right knee joint is 100-1500,
there is no lateral mobility. What is the most likely diagnosis?
100% Internal meniscus injury
0% Anterior cruciate ligament injury
0% Patellar subluxation, possibly habitual
0% Damage to the internal lateral ligament

337

Downloaded from the site - online testing step

0% Strained hemarthrosis of the knee joint

https://translate.yandex.com/en/doc 324/540
22:20 ,27.6.2023 �� C : ; 5 B

1469. Blood pressure and age were studied in 200 hypertensive patients
. What statistical value should I use to measure the strength of the relationship between
these attributes?
100% Correlation coefficient
0% Student's Coefficient
0% Coefficient of variation
0% Representativeness Error
0% Sigma deviation

1470. A 24-year-old woman complained of long-term fever and night sweats. Over
the past three months, I have lost 7 kg of weight. Had promiscuous sexual relations. Objectively: enlargement of all
groups of lymph nodes, hepatolienal syndrome. In the blood: leuc.- 2, 2 * 109/l. what
disease can be suspected?
100% hiv infection
0% Lymphogranulomatosis
0% Tuberculosis
0% Infectious mononucleosis
0% Chroniosepsis

1471. in a patient with rheumatism, a diastolic tremor of the chest wall ("cat
purr") is determined, an increased I tone at the apex, diastolic noise from the press-capital amplification,
the tone of opening the mitral valve flaps, and the accent of the II tone above the pulmonary artery. What
is the patient's heart defect?
100% Stenosis of the left atrioventricular opening
0% Aortic valve insufficiency
0% Pulmonary artery stenosis
0% Mitral valve insufficiency
0% Open Ductus arteriosus

1472. a 3-year-old child developed a cough and runny nose. Two other family members had the same
symptoms. On the third day after the onset of the disease, the cough increased, became dry,
obsessive; the temperature rose to 37.80 C. objectively: breathing with the participation of auxiliary
muscles; with percussion-a pulmonary sound with a boxy tinge on both sides. Breathing is hard,
exhalation is prolonged, scattered medium-and large-bubble wet and sometimes dry wheezing.
What is the most likely diagnosis?
100% Acute obstructive bronchitis
0% Acute bronchiolitis
0% Stenotic laryngotracheitis
0% Bronchial asthma
0% Bilateral bronchopneumonia

1473. a 13-year-old boy complains of a dry cough and shortness of breath. He has been ill for a year. Attacks

338

Downloaded from the site - online testing step

short-term suffocation-1-2 times a month. Objectively: the child is restless, pale skin, nasolabial triangle cyanosis
, expiratory dyspnea. PDR-48 / min. Percussion: above the lungs
sound with a boxy tinge; auscultation - breathing is weakened, dry wheezing on both

https://translate.yandex.com/en/doc 325/540
22:20 ,27.6.2023 �� C : ; 5 B
sides. The volume of forced exhalation is 80% of the required volume. Which of the drugs is most appropriate
100% to the
to prescribe Salbutamol
patient?
0% Prednisone
0% Euphyllinum
0% Suprastin
0% Indomethacin

1474. A 31-year-old patient has been suffering from systemic scleroderma for 14 years. She was repeatedly
treated in hospitals. Complains of periodic dull pain in the heart, palpitations,
shortness of breath, headache, swelling of the eyelids, weight loss, pain and deformity of the joints of the extremities.
What organ damage worsens the prognosis of the disease?
100% Kidneys
0% Heart
0% Lungs
0% Gastrointestinal tract
0% Skin and joints

1475. in a 40-year-old woman, at the 40th week of the 6th pathological pregnancy (threat of failure, gestosis and type
II half), a boy was born in asphyxia from 3 deliveries. The child's condition is severe, weight 2 kg,
signs of immaturity, hydrocephalus syndrome. The skin is pale, with a jaundiced tinge, acrocyanosis.
Heart sounds are muffled, rough systolic murmur at all points of auscultation. Belly enlarged,
liver + 3 cm. Urine is saturated, feces are light. The optometrist revealed chorioretinitis. What is the most
likely diagnosis?
100% Congenital toxoplasmosis
0% Hemolytic disease of the newborn
0% Sepsis
0% Congenital heart disease
0% Congenital hepatitis

1476. A 13-year-old girl complains of an increase in body temperature to febrile figures


within a month, joint pain, and periodic skin rashes. During the examination
, a persistent increase in SSE, LE-cells was found in the blood. What is the most likely diagnosis?
100% Systemic lupus erythematosus
0% Juvenile rheumatoid arthritis
0% Systemic scleroderma
0% Acute lymphoblastic leukemia
0% Rheumatism

1477. a boy in the maternity hospital was diagnosed with a congenital heart defect (
ventricular membrane defect). At the age of 2 months, there were complaints of shortness of breath. Objectively:
PDR up to 60 / min, tachycardia up to 170 / min, liver 3 cm below the edge of the costal arch. What medications

339

Downloaded from the site - online testing step

do you need to assign a priority to your child?


100% Cardiac Glycosides
0% Nonsteroidal anti-inflammatory drugs
0% Potassium preparations
0% b-blockers

https://translate.yandex.com/en/doc 326/540
22:20 ,27.6.2023 �� C : ; 5 B
0% Glucocorticoids

1478.A 50 - year-old woman suffering from chronic pyelonephritis was


prescribed a combination of antibacterial agents-gentamicin (80 mg 3 times a day) and
Biseptol (960 mg 2 times a day). What are the consequences of prescribing such
a combination of antibiotics?
100% Acute renal failure
0% Ilomerulosclerosis
0% Chronic renal failure
0% The combination of antibiotics is optimal and quite safe
0% Acute adrenal insufficiency

1479. A 60-year-old patient had a short-term decrease in strength in the left


extremities for a month. Later in the morning after sleep, persistent weakness in the limbs appeared. Objectively:
AT140 / 90 mm Hg, conscious, central paresis of the VII and XII pairs of cranial nerves on the left,
central hemiparesis and hemihyperesthesia on the same side. What are the drugs of choice for differential
treatment of this patient?
100% Anticoagulants
0% Iemostatics
0% Antihypertensive drugs
0% Diuretics
0% Corticosteroids

1480. a 52-year-old patient complains of attacks of compressive chest pain radiating to the
left arm, which occur rarely, with significant physical exertion. He has been ill for 1 year.
Objectively: the size of the heart is extended to the left, the tones are muted. Ps-76 / min, rhythmic, blood pressure-155/80
mm Hg on the ECG: deviation of the electrical axis of the heart to the left, other indicators without features.
What additional research is needed to confirm the diagnosis?
100% Bicycle ergometric test
0% Echocardioscopy
0% Determination of blood lipoproteins
0% General blood test
0% Blood transaminases

1481. A 70-year-old patient complains of weakness, dizziness, short periods


of fainting, and pain in the heart. Objectively: heart rate-40 / min, rhythmic tones, I-th tone is muffled,
periodically significantly amplified. AT - 180/90 mm Hg. what is the most likely cause
of hemodynamic disorders?
100% AV block of the third degree
0% AV block of the first degree

340

Downloaded from the site - online testing step

0% Bradysystolic form of atrial fibrillation


0% Sinus bradycardia
0% Complete blockage of the left bundle branch of Gis

1482. a 65-year-old patient complains of suffocation, coughing with


foaming pink sputum, a feeling of lack of air, and fear of death. Objectively: the body is clear, the skin is pale,
acrocyanosis, cold sticky sweat. Respiration is hard, in the lower-posterior parts on both sides-

https://translate.yandex.com/en/doc 327/540
22:20 ,27.6.2023 �� C : ; 5 B
moist small - and medium-puffed wheezes. PDR-40 / min. The heart tones are sharply muted. At
the top of the heart-the rhythm of a gallop. What is the most likely diagnosis?
100% Pulmonary edema
0% Asthmatic status
0% Croup pneumonia
0% !infarction-pneumonia
0% Pulmonary embolism

1483. a 32-year-old patient who was treated in a hospital for an acute abscess of the right lung
suddenly developed heavy breathing, cyanosis,and pain in the right side of the chest after coughing.
What complication is most likely to occur in the patient?
100% Pyopneumothorax
0% !infarction-pneumonia
0% !nfarction of the myocardium
0% Esophageal perforation
0% Exudative pleurisy

1484.during a fluorographic examination of a 45-year-old man, for the first


time, small foci of low intensity with indistinct contours were found on the top of the right lung.
The patient's state of health is not disturbed. Smokes for many years. Objectively: there is a percutaneous-pulmonary
sound above the lungs, vesicular respiration, and wheezing is not audible. The blood test is unchanged. What is the most
likely diagnosis?
100% Focal tuberculosis of the lungs
0% Peripheral lung cancer
0% Eosinophilic pneumonia
0% Bronchopneumonia
0% Disseminated pulmonary tuberculosis

1485. an 18-year-old patient was admitted to the hematology department with complaints of headache,
general weakness, lack of appetite, fever up to 390C,
and swelling of the neck. Objectively: t0-380C, skin and mucous membranes are very pale, packages
of lymph nodes on both sides of the neck up to 1 cm in size, non-painful. Liver + 1 cm,not ill,
spleen +0.5 cm. In the blood: HB-98 g / l, ep.- 2,9 * 1012/ l, lake.- 32 * 109/l, p-0%, c-28%, m-2%,
l39%, blast-31%, pet-31%, blood clot.-120 * 109 / l, WSE-36mm / hour. What is the patient's form of leukemia?
100% Acute lymphoblastic leukemia
0% Acute myeloblastic leukemia
0% Chronic lymphocytic leukemia
0% Chronic myeloid leukemia

341

Downloaded from the site - online testing step

0% Undifferentiated leukemia

1486. the mother of a 5-year-old girl complained of nighttime urinary incontinence, night
terrors, disturbed sleep, and a lag in body weight gain. Objectively: a girl of low
nutrition, intellectually well developed, can read, explains life situations in an adult
way. The skin is pale, the liver is enlarged. My mother has cholelithiasis. What type
of diathesis is most likely to occur in a child?
100% Neuro-arthritic
0% Urate
0% Exudative-catarrhal
https://translate.yandex.com/en/doc 328/540
22:20 ,27.6.2023 �� C : ; 5 B

0% Allergic
0% Lymphatic-hypoplastic

1487. a 35-year-old patient who has been undergoing hemodialysis for 3 recent years due to chronic glomerulonephritis
has experienced heart failure, hypotension, increasing
weakness, and shortness of breath. On the ECG: bradycardia, atrioventricular block of the first century, high pointed
teeth of the pelvis - a gross violation of drinking and dietary regimes. What biochemical changes are
the most likely cause of the above clinical picture?
100% Hyperkalemia
0% Hyperhydration
0% Hypokalemia
0% Hypernatremia
0% Hypocalcemia

1488. A 60-year-old patient complains of pain in the interphalangeal joints of the hands, which increases
during work. Objectively: the distal and proximal joints of the fingers II-IV are defigured, with nodes
of Ge-burden, Bouchard, painful, with limited mobility. X-ray of joints:
narrowed joint spaces, marginal osteophytes, subchondral sclerosis. What is the most likely diagnosis?
100% Deforming osteoarthritis, nodular form
0% Reiter's disease
0% Ankylosing spondylitis
0% Rheumatic arthritis
0% Psoriatic arthritis

1489. a 42-year-old man who has been suffering from a duodenal ulcer for 20 years
has a constant feeling of heaviness in the stomach after eating, belching rotten, vomiting
food consumed the day before, and losing weight. Objectively: the condition is relatively satisfactory, tissue turgor is reduced.
The abdomen during palpation is soft, there are no symptoms of irritation of the peritoneum, "splashing noise"in
the epigastrium. Emptying once every 3 days. What complication is most consistent with the patient's condition and
the specified clinic?
100% Ulcerative stenosis of the output part of the stomach
0% Covered ulcer perforation
0% Stomach cancer
0% Ulcer penetration
0% Chronic pancreatitis

342

Downloaded from the site - online testing step

1490. a 46-year-old patient complains of itchy skin, sweating, especially at night,


fever up to 38.60 C. objectively: on the skin of the chest there are traces of uncovering, supraclavicular lymph
nodes the size of a pigeon's egg, they are not soldered to the skin. What is the most
appropriate research method?
100% Puncture of an enlarged lymph node
0% General blood test
0% Chest X-ray survey
0% Immunogram
0% Total protein and protein fractions

1491. A 32-year-old patient lives in an area endemic with echinococcosis. For the last 6 months, he
has been worried about pain in the right hypochondrium, fever. Suspected echinococcal
https://translate.yandex.com/en/doc 329/540
22:20 ,27.6.2023 �� C : ; 5 B
liver damage. What kind of research is most informative in this case?
100% Ultrasound examination
0% Overview radiography of the abdominal cavity
0% Biochemical laboratory research
0% Angiography
0% Liver scan

1492. a 30-year-old child who is giving birth for the first time has intense attempts with an interval of 1-2
minutes, lasting 50 seconds. Eruption of the fetal head occurs. The crotch, which
is 4 cm high, turned pale. What should be done in this situation?
100% Episiotomy
0% Perineal protection
0% Perineotomy
0% Vacuum extraction of the fetus
0% Stick to wait-and-see tactics

1493. A 28-year-old patient was hospitalized on the 9th day of illness with complaints of
fever up to 390C, headache, general weakness, constipation, and sleep disorders. Objectively: on
the skin of the abdomen-single roseoli, Ps-78 / min, the liver is enlarged by 2 cm. What is the most
likely diagnosis?
100% Typhoid fever
0% Typhus fever
0% Sepsis
0% Brucellosis
0% Leptospirosis

1494. A 50-year-old patient was admitted with complaints of blood in the urine. Pain and
urinary disorders were not observed; hematuria continues for 3 days. Objectively:
the kidneys are not palpable, the suprapubic area is without features, the external genitalia are without
pathology. During rectal examination, the prostate gland is not enlarged.
Cystoscopy revealed no changes. What disease should you think about first?
100% Kidney cancer
0% Tuberculosis of the bladder

343

Downloaded from the site - online testing step

0% Varicocele
0% Kidney dystopia
0% Necrotizing papillitis

1495. A 48-year-old patient came to the doctor, whose blood test showed that the Vaserman reaction and
sedimentary reactions were positive three times. A blood test for RIBT also gave a positive
result. When examining the patient, no clinical manifestations of syphilis were detected either from the skin or from the
internal organs and nervous system. What is the most likely diagnosis?
100% Latent early syphilis
0% Primary seropositive syphilis
0% Secondary recurrent syphilis
0% Tertiary syphilis
0% Latent recurrent syphilis

https://translate.yandex.com/en/doc 330/540
22:20 ,27.6.2023 �� C : ; 5 B

1496. A 35-year-old female patient developed areas of redness and swelling on the back of her hands after severe nervous tension
, followed by the formation of small
inflammatory nodules, blisters, and then erosions with significant serous fluid release. The process
is accompanied by severe itching. What is the most likely diagnosis?
100% Real eczema
0% Allergic dermatitis
0% Microbial eczema
0% Simple contact dermatitis
0% Toxicoderma

1497. a patient complains of soreness and redness of the skin on the right calf. Objectively:
body temperature 38.50 c, enlarged and painful inguinal lymph nodes on the right side.
The skin of the limb is edematous, hyperemic, with elements of a rash in the form of bubbles with a dark
liquid inside; palpation is painful. There is a clear line between redness and healthy
skin. What is the most likely diagnosis?
100% Erysipelas, hemorrhagic form
0% Anthrax, skin form
0% Herpes infection
0% Chickenpox
0% Phlegmon of the lower leg

1498. an employee of a pig farm, on the background of full health, developed a sharp chill, an increase
in body temperature to 39.90 C, intense headache, nausea. The next day
, there was pain in the muscles of the lower extremities, nosebleeds. Objectively: on the 3rd day of the disease
, the condition is severe, the face is hyperemized, scleritis, scleral subictericity. Liver + 3 cm. Daily
diuresis of 700 ml. What is the most likely diagnosis?
100% Leptospirosis
0% Viral hepatitis
0% Yersiniosis
0% Flu
0% Hemorrhagic fever with renal syndrome

344

Downloaded from the site - online testing step

1499. a 1.5-year-old child became acutely ill: chills, an increase in body temperature to 40.10 c, then
a rapid decrease to 36.20 c, a profuse hemorrhagic rash on the skin, purple-cyanotic spots.
The limbs are cold, and the facial features are pointed. Diagnosis: meningococcal infection, lightning
-fast form, infectious and toxic shock. What antibacterial drug should be used at
the pre-spital stage?
100% Levomycetin succinate, soluble
0% Penicillin
0% Lincomycin
0% Ientamycin
0% Sulfamonomethoxine

1500. A 41-year-old patient after lifting a heavy object noted sharp pain in the right side
of the chest, shortness of breath sharply increased. The patient's condition is severe: cyanosis of the lips and mucous membranes,
PDR-28/min, Ps-122 / min. Over the right half of the chest percussion-timpanitis,
auscultation-sharply weakened breathing; accent of the second tone over the pulmonary artery. AT-80/40
mmHg. what is the main emergency measure at the pre-spital stage?

https://translate.yandex.com/en/doc 331/540
22:20 ,27.6.2023 �� C : ; 5 B
100% Aspiration of air from the pleural cavity
0% Introduction of epinephrine
0% Introduction of eufillin
0% Call a cardiology team
0% Inhaling oxygen

1501. a 38-year-old woman has episodes of paroxysmal increase in blood pressure


up to 240/120 mm Hg, accompanied by nausea, vomiting, tachycardia, and increased
sweating. In the blood during the attack-hyperglycemia. After the attack, profuse urination.
Sonography of the kidneys revealed an additional formation adjacent to the upper pole of the right
kidney, which may relate to the adrenal glands-ka. What laboratory test can help clarify
the diagnosis?
100% Determination of urinary excretion of catecholamines and vanillylmigdalic acid
0% Determination of insulin and C-peptide in the blood
0% Determination of glomerular filtration rate by endogenous creatinine
0% Determination of thyroxine and thyroid-stimulating hormone in the blood
0% Determination of blood renin levels

1502. A 28-year-old patient is undergoing treatment for pulmonary tuberculosis. Complaints of


sudden acute pain in the right side of the chest, shortness of breath. Percussion over
the right lung is detected box sound, breathing is not listened to.
Radiographically: the right lung is anchored to the root by 1/2 volume, the heart and
mediastinal organs are shifted to the left. What complication did the patient have?
100% Spontaneous pneumothorax
0% Dry pleurisy
0% Pleural empyema
0% Lung infarction
0% Exudative pleurisy

345

Downloaded from the site - online testing step

1503. A 48-year-old farmer was admitted to the clinic with complaints of headache,nausea,
wet cough, difficulty breathing, poor vision, sweating, and drooling. He worked in
a team that treated the garden with organophosphate pesticides. In the blood: Er.- 4,1 * 1012/ l, HW-136 g / l,
CP-0.9, leuc. -13.0* 109/l, WSE-17 mm / year. The diagnosis was established: acute intoxication
with organophosphate pesticides. What is the most important diagnostic criterion for this
pathology?
100% Reduced cholinesterase levels
0% Reticulocytosis
0% Leukocytosis
0% Anemia
0% Thrombocytopenia

1504. a 74-year-old patient complains of pain and bloating, nausea. He suffers from coronary
heart disease, post-infarction and atherosclerotic cardiosclerosis. Objectively: the condition is severe,
the abdomen is swollen, the abdominal wall is poorly involved in the act of breathing. During laparoscopy:
there is a small amount of cloudy effusion in the abdominal cavity, one of the loops of the small intestine
is dark-blue in color. What is the most likely diagnosis?
100% Mesenteric vascular thrombosis
0% Inversion of the bowel
https://translate.yandex.com/en/doc 332/540
22:20 ,27.6.2023 �� C : ; 5 B

0% Acute intestinal obstruction


0% Ischemic abdominal syndrome
0% Erysipelas

1505. a 10-year-old child in the oligoanuric stage of acute renal


failure developed tingling sensations in the oral mucosa,tongue,
numbness of the extremities, decreased reflexes, respiratory disorders, and arrhythmia. What causes these
symptoms?
100% Hyperkalemia
0% Hyponatremia
0% Hyperazotemia
0% Acidosis
0% Alkalosis

1506. a 30-year-old woman has repeated labor lasting 14 hours. Fetal heartbeat is muffled,
arrhythmic, 100 / min. Weight study: the opening of the cervix is complete, the fetal head is in
the plane of exit from the small pelvis. Sagittal suture in straight size, small crown near the womb.
What are the further tactics of labor management?
100% Using the original obstetric forceps
0% Oxytocin stimulation of labor
0% Caesarean section
0% Skin-main forceps by Ivanov
0% Use of abdominal obstetric forceps

1507. an employee who was involved in the elimination of a fire in a room where 2 kg
of metallic mercury was stored was taken to the clinic with complaints of emotional incontinence and palpitations,

346

Downloaded from the site - online testing step

sweating, trembling of the body, pain in the heart. The condition worsened during the day.
Objectively: the skin is pale and moist. The patient has a state of depression, persistent red dermographism,
erethism, unstable blood pressure. Which of the drugs is the antidote in this
case?
100% Unitiol
0% Atropine Sulfate
0% Calcium Tetacin
0% Amyl Nitrite
0% Dipiroxam

1508. a 63-year-old woman complains of unmotivated weakness, rapid fatigue, decreased


appetite, and a desire for meat products. Two weeks ago, a stomach bleed. Ob'ektivno:
t0-37,50 S, PDR-20/xv, Ps-96/xv, AT-110/75 mm Hg during palpation in the epigastrium-pain and
muscle tension. In the blood: HB-82 g / l, SSE-35 mm / h. Which study
will most reliably establish the diagnosis?
100% Cytological
0% X-ray examination
0% Endoscopic treatment
0% Examination of stomach contents
0% Coprological research

https://translate.yandex.com/en/doc 333/540
22:20 ,27.6.2023 �� C : ; 5 B

1509. a 32-year-old patient developed general weakness, sweating,


fever, and cough after hypothermia. Objectively: the general condition is moderate,
the skin is moist, lip cyanosis. the chest is symmetrically involved in breathing, on the right side under
the scapula there is a slight blunting of percussion sound, weakened vesicular respiration, wet
sonorous wheezing. Above the rest of the surface of the lungs - hard breathing, scattered dry
wheezes. Heart tones are enhanced, accent tone II over the pulmonary artery. In the blood: lake.- 8,2 * 109/ l,
wSe - 21 mm/hour. What is the most likely diagnosis?
100% Right-sided focal pneumonia
0% Chronic non-obstructive bronchitis in the acute stage
0% Acute bronchitis
0% Lung cancer
0% Tuberculosis

1510. a 42-year-old woman suffers from micro-nodular cryptogenic cirrhosis of the liver. During
the last week, the condition has worsened: convulsions, confusion,
jaundice has increased. What kind of research can explain the reason for the deterioration of the condition?
100% Determination of serum ammonia
0% Determination of cholesterol esters
0% Determination of a-fetoprotein content
0% Definitions of AlAT and AsAT
0% Determination of alkaline phosphatase level

1511. the mother of a newborn child suffers from chronic pyelonephritis


and had acute respiratory viral infections before giving birth. Delivery on time, long waterless period. On the 2nd day, the child appeared

347

Downloaded from the site - online testing step

erythema-a tous rash, later-blisters about 1 cm in size, filled


with serous contents; when opened, they form erosions. Nikolsky's symptom is positive.
The child is sluggish, the body temperature is subfebrile. What is the most likely diagnosis?
100% Newborn's pemphigus
0% Vesiculopustulosis
0% Pseudofurunculosis
0% Sepsis
0% Ritter's dermatitis

1512. the child has a sudden cessation of breathing,blueness of the skin,


disappearance of the pulse on the main vessels, narrowing of the pupils. What are the priority measures?
100% Ventilator, closed heart massage
0% Intravenous administration of eufillin
0% Gastric lavage
0% Oxygen Therapy
0% Intracardiac administration of epinephrine

1513. a 44-year-old patient suffering from hypertension developed headache,


palpitations, and fear. Objectively: Ps-100 / min, BP-200/100 mm Hg. the left border of cardiac dullness
is shifted to the left by 1.5 cm, respiration is vesicular. ECG: sinus tachycardia, signs of
left ventricular hypertrophy. What medication should be prescribed for emergency care?
100% Obzidan

https://translate.yandex.com/en/doc 334/540
22:20 ,27.6.2023 �� C : ; 5 B
0% Dibazol
0% Reserpine
0% Magnesium Sulfate
0% Furosemide

1514. a patient was admitted to the clinic with a diagnosis of anaerobic gas gangrene of the thigh. What
antiseptic must be used during the treatment of instruments after
dressing?
100% 6% hydrogen peroxide solution
0% Furacilin solution
0% 5% iodine solution
0% 0.02% decamethoxin solution
0% 0.1% potassium permanganate solution

1515. in the technical school, where mostly girls and boys aged 14 years study, it is necessary to make
a routine preventive examination for tuberculosis. Which of the following methods should be
used?
100% Mantoux test with 2 TO
0% Fluorography
0% Sputum testing at the Office
0% General blood test
0% Physical examination

348

Downloaded from the site - online testing step

1516. A 32-year-old woman has been bothered by pain in the lumbar region, sub-febrility,
and frequent urination for 5 months. In the urine: moderate proteinuria, white blood cells in the entire field of vision, bacteriuria. In
the blood: leukocytosis, increased SSE. What is the most likely diagnosis?
100% Chronic pyelonephritis
0% Acute glomerulonephritis
0% Chronic glomerulonephritis
0% Acute pyelonephritis
0% Urolithiasis

1517. a 30-year-old woman developed tonic convulsions after an emotional shock, which were replaced
by clonic ones; they were accompanied by fainting, foaming at the mouth,
and involuntary urination. No focal symptoms were detected. AT - 120/60 mm Hg. what is the most
likely diagnosis?
100% Epilepsy
0% Neurasthenia
0% Eclampsia
0% Syncope
0% Stroke

1518. a 2-year-old boy was hospitalized with complaints of weight loss, unstable
bowel movements, and anorexia that appeared after the introduction of semolina porridge into the diet (from 5 months).
The child is adynamic, lethargic, the skin is pale, dry, the subcutaneous fat layer is significantly thinned.
The abdomen is swollen, tense, during percussion in the upper part of the abdomen tympanitis, splashing noise

https://translate.yandex.com/en/doc 335/540
22:20 ,27.6.2023 �� C : ; 5 B
,frothy, light-colored, smelly bowel movements. Coprocytogram: neutral fat-a lot.
100% Celiac disease
What is the most likely cause of the disease?
0% Cystic fibrosis
0% Intestinal dysbiosis
0% Chronic enteritis
0% Disaccharidase deficiency

1519. a child with chronic carditis, NC IIA, who is being treated with digoxin,
developed increased bradycardia, nausea, vomiting, dizziness, and sleep disorders. On
an Ecgextrasystole, PQ is 0.18. what is the most likely cause of this condition?
100% Overdose or intolerance of cardiac glycosides
0% Pulmonary edema
0% Grade I atrioventricular block
0% Acute intestinal infection
0% Hypokalemia

1520. when examining the corpse of a person who died as a result of hanging, it was found: cadaveric spots
disappear under pressure, are restored after 50 seconds, rigor mortis is moderately expressed
only in the masticatory muscles, in the muscles of the neck and fingers of the hand. Body temperature 31.0 aboutC. determine
the time of death?
100% 6-7 hours

349

Downloaded from the site - online testing step

0% 1-2 hours
0% 16-24 hours
0% 8-10 hours
0% 10-18 hours

1521.during a hernia repair, a 12-year-old patient was found to have a testicle in the hernial
sac. What is the patient's hernia?
100% Congenital oblique inguinal hernia
0% Acquired oblique inguinal hernia
0% Direct inguinal hernia
0% Femoral hernia
0% Zatulnaya hernia

1522. an ambulance team delivered a patient with a diagnosis of acute respiratory viral infection to the hospital.
He became acutely ill with an increase in body temperature to 39.9 aboutC. complaints of headache in the frontotemporal
areas, pain in the eyeballs, aching all over the body, nasal congestion, scratching in the throat, dry
cough. I had a nosebleed twice at home. What type of acute respiratory viral infection does the patient have?
100% Flu
0% Adenovirus infection
0% Parainfluenza
0% MS infection
0% Enterovirus infection

1523. A 25-year-old repeat mother was admitted in the first stage of labor with
moderate contractions. This pregnancy is the third. In the anamnesis - Cesarean section for a clinically narrow

https://translate.yandex.com/en/doc 336/540
22:20 ,27.6.2023 �� C : ; 5 B
pelvis and 1 artificial abortion. Suddenly, Rodilli developed severe abdominal pain and weakness;
her atrrial pressure dropped to 90/50 mm Hg. Moderate spotting appeared from the vagina.
The fetal heartbeat is not monitored. Labor activity stopped. Most likely
diagnosis:
100% Uterine rupture
0% Premature detachment of the normally located placenta
0% Amniotic fluid embolism
0% Inferior vena cava syndrome
0% Placenta previa

1524. A 52-year-old patient was admitted with complaints of shortness of breath during moderate physical
exertion, cough with severe sputum discharge. He has been ill for 12 years.
Objectively: PDR-26 / min. Percussion-pulmonary tone with a boxy tinge; weakened
vesicular respiration with prolonged exhalation, scattered dry wheezing. Previously treated only
with teopec, or eufillin IV. appoint basic therapy after the elimination of exacerbation:
100% Atrovent
0% Alupent
0% Ingacort
0% Timed Out

350

Downloaded from the site - online testing step

0% Aminophylline

1525. A survey radiography of the patient's abdominal cavity reveals several


areas of hemispherical illumination located above clear horizontal
levels. What causes such an X-ray picture?
100% Intestinal obstruction
0% Perforating ulcer
0% Flatulence
0% Colon cancer
0% Price's Disease

1526. as a result of actual drowning in seawater, a patient was pronounced clinically


dead. Specify the correct sequence of resuscitation measures:
100% Restoration of patency of the upper respiratory tract, mechanical ventilation, indirect heart massage
0% Indirect heart massage, mechanical ventilation, restoration of patency of the upper respiratory tract
0% Mechanical ventilation, indirect heart massage, restoration of patency of the upper respiratory tract
0% Indirect heart massage, mechanical ventilation
0% -

1527. A 28-year-old patient, who 2 days ago injured the distal phalanx of the right index finger
with a needle, complains of sharp pain of a pulsating nature and swelling in the area of this phalanx,
a pronounced violation of finger mobility. Because of the pain, I couldn't sleep last night. Objectively:
the index finger is slightly bent, its distal phalanx is hyperemic, significantly increased in
volume. The probe clearly defines the point of maximum pain. Body temperature 36.90 C.
What should be the local treatment?
Autopsy and drainage of the purulent process of the finger after anesthesia by
100%
Oberst Mukashevich
0% Opening and drainage of the abscess after local infiltrative anesthesia

https://translate.yandex.com/en/doc 337/540
22:20 ,27.6.2023 �� C : ; 5 B
0% Pricking the affected area with antibiotics with novocaine
0% Application of semi-alcohol compresses
0% Resection of the distal phalanx of the finger

1528. a patient with a cerebral vessel aneurysm suddenly developed


severe headache, short-term fainting, and vomiting during physical exertion. Objectively: the patient is excited,
wants to run somewhere, Ps-62/min, rhythmic, blood pressure-140/90 mm Hg, t0-37.50 s. stiffness
of the occipital muscles, Kernig's symptom are detected. There are no focal neurological symptoms.
Determine the preliminary diagnosis:
100% Subarachnoid hemorrhage
0% Ischemic stroke
0% Meningitis
0% Hypertensive crisis
0% Brain hemorrhage

1529. a 39-year-old man suddenly developed a rash on the skin of his torso, consisting of small
red rashes that itch during the day. Elements of the rash are scattered over the skin
351

Downloaded from the site - online testing step

disordered and mostly focused. The patient associates the appearance of a rash with a visit
to a sports and wellness center and sauna a few days before. Similar symptoms
are observed in a friend who was then with him. What is the most likely diagnosis?
100% Lousiness
0% Eczema
0% Contact dermatitis
0% Allergic dermatitis
0% Scabies

1530. the patient had acute chills, headache, vomiting, fever


up to 38.50 C. In the late afternoon, stiffness of the muscles of the back of the head, Kernig's symptom, appeared. On the mucous
membranes of the lips and
nose, herpetic blisters are noted. No focal neurological symptoms were detected.
What100%
is the mostMeningococcal
likely diagnosis?
meningitis
0% Subarachnoid hemorrhage
0% Herpetic encephalitis
0% Brain abscess
0% Brain hemorrhage

1531. a patient in a somatic hospital with a high fever developed


psychomotor agitation: he tried to run around the department; he believed that water was flowing on the walls, and
rats and cockroaches were running on the floor. He claimed that he was in a hostel, "late-vav" friends.
After sedation, he fell asleep. The next morning I remembered about the transferred state. Identify
the psychopathological syndrome:
100% Delusional syndrome
0% Oneiroid syndrome
0% Twilight disorder of consciousness
0% Hallucinatory-paranoid syndrome
0% Manic syndrome

https://translate.yandex.com/en/doc 338/540
22:20 ,27.6.2023 �� C : ; 5 B
1532. a 33-year-old patient with stopped recurrent ulcerative
bleeding was admitted to the hospital. During the examination, emaciated, pale. In the blood: HB-77 g / l, Ht-0.25. due to
the presence of anemia, a single-group A(II)transfusion was attempted twiceRh+ of blood. Both times
the transfusion was stopped due to the development of an anaphylactic reaction. Transfusion of what
transfusion medium is desirable in this case?
100% Washed red blood cells
0% Fresh Blood Citrate
0% Red blood cell mass (native)
0% Red blood cell mixture
0% Red blood cell mass, which is poor in white blood cells and platelets

1533. a 19-year-old boy who received a closed abdominal injury was admitted to a surgical hospital
. During the operation, multiple ruptures of the spleen and small intestine were found. Blood pressure gradually decreases.
There was a need for blood transfusion. Who can
determine the victim's blood type and Rh status?

352

Downloaded from the site - online testing step

100% Doctor of any specialty


0% Laboratory assistant
0% Surgeon
0% Traumatologist
0% Doctor-anesthesiologist

1534. A 20-year-old patient complains of amenorrhea. Objectively: hirsutism, obesity with a predominant
distribution of adipose tissue on the face, neck, upper body. On the face - acne vulgaris,
on the skin - stretch marks. AT - 170/100 mm Hg. mental development and intelligence are not
impaired. Gynecological status: hairiness on the external genitalia is moderate, acute
hypoplasia of the vagina and uterus. What is the most likely diagnosis?
100% Itsenko-Cushing Syndrome
0% Shereshevsky-Turner syndrome
0% Stein-Leventhal syndrome
0% Sheehan's syndrome
0% Pehrantz-Babinski-Fröhlich syndrome

1535. A 22-year-old woman applied to a antenatal clinic for a pregnancy of 1112 weeks.
During the examination, Wasserman was found to have a positive reaction. The dermatologist diagnosed secondary
latent syphilis. What are the tactics of managing this pregnancy?
100% Artificial termination of pregnancy after a course of antisyphilitic therapy
0% Artificial termination of pregnancy after diagnosis
0% Artificial termination of pregnancy after normalizing the Wasserman reaction
0% Prolongation of pregnancy after a course of antisyphilitic therapy
0% Antisyphilitic treatment three times during pregnancy

1536. An 18-year-old patient was admitted to the hospital on the 7th day of illness with complaints of headache,
general weakness, fever, and sore throat. Objectively: an increase in all
groups of lymph nodes up to 1-3 cm in diameter. During palpation: the lymph nodes are dense, elastic,
low-bolus, not soldered together. On the enlarged tonsils-purulent layers. Liver + 3
cm. In the blood: leukocytosis, relative lymphomonocytosis, virocytes -15%. What is the most likely
diagnosis?

https://translate.yandex.com/en/doc 339/540
22:20 ,27.6.2023 �� C : ; 5 B
100% Infectious mononucleosis
0% Adenovirus infection
0% Tonsillitis
0% Diphtheria
0% Acute lymphocytic leukemia

1537. a 72-year-old woman was hospitalized with a nosebleed. Over the past 6 years, blood pressure
has increased to 180/100 mmr.st. objectively: the skin is pale, the heart tones are quite sonorous, the accent is II
tones above the aorta, a systolic murmur is heard. AT-150/80 mm Hg in the blood: HB-92 g / l, ep. - 2, 7 *
1012/l in the urine: pit. weight -1022, lake.- 3-7 v p/ s, ep.- 0-2 v p/S. what is the most likely cause
of hypertension?
100% Aortic atherosclerosis
0% Hypertension

353

Downloaded from the site - online testing step

0% Chronic pyelonephritis
0% Chronic glomerulonephritis
0% Aortic coarctation

1538. enterprises with harmful working conditions carry out a set of measures to
reduce the level of morbidity. To achieve a higher effect, the shop doctor identifies
a group of long-term and frequent patients. What type of morbidity determines the group of long
-term and frequent patients?
100% Morbidity with temporary disability
0% Incidence of the most important non-epidemic diseases
0% General morbidity rate
0% Acute infectious diseases
0% Hospitalized morbidity rate

1539. a 5-year-old girl was accidentally locked in a dark room for several minutes. When the door
was opened, the child stood motionless in the middle of the room, his eyes fixed on one point,
his face frozen in a grimace of horror, did not react to stimuli. After 40 minutes, the girl's condition
changed to crying. The next day, I didn't remember anything about yesterday's incident. What is the most
likely mechanism of this reaction?
100% Psychogenic
0% Endogenous
0% Exogenous-organic
0% Endogenous-organic
0% Conditioned reflex mode

1540. A 42-year-old patient was admitted 3 hours after the injury with severe subcutaneous
emphysema of the upper half of the trunk, shortness of breath, and tachycardia 120/min. Radiologically
, no pneumothorax was detected, significant expansion of the mediastinum in both directions. What kind of emergency
care?
100% Drainage of the anterior mediastinum
0% Puncture of the pleural cavity
0% Drainage of the pleural cavity
0% Thoracoscopy
0% Thoracotomy

https://translate.yandex.com/en/doc 340/540
22:20 ,27.6.2023 �� C : ; 5 B

1541. a 60-year-old patient who is in a hospital for left-sided exudative


pleurisy has rapid accumulation of exudate after each evacuation. What
disease does this dynamic correspond to?
100% Blastomatous process
0% Dressler's syndrome
0% Systemic lupus erythematosus
0% Post-pneumonic pleurisy
0% Traumatic pleurisy

354

Downloaded from the site - online testing step

1542. A 54-year-old patient who suffered from hepatitis B virus 4 years ago and abuses alcohol
has been experiencing heartburn and burning pain behind the sternum for the last two months. In the morning, after eating and
lifting weights, vomiting of fresh dark blood appeared. objectively: the skin
is pale, moist, Ps-92 / min, blood pressure-90/60 mm Hg. sclera is jaundiced, the abdomen is enlarged
due to ascites, hepatosplenomegaly. What is the most likely cause of the bleeding?:
100% Esophageal varicose vein rupture
0% Malory-Weiss syndrome
0% Budd-Chiari syndrome
0% Achalasia of the esophagus
0% Peptic ulcer of the duodenum 12

1543. an 8-year-old boy who had hepatitis B a year ago has been complaining of
fatigue, sleep disorders, loss of appetite, and nausea for the past two months, especially in the morning.
The skin is not jaundiced, the liver and spleen are palpable 1 cm below the costal
edges, and are painless. ALT activity is 2.2 mmol/l. How can this condition be assessed?
100% Development of chronic hepatitis
0% Relapse of viral hepatitis B
0% Biliary dyskinesia
0% Residual effects of hepatitis B transmission
0% Development of cirrhosis of the liver

1544. a 2.5-month-old child developed muscle hypotension, sweating, and baldness at the back of the head. Along with
massage and therapeutic gymnastics a vitamin is prescribed specify its dosage and
frequency of administration:
100% 3000 IU daily
0% 500 IU daily
0% 1000 IU daily
0% 500 IU every other day
0% 1000 IU every other day

1545. a 25-year-old patient complains of an increase in body temperature to 370C, pain in the lower abdomen
and vaginal discharge. Three days ago, at 10 weeks of pregnancy, an artificial abortion was performed.
Objectively: the cervix is clean, the uterus is slightly enlarged, painful. Uterine appendages are not
detected. The arches are deep and painless. Vaginal discharge is purulent-bloody. What
is the most likely diagnosis?
100% Post-abortion metroendometritis
0% Hematometer
https://translate.yandex.com/en/doc 341/540
22:20 ,27.6.2023 �� C : ; 5 B

0% Pelvioperitonitis
0% Uterine perforation after abortion
0% Parametrite

1546. A 67-year-old patient complains of shortness of breath, chest pain, and general weakness. He's been ill for 5
months. Objectively: t0 - 37, 30C, Ps-96/min. Above the right lung, the vocal tremor is not
detected, the percussion sound is dull, and breathing is not listened to. In sputum-admixtures
of blood diffusely mixed with mucus. What is the most likely diagnosis?

355

Downloaded from the site - online testing step

100% Lung cancer


0% Bolshegnishchevaya pneumonia
0% Bronchiectasis
0% Focal tuberculosis of the lungs
0% Exudative pleurisy

1547. for the population living in a radiation-contaminated area,


it is recommended to include pectins in the diet in order to remove radionuclides from the body. Which of
the following foods are the main source of pectins?
100% Vegetables and fruits
0% Bread
0% Milk
0% Meat
0% Pasta

1548. a 25-year-old pregnant woman at 34 weeks was hospitalized in a maternity hospital in a serious
condition. Complains of headache, visual impairment, nausea. Objectively: continuous edema,
AT170 / 130 mm Hg. suddenly, the pregnant woman had fibrillar twitching of the facial muscles, tonic
and clonic convulsions, stopped breathing. After 1.5 min. his breathing was restored,
and blood-stained foam came out of his mouth. in the urine: protein-3.5 g / l. what is the most likely diagnosis?
100% Eclampsia
0% Epilepsy
0% Brain hemorrhage
0% Brain edema
0% Stomach ulcer

1549. The Carpathian region is characterized by consistently high (more than 80%)
atmospheric humidity. During the cold period of the year, with moderately low air temperatures,
the population of this region experiences severe cold. Which heat transfer path does this increase
?
100% Convection
0% Radiation
0% Evaporation
0% Conducting system
0% Radiation exposure

1550. a 51-year-old patient has a 2.5-month delay in menstruation. Complains of significant


spotting from the vagina for 15 days, increased irritability, sleep disorders. In the anamnesis-
https://translate.yandex.com/en/doc 342/540
22:20 ,27.6.2023 �� C : ; 5 B
violation of menstrual function during the year. On ultrasound: the uterus meets the age standards,
appendages without features, endometrial thickness of 14 mm. What are the doctor's tactics?
100% Diagnostic curettage of the uterine cavity walls
0% Conservative treatment of bleeding
0% Hysterectomy
0% Superpichval amputation of the uterus without superstructures

356

Downloaded from the site - online testing step

0% Testing for TORCH infection

1551. an 18-year-old female patient complains of soreness and engorgement of the mammary glands, headache,
irritability, and swelling of the lower extremities. These symptoms are disturbing from the beginning of menarche,
they appear 3-4 days before the start of the next menstruation. During the gynecological examination
, no pathology was detected. What is the most likely diagnosis?
100% Premenstrual syndrome
0% Neurasthenia
0% Kidney diseases
0% Mastopathy
0% Diseases of the cardiovascular system

1552. a 22-year-old patient complains of aching pain in the right swollen area for a week,
morning sickness, and a change in taste. Delayed menstruation for 3 weeks. Objectively: AT-110/70 mm Hg,
Ps-78 / min, t0-37.00 s. Bimanually: the uterus is slightly enlarged, soft, mobile, painless. Palpation
of the appendages: on the right side there is a painful formation of 3x4 cm, tightly elastic consistency, moderately
mobile. What is the most likely diagnosis?
100% Progressive tubal pregnancy
0% Aborted tubal pregnancy
0% Right ovarian cyst
0% Uterine pregnancy
0% Acute appendicitis

1553. a 29-year-old female patient complains for 2 months of pain in the left half of the chest,
cough, shortness of breath, body temperature rising to 39.60 C. objectively: the left half
of the chest lags behind in the act of breathing, weakening of vesicular respiration and shortening
of percussion sound on the left. Radiologically, a rounded shadow is detected in the lower lobe of the left
lung. What is the preliminary diagnosis?
100% Lung abscess
0% Purulent pleurisy
0% Lung cancer
0% Pleural empyema
0% Chronic pneumonia

1554. a woman in labor on the fifth day after an emergency delivery experienced an increase
in body temperature to 38.80 C, abdominal pain, and general weakness. Objectively: AT-120/80 mm Hg, Ps-100 / min.
The mammary glands are moderately rough. The abdomen is soft and participates in the act of breathing. The bottom of the uterus is
three
fingers below the navel. During vaginal examination: the cervix passes a finger, the uterus
of a soft consistency, enlarged up to 13 weeks of pregnancy, is painful. Discharge from the genital tract
is purulent-serous,
100% with an unpleasant
Postpartum smell. What is the most likely diagnosis?
Purulent metroendometritis
https://translate.yandex.com/en/doc 343/540
22:20 ,27.6.2023 �� C : ; 5 B

0% Postpartum endometritis
0% Subinvolution of the uterus
0% Lactostasis
0% Septic shock

357

Downloaded from the site - online testing step

1555. the patient is 15 years behind in physical development, periodically yellowing


of the skin is noted. Objectively: spleen 16x12x10 cm, cholecystolithiasis, skin ulcer of the lower third
of the left shin. In the blood: Er.- 3,0 * 1012/ l, Hb-90 g/l, CP-1.0, microspherocytosis, reticulocytosis.
Total serum bilirubin - 56 mmol/l, indirect - 38 mmol/l. which treatment method
is most appropriate?
100% Splenectomy
0% Spleen transplant
0% Portocaval anastomosis
0% Omentosplenopexy
0% Omentohepatopexy

1556. in rodilla 38 years old III birth, 5 artificial abortions in the anamnesis. 5 minutes after
the birth of the child, bleeding from the genital tract began (350 ml). The woman
's condition is satisfactory, Ps-92 / min, BP-100/60 mm Hg. there are no signs of placental separation.
Manual separation was performed: the placenta exfoliated with some difficulty in some areas. What
is the most likely diagnosis?
100% Partial tight attachment of the placenta
0% Pinching of the separated placenta
0% Placental increment
0% Complete tight attachment of the placenta
0% Hypotonic bleeding

1557. A 17-year-old female patient was admitted to the gynaecological department with complaints of profuse
spotting from the genital tract and cramping pain in the lower abdomen. My last period was 10
weeks ago. Objectively: AT-100/60 mm Hg, Ps-90 / min. During vaginal examination: the external
peephole of the cervix passes the finger. The uterus is enlarged up to 6 weeks of pregnancy, painless.
Appendages are not defined, arches are free. What is the most likely diagnosis?
100% Incomplete abortion
0% Abortion that started
0% Complete abortion
0% Dead pregnancy
0% The threat of abortion

1558. a 3-year-old child has difficulty breathing, refusal to eat, t0-400C, nasal
voice. Within 10 days there were rhinitis symptoms. Objectively: the soft palate is symmetrical,
hyperemia and explosion of the spherical shape of the posterior pharyngeal wall, more on the right. Along
the anterior edge of the sternoclavicular-mascoid muscle, along the angle of the lower jaw,
swelling, painful during palpation. Breathing is difficult, moderate stridor. What is the most
likely diagnosis?
100% Pharyngeal abscess
0% Right-sided paratonsillitis
0% Right-sided paratonsillar abscess
0% Right-sided adenophlegmon of the neck

https://translate.yandex.com/en/doc 344/540
22:20 ,27.6.2023 �� C : ; 5 B
0% Oropharyngeal diphtheria

358

Downloaded from the site - online testing step

1559.in a patient in a state of clinical death, artificial ventilation of the lungs is performed using the "mouth
-to-mouth"method and indirect heart massage. The doctor drew attention to the fact that air does not enter the
patient's respiratory tract, and his head and torso are in the same plane. What is the reason
for the ineffectiveness of artificial respiration in this case?
100% Sinking of the tongue
0% Small volume of inhaled air
0% Performing an indirect heart massage
0% Lack of a probe in the stomach
0% Small mouth of the patient

1560. a 42-year-old man complained of a feeling of heaviness in the lower abdomen, frequent and
painful urination, and blood staining of urine at the end of urination. Got sick after
hypothermia. Digital rectal examination of the prostate gland does not
detect any changes. In the urine: protein-0.99 g / l, white blood cells-30-40 v / s, red blood cells - for all p/ s, a large
number of bacteria. What disease should I think about?
100% Acute cystitis
0% Acute prostatitis
0% Tuberculosis of the bladder
0% Bladder cancer
0% Prostate cancer

1561. a worker with a metal fragment penetrating his right eye applied to the factory's medical
center. Half of the fragment is located in the anterior chamber of the eye, the other half is outside.
The doctor removed the fragment, applied a binocular bandage, administered tetanus serum and
immediately sent the patient to the hospital. What mistake did the doctor make when providing first
aid?
100% Removed a piece of metal
0% I put on a binocular patch
0% Introduced tetanus serum
0% Mydriatic did not drip
0% Did not refer the patient for an X-ray of the eye socket

1562. In a city with a population of 400,000 people, 5,600 deaths were registered per year,
including 3,300 cases from diseases of the circulatory system and 730 from neoplasms. What indicator
can characterize the mortality rate from diseases of the circulatory system in the city?
100% Intensive indicator
0% Extensive indicator
0% Relative intensity indicator
0% Visibility indicator
0% Ratio indicator

1563. a 12-week-old pregnant woman developed lower abdominal pain and minor
spotting for no apparent reason. During vaginal examination: the cervix is formed, the outer
eye is closed. The body of the uterus is enlarged accordingly during pregnancy. Specify
the most likely diagnosis:
https://translate.yandex.com/en/doc 345/540
22:20 ,27.6.2023 �� C : ; 5 B

359

Downloaded from the site - online testing step

100% Threatening abortion


0% Abortion that started
0% Abortion is in progress
0% Incomplete abortion
0% Undeveloped pregnancy

1564.during the medical examination, the driver of the port crane complained of dizziness,
nausea, pressure on the eardrums, tremor, choking, coughing. Work is performed at
a height associated with nervous and emotional stress. In addition, workers are affected by vibration (general
and local), noise, infrasound, and a microclimate that heats up in summer and cools down in winter.
What factor can be associated with the worker's complaints?
100% Infrasound
0% Noise
0% Vibration
0% Labor intensity
0% Working at height

1565. the analysis of inpatient care for the population of the district is carried out. Which of the above
indicators should be used when deciding whether to reduce the available
number of hospital beds?
100% Average annual bed employment
0% Lethality rate
0% Average length of hospital stay
0% Average duration of treatment
0% Bed turnover

1566. a 53-year-old patient complains of aching pain in the lower abdomen, a significant increase in it over
the past 5 months, weight loss, weakness. Objectively: the cervix is clean, the uterus is not enlarged,
painless, sedentary. On both sides, tumors with a size of 10x13 cm, with an uneven
surface, dense consistency, and immobile are determined. During percussion of the abdomen, a fluctuation is detected.
What is the most likely diagnosis?
100% Ovarian cancer
0% Uterine fibromyoma
0% Tubo-ovarian tumor
0% The wandering kidney
0% Endometriosis

1567. A 33-year-old woman has had two ectopic pregnancy surgeries in the past,
and both fallopian tubes have been removed. I asked about the possibility of having a child. What should
be recommended for pregnancy in this case?
100% In Vitro fertilization
0% Male sperm insemination
0% Surrogate motherhood
0% Artificial insemination with donor sperm

360

https://translate.yandex.com/en/doc 346/540
22:20 ,27.6.2023 �� C : ; 5 B

Downloaded from the site - online testing step

0% Ovulation induction

1568. a 42-year-old patient complains of aching pain in the lower back, more on the right side, sometimes
an increase in body temperature to subfebrile, the main one is 6 degrees. 10 years ago, during pregnancy
, the patient had an attack of pain in the right side of the lower back, which was accompanied by a sharp
increase in temperature. 5 years ago, there was an increase in blood pressure to 200/110 mm Hg in
the urine: protein-0.99 g / l, leuc.- 10-15 v / s, ep. - 2-4 v / s, hyaline cylinders-1-2 v / s, creatinine-102
mmol/l. what is the most likely diagnosis?
100% Chronic pyelonephritis
0% Chronic glomerulonephritis
0% Tuberculosis of the kidneys
0% Amyloidosis of the kidneys
0% Hypertension

1569. a 30-week pregnant woman suffered an eclampsia attack at home. When hospitalized in
the maternity ward AT-150/100 mm Hg, the estimated fetal weight is 1500 g. pasty
of the face and lower legs. Protein in the urine - 0.66 g / l. the birth canal is not ready for delivery.
Intensive complex therapy was started. What is the correct management strategy for this case?
100% Give birth by caesarean section
0% During treatment, prolong pregnancy for 1-2 weeks
0% During treatment, prolong pregnancy for 3-4 weeks
Start labor stimulation by intravenous administration of oxytocin or
0%
prostaglandins
0% Treat preeclampsia and give birth conservatively

1570. what are the mandatory actions of a doctor when a child or teenager is detected on an X-ray for the first
time intra-thoracic lymph nodes with lateral calcification?
100% Apply for a dispensary registration and prescribe treatment
0% Prescribe chemoprophylaxis with one drug
0% Prescribe chemoprophylaxis with two drugs
0% Take it for medical observation
0% -

1571.a sample of milk was sent to the laboratory for research. The following
data were established: color-whitish, smell - without features, taste-characteristic of milk, density-1.038,
key content - 350 Turner, fat content-3.2%. What is the degree of milk quality?
100% Milk of poor quality
0% Milk of good quality
0% Reduced quality milk
0% Adulterated milk
0% Milk is conditionally suitable

1572. A 74-year-old patient has been suffering from benign prostatic hyperplasia for the last 5
years. 4 days ago, after drinking alcohol, there was an acute urinary retention. At
the prehospital stage, the bladder was catheterized twice a day with a metal catheter.

361

https://translate.yandex.com/en/doc 347/540
22:20 ,27.6.2023 �� C : ; 5 B

Downloaded from the site - online testing step

examination: the appendage of the right testicle is enlarged, compacted, painful, there is purulent discharge from the urethra.
What type of emergency care should I choose?
100% Trocar or open epicystomy
0% Transurethral resection or prostatectomy
0% Installing a permanent urethral catheter
0% Microwave Prostate Thermotherapy
0% Installing an intraprostatic stent

1573. a 30-year-old man complains of severe pain, redness of the skin, swelling in the
ankle joint, fever up to 390C. got sick suddenly. In the past
, there were similar attacks lasting up to 5-6 days without any residual changes in the joint. The skin above the joint
is hyperemic without clear contours and an infiltrative shaft on the periphery. What is the most
likely diagnosis?
100% Gout
0% Infectious arthritis
0% Rheumatoid arthritis
0% Erysipelas
0% Osteoarthritis

1574. a boy was admitted to the surgical department on the first day after birth with
foamy discharge from the nose and mouth, attacks of cyanosis. Radiologically: blind end
of the esophagus at the level of the II thoracic vertebra, gas bubble of the stomach under the left dome of the diaphragm.
What is the most likely diagnosis?
100% Esophageal atresia, tracheo-esophageal fistula
0% Total esophageal atresia
0% Paraesophageal hernia of the stravochoid orifice of the diaphragm
0% Esophageal atresia without fistulas
0% Bronchoesophageal fistula

1575. a 15-year-old boy has been suffering from atopic dermatitis since childhood with an allergy to
crustacean meat. The last 3 months, after buying aquarium fish, rhinitis,
conjunctivitis, itchy nose appeared. What level of immunological indicator should be determined in the
patient?
100% IgE
0% IgJ
0% IgM
0% IgA
0% Circulating immune complexes

1576.when studying the incidence of pulmonary tuberculosis, data were obtained on


the socio-economic living conditions and bad habits of patients. Which of these methods allows us to assess
the degree of influence of these factors on the incidence of tuberculosis?
100% Calculating the correlation coefficient
0% Calculating the compliance indicator
0% Calculating the regression coefficient

362

Downloaded from the site - online testing step

https://translate.yandex.com/en/doc 348/540
22:20 ,27.6.2023 �� C : ; 5 B
0% Calculating standardized metrics
0% Calculating the probability coefficient

1577. within 2 hours after the birth of the fetus, the state of the child is good: the uterus is dense, spherical,
its bottom is at the level of the navel, there is no bleeding. The clamp applied to the umbilical cord segment
is at a preliminary level, with a deep breath and when pressing the edge of the palm
over the symphysis, the umbilical cord is pulled into the vagina. There is no bloody discharge from the genital tract.
What will be the doctor's next strategy?
100% Perform manual placental separation
0% Apply the Abuladze method
0% Apply the Kredit-Lazarevich method
0% Perform curettage of the uterine cavity
0% Intravenous administration of oxytocin

1578. after surgery for a perforated stomach ulcer, end-stage


diffuse peritonitis, and endotoxic shock, the patient is given artificial
ventilation with 60% oxygen inhalation in the postoperative period. Blood gases: Pa02-70-78 mmHg, hypoxemia does not
decrease, CVT-150-180 mmHg, AT-90/60 mmHg (against the background of high doses
of dopamine). On the R-gram diffuse infiltration of the lungs. What is the cause of persistent arterial
hypoxemia?
100% Respiratory distress syndrome
0% Bilateral pneumonia
0% Pneumothorax
0% Mendelssohn's syndrome
0% Pulmonary edema

1579. a 56-year-old patient complains of shortness of breath with difficulty exhaling, sputum
discharge in the morning for 22 years. Smokes 1 pack of cigarettes a day from the age of 18. During
the FDD test, the reversibility of the obstruction is 10%. What medications should
be prescribed at the beginning of treatment?
100% Inhaled anticholinergics (atrovent)
0% Antibiotics
0% Inhaled sympathomimetics (berotek)
0% Membrane stabilizers (Intal, tailed)
0% Inhaled glucocorticosteroids

1580. a 66-year-old patient complains of palpitations, general weakness, paresthesia, decreased


sensitivity in the legs, impaired taste and smell. In the blood: Er.- 2,1 * 1012/ l, HB-84 g / l, CP-1,2,
macrocytosis, Poly-segmentation of neutrophil nuclei. What plays a leading role in the pathogenesis of this
condition?
100% Violation of DNA synthesis, megaloblastic type of hematopoiesis
0% Impaired hemoglobin synthesis due to chronic iron deficiency
0% Dyseritropoiesis due to impaired iron utilization at the bone marrow level
0% Increased destruction of red blood cells
0% Bone marrow damage caused by toxic factors

363

Downloaded from the site - online testing step

1581. a 1-year-old child has t0=39.80 S, liquid bowel movements. For 5 days
, I had a fever with periods of temperature rising to high values. Haggard, pale.

https://translate.yandex.com/en/doc 349/540
22:20 ,27.6.2023 �� C : ; 5 B
Tissue turgor is reduced. Heart tones are muted. The abdomen is swollen, rumbling in the navel area, soreness
during palpation. Liver + 3.5 cm. Bowel movements are liquid, 10 times a day, green in color, with
mucus. What is the most likely diagnosis?
100% Salmonellosis
0% Dysentery
0% Viral hepatitis
0% Escherichiosis
0% Rotavirus infection

1582. based on the results of hospital-pedagogical observation of a physical education lesson in the 9th
grade, a physiological curve is constructed, characterized by a gradual increase
in the pulse rate in the introductory part, an increase in the pulse rate by 80% during the main part, the curve has
a 4-toothed form. How can I evaluate the organization of a physical education lesson?
100% Long intervals between exercises
0% The lesson is built correctly
0% Physical activity is insufficient
0% Excessive physical activity
0% Physical activity is sufficient

1583. a 16-year-old patient. Over the past year, my behavior has gradually changed: I began
to withdraw, lost interest in my friends and studies. He became indifferent to his family, was
unreasonably rude, talked to himself or laughed. The question is answered formally
correctly, with little justification. He considers himself quite healthy, but somewhat tired, says that
he is considering writing a book "the projection of humanity on the plane of the Universe", carries a notebook
with its pages filled with many identical crosses. What is the most likely diagnosis?
100% Schizophrenia
0% Depressive disorder
0% Schizoid personality disorder
0% Autistic Personality Disorder
0% Pick's disease

1584. the employee was temporarily disabled for 16 days due to illness.
He was treated on an outpatient basis. The doctor-curator issued a disability certificate first for 5 days and
extended it to 10 days. Who can continue the disability certificate for this
employee?
100% Doctor-curator together with the head of the department
0% Medical Advisory Commission
0% Curator doctor with the permission of the chief Physician
0% Deputy Chief Physician for Disability Assessment
0% Department Head

1585. A 13-year-old patient was treated in a dermatological hospital for


acute atopic dermatitis. He was discharged in clinical remission. What are your recommendations

364

Downloaded from the site - online testing step

should the doctor give the patient skin care to prevent new
flare-ups?
100% Use of indifferent creams for protective purposes
0% Frequent skin washing with detergents

https://translate.yandex.com/en/doc 350/540
22:20 ,27.6.2023 �� C : ; 5 B
0% Systematic use of topical corticosteroids
0% Systematic treatment of the skin with disinfecting agents
0% Prevention of sun exposure to the skin

1586. an 18-year-old patient developed jaundice. After the examination


, Gilbert-Meilengracht syndrome was diagnosed. What is the mechanism of jaundice development in the patient?
100% Lack of glucuronyltransferase
0% Lack of glutamate transferase
0% Intravascular hemolysis
0% Obturation of the common bile duct
0% Tyrosine sulfotransferase deficiency

1587. a full-term child suffered from ante-and intra-natal hypoxia and was born in asphyxia
(Apgar score 2-5 points). After birth, the child
's agitation progresses,vomiting, nystagmus, convulsions, strabismus, spontaneous Moreau and
Babinsky reflexes are noted. What localization of intracranial hemorrhage is most likely?
100% Subarachnoid hemorrhage
0% Minor brain tissue hemorrhages
0% Subdural hemorrhage
0% Periventricular hemorrhage
0% Hemorrhages in the ventricles of the brain

1588. A 25-year-old patient was hospitalized due to pericarditis effusion. Complains of


shortness of breath, weakness, tightness in the chest. She thinks she got sick 2 weeks ago, after catching a cold. Which
of the features identified during the examination will indicate a chronic nature
of pericardial damage?
100% Signs of pericardial calcification
0% Reduction of electrocardiogram voltage
0% Muted heart tones
0% Signs of stagnation in a large circle
0% Significant expansion of the heart tissue, weakening of pulsation

1589. in a 37-year-old pregnant woman giving birth for the first time, labor continues for 10 hours.
Contractions for 20-25 seconds after 6-7 minutes. The position of the fetus is longitudinal, the head
lies pressed against the entrance to the small pelvis. During the vaginal examination: the cervix is up to 1
cm long, passes 2 transverse fingers. There is no fertile bubble. What is the most likely diagnosis?
100% Primary weakness of labor activity
0% Secondary weakness of labor activity
0% Normal labor activity
0% Discoordination of labor activity

365

Downloaded from the site - online testing step

0% Pathological preliminary period

1590. a 64-year-old patient 2 hours ago developed compressive pain behind the sternum radiating to the
left shoulder, pronounced weakness. Objectively: the skin is pale, cold sweat. Ps-108 beats / min., AT-70/50
mm Hg. heart tones are deaf. Respiration is vesicular. The belly is soft, painless.

https://translate.yandex.com/en/doc 351/540
22:20 ,27.6.2023 �� C : ; 5 B
Varicose veins on the left shin. ECG: sinus rhythm, heart rate-100/min, sharp rise
100%
of the Cardiogenic
ST segment above theshock
isoline in leads II, III, aVF. What pathology did the patient have?
0% Cardiac asthma
0% Pulmonary embolism
0% Dissecting aortic aneurysm
0% Cardiac tamponade

1591. at the reception, the patient complains of fever up to 38, 20C, the presence of edema
in the upper lip. On examination: the upper lip is sharply swollen, with a cone-shaped swelling in the center of the edema
. The skin and mucous membrane above it are dark red. Diagnosis:
upper lip furuncle. The surgeon performed an autopsy of the boil, treated the wound with a solution of hydrogen peroxide and
applied a bandage with a hypertonic solution. What treatment regimen should be recommended
to the patient?
100% Inpatient treatment with a general regimen
0% Outpatient treatment, followed by inpatient treatment
0% Outpatient treatment
0% Treatment is inpatient with a bed regime
0% -

1592. A 50-year-old patient who has been suffering from chronic obstructive
pulmonary disease (COPD) for a long time complains of a gradual increase in pain in the extremities, especially in the wrists,
and notes a loss of body weight (about 4 kg). Objectively: the skin of the wrists is warm and hyperemic.
Overview Rh-graph: periosteal thickening, possibly osteomyelitis. What is the doctor's next action?
100% Appointment of chest radiography
0% Appointment of a biopsy of both wrists
0% Administration of ciprofloxacin
0% Prescribing gold preparations
0% -

1593. a 50-year-old patient, a construction worker, retains elevated blood pressure even after prescribing
three antihypertensive medications. The patient does not take drugs that increase blood pressure
. Objectively: blood pressure is 160/95 mm Hg. the results of physical examination are normal,
blood serum electrolytes are not disturbed. What are the most appropriate next steps for your doctor?
100% Check how the patient is taking medications
0% Examine the patient for Cushing's disease
0% Examine the patient for aortic coarctation
0% Assign an angiography of the kidneys
0% Assign a cranial X-ray
1594. a 60-year-old woman complains of increased fatigue and shortness of breath. He smokes. Which

366

Downloaded from the site - online testing step

what would be the most specific evidence of congestive heart failure in this patient?
100% Protodiastolic gallop rhythm
0% Edema in the pit area
0% Congestive wheezing in the lungs
0% Weight gain
0% Increased shortness of breath

1595. A 50-year-old patient was found to have jaundice, muscle tension in the upper right
https://translate.yandex.com/en/doc 352/540
22:20 ,27.6.2023 �� C : ; 5 B
quadrant of the abdomen, vascular asterisks on the skin, and ascites. He doesn't take any medications.
Abusing alcohol. What is most likely to be detected during the examination of the patient?
100% Malory corpuscles in liver biopsy
0% Strained jugular veins
0% AlAT is much higher than AsAT
0% Rapid clinical recovery after alcohol withdrawal
0% Splenomegaly

1596. a 40-year-old patient with clinical signs of peptic ulcer disease in combination with severe
diarrhea. In the anamnesis-three duodenal ulcers, which often recur. One of
the ulcers is located close to the small intestine. Gastrin serum 200 pg / ml. What
would be the most informative study in this case?
100% Secretin Injection Test
0% Colonoscopy
0% Endoscopic retrograde cholangiography
0% CT scan of the abdominal cavity
0% Radiography of the OCP

1597. A 40-year-old man complains of weakness, weight loss, and abdominal pain. Objective:
diffuse hyperpigmentation of the skin, polyarthritis (joints of the wrists and hip joint),
palpable enlarged liver. Blood sugar is 9.25 mmol / L. What is the most likely diagnosis?
100% Hemochromatosis
0% Addison's Disease
0% Pancreatic carcinoma
0% Insulin-dependent diabetes mellitus
0% Chronic hepatitis

1598. a 30-year-old man has a thyroid nodule. The patient's father died of
thyroid cancer, and his brother had urolithiasis. In the blood: calcitonin 2000 pg / ml (N<100);
serum calcium and phosphorus in the normal range. What diagnostic tests should a
doctor perform before referring a patient to a surgeon?
100% Determination of the concentration of catechol-min in the urine
0% Liver scan
0% Calcium Infusion Test
Determination of the concentration of thyroid-stimulating hormone against the background of a suppressive dose
0%
of thyroxide

367

Downloaded from the site - online testing step

0% Diagnostic course of treatment with radioactive iodine

1599. a 55-year-old woman suffers from severe depression, polyuria, nocturia, and extreme thirst. A
history of radical mastectomy for breast cancer (one year ago). In the blood:
glucose - 5.5 mmol/ l, Na + -149 mmol/l; K+- 3.6 mmol/l; Ca2+-2.37 mmol/l (serum);
urea (serum)- 10.71 mmol/l. Urine osmosis is 150 mOsm / l. What is the most
likely diagnosis?
100% Diabetes insipidus
0% Psychogenic polydipsia
0% Renal glucosuria

https://translate.yandex.com/en/doc 353/540
22:20 ,27.6.2023 �� C : ; 5 B
0% Hypercalciuria
0% Syndrome of inappropriate anti-diuretic hormone secretion (SIADH)

1600. A patient with subfebrile fever and weight loss has a poor
excursion of the diaphragm, bluntness during percussion, and weakened breathing on the right side. The trachea
is deviated to the right. What is the most likely diagnosis?
100% Pleural effusion due to histoplasmosis
0% Pneumothorax
0% Atelectasis
0% Mediastinal tumor
0% Diaphragm keel

1601. A 30-year-old man complains of fever and sore throat for several days. On the day
of the visit to the doctor, the patient developed hoarseness, difficulty breathing and drooling.
Objectively: febrile fever and wheezing in the lungs on inhalation. What is the doctor's next strategy?
100% Hospitalization in the IT department with an otolaryngologist's consultation
0% Outpatient ampicillin treatment
0% Purpose of pharyngeal smear seeding
0% Appointment of chest radiography
0% Appointment of bronchoscopy

1602. A 40-year-old nurse was hospitalized for a high temperature (39.40 C). This
temperature persists for 2 weeks, the etiology of fever is not established. What
disease should not be considered during differential diagnosis and further
examination of the patient?
100% Respiratory infection
0% Latent bacterial infection
0% Lymphoma
0% Still's disease
0% Artificial hyperthermia

1603. A 60-year-old man was taken to the hospital for acute pancreatitis. Laboratory
parameters: Ht-42%, lake.- 14,0* 109/ l, calcium-2.1 mmol / l, urea-5 mg / dl, amylase-1000
U/l. laboratory tests reflecting liver function are within the norm. After 48

368

Downloaded from the site - online testing step

hours of infusion therapy and follow-up, which of the listed indicators should indicate an
unfavorable prognosis?
100% Calcium 1.8 mmol / l
0% ALT 0.72 mmol/(year*l)
0% Amylase 2000 U / l
0% Bilirubin 42 mmol / l
0% Glucose 12.0 mmol / l

1604. A 29-year-old woman was admitted to the hospital complaining of progressive severe pain in the lower
abdomen and vomiting during the last 2 days. Notes a 6-day delay in menstruation. Lives
a sexual life and does not use contraceptives. The temperature is 38, 30C.on examination
, the tension of the anterior abdominal wall in the lower parts is noted. During the vaginal

https://translate.yandex.com/en/doc 354/540
22:20 ,27.6.2023 �� C : ; 5 B
examination, leucorrhea from the cervical canal and uterine tension during
the biman study are noted. The area of the appendages is somewhat tense, but no seals are
palpable. What is the most appropriate study to establish a diagnosis?
100% Sowing from the cervical canal
0% Posterior arch puncture
0% Laparoscopy
0% Ultrasound of the pelvic organs
0% Determination of human choriotropic gonadotropin

1605. A 65-year-old man who is taking di-goxin for chronic heart failure
was taken to the hospital for a sudden spread of abdominal pain that occurred 2 hours
ago. On examination: the skin is moist, the temperature is normal. Ps-110 beats / min, arrhythmic.
AT120 / 70 mm Hg on palpation, tension of the anterior abdominal wall
is noted throughout the entire abdomen. There are no intestinal noises. The rectal ampoule is empty. Rh-graph
shows air-liquid levels in the intestinal area. ECG: atrial fibrillation without
signs of myocardial ischemia. In the blood: leuc. -17.0 * 109/l, arterial blood pH-7.33, amylase-110
U/l. the most likely diagnosis is:
100% Mesenteric vascular thrombosis
0% Intestinal obstruction due to a tumor
0% Diverticulosis
0% Intestinal ischemia
0% -

1606. An 8-month-old child was taken to the hospital complaining of difficulty breathing. The mother
of the child reported that 5 days ago she had a runny nose, a slight cough and an increase
in body temperature to 38.30 ° C. the fever subsided, but the cough remained. In the morning
, the child's breathing became faster and the skin was covered with spots. Objectively: the temperature is normal, but
general malaise persists, and wheezing sounds can be heard at a distance. BH-80 / min. Ps-180 bpm.
On percussion of lungs, a sound with a boxy tinge; on auscultation, various dry wheezes.
The most important step in the survey plan is:
100% Determination of arterial blood gases
0% Conducting a bronchoscopy
0% Nasopharyngeal seeding
0% Examination of cerebrospinal fluid

369

Downloaded from the site - online testing step

0% Detailed general blood test

1607. a 41-year-old man complains of coughing, fever up to 39.4°C and pain in


the left side of the chest for 3 days. There is no history of respiratory diseases
. Chest radiography shows infiltration in the lower lobe
of the left lung. Gram staining revealed exfoliated epithelial cells, single
neutrophils, and mixed gram-positive and gram-negative flora. What is the doctor's next strategy?
100% Administration of erythromycin
0% Hospitalization of a patient for a transbronchial biopsy
0% Study of bronchial flushing waters
Conducting a 3-time sputum test for the presence of acid
0%
-resistant microflora
0% Tomography of the lower lobe of the left lung

https://translate.yandex.com/en/doc 355/540
22:20 ,27.6.2023 �� C : ; 5 B
1608. a 12-year-old boy was taken to the hospital with complaints of pain in the lower right
quadrant of the abdomen. According to the parents, the pain occurred yesterday in the belyapupochny area and was accompanied
by double vomiting during the night. The temperature is normal. On examination, there
is tension in the anterior abdominal wall in the lower right quadrant of the abdomen.
I listen for intestinal noises. In the blood: lake.- 10,0 * 109/L. in the urine: lake.- 5-10 in the field of view. Please indicate when
you need to consult a surgeon, given this clinical picture?
100% Immediately
0% In case of an increase in body temperature
0% In case of symptoms of peritoneal irritation
0% In case of leukocytosis above 15.0 * 109 / l
0% No need

1609. A 74-year-old woman complains of burning, shingling pain in the upper right quadrant
of the abdomen, the epigastric region, which radiates to the back. On examination: there is no abnormal tension
of the anterior abdominal wall. During the ultrasound, the gallbladder is normal. Blood
amylase levels are normal. What is the most likely diagnosis?
100% Herpes zoster
0% Stone-free cholecystitis
0% Exacerbation of chronic pancreatitis
0% Cecal diverticulosis
0% Penetrating ulcer of the duodenum bulb 12

1610. a 64-year-old man went to the doctor with complaints of pain behind the sternum, which has been bothering
him for the last 2 months. The pain is short-lived, occurs several
times a day and lasts 10-15 minutes. During physical examinations, no pathological changes were
detected. ECG - within the age norm. What is the next step to
evaluate existing symptoms?
100% Veloergometry
0% Chest radiography
0% Echocardiography
0% FEGDS

370

Downloaded from the site - online testing step

0% Holter monitoring

1611. the mother of a 9-month-old child went to the doctor because "
there was blood in the child's urine". The child's general condition is satisfactory. Her temperature is normal and there was
no urethral injury. On macroscopic examination, the urine is pink, but on
microscopy, red blood cells are absent. The most likely explanation for this phenomenon is:
100% The presence of chromogens in the child's food
0% Hemoglobinuria
0% Lysis of red blood cells
0% Myoglobinuria
0% Porphyria

1612. a 2-year-old boy has had eye and bone swelling in the last
week. Objectively: AT-100/60 mm Hg, Ps-110/Xv, BH-28/xv. Abdominal enlargement with positive
wave symptom. Serum creatinine concentration-0.45 mmol / l, albumin-14 g/l and

https://translate.yandex.com/en/doc 356/540
22:20 ,27.6.2023 �� C : ; 5 B
cholesterol-11.8 mmol / l. in the Section 6 cells ++++, red blood cells are not detected what is the most
100% Nephrotic syndrome
likely diagnosis?
0% Iemolytic-uremic syndrome
0% Acute post-streptococcal glomerulonephritis
0% Nephrotic syndrome due to focal and segmental glomerulosclerosis
0% Schonlein-Yenoch disease with nephritis

1613. A 15-year-old boy suffered severe anaphylactic shock twice after bee stings. Which
of the following is the most effective method of prevention?
100% Desensitization with bee venom extract
0% Prescribing corticosteroids during the summer
0% Long-term preventive treatment with antihistamines
0% Restriction of outdoor use during the summer months
0% Protective cloth

1614. in a healthy 75-year-old woman leading a moderately active lifestyle, a preventive


examination revealed a serum concentration of total cholesterol at the level of 5.1 mmol / l and
HDL cholesterol-70 mg / dl. ECG without pathology. Which of these dietary recommendations
is most acceptable?
100% No dietary changes
0% Reducing cholesterol intake
0% Reduced saturated fat intake
0% Reduce your intake of simple carbohydrates
0% Increase your fiber intake

1615. a 33-year-old woman, 1 pregnancy and 1 birth in anamnesis, gave birth to a girl of 2460 g at the 38th
week of pregnancy (physiological delivery). Newborn hepa has tosplenomegaly, an open
ductus arteriosus, and cataracts. At the 8th week of pregnancy, the mother had

371

Downloaded from the site - online testing step

maculopapular rash, enlarged cervical lymph nodes, sore throat, and arthralgia that
spontaneously stopped after 1 week. In the subsequent prenatal period, there were no complications
. Which of these tests during pregnancy is most likely to
predict such abnormalities in the fetus?
100% Serial titers of rubella virus antibodies
0% Culture test for Herpes Smplex virus
0% Syphilis test
0% Amniocentesis for karyotype determination
0% Urinalysis for cytomegalovirus

1616. a 33-year-old man with no complaints of AT-166/112 mm Hg has a normal level of electrolytes in
the blood serum. To reduce the likelihood of developing what condition is antihypertensive therapy necessary
?
100% Stroke
0% Myocardial infarction
0% Congestive heart failure
0% Aortic aneurysm
0% Kidney failure

https://translate.yandex.com/en/doc 357/540
22:20 ,27.6.2023 �� C : ; 5 B

1617. a 10-year-old girl developed macrohematuria 14 days after having a sore throat. AT-170/100 mm
Hg, moderate swelling of the feet and shins. Urea serum nitrogen is 32 mmol / l. Which of
the following is the most likely cause of these symptoms?
100% Increased volume of intracardiac fluid
0% Aortic coarctation
0% Reduced production of endothelial relaxing factor
0% Increased catecholamine production
0% Increased aldosterone production

1618. A 32-year-old male homosexual has purulent discharge from the urethra.
Neb sseria gonorrhoeae sensitive to penicillin was isolated during seeding. A week after
stopping penicillin treatment, the patient has a recurrence of discharge. Seeding again
identifies penicillin-sensitive N. gonorrhoeae. Both the patient and his partner face to face. During
the examination, an anal fissure was found in the sexual partner; culture from the urethra of N. gonorrhoeae was not
found. Which of the following is the most likely cause of recurrent urethral infection?
100% Reinfication from a partner
0% Occurrence of bacterial resistance
0% Inadequate penicillin therapy
0% Concomitant infection with the herpes virus
0% -

1619. A 52-year-old man has progressive dyspnea and cough with purulent
sputum for 2 days. He has been smoking a pack of cigarettes a day for 30 years.
The temperature is 37, 20C. Breathing is weak, with occasional dry and wheezing wheezes. In
the blood: lake. 9 * 109 / l, the formula has not been changed. A Gram-based sputum smear shows a large
number of neutrophils and gram-negative diplococci. On the chest X-ray

372

Downloaded from the site - online testing step

increased lung function. What is the most likely diagnosis?


100% Bronchitis
0% Streptococcal infection
0% Asthma
0% Bronchiectasis
0% Pulmonary embolism

1620. a 32-year-old man notes increasing weakness in his limbs for 4 days. He was
previously healthy, but suffered a respiratory infection 10 days ago. Temperature 37.80 C,
AT-130/80 mm Hg, Ps - 94/Hv, BDR-42 / min. breathing is shallow. There is a symmetrical
weakness of the muscles of both halves of the face and the proximal and distal muscle groups of the extremities.
Sensitivity is preserved. Deep tendon reflexes are absent; flexion
plantar reflexes are noted. What is the most likely diagnosis?
100% Guillain-Barre Syndrome
0% Acute disseminated encephalomyelitis-lit
0% Myasthenia Gravis
0% Polio
0% Polymyositis

1621. a 55-year-old woman with decompensated cirrhosis of the liver, who is in a hospital,

https://translate.yandex.com/en/doc 358/540
22:20 ,27.6.2023 �� C : ; 5 B
receives spironolactone, potassium chloride solution and furosemide. At the moment, the patient
has pronounced lethargy and hypotension without respiratory disorders. There are symptoms
characteristic of chronic liver disease, ascites and small peripheral edema. The
ECG shows a regular, slow (55 / min) rhythm, the P wave is absent, the extended
QRS complex changes to the extended ST segment and the T wave. which of the following should
be administered intravenously?
100% Potassium solution
0% Calcium solution
0% Lidocaine solution
0% Magnesium solution
0% Saline solution

1622. a previously healthy 15-year-old boy has cramping pain in the


navel area; after a few hours, the pain shifted to the lower right quadrant and became permanent.
There were several episodes of vomiting. The abdomen is painful during deep palpation in
the lower right quadrant. X-ray examination of the thoracic and abdominal organs shows no
abnormalities. In the blood of Lake. 15 * 109 / l. in the urine, 3 leukocytes per day. which of the above is
the primary action in relation to this patient?
100% Diagnostic laparotomy
0% Computed tomography of the abdominal cavity
0% Intravenous pyelography and cystography
0% Irrigoscopy
0% Symptomatic treatment at home; re-hospitalization in case of increased pain

1623. An 8-year-old boy was bitten by a domestic dog. The child's medical history included the following:

373

Downloaded from the site - online testing step

all routine vaccinations. The dog was vaccinated against rabies. On examination:
there is a 1 cm long wound on the thigh, treated and closed with a plaster. What additional preventive actions are needed?
100% Taking antibiotics is impractical
0% Single dose of cephalexin
0% Taking erythromycin for 5 days
0% Taking penicillin for 2 days
0% Taking penicillin for 5 days

1624. A 64-year-old man suffering from severe pulmonary emphysema and receiving oxygen inhalation at
home was hospitalized due to bleeding from the upper gastrointestinal tract. The bleeding soon
stopped, but the patient developed agitation and disorientation, for which he
was given 5 mg of Diazepam intravenously. It doesn't respond to your voice after 20 minutes. The patient is in a stupor,
wakes up to painful irritation, there is swelling of the papilla of the optic nerve. Blood gases:
pH-7d7; Pa02-42 mmHg; PC02-95 mmHg. what should be emergency therapy?
100% Intubation
0% Correction of hypoxia by inhalation of a gas mixture with a high oxygen content
0% Correction of acidosis by bicarbonate infusion
0% Intravenous administration of 10 mg dexamethasone
0% Neurosurgeon's consultation

1625. A 37-year-old woman was found to have a 6 cm diameter lesion on the anterior surface of the thigh, which
has been observed for 10 months. The formation is fixed to the underlying muscles, but the skin above
the formation is mobile. What should be the doctor's tactics?
https://translate.yandex.com/en/doc 359/540
22:20 ,27.6.2023 �� C : ; 5 B

100% Puncture biopsy


0% Amputation
0% Open biopsy
0% Skeletal bone scans
0% CT scan of the abdominal cavity

1626. An 80-year-old man, who had never been hospitalized before, was admitted to the surgical
department with signs and symptoms of small bowel obstruction. Which of the following
clinical findings will be most effective in making a diagnosis?
100% The presence of Kloiberg "bowls" on the X-ray survey
0% Coffee grounds stomach aspirate
0% Leukocytosis 40
0% pH-7.5; PC02-50 mmHg.
0% Formation of the palpable pelvis

1627. an 18-year-old young man was taken to the emergency department after an accident.
Conscious, fully oriented, but witnesses report a short period
of fainting after the injury. The skull image shows a fracture of the left temporal bone.
After radiography, the patient suddenly loses consciousness and the doctor notes dilation of the left pupil.
What is the most likely diagnosis?
100% Epidural hematoma
0% Rupture of a congenital aneurysm

374

Downloaded from the site - online testing step

0% Acute subdural hematoma


0% Intra-abdominal bleeding
0% Arteriovenous malform rupture

1628. a newborn has swollen eyelids, hyperemic conjunctiva and a small


amount of clear discharge from the eyes 6 hours after birth. What is the most likely
diagnosis?
100% Chemical conjunctivitis
0% Dacryocystitis
0% Pneumococcal conjunctivitis
0% Gonococcal conjunctivitis
0% Chlamydial conjunctivitis

1629. a newborn baby after giving birth is slowed down, its movements are slow. Born with
a normal weight from the 4th pregnancy to a 28-year-old mother. The pregnancy was uneventful,
the delivery was quick, local anesthesia and intravenous administration of promedol were used. What
should be done to improve the condition of the newborn?
100% Enter naloxone
0% Conduct an infusion of 10% glucose solution
0% Introduce vitamin K
0% To study the concentration of blood serum electrolytes
0% Invite a neurologist for a consultation

1630. the child periodically has a fever of up to 40 ° C, swelling


https://translate.yandex.com/en/doc 360/540
22:20 ,27.6.2023 �� C : ; 5 B
of the finger joints. Complaints of upper chest pain. What is the most likely diagnosis?
100% Juvenile rheumatoid arthritis
0% Rheumatism
0% Toxic Synovitis
0% Septic arthritis
0% Osteoarthritis

1631. a child with staphylococcal pneumonia suddenly developed tachypnea


and shallow breathing. What is the most likely diagnosis that requires immediate action?
100% Strained pneumothorax
0% Pneumocele formation
0% Progression of pneumonia
0% Fear attack
0% Pleural effusion

1632. a 7-year-old child was taken to the emergency department. Within 3 hours, she
has a fever of 39.5 degrees Celsius and a sore throat. The child is frightened, breathing
is moderate inspiratory. What should I do immediately?
100% Prepare Everything for installing the air duct
0% Take a pharyngeal swab for culture

375

Downloaded from the site - online testing step

0% Examine blood gases and install an IV catheter


0% Perform chest X-ray, neck X-ray in the lateral projection
0% Hospitalize the child and place him in a high-humidity cuvette

1633. a child aged 1 week was admitted to the emergency department due to the acute
onset of the disease. About-but: vomiting with bile admixtures. An overview of the abdominal cavity
indicates an obstruction of the duodenum. Using a barium enema
, the disposition of the cecum was revealed. What is the most likely diagnosis?
100% Congenital intestinal pathology (bowel rotation disorder)
0% Empty bowel atresia
0% Hypertrophic pyloric stenosis
0% Acute appendicitis
0% Intestinal intussusception

1634. a 4-week-old boy experiences vomiting for 10 days, and its frequency
increases. The vomit is not colored with bile. The child eats with an appetite, looks good, but
loses weight. What is the most likely diagnosis?
100% Pyloric stenosis
0% Small bowel obstruction
0% Gastroenteritis
0% Invagination
0% Brain tumor

1635. a 7-year-old child has cramping abdominal pain and rash on the back, legs, buttocks, and
extended forearm surfaces. In laboratory parameters-proteinuria and
microhematuria. what is the most likely diagnosis?

https://translate.yandex.com/en/doc 361/540
22:20 ,27.6.2023 �� C : ; 5 B
100% Anaphylactoid purpura
0% Systemic lupus erythematosus
0% Post-streptococcal glomerulonephritis
0% Takayasu arteritis
0% Dermatomyositis

1636. in which of the listed comorbidities should pregnancy be terminated?


100% Eisenmenger syndrome
0% Atrial membrane defect
0% Ventricular membrane defect
0% Open Ductus arteriosus
0% Wolf-Parkinson-White arrhythmia (WPW syndrome)

1637. a pregnant woman with concomitant diabetes mellitus applied to a women's consultation. What should
a doctor expect and consider when managing such a patient?
100% Increased need for insulin in late pregnancy
0% Reduced placental glucose transport as a result of hyperglycemia
0% Increased need for insulin in early pregnancy

376

Downloaded from the site - online testing step

0% Tendency to develop hyperglycemia in early pregnancy


0% Tendency to develop acidosis in early pregnancy

1638. a pregnant woman in the third trimester was diagnosed with an acute urinary tract infection.
Which antibacterial drug should be preferred in this case?
100% Cephalexin
0% Tetracycline
0% Sulfonamide
0% Nitrofurantoin
0% Gentamicin

1639. a 39-year-old woman with a history of 3 pregnancies and 3 deliveries complains of uterine bleeding.
The survey revealed a secondary progressive nature of dysmenorrhea. During
the vaginal examination, a hard, diffusely enlarged uterus is determined.
Endometrial biopsy results without pathology. What is the most likely diagnosis?
100% Endometriosis
0% Endometritis
0% Adenomyositis
0% Uterine sarcoma
0% Leiomyoma

1640. A 54-year-old woman underwent laparotomy for a large mass in her pelvis, which turned
out to be a unilateral ovarian tumor with significant metastases to the omentum. The most appropriate
intraoperative strategy involves::
100% Removal of the omentum, uterus and both ovaries with tubes
0% Omentum biopsy
0% Ovarian biopsy
0% Removal of ovary and omentum metastases

https://translate.yandex.com/en/doc 362/540
22:20 ,27.6.2023 �� C : ; 5 B
0% Removal of the omentum and both ovaries with tubes

1641. a 28-year-old woman without pregnancy complains of bleeding between menstrual


cycles and heavy menstrual bleeding. In the last 2 years, she has had two scrapings
that have not resolved this problem. Oral contraceptives and antiprostaglandins
were also ineffective. What should be the drug tactic?
100% Hysterectomy
0% Hysteroscopy
0% Uterine curettage (complete removal of the endometrium)
0% Appointment of GnRH antagonists
0% Start high-dose progestin therapy

1642. a newborn girl on the 4th day has swelling of the mammary glands with
secretions, swelling of the vulva, bloody discharge from the genital tract. What is the most
likely cause of this condition?
100% Transfer of mother's estrogens to the fetus

377

Downloaded from the site - online testing step

0% Taking hormones by the mother during pregnancy


0% Increased thyroid-stimulating hormone levels in a newborn
0% Granulocellular tumor
0% Testicular feminization

1643. the district doctor was instructed to prepare a plan for carrying out a complex
of therapeutic and preventive measures among the population in the subordinate territory. What
secondary disease prevention measures should it include in this plan?
100% Prevention of disease complications
0% Prevention of diseases
0% Elimination of disease-causing factors
0% Improving the living conditions of the population
0% Implementation of rehabilitation measures

1644. The chief district pediatrician should conduct an analysis of the infant mortality rate
. What should it take as a unit of observation?
100% Case of death of a child under 1 year of age
0% Case of death of a child in the first month of life
0% Case of death of a child after 28 days of life
0% Case of death of a child in the first 7 days of life
0% Case of death of a child during childbirth

1645. the general practitioner was instructed to analyze the morbidity of the population at the site. Which of
the sources of morbidity studies provide a more complete account of acute diseases?
100% Appeal to outpatient clinics
0% Specially organized study
0% Causes of death
0% Population survey
0% Routine checkups

https://translate.yandex.com/en/doc 363/540
22:20 ,27.6.2023 �� C : ; 5 B

1646. A 43-year-old female patient complains of neoplasms,pain in the right breast,


fever up to 37.2°C during the last 3 months. The state of health worsens
before menstruation. Objectively: the right breast is swollen, hyperemic, and the nipple is retracted.
Indistinct painful infiltrate is palpated in the lower quadrants. What is the most likely
diagnosis?
100% Right breast cancer
0% Right acute mastitis
0% Right-sided chronic mastitis
0% Premenstrual syndrome
0% Tuberculosis of the right breast

1647. The Deputy Chief Physician for Medical Work conducted a study of the morbidity
rate of the population served in the polyclinic over the past 5 years. What
statistics can it use to calculate the prevalence of diseases?

378

Downloaded from the site - online testing step

100% Relative values


0% Standardized values
0% Average values
0% Absolute values
0% Dynamic range

1648. a 37-year-old woman complains of constrictive pain behind the sternum that occurs during
exercise. Objectively: the heart borders are extended to the left, systolic murmur in the aorta,
4cc=Ps-72 beats / min, AT-130/80 mm Hg on the ECG-signs of left ventricular hypertrophy. Which
survey method is the most informative in this case?
100% Echocardiography
0% Phonocardiography
0% Coronarography
0% Sphygmography
0% Radiography

1649. a 14-year-old girl complains of sleep disorders, weight loss, palpitations,


cardialgia, and fatigue. Hyperplasia of the thyroid gland of the II century, exophthalmos was noted. What
disorders of hormone levels are most characteristic of this disease?
100% Increased levels of thyroxine and triiodothyronine
0% Increased thyroid-stimulating hormone levels
0% Increased protein-bound iodine levels
0% Reduced thyroxine levels
0% Reduced triiodothyronine levels

1650. a 32-year-old patient complains of chills, pain in the lower back, lower leg muscles, weight loss up to 10
kg, and dry hands. Objectively: the skin is pale. Heart tones are muted, t0 to 38.50 S,
heart rate= Ps-110 beats / min, AT-190/115 mm Hg. Pasternatsky's symptom is weak-copositive on both
sides. In the blood: Er.- 2,8 * 1012/ l, Hb-65 g / l, WSE-58 mm / year, A/G-0.82, y-globe.- 30%. In the urine: protein -0.8
g / l, leuc.- 20-25 v / s, ep. up to 100 v / s, creatinine - 0.190 mmol/l. what is the most likely
diagnosis?
100% Nodular periarteritis
0% Systemic lupus erythematosus
https://translate.yandex.com/en/doc 364/540
22:20 ,27.6.2023 �� C : ; 5 B

0% Sharp jade
0% Dermatomyositis
0% Kidney stone disease

1,651. 500 cases of urolithiasis per 10,000 inhabitants were registered among the city's population
. What statistical value is used to display this data?
100% Intensity indicator
0% Ratio indicator
0% Visibility indicator
0% Extensity indicator
0% Compliance indicator

379

Downloaded from the site - online testing step

1652. At the end of the year, the hospital received data on the number of patients treated during
the year and the average annual number of beds used to treat patients. Which of
the hospital performance indicators can be calculated based on these data?
100% Bed turnover
0% Hospital bed capacity
0% Average annual bed employment
0% Average length of hospital stay for patients
0% Average bed downtime

1653. A 52-year-old patient fell from a height of 3 m on a flat surface of the Ground with his right lumbar
region. Complaints of pain in the right lumbar region. In the urine-microhematuria. According
to excretory urography, renal function is satisfactory. What is the most likely diagnosis?
100% Kidney injury
0% Subcapsular kidney rupture
0% Multiple kidney ruptures
0% Paranephral hematoma
0% Kidney detachment

1654. A 14-year-old girl complained of heavy spotting from the genital tract
within 10 days after a 1.5-month delay in her period. Such bleeding is repeated
periodically from the age of 12 on the background of an unspecified cycle. During rectal examination
, pathology from the internal genitalia was not detected. In the blood: HB-70 g / l, ep.- 2,3 * 1012/ l,
Ht-20. what is the most likely diagnosis?
100% Juvenile hemorrhage, post-hemorrhagic anemia
0% Werlhof's disease
0% Polycystic ovary syndrome
0% Iormon-producing ovarian tumor
0% Incomplete spontaneous abortion

1655. a 33-year-old patient with newly diagnosed diabetes mellitus uses a diet to maintain
glycemia after eating less than 10.0 mmol / l. he refrains from insulin therapy. What
is the most important study to differentiate between type 1 (insulin-dependent) and type 2
(insulin-dependent) diabetes?
100% Determination of antibodies to islet cells
0% Ilukozotolerantny test

https://translate.yandex.com/en/doc 365/540
22:20 ,27.6.2023 �� C : ; 5 B
0% Fasting blood glucose test
0% Determination of glycosylated hemoglobin in the blood
0% Determination of fructosamine in the blood

1656. A 14-year-old patient became acutely ill when he developed a severe fever and sharp pain in his right
shin. Two weeks later, on the radiograph in the projection of the middle third
of the tibial diaphysis, areas of enlightenment (destructive foci) with
uneven contours are determined. Along the border of the bone at a distance of 1-2 mm from its surface, a
narrow strip of darkening (periostitis) is found. What pathology is characterized by such an X-ray
picture?

380

Downloaded from the site - online testing step

100% Osteomyelitis of the right shin


0% Tuberculosis of the right lower leg
0% Syphilis of the right lower leg
0% Bone cyst
0% Right shin injury

1657. a patient was admitted to the clinic complaining of severe weakness, fever
up to 39.20 C, sore throat during swallowing, and a hemorrhagic rash on the skin of the trunk. In the blood
, anemia, thrombocytopenia, blasts - 14%, SSE-40 mm / h. What examination should be performed to
clarify the diagnosis?
100% Sternal puncture
0% X-ray of the skull bones
0% Detailed blood test
0% Sowing from tonsils
0% Coagulogram

1658. A 43-year-old patient underwent a cholecystectomy 6 years ago for chronic


calculous cholecystitis, and has been experiencing pain in the right hypochondrium and
transient jaundice for the past six months. The last 2 weeks of jaundice does not go away. During the examination
, stenosing papillitis with a length of up to 0.5 cm was detected. Choose the best treatment option for the patient:
100% Perform endoscopic papilosphincterotomy
Conduct conservative treatment: antispasmodics, antibiotics, anti-inflammatory
0%
drugs
0% Perform external drainage of khole-dokha
0% Perform transduodenal papilo-sphincterotomy
0% Perform choledochoduodenostomy

1659. in one of the sections of the railway station where sand wagons are unloaded
in bulk, a physiological and hygienic study of the nature of the work of movers who manually
break up the baked mass with a shovel and transfer it found that their work belongs to
the third degree of severity. Which of these criteria could be used to evaluate the work
of movers?
100% Maximum mass of cargo to be moved
0% Static load value per change
0% Time of active actions, % of the shift duration
0% Passive monitoring time, % of the change duration
0% Intellectual tension
https://translate.yandex.com/en/doc 366/540
22:20 ,27.6.2023 �� C : ; 5 B

1660. workers of fisheries are exposed to low ambient temperatures


from - 50C to-150C. diseases of what organs and systems are most often found
in workers of these industries?
100% Respiratory system
0% Cardiovascular system
0% Blood
0% The liver

381

Downloaded from the site - online testing step

0% Gastrointestinal tract

1661. In an industrial district of one of the industrial cities, preschool-age


children have become increasingly ill with chronic asthmatic bronchitis and bronchial asthma. Which of
the persistent air pollutants could have caused them?
100% Sulfur dioxide
0% Carbon monoxide
0% Lead
0% Nitrogen oxides
0% Products of photochemical reactions

1662. A 7-year-old boy has been undergoing medical treatment for a month. During hospitalization
, severe edema was observed, proteinuria-7.1 g / l, Protein in the daily urine-4.2 g. the
blood microbiochemical analysis showed hypoproteinemia (43.2 g/l), hypercholesterolemia (9.2 mmol/l). Which of
these variants of glomerulonephritis is most likely to occur in the patient?
100% Nephrotic
0% Nephritic
0% Isolated urinary tract
0% Hematuric
0% Mixed

1663. a 30-year-old patient is disturbed by attacks of chills, fever, profuse perspiration, which recur
every third day. He's been ill for 2 weeks. The sclera and skin turn bright yellow. The liver and spleen
are enlarged. A year ago I worked in Africa. Which diagnosis is most likely?
100% Malaria
0% Cancer of the head of the pancreas
0% Sepsis
0% Viral hepatitis
0% Leptospirosis

1664. examination of a newly born placenta revealed the presence of


a 2x3 cm defect. There is no bleeding. Which tactic is most justified?
100% Manual revision of the uterine cavity
0% Appointment of uterotonic drugs
0% External uterine massage
0% Supervision of a woman in labor
0% Instrumental revision of the uterine cavity

https://translate.yandex.com/en/doc 367/540
22:20 ,27.6.2023 �� C : ; 5 B

1665. A 27-year-old pregnant woman at 17 weeks was admitted to the hospital for treatment. In the anamnesis-2
spontaneous miscarriages. During bimanual examination: the uterus is enlarged up to 17 weeks
of pregnancy, the cervix is shortened, the peephole passes the tip of the finger. The following diagnosis was made:
isthmic-cervical insufficiency. What are the doctor's tactics?
100% Apply a suture to the cervix
0% Tocolytic therapy

382

Downloaded from the site - online testing step

0% Termination of pregnancy
0% Perform hormone treatment
0% Do an amniocentesis

1666. The district pediatrician conducted an analysis of child mortality in the subordinate territory.
What indicators were used by them?
100% Mortality of children under 1 year of age, gender, and causes
0% Mortality of children under 1 year, stillbirth rate
0% Mortality of children by age, in hospitals
0% Teen mortality in the area
0% Infant mortality in the area

1667. a doctor studies the levels of general morbidity in the population of a rural medical
district. Which accounting and statistical document is the source of this information?
100% Statistical coupon for registration of final (updated) diagnoses
0% Control card of dispensary observation
0% Medical card of an outpatient patient
0% Doctor's appointment card
0% Medical record of a prophylactically examined patient

1668. A 12-year — old girl's body temperature rose to 39-400 ° C in 4-5 hours after taking two aspirin
tablets.complains of general malaise, dizziness, sudden
appearance of red spots on the skin with the formation of blisters or peeling of the epidermis, with
erosion of the surface. The lesions on the skin looked like second-degree burns. Positive
ni-Kola symptom. What is the most likely diagnosis?
100% Acute epidermal necrolysis
0% Pemphigus vulgarus
0% Polymorphic exudative erythema
0% Bullous dermatitis
0% Duhring's herpetiform dermatitis

1669. A 23-year-old woman who suffered from Raynaud's syndrome for 5 years had a
fever of 38.5-39.20 C, joint pain, facial flushing, weight loss,
and weakness a month ago. In the blood: leuc. - 3, 2 * 109/l, WSE-38 mm / h. The study of which immunological
parameters will have the greatest diagnostic value?
100% DSDNA antibodies
0% Antibodies to phospholipids
0% Serum immunoglobulin level
0% Number of T-lymphocytes
0% Number of T-suppressors

https://translate.yandex.com/en/doc 368/540
22:20 ,27.6.2023 �� C : ; 5 B

1670. the patient complains about the presence of a pathological formation that appears in the right
inguinal region during physical exertion. The formation of a rounded shape, with a diameter
of about 4 cm, with palpation of a soft-elastic consistency, is located near the medial

383

Downloaded from the site - online testing step

parts of the pupart ligament. The formation is located to the north of the spermatic cord. What
is the preliminary diagnosis?
100% Right-sided straight inguinal keel
0% Right-sided oblique inguinal keel
0% Right femoral keel
0% Varicose veins of the right thigh
0% Lipoma of the right inguinal region

1671. a 5-year-old child had an acute attack of palpitations, accompanied by nausea,


dizziness, and general weakness. The ECG showed tachycardia with a heart rate of 220 / min.
Ventricular complexes are deformed and dilated. The P-prong is missing. What is the main
drug that should be prescribed for emergency care?
100% Lidocaine
0% Isoptin
0% Seduxenum
0% Novocainamide
0% Strophanthin

1672. a 37-year-old patient complains of a sharp pain that suddenly appeared in the right hypochondrium, and
then quickly spread to the entire abdomen. The abdomen is retracted, palpation is painful,
and the Blumberg-Shchetkin symptom is positive. On the X-ray survey of the abdominal organs
performed in an upright position of the patient,a crescent-shaped illumination under
the dome of the diaphragm is determined. What is the most likely diagnosis?
100% Gastric ulcer perforation
0% Acute cholecystitis
0% Acute appendicitis
0% Hepatic colic
0% Acute intestinal obstruction

1673. a 52-year-old patient went to the doctor with complaints of weakness, painful itching of the skin
after washing, washing in the bathroom, and heaviness in the head. Objectively: the skin of the face,neck,
and extremities is hyperemic. AT-180/100 mm Hg spleen 4 cm below the edge of the costal arch.
What is the most likely diagnosis?
100% ^ ritremia
0% Hypertension
0% Dermatomyositis
0% Allergic dermatitis
0% Systemic scleroderma

1674. A 22-year-old patient complains of a pad-like whiteness of the fingertips


that develops when she cools down. When warmed, the fingers first turn bluish, and then
beet color. The drug of choice for the treatment of this syndrome is:
100% Nifedipine

https://translate.yandex.com/en/doc 369/540
22:20 ,27.6.2023 �� C : ; 5 B
0% Dipyridamole
0% Captopril

384

Downloaded from the site - online testing step

0% Aspirin
0% Nitrates

1675. the district doctor must prepare a report on the state of health of the population of their
service area. What medical indicators of the population's health should they use when
doing this?
100% Morbidity, disability, demographic, physical development
0% Social welfare, satisfaction with the quality of life
0% Lifestyle factors, environmental pollution, genetic factors
0% Average duration of patient's treatment, specific weight of complications
0% Average life expectancy

1676. an 8-year-old boy suffering from hemophilia a experienced


massive bleeding from the hole of a missing tooth 8 hours after the fall. Specify the leading therapy method:
100% Introduction of cryoprecipitate
0% Red blood cell mass transfusion
0% Transfusion of fresh frozen plasma
0% Dicynonum
0% Vikasol

1677. a 28-year-old woman went to the doctor with complaints of puffiness of the face, slight
swelling of the legs, and sometimes notes urine the color of "meat slop". As a teenager, she often
had sore throats. Objectively: pale skin, temperature 36.80 C, Ps-68 bpm, rhythmic.
AT170 / 110 mmHg. what changes in urine are most likely?
100% Proteinuria, hematuria, and cylindruria
0% Increased relative density, hematuria, bacteriuria
0% Reduced relative density, proteinuria, poor urinary sediment
0% Erythrocyturia and urinosuria
0% Reduced relative density, proteinuria, myoglobinuria

1678. A 36-year-old female patient was hospitalized with complaints of acute pain behind the sternum, which occurred
after accidentally swallowing a fish bone. Esophago-gastroscopy failed to detect a foreign
body. The pain increased, localized between the shoulder blades. A day
later, the body temperature increased, the condition worsened, and dysphagia increased. What is the complication?
100% Esophageal perforation with mediastinitis development
0% Esophageal hemorrhage
0% Esophageal obturation
0% Atelectasis of the lung
0% Aspiration pneumonia

1679. a patient was delivered urgently with complaints of lower abdominal pain radiating into
the rectum, spotting from the genital tract, and dizziness. Complaints appeared suddenly.
Last menstruation 2 weeks ago. The skin is pale, Ps-102 beats / min, AT-90/60 mm Hg.
the abdomen is tense, slightly painful in the lower parts, symptoms of irritation of the peritoneum

https://translate.yandex.com/en/doc 370/540
22:20 ,27.6.2023 �� C : ; 5 B

385

Downloaded from the site - online testing step

weakly positive ones. What is the most likely diagnosis?


100% Ovarian apoplexy
0% Fibroid that is born
0% Intestinal obstruction
0% Ectopic pregnancy
0% Abortion that started

1680. a full-term child from the 1st uneventful pregnancy, burdened labor, had a
cephalohematoma. On the 2nd day jaundice appeared, on the 3rd - changes in the neurological status:
nystagmus, Grefe's syndrome. Urine is yellow, feces are golden yellow. Mother's blood
type A (II)Rh -, child-A (II) Rh+. On day 3, Hb in a child is 200 g / l, ep.- 6,1 * 1012/ l, bilirubin in the blood-58
mmol / l due to the unbound fraction, Ht-0.57 how to explain jaundice in a child?
100% Craniocerebral birth trauma
0% Physiological jaundice
0% Hemolytic disease of newborns
0% Biliary tract atresia
0% Fetal hepatitis

1681. A 7-week pregnant woman is referred for an artificial abortion. During the operation, when
dilating the cervical canal with the Iegar No. 8 dilator, the doctor suspected
uterine perforation. What are the main tactics of the doctor to confirm the diagnosis?
100% Probing the uterine cavity
0% Bimanual research
0% Ultrasound examination
0% Laparoscopy
0% Metrosalpingography

1682. an 18-year-old patient complains of pain in the knee and ankle joints,
an increase in body temperature up to 39.50 C. A week and a half before that, he suffered a respiratory
illness. Objectively: body temperature 38.50 C. There is swelling of the knee and
ankle joints. Ps-106 bpm, rhythmic. AT-90/60 mm Hg. heart borders are not
changed, tones are weakened, soft systolic murmur at the apex. What indicator is most
associated with the possible etiology of the process?
100% Antistreptolysin-O
0% 1-antitrypsin
0% Creatine Kinase
0% Rheumatoid factor
0% Seromucoid

1683. A 26-year-old woman was taken to the emergency department with complaints of sudden spitting in
the lower abdomen, weakness, and loss of consciousness at home. There was no previous menstruation.
Hb-106 g / L, Ps-120 bpm, AT-80/50 mmHg pain and symptoms of peritoneal irritation
lower right. What is the most likely diagnosis?
100% Disrupted tubal pregnancy
0% Acute appendicitis

386

https://translate.yandex.com/en/doc 371/540
22:20 ,27.6.2023 �� C : ; 5 B

Downloaded from the site - online testing step

0% Right acute adnexitis


0% Twist the legs of the ovary
0% Ovarian apoplexy

1684. A 28-year-old patient without a permanent place of residence, who was hospitalized with a preliminary
diagnosis of “flu", developed a roseolous-petechial rash on the trunk and
internal surfaces of the limbs on the 5th day of illness. Temperature 410c, euphoria, flushing of the face, redness
of the sclera, tremor of the tongue, tachycardia, splenomegaly, agitation. What is the most likely diagnosis?
100% Typhus fever
0% Alcoholic delirium
0% Leptospirosis
0% Measles
0% Typhoid fever

1685. a 65-year-old patient palpating the abdomen in the navel area and above palpates a tumor
measuring 13x8 cm, moderately painful during palpation, does not move, pulsates. Auscultation:
systolic murmur. What is the most likely diagnosis?
100% Abdominal aortic aneurysm
0% Stomach tumor
0% Arterio-venous aneurysm
0% Tricuspid valve insufficiency
0% Mitral valve insufficiency

1686. A 30-year-old patient complained to a general practitioner about skin itching


that worsened in the evening. Ill for 1.5 months. Objectively: on the skin of the interdigital folds of the hands,
flexor surfaces of the limbs, abdomen, thighs, and lower legs,a rash
consisting of paired papules covered with bloody crusts, linear swelling is observed. What
additional examination methods should be used to clarify the diagnosis?
100% Investigation of scouring of loose elements
0% Definition of dermography
0% Serological blood testing
0% Determination of blood glucose level
0% Helminth screening

1687. a 28-year-old patient complains of pain in his legs while walking, chilly feet
and toes. He has been ill for a year. Objectively: the skin of the feet is pale, cool,turgor is low,
hypotensive. Pulsation on the femoral and popliteal arteries is weakened, on the arteries of the feet
palpated after a nitroglycerin test. Rheographic index < 1. what is the most likely
diagnosis?
100% Obliterating endarteritis
0% Chronic thrombophlebitis
0% Obliterating atherosclerosis
0% Raynaud's disease
0% Burger's disease
1688. during a medical examination of a 3rd-grade student, a disharmonious physical state was established.

387

https://translate.yandex.com/en/doc 372/540
22:20 ,27.6.2023 �� C : ; 5 B
Downloaded from the site - online testing step

development. Objectively: the biological age corresponds to the calendar age, there is compensated
chronic bronchitis. There are no clinical functional changes on the part of other organs. What
health group does the student belong to?
100% Group III
0% Group I
0% Group IV
0% Group II
0% V group

1689. a 58-year-old patient is examined by a therapist due to general weakness,


fatigue, moderate pain in the left hypochondrium, and sometimes frequent urination with cutting
pain. Moderate splenomegaly was detected. In the blood: neutophilic leukocytosis with a shift to
myelocytes, basophils-2%, eosinophils-5%. In the urine: urate mass, ep. - 2-3 in the field of vision. What
is the preliminary diagnosis?
100% Chronic myeloid leukemia
0% Leukemoid reaction
0% Lymphogranulomatosis
0% Cirrhosis of the liver
0% Urolithiasis

1690. when a child was admitted to a children's group, a systolic murmur with punctum
maximum was detected in the II-III intercostal space on the left edge of the sternum. What congenital heart disease can
be suspected?
100% Septal defect
0% Aortic stenosis
0% Aortic coarctation
0% Fibroelastosis
0% Open Ductus arteriosus

1691. A 62-year-old patient complained of periodic discharge of vermi-podiatric


blood clots in the urine. In the right half of the abdomen,a bumpy,
painless, mobile formation is detected during palpation. Which of these survey methods should
be used first?
100% Ultrasound of the kidneys and retroperitoneal space
0% Chromocystoscopy
0% Excretory urography
0% Cystoscopy
0% Computed tomography of the pelvis

1692. a patient was hospitalized in a medical and preventive institution with a diagnosis
of diphyllobothriosis. During the consumption of which fish was infected?
100% Summer carp
0% Pollock
0% Flounder

388

Downloaded from the site - online testing step

https://translate.yandex.com/en/doc 373/540
22:20 ,27.6.2023 �� C : ; 5 B
0% Sea Bass
0% Sea halibut

1693. A 65-year-old patient reported difficulty opening her mouth 10 days after the injury to the foot area
. The next day, I could hardly eat, joined the tension
of the muscles of the back of the head, back, and abdomen. On the third day of the disease, tonic tension of all
muscle groups was noted, generalized convulsions every 10-15 minutes. What is the most likely diagnosis?
100% Tetanus
0% Tetany
0% Meningoencephalitis
0% Hemorrhagic stroke
0% Epilepsy

1694. A 60-year-old woman complains of unbearable pain in her right hypochondrium. a history of acute
pancreatitis. Temperature-38.20 C. objectively: zhovtyannost sclera.
There are no symptoms of peritoneal irritation. Positive symptoms of Ortner, Jubergritz-Skulpsky. Urine diastasis - 320
g / hour. What is the most likely diagnosis?
100% Chronic pancreatitis
0% Acute cholangitis
0% Chronic cholecystitis
0% Acute cholecystitis
0% Pancreatic cancer

1695. A 20-year-old patient became acutely ill. On the 2nd day of the illness, he complains of a severe
headache in the temples and orbits, body aches, and a dry, painful cough. Objectively: e°-39°C,
the patient is adynamic, the mucous membrane of the pharynx is "burning", wheezing is not
heard in the lungs. What is the most likely diagnosis?
100% Flu
0% Parainfluenza
0% Respiratory mycoplasmosis
0% Pneumonia
0% Meningococcal infection

1696. a 55-year-old patient complains of weakness for 2 months, pain in the right side
of the chest, cough, sputum production with streaks of blood. Radiologically:
intense triangular shadow on the lower lobe associated with the mediastinum. What
disease should you think about?
100% Lung cancer
0% Tuberculosis of the lungs
0% Bronchiectasis
0% !nfarct of the lungs
0% Pleuropneumonia

1697. in a forest summer sanatorium, children are given various exercises to harden their bodies.

389

Downloaded from the site - online testing step

procedures. Which of the following procedures is the most tempering?


100% Contrast shower
0% Morning exercise in the fresh air
https://translate.yandex.com/en/doc 374/540
22:20 ,27.6.2023 �� C : ; 5 B

0% Hygienic shower
0% Walking in the fresh air
0% Hydro massage bath

1698. a 65-year-old patient complains of pain in the right side of the face,headache,
fever. Ill for 3 days, the disease is associated with hypothermia.
Objectively: in the area of the right half of the forehead skin-inflammatory hyperemia, edema. At the sites
of blisters-erosion with necrotic plaque. On the edges of the focus-vesicles with inflammatory hyperemia.
What is the most likely pathology?
100% Shingles
0% Erysipelas
0% Dermatitis
0% Eczema
0% Lichen planus erythematosus

1699.Among the population living near a pesticide production facility


, the rate of congenital malformations,which manifest themselves in Central
paralysis, idiocy, and neonatal blindness, is dynamically increasing. Compounds of what environmental
pollutant can cause the development of this pathology?
100% Mercury
0% Strontium
0% Cadmium
0% Iron
0% Chrome

1700. It is planned to build a bagatoprofile hospital in one of the central districts


of the city. What type of development is most appropriate in this case?
100% Centralized and block-based
0% Centralized
0% Decentralized
0% Mixed
0% Blocky

1701. on the third day after venectomy for saphenous vein thrombophlebitis, a 60-year-old patient
suddenly developed an acute feeling of lack of air. The skin became at first sharply cyanotic, and
then ashen in color. Pronounced psychomotor agitation, tachypnea, chest pain.
What complication of the postoperative period occurred?
100% Pulmonary embolism
0% Bleeding
0% Hypostatic pneumonia
0% Myocardial infarction

390

Downloaded from the site - online testing step

0% Valvular pneumothorax

1702.as a result of an accident, the victim received numerous fractures of the limbs and pelvic bones. In the anamnesis:
hemophilia A. During the examination, hematomas are formed on the damaged areas. The condition

https://translate.yandex.com/en/doc 375/540
22:20 ,27.6.2023 �� C : ; 5 B
is getting worse. AT-90/50 mm Hg. what is the most appropriate combination of infusion agents for
the treatment of a patient after the use of polyglucin and saline solutions?
100% Cryoprecipitate, red blood cell mass
0% Fresh frozen plasma, albumin
0% Red blood cell mass, freshly frozen plasma
0% Cryoprecipitate, glucose
0% Red blood cell mass

1703. a 70-year-old man has coronary heart disease. The mood is noticeably low,
anxious. Against the background of prolonged insomnia, there were fears, unwillingness to live, thoughts
of suicide. He sits for a long time without changing his position, does not answer immediately, quietly, in a monotonous voice.
An expression of suffering, pain, and fear. What is the leading psychopathological syndrome?
100% Depressive
0% Paranoid
0% Asthenic
0% Phobic
0% Obsessive

1704. a patient has been suffering from rheumatoid arthritis for more than 1 year. The course of the disease
is rapidly progressing. X-ray examination confirmed the presence of marginal usurs. Treatment with what
basic drug is the most appropriate?
100% Methotrexate
0% Delagil
0% Prednisone
0% Diclofenac-sodium
0% Aspirin

1705. A 31-year-old patient has been suffering from systemic scleroderma for 14 years. She was repeatedly
treated in hospitals. Complains of periodic dull pain in the heart, palpitations,
shortness of breath, headache, swelling of the eyelids, weight loss, pain and deformity of the joints of the extremities.
What organ damage worsens the prognosis of the disease?
100% Kidneys
0% Heart
0% Lungs
0% Gastrointestinal tract
0% Skin and joints

1706. a child born in the 8th month of pregnancy was found to have microcephaly, cataracts,
and heart defects. The mother of the child in the 2nd month of pregnancy was ill: there was a short rise in temperature
to 37.50 C, an increase in lymph nodes and a small-spotted rash on the face, torso and

391

Downloaded from the site - online testing step

on the limbs, which passed without residual phenomena. What disease did the mother suffer from during
pregnancy?
100% Rubella
0% Cytomegalovirus infection
0% Herpes infection
0% Chlamydia infection
0% Toxoplasmosis
https://translate.yandex.com/en/doc 376/540
22:20 ,27.6.2023 �� C : ; 5 B

1707.A 50 - year-old woman suffering from chronic pyelonephritis was


prescribed a combination of antibacterial agents-gentamicin (80 mg 3 times a day) and
Biseptol (960 mg 2 times a day). What are the consequences of prescribing such
a combination of antibiotics?
100% Acute renal failure
0% Glomerulosclerosis
0% Chronic renal failure
0% The combination of antibiotics is optimal and quite safe
0% Acute adrenal insufficiency

1708. a 32-year-old patient who was treated in a hospital for an acute abscess of the right lung
suddenly developed heavy breathing, cyanosis, and pain in the right side of the chest after coughing.
What is the most likely complication?
100% Pyopneumothorax
0% Heart attack-pneumonia
0% Myocardial infarction
0% Esophageal perforation
0% Exudative pleurisy

1709. a 50-year-old grinder turned to a shop doctor with complaints of general weakness,
stiffness of the end phalanges of the fingers and severe pain in them. Objectively: paleness of the skin
of the fingers. During the study of pain, tactile and temperature sensitivity, minor
violations were detected. There were no deviations from other bodies and systems. What is the most likely
diagnosis?
100% Vibration sickness
0% Polyneuritis
0% Raynaud's disease
0% Syringomyelia
0% Deforming osteoarthritis

1710. a 35-year-old patient suffering from chronic glomerulonephritis and


who has been on hemodialysis for the last 3 years has experienced heart failure, hypotension, growing
weakness, and shortness of breath. On the ECG: bradycardia, atrioventricular block and B., high pointed
t teeth. The day before - a gross violation of drinking and dietary regimes. What is the most likely
reason for these changes?
100% Hyperkalemia
0% Hyperhydration

392

Downloaded from the site - online testing step

0% Hypokalemia
0% Hypernatremia
0% Hypocalcemia

1711. an 18-year-old girl complains of weakness, dizziness, decreased appetite,


and menorrhagia. Objectively: on the skin of the upper extremities, petechiae are of different colors. In the blood: Hb-105 g / l,
Er-3.2 * 1012/l, CP-0.95, blood clot.-20 * 109 / l. blood clotting time According to Lee-White - 5'; Duration
of bleeding according to Duke-8', pinch and tourniquet tests (+). What is the most likely diagnosis?
100% Thrombocytopenic purpura
https://translate.yandex.com/en/doc 377/540
22:20 ,27.6.2023 �� C : ; 5 B

0% Hemophilia
0% Hemorrhagic diathesis
0% Iron deficiency anemia
0% Marchiafava-Micheli disease

1712. A 28-year-old patient was hospitalized on the 9th day of illness with complaints of
fever up to 390C, headache, general weakness, stool retention and
sleep disturbance. On the abdomen-single roseoli, Ps-78 bpm, liver enlarged by 2 cm. What is the most
likely diagnosis?
100% Typhoid fever
0% Typhus fever
0% Sepsis
0% Brucellosis
0% Leptospirosis

1713. a child aged 40 hours has hyperesthesia, central nervous system depression, and impaired
appetite. Sepsis is suspected. What should be used for differential diagnosis?
100% Hypoglycemia
0% Hypocalcemia
0% Hyperbilirubinemia
0% Hyperkalemia
0% Hypomagnesemia

1714. a patient in an unconscious state, with head injuries in the form of a fracture of the bones
of the arch, blood pressure-60/40 mm Hg, with pathological neurological symptoms. On what grounds
can these injuries be considered severe?
100% Life - threatening symptoms
0% Neurological pathologic symptoms
0% Persistent disability
0% Timing of a health disorder
0% Nature of the wound

1715. a 35-year-old female patient


had reddening of the skin of the lower third of the forearm and hands for two to three weeks after applying synthetic detergent, which
was accompanied by itching. Objectively: hyperemia, edema, disseminated papular lesions

393

Downloaded from the site - online testing step

elements. What is the most likely diagnosis?


100% Allergic dermatitis
0% Simple dermatitis
0% Limited neurodermatitis
0% Professional eczema
0% Toxicoderma

1716. in the first-born child with a pelvis size of 25-28-31-20 cm, active labor activity began.
The water's gone, clean. The weight of the fetus is 4500 g. the head is pressed against the entrance to the small pelvis.
The Vaste attribute is positive. Full opening of the cervix. There is no fertile bubble. Fetal heartbeat
is clear, rhythmic, 136 / min. What are the tactics of labor management?
https://translate.yandex.com/en/doc 378/540
22:20 ,27.6.2023 �� C : ; 5 B

100% Caesarean section


0% Vacuum extraction of the fetus
0% Obstetric forceps
0% Conservative labor management
0% Stimulation of labor activity

1717. a 41-year-old patient, after lifting a heavy object, noted sharp pain in the right side
of the chest, shortness of breath sharply increased. The patient's condition is severe: cyanosis of the lips and mucous membranes,
PDR-28 / min, Ps-122 beats / min. Over the right half of the chest percussion-timpanitis,
auscultation-sharply weakened breathing; accent of the second tone over the pulmonary artery. AT-80/40
mmHg. what is the main emergency measure at the pre-spital stage?
100% Aspiration of air from the pleural cavity
0% Introduction of epinephrine
0% Introduction of eufillin
0% Call a cardiology team
0% Inhaling oxygen

1718. a 6-year-old child has been bothered by thirst, polyuria, and increased appetite for two months.
At the same time, there is a decrease in body weight by 3 kg. During the week, nocturnal
enuresis was added. During the examination, hyperglycemia of 14 mmol/l was detected. the diagnosis was established: diabetes
mellitus, type I. what is the most likely genesis of this disease?
100% Autoimmune
0% Viral
0% Bacterial
0% Neurogenic
0% Viral-bacterial

1719. a 74-year-old patient complains of pain and bloating, nausea. He suffers from coronary
heart disease, post-infarction and atherosclerotic cardiosclerosis. Objectively: the condition is severe,
the abdomen is swollen, the abdominal wall is poorly involved in the act of breathing. During laparoscopy:
there is a small amount of cloudy effusion in the abdominal cavity, one of the loops of the small intestine
is dark-blue in color. What is the most likely diagnosis?
100% Mesenteric vascular thrombosis
0% Inversion of the bowel

394

Downloaded from the site - online testing step

0% Acute intestinal obstruction


0% Ischemic abdominal syndrome
0% Erysipelas

1720. A 56-year-old man who regularly consumed alcohol repeatedly


experienced intense abdominal shingles pain for 20 years. For the last 2 years, unformed profuse
stools have appeared, 23 times a day. I lost 8 kg in 2 years. Objectively: the abdomen is soft and painless.
Blood amylase-12 g / l. in the analysis of feces-neutral fat 15 g / day, starch grains. What is most
appropriate to prescribe for the treatment of the patient at this stage?
100% Pancreatin
0% Kontrikal
0% Aminocislaproic acid
0% Levomycetin
https://translate.yandex.com/en/doc 379/540
22:20 ,27.6.2023 �� C : ; 5 B

0% Imodium

1721. A 30-year-old serviceman received a single dose of 15 Gy of radiation in a


radioactively contaminated area. There were complaints of a compressive headache,
fever up to 380C, nausea, double vomiting. Clinic of what form of acute radiation
sickness will this person develop?
100% Intestinal
0% Bone Marrow surgery
0% Toxemic
0% Cerebral
0% -

1722. a 30-year-old woman has repeated labor lasting 14 hours. Fetal heartbeat
is muffled, arrhythmic, 100 / min. Weight study: the opening of the cervix is complete,
the fetal head is in the plane of exit from the small pelvis. Sagittal suture in straight size, small
crown near the womb. What are the further tactics of childbirth?
100% Using the original obstetric forceps
0% Oxytocin stimulation of labor
0% Caesarean section
0% Skin-main forceps by Ivanov
0% Use of abdominal obstetric thorns

1723. a 31-year-old man has been mentally ill for many years. Against the background of prolonged insomnia, there
were fears, thoughts of suicide, tried to hang himself. The mood is noticeably reduced, refuses
treatment. What are the most appropriate measures to prevent suicide in this
case?
100% Hospitalization in a mental hospital
0% Hospitalization in the neurological department
0% Outpatient treatment
0% Psychotherapy conversation
0% Strict supervision at home
1724. a 43-year-old woman complains of stabbing pain in the heart and shortness of breath for 3 weeks,

395

Downloaded from the site - online testing step

interruptions in the activity of the heart, increased fatigue. I had an acute respiratory infection a month ago. Objectively:
heart borders + 1.5 cm to the left, muted tones, soft systolic murmur at the apex and in Botkin's body,
heart rate-98 / min, single extrasystoles. AT-120/80 mm Hg. the liver is not palpable,
there is no edema. In the blood: leuc. - 6, 7 * 109/l., SHZ-21 mm / year. What is the most likely diagnosis?
100% Acute myocarditis
0% Climacteric myocardial dystrophy
0% IHD, angina pectoris
0% Rheumatism, mitral insufficiency
0% Hypertrophic cardiomyopathy

1725. a 17-year-old boy complains of a sudden palpitation that occurred 10 minutes ago.
Objectively: normal skin color. Heart boundaries are not changed, heart rate=Ps-200 beats / min, the rhythm
is correct. AT-135/75 mmHg heart rate of ECG amplifications: QRS-0.09 sec. What actions should
I take during the first stage of help?
100% Perform vagal tests

https://translate.yandex.com/en/doc 380/540
22:20 ,27.6.2023 �� C : ; 5 B
0% Enter novocainamide intravenously
0% Give Anaprilin per os
0% Perform a cardioversion
0% Refrain from treatment

1726. a 54-year-old patient with a myocardial infarction suddenly experienced suffocation and general weakness on the second day
. Objectively: cyanosis, HR-36 / min, HR-110 / min, AT-130/70 mm Hg. above the lungs
, breathing is weakened, in the lower parts - wet wheezing. Heart tones are weakened, the accent is tone II over
the pulmonary trunk. What drug is the most rational way to start treatment?
100% Nitroglycerin
0% Lasix
0% Dopamine
0% Euphyllinum
0% Anaprilin

1727. a 35-year-old patient who has been working as a tunneller in a mine for 10 years complains of
shortness of breath during physical exertion, coughing with a small amount
of sputum. Body temperature is normal. In the lungs, wheezing is not heard. SSE - 8 mm / h,
Mantoux reaction with 2 to-10 mm papule. On the X-ray: small,
high-intensity foci that are clearly outlined throughout the entire volume of both lungs. What is the preliminary diagnosis?
100% Anthracosis
0% Bronchiolitis
0% Pneumonia
0% Carcinomatosis
0% Tuberculosis

1728. A 60-year-old patient complains of severe pain in the right eye, photophobia, lacrimation,
decreased vision in this eye, and pain in the right side of the head. The pain started 2 days ago. Objectively:
S OD-0.03, congestive injection of the eyeball, sharp corneal edema,deep anterior chamber,
narrow pupil, atrophic ry-arch, glau-comatose excavation of the optic nerve on the fundus.

396

Downloaded from the site - online testing step

HERE-38 mmHg ^S OS-0.8, not corrected. The eye is calm and healthy. HERE-22 mm Hg. what
is the most likely diagnosis?
100% Acute glaucoma attack
0% Uveitis of the right eye
0% Keratitis of the right eye
0% Optic neuritis
0% Macular degeneration

1729. a 45-year-old man with hypertension of the 2nd degree and ventricular
extrasystoles notes the appearance of stenocarditis attacks during physical activity. Objectively: Ps-80
beats / min, AT-170/100 mm Hg. which drug is most appropriate for the patient's treatment?
100% Propranolol
0% Enalapril
0% Nifedipine
0% Isosorbide dinitrate
0% Verapamil

https://translate.yandex.com/en/doc 381/540
22:20 ,27.6.2023 �� C : ; 5 B

1730. A 41-year-old patient has been suffering from ulcerative colitis for 5 years. During
rectoromanoscopy, a pronounced inflammatory process of the lower intestines
was found,as well as pseudopoli-posal changes in the mucous membrane. In the blood: lake.- 9,8 * 109/ l, Er.- 3,0 * 1012/ l, WsE-52
mm / year. What drug is pathogenetic in the treatment of this disease?
100% Sulfosalazine
0% Motillium
0% Vikasol
0% Linex
0% Creon

1731. A 49-year-old patient was admitted with complaints of severe headache attacks, which
were accompanied by a throbbing sensation in the temples, an increase in blood pressure up to 280/140 mm Hg.
pheochromocytoma is suspected. What is the mechanism of hypertensive crisis in this patient?
100% Increase in catechol concentration-min
0% Increased blood levels of aldosterone
0% Increased plasma renin activity
0% Increased vasopressin release
0% Increased thyroxine release

1732. in order to quickly replenish blood loss, 1000 ml of single-group Rh-compatible


donor blood preserved with sodium citrate was transfused to the patient. At the end of the blood transfusion, the patient
experienced agitation, pallor of the skin, tachycardia, muscle spasms. What
is the first complication to think about?
100% Citrate intoxication
0% Citrate shock
0% Allergic reaction
0% Anaphylactic shock

397

Downloaded from the site - online testing step

0% Pyrogenic reaction

1733. a 24-year-old female patient, emotionally labile in nature,


complains of irritability, depressed mood, palpitations, a feeling of intermittent and
stabbing pain in the heart, and general weakness after her divorce from her husband. Objectively: hyperhidrosis of the palms, Ps-72-78
beats / min, labile, heart - without pathology. The ECG is normal. What is the most likely pathology that determines
such a picture?
100% Neurasthenia
0% Hypochondriac neurosis
0% Obsessive-compulsive disorder
0% Low-grade schizophrenia
0% Depressive neurosis

1734. a 58-year-old patient complains of pain in the left lower limb, which increases when
walking, a feeling of cold and numbness in both feet. He has been ill for 6 years. Objectively: the skin
is pale, dry; hyperkeratosis. On the left shin, the hairline is poorly developed, a positive
symptom of a "furrow". Pulse on the arteries of the foot, popliteal artery is not detected, on the femoral
artery-weakened. On the right limb, the popliteal artery pulsation is preserved. What
is the most likely diagnosis?

https://translate.yandex.com/en/doc 382/540
22:20 ,27.6.2023 �� C : ; 5 B
100% Obliterating atherosclerosis of the lower limb arteries
0% Obliterating endarteritis
0% Femoral artery thrombosis
0% Raynaud's disease
0% Burger's disease

1735. a 30-year-old woman developed tonic convulsions after an emotional shock, which changed to
clonic convulsions with fatigue, accompanied by the appearance of foam from the mouth, and involuntary
urination. No focal symptoms were detected. AT - 120/60 mm Hg. what is the most
likely diagnosis?
100% Epilepsy
0% Neurasthenia
0% Eclampsia
0% Syncopal state
0% Stroke

1736.during the medical examination, the doctor drew attention to a student with a height of 162 cm and
a body weight of 59 kg, who complained that in the evening she loses the ability to clearly see
objects that surround her. Objectively, there is dry skin, hyperkeratosis. The
following content of essential vitamins was found in the student's daily diet: Vitamin A-0.5 mg, Vitamin
B and -2.0 mg, Vitamin B2-2.5 mg, Vitamin B6-2 mg, Vitamin C-70 mg. Determine the type of hypovitaminosis:
100% A-hypovitaminosis
0% B ^ hypovitaminosis
0% B2-hypovitaminosis
0% B6-hypovitaminosis
0% C-hypovitaminosis

398

Downloaded from the site - online testing step

1737. A 25-year-old repeat mother was admitted in the first stage of labor with
moderate contractions. This pregnancy is the third. In the anamnesis - Cesarean section for a clinically narrow
pelvis and 1 artificial abortion. Suddenly, Rodilli developed severe abdominal pain, weakness; blood pressure
dropped to 90/50 mm Hg. moderate spotting appeared from the vagina. The fetal heartbeat
is not detected. Labor activity stopped. Most likely diagnosis:
100% Uterine rupture
0% Premature detachment of the normally located placenta
0% Amniotic fluid embolism
0% Inferior vena cava syndrome
0% Placenta previa

1738. when determining the adaptation of 1st - grade students to school, it was found that they
have a low rate of information processing in the classroom, the predominance of inhibition processes over
arousal. What is the degree of adaptation of these students?
100% Tense
0% Satisfactory
0% Good
0% Unsatisfactory
0% Slow motion

1739.during the vaginal examination, the fetal head is determined, performing the posterior surface
https://translate.yandex.com/en/doc 383/540
22:20 ,27.6.2023 �� C : ; 5 B
of the pubic joint and the sacral cavity, palpation is available to the lower edge of the pubic
joint, the awns of the sciatic bones, the sacrococcygeal joint. In what plane of the small
pelvis is the fetal head located?
100% In the plane of the narrow part of the pelvic cavity
0% In the plane of the broad part of the pelvic cavity
0% Above the entrance to the pelvis
0% In the plane of the entrance to the small pelvis
0% In the plane of the exit from the small pelvis

1740. a 42-year-old patient developed swelling, redness, and a local increase in skin temperature on the 5th day after falling on broken
glass in the area of raising the first
finger of the right upper limb
. The indicated area is painful during movement and palpation, characterized
by fluctuation
100% phenomena. What treatment
Phlegmon disclosure, should
drainage, I use?
antibacterial therapy
0% General antibacterial therapy
0% UHF therapy
0% Puncture of the suppuration site with the introduction of proteolytic enzymes
0% Semi-alcohol compresses and antibiotic therapy

1741. on the land plot of a rural district hospital with 50 beds, the following zones are allocated:
a medical building for non-infectious patients, a medical building for infectious patients
with separate garden and park areas, as well as an economic zone where
the pathology department is located. What violation was made when planning the site?
100% Combination of economic and pathoanatomic zones

399

Downloaded from the site - online testing step

0% Department of Garden and Park Areas


0% Location of the infectious diseases department on the hospital site
0% Delineation of zones for infectious and non-infectious patients
0% Location of the pathology department on the hospital site

1742. in a 30-year-old patient with deep burns up to 30% of the body surface
, constant fever, lack of appetite, and night sweats are detected after 30 days. The burn surface is sluggishly granulated.
Determine the stage of burn disease:
100% Septicotoxemia
0% Primary burn shock
0% Secondary burn shock
0% Acute burn toxemia
0% Recovery

1743. a 50-year-old patient complains of fog in front of the left eye, iridescent circles,headache,
and nausea. Such complaints have already occurred twice, but gradually all the symptoms passed, vision
was restored. Now all the symptoms last more than 2 days. Objectively: edema of the eyelids, stagnant
injection, cornea dull, small anterior chamber, pupil wide, does not respond to light.
Palpation (T+3). What is the most likely diagnosis?
100% Prolonged acute attack of glaucoma
0% Iridocyclite
0% Keratitis
0% Irit
https://translate.yandex.com/en/doc 384/540
22:20 ,27.6.2023 �� C : ; 5 B
0% Cyclite

1744. a patient complains of unpleasant sensations that occur in various parts of the body and
internal organs, and sometimes "very painful sensations of tightening, twisting, turning over".
She repeatedly turned to doctors, was examined in a therapeutic clinic, and then
sent to a psychiatrist. What is the most likely psychopathological disorder?
100% Senestopathies
0% Synesthesia
0% Parasthesia
0% Hypochondria
0% Depersonalization

1745. a 33-year-old patient with stopped recurrent ulcerative


bleeding was admitted to the hospital. During the examination, emaciated, pale. In the blood: HB-77 g / l, Ht-0.25. due to
the presence of anemia, a single transfusion was attempted twice-group A (II)DL,+ blood. Both times
the transfusion was stopped due to the development of an anaphylactic reaction. Transfusion of what
transfusion medium is desirable in this case?
100% Washed red blood cells
0% Fresh Blood Citrate
0% Red blood cell mass (native)
0% Erythrocyte suspension

400

Downloaded from the site - online testing step

0% Red blood cell mass, which is poor in white blood cells and platelets

1746. a 19-year-old boy who received a closed abdominal injury was admitted to a surgical hospital
. During the operation, numerous ruptures of the spleen and small intestine were found. Blood pressure gradually decreases.
There was a need for blood transfusion. Who can
determine the victim's blood type and Rh status?
100% Doctor of any specialty
0% Laboratory assistant
0% Surgeon
0% Traumatologist
0% Doctor-anesthesiologist

1747. A 75-year-old patient with a perforated gastric ulcer was admitted to the surgical department
. 2 weeks earlier, the patient had suffered a myocardial infarction, with a history
of acute cerebrovascular accident. The patient and his relatives categorically refuse the operation. What
are the doctor's tactics in this situation?
100% The Taylor method
0% Infusion therapy
0% Antibacterial treatment
0% Forced diuresis method
0% Hyperbaric oxygenation method

1748. a 54-year-old patient complains of shortness of breath during minor physical exertion,
cough with severe sputum discharge. Objectively: diffuse cyanosis. The chest
is barrel-shaped. In the lungs, vesicular respiration is weakened with an extended exhalation, dry
https://translate.yandex.com/en/doc 385/540
22:20 ,27.6.2023 �� C : ; 5 B
wheezing. AT-140/80 mm Hg, Ps-92 beats / min., rhythmic. Spirography: core/Jel-65%,
FEV1 / FVC-50%. Determine the type of respiratory failure (DN) in the patient:
100% Mixed type of DN with a predominance of obstruction
0% Restrictive DN type
0% Obstructive type of DN
0% Mixed type of DN with a predominance of restriction
0% No respiratory failure

1749. A 38-year-old patient developed pain in the lumbar spine with


radiating along the back of the left leg after lifting the load. The pain increases when changing the position of the body and in
an upright position. During the examination, positive symptoms of tension were revealed. What
is the preliminary diagnosis?
100% Pathology of intervertebral discs
0% Spinal cord tumor
0% Arachnomyelitis
0% Polyneuritis
0% Myelopathy

1750. a 36-year-old female patient came to the antenatal clinic complaining of high blood pressure.

401

Downloaded from the site - online testing step

irritability, tearfulness, headache, palpitations, swelling of the hands and feet, decreased
urination, engorgement of the mammary glands. These symptoms appear and gradually increase a
few days before menstruation and disappear with its onset. Menstrual cycle without violations.
I started noticing these complaints during the last year. What is the most likely
diagnosis?
100% Premenstrual syndrome
0% Sheehan's syndrome
0% Stein-Leventhal syndrome
0% Adrenogenital syndrome
0% Menopausal syndrome

1751. a 50-year-old patient was removed from fresh water with a real drowning clinic:
agitation, skin cyanosis, rapid breathing, respiratory tract noises can be heard at a distance, coughing,
tachycardia, hypotension, vomiting. What is the main priority action?
100% Immediately release the airway
0% Enter subcutaneously cordiamine to excite the respiratory center
0% Start massive infusion therapy
0% Start artificial respiration and closed heart massage
0% Ensure that oxygen is inhaled

1752. A 7-month-old child is discharged from a surgical hospital after conservative


treatment of ileocecal intussusception. What recommendations should the mother's doctor give to
prevent relapse of this disease?
100% Strict adherence to the child's feeding regimen
0% Prevention of colds
0% Observation of the nature of the child's stool
0% Prevention of gastrointestinal diseases

https://translate.yandex.com/en/doc 386/540
22:20 ,27.6.2023 �� C : ; 5 B
0% Tempering the child

1753. the patient developed swelling of the lower legs, face and eyelids, increased blood pressure to 160/100 mm
Hg, Ps-54 beats / min., daily loss of protein in the urine-4 g. what therapy is pathogenetic in this
case?
100% Corticosteroids
0% Diuretics
0% NSAIDs
0% Calcium antagonists
0% Antibiotics

1754. a 34-year-old woman who had 4 pregnancies, without a


history of gynecological diseases, experienced uterine bleeding on the 17th day of the menstrual cycle. Which survey
is most important in this basin?
100% Diagnostic curettage of the uterine cavity
0% Determining the thyroid profile
0% Hysterosalpingography

402

Downloaded from the site - online testing step

0% Colpocytology
0% Colposcopy

1755. a 3-month-old boy is fed cow's milk. Well gaining body weight,
prevention of rickets with vitamin D was not carried out. The child sweats, the back of the head is flattened,
craniotabes, the hair on the back of the head is rolled out, does not sleep well. The blood has a high level of alkaline
phosphatase. What is the most likely diagnosis?
100% Rickets is mild, at its height, acute
0% Rickets is mild, initial period, acute course
0% Rickets is mild, ignition period, hot.
0% Hypervitominosis D
0% Rickets of moderate severity, ignition period, acute course

1756. after lifting a heavy bag, a patient suddenly developed acute pain in the lower back. Movement in
the spine is limited. The Achilles reflex on the left side is not caused, pain
sensitivity anesthesia has appeared on the outer surface of the left shin. What disease will you suspect?
100% Lumbosacral sciatica
0% Lumbago
0% Lumbalgia
0% Neuritis of the femoral nerve
0% Spinal arachnoiditis

1757. Purulent meningitis was diagnosed in a patient with severe meningeal syndrome, petechial skin rashes,
chills, body temperature of 39°C, inflammatory changes in peripheral blood and neutrophil
pleocytosis in the cerebrospinal fluid. Which of the existing syndromes in a patient is
crucial for making a diagnosis of meningitis?
100% Neutrophilic pleocytosis
0% Petechial skin rashes
0% Meningeal syndrome
https://translate.yandex.com/en/doc 387/540
22:20 ,27.6.2023 �� C : ; 5 B
0% Increased body temperature
0% Inflammatory changes in the blood

1758. A 31-year-old patient developed a sharp chill, a temperature of 39°C, which lasted for 9 hours and
suddenly dropped to normal with heavy sweating. The next day the temperature
is 36.8°C, Ps - 70/min. The abdomen is soft, not painful. Liver + 2 cm, spleen + 3 cm. A year ago
, I returned from South Asia, where I had several bouts of fever. What
is the best study to conduct to quickly confirm the diagnosis?
100% Testing a thick drop of blood for malaria
0% Blood test for sterility
0% Bacterioscopy of a thick drop of blood for meningococcus
0% Blood test for anti-HEV IgM
0% Bacterioscopy of a thick drop of blood for leptospirosis

1759. a 37-year-old patient developed a spot 2 days after the cattle incision

403

Downloaded from the site - online testing step

on the hand, which turned into a black-bottomed pustule within a day, painless when touched, with
a crown of daughter vesicles on the periphery. There is a painless swelling on the arm and shoulder.
The body temperature increased to 39° Ps - 100/min, AT - 95/60 mm Hg, BH-30 / min. What is the most
likely diagnosis?
100% Anthrax
0% The plague
0% Tularemia
0% Brucellosis
0% Herpes zoster

1760. a 17-year-old boy at the draft board complained of tinnitus, which increases
with physical exertion. AT-150/30 mm Hg, diastolic murmur is heard over
the aorta. An overview radiograph shows a shadow of the aortic heart, enlarged aortic arch and
left ventricle. The lung pattern has not been changed. Aortic pulsation is increased. What is the most
likely diagnosis?
100% Aortic insufficiency
0% Aortic atherosclerosis
0% Hypertension
0% Aortic dextraposition
0% Aortic coarctation

1761. the child is 5 years old. He is currently registered with a cardiologist for a
congenital heart defect. The mother went to the doctor with complaints of shortness of breath during physical
exertion and at rest, unstable swelling of the legs and feet, which at the end of the day increases and
disappears after a night's sleep. From the medical history, it is known that 3 weeks ago the child had a sore throat.
Specify the origin of peripheral edema:
100% Cordial
0% Orthostatic
0% Cirrhotic
0% Kidney disease
0% Due to thrombophlebitis

https://translate.yandex.com/en/doc 388/540
22:20 ,27.6.2023 �� C : ; 5 B

1762. A 42-year-old patient complains of persistent epigastric pain radiating to the


back, nausea, vomiting, and heartburn. He has been suffering from duodenum peptic ulcer for 20 years.
Objectively: low nutrition, tongue covered, wet. The abdomen is tense and painful
on palpation in the pyloroduodenal zone. Positive symptom of Obraztsov. Subfebrility. In
the blood: lake.- 10,0 • 109/l, WSE-24 mm / hour. Radiologically: a
0.7 x 1.0 cm three-layer niche is submerged in the duodenum bulb. Gregersen's reaction is negative. What is the complication in this
case?
100% Penetration
0% Perforation
0% Bleeding
0% Malignization
0% Pyloric stenosis

404

Downloaded from the site - online testing step

1763. a 55-year-old man complains of general weakness, decreased urination,


and itching of the skin. He has been suffering from chronic pyelonephritis for 15 years. Objectively:
the skin is dry, with a yellowish tinge. Ps-80 / min, rhythmic, blood pressure-100/70 mm Hg. during
auscultation, heart tones are deaf, pericardial friction noise is heard. Blood creatinine -1.1
mmol / l, glomerular filtration 5 ml / min. What treatment is indicated for the patient?
100% Hemodialysis
0% Plasmapheresis
0% Neohemodez
0% Enterosorbent
0% Diuretics

1764. a child was born due to physiological labor on time. On the second day, the child developed
icteric skin and mucous membranes. Indirect bilirubin was equal to 136 mmol/l. The
mother has blood type 0(1) Rh-, the child has A(II)Rh+. What is the mechanism of jaundice?
100% Hemolysis of red blood cells
0% Cholestasis
0% Hepatitis
0% Violation of the outflow of bile
0% Violation of bilirubin metabolism

1765. the patient 4 days ago developed a rash on the skin, accompanied by itching. Bothered
by bloating, dull pain in the right hypochondrium, constipation. The day before, I ate smoked meat.
Previously, similar reactions appeared after consuming a significant amount of tomatoes, strawberries, and chocolate.
Objectively: there is an urticarial rash on the skin of the face, torso, and limbs. The total IgE level is
normal. What is the most likely diagnosis?
100% Pseudoallergy
0% Idiosyncrasy
0% Food allergies
0% Atopic dermatitis
0% Chronic urticaria

1766.during a conversation with the doctor, the patient said: "every night when I fall asleep
, the"messenger of death" comes to me, I pass by her and pretend that I don't see her. Then she sneaks

https://translate.yandex.com/en/doc 389/540
22:20 ,27.6.2023 �� C : ; 5 B
up to my bed with an axe, the axe glinting and her eyes blazing with rage. This happens many
times a night. Determine the patient's condition:
100% Hypnagogic hallucinations
0% Pseudo-hallucinations
0% Violent memories
0% Menthism
0% Pathological affect

1767. a 57 - year-old woman with third-century obesity. Two months ago, I was diagnosed with diabetes
mellitus. The endocrinologist of the patient recommended a subcaloric diet and dosed physical
activity. Fasting blood glucose level is 9.2 mmol / l. what hypoglycemic drug can
be recommended for the patient?

405

Downloaded from the site - online testing step

100% Metformin
0% Glibenclamide
0% Gliquidone
0% Glipizid
0% Repaglinide

1768. A 73-year-old female patient complains of weakness, drowsiness, chilliness, severe


memory loss, hair loss, constipation, and edema. Objectively: normal nutrition. The skin
is dry and yellowish. The face and limbs are swollen, and when pressed, no fossa remains. Heart
tones are muted, bradycardia. The size of the heart is expanded. The volume of the thyroid gland
is reduced. In the blood: HB - 85 g / l, cholesterol-8.5 mmol / l, TSH-20.5 mmol/l. what is the preliminary
diagnosis?
100% Hypothyroidism
0% Cardiosclerosis, heart failure
0% Chronic hepatitis
0% Kidney failure
0% Severe atherosclerosis of the cerebral vessels

1769.A 20-year-old patient with polyarthritis was admitted to the rheumatology department. On
the skin of the face - redness in the form of a"butterfly". In the urine-protein 4,8 g / l. the doctor suspected
the presence of systemic lupus erythematosus in the patient. What additional research method is
the most informative for verifying the diagnosis?
100% Blood test for antinuclear antibodies
0% General blood test
0% Immunological blood testing
0% LE-cell blood test
0% Blood test for rheumatoid factor

1770. a 19-year-old patient complains of itching in the interdigital folds of the hands,
abdominal skin, which increases in the evening. Gets sick for 2 weeks. Objectively: in the interdigital
folds of the hands, on the skin of the abdomen,a large number of papulovesicles,
uncovers, hemorrhagic crusts placed in pairs. Sarcoptes scabiei hominis was found in the scrapings. What is the most
likely diagnosis?
100% Scabies
0% Microbial eczema

https://translate.yandex.com/en/doc 390/540
22:20 ,27.6.2023 �� C : ; 5 B
0% Neurodermatitis
0% Contact dermatitis
0% Lichen planus erythematosus

1771. a 1-month-old boy vomits in a fountain after each feeding.


The vomit masses are fermented milk and exceed the volume of pre
-feeding. The child gained 200 g in weight in the first month. Urination is liquid, bowel
movements are scanty, irregular. What examination method will you use to verify the diagnosis?
100% Gastrofibroscopic examination
0% Ultrasound examination

406

Downloaded from the site - online testing step

0% Biochemical research
0% X-ray of the abdominal cavity
0% Coprological research

1772. a 39-year-old patient has been suffering from diabetes mellitus for 10 years. The last year marks
cold toes, pain and numbness. Objectively: the skin of the lower extremities is dry,
elegant, cold to the touch, pulsation on the femoral and popliteal arteries is preserved. What
is the most likely diagnosis?
100% Diabetic vascular microangiopathy of the lower extremities
0% Diabetic macroangiopathy of the vessels of the lower extremities
0% Raynaud's disease
0% Obliterating atherosclerosis of the vessels of the lower extremities
0% Obliterating endarteritis of the vessels of the lower extremities

1773. a 48-year-old patient, after consuming 700 ml of vodka and a large amount of food, developed
unrestrained vomiting, intense pain behind the sternum and in the epigastric region radiating to the
spine, and shortness of breath. Lost consciousness. Objectively: BH-28 / min. Ps-54 / min. AT-80/50 mm Hg
subcutaneous crepitation in the neck, cyanosis of the face. Heart tones are weakened. Respiration
is vesicular on both sides. Muscle tension in the epigastric region. What can be
the cause of the patient's condition?
100% Esophageal rupture
0% Acute myocardial infarction
0% Pneumothorax
0% Dissecting aortic aneurysm
0% Pulmonary embolism

1774. A 24-year-old patient went to the doctor complaining of enlarged submandibular


lymph nodes. Objectively: enlarged submandibular, axillary and inguinal lymph nodes.
Chest X-ray shows enlarged mediastinal lymph nodes. In the blood: Er.- 3,4 •
1012/ l, HB-100 g / l, CP-0.88, tr. -190 * 109/l, lake.- 7 • 5 * 109/l, E.-8%, P.-2%, C.-67%,
lymph. 23%, ESR-22 mm/h. What research is indicated to verify the cause of lymphadenopathy?
100% Open lymph node biopsy
0% Ultrasound examination of the abdominal organs
0% Tomography of the mediastinum
0% Puncture biopsy of lymph nodes
0% Sternal puncture

https://translate.yandex.com/en/doc 391/540
22:20 ,27.6.2023 �� C : ; 5 B
1775. a 32-year-old patient complained of heart failure, dizziness,
shortness of breath during physical exertion. I haven't been ill yet. Objectively: Ps-74 / min,
rhythmic. AT - 130/80 mm Hgon auscultation: first tone of normal sonority,
systolic murmur over the aorta. On the ECG: left ventricular hypertrophy, signs
of repolarization disorders in the I, V5, V6 leads. Echocardiography: 2 cm interventricular septum. What is the most
likely diagnosis?
100% Hypertrophic cardiomyopathy
0% Aortic stenosis

407

Downloaded from the site - online testing step

0% Hypertension
0% Myocardial infarction
0% Aortic coarctation

1776. a 6 - year-old girl on the background of GDV! there were changes in the urine analysis: traces of protein, white blood cells-
30-40 in p/s, red blood cells (fresh) -10-12 in p/s. AT - 100/60 mm Hg. which of the above diagnoses
is most likely?
100% Urinary system infection
0% Acute glomerulonephritis
0% Hemorrhagic vasculitis
0% Vulvovaginitis
0% Urolithiasis

1777. A 47-year-old woman was admitted to the infectious diseases department with complaints of high
fever, chills, severe weakness, headache, nausea and vomiting. The stool is frequent and loose.
The disease started 12 hours after eating a duck egg omelet. What
is the most likely diagnosis?
100% Salmonellosis
0% Food poisoning with Staphylococcus
0% Botulism
0% B. cereus food poisoning
0% Food poisoning caused by V. Parahaemolyticus

1778. In the morning, the patient experienced nausea, single vomiting, and dry mouth. In the evening, I noticed
double vision, " fog " in front of my eyes, and choking when swallowing food. Objectively:
temperature 36.4°C, ptosis, mydriasis, anisocoria, lack of gag and swallowing reflexes,
dry mucous membranes. No abnormalities were detected on the part of the internal organs. What is the most
likely medical condition?
100% Botulism
0% Acute cerebrovascular accident
0% Viral meningoencephalitis
0% Tuberculosis meningoencephalitis
0% Stem encephalitis

1779. a 16-year-old patient suffering from severe bleeding with small cuts
and wounds had a question about the need to extract the roots of the teeth. Objectively: an increase in
the volume of the right knee joint, limited mobility. There are no other changes. In the blood: tendency
to anemia (Hb-120 g / l). What should be done to prevent bleeding before
the dentist's intervention?

https://translate.yandex.com/en/doc 392/540
22:20 ,27.6.2023 �� C : ; 5 B
100% Cryoprecipitate
0% Epsilon-Hydrocaproic acid
0% Fibrinogen
0% Dry blood plasma
0% Infusion of calcium chloride
1780. a 40-year-old patient complains of pain in the left hip joint, which increases under pressure.

408

Downloaded from the site - online testing step

walking time, temperature increase to 37.7°C for 2 months. In the anamnesis: focal
tuberculosis of the lungs. On the radiograph of the left hip joint, destruction
of the adjacent surfaces of the femoral head and roof of the acetabular cavity, narrowing of the joint
gap are determined. Mantoux test z 2 TO - papule 14 mm. Which of these diagnoses is most likely?
100% Tuberculosis coxitis on the left
0% Sarcoma of the left hip joint
0% Osteoarthritis of the left hip joint
0% Rheumatoid arthritis
0% Rheumatic arthritis

1781. a 30-year-old patient, a smoker, developed a sharp pain in the right side
of the chest, shortness of breath at rest, and a sharp general weakness during a coughing attack. Suspected pneumothorax. Above
the right half of the chest, tympanitis, no respiration. BH-36 / min, Ps-120 / min, rhythmic,
blood pressure-90/60 mm Hg. in the pathogenesis of this condition, the leading role belongs to such a factor:
100% Sharp increase in intrapleural pressure
0% Increased pressure in the small circulatory system
0% Reduced left ventricular contractility
0% Mechanical blockage of the pulmonary artery branches
0% Small coronary artery thrombosis

1782. a 34-year-old patient became acutely ill. Complaints of weakness, lethargy, tightness in
the chest, difficulty breathing. The patient has a paradoxical pulse, overflow of the neck veins. What
is the preliminary diagnosis?
100% Acute pericarditis
0% Acute pneumonia
0% Tuberculous lobitis
0% Lung tumor
0% Myocardial infarction

1783. the district center is not drained, sewage is collected in the cesspools of the yard
latrines. To neutralize the sewage of the district center, you should apply:
100% Sanitation fields
0% Irrigation fields
0% Filter fields
0% Biological bids
0% Agricultural irrigation fields

1784. A 40-year-old homosexual patient complained of prolonged fever, night


sweats, severe weight loss, muscle, throat, and joint pain, and recurrent diarrhea.
Objectively: generalized lymphadenopathy. What disease should be suspected?
100% AIDS

https://translate.yandex.com/en/doc 393/540
22:20 ,27.6.2023 �� C : ; 5 B
0% Lymphogranulomatosis
0% Sepsis
0% Tuberculosis

409

Downloaded from the site - online testing step

0% Lymphocytic leukemia

1785. a 42-year-old patient suddenly, on the background of full health, felt pain like a "blow to the
head", after which he developed vomiting, lost consciousness, and generalized convulsions. Objectively:
coma and stiffness of the occipital muscles, the left pupil is larger in size than the right, the right limbs
are rotated, muscle tone is significantly reduced. What is the most likely diagnosis?
100% Ruptured brain aneurysm
0% Meningitis
0% Epilepsy
0% Ischemic stroke
0% Intracranial hematoma

1786. a 13-year-old girl complains for 5 years of pain in the right hypochondrium
radiating to the right scapula, attacks of pain are associated with a violation of diet, they are short-lived, easily
relieved by antispasmodic agents. During an attack of pain, palpation of the abdomen is painful,
as much as possible at the point of projection of the gallbladder. The patient is most likely to have:
100% Biliary dyskinesia
0% Chronic cholecystitis
0% Chronic gastroduodenitis
0% Chronic pancreatitis
0% Duodenal ulcer disease 12

1787. A 48-year-old patient experienced


a sharp abdominal pain during esophageal augmentation due to cicatricial stricture. Previously, he was ill with duodenal ulcer of the 12th
duodenum. On examination: the abdomen
is sharply tense, painful; Ps-110 / min. Painful swallowing of saliva. Pallor of the skin.
What100%
is the mostPerforation
likely diagnosis?
of the abdominal esophagus
0% Acute myocardial infarction
0% Pinching of the diaphragmatic keel
0% Perforation of duodenal ulcer 12
0% Mesenteric vascular thrombosis

1788. a patient complains of paroxysmal pain in the right hypochondrium radiating to the right shoulder,
which occurs after eating fried food. Periodically notes an increase in body temperature
to subfebrile numbers. He has been ill for about 5 years. What examinations should the patient conduct
first?
100% Ultrasound of the abdominal cavity
0% Cholecystography
0% Duodenal probing
0% Overview radiography
0% Fibrogastroduodenoscopy

1789. during the last 6 months, a 23-year-old patient became withdrawn, avoided communication with
https://translate.yandex.com/en/doc 394/540
22:20 ,27.6.2023 �� C : ; 5 B
others, went into seclusion, and talked to himself. When talking to a psychiatrist

410

Downloaded from the site - online testing step

at first, he hid his feelings, but then he told about the special, unreal voices
contained in himself ("nested in the brain"). What are these perceptual disorders?
100% Pseudo-hallucinations
0% Illusions
0% The hallucinations are real
0% Metamorphopsies
0% Derealization

1790. in a 16-year-old boy with no clinical symptoms, cardiac auscultation revealed


an accent of tone II and systolic murmur on the pulmonary artery. The heart tones are sonorous and rhythmic. What
is the most likely diagnosis?
100% Functional noise
0% Stenosis of the pulmonary artery mouth
0% Pulmonary artery valve insufficiency
0% Non-infection of the botal duct
0% Atrial septal defect

1791. A 10-year-old girl was admitted to the department with symptoms of carditis. In the anamnesis: two weeks
ago there was an exacerbation of chronic tonsillitis. What etiological factor of carditis is most
likely in this case?
100% Streptococcus
0% Staphylococcus aureus
0% Pneumococcus
0% Klebsiela
0% Proteus

1792. A 3-year-old child with a body weight deficit has a persistent wet cough. In
the anamnesis of several previous pneumonias, which occur with the phenomena of obstruction. Objectively:
the chest is swollen, over the lungs there is a shortening of percussion sound in the lower parts,
auscultation-a large number of different-sized wet wheezes. The level of sweat chlorides is 80
mmol / l. Please indicate a preliminary diagnosis:
100% Cystic fibrosis
0% Bronchial asthma
0% Recurrent bronchitis
0% Bronchiectasis
0% Hypoplasia of the lungs

1793. a 1-year-old boy developed swelling and soreness


of the left ankle joint after falling on a walk. Previously, after minor injuries, significant
bruising was noted. The duration of Duke bleeding is 3 minutes. Lee-White blood coagulation - 24
minutes. What disease can be assumed in a boy?
100% Hemophilia
0% JURASSIC, mostly articular form
0% Hemorrhagic vasculitis, articular form

https://translate.yandex.com/en/doc 395/540
22:20 ,27.6.2023 �� C : ; 5 B
411

Downloaded from the site - online testing step

0% Idiopathic thrombocytopenic purpura, acute course


0% Idiopathic thrombocytopenic purpura, chronic course

1794. milk samples were taken at a dairy farm for the content of the organochlorine pesticide
lindane. The batch size is 1000 liters. In the toxicological laboratory of the SES, it was found that the content
of lindane exceeds the maximum permissible levels by 4 times. Give a conclusion about the quality
of milk:
100% Poor quality
0% Good quality
0% Conditionally acceptable
0% Falsified information
0% Biologically valuable product

1795. when conducting a glucose tolerance test for a 13-year-old child with
grade III obesity, it was established: fasting blood sugar - 5.4 mmol/l, 1 hour after carbohydrate
loading - 10 mmol/l, 2 hours later-7.8 mmol/l. what measures should be taken to
normalize carbohydrate metabolism?
100% Diet, exercise regime
0% Sugar-lowering herb collections
0% Biguanide-type preparations
0% Sulfonamide preparations
0% Insulin

1796. A 10-year-old boy developed acute signs


of drug allergy in the form of generalized urticaria and massive angioedema due to the use of erythromycin.
The child's condition is serious. What medications should I start intensive care with?
100% Glucocorticoids
0% Antihistamines
0% Diuretics
0% Sedatives
0% Calcium Gluconate

1797. primiparous 30 years old. Head in the pelvic cavity. The fetal heartbeat began
to slow down, and an arrhythmia appeared. What to do?
100% Abdominal obstetric forceps
0% Weekend Obstetric Forceps
0% Caesarean section
0% Perineotomy
0% Skin-head forceps

1798. in the district, the prevalence of diseases among the population was 1156 cases per
1000 population. Which of the following indicators characterizes the budget distribution of
diseases?
100% Intensive

412

https://translate.yandex.com/en/doc 396/540
22:20 ,27.6.2023 �� C : ; 5 B

Downloaded from the site - online testing step

0% Ratio
0% Extensive
0% Standardized
0% For visibility purposes

1799. in a newborn with suspected intracranial birth trauma, a


lumbar puncture was performed. A bloody cerebrospinal fluid was obtained. What kind of hemorrhage occurs in this
case?
100% Subarachnoid
0% Cephalohematoma
0% Epidural
0% Supratentorial
0% Subtentorial

1800. A 4-year-old boy was admitted to the hospital with complaints of shortness of breath, rapid
fatigue. A history of frequent respiratory diseases. Percussion: the heart borders
are extended to the left and up. Auscultation: amplification of the II tone above the pulmonary artery,in the II-III
intercostal space to the left of the sternum,a rough systolic "machine"noise is heard,
conducted to all other points and to the back. What is the most likely diagnosis?
100% Open Ductus arteriosus
0% Ventricular septal defect
0% Isolated stenosis of the pulmonary artery opening
0% Atrial septal defect
0% Valvular aortic stenosis

1801. A 42-year-old patient complains of palpitations, constant weakness, pain in the heart
area radiating to the left shoulder and increasing with breathing, and an increase in body temperature up to 38.5 ° C.
Objectively: AT-105/50 mm Hg, Ps-120 / min. In the zone of absolute dullness of the heart
, a two-phase noise is heard. In the lungs, breathing is weakened in the lower parts. On the ECG-concordant
rise of the ST segment in V2-V6. What is the most likely diagnosis?
100% Acute pericarditis
0% Myocardial infarction
0% Lung infarction
0% Dressler's syndrome
0% Extrahospital pneumonia

1802. A 20-year-old patient complains of severe headache, double vision, general


weakness, fever, irritability. Objectively: the body temperature is 38.1°C, it comes into
contact reluctantly, reacts painfully to stimuli. Ptosis of the left eyelid, divergent strabismus,
anisocoria S > D pronounced meningeal syndrome. During lumbar puncture, the cerebrospinal fluid flowed out under
a pressure of 300 mm water. st., transparent, with a slight opalescence, a fibrinous film fell out after a day
. Protein -1.4 g / l, lymphocytes-600/3 in mm3, sugar-0.3 mmol / l. what is the preliminary diagnosis?
100% Tuberculosis meningitis
0% Meningococcal meningitis
0% Armstrong lymphocytic meningitis

413

https://translate.yandex.com/en/doc 397/540
22:20 ,27.6.2023 �� C : ; 5 B
Downloaded from the site - online testing step

0% Syphilitic meningitis
0% Mumps meningitis

1803. in a patient with symptoms of malnutrition, steatorrhea, 4 hours after eating


, abdominal pain occurs, especially above the navel and to the left. Diarrhea can be replaced by constipation for up to 3-5 days.
Palpation: moderate pain in the choledochopancreatic zone. The level of amylase in the blood does not
increase. Ultrasound revealed calcifications located in the area of the pancreas.
What is the preliminary diagnosis?
100% Chronic pancreatitis
0% Chronic gastroduodenitis
0% Duodenal ulcer 12
0% Zollinger-Ellison syndrome
0% Chronic calculous cholecystitis

1804. a 54-year-old patient complains of frequent painful urination, chills, and an increase
in body temperature up to 38°C. In the urine: protein -0.33 g / l, white blood cells up to 50-60 at p/s, red blood cells-5-8 at
p/ s, gram-negative rods. Which of the presented antibacterial drugs is best suited in
this case?
100% Ciprofloxacin
0% Oxacillin
0% Erythromycin
0% Tetracycline
0% Tseporin

1805.convalescents after
infectious diseases, often and long-term ill people, and people with chronic pathology are registered at the family doctor's dispensary.
Which of
these100%
patients should
Peoplebesuffering
assignedfrom
to thechronic
third health group?
diseases
0% Those who are often and long-term ill
0% Chronic pathology and those who are often and long-term ill
0% Recovering from infectious diseases and people with chronic pathology
0% All categories of patients listed in the terms and conditions

1806. A 62-year-old patient complained of periodic discharge of vermiform


blood clots in the urine. In the right half of the abdomen,a bumpy,
painless, mobile formation is detected during palpation. Which of these survey methods should
be used first?
100% Ultrasound of the kidneys and retroperitoneal space
0% Chromocystoscopy
0% Excretory urography
0% Cystoscopy
0% Computed tomography of the pelvis

1807. the chief doctor of the polyclinic gave a task to the general practitioner-family medicine -

414

Downloaded from the site - online testing step

determine the incidence of the nth disease in the adult population. What document will make

https://translate.yandex.com/en/doc 398/540
22:20 ,27.6.2023 �� C : ; 5 B
it possible to determine the pathological lesion of the population?
100% Journal of preventive examinations
0% Ticket for registration of final (updated) diagnoses with the sign (+) and ( -)
0% Ticket for registration of final (updated) diagnoses with a ( + )sign
0% Ticket for registration of final (updated) diagnoses with the ( -)sign
0% Medical card of an outpatient patient

1808. a 68 - year-old man complains of a cough with sputum discharge, which has been bothering him
for several years, hoarseness of voice, general weakness. Lives near
an asbestos processing plant. Objectively: in the lungs on the right - weakened breathing with an extended exhalation,
dry wheezing. On the radiograph: in the area of the root and basal zone, the case is heterogeneous, with
indistinct darkening contours, increased airiness of the lungs. ENT-paresis of the right vocal
cord. What is the most likely diagnosis?
100% Central cancer of the right lung
0% Right-sided basal pneumonia
0% Chronic dust bronchitis
0% Asbestosis
0% Tuberculosis of the lungs

1809. a 58 - year-old woman complains of gratuitous bruising, weakness, bleeding


gums, and dizziness. Objectively: the mucous membranes and skin are pale, with numerous
hemorrhages of various antiquities. The lymph nodes are not enlarged. Ps-100 / min., AT-110/70 mm Hg.
no changes were detected on the part of the internal organs. In the blood: Er.- 3,0-1012/ l, HB-92 g / l, CP-0.9,
anisocytosis, poikilocytosis, leukemia.- 10-109/l, e-2%, p-12%, c-68%, l-11%, m-7%, WZ-12 mm / year.
Additional determination of which laboratory parameter is most appropriate for establishing
a diagnosis?
100% Platelets
0% Reticulocytes
0% Blood clotting time
0% Osmotic resistance of red blood cells
0% Fibrinogen

1810. A 60-year-old patient was admitted to the clinic with complaints of shortness of breath, heaviness in the right
hypochondrium, and abdominal enlargement. The phenomena grew during the year. With auscultation of the heart - the rhythm
of a gallop. Objectively: swelling of the cervical veins, ascites, palpable liver and spleen. What
disease needs to be differentially diagnosed?
100% Constrictive pericarditis
0% Cirrhosis of the liver
0% Lung cancer with pleural growth
0% Chronic pulmonary heart disease
0% Pulmonary embolism

1811. a 50-year-old patient suddenly developed pain in the occipital region, vomiting. Objectively:
sopor, hyperemia of the facial skin, blood pressure-210/120 mm Hg, Ps-60 / min, tense, body temperature

415

Downloaded from the site - online testing step

- 37.8°E. horizontal nystagmus. Reflexes of oral automatism are expressed. Tendon


reflexes are uniform. Stiffness of the occipital muscles, bilateral Kernig's symptom. What
is the preliminary diagnosis?

https://translate.yandex.com/en/doc 399/540
22:20 ,27.6.2023 �� C : ; 5 B
100% Subarachnoid hemorrhage
0% Hemorrhagic parenchymal stroke
0% Subdural hematoma
0% Acute hypertensive encephalopathy
0% Meningococcal meningitis

1812. an 8-year-old child suffering from diabetes mellitus for 3 years was taken to the
department in a state of hyperglycemic coma. The primary dose of insulin should be given
at the rate of:
100% 0.1-0.2 U/kg of body weight per hour
0% 0.05 U / kg of body weight per hour
0% 0.2-0.3 U/kg of body weight per hour
0% 0.3-0.4 U/kg of body weight per hour
0% 0.4-0.5 U/kg of body weight per hour

1813. A 28-year-old patient complains of cutting abdominal pain. I fell ill abruptly 1.5
hours ago, when I felt a sharp pain under my chest. I've never been ill before. Objectively: the skin
is pale, the patient is lying on his back. When you change the position of the body, the pain worsens. P8-70 / min,
rhythmic, AT-100/60 mm Hg. there is no pathology on the part of the heart and lungs. The tongue is dry and clean.
The abdomen is retracted, does not participate in breathing, during palpation it is sharply tense and painful in
all parts. The Shchetkin-Blumberg symptom is positive. What is the most likely diagnosis?
100% Perforated stomach ulcer
0% Acute pancreatitis
0% Acute cholecystitis
0% Acute intestinal obstruction
0% Acute appendicitis

1814. a child is undergoing inpatient treatment for acute staphylococcal


destruction of the right lung. Suddenly there was a sharp pain in the right side of the chest,
shortness of breath, cyanosis. The right side of the chest lags behind in the act of breathing. Percussion
at the bottom right-dullness, in the upper sections - box sound. The boundaries of relative cardiac dullness are shifted
to the left. What complication is most likely to occur in the child?
100% Pyopneumothorax on the right
0% Pleural empyema
0% Spontaneous pneumothorax
0% Exudative pleurisy
0% Right lung abscess

1815.A patient who was diagnosed with acute respiratory infections after examination turned to the family doctor for an outpatient
appointment
. What registration document should a doctor fill out to
register
100% this caseStatistical
of the disease?
ticket for registering the final diagnosis

416

Downloaded from the site - online testing step

0% Medical card of an outpatient patient


0% Sick leave form
0% Emergency notification of an infectious disease
0% Doctor's appointment card

https://translate.yandex.com/en/doc 400/540
22:20 ,27.6.2023 �� C : ; 5 B

1816. a 12-year-old girl notes bleeding from the genital tract for the last 2 weeks, after
a delay in menstruation for 3 months, weakness, headache, dizziness.
Menstruation from the age of 10. Objectively: the skin is pale, tachycardia, blood pressure-100/60 mm Hg in
the blood: Hb-100 g/l, platelets 200 * 109/L. gynecological examination: virgo. During rectal
examination: the body of the uterus and appendages without pathology. What is the most likely diagnosis?
100% Juvenile bleeding
0% Aborted pregnancy
0% Werlhof's disease
0% Sclerocystic ovary syndrome
0% Hemorrhagic diathesis

1817. the child is 1 day old. In childbirth, vyvazhivanie shoulders is difficult. Body weight 4,300 g. The right arm
hangs down along the torso, the hand is new, there are no movements in the hand. Positive symptom of "scarf"
indicate the most likely diagnosis:
100% Total obstetric paralysis on the right
0% Proximal type of obstetric paralysis on the right
0% Distal type of obstetric paralysis on the right
0% Hemiparesis
0% Tetraparesis

1818. to determine the influence of the microclimate on the functional state of the body, the
following physiological indicators of tension in the functions of organs and systems involved in
heat exchange processes were studied: pulse, blood pressure, number of breaths per minute,
skin temperature, sweating rate per minute, speed of visual and auditory reactions. Name
the indicator that most objectively reflects the intensity of the body's thermoregulation processes:
100% Skin temperature
0% Pulse and blood pressure
0% Number of breathing acts per minute
0% Sweating rate per minute
0% Visual and auditory response rate

1819. A 22-year-old woman went to a antenatal clinic for an 11-12-week pregnancy.


During the examination, Wasserman was found to have a positive reaction. The dermatologist diagnosed
secondary latent syphilis. What are the tactics of managing this pregnancy?
100% Artificial termination of pregnancy after a course of antisyphilitic therapy
0% Artificial termination of pregnancy before the course of antisyphilitic therapy
0% Urgent termination of pregnancy
0% Prolongation of pregnancy after the first course of antisyphilitic therapy
0% Antisyphilitic treatment three times during pregnancy

417

Downloaded from the site - online testing step

1820. A 36-year-old woman went to a women's clinic with complaints of heavy painful
menstruation," smearing", chocolate-colored vaginal discharge before and after menstruation. I have a
history of 4 abortions. Gynecological status: external genitalia, cervix and vagina are not
changed, uterus in retroflexion, size 9x7x8 cm, limited mobility. The appendices are not changed,
the vaginal arches are free. What is the diagnosis?
100% Endometriosis of the uterine body
0% Uterine fibroids
0% Endometrial cancer
https://translate.yandex.com/en/doc 401/540
22:20 ,27.6.2023 �� C : ; 5 B

0% Dysfunctional bleeding
0% Chronic endometritis

1821. an 18-year-old female patient went to the gynecologist with complaints about the appearance of warty
growths in the genital area. When examining the external genitalia on the labia majora and
labia minora, papillary growths are determined, soft consistency,
painless. Gynecological examination revealed no pathology on the part of the internal genitalia
. What is the preliminary diagnosis?
100% Genital warts
0% Papillomatosis
0% Syphilitic warts
0% Vegetative pemphigus
0% Vulvar cancer

1822. a 14-year-old boy with chronic tonsillitis and sinusitis developed feelings
of heart failure and additional pulse beats. Heart rate-83 / min. On the ECG: after every two
sinus contractions, impulses regularly occur in which the P Wave is absent, QRS is extended
for more than 0.11 seconds, the discordant T wave is sharply deformed, after which a complete
compensatory pause is recorded. Specify the nature of rhythm disturbances:
100% Trigeminal extrasystole
0% Bigeminia-type extrasystole
0% Partial AV blockage
0% Complete AV blockade
0% Left bundle branch block

1823. a two-year - old child coughs for about a month. Over the past 3 weeks, the cough has become
paroxysmal. After a coughing fit, vomiting occurs. Objectively: puffy face.
Right subconjunctival hemorrhage. On the skin of the neck and chest there are single petechiae,
an ulcer on the frenulum of the tongue. What is the most likely diagnosis?
100% Whooping cough
0% Tuberculous bronchoadenitis
0% Spastic bronchitis
0% Hemorrhagic vasculitis
0% MS infection

1824. a full-term baby was born with a body weight of 3200 g, a body length of 50 cm, with
an Apgar score of 8-10 points. What is the optimal time for the first application to the breast?

418

Downloaded from the site - online testing step

100% In the first 30 minutes


0% In the first 6 hours
0% In the first 24 hours
0% In the first 48 hours
0% After 48 hours

1825. after a conversation with the mother of a seven-month-old boy who is on natural
feeding, the pediatrician found out that the child is fed 7 times a day. What number of feedings
is set for a child of this age?

https://translate.yandex.com/en/doc 402/540
22:20 ,27.6.2023 �� C : ; 5 B
100% 5 times
0% 3 times
0% 4 times
0% 6 times
0% 7 times

1826. A 12-year-old boy complains of "hungry" nighttime abdominal pain, constipation, and heartburn. He has been ill
for more than two years. What is the most likely medical condition?
100% Duodenal ulcer disease 12
0% Acute gastritis
0% Chronic gastritis
0% Biliary dyskinesia
0% Peptic ulcer disease of the stomach

1827. a 40-year-old patient suffers from rheumatism. Complains of severe pain in the left eye,
especially at night, blurred vision, photophobia, lacrimation. The disease does not connect with anything.
Objectively: weak pericorneal injection, smoothness of the iris relief, change in its color.
What is the most likely diagnosis?
100% Iridocyclite
0% Irit
0% Keratitis
0% Choroiditis
0% Acute glaucoma attack

1828. A 67 - year-old patient complains of shortness of breath, chest pain, and general weakness. He's been ill for 5
months. Objectively: t° - 37.3°C, Ps-96 / min. Above the right lung, the vocal tremor is not
detected, the percussion sound is dull, and breathing is not listened to. In sputum-admixtures
of blood diffusely mixed with mucus. What is the most likely diagnosis?
100% Lung cancer
0% Bolshegnishchevaya pneumonia
0% Bronchiectasis
0% Focal tuberculosis of the lungs
0% Exudative pleurisy

1829. insufficiently treated waste from an industrial enterprise is discharged into a river, the water from which

419

Downloaded from the site - online testing step

it is used for domestic drinking water supply. This causes the death of some
microorganisms, disruption of water self-purification processes and deterioration of its quality, which can
have a negative impact on people's health. This effect of environmental factors
is called:
100% Indirect
0% Direct line
0% Combined
0% Comprehensive
0% Combined

1830.for a long time, herbicides were used to control weeds on agricultural land

https://translate.yandex.com/en/doc 403/540
22:20 ,27.6.2023 �� C : ; 5 B
, which are classified as persistent in terms of their degree of resistance in the environment.
Indicate the most likely route of their entry from the soil into the human body:
100% Soil-plants-people
0% Soil-microorganisms-humans
0% Soil-animals-humans
0% Soil-protozoa-man
0% Soil-insects-humans

1831. A 51-year-old patient complains of significant spotting from the vagina for 15 days.
From the anamnesis: violation of menstrual function during the year, the patient notes increased
irritability, sleep disturbance. Ultrasound: the uterus meets the age standards, applications without
features, endometrial thickness of 14 mm. What are the doctor's tactics?
100% Diagnostic curettage of the uterine cavity walls
0% Conservative treatment of bleeding
0% Hysterectomy
0% Superpichval amputation of the uterus without superstructures
0% Testing for TORCH infection

1832. an 18-year-old patient complains of soreness and engorgement of the mammary glands, headache,
irritability, and swelling of the lower extremities. These symptoms are disturbing from the beginning of menarche,
they appear 3-4 days before the start of the next menstruation. Gynecological examination
revealed no pathology. What disease does a woman have?
100% Premenstrual syndrome
0% Neurasthenia
0% Kidney diseases
0% Mastopathy
0% Diseases of the cardiovascular system

1833. A 23-year-old patient was taken to the clinic after a road injury. The patient's condition is severe,
conscious, pale skin, Ps-100 / min., AT-90/60 mm Hg, BH-29 / min., breathing is not
heard, pronounced subcutaneous emphysema is determined. Radiologically,the right lung
is collapsed, V-V and right ribs are broken. What are the doctor's tactics?
100% Drainage of the pleural cavity
0% Urgent thoracotomy with suturing of the damaged lung

420

Downloaded from the site - online testing step

0% Performing bronchoscopy
0% Performing intercostal alcohol-novocaine blockade
0% Conducting anti-shock therapy

1834. a 15-year-old patient lags behind in physical development, periodic yellowing of the skin.
Objectively: spleen 16x12x10 cm, cholesi-vekiasis, skin ulcer of the lower third of the left
shin. In the blood: Er.- 3,0 • 1012/ l, HB-90 g / l, CP-1.0, microspherocytosis, reticulocytosis.
Blood bilirubin is 56 mmol/l, indirect-38 mmol/ l. choose a treatment method:
100% Splenectomy
0% Spleen transplant
0% Portocaval anastomosis
0% Omentosplenopexy
0% Omentohepatopexy

https://translate.yandex.com/en/doc 404/540
22:20 ,27.6.2023 �� C : ; 5 B

1835. the girl is 8 months old and was born prematurely. During the examination: suffocation,
tachycardia, hepatosplenomegaly, lag in physical development, cyanosis of the extremities are noted.
Parasternal heart hump is determined
, systolic diastolic murmur is heard in the II intercostal space on the left, blood pressure is 90/0 mm Hg. what disease should I think
about?
100% Open Ductus arteriosus
0% Aortic coarctation
0% Aortic valve stenosis
0% Pulmonary artery stenosis
0% Non-closure of the interventricular septum

1836. an ambulance doctor came to a call to a 76-year-old patient who fell on her right
leg. Complaints of pain in the right hip joint. Violation of the supporting function of the right
lower limb. Objectively: the right lower limb in the external rotation position,
a positive symptom of "stuck heel", pain when loading along the axis on the large swivel.
What is the most likely diagnosis?
100% Right hip neck fracture
0% Bruised right hip joint
0% Posterior and Lower hip dislocation
0% Right-sided coxarthrosis
0% Pearl of the upper third of the right thigh

1837. a patient with frostbite on both feet was taken to the emergency department. What
does the victim need to do?
100% Apply a bandage, enter vasodilators
0% Prescribe heart medications
0% Put your feet in hot water
0% Rub snow on your feet
0% Apply an alcohol compress

1838. on the fourth day after receiving a stab wound to the right foot, the patient's blood pressure increased.

421

Downloaded from the site - online testing step

the body temperature is up to 38oC, the inguinal lymph nodes have increased, become painful, the skin above
them has turned red. What complication of the wound can you think about?
100% Lymphadenitis
0% Lymphangitis
0% Phlegmon
0% Tetanus
0% Erysipelas

1839. A 50-year-old patient complained of an increase in body temperature up to 39°C,


throbbing pain and swelling of the right hand. The day before, I pricked my hand with a fish bone. On
examination, there is edema and sharp pain in the palm and in the area of raising the I finger of the hand. In addition
, there is swelling of the soft tissues of the back of the hand, redness of the skin and sharp pain when
moving the fingers. What is the patient's medical condition?
100% Phlegmon of the right hand
0% Carbuncle of the hand

https://translate.yandex.com/en/doc 405/540
22:20 ,27.6.2023 �� C : ; 5 B
0% Erysipelas of the hand
0% Tendon panaritium of the first finger of the hand
0% Foreign body of the hand

1840. a 22-year-old woman went to a women's clinic on the 15th day after cesarean
section with complaints of pain in the right breast, fever up to 39°C,
and chills. Objectively: the mammary gland is enlarged, hyperemic, compacted, painful during
palpation. The child is breastfed from the 8th day, the milk is expressed irregularly. What is the prevention
of this condition?
Regular breast pumping after feeding, prenatal preparation
100%
of nipples and mammary glands
0% Regulation of the menstrual cycle
0% Regular medical examinations, use of the IUD
0% Expressing both mammary glands after each feeding
0% Conducting professional examinations

1841. A 73-year-old patient was admitted to the clinic on an emergency basis with a pinched femoral
keel. In addition, the patient has varicose veins of the left lower limb, obesity IV B. what
is the most likely complication that may occur in the postoperative period?
100% Pulmonary embolism
0% Sepsis
0% Acute respiratory failure
0% Myocardial infarction
0% Acute pneumonia

1842. A 28-year-old patient was admitted to the intensive care unit with complaints of
abdominal pain, vomiting, and double vision. Objectively: disoriented, pupils dilated, reaction to
light weakened, skin dry, hyperemic, minor cyanosis, blood pressure-90/60 mm Hg, PS-100 / min. From
the medical history, it is known that he consumed alcohol of unknown origin two days before hospitalization.
Specific therapy consists of intravenous administration:

422

Downloaded from the site - online testing step

100% Of ethanol
0% Sodium Bicarbonate
0% 40% glucose solution
0% Thiamine
0% Essentiale

1843. a 50-year-old patient has a stable increase


in jaundice and anemia with periodic rises in body temperature for one and a half months. During palpation
, an enlarged and painless gallbladder was found. The development of what disease can be assumed?
100% Cancer of the head of the pancreas
0% Gallbladder cancer
0% Cholelithiasis
0% Echinococcosis of the liver
0% Primary sclerosing cholangitis

1844. a pregnant woman with a gestation period of 7 weeks was admitted to the hospital in a serious condition with
complaints of vomiting up to 20 times a day, weakness, dizziness, and immediately eating. During
https://translate.yandex.com/en/doc 406/540
22:20 ,27.6.2023 �� C : ; 5 B
pregnancy, the weight decreased by 10 kg. Ps-105 / min, rhythmic, blood pressure-90/60 mm Hg,
body temperature 37.9°C. in the blood: Hb-154 g / l, in the urine acetone (++++). Complex therapy
is ineffective. What are the obstetric tactics?
100% Termination of pregnancy
0% Continue conservative treatment for preeclampsia
0% Apply plasmapheresis in the treatment of patients
0% Transfer the patient to the gastroenterology department
Continue treatment for 1 week, then decide on the possibility
0%
of prolonging the pregnancy

1845. with a gestation period of 32 weeks, the pregnant woman began to give birth prematurely. Contractions in
10-15 minutes, 15-20 seconds each. Fetal heartbeat is clear, rhythmic-145 / min. During vaginal
examination, the cervix is shortened, the external eye is opened by 1.5 cm, the fetal bladder is intact,
the head is present, the discharge is mucous, the body temperature is 36.5°C. in the blood: no deviations. In
a vaginal smear, there are 4-5 white blood cells in the n / A. What is the obstetric tactic?
100% Tocolysis. Prevention of fetal distress syndrome
0% Antispasmodics, analgesics
0% Give birth through the natural birth canal
0% Caesarean section operation
0% Give intravenous anesthesia to relieve labor

1846. a shop doctor forms a group of people who are often ill for in-depth observation.
In this case, it takes into account the number of etiologically related cases of diseases with
temporary disability during the last year for each of the workers. What
should this number be in order for the employee to be assigned to the specified group?
100% 4 or more
0% 1 or more
0% 2 or more

423

Downloaded from the site - online testing step

0% 3 or more
0% 6 or more

1847. A 53-year-old patient was admitted to the clinic from a district hospital, where 5 days ago
a back carbuncle was opened by a linear incision. In the postoperative period, the condition remained severe.
Tachypnea, tachycardia, leukocytosis, hyperthermia were detected. During bacteriological examination
of the blood, hemolytic streptococcus was isolated. What is the most likely diagnosis?
100% Sepsis
0% Carbuncle of the back in the stage of abcedation
0% Toxicoresorptive fever
0% Systemic inflammatory response syndrome
0% Septic shock

1848. a 32-year-old pregnant woman complains of episodic loss of consciousness,spontaneous syncope,


which quickly disappears when the body position changes. Syncope may be accompanied
by rapidly passing bradycardia. Otherwise, pregnancy proceeds without complications.
What is the most likely cause of this condition?
100% Compression of the inferior vena cava by a pregnant uterus
0% Increased pressure in the veins of the upper extremities

https://translate.yandex.com/en/doc 407/540
22:20 ,27.6.2023 �� C : ; 5 B
0% Reduction of pressure in the veins of the lower extremities
0% Vegetative-vascular dystonia by cardiac type
0% Psychosomatic disorders

1849. on the 15th day after giving birth, a woman went to the gynecologist complaining of pain in the right
breast, fever up to 38°C, and weakness. Objectively: in the upper
quadrant of the right breast, a painful infiltrate of 10x10 cm is palpated, the skin above it
is hyperemic, and purulent discharge from the nipple . What is the most likely diagnosis?
100% Purulent mastitis
0% Lactostasis
0% Fibrocystic mastopathy
0% Mastopathy
0% Cyst of the right breast

1850. A 33-year-old woman has had two previous operations for an ectopic pregnancy,
and both fallopian tubes have been removed. I applied for a consultation with the question: what can I do
to get pregnant?
100% In Vitro fertilization
0% Male sperm insemination
0% Surrogate motherhood
0% Artificial insemination with donor sperm
0% Ovulation induction

1851. An 18-year-old patient complains of fever


, pain and swelling in the knee joints, and a rash in the form of red rings on the lower legs 2 weeks after having a sore throat.

424

Downloaded from the site - online testing step

After a few days, I started to worry about pain in my ankles, then in my elbows. What
disease is characterized by such symptoms?
100% Acute rheumatic fever
0% Rheumatoid arthritis
0% Reactive arthritis
0% Toxic-allergic dermatitis
0% Deforming osteoarthritis

1852. a 62-year-old woman has weakness, increased sweating. In the anamnesis: frequent
colds. Objectively: the temperature is 37.5°C. palpable cervical lymph
nodes up to 1.5-2 cm in size, painless during palpation, not soldered, tightly elastic consistency.
In the blood: white blood cells - 30 • 109/l, lymphocytes-68%, prolymphocytes-6%. Leukolysis cells
were detected. Which of the following diagnoses is most likely to occur?
100% Chronic lymphocytic leukemia
0% Lymphoma
0% Regional lymphadenitis
0% Lymphosarcoma
0% Lymphogranulomatosis

1853. during an emergency delivery, a maternity patient developed


acute abdominal pain, labor stopped, and hypovolemic shock phenomena began to increase against the background of rapid labor activity

https://translate.yandex.com/en/doc 408/540
22:20 ,27.6.2023 �� C : ; 5 B
. External
100% bleeding is insignificant.
Uterine rupture What is the most likely diagnosis?

0% Secondary weakness of labor activity


0% Amniotic fluid embolism
0% Placenta previa
0% Detachment of the naturally located placenta

1854. a patient with alcoholism claims that he "constantly hears men and women's voices outside the window"
arguing about him:" some call him an alcoholic, a jerk who should
be stabbed or hanged, others - a kind father, because he brought the child an apple." What kind
of psychopathological syndrome can you think about?
100% Hallucinatory
0% Delusional
0% Paranoid
0% Abstinent
0% Oneiroid

1855. a 3-month-old girl has a runny nose, shortness of breath, and a dry cough. He is ill on the 2nd day.
Objectively: pale skin, acrocyanosis, shallow respiration, Ps-80 / min., boxy sound over the entire surface
of the lungs, a large number of small-bubble wheezes. What is the most likely
diagnosis?
100% Acute bronchiolitis
0% Pneumonia
0% Cystic fibrosis

425

Downloaded from the site - online testing step

0% Foreign body of the respiratory tract


0% Acute bronchitis

1856. A 24-year-old patient went to the clinic complaining of weight gain and increased
appetite. Objectively: hypersthenic build, body mass index 33.2 kg / m2, waist circumference 100 cm.
The ratio of waist circumference to hip circumference is 0.95. what is the preliminary diagnosis?
100% Alimentary-constitutional obesity, first century, abdominal type
0% Hypothalamic obesity according to the Itsenko-Cushing type, II century, genoid type
0% Alimentary-constitutional obesity, III century, genoid type
0% Alimentary-constitutional obesity, II century, abdominal type
0% Hypothalamic obesity according to the Itsenko-Cushing type, I century, abdominal type

1857. a 33-year-old patient has acute blood loss (ep.- 2,2 • 1012/ l, HB-55 g / l), blood
type a (II)KN+. He was mistakenly transfused with donor erythromas AB (IV) Rh+. After
an hour, there were feelings of anxiety, pain in the lower back, stomach. Ps-134 / min, AT-100/65 mm Hg,
body temperature-38.6°C. during catheterization of the bladder, 12 ml/h
of dark brown urine was obtained. What complication did the patient have?
100% Acute renal failure
0% Cardiac shock
0% Allergic reaction to donor red blood mass
0% Citrate intoxication
0% Infectious and toxic shock

https://translate.yandex.com/en/doc 409/540
22:20 ,27.6.2023 �� C : ; 5 B
1858. a 35-year-old patient was admitted to an infectious diseases hospital with complaints of cold,
fever up to 39°C, nausea, repeated vomiting, aching abdominal pain, frequent
liquid bowel movements with an unpleasant smell, green color up to 10-12 times a day.
The skin is pale, the turgor of the tissues is somewhat reduced, the abdomen is soft, painful in the mesogastrium, rumbling in
the right iliac region. What is the most likely diagnosis?
100% Salmonellosis
0% Food toxicoinfection
0% Cholera
0% Shigellez
0% Rotavirus infection

1859. a 42-year-old patient was hospitalized with complaints of headache and body rash. During
the examination: t° - 40°c, agitated, delirious, face hyperemic, tongue enlarged,
jerky, petechial rash on the trunk, limbs, tachycardia, hypotension,
hepatosplenomegaly. No permanent place of residence, unemployed. What is the most
likely diagnosis?
100% Typhus fever
0% hiv infection
0% Flu
0% Leptospirosis
0% Typhoid fever
1860. A 60 - year-old woman began to feel weak and dizzy during the last year,

426

Downloaded from the site - online testing step

rapid fatigue. Recently-shortness of breath, paresthesia. Objectively: the skin and mucous
membranes are pale with an icteric tint. The papillae of the tongue are smoothed. Liver, spleen at the edge
of the costal arch. In the blood: Hb-70 g / l, Er-1, 7 * 1012/l, CP-1,2, macrocytes. Which
drug is pathogenetically justified for its use?
100% Vitamin B12
0% Vitamin B6
0% Ascorbic acid
0% Iron preparations
0% Vitamin B1

1861. polyclinic doctors conduct a statistical study of the results of diseases


of two groups of patients (registered and not registered)
, depending on their age, gender, and level of hygiene culture. Which type of statistical tables
is most appropriate to use for an in-depth analysis of the relationship between these features?
100% Combination options
0% Analytical services
0% Group settings
0% Simple ones
0% Currently under development

1862. a 57-year-old woman complains of a feeling of compression in the esophagus, palpitation,


difficulty breathing when eating solid food, sometimes vomiting with her mouth full, and at night-
a symptom of a "wet pillow". Ill for about 6 months. Objectively: temperature-36.5°C,
Height-168 cm, weight -72 kg, P8-76/min., AT-120/80 mm Hg. Radiologically: esophagus is significantly
expanded, in the cardiac part - narrowed. What pathology is most likely to cause
dysphagia in the patient?

https://translate.yandex.com/en/doc 410/540
22:20 ,27.6.2023 �� C : ; 5 B
100% Achalasia of the cardia
0% Primary esophagospasm
0% Hiatal hernia
0% Esophageal cancer
0% Reflux esophagitis

1863. a 35-year-old woman complains of weakness, frequent bowel movements with blood impurities,
a decrease in body weight by 8 kg in 6 months, periodically-an increase in temperature to 37.8°C.
objectively: t° - 37.5°C, P8-86 / min, AT-110/70 mm Hg. the skin is pale palpation of the abdomen is moderately
painful. In the blood: Hb-92 g / l, SSE-35 mm / h. Colonoscopy: the mucosa of the rectum and sigmoid
colon is granular, hyperemic, edematous, unevenly thickened (pseudopolyps), bloody,
with erosions and ulcers on its surface. What pathology is most likely to cause intestinal damage in the
patient?
100% Ulcerative colitis
0% Crohn's disease
0% Irritable bowel syndrome
0% Rectal cancer
0% Intestinal polyposis

427

Downloaded from the site - online testing step

1864. after lifting the load, a 47-year-old man developed a sharp pain in the right side
of the chest, suffocation, cyanosis of the skin and mucous membrane began to increase. The patient
had infiltrative tuberculosis of the right lung 7 years ago. I haven't been examined in the last three years. On
objective examination, tympanitis is percussive, and on auscultation, respiration is sharply
weakened. What complication did the patient develop?
100% Spontaneous pneumothorax
0% Pulmonary embolism
0% Exudative pleurisy
0% Right lung atelectasis
0% Hospital-acquired pneumonia

1865. a 50-year-old patient who has been suffering from hypertension for more than 10 years
suddenly had a high blood pressure of 200/110 mm Hg due to stress.The condition was accompanied by trembling
of the body, headache, tachycardia, general agitation, a feeling of heat and dry mouth.
What medications are the most appropriate ones to prescribe?
100% ^ - adrenoblockers
0% Calcium dihydropyridine antagonists
0% Diuretics
0% Non-dihydropyridine calcium antagonists
0% Angiotensin II receptor blockers

1866. a 64-year-old patient on the fourth day of her stay in a therapeutic hospital with
the diagnosis: "CHD: tension angina of FC III" had a palpitation and a sharp deterioration
in her state of health. Objectively: heart sounds are dull, arrhythmic, systolic murmur at the apex.
HR94 / min., no pulse deficit. AT-130/85 mm Hg on the ECG: group, polytopic ventricular
extrasystoles, episodes of arrhythmia. What fatal clinical situation can
be predicted?
100% Ventricular fibrillation

https://translate.yandex.com/en/doc 411/540
22:20 ,27.6.2023 �� C : ; 5 B
0% Paroxysm of atrial fibrillation
0% Complete atrioventricular block
0% Development of sinus node weakness syndrome
0% Sinoatrial blockade

1867. a 46-year-old woman who had been suffering from hypertension for 5 years
developed a hypertensive crisis. Complaints of palpitation, throbbing sensation in the head, heart rate-100 / min,
AT-190/100 mm Hg (hyperkinetic type of hemodynamics). Which drug should I give
preference to?
100% beta-blocker
0% ACE Inhibitor
0% Diuretic
0% a-adrenoblocker
0% Dihydropyridine calcium antagonist

1868. a 54-year-old patient has mild primary arterial hypertension, 1xc: tension angina
II FC, HF II B. Concomitant diagnosis: gastroesophageal reflux disease, erosive esophagitis II

428

Downloaded from the site - online testing step

stage. Constant use of which of the drugs can cause the patient to increase the manifestations
of gastroenterological pathology?
100% Isosorbide dinitrate
0% Metoprolol
0% Enalapril maleate
0% Omeprazole
0% Hydrochlorothiazide

1869. a patient with type I diabetes mellitus, who was taken to the intensive care unit in a state of
ketoacidotic coma, was administered intravenously in the first three hours:
saline solution of 1.2 liters, Ringer's solution of 400 ml,potassium chloride solution of 7.5% 200 ml,
cocarboxylase 5 ml. What other remedy must be administered to the patient?
100% 4% sodium bicarbonate solution
0% Red blood cell mass
0% 10% calcium chloride solution
0% Blood plasma
0% Promedol

1870. A 17-year-old patient is undergoing inpatient treatment for glomerulonephritis.


Complaints of severe edema all over the body, a decrease in the amount of urine, headache. In the urine:
protein 7.1 g / l, leuc.- 1-2 v / s, ep. - 3-4 v / s. protein in daily urine -3.8 g / l, diuresis-800 ml.
Total protein - 43.2 g/l, urea-5.2 mmol/l. Cholesterol - 9.2 mmol / l. which of
the listed glomerulonephritis syndromes is most likely to occur in the patient?
100% Nephrotic
0% Nephritic
0% Urinary tract infection
0% Hematuric
0% Mixed

https://translate.yandex.com/en/doc 412/540
22:20 ,27.6.2023 �� C : ; 5 B
1871. in a 1-month-old boy with symptoms of agitation, the outline of the head is 37 cm, the size
of the large crown is 2x2 cm. The child regurgitates after feeding small portions of milk;
the stool is normal in composition and volume. Muscle tone is normal. What
is the most likely diagnosis?
100% Pylorospasm
0% Meningitis
0% Pylorostenosis
0% Microcephaly
0% Craniostenosis

1872. a venereologist was visited by a patient who was diagnosed with trichomoniasis based on clinical manifestations and
laboratory examination data. When prescribing treatment
to this patient, the main drug should be considered:
100% Tinidazole
0% Interferon
0% Biseptol

429

Downloaded from the site - online testing step

0% Pimidel
0% Tetracycline

1873. an employee of a leather processing plant,12 years of experience in contact with ursol,
complains of severe itching of the skin. Objectively: on the hands and fingers, forearm,
face and neck there are symmetrical papular polymorphic rashes. The condition worsens after
work, on weekends and during holidays feels better. Which of the following should
be applied in this case?
100% Antihistamines
0% Disinfection solutions
0% Sulfur preparations
0% Radioactive isotopes
0% X-ray therapy

1874. a 62 - year-old woman, after lifting a load, felt acute pain in the lumbar region, buttock,
posterolateral surface of the right thigh, the outer surface of the right shin and the back
of the foot. Objectively: weakness of the anterior tibial muscle, long extensor
of the thumb, short extensor of the fingers of the right foot. Reduced Achilles reflex on the right side.
Positive Lasega symptom. What is the most informative method of investigation to clarify
the diagnosis of discogenic compression of the L5 root?
100% Magnetic resonance imaging
0% Radiography of the spine
0% Electromyography
0% Angiography
0% Lumbar puncture

1875.a patient with hoarse breathing (more


difficult breathing), skin cyanosis, tachycardia, and arterial hypertension was admitted to the emergency department of the hospital. A
history
of bronchial asthma. An hour ago, I inhaled salbutamol and forgot to remove the cap, which
I aspirated
100% with Perform
a deep breath. What trick
the Eimlich are the doctor's actions?

0% Immediately perform a conicotomy

https://translate.yandex.com/en/doc 413/540
22:20 ,27.6.2023 �� C : ; 5 B
0% Call an anesthesiologist and wait for him to show up
0% Make an inhalation of beta-2-adrenomimetics
0% Enter dexamethasone subcutaneously

1876.a patient complaining of pelvic pain was taken to the sanitarium. Two
hours ago, during the explosion, he was pinned down by an overturned car. Moans in pain.
Objectively: AT - 70/40 mm Hg, heart rate-115/min. The pelvis is deformed. Shortening of the right lower
limb. Abdominal organs without pathology. Choose the best way
to stop the pain syndrome:
100% Narcotic analgesic
0% Intraosseous anesthesia in the iliac wing
0% Intra-pelvic anesthesia
0% Non-narcotic analgesic

430

Downloaded from the site - online testing step

0% Conducting anesthesia

1877. a 37-year-old patient has frequent painful urination, a feeling of incomplete emptying
of the bladder. He has been ill for about 15 years. In the urine: specific gravity-1020, protein-0.04 g / l,
leuc. 20-25 in p/ s, ep. - 3-4 in p/s. ultrasonography shows no special features of the kidneys, the volume of the bladder
is 300 ml, its wall is thickened to 0.5 cm, and the trabecular lumen shows echosavis. What method
of additional research is necessary for this patient in the first place to clarify the diagnosis?
100% Cystoscopy
0% Backseeding of urine
0% Excretory urography
0% Overview of urography
0% Nechiporenko's test

1878. a 77-year-old patient complains of the inability to urinate, bursting pain over
the womb. I was acutely ill 12 hours ago. Objectively: a full bladder is palpated above the womb
. Rectally: the prostate is enlarged, tightly elastic, with clear contours, without nodes.
The interparticle groove is pronounced. In ultrasonography, the prostate volume is 120 cm3, it
is pushed into the bladder cavity, the parenchyma is uniform. Prostate-specific antigen -5
ng / ml. What is the most likely disease that caused acute urinary retention?
100% Prostatic hyperplasia
0% Prostate cancer
0% Prostate sclerosis
0% Tuberculosis of the prostate
0% Acute prostatitis

1879. a 2-year-old boy on the second day after the preventive vaccination had
a fever of up to 38°C and abdominal pain without a specific localization. On day 3, the child
developed a red papular-hemorrhagic rash on the extensor surfaces of the limbs and
around the joints. There is swelling and slight soreness of the knee joints. From
other organs and systems-without pathological changes. What is the most likely diagnosis?
100% Hemorrhagic vasculitis
0% Thrombocytopenic purpura
0% Meningococcemia
0% Urticaria

https://translate.yandex.com/en/doc 414/540
22:20 ,27.6.2023 �� C : ; 5 B
0% DIC-syndrome

1880. a 73-year-old patient went to the doctor with complaints of palpitations, interruptions in the activity
of the heart, suffocation with a predominance of difficult breathing. P8-96 / min, heart rate-128 / min, atrial
fibrillation. Heart sounds are weakened, systolic murmur is on top. Liver + 5 cm.
Lower leg edema, EF-51%. To control the heart rate, it is advisable to use:
100% Digoxin
0% Lisinopril
0% Furosemide
0% Trimetazidine
0% Amlodipine

431

Downloaded from the site - online testing step

1881. a 22-year-old patient complains of coughing with the release of "rusty" sputum, episodes
of confusion, fever up to 40.1°C, shortness of breath, muscle pain.
Objectively: BH-36 / min. On the right, below the angle of the scapula-a dull percussion sound, during auscultation
-bronchial respiration. In the blood: lake.- 17,8 • 109/ l, WSE-39 mm / hour. What is the most
likely diagnosis?
100% Non-hospital-acquired pneumonia
0% Tuberculosis of the lungs
0% Bronchiectasis
0% Acute bronchitis
0% Lung cancer

1882. to assess the state of health of the population,a report on diseases


registered in the population of the service area was compiled and analyzed (f. 12). What indicator is calculated based
on this report?
100% Overall morbidity rate
0% Indicator of pathological lesion
0% Morbidity rate with temporary disability
0% Rate of hospitalized morbidity
0% Indicator of the main non-epidemic morbidity

1883.A 12-year-old child periodically develops a fever of up to 38.5


°C, cough, shortness of breath, and hemoptysis once during 6 months. BCG scar is absent. My grandfather
has tuberculosis. The lungs are scattered with dry and mixed wet wheezes. Radiologically:
uniform shallow-focal lung infiltration of the same type, thin-walled cavern in the upper lobes with
weakly expressed perifocal infiltration. What disease is likely?
100% Tuberculosis of the lungs
0% Focal pneumonia
0% Chronic bronchitis
0% Bronchial asthma
0% Haman-Rich syndrome

1884. a patient suddenly developed stridorotic respiration after exposure to chemicals in the workplace
. Hoarseness of voice, "barking" cough, shortness of breath began to increase.
Objectively: acrocyanosis. What is the preliminary diagnosis?
100% Laryngeal edema
0% Laryngeal cancer

https://translate.yandex.com/en/doc 415/540
22:20 ,27.6.2023 �� C : ; 5 B
0% bodies
0% Atelectasis of the lungs
0% Pneumothorax

1885. a 37-year-old patient was hospitalized in the intensive care unit due to
repeated convulsive tonic-clonic seizures every half hour.
He doesn't regain consciousness between seizures. AT-120/90 mm Hg, Ps-100 / min. I was at a wedding last night, drinking
alcohol. 5 years ago, he suffered a closed craniocerebral injury, a brain contusion, after
which he had isolated convulsive attacks with loss of consciousness. Patient's course of antiepileptic medicine

432

Downloaded from the site - online testing step

the treatment did not take place. What drug should be administered first for
emergency care?
100% Sibazonum
0% Magnesium Sulfate
0% Sodium oxybutyrate
0% Aminazine
0% Sodium thiopental

1886.during the forensic examination of the corpse, the forensic expert described a
linear wound in the crown of the head on the right, measuring 6.4 cm in size with the edges drawn together, the edges are uneven,
covered with abrasions; tissue membranes are visible in the depth of the wound. Give a name to the described wound:
100% Clogged
0% Dissected
0% Chopped
0% Knees
0% Sliced

1887. A 60 - year-old patient complains of an almost constant feeling of heaviness and fullness
in the epigastrium, which increases after eating, belching with a rotten smell, sometimes vomiting of food eaten 1-2
days ago, and weight loss. 12 years ago, a pyloric canal ulcer was first discovered.
Noted periodic "hungry" pains, for which he took Ranitidine. Deterioration within
3 months. Objectively: the "splashing noise" in the epigastrium is determined. What complication
are we talking about?
100% Pyloric stenosis
0% Penetration of a stomach ulcer
0% Functional spasm of the goalkeeper
0% Foreign body of the stomach (bezoar)
0% Malignancy of a stomach ulcer

1888. a 23-year-old woman. After the stress, thirst,polydipsia, polyuria appeared,


she lost weight, weakness increased, then nausea, vomiting, drowsiness appeared, and she lost consciousness.
She was hospitalized. Glycemia - 28 mmol / l, acetone in the urine is sharply positive. Treatment
of ketoacidotic coma has been initiated. When is it advisable to start prevention of hypoglycemia by introducing
a 5% glucose solution?
100% After reducing the level of glycemia to 13-14 mmol / l
0% 2 hours after the start of insulin therapy
0% After the patient regains consciousness
0% After normalization of the glycemic level

https://translate.yandex.com/en/doc 416/540
22:20 ,27.6.2023 �� C : ; 5 B
0% When the rate of decrease in the level of glycemia exceeds 5 mmol / l per hour

1889. A 15-year-old teenager was examined at the military registration and enlistment office and found to have an interval systolic
murmur at the apex of the heart, an accent of the second tone over the pulmonary artery, and tachycardia. Which of the additional
examination methods is most informative for making a diagnosis?
100% Echocardiography
0% Electrocardiography

433

Downloaded from the site - online testing step

0% Radiography
0% Phonocardiography
0% Rheography

1890. a 5-year-old girl complains of pain in the vaginal area, significant purulent
discharge, which worries for 5 days and gradually increases. On examination, the doctor found
significant swelling of the external genitalia, redness, purulent discharge from the vagina with
an unpleasant smell. Ultrasound scans show an echo-positive shadow in the vaginal area. What reason
can lead to this condition in children?
100% Presence of a foreign body in the vagina
0% Vaginal tumor
0% Cervical tumor
0% Purulent colpitis
0% Vulvovaginitis

1891. the victim was taken in serious condition from the scene of an accident. During examination, pain with
axial load on the pelvic wings and their mobility. In what position should
the patient be immobilized?
100% On the back in the position behind Volkovich
0% On the side with the hips brought to the stomach
0% On the stomach with a pillow under the pelvis
0% In a half-sitting position
0% On the back with a pillow under the pelvis

1892. a 22-year-old patient with reduced nutrition, vegetarian, went to the polyclinic with
complaints of distortion of smell, taste, "jamming" in the corners of the mouth. Objectively: pronounced blueness
of the sclera. The diagnosis was iron deficiency anemia. Which clinical syndrome has
the advantage?
100% Sideropenic
0% Anemic
0% Iemological
0% Hemolytic
0% Myelodysplastic

1893. a 37-year-old patient has a sharp pain in the frontal area


on the right side, deterioration of breathing through the nose and mucopurulent discharge from the right side for a week.
Objectively: edema of the nasal mucosa, mucopurulent discharge from the middle nasal
passage. Which of these diseases is most characterized by these symptoms?
100% Frontit
0% Sphenoiditis
https://translate.yandex.com/en/doc 417/540
22:20 ,27.6.2023 �� C : ; 5 B

0% Sinusitis
0% Etmoidite
0% Gemisinuit

434

Downloaded from the site - online testing step

1894. A 28-year-old patient developed severe weakness,dizziness,


darkening of the eyes, nausea and loss of consciousness without convulsions after stress. Objectively: the patient is unconscious, her skin
is pale, and her limbs are cold. Pupillary and tendon reflexes are preserved. AT-80/50 mm Hg,
Ps102 / min., low filling. Blood glucose is 5.7 mmol / l. what is the most likely diagnosis?
100% Syncopal state
0% Epileptic syndrome
0% A hysterical fit
0% Hypoglycemic coma
0% Transient cerebrovascular accident

1895. in an 18-year-old patient, blood pressure-120/70 mm Hg in the III intercostal space to the left of the sternum edge-
systolic tremor. The left border of the heart is shifted 1 cm outwards. In the II-III intercostal space on the left
, systolic-diastolic murmur, which increases during systole, accent of the II tone. What
is the most likely diagnosis?
100% Open Ductus arteriosus
0% Aortic coarctation
0% Ventricular septal defect
0% Atrial septal defect
0% -

1896. 2 weeks after giving birth, a woman in labor developed pain in the mammary gland, which increased
within 3 days. Objectively: body temperature 39°C, chilliness, weakness, hyperemia
of the skin, enlargement, soreness and deformity of the breast. Palpation of the infiltrate
reveals the area of softening and fluctuation. What is the most likely diagnosis?
100% Infiltrative-purulent mastitis
0% Phlegmonous mastitis
0% Lactostasis
0% Serous mastitis
0% Mastopathy

1897.A 3-year-old child was prescribed Biseptol, paracetamol,and nasoferon due to acute respiratory viral
infection. On the third day, the child's condition worsened: there was a sore throat, stomatitis,
conjunctivitis, hypersalivation, painful spots of dark red color on the neck, face, chest and
limbs, then blisters appeared on the surface of the spots. The impression of mucous membranes
around the mouth and anus was noted. What is the preliminary diagnosis?
100% Stevens-Johnson syndrome
0% Atopic dermatitis
0% Chickenpox
0% Serum sickness
0% Bullous dermatitis

1898. A 20-year-old patient complains of a lack of air, prolonged aching pains in the
https://translate.yandex.com/en/doc 418/540
22:20 ,27.6.2023 �� C : ; 5 B
heart, and irritability. During the examination: general condition is satisfactory, pulse lability,
AT130 / 60 mm Hg ECG-violation of repolarization processes. The patient was diagnosed
with neurocirculatory dystonia of the cardiac type. Specify the conditions in which the patient should

435

Downloaded from the site - online testing step

receive treatment:
100% Outpatient treatment
0% Inpatient treatment in the therapeutic department
0% Inpatient treatment in the cardiology department
0% Inpatient treatment in the cardiac surgery department
0% Inpatient treatment in a psychiatric ward

1899. a 24-year-old woman in labor has a postpartum period, on the 4th day. The delivery was the first,
timely, without complications. The general condition of the woman in labor is satisfactory. Body temperature 36.6 °
C Ps78 / min, rhythmic. Mammary glands in a state of engorgement. The bottom of the uterus is 2 cm below the navel. The uterus is in
normal condition, painless. Lochia are bloody, moderate. Stool and urination are normal.
What treatment is appropriate?
100% Uterotonic drugs
0% Analgesics
0% Antibiotics
0% Estrogenic hormones
0% Medications that stop lactation

1900. A 47-year-old patient is concerned about a tumor-like formation on the anterior surface of the neck in the
thyroid gland. Notes a progressive increase in education. Objectively: in the right lobe
of the thyroid gland there is a formation with a diameter of about 4 cm with a smooth surface,
mobile, high density. Slightly expressed symptoms of thyrotoxicosis. Which of
the additional examination methods is the most informative for clarifying the diagnosis?
100% Fine needle puncture biopsy
0% Neck radiography
0% Ultrasound examination
0% Determination of thyroid hormones
0% Determination of protein-bound iodine

1901. A 20-year-old patient who was delivered from the street in the summer with brachial artery bleeding was put on a
tourniquet during first aid to temporarily stop
the bleeding. Specify the maximum exposure of the harness:
100% 120 minutes
0% 15 minutes
0% 30 minutes
0% 60 minutes
0% 180 minutes

1902. Purulent meningitis was diagnosed in a 36-year-old patient with severe meningeal syndrome, petechial
skin rashes, coldness, body temperature - 39°C, inflammatory changes in peripheral blood, and
neutrophil pleiocytosis in the cerebrospinal fluid. Which of the existing
syndromes in a patient is crucial for making a diagnosis of meningitis?
100% Neutrophilic pleiocytosis
0% Petechial skin rashes

https://translate.yandex.com/en/doc 419/540
22:20 ,27.6.2023 �� C : ; 5 B
0% Meningeal syndrome

436

Downloaded from the site - online testing step

0% Increased body temperature, cooling down


0% Inflammatory changes in the peripheral blood

1903. a patient who was injured with a broken collarbone developed flabby atrophic paralysis
of the right hand with a violation of all types of sensitivity in it. What disease can be assumed?
100% Plexitis of the brachial plexus
0% Cervical-thoracic sciatica
0% Cubital canal syndrome
0% Cervicothoracalgia
0% Polyneuritis

1904. a patient developed pulmonary edema due to acute left ventricular heart failure
. What inhalations are indicated in this case together with oxygen therapy?
100% Inhalation of alcohol vapors
0% Inhalation of chamomile infusion
0% Inhaling soda
0% Inhalation of propolis infusion
0% Ultrasonic inhalations

1905. A 48-year-old female patient complains of heavy menstruation. Births - 2, abortions-2.


I haven't seen a gynecologist in the last 2 years. Objectively: the external genitalia are free of pathology,
the cervix is cylindrical, clean. The body of the uterus is enlarged to 14-15 weeks of pregnancy, uneven surface,
mobile, painless. The arches are deep. Appendages are not defined, the site of appendages
is painless. The parameters are free. Mucosal discharge. What is the most likely diagnosis?
100% Uterine fibromyoma
0% Uterine body sarcoma
0% Pregnancy 14-15 weeks
0% Chorionic epithelioma
0% Endometrial cancer

1906. A 62-year-old female patient was hospitalized with complaints of enlarged cervical, supraclavicular, and
iliac lymph nodes, general weakness, increased sweating, and sub-febrile
fever during the last 3 months. In the blood: white blood cells - 64 • 109/l, in the Formula-
lymphocytes 72%. What research method should be used to clarify the diagnosis?
100% Myelogram
0% Lymphography
0% Lymphoscintigraphy
0% X-ray examination
0% Thermography

1907. based on the results of a hospital-pedagogical observation of a physical education lesson in the 9th
grade, a physiological curve is constructed, characterized by a gradual increase
in the pulse rate in the introductory part, an increase in the pulse rate by 80% during the main part, the curve
has a 4-toothed form. How can I evaluate the organization of a physical education lesson?

https://translate.yandex.com/en/doc 420/540
22:20 ,27.6.2023 �� C : ; 5 B
437

Downloaded from the site - online testing step

100% Long intervals between exercises


0% The lesson is built correctly
0% Physical activity is insufficient
0% Excessive physical activity
0% Physical activity is sufficient

1908.A 20 - year-old patient with polyarthritis symptoms was admitted to the rheumatology department. On
the skin of the face - redness in the form of a "butterfly" in Sich: 6 tablets - 4.8 g / l. the doctor suggested
that the patient has systemic lupus erythematosus. What additional research method is
the most informative for verifying the diagnosis?
100% Blood test for antinuclear antibodies
0% General blood test
0% Immunological blood testing
0% LE-cell blood test
0% Blood test for rheumatoid factor

1909. A 39-year-old patient has been suffering from diabetes mellitus for 10 years. The last year marks
the cooling of the toes, pain and a feeling of zaterpaniya. Objectively: the skin of the lower extremities is dry,
thinned, cold to the touch; pulsation on the femoral and popliteal arteries is preserved. What
is the most likely diagnosis?
100% Diabetic vascular microangiopathy of the lower extremities
0% Diabetic macroangiopathy of the vessels of the lower extremities
0% Raynaud's disease
0% Obliterating atherosclerosis of the vessels of the lower extremities
0% Obliterating endarteritis of the vessels of the lower extremities

1910. a 26-year-old patient went to a women's clinic with complaints of lower abdominal pain,
which increases during menstruation, spotting spotting before and after menstruation.
The disease is associated with an artificial abortion. In the mirrors: there are 5 dark red inclusions on the cervix
. What is the most likely diagnosis?
100% External endometriosis
0% Cervical polyposis
0% Cervical cancer
0% Cervical erosion
0% Cervical dysplasia

1911. a patient has been suffering from gastric ulcer disease for more than 15 years, and has been repeatedly
treated for exacerbations. During the year, he notes a feeling of heaviness in the stomach, belching
with an unpleasant smell. Over the past three days, I have vomited several times with unsweetened food.
Which of the above research methods is the most informative for verifying the diagnosis?
100% X-ray examination
0% Ultrasound examination
0% pH-metric
0% Multi-stage duodenal probing

438

https://translate.yandex.com/en/doc 421/540
22:20 ,27.6.2023 �� C : ; 5 B

Downloaded from the site - online testing step

0% Helicobacter pylori Study

1912. A 24-year-old patient went to the doctor complaining of enlarged submandibular


lymph nodes. Objectively: enlarged axillary stumps, axillary and inguinal lymph nodes.
Chest X-ray shows enlarged mediastinal lymph nodes. In the blood: Er.- 3,4 •
1012/ l, Hb-100 g / l, CP-0.88, tr.- 190 • 109/ l, lake.- 7 • 5 * 109/l, E.-8%, P. -2%, C.-67%,
lymph. 23%, ESR-22 mm/h. What research is indicated to verify the cause of lymphadenopathy?
100% Open lymph node biopsy
0% Ultrasound examination of the abdominal organs
0% Tomography of the mediastinum
0% Puncture biopsy of lymph nodes
0% Sternal puncture

1913. in a 35-year-old patient, on the 8th day after surgical treatment of a purulent focus, the wound
was cleared of purulent-necrotic contents, granulations appeared. Nevertheless, against the background
of antibacterial therapy, the body temperature is kept at 38.5-39.5 oC, colds,sweating,
euphoria, pulse rate -120/min. What complication of the local purulent-inflammatory process can
be assumed?
100% Sepsis
0% Purulent-resorptive fever
0% Thrombophlebitis
0% Meningitis
0% Pneumonia

1914. a 61-year-old man was talking while eating (pumpkin seeds) and coughed. Continuous
coughing changed to periodic coughing after 2 hours. On the exhalation during coughing at a distance
, ”flapping " is heard during auscultation, pulmonary respiration is heard from both sides.
On the X-ray of the chest cavity, no pathological changes were found. Where
is the balloting foreign body located?
100% Trachea
0% Ventricles of the larynx
0% Right bronchus
0% Left bronchus
0% Hypopharynx

1915. a patient complains of a feeling of sand in the left eye, mucopurulent discharge, and clumping
of the eyelids, especially in the morning. Objectively: during the examination of the conjunctival cavity
, mucopurulent discharge and conjunctival injection are observed. Visual acuity and other structures of the eye are not
affected. What is the most likely diagnosis?
100% Acute purulent conjunctivitis
0% Adenoviral conjunctivitis
0% Superficial foreign bodies of the conjunctiva
0% Allergic conjunctivitis
0% Chronic dacryocystitis
1916. a patient with probable dysentery was admitted to the infectious diseases department.

439

Downloaded from the site - online testing step

https://translate.yandex.com/en/doc 422/540
22:20 ,27.6.2023 �� C : ; 5 B

feces mixed with blood, similar to red currant jelly). A day


later, the final diagnosis was established-invagination. The patient was transferred to a specialized
department. What treatment methods can be used?
100% Operational disinvagination
0% Conservative disinvagination
0% !nfusion therapy with antispasmodics
0% Manual disinvagination
0% Paranephral blockade

1917. A 10-year-old boy suffered a blunt chest injury a week ago. The second day
is in the cardiology department for traumatic pericarditis.
Echocardiogram shows signs of pericardial effusion. Signs of heart failure have been progressing in the last hour
. There was a threat of cardiac tamponade. What should be the primary medical
tactics?
100% Pericardial puncture
0% IV Cardiac glycosides
0% Permanent oxygen therapy
0% Intravenous diuretics
0% Intravenous antibiotics

1918. A 44-year-old patient complained of difficulty urinating and a feeling of incomplete


emptying of the bladder. During the sonographic examination of the bladder
,an oval hyperechoic formation with equal contours measuring 2x3 cm was found at the entrance to the urethra
, which was moved during the study. What conclusion can be drawn?
100% Concretion
0% Malignant tumor of the bladder
0% Polyp of the bladder
0% Prostate adenoma
0% Tumor of the initial part of the ureter

1919. a 15-year-old girl was diagnosed with chronic cholecystocholangitis 2 years ago. I didn't
follow a diet. The condition has worsened in the last 3 months. There is an increase
in body temperature. Abdominal pain of a pad-like nature after fatty, spicy food. Worries about itchy
skin. The tongue is covered with a white coating. Stomach is soft, liver + 3 cm, palpation is painful,
positive cystic symptoms. In the blood: lake.-12 * 109 / l, ESR-20 mm / hour. Which drug should
be included in the treatment package?
100% Antibiotics
0% Hepatoprotectors
0% Prokinetics
0% Enzymes
0% Enterosorbents

1920. A 12-year-old boy is in a local hospital for croup pneumonia with


severe intoxication and respiratory failure. He receives lincomycin in
combination with biseptol for 3 days. Against the background of the therapy, hectic fever persists,

440

Downloaded from the site - online testing step

short, painful cough, weak breathing. What treatment measures should be applied?
100% Replacement of antibacterial therapy
https://translate.yandex.com/en/doc 423/540
22:20 ,27.6.2023 �� C : ; 5 B

0% Conducting therapeutic bronchoscopy


0% Intrapleural administration of antibiotics
0% Appointment of eufillin
0% Administration of cardiac glycosides

1921. For the second time, 26-year-old Rodilla arrived at the maternity ward due to a 40
-week pregnancy and the beginning of labor. 2 hours ago, the amniotic fluid left.
Fetal position is longitudinal, main presentation. Coolant-100 cm, VDM-42 cm. Contractions in
4-5 minutes, 25 seconds each. Internal obstetric examination: the cervix is smoothed,
opening 4 cm. There is no fetal bladder. The fetal head is pressed against the entrance to the small pelvis. What
complication occurred during childbirth?
100% Premature discharge of amniotic fluid
0% Primary weakness of labor activity
0% Secondary weakness of labor activity
0% Discoordinated labor activity
0% Clinically narrow pelvis

1922. a child with polytrauma has no reaction to external stimuli. The skin
is marbled with an earthy hue, covered with cold sweat. Pupils dilated, weakly react to
light. Limbs are cold, hyperthermia. Signs of respiratory failure. Ps-132 / min.,
threadlike. Hypotension. Anuria. What condition did the child have?
100% Traumatic shock
0% Acute adrenal insufficiency
0% Cardiogenic shock
0% Pneumothorax
0% Hypoglycemic coma

1923. a 13-year-old girl complains for 5 years of pain in the right hypochondrium,
echo in the right scapula, attacks of pain are associated with a violation of diet, they are short-lived, easily
relieved by antispasmodic agents. During an attack of pain, palpation of the abdomen is painful,
as much as possible at the point of projection of the gallbladder. The patient is most likely to have:
100% Biliary dyskinesia
0% Chronic cholecystitis
0% Chronic gastroduodenitis
0% Chronic pancreatitis
0% Duodenal ulcer disease 12

1924. A 23-year-old patient during the last 6 months became withdrawn, avoided communication with
others, went into seclusion, and talked to himself. During a conversation with a psychiatrist
, at first he hid his feelings, but then he told about the special, unreal voices
contained in himself ("nested in the brain”). What are these perceptual disorders?
100% Pseudo-hallucinations
0% Illusions

441

Downloaded from the site - online testing step

0% The hallucinations are real


0% Metamorphopsies
0% Derealization

https://translate.yandex.com/en/doc 424/540
22:20 ,27.6.2023 �� C : ; 5 B

1925. workers of the refrigeration shops of fisheries work in conditions of low


ambient temperatures from -5°C to -15°C. Which of the listed changes in the body of workers are
the most characteristic and leading under the above effects?
100% Vasoconstriction of the skin and muscles
0% Increased oxygen consumption
0% Acceleration of breathing
0% Changes in systolic blood volume
0% Changes in minute blood volume

1926. 1,200 cases


of disability and 12,000 days of disability were registered among employees of an industrial enterprise. What indicator of morbidity with
temporary
disability
100%can beAverage
calculated from these
duration of onedata?
case
0% Number of cases of disability per 100 employees
0% Number of days of disability per 100 employees
0% Percentage of workers who never got sick during the year
0% Percentage of people who have been ill for a long time and often

1927. A 43-year-old patient complains of contact bleeding during the last 6 months.
Bimanually: the cervix is enlarged in size, limited in mobility. In the mirrors-
the cervix in the form of 'cauliflower'."The worm and Schiller samples are positive. What is the most
likely diagnosis?
100% Cervical cancer
0% Cervical polyp
0% Cervical pregnancy
0% Emerging myomatous node
0% Leukoplakia

1928. A 22-year-old patient complains of napa - like whiteness of the fingertips,


which develops as a result of cooling. When warmed, the fingers first acquire a bluish,
then purple color. The drug of choice for the treatment of this syndrome is:
100% Nifedipine
0% Dipyridamole
0% No-shpa
0% Aspirin
0% Nitrates

1929. A 39-year-old patient went to the doctor complaining of morning headache, decreased
appetite, nausea, morning vomiting, and periodic nosebleeds. At the age of 15, he suffered
from acute glomerulonephritis. During the examination, an increase in blood pressure to

442

Downloaded from the site - online testing step

220/130 mm Hg, 6lidity of the skin and mucous membranes. Which of the above blood
microbiochemical parameters is of diagnostic significance in this case?
100% Creatinine
0% Bilirubin
0% Sodium
0% Uric acid
https://translate.yandex.com/en/doc 425/540
22:20 ,27.6.2023 �� C : ; 5 B

0% Fibrinogen

1930. a 30-year-old patient suffering from headaches, while lifting a heavy object
, felt a strong headache similar to a blow to the head; nausea, vomiting, and light
dizziness appeared. A day later-objectively expressed meningeal syndrome, body temperature -37.6
oC. The doctor suggested a subarachnoid hemorrhage. What additional examination should
be performed to confirm the diagnosis?
100% Lumbar puncture with examination of cerebrospinal fluid
0% X-ray of the skull
0% Computed tomography
0% Rheoencephalography
0% Cerebral angiography

1931. a child is hospitalized with focal changes in the skin folds. During the examination, the child
is restless, the skin is dry, with separate papular elements and areas of lichenification with
indistinct edges. A rash on the skin is accompanied by severe itching. Improvement of
the child's condition is observed in the summer months, deterioration - in winter. A child from 2 months on artificial
feeding, underwent exudative diathesis. My maternal grandmother suffers from
bronchial asthma. What is the most likely diagnosis?
100% Atopic dermatitis
0% Contact dermatitis
0% Seborrheic eczema
0% Strofulus
0% Urticaria

1932. in the mental status of a 32-year-old patient, a persistent pathological mood decline.
Contact, briefly, but essentially answers questions. The speech output is extremely concise, in
slow motion. Movement is constrained and inhibited. Expresses ideas of self-blame and
self-deprecation. Which group of drugs are primarily indicated for the patient?
100% Antidepressants
0% Neuroleptics
0% Anxiolytics
0% Tranquilizers
0% Normotimics

1933. in a 40-year-old woman, during palpation of the thyroid gland,a node was found in the left lobe,
compacted, moderately painful during palpation. During ultrasound examination, it
is of increased density, "cold" in scintigraphy with I131. which examination is most appropriate
to clarify the diagnosis?

443

Downloaded from the site - online testing step

100% Aspiration fine needle biopsy


0% Determination of TSH levels in the blood
0% Reflexometry
0% Determination of urinary iodine excretion
0% Thermography

1934. A 62-year-old patient complains of a bulge in the left groin area, which

https://translate.yandex.com/en/doc 426/540
22:20 ,27.6.2023 �� C : ; 5 B
gradually increases. Objectively: in the left inguinal region, the protrusion is 5-6 cm in size,
dense, elastic, painful, does not set in the abdominal cavity, the skin above it is not changed,
the outer inguinal ring passes the tip of the finger. What is the most likely diagnosis?
100% Inguinal lymphadenitis
0% Skilful left-sided inguinal hernia
0% Left-sided pinched inguinal hernia
0% Nonrecoverable left-sided inguinal hernia
0% Tumor formation

1935. a 58-year-old patient complains of bloating, constipation, severe weakness,


and weight loss. During the last month of bowel movements only after taking laxatives.
During palpation of the abdomen, a dense formation is observed in the left iliac region. What
is the most likely diagnosis?
100% Sigmoid colon cancer
0% Colorectal cancer
0% Tumor of the mesentery of the colon
0% Tumor of the retroperitoneal space on the left
0% Cancer of the left ureter

1936. The electric welder of mechanical workshops performs metal welding and cutting
operations accompanied by intense UV radiation at a welding station
equipped with efficient mechanical ventilation. What occupational
disease is most likely to develop in an electric gas welder?
100% Electroophthalmia
0% Heat stroke
0% Vegetative vascular dystonia
0% Chronic overheating
0% Pneumoconiosis

1937. in a patient with duodenal ulcer, the clinical picture has changed: the pain
is constant with radiation to the back, which increases after eating, and is poorly stopped by atropine.
Weight loss in six months up to 6 kg. Endoscopically revealed an ulcer of the duodenum 12 bulb with
dense edges, deformity of the bulb. What can explain the changes in the clinical picture?
100% Penetration
0% Perforation
0% Duodenospasm
0% Malignization

444

Downloaded from the site - online testing step

0% Pyloric stenosis

1938. a 47-year-old man has developed widespread edema over the past month. 10 years
ago I was treated for tuberculosis of the lungs. Objectively: the face is puffy,swollen legs,
lower back. AT-160/100 mm Hg In the blood: ep.- 3,0 • 1012/ l, lake. - 4, 5 * 109/l, ESR-50 mm / hour.
Total protein-50 g / l, albumins-42%, globulins-58%, total cholesterol-7.2 mmol / l
. in the urine: specific gravity-1020, ep. - 1-2 in p/ s, leuc. - 4-5 in p/s, hyaline cylinders-2-4 in p / s, protein-4.5
g / day. What is the most likely diagnosis?
100% Secondary amyloidosis
0% Chronic glomerulonephritis

https://translate.yandex.com/en/doc 427/540
22:20 ,27.6.2023 �� C : ; 5 B
0% Acute glomerulonephritis
0% Renal vein thrombosis
0% Chronic pyelonephritis

1939. A 27-year-old female patient developed


diplopia, bilateral ptosis, swallowing disorders, shallow breathing with a frequency of 40/min,
muscle weakness, and intestinal paresis 10 hours after eating canned mushrooms. What medical event should be held first?
100% Tracheal intubation for artificial respiration
0% Gastric and intestinal lavage
0% Introduction of anti-botulinum serum
0% Introduction of glucocorticosteroids
0% Intravenous detoxification therapy

1940. a 19-year-old patient complains of sharp pain and redness of the upper eyelid. He has been ill for 2
days. Objectively: body temperature-38.2 oC. Hyperemia and swelling of the upper eyelid, more in
the central area. There is also a sharp pain on palpation, fluctuation. The edge of the century has not
been changed. What is the most likely preliminary diagnosis?
100% Upper eyelid abscess
0% Blepharitis
0% Upper eyelid chalazion
0% Dacryoadenite
0% Phlegmon of the orbit

1941. A 41-year-old woman has been suffering from chronic cholecystitis for 8 years. Bothered by almost
constant monotonous aching pain or a feeling of heaviness in the right hypochondrium, bitterness in the mouth
in the morning, constipation. During palpation of the abdomen, there is a slight soreness at the point of projection
of the gallbladder. The volume of the bladder after a choleretic breakfast decreased only by 15% (
according to ultrasound). Which medications are most appropriate to prescribe?
100% Cholekinetics
0% M-holinolytics
0% Myospasmolytics
0% Cholespasmolytics
0% Choleretics

445

Downloaded from the site - online testing step

1942. A 27-year-old electrician was electrocuted after touching a bare


electrical wiring with his hand. Circulatory and respiratory arrest was observed. Resuscitation measures
provided restoration of cardiac activity in 5 minutes. Which of the complications is most
likely a few hours or even days after an electric shock?
100% Circulatory arrest
0% Respiratory arrest
0% Acute liver failure
0% Acute renal failure
0% Pulmonary edema

1943. A 4-year-old boy was hospitalized with complaints of shortness of breath and rapid
fatigue. A history of frequent respiratory diseases. Percussion: the heart borders
https://translate.yandex.com/en/doc 428/540
22:20 ,27.6.2023 �� C : ; 5 B
are extended to the left and up. Auscultation: amplification of the II tone above the pulmonary artery,in the II-III
intercostal space to the left of the sternum,a rough systolic-diastolic "machine” noise is heard,
conducted to all other points and to the back. What is the most likely diagnosis?
100% Open Ductus arteriosus
0% Ventricular septal defect
0% Isolated stenosis of the pulmonary artery opening
0% Atrial septal defect
0% Valvular aortic stenosis

1944. a 34-year-old woman complains of pain in the heart area (”pinching, drilling"), which occurs
mainly in the morning hours in the autumn-spring period, with pain radiating to the neck, back, stomach;
rapid heartbeat, as well as a decrease in overall vitality. The occurrence of this condition is not
related to physical activity. In the evening, the condition improves. Somatic,
neurological status and ECG - without pathology. What is the most likely pathology that caused this
clinical picture?
100% Somatized depression
0% Resting angina pectoris
0% Neurosis-like schizophrenia
0% Neurocirculatory dystonia
0% !chondriacal depression

1945. on the 4th day after suffering a cold, a patient was hospitalized with complaints of coughing with
the release of single spits of mucosal sputum. On day 2
, about 250 ml of purulent sputum with streaks of blood was released once. Objectively: the condition is of moderate severity.
PDR-28-30 / min., Ps-96 / min., AT-110/70 mm Hg. respiration over the left lung is vesicular, over
the right-weakened, various wet wheezes over the lower lobe and amphoric respiration
near the angle of the scapula. What is the most likely diagnosis?
100% Acute lung abscess
0% Exudative pleurisy
0% Acute focal pneumonia
0% Pleural empyema
0% Pyopneumothorax

446

Downloaded from the site - online testing step

1946. working under the influence of electromagnetic waves can cause functional disorders
of certain body systems. Which systems are most vulnerable to
radio frequency electromagnetic waves?
100% Nervous and cardiovascular systems
0% Gastrointestinal tract and respiratory system
0% Cardiovascular and respiratory systems
0% Cardiovascular system and water-salt metabolism
0% Nervous system and gastrointestinal tract

1947. A 30-year-old patient complained to a general practitioner of itchy skin,


which worsened in the evening. Ill for 1.5 months. Objectively: on the skin of the interdigital folds of the hands,
flexor surfaces of the limbs, abdomen, thighs, buttocks,there is a rash
consisting of paired papules covered with bloody crusts, linear combs. What additional
examination methods should be used to clarify the diagnosis?

https://translate.yandex.com/en/doc 429/540
22:20 ,27.6.2023 �� C : ; 5 B
100% Investigation of scouring of loose elements
0% Definition of dermography
0% Serological blood testing
0% Determination of blood glucose levels
0% Helminth screening

1948. A 19-year-old patient has been suffering from epileptic disease for ten years. I went to
a doctor's appointment and didn't come back. Three days later, she came home ragged and cold.
She behaved strangely: she did not talk, did not communicate with anyone, looked at one point, did not
sleep at night. The next day, the condition returned to normal. The patient could not remember what happened to her
during the last four days. To those around her, she seemed thoughtful and sleepy. What
psychopathological syndrome is suspected to have occurred in the patient?
100% Twilight disorder of consciousness
0% Stunning
0% Delusional syndrome
0% Oneiroid syndrome
0% Amenity syndrome

1949. the head of the newborn has a dolichocytic-phallic shape, elongated from front to back. During
the examination of the head on the occipital part, a generic tumor is determined, located in the middle
between the large and small crown. At what presentation of the fetal head did labor occur?
100% Posterior view of occipital presentation
0% Anterior view of occipital presentation
0% Anteroparietal presentation
0% Frontal presentation
0% Facial presentation

1950.convalescents after
infectious diseases, often and long-term ill people, and people with chronic pathology are registered at the family doctor's dispensary.
Who of
the above
100% patients should
People be enrolled
suffering from in the IIIdiseases
chronic health group?

447

Downloaded from the site - online testing step

0% Those who are often and long-term ill


0% Chronic pathology and those who are often and long-term ill
0% Recovering from infectious diseases and people with chronic pathology
0% All categories of patients listed in the condition

1951. on the 14th day after giving birth, a woman in labor went to the doctor complaining of
sudden pain, hyperemia and compaction in the left breast, fever up
to 39°C, headache, malaise. Objectively: a crack in the nipple area, an increase in the size
of the left breast, increased pain during palpation. What pathology can be assumed?
100% Lactation mastitis
0% Cyst of the left breast with suppuration
0% Fibroadenoma of the left breast
0% Breast cancer
0% Phlegmon of the breast

https://translate.yandex.com/en/doc 430/540
22:20 ,27.6.2023 �� C : ; 5 B
1952. A 42-year-old woman complains of bruising on her legs and prolonged menstruation, general
weakness, and noise in her head. Objectively: a large number of spotty hemorrhages on the legs and torso.
Tachypnea, tachycardia, systolic murmur at all points. AT-75/50 mm Hg in the blood: Er.- 1,9 •
1012/ l, HB-60 g / l, CP-0,9, lake.- 6 • 5 * 109/l, tr.- 20 • 109/ l, ESR - 12 mm / hour. The duration
of bleeding for Duke is 12 minutes. In the bone marrow-a large number of young immature forms
of megakaryoblasts without signs of platelet exfoliation. What is the most likely diagnosis?
100% Ideopathic thrombocytopenic purpura
0% Hemophilia A
0% Willebrandt's disease
0% Acute megakaryoblastic leukemia
0% Hemophilia B

1953. a 68 - year-old man complains of coughing with sputum, which has been plaguing him
for several years, a weak voice, and general weakness. Lives near
an asbestos processing plant. Objectively: in the lungs on the right - weakened breathing with an extended exhalation,
dry wheezing. On the radiograph: in the area of the root and basal zone, the case is heterogeneous, with
indistinct darkening contours, increased airiness of the lungs. ENT-paresis of the right vocal
cord. What is the most likely diagnosis?
100% Central cancer of the right lung
0% Right-sided basal pneumonia
0% Chronic dust bronchitis
0% Asbestosis
0% Tuberculosis of the lungs

1954. A 55-year-old man was admitted to the clinic due to an attack of renal colic, which
periodically recurs throughout the year. Objectively: in the area of the auricles and the right
elbow joint there are nodular formations covered with thin shiny skin.
Ps88 / min, AT-170/100 mm Hg. positive Pasternatsky symptom on both sides. The patient
is scheduled for an examination. The study of which laboratory parameter is most appropriate for
establishing a diagnosis?

448

Downloaded from the site - online testing step

100% Uric acid


0% Rheumatoid factor
0% ESR
0% Urine sediment
0% Lactic acid

1955. A 20-year-old woman complains of pain, unpleasant heavy sensation in the heart, rapid
heartbeat. ECG - no pathology. Sedatives relieved the pain,but there was still anxiety,
the certainty of a serious heart disease, the fear of death, the expectation of a new attack and the fear
of it. What is the leading psychopathological syndrome?
100% Cardiophobic
0% Hypochiondrial
0% Obsessive
0% Hysterical
0% Depressive

1956. A 60-year-old patient was admitted to the clinic complaining of shortness of breath, heaviness in the right

https://translate.yandex.com/en/doc 431/540
22:20 ,27.6.2023 �� C : ; 5 B
hypochondrium, and abdominal enlargement. The phenomena grew during the year. During auscultation of the heart-
presystolic gallop rhythm. Objectively: swelling of the cervical veins, ascites, palpable liver and
spleen. What disease should be dif-ferently diagnosed?
100% Constrictive pericarditis
0% Cirrhosis of the liver
0% Lung cancer with pleural growth
0% Chronic pulmonary heart disease
0% Pulmonary embolism

1957. A 14-year-old female patient has dramatically gained 7 kg in weight during the last year, she has frequent
head wobble, and general weakness. Objectively: the accumulation of fat is more
pronounced on the neck, chest, and abdomen. There are stretch marks on the sides of the abdomen. The mammary
glands are not sufficiently developed, there is no menstruation, and male
-type hair is observed. AT-160/100 mm Hg, heart rate-92/min. Radiographs show marked
osteoporosis and an enlarged Turkish saddle. Clinical diagnosis:
100% Itsenko-Cushing's disease
0% Itsenko-Cushing Syndrome
0% Semi-pubertal dyspituitarism
0% Lawrence-Moon-Bardet-Bidl syndrome
0% Stein-Leventhal syndrome

1958. A 40-year-old man suffers from autoimmune hepatitis. In the blood: a/G coefficient-0.8, bilirubin
- 42 mmol/l, transaminases: ALT - 2.3 mmol/l, AsAt-1.8 mmol/l. Which of the following is
most effective in treatment?
100% Glucocorticoids, cytostatics
0% Antibacterial agents
0% Iepathoprotectors

449

Downloaded from the site - online testing step

0% Antiviral drugs
0% Iemosorption, vitamin therapy

1959. A 43-year-old patient was hospitalized for a perforated stomach ulcer 5


hours after the onset of the disease. The diagnosis was confirmed by the presence of signs of pneumoperitoneum on
a survey X-ray of the abdominal cavity. Clinically, the patient develops signs of diffuse
peritonitis. Concomitant disease-schizophrenia. Doesn't have any relatives or relatives. From surgical
intervention categorically refused. What are the most appropriate actions for a curator surgeon?
Convene a consultation with the participation of the hospital administration and immediately
100%
operate on the patient according to its decision
Refuse surgical intervention and conduct comprehensive conservative
0%
treatment
0% Continue to persuade the patient until he gets his consent to the operation.
0% Conduct conservative treatment according to the Taylor method
Notify the head physician of the hospital and conduct conservative treatment until
0%
the patient agrees to the operation

1960. A 46-year-old motor seamstress complained of heart pain, general


weakness, hearing loss and fatigue. Objectively: pallor of the skin, Ps-80 / min.

https://translate.yandex.com/en/doc 432/540
22:20 ,27.6.2023 �� C : ; 5 B
Heart activity isNeuritis
100% rhythmic.
of AT-120/85
the auditorymm Hg, the audiogram showed a decrease in the threshold
nerves
of auditory sensitivity. What disease is possible in a female patient?
0% Vibration sickness
0% Neurocirculatory dystonia
0% Raynaud's disease
0% Violation of bone conduction

1961.during topographic percussion of the lungs in a patient who received a significant


barotrauma at the enterprise, it was found that the lower borders of the lungs are located one rib below the norm,
the height of the tops of both lungs and the Krenig field are significantly increased. What disease can
a doctor suggest in the first place?
100% Emphysema of the lungs
0% Exudative pleurisy
0% Chronic bronchitis
0% Bronchial asthma
0% Pneumothorax

1962. a 50-year-old patient suddenly developed pain in the occipital region, vomiting. Objectively:
sopor, hyperemia of the facial skin, blood pressure-210/120 mm Hg, Ps-B0 / min., tense,
body temperature-37.8 oC. Horizontal nystagmus. Reflexes of oral automatism are expressed.
Tendon reflexes are uniform. Stiffness of the occipital muscles, bilateral Kernig's symptom.
What is the preliminary diagnosis?
100% Subarachnoid hemorrhage
0% Hemorrhagic parenchymal stroke
0% Subdural hematoma

450

Downloaded from the site - online testing step

0% Acute hypertensive encephalopathy


0% Meningococcal meningitis

1963. a full-term child developed hemorrhages on the skin on the 3rd day, bloody vomiting,
ground blood. Hemorrhagic disease of newborns was diagnosed. What medication does your child need
as an emergency?
100% Fresh frozen plasma
0% Red blood cell mass
0% Platelet mass
0% Ethamzylate
0% Calcium Chloride

1964. A 27-year-old patient complains of unbearable headache and repeated vomiting on the 2nd day of illness
. Objectively: the condition is serious. Conscious, adynamic. In bed, he took
a forced position with his head thrown back. Skin without rash. Stiffness
of the occipital muscles, symptoms of Kern-nig, Brudzinsky are expressed. Temperature-39.5 ° C, Ps-120/min, AT-130/80 mm
Hg. what causes the leading syndrome of the disease?
100% CSF hypertension
0% CSF hypotension
0% Damage to the cranial nerve nuclei
0% Adrenal hemorrhage
0% Hyperthermia
https://translate.yandex.com/en/doc 433/540
22:20 ,27.6.2023 �� C : ; 5 B

1965. A 24-year-old patient complains of general weakness, dizziness,


fever up to 37.5 ° C, sore throat, neck edema, enlarged submandibular lymph nodes.
Objectively: the oropharyngeal mucosa is swollen and cyanotic, the tonsils are enlarged, covered
with films extending beyond them, and are difficult to remove. What is the main mechanism of development
of this disease?
100% Action of bacterial exotoxin
0% Action of bacterial endotoxin
0% Allergic component
0% Dysbiotic changes
0% Metabolic disorders

1966. A 40-year-old patient was bitten by an unknown dog an hour ago. On the left shin, the bite mark
is a wound measuring 4x2x0. 5 cm. What surgical treatment is most appropriate in this case?
100% Toilet the wound with soapy water, provision stitches on the wound
0% Aseptic dressing
0% Ointment dressing
0% Blind seam
0% Provision stitches for the wound

1967. parents of a 2-month-old child complain of frequent regurgitation and vomiting after eating
undigested milk. Objectively: restless. Body weight corresponds to age. Turgor of the soft ones

451

Downloaded from the site - online testing step

the result is satisfactory. The abdomen is moderately swollen and soft. Bowel movements 5 times a day, yellow, without
mucus. Coprogram and bacterial examination of stool without pathology. What is the most
likely diagnosis?
100% Pylorospasm
0% Pylorostenosis
0% Intestinal infection
0% Malformation of the bowel
0% Intestinal dysbiosis

1968. In a 3-year-old child with periodic abdominal pain and pyuria, ultrasound revealed
an enlarged kidney and rounded cavities connected to an enlarged bowl. The right kidney
is not changed. What is the most likely diagnosis?
100% Hydronephrosis
0% Polycystic kidney disease
0% Multicystic fibrosis
0% Echinococcus
0% Coralloid kidney stone

1969. A 60-year-old man developed dysphagia, which progresses rapidly over several
weeks. There is loss of blood, anemia. What is the most likely diagnosis?
100% Esophageal cancer
0% Foreign body of the esophagus
0% Achalasia of the cardia
0% Esophageal diverticulum
https://translate.yandex.com/en/doc 434/540
22:20 ,27.6.2023 �� C : ; 5 B
0% Keel of the esophageal orifice of the diaphragm

1970. a 3-year-old girl was taken to the surgical department in serious condition. Objectively:
Ps126 / min., body temperature -40.3 oC. The lower limb is bent at the knee, slightly brought to
the middle, passive movements in the knee and hip joints are painful. Palpation of the lower third
of the left thigh also causes a sharp concern of the patient. There is no hyperemia on the skin. Regional
lymph nodes are not enlarged. What is the most likely diagnosis?
100% Acute hematogenous osteomyelitis
0% Tuberculosis of the femur
0% Rheumatism
0% Typhoid lesion of the left femur
0% Osteosarcoma

1971. a patient complains of hip pain after an accident. The leg in the position
of flexion, adduction and internal rotation is significantly reduced. Elastic resistance when attempting
passive adduction or abduction of a limb. The big swivel is located high above
the Roser-Nelaton line. Significant lordosis is observed. What is the preliminary diagnosis?
100% Iliac hip dislocation
0% Dislocated femoral neck fracture
0% Acetabular fracture with central hip dislocation

452

Downloaded from the site - online testing step

0% Gluteal hip dislocation


0% Hip hypertrol fracture

1972. a child is undergoing inpatient treatment for acute staphylococcal


destruction of the right lung. Suddenly there was a sharp pain in the right side of the chest,
shortness of breath, cyanosis. The right side of the chest lags behind in the act of breathing. First-cut
from the bottom right-dullness, in the upper parts - box sound. The boundaries of relative cardiac dullness are shifted
to the left. What complication is most likely to occur?
100% Pyopneumothorax on the right
0% Pleural empyema
0% Spontaneous pneumothorax
0% Exudative pleurisy
0% Right lung abscess

1973. A photochronometric study of the admission of patients by general practitioners of polyclinic No. 1
in 2007 showed that 10.6% of the total time spent on preparing and familiarizing with the medical card
, 15.1% on interviewing, 35.9% on examination and examination, and 38.4% on other work elements
of the total time spent on receiving one patient. What diagrams can be used to visually
illustrate the results of the study?
100% Pie Chart
0% Cartodiagram
0% Line Chart
0% Radial diagram
0% Bar chart

1974. A 68-year-old female patient went to the doctor complaining of a tumor in her left
https://translate.yandex.com/en/doc 435/540
22:20 ,27.6.2023 �� C : ; 5 B
breast. On examination, in the upper inner quadrant of the left breast
there is a formation with a diameter of up to 2.5 cm, dense, lumpy, painless during palpation.
Regional lymph nodes are not enlarged. What is the most likely diagnosis?
100% Cancer
0% Cyst
0% Fibroadenoma
0% Mastopathy
0% Lipoma

1975. a 49-year-old patient with a prolonged attack of bronchial asthma was hospitalized. In
the lungs, breathing is weakened, a lot of dry wheezing. BDR-32 / min, AT-140/90 mm Hg, Ps-90 / min.
Made in / M injection of 1 ml of epinephrine. The condition has not improved: shortness of breath persists for 28 / min,
the number of dry wheezes in the lungs has decreased. Blood pressure rose to 170/110 mm Hg,
tachycardia 130 / min. For emergency care, it is advisable to apply:
100% Verapamil IV
0% ^- iv blockers
0% Atrovent via nebulizer
0% Chew Corinthine
0% Cordaron internally

453

Downloaded from the site - online testing step

1976.during grain harvesting in July, the outdoor temperature was 31oC, the temperature in the
driver's cabin of the combine was 35oC, the air speed was 0.2 m / s, the relative
humidity was 55%, and the temperature of the walls and ceiling was 45oC. It is possible to normalize the microclimate in the cabin
by:
100% Air cooling system
0% Increase in air velocity
0% Increase in air humidity
0% Reducing air humidity
0% Thermal insulation of walls and ceilings

1977. An 11-year-old boy has suffered from acute respiratory diseases five times in the last year
. What health group should the child be assigned to by the family doctor?
100% To the second group
0% To the first group
0% To the third group
0% To the fourth group
0% To the fifth group

1978. A baby was born from a second full-term pregnancy, a second urgent delivery from
a mother with blood type A (II) Rh_. The child's blood type is 0 (I) Rh+. The level of indirect bilirubin
in the blood from the umbilical cord was 45 mmol / l. After 3 hours, an indicator of the content
of indirect bilirubin in the child's blood serum was 170 mmol/ l. At this point, the child should
be assigned:
100% Replacement blood transfusion
0% Phototherapy
0% Phenobarbital
0% Enterosorbents
0% Infusion therapy

https://translate.yandex.com/en/doc 436/540
22:20 ,27.6.2023 �� C : ; 5 B

1979. A 22-year-old woman went to a antenatal clinic for an 11-12-week pregnancy.


During the examination, Wasserman was found to have a positive reaction. The dermatologist diagnosed
secondary latent syphilis. What are the tactics of managing this pregnancy?
100% Artificial termination of pregnancy after a course of antisyphilitic therapy
0% Artificial termination of pregnancy before the course of antisyphilitic therapy
0% Urgent termination of pregnancy
0% Prolongation of pregnancy after the first course of antisyphilitic therapy
0% Antisyphilitic treatment three times during pregnancy

1980. a 54-year-old patient has splenomegaly without any particular subjective complaints. In the blood: HB-142 g / l;
white blood cells - 32 • 109/l, E.-5%, basal -2%, myeloblasts-6%, promyelocytes-5%, myelocytes-6%,
juvenile-8%, rod-shaped-4%, C.-46%, L. - 12%, m. - 6%; ESR - 19 mm / h. For the treatment of the patient
, it is advisable to use:
100% Myelosan
0% Cyclophosphamide
0% Scheme 5+2

454

Downloaded from the site - online testing step

0% Prednisone
0% Treatment is not indicated

1981. in an 8 - year-old boy, intermittent fever with colds is observed for 3 weeks
; in febrile periods, manifestations of polyarthritis increase,
spot-like rashes appear on the trunk and face. Tachycardia, resonant tones. Hepa-tolienal syndrome.
Antibacterial therapy for 2 weeks had no effect. What is the most likely
pathology that determines the described picture?
100% Juvenile rheumatoid arthritis
0% Rheumatism
0% Sepsis
0% Systemic lupus erythematosus
0% Systemic scleroderma

1982. a 62-year-old woman complains of difficulty urinating and defecating,


protruding from the genital cleft of a tumor that interferes with walking. Gynecological status:
a tumor emerges from the genital fissure, with a hole visible at the lower pole. The walls of the tumor look like a
matte-shiny dry skin, several ulcers are visible. In the tumor, the part of the uterus that
completely leaves the genital cleft is palpated. What is the most likely diagnosis?
100% Complete uterine prolapse with decubital ulcer
0% Omission of the anterior vaginal wall
0% Incomplete uterine prolapse with cervical ulcers
0% Prolapse of the cervix with bedsores
0% Prolapse of the posterior vaginal wall

1983. a 14-year-old boy with chronic tonsillitis and sinusitis developed feelings
of interruptions in the heart and additional pulse beats. Heart rate-83 / min. On the ECG: after every two
sinus contractions, there are regular impulses in which the P Wave is absent, QRS is extended
for more than 0.11 seconds, the discordant T Wave is sharply deformed, and then a complete
compensatory pause is recorded. Specify the nature of rhythm disturbances:

https://translate.yandex.com/en/doc 437/540
22:20 ,27.6.2023 �� C : ; 5 B
100% Trigeminal extrasystole
0% Bigeminia-type extrasystole
0% Partial AV blockage
0% Complete AV blockage
0% Left bundle branch block

1984. The employee was hospitalized for pneumonia for 16 days. What
is the procedure for issuing a disability certificate in this case?
100% By a curator doctor together with the head of the department for the entire period of treatment
0% By a curator doctor from the day of admission to the hospital for a maximum of 5 days
0% By a doctor-curator from the day of admission to the hospital up to a maximum of 10 days
0% By a curator doctor from the day of admission to the hospital up to a maximum of 14 days
0% According to the conclusion of the medical advisory commission for the entire period of treatment

455

Downloaded from the site - online testing step

1985. in a child on the 3rd day of life, the skin became jaundiced. The baby was born with
a weight of 3200 kg, body length 52 cm. Active. Puerile breathing over the lungs. BH-36 / min.
The heart sounds are rhythmic. Heart rate-130 / min. The belly is soft. The liver protrudes 2 cm from under the costal arch,the
spleen is not palpable. Bowel movements in the form of meconium. What is the most likely diagnosis?
100% Physiological jaundice
0% Hemolytic disease of newborns
0% Neonatal sepsis
0% Minkowski-Shofar anemia
0% Biliary tract atresia

1986. a 36-year-old woman went to the gynecological hospital complaining of significant


bleeding from the genital tract and a monthly delay in menstruation. Bimanual examination: the cervix
of the uterus is barrel-shaped, soft consistency. The uterus is of normal size, somewhat softened.
Appendages without features on both sides. Mirror examination: the cervix is cyanotic,
enlarged in size, the outer eye is opened up to 0.5 cm. A urine test for HCG is positive.
What is the most likely diagnosis?
100% Cervical pregnancy
0% Uterine pregnancy
0% Abortion in progress
0% Threat of termination of pregnancy
0% Ectopic pregnancy

1987. A 5-year-old child's general condition worsened after acute respiratory viral infection and developed rapid
fatigue. The skin is pale, the borders of the heart are extended to the left, and the tone at the apex is dull, a gentle
systolic murmur above the apex. ECG shows signs of left ventricular overload. What
is the most likely diagnosis?
100% Non-rheumatic myocarditis
0% Rheumatic carditis
0% Congestive cardiomyopathy
0% Congenital carditis
0% Acquired heart disease

https://translate.yandex.com/en/doc 438/540
22:20 ,27.6.2023 �� C : ; 5 B
1988. during the examination of a child, it was found that he can watch a bright
toy for a long time, smiles. In the stomach position-raises and holds the head. It doesn't sit. Age
of the child, based on its psychosomatic development:
100% 2 months
0% 4 months
0% 5 months
0% 6 months
0% 7 months

1989. during the examination of a 1.5-year-old child, it was revealed that he became acutely ill,
the temperature rose to 39°C, and vomiting occurred up to 5 times. A study of the nervous system revealed
positive symptoms of Kernig and Bruzinsky. These symptoms include::
100% Meningeal signs

456

Downloaded from the site - online testing step

0% Discoordination syndrome
0% Movement disorders syndrome
0% Encephalic syndrome
0% Signs of infectious toxicosis

1990. a 2.5-month-old child developed muscle hypotension, sweating, and baldness at the back of the head. Along with
massage and therapeutic gymnastics, vitamin D is prescribed. Specify its dosage and frequency
of administration:
100% 3000 IU daily
0% 500 IU daily
0% 1000 IU daily
0% 500 IU every other day
0% 1000 IU every other day

1991. A 67 - year-old patient complains of shortness of breath, chest pain, and general weakness. He's been ill for 5
months. Objectively: t° - 37, 3°C, Ps - 96/min. Above the right lung, the vocal tremor is not
detected, the percussion sound is dull, and breathing is not listened to. In sputum-admixtures
of blood diffusely mixed with mucus. What is the most likely diagnosis?
100% Lung cancer
0% Bolshegnishchevaya pneumonia
0% Bronchiectasis
0% Focal tuberculosis of the lungs
0% Exudative pleurisy

1992. insufficiently treated industrial waste is discharged to the river, the water from
which is used for domestic drinking water supply. This causes the death
of some microorganisms, disruption of water self-purification processes and deterioration of its quality, which
can have a negative impact on people's health. This effect
of environmental factors is called:
100% Indirect
0% Direct line
0% Combined
0% Comprehensive
0% Combined

https://translate.yandex.com/en/doc 439/540
22:20 ,27.6.2023 �� C : ; 5 B

1993. an 8-month-old girl was born prematurely. During the examination: suffocation,
tachycardia, hepatosplenomegaly, lag in physical development, cyanosis of the extremities are observed.
Parasternal heart hump is determined
, systolic diastolic murmur is heard in the II intercostal space on the left, blood pressure is 90/0 mm Hg. what disease can be assumed?
100% Open Ductus arteriosus
0% Aortic coarctation
0% Aortic valve stenosis
0% Pulmonary artery stenosis
0% Non-closure of the interventricular septum

457

Downloaded from the site - online testing step

1994. 3 days ago, a boy had a foreign body removed from under his nail plate. After 2 days
, there was a sharp throbbing pain at the end of the nail phalanx, especially during pressure,
hyperemia of the nail roller, the body temperature rose to 37.5 ° C, the color of the nail plate changed
. What is the most likely diagnosis?
100% Subungual panaritium
0% Erysipelas
0% Paronychus
0% Erysipeloid
0% Abscess

1995. on the fourth day after receiving a stab wound to the right foot, the patient
's body temperature rose to 38oC, the inguinal lymph nodes increased, became painful, and the skin above
them turned red. What complication of the wound can be assumed?
100% Lymphadenitis
0% Lymphangitis
0% Phlegmon
0% Tetanus
0% Erysipelas

1996. A 50-year-old patient complained of an increase in body temperature up to 39 ° C,


throbbing pain and swelling of the right hand. The day before, I had pricked my hand with a fish bone. On
examination, there is edema and sharp pain in the palm and in the area of raising the I finger of the hand. In addition
, there is swelling of the soft tissues of the back of the hand, redness of the skin and sharp pain when
moving the fingers. What is the patient's medical condition?
100% Phlegmon of the right hand
0% Carbuncle of the hand
0% Erysipelas of the hand
0% Tendon panaritium of the first finger of the hand
0% Foreign body of the hand

1997. second emergency delivery in a 30-year-old child, second period. After one of the attempts
, there is an appearance of bloody discharge from the vagina and a deterioration in the condition of rodilli. The uterus is
highly toned, and there is a painful protrusion in the left corner of the uterus. Ps-100 / min, rhythmic,
blood pressure-160/90 mm Hg, fetal heartbeat is dull, rhythmic, 180 / min. During internal
examination: the cervix is smoothed, the opening of the eye is complete. The fetal bladder is absent.
Fetal head in the narrow part of the pelvic cavity, arrow-shaped suture in the left oblique
size. What are the doctor's tactics?

https://translate.yandex.com/en/doc 440/540
22:20 ,27.6.2023 �� C : ; 5 B

100% Accelerate delivery by using hollow obstetric forceps

0% Accelerate delivery with intravenous oxytocin


0% Prescribe antispasmodics and drugs that enhance hemocoagulation
0% Conduct treatment of fetal hypoxia and monitor the state of the parent.
Perform a perineotomy and use a Willow bandage to speed
0%
up delivery

458

Downloaded from the site - online testing step

1998. a woman with gout was admitted to the hospital. Which food items are allowed
to transfer gout patients to the hospital:
100% Kefir
0% White bread
0% Fried meat
0% Black bread
0% Fried fish

1999. A 43-year-old patient was admitted with a clinical picture of ischiorectal paraproctitis. On
the 12th day of treatment, the patient's condition worsened sharply: the level of intoxication and
liver failure began to increase, the body temperature was hectic, blood pressure was 100/60 mm Hg. Ultrasound
revealed a hydrophilic formation in the liver. In the blood: lake.- 19,6 • 109/ l, Er.- 3,0 • 1012/ l, HB-98 g / l. what
is the complication of the course of the disease?
100% Liver abscess
0% Pylephlebitis
0% Liver cyst
0% Liver necrosis
0% Budd-Chiari syndrome

2000. A 28-year-old patient was admitted to the intensive care unit with complaints of
abdominal pain, vomiting, and double vision. Objectively: disoriented, pupils dilated, reaction to
light weakened, skin dry, hyperemic, minor cyanosis, blood pressure-90/60 mm Hg, Ps-0.00 / min. From
the medical history, it is known that he consumed alcohol of unknown origin two days before hospitalization.
Specific therapy consists of intravenous administration:
100% Of ethanol
0% Sodium Bicarbonate
0% 40% glucose solution
0% Thiamine
0% Essentiale

2001. A stout 73-year-old woman developed a clinical picture of submassive pulmonary embolism on the 4th day after umbilical keel
surgery
. Duplex sonography of the
lower extremity veins revealed a floating blood clot in the femoral vein. What is the best way
to prevent
100% recurrent embolism?
Implantation of a coffee filter
0% Continuous intravenous infusion of heparin
0% Low molecular weight heparins
0% Thrombectomy
0% Plication of the inferior vena cava
https://translate.yandex.com/en/doc 441/540
22:20 ,27.6.2023 �� C : ; 5 B

2002. a 50-year-old patient has a stable increase


in jaundice and anemia with periodic increases in body temperature for one and a half months. During palpation
, an enlarged and painless gallbladder was found. The development of what disease can be assumed?
100% Cancer of the head of the pancreas
0% Gallbladder cancer

459

Downloaded from the site - online testing step

0% Cholelithiasis
0% Echinococcosis of the liver
0% Primary sclerosing cholangitis

2003. at the gestational age of 32 weeks, a pregnant woman began to give birth prematurely. Contractions in
10-15 minutes, 15-20 seconds each. Fetal heartbeat is clear, rhythmic, 145 / min. During the vaginal
examination, the cervix is shortened, the external eye is opened by 1.5 cm, the fetal bladder is intact,
the head is present, the mucous membranes are removed, the body temperature is 36.5 ° C. In the blood: no abnormalities. In
a vaginal smear, there are 4-5 white blood cells in the n / A. What is the obstetric tactic?
100% Tocolysis. Prevention of fetal distress syndrome
0% Antispasmodics, analgesics
0% Give birth through the natural birth canal
0% Caesarean section operation
0% Give intravenous anesthesia to relieve labor

2004. A shop doctor forms a group of long-term patients for in-depth observation.
At the same time, it takes into account the duration of etiologically related cases of diseases with
temporary disability during the last year for each of the workers. How
long should this duration be for the employee to be enrolled in the specified group?
100% 40 or more days
0% 60 or more days
0% 20 or more days
0% 30 or more days
0% 10 or more days

2005. A 24-year-old female patient was taken to the hospital by an ambulance team due to
complaints of cramping pains in the lower abdomen, copious, clot-filled spotting from
the genital tract, and weakness. AT-100/60 mm Hg, Ps-90 / min. Last normal menstruation 2
months ago. During the examination of the cervix, remnants of fetal
tissue are observed in mirrors. During bimanual examination: the uterus is enlarged up to 6 weeks of pregnancy,
painless, the cervical canal passes the finger. What is the preliminary diagnosis?
100% Incomplete abortion
0% Dysfunctional uterine bleeding
0% Disturbed ectopic pregnancy
0% Inflammation of the uterine appendages
0% Uterine fibroids

2006. The ambulance doctor was called to the victim. A circular saw was used to
detach the left hand at the level of the inter-wrist joint in a 25-year-old worker. There is

https://translate.yandex.com/en/doc 442/540
22:20 ,27.6.2023 �� C : ; 5 B
arterial
100%bleeding. Wherethird
Middle should a tourniquet
of the shoulder be applied to temporarily stop arterial
bleeding?
0% Lower third of the forearm
0% Upper third of the forearm
0% Elbow joint area

460

Downloaded from the site - online testing step

0% -

2007. A patient with congestive heart failure II B, systolic variant, FC III, was prescribed digoxin, kapoten,
Veroshpiron, and preduktal. After 3 weeks, loss of appetite, nausea, and dizziness occurred. On
the ECG: heart rate-52 / min, shluno-chkova bigemenia. PO2 - 0.26 seconds. Which of the medications used
provoked this condition?
100% Digoxin
0% Nitrogranulong
0% Kapoten
0% Veroshpiron
0% a-tocopherol

2008. An 18-year-old patient complains of fever


, pain and swelling in the knee joints, and a rash in the form of red rings on the lower legs 2 weeks after having a sore throat.
After a few days, I started to worry about pain in my ankles, then in my elbows.
What disease is characterized by such symptoms?
100% Acute rheumatic fever
0% Rheumatoid arthritis
0% Reactive arthritis
0% Toxic-allergic dermatitis
0% Deforming osteoarthritis

2009. A 47-year-old woman complains of joint pain and muscle weakness, rapid
fatigue, sweating, palpitations, fever. Objectively: dysphagia,
red rash around the eyes, on the cheekbones and over the interphalangeal joints, edema
of the upper eyelids, tachycardia. In the blood: increased ESR and C-reactive protein. What disease
determines this picture?
100% Dermatomyositis
0% Myasthenia gravis
0% Rheumatoid arthritis
0% Systemic lupus erythematosus
0% Systemic scleroderma

2010. A 50-year-old patient has been suffering from chronic obstructive bronchitis for 15 years.
Smokes for 30 years. During the spirographic study, it was established: vein < 40% of
the prescribed value, Fev < 40%, Tiffno index < 40%, MVL < 30%. What degree of ventilation disorders
does the patient have?
100% IV
0% II
0% I
0% III
0% 0

https://translate.yandex.com/en/doc 443/540
22:20 ,27.6.2023 �� C : ; 5 B

2011. A 29-year-old patient was hospitalized on the 10th day of his illness. Onset of the disease

461

Downloaded from the site - online testing step

gradual, worried about intense headache, lack of appetite, constipation, poor sleep,
fever up to 39oc. Objectively: the condition is severe, Ps-80 / min, AT-100/60 mm Hg. on
the skin of the abdomen, a single roseolous rash. The abdomen is healthy, the liver and spleen are enlarged.
What is the first disease that can be assumed?
100% Typhoid fever
0% Typhus fever
0% Leptospirosis
0% Flu
0% Yersiniosis

2012. A 60 - year-old woman has been experiencing weakness, dizziness,


and rapid fatigue over the past year. Recently-shortness of breath, paresthesia. Objectively: the skin and mucous
membranes are pale with an icteric tint. The papillae of the tongue are smoothed. Liver, spleen at the edge
of the costal arch. In the blood: Hb-70 g / l, Er-1, 7 * 1012/l, cP-1,2, macrocytes. Which
drug is pathogenetically justified for its use?
100% Vitamin B12
0% Vitamin B6
0% Ascorbic acid
0% Iron preparations
0% Vitamin B1

2013. a 50-year-old patient with 18 years of experience as a welder complains of suffocation and dry
cough. During X-ray examination of UCP: in the lungs, against the background of an enhanced,
reticulated pulmonary pattern, a large number of small, rounded
, clearly defined shadows are detected in the middle and lower parts on both sides, symmetrically. The roots of the lungs
are compacted, expanded. What is the most likely diagnosis?
100% Pneumoconiosis
0% Disseminated pulmonary tuberculosis
0% Miliary carcinomatosis
0% Cardiogenic pneumosclerosis
0% Sarcoidosis

2014. a 35-year-old woman complains of weakness, frequent bowel movements with blood impurities,
a decrease in body weight by 8 kg in 6 months, and periodically-an increase in temperature to 37.8 oC.
Objectively: to-37, 5oC, Ps-86 / min, AT-110/70 mm Hg. skin is pale, palpation of the abdomen is moderately
painful. In the blood: HB-92 g / l, ESR-35 mm / h. Circuloscopy: the rectal and sigmoid
colon mucosa is granular, hyperemic, edematous, unevenly thickened (pseudopolyps), bloody,
with erosions and ulcers on its surface. What pathology is most likely to cause intestinal damage in the
patient?
100% Ulcerative colitis
0% Crohn's disease
0% Irritable bowel syndrome
0% Rectal cancer
0% Intestinal polyposis

https://translate.yandex.com/en/doc 444/540
22:20 ,27.6.2023 �� C : ; 5 B

462

Downloaded from the site - online testing step

2015. a 40-year-old man was taken by ambulance after a traffic


accident with complaints of severe shortness of breath, pain in the right side of the chest and sternum,
coughing with a small amount of bright foamy blood. Objectively: the patient's condition is extremely severe,
facial cyanosis, severe subcutaneous emphysema of the chest wall, neck and face. Ps-110 / min,
rhythmic, blood pressure-90/60 mm Hg during auscultation of the lungs-on the right, breathing is sharply weakened
throughout, on the left-satisfactory. What examination method should be used to diagnose
pathology in this patient?
100% Chest X-ray and fibrobronchoscopy
0% Electrocardiography
0% Computed tomography of the brain
0% Fibroesophagoscopy
0% Ultrasound examination

2016. A 64-year-old patient on the fourth day of her stay in a therapeutic hospital with
a diagnosis of "CHD: tension angina of FC III" developed a palpitation and
her state of health deteriorated sharply. Objectively: heart sounds are dull, arrhythmic, systolic murmur at the apex.
HR94 / min., no pulse deficit. AT-130/85 mm Hg on the ECG: group, lethopal ventricular
extrasystoles, episodes of alorhythmia. What fatal clinical situation can
be expected to develop?
100% Ventricular fibrillation
0% Paroxysm of atrial fibrillation
0% Complete atrioventricular block
0% Development of sinus node weakness syndrome
0% Sinoatrial blockade

2017. a 46-year-old woman who had been suffering from hypertension for 5 years
developed a hypertensive crisis. Complaints of palpitation, throbbing sensation in the head, heart rate-100 / min,
AT-190/100 mm Hg (hyper-kinetic type of hemodynamics). Which drug should I give
preference to?
100% b-adrenoblocker
0% ACE Inhibitor
0% Diuretic
0% a-adrenoblocker
0% Dihydropyridine calcium antagonist

2018. a worker of a cement manufacturing plant, 21 years of work experience, came in with complaints of
severe itching of the skin. On examination, polymorphic rashes were detected in different areas of the skin
. First, they appeared on the skin of the hands and fingers, forearms, and face, and over
the past six months they have spread to other areas. In some places there are elements of vesiculation and
diaper rash. While on vacation, these symptoms disappeared. A skin test with chromium gave
a positive result. What is the most likely diagnosis?
100% Professional eczema
0% Limited neurodermatitis
0% Microbial eczema
0% Professional urticaria
0% Seborrheic eczema

463

https://translate.yandex.com/en/doc 445/540
22:20 ,27.6.2023 �� C : ; 5 B

Downloaded from the site - online testing step

2019. A 20-year-old patient was prescribed antibiotics due to a history of angina


, which she refused. After 2 weeks, facial edema appeared, blood
pressure increased, and the amount of urine released decreased. Objectively: AT-150/110 mm Hg in the urine: specific
gravity-1020; protein-6.3 g / l; red blood cells-30-40 p / s; single hyaline cylinders. Blood creatinine
-340 mmol / l, urea-4.2 mmol / l. what disease did the patient develop?
100% Acute glomerulonephritis
0% Acute pyelonephritis
0% Chronic glomerulonephritis
0% Chronic pyelonephritis
0% Chronic renal failure

2020. a 10-year-old boy was diagnosed with streptococcal impetigo. Which of these drugs
are most suitable for the treatment of this disease?
100% Tetracycline ointment
0% Flucinarum
0% Zovirax
0% Nizoral
0% 20% water-soap emulsion of benzyl benzoate

2021. a patient complaining of pelvic pain was taken to the sanitary checkpoint. Two
hours ago, during the explosion, he was pinned down by an overturned car. Moans in pain.
Objectively: AT - 70/40 mm Hg, heart rate-115/min. The pelvis is deformed. Shortening of the right lower
limb. Abdominal organs without pathology. Choose the best way to stop
the pain syndrome:
100% Narcotic analgesic
0% Intraosseous anesthesia in the iliac wing
0% Intra-pelvic anesthesia
0% Non-narcotic analgesic
0% Conducting anesthesia

2022. on the 6th day of life, the child developed vesicles


filled with serous-purulent contents in the back of the head,neck and buttocks, which tightly cover the skin. The child's general condition
is not
affected.
100%What disease can be assumed?
Vesiculopustulosis
0% Pemphigus of newborns
0% Sweaty skin
0% Impetigo
0% Epidermolysis bullosa

2023. a 22-year-old patient complains of coughing with the release of "rusty" sputum, episodes
of confusion, fever up to 40.1 oC, shortness of breath, muscle pain.
Objectively: BH-36 / min. On the right, below the angle of the scapula-a dull percussive sound, during auscultation
-bronchial respiration. In the blood: lake.- 17,8 • 109/ l, ESR-39 mm / hour. What is the most
likely diagnosis?
100% Non-hospital-acquired pneumonia

464

https://translate.yandex.com/en/doc 446/540
22:20 ,27.6.2023 �� C : ; 5 B

Downloaded from the site - online testing step

0% Tuberculosis of the lungs


0% Bronchiectasis
0% Acute bronchitis
0% Lung cancer

2024. A 25-year-old woman was prescribed a 10-day


complex of antibiotic therapy to treat acute pyelonephritis. After the end of taking antibiotics, thick white
vaginal discharge of a curd-like nature appeared, which was accompanied by hyperemia
of the external genitalia, pronounced itching of the vulva. For the described type of colpitis, choose a rational
therapy:
100% Antifungal medications
0% Douching with boric acid solution
0% Estrogen-containing creams
0% Vaginal suppositories with sulfonamides
0% Metronidazole

2025. a 25-year-old patient complains of infertility for three years, periodic delays
in menstruation from 2 to 4 months. Over the past year, the weight has increased by 12 kg. Severe hirsute
syndrome; basal temperature is monophasic. During the bimanual examination: the uterine body
is smaller than normal, mobile, painless, the ovaries are enlarged, dense, 5x6 cm in size, painless,
the vaginal arches are deep. What is the likely cause of infertility?
100% Sclerocystic ovary syndrome
0% Gonadal dysgenesis
0% Bilateral salpingo-oophoritis
0% Follicular cysts
0% Granulocellular tumor of the ovaries

2026. the parents of a 2-year-old boy complained to the polyclinic about the absence of the child
's right testicle in the gate. During the examination, hypoplasia of the right half of the scrotum is detected, the testicle
is missing. It is reduced in size, palpable along the inguinal canal, but does not
descend into the gate. What is the most likely diagnosis?
100% Right-sided cryptorchidism, inguinal form
0% Right testicular retraction (pseudopterychism)
0% Left-hand monarchism
0% Right-sided cryptorchidism, abdominal form
0% Ectopia of the right testicle, pubic form

2027. a diabetic patient with right-sided lower-lobe pneumonia on the 7th day
had a fever of up to 39.6°C with cooling, shortness of breath increased. Objectively: BH-26 / min.,
HR-100 / min., AT-100/60 mm Hg. below the angle of the right scapula, a dull percussive sound, breathing
is not listened to. Radiography of the OGC: to the right of the 6th rib downwards-intense darkening
of the lung tissue, the sinus does not differentiate. What complication did the patient develop?
100% Pleural empyema
0% Pneumothorax
0% Hydrothorax

465

Downloaded from the site - online testing step

https://translate.yandex.com/en/doc 447/540
22:20 ,27.6.2023 �� C : ; 5 B
0% Thromboembolism of small branches of the pulmonary artery
0% Atelectasis of the lower lobe of the right lung

2028. A 63-year-old patient complains of heart pain and shortness of breath. From the medical history, it is known
that 2 months ago she suffered an acute myocardial infarction. During the examination of the chest
, there is a pathological precardial pulsation in the IV intercostal space to the left of
the sternum. The ECG shows a steady rise in the ST interval and a negative T Wave in the thoracic leads.
During X-ray examination, a change in the configuration of the shadow of the heart due
to a sac-like protrusion on the left. What is the most likely diagnosis?
100% Post-infarction heart aneurysm
0% Repeated acute myocardial infarction
0% Mitral valve stenosis
0% Aortic valve stenosis
0% Mitral valve insufficiency

2029. a 60-year-old patient complains of an almost constant feeling of heaviness and fullness
in the epigastrium, which increases after eating, belching with a rotten smell, sometimes vomiting of food eaten 1-2
days ago, and weight loss. 12 years ago, a pyloric canal ulcer was first discovered.
Noted periodic "hungry" pains, for which he took Ranitidine. Deterioration within
3 months. Objectively: there is a "splashing noise" in the epigastrium. What complication
are we talking about?
100% Pyloric stenosis
0% Penetration of a stomach ulcer
0% Functional spasm of the goalkeeper
0% Foreign body of the stomach (bezoar)
0% Malignancy of a stomach ulcer

2030.a 23-year-old woman. After the stress, thirst, polydipsia,polyuria,


weight loss, increasing weakness appeared, then nausea, vomiting, drowsiness, fainting appeared.
She was hospitalized. Glycemia -28 mmol / l, acetone in the urine is sharply positive. Treatment
of ketoacidotic coma has been initiated. When is it advisable to start prevention of hypoglycemia by introducing
a 5% glucose solution?
100% After reducing the level of glycemia to 13-14 mmol / l
0% 2 hours after the onset of insulinotherapy
0% After the patient regains consciousness
0% After normalization of the glycemic level
0% When the rate of decrease in the level of glycemia exceeds 5 mmol / l per hour

2031. patient is 52 years old, secretary-typist, work experience of 30 years. Complains of cramps in
the right hand while working and inability to type or write. The load on
the brush is up to 80% of the working time. He has been ill for 2 years. Objectively: the right hand is tense,
the muscle tone is increased, and convulsions occur during writing attempts. During the examination, no pathological
manifestations of the central nervous system were found. What is the most likely diagnosis?
100% Convulsive form of coordination neurosis
0% Neuralgic form of coordination neurosis

466

Downloaded from the site - online testing step

0% Paretic form of coordination neurosis


0% Hysterical neurosis

https://translate.yandex.com/en/doc 448/540
22:20 ,27.6.2023 �� C : ; 5 B
0% Chronic manganese intoxication

2032. an electric welder with 15 years of work experience was found


to have dry wheezing in the lower parts of the lungs during a medical examination. Diffuse nodules
3-4 mm in size in the middle and lower parts of the lungs are observed on the X-ray. What disease can be assumed?
100% Metallokonioz
0% Silicosis
0% Silicatosis
0% Carboconiosis
0% Bronchitis

2033. a 7-year-old girl complains of pain in the vaginal area, significant purulent
discharge, which bothers for 5 days and gradually increases. During the examination, the doctor
found significant swelling of the external genitalia, redness, purulent discharge from the vagina with
an unpleasant smell. During ultrasound, an echo-positive shadow is detected in the vaginal area. What
reason can lead to this condition in children?
100% Presence of a foreign body in the vagina
0% Vaginal tumor
0% Cervical tumor
0% Purulent colpitis
0% Vulvovaginitis

2034. a 22-year-old female patient of reduced nutrition, vegetarian, applied to the polyclinic with
complaints of distortion of the sense of smell, taste, ”jamming” in the corners of the mouth. Objectively: pronounced blueness
of the sclera. The diagnosis was iron deficiency anemia. Which clinical syndrome has
the advantage?
100% Sideropenic
0% Anemic
0% Gemological Center
0% Hemolytic
0% Myelodysplastic

2035. A 49-year-old patient experienced burning pain for the first time in his life
, localized in the heart area, without radiation, after significant emotional stress. The duration of the attack is about
12 minutes. I took 1 tablet of nitroglycerin, which relieved the pain. During the inspection in 20 minutes:
increased nutrition, muted heart tones, blood pressure-140/90 mm Hg, heart rate-82 / min.,
vesicular respiration, no peripheral edema was detected. ECG - no pathological signs. What
is the preliminary diagnosis?
100% First-time angina pectoris
0% Progressive angina pectoris
0% Condition after uncomplicated hypertensive crisis
0% Asthenoneurotic syndrome
0% Small-focal myocardial infarction

467

Downloaded from the site - online testing step

2036. A 67-year-old female patient complains of swelling of the face and legs, lower back pain that increases
during movement, sharp weakness, sometimes nosebleeds, and an increase in body temperature up to 38.4 ° C.
Objectively: pain on palpation of the spine and ribs. During a laboratory examination:
proteinuria -4.2 g / day, ESR-52 mm / hour. What changes in laboratory parameters are most likely?

https://translate.yandex.com/en/doc 449/540
22:20 ,27.6.2023 �� C : ; 5 B
100% Total serum protein-101 g / l
0% White blood cells -15.3 g / l
0% Hemoglobin-165 g / l
0% Albumins-65%
0% y-globulins -14%

2037. According to the annual report of the city hospital's inpatient department, data were obtained on the actual
number of bed days and the number of patients treated during the year. Which of
the hospital performance indicators can be calculated based on these data?
100% Average length of a patient's hospital stay
0% Average bed occupancy
0% Bed turnover
0% Lethality rate
0% Efficiency of using the bed fund

2038. A 20-year-old man was taken to the hospital with bleeding after tooth extraction, which lasts
for 4 hours. Objectively: the skin is pale,the right knee joint is deformed,
and movement is restricted. In the blood: Er.- 3,2 • 1012/ l, Hb-98 g / l, CP-0.92, lake.- 7,4-109/ l,
blood clot.240-109/l, ESR-11 mm / h. Bleed time according to Du-com is 3 minutes, blood clotting time according
to Lewhite is 20 minutes. What is the most likely diagnosis?
100% Hemophilia
0% Werlhof's disease
0% Hemorrhagic vasculitis
0% Randu-Osler's disease
0% DIC syndrome

2039. A 20-year-old patient is being treated by a hematologist for hemophilia A. After falling from
a horizontal bar, acute hemarthrosis of the knee joint has developed. Objectively: the right knee joint is sharply
enlarged in volume, the skin over it is hyperemic. Duke's bleeding time is 3 minutes,
Lee-White's clotting time is 20 minutes. Which drug is most effective?
100% Recombinant factor VIII
0% Recombinant X Factor
0% Tromboconcentrate
0% Recombinant factor IX
0% Aminocislaproic acid

2040. a 47-year-old man suffers from liver cirrhosis of unknown etiology. Objectively: the abdomen
is enlarged, the presence of free fluid in the abdominal cavity is established. What diuretic
drug should be included in the complex therapy of this patient?
100% Veroshpiron
0% Hypothyazide

468

Downloaded from the site - online testing step

0% Mannitol
0% Diacarb
0% !ndapamide

2041. in a 23-year-old woman in labor, on the tenth day after giving birth, her general condition worsened:
a cold appeared, pain in the right mammary gland, and her body temperature rose to 39oc. Objectively:
https://translate.yandex.com/en/doc 450/540
22:20 ,27.6.2023 �� C : ; 5 B
the right mammary gland is slightly enlarged, a painful
dense infiltrate measuring 5x7 cm with a softening area in the center is palpated in the Upper-outer quadrant, the skin above it
is hyperemic with a bluish tinge. Axillary lymph nodes on the right are enlarged, painful during
palpation. What is the most likely diagnosis?
100% Purulent mastitis
0% Lactostasis
0% Serous mastitis
0% Breast fibroadenoma
0% Breast cancer

2042. A 38-year-old female patient complains of hot flashes and a feeling of heat that recurs up to 5
times a day, headache in the occipital region with increased blood pressure,
palpitation, dizziness, rapid fatigue, irritability, memory loss. 6
months ago, an operation was performed to extirpate the uterus with appendages. What is the most
likely diagnosis?
100% Post-castration syndrome
0% Premenstrual syndrome
0% Early pathological menopause
0% Secondary psychogenic amenorrhea
0% Physiological premenopause

2043. A 28-year-old patient developed severe weakness, dizziness,


darkening of the eyes, nausea and loss of consciousness without convulsions after stress. Objectively: the patient is unconscious, her skin
is pale, and her limbs are cold. Pupillary and tendon reflexes are preserved. AT-80/50 mm Hg,
PsI02 / min., low filling. What is the most likely diagnosis?
100% Syncopal state
0% Epileptic syndrome
0% !steric seizure
0% Vegetative-vascular paroxysm
0% Transient cerebral circulatory disorders

2044. in a two-year-old child with a satisfactory condition, moderate


proteinuria and microhematuria are periodically noted. During ultrasound examination, the left kidney is not detected,
the right kidney is enlarged in size, there are signs of doubling of the calico-pelvic system. What
kind of research does the child first need to conduct to clarify the diagnosis?
100% Excretory urography
0% Mycological cystography
0% Retrograde urography
0% Renal vascular Doppler imaging

469

Downloaded from the site - online testing step

0% Radioisotope scanning of the kidneys

2045. as a result of the explosion of a benzene tank at a chemical plant, there are many dead and injured
(more than 50 people) with burns, mechanical injuries and poisoning. Specify
the main elements that provide medical evacuation support for the population in this
situation:
100% Triage, medical care, evacuation
0% Sorting, evacuation, treatment

https://translate.yandex.com/en/doc 451/540
22:20 ,27.6.2023 �� C : ; 5 B
0% Medical assistance, evacuation, isolation
0% Isolation, rescue, recovery
0% Sorting, updating, saving

2046. A 19-year-old patient has been sluggish and indifferent to himself and others for the last six months. He started
to study poorly, does not assimilate the material, and began to skip classes. Communicates little, withdrawn into
himself, emotions are indistinct. He spends most of his time in bed, is untidy, indifferent to
appearance, does not wash, and is prone to impulsive actions. What is the most likely
diagnosis?
100% Simple schizophrenia
0% Catatonic schizophrenia
0% Hebephrenic schizophrenia
0% Paranoid schizophrenia
0% Residual schizophrenia

2047. A 35-year-old female patient experienced weakness, sweating, subfebrile


fever, pain, and restricted movement in her left knee joint about a year ago. 2 years ago, she was treated for
disseminated pulmonary tuberculosis. Objectively: the position of the left limb is forced,
the knee joint is enlarged, painful during palpation, movement in it is limited. Mantoux
test Z 2 TO - papule 23 mm. In the lungs, focal shadows of increased intensity. What is the preliminary
diagnosis?
100% Tuberculosis drives
0% Post-traumatic arthritis
0% Chronic osteomyelitis
0% Knee joint tumor
0% Rheumatoid arthritis

2048. A 64-year-old diabetic patient was treated with metformin. Due to high
blood pressure, I took diuretics. Gradually, the effectiveness of the diuretic decreased,
nausea and vomiting appeared. Contact with the patient is difficult. The skin is dry.
There is no smell of acetone. AT-180/100 mm Hg. heart sounds are deaf. Ps-98 / min. Respiration is vesicular. The abdomen
is painful in the epigastrium. Liver + 4 cm. Blood glucose-48 mmol / l, Na-156 mmol/ L, K-5.2
mmol/ L, urea-15 mmol / l. what pharmacological approach should I start with?
100% Emergency hydration of the patient
0% Correction of Na + content in the blood
0% Purpose of sodium bicarbonate
0% Correction of blood pressure with sodium sulfate

470

Downloaded from the site - online testing step

0% Transfer the patient to protamine-zinc insulin treatment

2049. a 7-year-old child was hospitalized with complaints of fever up to 39.8°C,


lethargy, moderate headache, and vomiting. Examination revealed meningeal symptoms.
A lumbar puncture was performed. A liquid under high pressure, transparent, cytosis of 450
cells in 1 µl (mainly lymphocytes-90%), glucose content-2.6 mmol/l was obtained. what pathogen
can be caused by the disease in a child?
100% Enterovirus
0% Meningococcus
0% Tuberculosis bacillus

https://translate.yandex.com/en/doc 452/540
22:20 ,27.6.2023 �� C : ; 5 B
0% Staphylococcus aureus
0% Pneumococcus

2050. A 28-year-old female patient discovered a tumor in the upper outer


quadrant of the right breast during self-treatment. During palpation - a painless, solid, mobile formation in
the mammary gland with a diameter of 2 cm, peripheral lymph nodes are not changed. Ultrasound
examination of the mammary glands: in the upper outer quadrant of the right mammary gland
, a volume formation of increased echogenicity, measuring 21x18 mm. What is the most likely
diagnosis?
100% Fibroadenoma
0% Breast cyst
0% Diffuse mastopathy
0% Breast cancer
0% Mastitis

2051. A 23-year-old patient complains of a tumor in the lower outer quadrant


of the left breast during the 1st year, which becomes painful and increases
in size before menstruation. During palpation: a mobile formation filled with fluid, up to 3 cm, with clear
contours, peripheral lymph nodes are not changed. As a result of ultrasound
examination of the mammary glands: in the Lower outer quadrant of the left mammary gland
, a volume formation of reduced echogenicity, measuring 31x29 mm, was found. What is the preliminary
diagnosis?
100% Breast cyst
0% Fibroadenoma
0% Fibrotic mastopathy
0% Breast cancer
0% Mastalgia

2052.A 3-year-old child was prescribed Biseptol, paracetamol,and nasoferon due to acute respiratory viral
infection. On the third day, the child's condition worsened: there was a sore throat, stomatitis,
conjunctivitis, hypersalivation, painful spots of dark red color on the neck, face, chest and
limbs, then blisters appeared in place of the spots. There was a lesion of the mucous membranes
around the mouth and anus. What is the preliminary diagnosis?
100% Stevens-Johnson syndrome
0% Atopic dermatitis

471

Downloaded from the site - online testing step

0% Chickenpox
0% Serum sickness
0% Bullous dermatitis

2053. a 22-year-old woman complains of tearfulness,depressive mood,aggressiveness,


drowsiness, loss of appetite, roughness of the mammary glands, which are observed 4-5 days
before menstruation and disappear after it begins. Menstruation after 31 days, within 4-5 days,
regular, painless, unexplained; sexual life lives from 20 years; pregnancies-0. During
the vaginal examination, no pathology of the genitals was detected. What is the most likely diagnosis?
100% Premenstrual syndrome
0% Dysmenorrhea
0% Fibrocystic mastopathy

https://translate.yandex.com/en/doc 453/540
22:20 ,27.6.2023 �� C : ; 5 B
0% Psychoneurotic syndrome
0% Thyrotoxicosis

2054. a 38-year-old man works in the zone of ionizing radiation. During the
periodic medical examination, he does not make any complaints. In the blood: Er. - 4, 5 1012/l, Hb-80 g/l,
leuc.2.8 • 109/l, platelets -30 * 109/l. Can this person be allowed to work with
ionizing radiation sources?
It is contraindicated to work with radioactive substances and other sources
100%
of ionizing radiation
0% Allowed to work with radioactive substances
0% It can only work with low-activity radioactive substances
0% Allowed to work after an extended medical examination
0% It is allowed to work with radioactive substances with a limited working time

2055. a 24-year-old woman in labor has a postpartum period, on the 4th day. The delivery was the first,
timely, without complications. The general condition of the woman in labor is satisfactory. Body temperature-36.6°C.
Ps78 / min, rhythmic. Mammary glands in a state of engorgement. The bottom of the uterus is 2 cm below the navel. The uterus is in
normal condition, painless. Lochia are bloody, moderate. Stool, urination is normal. What
treatment is appropriate?
100% Uterotonic drugs
0% Analgesics
0% Antibiotics
0% Estrogenic hormones
0% Medications that stop lactation

2056. on the 3rd day after instrumental revision of the uterine cavity for
termination of pregnancy, a woman complained of pain in the lower abdomen, an increase in body temperature
to 37.4°C. Bimanually: the uterus is enlarged to 6-7 weeks of pregnancy, painful on palpation.
Purulent-sukrovichnye discharge is observed. What pathology can be assumed?
100% Post-abortion metroendometritis
0% Post-export parametrite
0% Post-abortion peritonitis
0% Post-abortion thrombophlebitis

472

Downloaded from the site - online testing step

0% Post-abortion subinvolution of the uterus

2057. a 47-year-old patient is concerned about a tumor-like formation on the anterior surface of the neck in the
thyroid gland. Notes a progressive increase in education. Objectively: in the right lobe
of the thyroid gland there is a formation with a diameter of about 4 cm with a smooth surface,
mobile, high density. Slightly expressed symptoms of thyrotoxicosis. Which of
the additional examination methods is the most informative for clarifying the diagnosis?
100% Fine needle puncture biopsy
0% Neck radiography
0% Ultrasound examination
0% Determination of thyroid hormones
0% Determination of protein-bound iodine

2058. A 35-year-old patient was hospitalized with fever attacks, which were accompanied

https://translate.yandex.com/en/doc 454/540
22:20 ,27.6.2023 �� C : ; 5 B
by sweating and repeated every 4 days. Objectively: subictericity of sclera, pallor
of the skin, enlargement of the liver by 2 cm and its compaction, enlargement of the spleen by 5 cm. What
kind of research will allow you to verify the diagnosis?
100% Microscopy of a thick drop and a blood smear for malarial plasmodium
0% Determination of the level of bilirubin in the blood serum
0% Serological blood testing
0% Study of AlAT and AsAT activity
0% Biochemical blood test

2059. A 35-year-old patient went to the doctor complaining of a painful formation in the
back of the neck, which appeared 1.5 weeks ago and is accompanied by soreness and
headache. Objectively: the indicated area has a sharply painful infiltrate of bright red color with
puffiness around. Pus is released from the hole on the surface of the infiltrate. During palpation
, there is a distinct soreness. What is the most likely diagnosis?
100% A boil
0% Neurodermatitis
0% Eczema
0% Scabies
0% Toxidermy

2060. a 64-year-old patient has been suffering from diabetes mellitus for 14 years. About three weeks
ago, the skin of the distal phalanx of the first toe of the left foot turned cold, bluish-black. The pain
is not intense. Pulse on the arteries of the foot is not detected, on the popliteal-preserved.
The glycemic level is 12.4 mmol/l. Ultrasound scanning - stenosis of the lower leg arteries, bloodshed-collateral
compensated. Ankle index-0.7 on the radiograph of the foot-destruction
of the distal phalanx of the first finger. It will determine the degree of diabetic foot by Wagner:
100% The fourth
0% First
0% The second one
0% Third
0% Fifth

473

Downloaded from the site - online testing step

2061. A 3-year-old girl who was not vaccinated due to her parents ' refusal to take preventive vaccinations
became acutely ill: her body temperature rose to 38 ° C,nasal mucosa appeared,
and coughing. After 3 days, the body temperature dropped to 36.6 oC. In the morning, the girl
complained of pain in her right leg and stopped standing up. The hospital examination
revealed restricted movement, reduced muscle tone, and the absence of tendon reflexes in the
right leg; sensitivity was preserved. What is the most likely diagnosis?
100% Polio
0% Encephalitis
0% Enterovirus infection
0% Polyneuropathy in diphtheria
0% Acute infectious myelitis

2062. a 5-year-old child attending kindergarten, after 3 days of moderate lethargy


, developed fever up to 39oc, membranous tonsilopharyngitis, cervical lymphadenopathy in combination
with hepatosplenomegaly. The pediatrician diagnosed infectious mononucleosis, which was confirmed
by serological studies. What quarantine measures are carried out in the kindergarten?
100% Anti-epidemic measures are not carried out

https://translate.yandex.com/en/doc 455/540
22:20 ,27.6.2023 �� C : ; 5 B
0% Group quarantine for 21 days
0% Vaccination of non-immune persons
0% Administration of specific immunoglobulin to contact persons
0% Prescribing acyclovir to all contact persons

2063. A 25-year-old patient was admitted to the emergency department of a multidisciplinary hospital in
an unconscious state. Breathing is difficult, the eyes do not flatten themselves, the pupils are sharply narrowed,
there is no speech, tonic flexion in response to a painful stimulus. On the skin of the inner side
of the shin, in the groin, traces of injections. Name the first priority measure of emergency care for the patient:
100% Administration of naloxone
0% Introduction of tranquilizers
0% Intramuscular administration of stimulating neuroleptics
0% Intramuscular administration of bemegrid
0% Administration of adrenomimetics

2064. a 32-year-old patient complains of acute "dagger" abdominal pain that suddenly occurs
in the epigastrium against the background of complete health. there was an increase in body temperature to 38.7 ° C,
a complex of medical measures is used, which are performed by professionally trained
specialists who have the right to do so in accordance with the legislation. What components of medical care
should be included in this case?
Examination of the patient, diagnosis of the disease, urgent hospitalization and
100%
urgent surgery
0% Admission to a surgical hospital and urgent surgery
0% Urgent hospitalization and medical treatment
0% Hospitalization in surgical statsyunar and elective surgery
Examination of the patient, diagnosis of the disease, planned hospitalization and
0%
drug therapy

474

Downloaded from the site - online testing step

2065. the mother of a 1-year-old child complains of a constant obsessive, frequent unproductive
cough, sometimes to the point of vomiting. During an objective study, the patient has
rapid breathing, moderate retraction of the lower intercostal muscles, and an increase
in the anteroposterior size of the chest. During respiratory infections
, bronchial obstruction occurs. During the neonatal period, he suffered meconial ileus. Choose
a priority survey:
100% Research on sweat chlorides
0% Research on chlamydia and mycoplasma
0% Chest radiography
0% CT scan of the lungs
0% Genetic testing

2066. among all registered diseases of the population served


by the city polyclinic, patients with diabetes mellitus accounted for 21%. What statistical indicator is given?
100% Extensive
0% Intensive
0% For visibility purposes
0% Ratio
0% Correlations
https://translate.yandex.com/en/doc 456/540
22:20 ,27.6.2023 �� C : ; 5 B

2067. a 35-year-old patient experienced a sharp headache, palpitation,


and cold after emotional stress. In the anamnesis: has been suffering from arterial hypertension for 5 years. The patient is agitated,
flushing
of the face, tremor of the limbs, the skin is moistened. Ps-100 / min., AT-180/100 mm Hg. which drug
should be prescribed
100% first?
Bisoprolol
0% Nifedipine
0% Furosemide
0% Dibazol
0% Clonidine

2068. a 23-year-old patient was delivered urgently with complaints of pain in the lower abdomen, a more intense
case, with radiation to the rectum, dizziness. The above complaints appeared suddenly
at night. Last menstruation 2 weeks ago. Objectively: the skin is pale, Ps-92 / min,
body temperature-36.6 oC, blood pressure-100/60 mm Hg. the abdomen is somewhat tense,
slightly painful in the lower parts, the symptoms of irritation of the peritoneum are slightly positive.
Hemoglobin-98 g / l. what is the preliminary diagnosis?
100% Ovarian apoplexy
0% Disturbed ectopic pregnancy
0% Acute appendicitis
0% Intestinal obstruction
0% Renal colic

2069. a 62-year-old man went to a urologist with complaints of frequent urination at night
(5-6 times), a feeling of incomplete release of the bladder, pain in the lower abdomen, slowing
down the speed of urination. In the anamnesis-hypertension of stage II (maximum blood pressure figures-

475

Downloaded from the site - online testing step

160/100 mmHg). Palpation of the prostate gland is enlarged to the second degree. PSA is 2.2
ng / ml. Determine the drug for long-term therapy of combined pathology in the patient:
100% Doxazosin
0% Propranolol
0% Indapamide
0% Amlodipine
0% Captopril

2070. A 20-year-old patient with polyarthritis was admitted to the rheumatology department. On
the skin of the face - redness in the form of a "butterfly" in the urine: protein-4.8 g / l. the doctor suggested
that the patient has systemic lupus erythematosus. What additional research method is
the most informative for verifying the diagnosis?
100% Blood test for antinuclear antibodies
0% General blood test
0% Immunological blood testing
0% LE-cell blood test
0% Blood test for rheumatoid factor

2071. after eating fried eggs made at home from duck eggs, patients experienced
the following symptoms of the disease after 8 hours: fever-39oc, headache, vomiting, abdominal pain,

https://translate.yandex.com/en/doc 457/540
22:20 ,27.6.2023 �� C : ; 5 B
and subsequently diarrhea. Bowel movements are frequent, mucus-filled, and smelly. The duration of the disease is 3 days.
What is the microbial nature of the disease?
100% Salmonellosis
0% Food-borne botulism
0% Brucellosis
0% Tularemia
0% Anthrax

2072. on the 4th day after surgery for purulent mastitis, the patient's general
condition worsened, the temperature increased to 40oC, the pulse rate was 110 / min, and the respiratory rate was 24 / min.
Blood pressure-100/60 mm Hg in the blood test the number of white blood cells - 18 • 109/l, The number
of rod-shaped forms is 19%. What complication can be assumed?
100% Sepsis
0% Suppuration of the wound
0% Gas gangrene
0% Peritonitis
0% Acute respiratory disease

2073. in the morning, the patient experienced nausea, single vomiting, and dry mouth. In the evening, I noticed
double vision of objects, " fog " in front of my eyes, chowder while swallowing liquid food. Objectively:
body temperature-36.4 oC, ptosis, mydriasis, anisocoria, lack of gag and swallowing
reflexes, dry mucous membranes. No abnormalities were detected on the part of the internal organs. What
is the most likely medical condition?
100% Botulism
0% Acute cerebrovascular accident

476

Downloaded from the site - online testing step

0% Viral meningoencephalitis
0% Tuberculosis meningoencephalitis
0% Stem encephalitis

2074. a 15-year-old girl was diagnosed with chronic cholecystocholangitis 2 years ago. I didn't
follow a diet. The condition has worsened in the last 3 months. There is an increase in body temperature.
Abdominal pain of paroxysmal nature after fatty, spicy food. Worries about itchy skin.
The tongue is covered with a white coating. Stomach is soft, liver + 3 cm, palpation is painful, positive
cystic symptoms. In the blood: white blood cells -12109/l, erythrocyte sedimentation rate -20 mm / h.
Which drug should be included in the treatment package?
100% Antibiotics
0% Choleretic
0% Prokinetics
0% Pancreatic enzymes
0% Enterosorbents

2075.A 40-year-old woman went to a women's clinic complaining of


menstrual irregularities such as hyperpolymenorrhea for six months, pulling pains in the lower
abdomen, and weakness. During the gynecological examination, the body of the uterus is enlarged to 12
weeks of pregnancy, dense, mobile, painless. In the blood: Hb-90 g / l. what pathology can
be assumed?
100% Uterine fibroids
0% Cancer of the uterine body
https://translate.yandex.com/en/doc 458/540
22:20 ,27.6.2023 �� C : ; 5 B

0% Pregnancy
0% Ovarian cysts
0% Dysfunctional uterine bleeding

2076. A 26-year-old repeat mother arrived at the maternity ward due to


a 40-week pregnancy and the onset of labor. 2 hours ago, the amniotic fluid left.
Fetal position is longitudinal, main presentation. Coolant - 100 cm, VDM-42 cm. Grab it every
4-5 minutes, for 25 seconds. During internal obstetric examination:
the cervix is smoothed, the opening is 4 cm. There is no fetal bladder. The fetal head is pressed against the entrance to the
small pelvis. What complication occurred during childbirth?
100% Premature discharge of amniotic fluid
0% Primary weakness of labor activity
0% Secondary weakness of labor activity
0% Discoordinated labor activity
0% Clinically narrow pelvis

2077. a 37-year-old female patient has been ill for more than 10 years. She says that her brain has long
been possessed by "criminal scientists-physicists" who are testing various types of psychotropic weapons on her.
Feels the effects of laser beams, constantly hears messages that are transmitted
directly to her brain. Emotionally monotonous, she spends most of her days in bed, making
notes that she doesn't show to anyone. Clear consciousness, no formal intellectual and mnestic
disorders were detected. What is the diagnosis of this patient"?

477

Downloaded from the site - online testing step

100% Paranoid schizophrenia


0% Chronic delusional disorder
0% The involutional paranoid
0% Schizo-affective psychosis
0% Reactive Paranoid

2078. a 13-year-old girl has been complaining for 5 years of pain in the right hypochondrium
radiating to the right scapula, attacks of pain are associated with a violation of the diet, they are short-lived, easily
relieved by antispasmodic agents. During an attack of pain, palpation of the abdomen is painful,
as much as possible at the point of projection of the gallbladder. The patient is most likely to have:
100% Biliary dyskinesia
0% Chronic cholecystitis
0% Chronic gastroduodenitis
0% Chronic pancreatitis
0% Duodenal ulcer disease 12

2079. during a medical examination, the patient was found to have: stunted growth, hypogonadism,
sexual infantilism, lack of secondary sexual characteristics, enlarged liver and spleen.
This indicates a lack of nutrition of such an element.:
100% Zinc
0% Calcium
0% Copper
0% Iodine
0% Vitamin D

https://translate.yandex.com/en/doc 459/540
22:20 ,27.6.2023 �� C : ; 5 B

2080.A 37-year-old patient


who was temporarily disabled for 117 days during a year due to an exacerbation of chronic obstructive bronchitis applied to the district
therapist.
Determine
100% the legally correct
The patient is tactics
subjectof
tothe doctor
referral to in
thethis case: and social expert commission
medical
0% Extension of the disability certificate by the therapist
Referral to the medical consultation commission for the extension
0%
of the disability certificate
0% Issue of a labor (extra-paid) disability certificate
0% Referral of the patient for sanatorium treatment

2081. giving birth for the first time at the age of 30. Head in the pelvic cavity. The fetal heartbeat
began to slow down, and an arrhythmia appeared. What should I apply?
100% Abdominal obstetric forceps
0% Weekend Obstetric Forceps
0% Caesarean section
0% Perineotomy
0% Skin-head forceps

2082. a 20-year-old patient complains of periodic dizziness, headache, and nasal congestion.

478

Downloaded from the site - online testing step

bleeding, aching pain in the heart, leg muscle cramps, cold feet. Objectively:
physical development is accompanied by a disproportion of the muscular system (the muscles of the upper half
of the body are hypertrophic, with relative hypotrophy of the pelvic and lower limb muscles), the legs
are cold to the touch. Pulse-86 / min., rhythmic, blood pressure: on the hands-200/100 mm Hg, on the legs
-140/90 mm Hg. what disease can be assumed in the first place?
100% Aortic coarctation
0% Hypertension
0% Neurocirculatory dystonia
0% Takayasu's disease
0% Obliterative endarteritis

2083. A 46-year-old female patient has been suffering from diabetes mellitus for 9 years and receives 26 units of Mono-tard insulin
in the morning and 18 units in the evening. Complaints of weakness, lethargy in the morning after sleep, headache, sweating
at night. Objectively: pulse-72 / min., blood pressure-125/70 mm Hg. the heart borders are normal.
Liver + 4 cm. Blood sugar: 8: 00-14 mmol / L, 12: 00-9 mmol/ L, 17: 00-11 mmol/l. Urine sugar
in the range of 0.5-1%. What are the most likely causes of complaints that occur in the patient at night?
100% Excess dose of insulin in the evening
0% Insufficient Evening dose of insulin
0% Presence of hepatosis
0% Menopausal syndrome
0% Insufficient dose of insulin in the morning

2084. a 43-year-old patient complains of coughing with a small amount


of colorless sputum, pain in the right side of the chest when breathing, shortness
of breath, fever up to 39oc. He became acutely ill. I was taking an aspirin. Objectively: on the lips of herpes. In
the projection of the lower lobe of the right lung, there is a dulling of percussion sound, increased

https://translate.yandex.com/en/doc 460/540
22:20 ,27.6.2023 �� C : ; 5 B
vocal tremor, and bronchial respiration. Radiologically: homogeneous darkening of the particle with
100% Streptococcus pneumoniae
clear contours. What is the most likely etiology of pneumonia?
0% Staphylococcus pneumoniae
0% Mycoplasma pneumoniae
0% Legionella pneumoniae
0% Klebsiella pneumoniae

2085. A 24-year-old female patient went to the gynecologist with complaints about the appearance of outgrowths in
the genital area. After examining the patient, the doctor found
papillary growths resembling cauliflower, soft consistency, non-painful, non-erosive on the large and small bashful
lips. The patient is referred for consultation to a dermatologist. What is the most
likely diagnosis?
100% Genital warts
0% Broad warts
0% Vegetative pemphigus
0% Granulomatous candidiasis
0% Papillomatosis

479

Downloaded from the site - online testing step

2086. a 3-month-old boy was hospitalized due to prolonged jaundice and persistent
constipation. Patient since birth. The mother's pregnancy was complicated by toxicosis. During
the examination, little active, the face is swollen, macroglossia, icteric skin. Narrow eye slits.
Muscle tone is reduced. Bradycardia. What is the most likely diagnosis?
100% Congenital pylorostenosis
0% Down's disease
0% Rickets
0% Cystic fibrosis
0% Hirschsprung's disease

2087. a 21-year-old patient without a permanent place of residence complains of loose


stools for 2 months, weight loss of 13 kg, weakness, constant subfebrile
temperature, recurrent herpes. On the lips, herpetic rashes, generalized
lymphadenopathy, liver enlargement up to 2 cm. In the blood: red blood cells - 4,4 • 1012/l, Hb-115 mg/l,
erythrocyte sedimentation rate -15 mm / h, leukocytes -10.0 * 109/ l, E.-2%, P.-6%, C.-61%,
L. -17%, M.-3%. Atypical mononuclears-6%. What is the most likely diagnosis?
100% hiv infection
0% Intestinal amoebiasis
0% Infectious mononucleosis
0% Lymphogranulomatosis
0% Sepsis

2088. a 64-year-old patient complains of general weakness, noise in the head, hoarseness of voice.
Objectively: pallor with an icteric tinge, red tongue with smooth papillae,
asymmetry of tactile and pain sensitivity. Pulse-120 / min, blood pressure-80/50 mm Hg.
palpable spleen. In the blood: HB-58 g / l, red blood cells -1.24 * 1012 / l, white blood cells - 2,8-109/l,
platelets - 140 * 109/l, erythrocyte sedimentation rate-17 mm / h, anisocytosis, poikilocytosis
-expressed (++). What research will be crucial to elucidate the genesis of anemia?
100% Sternal puncture

https://translate.yandex.com/en/doc 461/540
22:20 ,27.6.2023 �� C : ; 5 B
0% Indirect Coombs test
0% Direct Coombs test
0% Fibrogastroscopy
0% Lumbar puncture

2089. A 20-year-old patient was delivered by an emergency medical team in a comatose


state. The pupils are sharply narrowed, cyanosis and dry mucous membranes,the skin of the face and trunk is pale,
moist and cold. Pulse-60 / min., blood pressure-80/60 mm Hg, respiratory rate-8-10 / min.
What is the most likely diagnosis?
100% Opiate poisoning
0% Atropine poisoning
0% Barbiturate poisoning
0% Poisoning with benzodiazepine derivatives
0% Alcohol surrogate poisoning

2090. a 35-year-old woman complains of pain in the heart area ("pinching ’ "drilling"), which

480

Downloaded from the site - online testing step

it occurs mainly in the morning hours in the autumn-spring period, with radiating pain in the neck,
back, stomach; rapid heartbeat, as well as a decrease in overall vitality. The occurrence
of this condition is not related to physical activity. In the evening, the condition improves. Somatic,
neurological status and ECG - without pathology. What is the most likely pathology that caused this
clinical picture?
100% Somatized depression
0% Resting angina pectoris
0% Neurosis-like schizophrenia
0% Neurocirculatory dystonia
0% Hypochondriac depression

2091. an 18-year-old patient went to a doctor due to severe weakness, fever,


progressive weight loss, and dizziness. Objectively: with a height of 165 cm, Weight-40 kg, the skin
is pale with a yellowish tinge, dry, flaky. In the blood: red blood cells -1,8-1012/l, HB-85 g/l,
leukocytes-500-109 / l, formula: myeloblasts-78%, neutr. - 15%, lymph. - 7%. What is the most
likely diagnosis?
100% Acute myeloblastic leukemia
0% Acute lymphoblastic leukemia
0% Chronic lymphoblastic leukemia
0% Chronic anemia
0% Leukemoid reaction

2092. A 42-year-old patient was hospitalized on the 12th day of her illness. The disease began acutely-
cold, followed by a feeling of heat, temperature-39.5 oC, intoxication. The fever attack
lasted 6 hours, after which the temperature dropped to 36.5°C with excessive sweating.
Fever attacks were repeated every 3rd day. Objectively: there is a herpetic rash on the lips,
slight jaundice of the sclera. Hepatolienal syndrome. Blood test: anemia, leukopenia,
lymphocytosis, erythrocyte sedimentation rate -19 mm / h. What is the most informative
diagnostic method for this disease?
100% A smear and a thick drop of blood
0% Backseeding of urine
0% Vidal's reaction
https://translate.yandex.com/en/doc 462/540
22:20 ,27.6.2023 �� C : ; 5 B

0% Liver tests
0% Blood culture for sterility

2093. the patient felt a slight malaise, minor headache, and weakness the day before.
Today, the temperature has increased to 38.5°C with cooling, significant headache, mainly
in the frontal region, and pronounced pain when moving the eyeballs. The skin of the face and conjunctiva
are hyperemic. There was a dry nagging cough. The pharynx is hyperemic,
there is a grainy enanthema on the soft palate, and there are spot hemorrhages in places. Over the lungs, breathing with a hard
tinge. What is the most likely diagnosis?
100% Flu
0% Typhus fever
0% Typhoid fever
0% Ku fever
0% Enterovirus infection

481

Downloaded from the site - online testing step

2094. a 36-year-old patient complains of shortness of breath, a feeling of compression in the right side
of the chest, fever up to 38.7 oC, cough with a small amount
of mucosal sputum. He's been ill for more than a week. Complaints are associated with hypothermia. Objectively:
slight acrocyanosis of the lips, pulse rhythmic, 90 / min., blood pressure-140/85 mm Hg. the right
half of the chest lags behind in the act of breathing. Percussion-to the right below the angle of the scapula,
dullness is heard with a limit to the top. There is no breathing in this area. What is the most
likely diagnosis?
100% Exudative pleurisy
0% Extrahospital pneumonia
0% Atelectasis of the lung
0% bodies
0% Lung abscess

2095. A 46-year-old patient was taken by ambulance to the emergency room complaining
of sharp, paroxysmal pain in the right lumbar region, which radiates to the groin area and
to the inner surface of the thigh. The pain came suddenly a few hours ago. The day before, the patient
had developed profuse, painless hematuria with wormlike blood clots. I've
never been ill before. What disease can be assumed in the first place?
100% Cancer of the right kidney
0% Bladder tumor
0% Necrotizing papillitis
0% Acute glomerulonephritis
0% Urolithiasis, right kidney stone

2096. A 30-year-old patient suffering from a headache


experienced a severe headache that resembled a blow to the head when lifting a heavy object. Nausea, vomiting, light
dizziness appeared. Objectively, after one day: severe meningeal syndrome, body temperature -37.6
°C. The doctor suggested a subarachnoid hemorrhage. What additional examination should
be performed first to confirm the diagnosis?
100% Lumbar puncture
0% X-ray of the skull
0% Computed tomography
0% Rheoencephalography
0% Cerebral angiography
https://translate.yandex.com/en/doc 463/540
22:20 ,27.6.2023 �� C : ; 5 B

2097. a 37-year-old patient complains of constant dull pain in the hypochondria radiating to the back,
which increases after eating. Worries about bloating, frequent bowel movements with impurities
of undigested food. She has been ill for more than 5 years, lost 15 kg of weight. Objectively: moderate bloating,
pain in the Shoffar area, Desjardins points, Mayo-Robson points. Which of the research methods will
be the most informative for confirming external
secretory pancreatic insufficiency?
100% Coprogram
0% Oral cholecystography
0% Retrograde pancreatography
0% EGDS
0% Ultrasound of the abdominal cavity

482

Downloaded from the site - online testing step

2098. A 59-year-old patient who has been suffering from hypertension for 15 years
has developed acute LV insufficiency against the background of a hypertensive crisis. Which drug
is most appropriate to prescribe to this patient?
100% Sodium Nitroprusside
0% Nifedipine
0% Propranolol
0% Spironolactone
0% Captopril

2099. a 62-year-old patient complains of a rash in the left shin area. Has been ill for more than 2 months.
He was treated independently, using various ointments and compresses, without improvement. Objectively:
there is a clear-edged focus of chronic inflammation on the anterior surface of the left shin.
The surface is uneven, papillary growths. When pressed, drops of pus are released.
The patient experiences minor pain. RW, PIT, RIF, Mantoux reaction-negative. What is the most
likely pathology in this patient?
100% Pyoderma
0% Tuberculosis lupus
0% Warty tuberculosis
0% Tertiary syphilis
0% Leishmaniasis

2100. 49-year-old jackhammer, 14 years of professional experience, 75% of the working time works
with a jackhammer that generates a vibration of 30-45 Hz. Complains of pain and numbness of the hands, disturbing
during rest, pain in the elbow and wrist joints, in the cervical
spine, poor sleep, periodic aching pains in the heart. What disease
can be predicted based on the patient's complaints?
100% Vibration sickness
0% Ulnar nerve neuritis
0% Deforming osteoarthritis of the elbow joints
0% Osteochondrosis of the cervical spine
0% Raynaud's syndrome

2101. the milkmaid has a 3x3 cm ulcer on her right hand, covered with a dense black scab. The brush
is sharply swollen, painless. The skin color in the area of edema is not changed. Enlarged axillary

https://translate.yandex.com/en/doc 464/540
22:20 ,27.6.2023 �� C : ; 5 B
lymph node on the right. Palpation of the lymph node is painless. Symptoms of intoxication are
100% Cutaneous form of anthrax
moderate. What is the most likely diagnosis?
0% Bubonic skin form of plague
0% Ulcerative-bubonic form of tularemia
0% Banal carbuncle
0% Erysipelas

2102. A 63-year-old female patient underwent surgery for a large-sized ba-gatonodular euthyroid goiter
. With technical difficulties, subtotal resection of both
thyroid lobes was forced. On the 4th day after the operation, convulsions of the muscles of the face and upper arms appeared

483

Downloaded from the site - online testing step

limbs, abdominal pain. Positive symptoms of Khvostek and Trousseau. What is the most likely reason
for this condition of the patient?
100% Parathyroid gland failure
0% Postoperative hypothyroidism
0% Thyrotoxic crisis
0% Reverse nerve damage
0% Tracheomalacia

2103. A 23-year-old woman complains of an increase in body temperature up to 37.4 ° C, the appearance
of a hemorrhagic rash on the lower extremities, lower back pain,and red urine. I got sick 3 days
ago after hypothermia. Objectively: the skin is pale, and there is a small
symmetrical hemorrhagic rash on the surface of the legs and thighs. Heart rate-90 / min, blood pressure
-115/90 mm Hg. Pasternatsky's symptom is weakly positive on both sides. In the blood: white blood cells
- 9,6*109/l, platelets - 180*109/l, erythrocyte sedimentation rate-31 mm / hour. In the urine: protein -0.33
g / l, ep. Changed-3-40 in p/s, white blood cells - 5-8 in p/s. Which of the above drugs is
pathogenetically justified for treatment in this case?
100% Heparin
0% Calcium Gluconate
0% Ascorbic acid
0% Vikasol
0% Rutin

2104. A 36-year-old woman complains of pain, restricted movement in the small joints of her hands,
difficulty swallowing solid food, weakness, and a dry cough. Objectively: the skin of the hands and
forearm is dense and smooth. The proximal joints of the fingers II-IV of the hands are swollen
and painful on palpation. Above the lungs there are dry scattered wheezes, the borders of the heart are shifted to the left by 2 cm, the
tones
are muted. In the blood: erythrocyte sedimentation rate - 36 mm / h, 7-globulins - 24%. In the urine:
no changes.
100% What is the most
Systemic likely diagnosis?
scleroderma
0% Systemic lupus erythematosus
0% Dermatomyositis
0% Rheumatoid arthritis
0% Sarcoidosis

2105. A 49-year-old woman complains of pain in the shoulder joints that increases during movement,
limited mobility, and short-term morning stiffness. He's been ill for several years. In the past
, she was engaged in gymnastics. The radiograph of the shoulder joints shows narrowing of the joint

https://translate.yandex.com/en/doc 465/540
22:20 ,27.6.2023 �� C : ; 5 B
gap, subchondral osteosclerosis, osteophytes in the
lower-inner
100% partViolation
of the humerus head.metabolism
of cartilage What is the basis of the pathogenesis of joint damage in a patient?

0% Deposits in the synovial membrane of immune complexes


0% Violation of uric acid metabolism
0% Calcium pyrophosphate deposits in the joints
0% Synovial membrane damage caused by an infectious agent

484

Downloaded from the site - online testing step

2106. A 40-year-old woman was admitted to the clinic with complaints of suffocation, which lasts for several
hours and does not go away from the action of a previously effective asthmopent. There was palpitation and anxiety.
He has been suffering from chronic bronchitis for 8 years. Objectively: the condition is severe, the patient is sitting with her hands
on the edge of the table, pale cyanosis, wheezing can be heard in the distance. In the lungs, against the background
of weakened breathing, scattered dry wheezes are heard in a small amount. Pulse -108 / min.,
blood pressure -140 / 80 mm Hg. what medications are
required in the first place in the patient's treatment?
100% Glucocorticoids
0% Adrenomimetics
0% Antihistamines
0% Holinolytics
0% Mast cell Membrane Stabilizers

2107. a 58-year-old woman complains of gratuitous bruising, weakness, bleeding


gums, and dizziness. Objectively: the mucous membranes and skin are pale, with numerous
hemorrhages of various antiquities. The lymph nodes are not enlarged. Pulse -100 / min., blood
pressure -110 / 70 mm Hg. no changes were found on the part of the internal organs. In the blood: red blood cells -3.01
*1012/l, HB-92 g/l, CP-0.9, anisocytosis, poikilocytosis, leukocytes-10-109/l, E.-2%, P.-12%,
S.-68%, L.-11%, M.-7%, erythrocyte sedimentation rate-12 mm / hour. Additional determination
of which laboratory parameter is most appropriate for establishing a diagnosis?
100% Platelets
0% Reticulocytes
0% Blood clotting time
0% Osmotic resistance of red blood cells
0% Fibrinogen

2108. a 34-year-old woman complains of weakness, weight loss of 12 kg in six months, sweating,
palpitations, irritability. Objectively: the thyroid gland of the third degree, elastic, against the background
of diffuse enlargement in the right lobe node. The cervical lymph nodes are not enlarged. What
is the most appropriate treatment strategy?
100% Surgery after antithyroid therapy
0% Administration of radioactive iodine
0% Immediate surgical intervention
0% Conservative antithyroid therapy
0% Immediate telegammotherapy

2109. A 25-year-old patient was diagnosed with a pelvic bone fracture. The condition is severe, pulse -116
/ min., blood pressure-100/60 mm Hg. the abdomen is tense, painful, symptoms
of irritation of the peritoneum are poorly expressed. In the blood: red blood cells - 3,2*1012/l; hB-110 g / l; Ht-0.37;

https://translate.yandex.com/en/doc 466/540
22:20 ,27.6.2023 �� C : ; 5 B
white blood cells - 10*109/l; stick-shaped -10%. Which additional survey method is most
100%
informative? Laparoscopy
0% Overview X-ray of the abdomen
0% Ultrasound examination of the abdomen
0% Laparocentesis
0% Fibrogastroduodenoscopy

485

Downloaded from the site - online testing step

2110. a 7-year-old boy has a history of a peak temperature of up to 40


° C, spindle-shaped swelling of the joints of the fingers, knee and
ankle joints, pain in the upper part of the sternum, and the cervical spine is observed for 3 months. What
is the most likely diagnosis?
100% Juvenile rheumatoid arthritis
0% Iostra rheumatic fever
0% Toxic Synovitis
0% Septic arthritis
0% Osteoarthritis

2111.as a result of a traffic accident, the victim received multiple fractures


of the limbs and pelvic bones. In the anamnesis: hemophilia A. During the examination, hematomas are formed on
the damaged areas. The condition is getting worse. Blood pressure-90/50 mm Hg indicate
the most appropriate combination of infusion agents for the treatment of the patient after the use
of polyglucin and saline solutions:
100% Cryoprecipitate, red blood cell mass
0% Fresh frozen plasma, albumin
0% Red blood cell mass, albumin
0% Cryoprecipitate, glucose
0% Red blood cell mass

2112. A 70-year-old patient complains of weakness, dizziness, short periods


of fainting, and pain in the heart. During auscultation of the heart: heart rate -40
/ min., rhythmic tones, I tone periodically significantly increased, blood pressure -160/90 mm Hg.
what is the most likely cause of hemodynamic disorders?
100% AV block of the third degree
0% AV block of the first degree
0% Bradysystolic form of atrial fibrillation
0% Sinus bradycardia
0% Complete blockage of the left bundle branch of Gis

2113. a 6-month-old child suddenly developed intense cramping abdominal pain 8 hours ago
. The child's periods of anxiety last 5 minutes, with an interval of 15 minutes.
There was no bowel movement. Rectal bleeding started an hour ago. During palpation: the abdomen is soft,
painful in the right half, where an elongated formation is indistinctly defined-10x4 cm.
Rectal examination shows traces of blood. What is the most likely diagnosis?
100% Ileocecal invagination
0% Intestinal bleeding
0% Acute appendicitis
0% Inversion of the bowel
0% Acute enterocolitis

https://translate.yandex.com/en/doc 467/540
22:20 ,27.6.2023 �� C : ; 5 B

2114. a 35-year-old patient suffering from chronic glomerulonephritis and


who has been on hemodialysis for the last 3 years has experienced heart failure, hypotension, increasing weakness,
and shortness of breath. ECG: bradycardia, first-century atrioventricular block, high pointed T-waves

486

Downloaded from the site - online testing step

The day before-a gross violation of drinking and dietary regimes. What is the most likely
reason for these changes?
100% Hyperkalemia
0% Hyperhydration
0% Hypokalemia
0% Hypernatremia
0% Hypocalcemia

2115. the patient complains of a sudden onset of the disease: fever up to 39.0 oC, of the wrong
type, with acute chest pain. Sputum with a putrid smell and blood impurities, up to 400 ml
per day. During percussion: over the affected area, shortening of percussion sound, increased
vocal tremor. Anaerobic streptococcus was isolated in sputum. What disease can
be assumed in the first place?
100% Gangrene of the lung
0% Lung abscess
0% Tuberculosis
0% Bronchiectasis
0% Abscessed pneumonia

2116. A 46-year-old patient complains of itchy skin, sweating, especially at night,


and fever up to 38.6 ° C. Objectively: on the skin of the breast there are traces of uncovering, supraclavicular
lymph nodes are enlarged to 3 cm, they are not soldered to the skin. What is the most
appropriate research method?
100% Enlarged lymph node biopsy
0% General blood test
0% Overview X-ray of the chest cavity
0% Immunogram
0% Blood proteins and protein fractions

2117. a 50-year-old patient suddenly developed dagger pain in the occipital region and
vomiting. Objectively: sopor, hyperemia of the facial skin, blood pressure-210/120 mm Hg,
pulse-60 / min., tense, body temperature-37.8°C. Horizontal nystagmus.
Reflexes of oral automatism are expressed. Tendon reflexes are uniform. Stiffness of the occipital
muscles, bilateral Kernig's symptom. What is the preliminary diagnosis?
100% Subarachnoid hemorrhage
0% Hemorrhagic parenchymal stroke
0% Subdural hematoma
0% Acute hypertensive encephalopathy
0% Meningococcal meningitis

2118. A 31-year-old patient went to the doctor complaining of enlarged lymph nodes on the left side of
the collarbone. Chest and abdominal pain, itchy skin at night. He has been ill for a year. Objectively:
enlarged non-painful lymph nodes are palpated on the left in the supraclavicular region. The liver and spleen

https://translate.yandex.com/en/doc 468/540
22:20 ,27.6.2023 �� C : ; 5 B
are not enlarged. In the blood: Hb-70 g / l, white blood cells - 19,6*109/l, EOS.- 1%, pal. - 8%, p. -83%, lymph. - 2%,
mon. - 6%, erythrocyte sedimentation rate-55 mm / h, platelets - 58*109/L. On the Ro-graph

487

Downloaded from the site - online testing step

in the upper right lung, an infiltrate is detected in the thoracic cavity, which contrasts with
the lung tissue. What tests are most appropriate to carry out to confirm
the diagnosis?
100% Lymph node biopsy
0% Trepanobiopsy
0% Sternal puncture
0% Ben-Jones protein urine test
0% X-ray examination of the stomach

2119. A 46-year-old patient first noticed swelling in the legs, weakness, a feeling of " fullness” and
heaviness in the right hypochondrium. He has been suffering from rheumatoid arthritis for 20 years. Enlarged liver and
spleen, dense consistency. Blood creatinine - 0.23 mmol / l, proteinemia-68 g / l,
cholesterol-4.2 mmol/l, specific gravity of urine-1012, proteinuria-3.3 g/l, single waxy
cylinders, red blood cells are leached in p / s, white blood cells-5-6 in p / s. what is the most likely complication?
100% Amyloidosis of the kidneys
0% Chronic glomerulonephritis
0% Acute glomerulonephritis
0% Heart failure
0% Chronic pyelonephritis

2120. a 12-year-old girl has had chronic viral hepatitis C for 7 years. The condition
has worsened in the last 6 months. Lost weight, appeared jaundice of the skin and sclera.
Pronounced itching of the skin, vascular "asterisks" nosebleeds. Ascites. Liver + 4 cm,
spleen at the level of the umbilical line. What syndrome is associated with the development of severe itching
of the skin in the patient?
100% Cholestasis
0% Hypersplenism
0% Edematous-ascitic syndrome
0% Portal hypertension
0% Hepatocellular insufficiency

2121. The 26-year-old athlete fell with an emphasis on the right shoulder joint. - Sharp pain,
restriction of passive movements, active movements in the shoulder joint are impossible. The injured arm that is withdrawn and bent at
the shoulder joint is supported by a healthy one. Objectively: the deltoid
muscle is occluded, the acromial process of the scapula protrudes sharply under the skin, and the head
of the humerus is palpated under the coracoid process. What is the most likely
clinical diagnosis?
100% Dislocated shoulder
0% Fracture of the anatomical neck of the shoulder
0% Surgical shoulder neck fracture
0% Fracture of the head of the humerus
0% Large humerus hill Detachmentedit

2122. A 33-year-old patient


developed coldness, drowsiness, apathy, decreased appetite, constipation, and voice changes 3 months after thyroidectomy due to diffuse

https://translate.yandex.com/en/doc 469/540
22:20 ,27.6.2023 �� C : ; 5 B
toxic goiter. 488

Downloaded from the site - online testing step

Objectively: the skin is dry, cold, pale, the face is puffy, dense swelling of the lips, limbs.
The thyroid gland is not palpable. Pulse-52 / min., blood pressure -100 / 60 mm Hg.
heart sounds are weakened. Daily proteinuria -100 mg the patient's condition is due to:
100% Development of hypothyroidism
0% Relapse of toxic goiter
0% Development of nodular toxic goiter
0% Heart failure
0% Nephrotic syndrome

2123. a 37-year-old man gradually


developed weakness of the legs, then the muscles of the trunk, arms, neck, face, difficulty swallowing,and speech over a week after an
acute respiratory infection
. Neurological status: dysphagia, dysarthria, dysphonia, respiratory disorders,
cardiac
100%activity,To
tetraparesis. It iscare
the intensive most likely that the patient should be referred:
unit
0% To the therapeutic department
0% To the Infectious diseases department
0% To the neurological department
0% For outpatient treatment

2124. a 7-year-old girl complains of weakness, increased fatigue,


fever up to 38°C, a small amount of urine released per day, the color of " meat slop”'
objectively: pallor of the skin, swelling of the face, arms, legs, lower back pain. In the blood: red blood cells -2.7
*1012 / l, Hb-90 g / l, white blood cells - 17*109/l, E.-10%, P.-4%, C.-60%, L.-16%, M.-10%, erythrocyte
sedimentation rate-30 mm / h. In the urine: leukocyte -15 v / s, erythr. -30 v / s, hyaline cylinders
-8-10 v / s, protein-4 g / l. Cholesterol in the blood-8 mmol/l, total protein-43 g/l. what
is the leading mechanism of edema development?
100% Reduction of oncotic blood pressure
0% Cardiac disorders
0% Diselectrolyte violations
0% Hyperaldosteronism
0% Reduction of osmotic blood pressure

2125. A 35-year-old female patient complains of pain in her right lower leg, which increases during walking.
Objectively: on the right shin along the course of the varicose small saphenous vein, the skin
is red, hot to the touch, during palpation in the upper third of the shin, an oblong
painful seal measuring 3x2 cm is determined. What complication of varicose
veins did the patient have?
100% Thrombophlebitis
0% Deep vein thrombosis
0% A boil
0% Lymphangitis
0% Lymphadenitis

2126. the mother brought a 5-year-old boy to the doctor's office with a complaint of a rash on his face
that appeared 5 days ago. Objectively: there are a few blisters on the face.

489

https://translate.yandex.com/en/doc 470/540
22:20 ,27.6.2023 �� C : ; 5 B

Downloaded from the site - online testing step

0.5-1.5 cm in diameter, with thin flabby lids, filled with serous cloudy exudate
and located on a hyperemic, edematous base. Along with these lesions
, erosions with the release of serous-purulent fluid and thick honey-yellow crusts are observed.
Make a clinical diagnosis:
100% Vulgar impetigo
0% Children's eczema
0% Simple pemphigus lichen
0% Contact dermatitis
0% Candidiasis

2127. A 5-month-old child had three recurrent bronchial obstructive syndrome without previous
catarrhal events, and cough with
sputum difficult to clear was almost constantly observed. Bowel movements are greasy and smelly from the first days of life.
Cystic fibrosis is suspected. What research can confirm the diagnosis?
100% Sweat Chlorides
0% Chest radiography
0% Coprocytogram
0% Scintigraphy of the lungs
0% Determination of blood lipids

2128. a worker has been working in a foundry with non-ferrous metals and alloys for 12 years. An
increased content of heavy metals, carbon and nitrogen oxides was recorded in the air of the working area.
During a regular medical examination, the patient was diagnosed with asthenic-vegetative syndrome,
sharp abdominal pain, constipation,and pain in the liver. In the urine: aminolevulinic acid and
coproporphyrin. In the blood: reticulocytosis, decreased hemoglobin levels. The reason for such
intoxication is:
100% Lead and its salts
0% Tin
0% Carbon monoxide
0% Nitric oxide
0% Zinc

2129. a patient who consumed canned mushrooms three days ago has
a visual disorder (diplopia, mydriasis), speech disorder, and swallowing disorder. What kind of food poisoning
occurs?
100% Botulism
0% Food toxicoinfection
0% Fusariotoxicosis
0% Honeysuckle poisoning
0% Lead salt poisoning

2130. a 24-year-old woman in labor suddenly had a body temperature of 38.7 oC on the 5th day.
Complains of weakness, headache, lower abdominal pain, irritability. Objectively:
blood pressure-120/70 mm Hg, pulse-92 / min., body temperature -38.7 oC. Bimanually:
the uterus is enlarged up to 12 weeks of pregnancy, dense, somewhat painful on palpation, cervical

490

https://translate.yandex.com/en/doc 471/540
22:20 ,27.6.2023 �� C : ; 5 B
Downloaded from the site - online testing step

the channel passes 2 p/p, the discharge is moderate, cloudy, with an unpleasant smell. In the blood:
left-shifted leukocytosis, lymphopenia, erythrocyte sedimentation rate-30 mm / h. What
is the most likely diagnosis?
100% Endometritis
0% Parametrite
0% Pelvioperitonitis
0% Metroflebitis
0% Lochiometer

2131. A 3-year-old child has been admitted from home, who


continues to have tonic-clonic generalized convulsions for 50 minutes against the background of hyperthermic syndrome caused by
influenza. No therapy was
performed.
100% For Enter
immediate treatment of convulsive syndrome, it is necessary to:
diazepam
0% Introduce calcium
0% Physical cooling methods
0% Phenobarbital
0% Antipyretics

2132. to determine the influence of the microclimate on the functional state of the body, the
following physiological indicators of stress on the functions of organs and systems involved in
heat exchange processes were studied: pulse, blood pressure, number of respiratory acts per minute,
skin temperature, sweating per minute, speed of visual and auditory reactions. Name
the indicator that most objectively reflects the intensity of the body's thermoregulation processes:
100% Skin temperature
0% Pulse and blood pressure
0% Number of breathing acts per minute
0% Sweating rate per minute
0% Speed of visual and auditory responses

2133. A 58-year-old female patient started having heavy uterine bleeding after 10 years of menopause. In the course
of bimanual examination and examination with the help of mirrors, in addition to abundant bloody
discharge, no other pathology was found. What disease can be assumed?
100% Cancer of the uterine body
0% Schroeder's hemorrhagic metropathy
0% Incomplete abortion
0% Uterine fibroids
0% Menstrual disorders of a climacteric nature

2134. a 12-year-old girl has a "butterfly" symptom on her face, pink-red


spots with whitish-gray scales on her hands, hair loss and joint pain,
persistent arterial hypertension, proteinuria, erythrocyturia, leukocyturia, and azotemia are observed. What
connective tissue disease can be assumed in the first place?
100% Systemic lupus erythematosus
0% Juvenile rheumatoid arthritis
0% Dermatomyositis

491

Downloaded from the site - online testing step

https://translate.yandex.com/en/doc 472/540
22:20 ,27.6.2023 �� C : ; 5 B
0% Scleroderma
0% Rheumatism

2135. A 22-year-old female patient went to a antenatal clinic complaining of delayed


menstruation for 1.5 months, nausea, fatigue, drowsiness, and irritability. During the
examination, pronounced pigmentation was found on her face and nipples.
No pathology was detected on the part of the internal organs. During the examination in mirrors, cyanosis of the vaginal
and cervical mucosa is determined; during the bimanual examination, an increase in the uterus,its hyperanteflexia,
and asymmetry are detected. What are the most likely causes of these complaints and the data from the bimanual
study?
100% Uterine pregnancy
0% Menstrual disorders
0% Diseases of the gastrointestinal tract
0% Ectopic pregnancy
0% Uterine tumor

2136. in a 10-year-old patient with chronic renal failure


, anemic syndrome continues to progress. Specify the drug of choice for pathogenetic treatment of the specified
syndrome:
100% Erythropoietin
0% Ferrum-lek
0% Cyanocobalamin
0% Red blood cell mass
0% Folic Acid

2137. in a 2-month-old patient, clearly defined


acute-burning foci in the form of spots are observed in the inguinal-femoral folds, slightly raised above the nearby areas due
to skin edema. The rash appeared within a week. Vesiculation and diaper rash were not
observed. What will be the preliminary diagnosis?
100% Diaper dermatitis
0% Children's eczema
0% Mycosis of smooth skin
0% Psoriasis
0% Complicated course of scabies

2138. the employee was hospitalized for 16 days for pneumonia. What
is the procedure for issuing a disability certificate in this case?
100% By a curator doctor together with the head of the department for the entire period of treatment
0% By a curator doctor from the day of admission to the hospital for a maximum of 5 days
0% By a doctor-curator from the day of admission to the hospital up to a maximum of 10 days
0% By a curator doctor from the day of admission to the hospital up to a maximum of 14 days
0% According to the conclusion of the medical advisory commission for the entire period of treatment

2139. a patient complains of pain in the lower abdomen, which increases during menstruation and sexual intercourse

492

Downloaded from the site - online testing step

and radiates into the vagina. During a vaginal examination


, dense, nodular, painful formations are found behind the uterus. What is the most likely diagnosis?
100% Retrocervical endometriosis
https://translate.yandex.com/en/doc 473/540
22:20 ,27.6.2023 �� C : ; 5 B

0% Adenomyosis
0% Perimeter
0% Chronic inflammation of the uterine appendages
0% Parametrite

2140. the child is ill on the 5th day. Worried about increased to 39-40oC body temperature, annoying,
frequent, deep, unproductive cough, photophobia, nasal congestion. Objectively:
puffy face, loose mouth mucosa, hemorrhagic enanthema on the soft palate,
remnants of Filatov-Koplik spots. What is the most likely diagnosis?
100% Measles
0% Scarlet fever
0% Rubella
0% Adenovirus infection
0% Flu

2141. in the course of studying the working conditions of workers engaged in the production of mercury thermometers,
mercury vapors were detected in the air of the working area in concentrations that exceed the maximum
permissible values. Please indicate the main possible route of mercury ingestion in the employee's body:
100% Respiratory organs
0% Intact skin
0% Damaged skin
0% Gastrointestinal tract
0% Mucous membranes

2142. assessing the state of health of drivers and road patrol inspectors, doctors found the presence of carboxyhemoglobin in
the blood of the subjects, a decrease in their reflex reactions, and a violation
of the activity of a number of enzymes. The identified health problems of people in these occupational categories
are most likely related to exposure to:
100% Carbon monoxide
0% Sulphurous anhydride
0% Nervous and emotional stress
0% Aromatic carbohydrates
0% Nitrogen oxides

2143. A 25-year-old female patient complains of fever up to 38 ° C. lower abdominal pain


and vaginal discharge. Three days ago, an artificial abortion was performed at 10 weeks of pregnancy. PV
the cervix is clear, the uterus is slightly enlarged, painful. Uterine appendages are not
detected. The arches are deep and painless. Vaginal discharge is purulent-bloody. What is the most
likely diagnosis?
100% Post-abortion metroendometritis
0% Hematometer
0% Pelvioperitonitis

493

Downloaded from the site - online testing step

0% Uterine perforation after abortion


0% Parametrite

2144. a 63-year-old patient complains of pain in the right groin area, bloating,

https://translate.yandex.com/en/doc 474/540
22:20 ,27.6.2023 �� C : ; 5 B
weakness, persistent constipation for more than 7 months. Objectively: the skin is pale and dry. In the right iliac
region, a 5-7 cm formation is palpated, sedentary, non-painful. Auscultation: increased
intestinal noise. In the blood: red blood cells - 2,9*1012/l, HB-80 g / l, erythrocyte sedimentation rate -32
mm / h. Blood in the stool. What is the most likely diagnosis?
100% Colorectal cancer
0% Crohn's disease
0% Polyp of the cecum
0% Cancer of the right kidney
0% Spastic colitis

2145. a 67-year-old patient complains of shortness of breath, chest pain, and general weakness. He's been ill for 5
months. Objectively: body temperature-37.3 oC, pulse - 96/min. Above the right lung, the vocal
tremor is not detected, the percussion sound is dull, and breathing is not listened to. In sputum
-admixtures of blood diffusely mixed with mucus. What is the most likely diagnosis?
100% Lung cancer
0% Bolshegnishchevaya pneumonia
0% Bronchiectasis
0% Focal tuberculosis of the lungs
0% Exudative pleurisy

2146. A 29-year-old patient complains for 2 months of pain in the left side
of the chest, cough, shortness of breath; body temperature-39.6 oC. Objectively: the left side of the chest
lags behind in the act of breathing, weakening of vesicular respiration and shortening of percussion sound
on the left. Ro-logically defined rounded shadow in the lower lobe of the left lung. What is the preliminary
diagnosis?
100% Lung abscess
0% Purulent pleurisy
0% Lung cancer
0% Pleural empyema
0% Chronic pneumonia

2147. A 45-year-old patient complains of discomfort while reading, redness


of the eyelid edges, and white foamy discharge in the corners of the eye slits during the last two months. Objectively
, hyperemia and thickening of the free edge of the eyelids, expansion of the excretory ducts
of the glands of the cartilage of the eyelids are observed. What will be the diagnosis?
100% Meibomian blepharitis
0% Adenoviral conjunctivitis
0% Blenorrheic conjunctivitis
0% Chronic canaliculitis
0% Acute dacryoadenitis
2148. an 8-month-old girl was born prematurely. During the examination: suffocation is observed,

494

Downloaded from the site - online testing step

tachycardia, hepatosplenomegaly, delayed physical development, cyanosis of the extremities.


Parasternal heart hump is detected, systolic
diastolic murmur is heard in the II intercostal space on the left, blood pressure is 90/0 mm Hg. what disease can
be assumed?
100% Open Ductus arteriosus
0% Aortic coarctation
0% Aortic valve stenosis

https://translate.yandex.com/en/doc 475/540
22:20 ,27.6.2023 �� C : ; 5 B
0% Pulmonary artery stenosis
0% Non-closure of the interventricular septum

2149. on the 20th day after the injury, the


patient has a sharp increase in body temperature, general weakness, frequent
vesicular respiration, exacerbation of facial features, blood pressure - 90/50 mm Hg, pulse -112
/ min. What complication can be assumed?
100% Sepsis
0% Pneumonia
0% Acute intoxication
0% Purulent bronchitis
0% Anaerobic infection

2150. This year


, 11 patients with coronary heart disease were referred to the hospital by general practitioners of the city polyclinic. In 3 cases, the
diagnosis was not confirmed. What
is the100%
best management
Analysisdecision to make
of each case in this case?
of divergence of diagnosis
0% Analysis of the quality of dispensary follow-up
0% Analysis of the quality of diagnostic tests
0% Analysis of the level of qualification of polyclinic doctors
0% Analysis of the polyclinic's material and technical base

2151. during a laboratory study of a pork sample, 1 dead trichinella was found in 24 sections
. Such meat should be transferred for:
100% Technical recycling
0% Implementations without restrictions
0% Sales through a network of public catering establishments
0% Processing for the production of boiled sausages
Freezing to a temperature of 10oC in the thickness of meat, followed by exposure
0%
for 15 days

2152. during a medical examination, a 43-year-old man was objectively found to have pale
skin and mucous membranes, smoothness of the papillae of the tongue, transverse furrows on the nails,
cracks in the corners of the mouth, and tachycardia. Hemoglobin content in the blood - 90 g / l, anisocytosis, poikilocytosis.
The most likely causative factor of this condition is insufficient intake into
the body:
100% The shackles
0% Midi

495

Downloaded from the site - online testing step

0% Of zinc
0% Magnesium content
0% Selena

2153. a 52-year-old patient has a lesion of the mucous membrane in the corners of the mouth with
the formation of cracks, erosions and ulcers, vertical cracks on the lips when they close (cheilosis),
changes in the tongue (glossitis), angular stomatitis, seborrheic peeling of the skin around the mouth and on
the wings of the nose, pericorneal injection. The above symptoms are typical for:
100% B2-hypovitaminosis
https://translate.yandex.com/en/doc 476/540
22:20 ,27.6.2023 �� C : ; 5 B

0% B1-hypovitaminosis
0% C-hypovitaminosis
0% PP-hypovitaminosis
0% A-hypovitaminosis

2154. a 27-year-old patient suffered a fracture of the right femur


in the lower third due to a fall from a height, with the distal fragment protruding under the skin. The limb is warm, the patient
moves the toes of the foot. The ambulance doctor refused the Dieterichs splint and applied
immobilization with three ladder splints on the bent limb at the knee. What possible
complication did he prevent?
100% Damage to the popliteal neurovascular bundle
0% Development of traumatic shock
0% Development of fat embolism
0% Secondary displacement of bone fragments
0% Muscle interposition and skin perforation

2155. A 28-year-old patient was admitted to the intensive care unit with complaints of
abdominal pain, vomiting, and double vision. Objectively: disoriented, pupils dilated, reaction to
light weakened, skin dry, hyperemic, minor cyanosis, blood pressure-90/60 mm
Hg, pulse -100 / min. From the medical history, it is known that he consumed alcohol
of unknown origin two days before hospitalization. Specific therapy consists of intravenous administration:
100% Of ethanol
0% Sodium Bicarbonate
0% 40% glucose solution
0% Thiamine
0% Omeprazole

2156. a 29-year-old patient complains of the absence of menstruation during the year, rapid
fatigue, hypotension with syncopal states of memory loss, and dry skin. From
the anamnesis, it is known that 1.5 years ago in the second stage of labor there was significant bleeding,
uterine extirpation, blood transfusion was performed, and she was on a ventilator for 3 days. What is the most likely
diagnosis?
100% Skien's syndrome
0% Chiari-Frommel syndrome
0% Farbs-Albright syndrome
0% Aronze del Castillo Syndrome

496

Downloaded from the site - online testing step

0% Adrenogenital syndrome

2157. a 50-year-old patient has a stable increase


in jaundice and anemia with periodic rises in body temperature for one and a half months. During palpation
, an enlarged and painless gallbladder was found. The development of what disease can be assumed?
100% Cancer of the head of the pancreas
0% Gallbladder cancer
0% Cholelithiasis
0% Echinococcosis of the liver
0% Primary sclerosing cholangitis

https://translate.yandex.com/en/doc 477/540
22:20 ,27.6.2023 �� C : ; 5 B

2158. a 54-year-old woman applied to the gynecologist with complaints of blood discharge from the vagina
for 1 month. My last period was 5 years ago. During the gynecological examination
, no pathology was detected. What will be the doctor's actions?
100% Fractional diagnostic curettage of the uterine cavity walls
0% Colposcopy
0% Ultrasound examination
0% Take a smear for cytological examination
0% Prescribe symptomatic therapy

2159. the ambulance doctor was called to the victim. A circular saw was used to
detach the left hand at the level of the wrist joint in a 25-year-old worker. There is
arterial bleeding. Where should I apply a tourniquet to temporarily stop arterial
bleeding?
100% Middle third of the shoulder
0% Lower third of the forearm
0% Upper third of the forearm
0% Elbow joint area
0% -

2160. a 35-year-old patient who received 400 ml of red blood cell mass developed
general restlessness, short-term agitation, pain behind the sternum, low back, shortness of breath, cyanosis,
tachycardia - 110 / min., blood pressure decreased to 90/40 mm Hg. what is the most likely
diagnosis?
100% Blood transfusion shock
0% Massive blood transfusion syndrome
0% Pulmonary embolism
0% Citrate intoxication
0% Allergic reaction

2161. A 38-year-old patient complains of purulent discharge from the left side of the nose, difficulty
in nasal breathing, headache, heaviness in the left cheek area, and an increase in body temperature up
to 37.5 ° C. Gets sick for 6 days. The disease is associated with hypothermia. Objectively:
soreness on palpation in the area of the left cheek. Mucosa of the left half of the nasal cavity

497

Downloaded from the site - online testing step

the cavity is hyperemic, swollen, and there is a purulent streak in the middle nasal passage. What is the most
likely diagnosis?
100% Left-sided acute sinusitis
0% Left-sided acute frontitis
0% Left-sided acute etmoiditis
0% Acute rhinitis (runny nose)
0% Curvature of the nasal septum to the left

2162. An 11-year-old boy's posture was determined during a routine medical examination
. It was found that the child's shoulders are tilted and brought forward,the head is tilted forward,
the chest is flattened, the abdomen is convex. In the spine, there is an increase in the depth
of the cervical and lumbar bends. What kind of posture is found in the child?
100% Kyphosis

https://translate.yandex.com/en/doc 478/540
22:20 ,27.6.2023 �� C : ; 5 B
0% Lordoz
0% Stooped
0% Fixed
0% Normal

2163. An 18-year-old patient complains of fever


, pain and swelling in the knee joints, and a rash in the form of red rings on the lower legs 2 weeks after having a sore throat.
After a few days, I started to worry about pain in my ankles, then in my elbows. What
disease is characterized by such symptoms?
100% Acute rheumatic fever
0% Rheumatoid arthritis
0% Reactive arthritis
0% Toxic-allergic dermatitis
0% Deforming osteoarthritis

2164. A 21-year-old female patient went to a antenatal clinic complaining of significant malaise,
severe pain in the vulva, irritation, itching, and frequent urination. From
the anamnesis, it was found out that the woman had extramarital sexual contact 10 days ago.
Examination of the external genitalia revealed
that numerous vesicles 2-3 mm in diameter are located on the hyperemic, edematous mucous membrane of the labia majora. After
carrying
out additional diagnostic methods, the herpes simplex virus was detected. What drug
is used to treat this
100% condition?
Acyclovir
0% Tetracycline
0% Sulfadimizine
0% Metronidazole
0% Progesterone

2165. A 47-year-old patient weighing 100 kg complained of aching pain in the knee and
hip joints, which occurs during movement and at rest, increases in the evening and during
physical exertion. Objectively: the knee joints are deformed, during palpation - pain in
the inner surface at the site of the projection of the joint gap, movements are accompanied by a crunch.

498

Downloaded from the site - online testing step

He has been suffering from duodenal ulcer for 16 years. What is


the most appropriate means of symptomatic therapy in this situation?
100% Celecoxib
0% Ibuprofen
0% Piroxicam
0% Diclofenac Sodium
0% Indomethacin

2166.A 24-year-old woman complained about a 10-week delay in her menstrual period when she went to a women's clinic
. A history of 2 spontaneous miscarriages. During ultrasound examination
, one fetal egg was found, which corresponds to 6-7 weeks of pregnancy, the fetal heartbeat is not detected. What
are the next tactics in this situation?
100% Scraping the walls of the uterine cavity
0% Application of uterotonic drugs
0% Prescribing hormone therapy

https://translate.yandex.com/en/doc 479/540
22:20 ,27.6.2023 �� C : ; 5 B
0% Follow-up for 1 week, followed by ultrasound monitoring
0% Prescribing antibacterial therapy

2167. a 47-year-old patient began to worry about constrictive pain behind the sternum, which occurs when
walking at 700-800 m. Drinks 2 liters of beer once a week. Arterial hypertension for
the last 7 years. Objectively: pulse-74 / min., blood pressure-120/80 mm Hg.
during VEM with a load of 75 W, ST-segment depression was recorded 2 mm below
the contour line in V4 — V6. What is the most likely diagnosis?
100% Angina pectoris of tension, functional class II
0% Angina pectoris of tension, functional class III
0% Angina pectoris of tension, functional class IV
0% Hypertonic vegetative-vascular dystonia
0% Alcoholic cardiomyopathy

2168. A 43-year-old patient was admitted to the therapeutic department 2 months after suffering from tonsillitis
with complaints of shortness of breath, pain in the heart, dizziness,
and palpitation. Objectively: the general condition is serious. Respiratory rate-35 / min., pulse-100 / min.,
arrhythmic, (extrasystole). Blood pressure-145/60 mm Hg. heart sounds are arrhythmic
(extrasystole) weakening and tones, systolic murmur above the apex. The liver is enlarged. ECG:
sinus tachycardia, single ventricular extrasystoles. Echocardiography of the heart cavity is within
the normal range, EF-50%. What is the most likely diagnosis?
100% Infectious myocarditis
0% Dressler's syndrome
0% Infectious pericarditis
0% Myocarditis cardiofibrosis
0% Dilated cardiomyopathy

2169. a 33-year-old patient has acute blood loss (erythrocytes - 2,2*1012/l, Hb-55 g/l), blood
type A (II) Rh (+). He was mistakenly transfused with donor erythromas AB (IV) Rh ( + ). After
an hour, he began to feel anxiety, lower back pain, and abdominal pain. Pulse-134 / min., blood pressure -

499

Downloaded from the site - online testing step

100/65 mm Hg, body temperature-38.6 oC. During catheterization of the bladder, 12


ml/h of dark brown urine was obtained. What complication did the patient have?
100% Acute renal failure
0% Cardiac shock
0% Allergic reaction to donor red blood mass
0% Citrate intoxication
0% Infectious and toxic shock

2170. a 45-year-old man developed increased activity over the past two weeks,
became talkative, euphoric, slept little, claimed that he could " save humanity”
, solve the problem of cancer and acne, and distributed money to strangers. Choose the most likely
diagnosis:
100% A manic attack
0% Panic disorder
0% Agitated depression
0% Schizoaffective disorder
0% Catatonic arousal

https://translate.yandex.com/en/doc 480/540
22:20 ,27.6.2023 �� C : ; 5 B

2171. a postoperative patient, who underwent removal of the lower lobe


of the left lung 6 days ago due to bacterial destruction and the development of pneumothorax, developed
shortness of breath up to 35/min, raO2 decreased to 70 mm Hg, coughed, and released a large amount
of sputum. During auscultation of the right and left lungs-a lot of different-sized wheezing, sharply
weakened breathing. What is the degree of respiratory failure in this patient?
100% II degree
0% III degree
0% I degree
0% 0 degree
0% IV degree

2172. a 27-year-old man complains of shortness of breath,itching and swelling of the face
that occurred 25 minutes ago. I ate crayfish and drank beer. The condition progressively worsens. Objectively:
agitated, puffy face, enlarged cheeks and lips. Respiratory rate-28 / min., audible
wheezing on inhalation and exhalation, with auscultation of the lungs conducting noises, maximum above
the sternum. Heart sounds are loud, heart rate is 108/min, blood pressure
is 150/90 mm Hg. the most likely cause of shortness of breath is:
100% Laryngeal edema
0% Bronchial spasm
0% Hypertensive crisis
0% Neurocirculatory dystonia
0% Alcohol intoxication

2173. in a newborn from the first pregnancy weighing 3500 g


, jaundice, lethargy, and decreased reflexes are observed from the first day. Objectively: jaundice of the skin of the second degree with
a saffron hue, liver + 2 cm, spleen +1 cm. Urine and stool are yellow. In the blood:
HB-100 g / l, red blood cells - 3,2 • 1012/l, white blood cells-18.7 • 109 / l, mother's blood 0 (i) Rh ( + ), blood

500

Downloaded from the site - online testing step

child A (II) Rh ( - ), bilirubin -170 mmol / l, indirect fraction. ALT and AST levels are normal. What
disease is suspected in the child?
100% Hemolytic disease of the newborn, AV0-conflict
0% Intrauterine hepatitis
0% Hemolytic disease of the newborn, Rh-conflict
0% Biliary tract atresia
0% Physiological jaundice

2174. a 6-year-old girl was admitted with complaints of body temperature rising to 37.2°C,
frequent and painful urination, which appeared after hypothermia. In the urine: cloudy urine,
specific gravity-1012, protein-0.033°/°°, microscopy: leukocytes-40-45 v / s, red blood cells-8-9 v / s
(fresh), flat epithelium: 5-8 v / s, mucus. What is the most likely etiological factor in this case
?
100% Escherichia coli
0% Staphylococcus aureus
0% Klebsiella pneumoniae
0% Proteus mirabilis
0% Candida albicans

https://translate.yandex.com/en/doc 481/540
22:20 ,27.6.2023 �� C : ; 5 B

2175. a 35-year-old woman complains of weakness, frequent bowel movements with blood impurities,
a decrease in body weight by 8 kg in 6 months, and periodically-an increase in temperature to 37.8 oC.
Objectively: body temperature -37.5°C, pulse-86 / min., blood pressure -110 / 70 mm Hg. skin
is pale, palpation of the abdomen is moderately painful. In the blood: HB-92 g / l, erythrocyte sedimentation
rate-35 mm/h. Colonoscopy: the rectal and sigmoid colon mucosa is granular, hyperemic,
edematous, unevenly thickened (pseudopolyps), bloody, on its surface - erosions and ulcers.
What pathology is most likely to cause intestinal damage in the patient?
100% Ulcerative colitis
0% Crohn's disease
0% Irritable bowel syndrome
0% Rectal cancer
0% Intestinal polyposis

2176. A 47-year-old man is a worker in a weaving shop, has 15 years of experience in this production
, and works in conditions of high-frequency intense noise. During a periodic
medical examination, he was diagnosed with occupational hearing loss.'What is the basis
for such a diagnosis?
100% Data from audiometry and hygienic assessment of working conditions
0% Work experience at this production facility
0% Characteristics of noise in this production
0% Results of the study of CNS indicators
0% Results of the study of the condition of the inner ear

2177. in district N, 30% of cases of breast cancer and 26% of cases of cervical cancer
are detected in stage IV. What management decision should be made in this situation?
100% Organize and conduct targeted medical examinations of women

501

Downloaded from the site - online testing step

0% Organize and conduct comprehensive medical examinations of women


0% Organize and conduct periodic medical examinations of women
0% Organize and conduct preliminary medical examinations of women
0% Organize and conduct total medical examinations of women

2178. The Health Department needs to assess the effectiveness of implementing


maternity and child protection programs in the region. The level of which integral indicator
is most appropriate to use for such an analysis?
100% Infant mortality rate
0% Morbidity of women
0% Morbidity of children
0% Disability of children
0% Physical development

2179.a woman, 6-7 weeks pregnant, applied to the doctor of the antenatal clinic. From the anamnesis
, it was found out that he has been suffering from severe diabetes mellitus since the age of 15. According to the oculist's conclusion
, retinopathy of the second century had 2 pregnancies that ended in stillbirth. The blood sugar content
is 15 mmol/l. Bimanual examination revealed that the uterus is enlarged (as in
pregnancy, up to 7 weeks). What are the doctor's tactics?
100% Termination of pregnancy is indicated
0% Prolonging pregnancy under the control of blood sugar levels
https://translate.yandex.com/en/doc 482/540
22:20 ,27.6.2023 �� C : ; 5 B
0% Geneticist's consultation
0% Endocrinologist's consultation
0% Hospitalization during critical pregnancy

2180. a 50-year-old patient who suffers from Addison's disease (developed after suffering
from tuberculosis of the lungs at a young age), during a fire in the apartment was very scared and
lost consciousness. The patient is pale, cold, pulse is threadlike, heart rate
-120 / min., blood pressure-60/30 mm Hg. what complication did the patient have?
100% Acute adrenal insufficiency
0% Thyrotoxic crisis
0% Morgagni-Adams-Stokes syndrome
0% Acute myocardial infarction
0% Cardiac tamponade

2181. a 32-year-old patient complains of " heartache”, bad mood, lack of appetite,
insomnia. These symptoms appeared gradually over 3 months for no apparent reason.
In mental status: answers the question in a low voice, the face is sad, the facial expressions are mournful,
the movements are slowed down, the mood background is reduced, emotionally depressed, the pace of thinking is slowed down.
The patient's condition improves in the evening and worsens early in the morning. Which specialist should
the patient be referred to?
100% A psychiatrist
0% A neurologist
0% An endocrinologist
0% A gastroenterologist

502

Downloaded from the site - online testing step

0% Medical psychologist

2182. a 24-year-old contact patient who is correctly oriented in place, time, and her own personality.
Agitated, can't sit still. She constantly says that her ’present condition is the fault
of the grandmothers 'whom she went to see; that' the voice in my head keeps repeating that I am not being treated correctly, and
constantly torments me'. Attention is narrowed in scope, riveted to experiences. There is no pre-condition criticism
. Which of the listed drugs can be prescribed to the patient as the main one?
100% Haloperidol
0% Shipramin
0% Trazadone
0% Benzobarbital
0% Lithium preparations

2183. a patient with type I diabetes mellitus, who was taken to the intensive care unit in a state of
ketoacidotic coma, was given intravenous treatment in the first three hours:
saline solution-2.0 l, Ringer's solution-800 ml, potassium chloride solution 7.5% - 40 ml.
Analysis of blood gases showed a blood pH of 6.85. what other remedy should be administered to the patient?
100% Sodium Bicarbonate solution
0% Red blood cell mass
0% Calcium chloride solution
0% Blood plasma
0% Promedol

https://translate.yandex.com/en/doc 483/540
22:20 ,27.6.2023 �� C : ; 5 B

2184. A 34-year-old patient is being treated in a psychiatric hospital for acute


schizophrenia. Objectively: he is in bed, mobile inhibited, there is no contact.
Doesn't answer the question. The pose is monotonous, the patient is hypomimic, there is a symptom of "proboscis"
waxy flexibility of the muscles, a symptom of "air cushion" in this state remains for about
a week. Nutrition is parenteral. Identify an existing motor-volitional disorder syndrome:
100% Catatonic stupor
0% Depressive stupor
0% Psychogenic stupor
0% Apathetic stupor
0% Exogenous stupor

2185. a 34-year-old patient complains of increased appetite, overweight, shortness of breath during
physical exertion. On examination: excessive accumulation of fat in the abdomen and
shoulder girdle, pale pink skin, male-type body hair,
no build, heart rate-90 / min., blood pressure-120/80 mm Hg. additional
studies: blood sugar-4.9 mmol / l, cholesterol-6.2 mmol / l. ophthalmoscopy: fundus
unchanged. Neurologist's examination: healthy. Make a diagnosis:
100% Primary alimentary-constitutional obesity, android type
0% Primary alimentary-constitutional obesity gynoid type
0% Secondary cerebral obesity
0% Secondary endocrine hypothyroid obesity
0% Secondary endocrine hypovarian obesity

503

Downloaded from the site - online testing step

2186. a 60-year-old woman complains of periodic pain in the knee joints, which occurs
more often during a long walk, going down stairs, in the evening, is disturbed in the first half of the night and
subsides in the morning after a long rest. The examination revealed an overweight
body. The joints are not externally changed, the amount of movement in them is not limited. An X-ray of the right
knee joint shows osteophytes. To prevent further progression
of the disease, you should recommend:
100% Avoid excessive loads on the knee joints
0% Limit your diet to foods rich in purines
0% Daily jogging
0% Regular intake of allopurinol
0% Short-term joint immobilization with a spar

2187. in a 30-year-old repeat child, labor lasts 8 hours. Contractions every minute for
50 seconds, active. Fetal heartbeat-156 / min., rhythmic. During external examination
, the head is located in the pelvic cavity. Vaginally: the opening of the cervix is complete,
the fetal head is in the plane of the exit from the small pelvis. Arrow-shaped seam in straight size, small
crown near the womb. What is the period of labor?
100% Second period of normal labor
0% Latent phase of the first period of normal labor
0% Active phase of the first period of normal labor
0% Rapid delivery
0% Preliminary period

2188. A 30-year-old female patient complains of not being pregnant for 3 years after
marriage. Increased nutrition, behind the midline of the abdomen, on the inner surface of the thighs

https://translate.yandex.com/en/doc 484/540
22:20 ,27.6.2023 �� C : ; 5 B
and in the periosal area, hair growth is noted. Menstruation from the age of 16, infrequent and
unexplained. Ultrasound: uterus of normal size, ovaries-4x5x5 cm, with a large number of cystic
inclusions.
100% WhatPolycystic
is the most likely
ovary diagnosis?
disease
0% Ovarian cysts
0% Chronic oophoritis
0% Menstrual disorders
0% Bilateral ovarian tumors

2189. a 38 - year-old patient complains of shortness of breath, a feeling of heaviness in the right hypochondrium.2
years ago I suffered from tuberculosis of the lungs. Her neck veins were swollen. Pulse-96 / min., rhythmic, small,
soft. The heart sounds are very weak. Liver + 7 cm. Free fluid in the abdominal cavity. ECG-
low voltage, EVS deviation to the right. Radiologically: 'small heart’.'What
is the most likely diagnosis?
100% Constrictive pericarditis
0% Hypertrophic cardiomyopathy, obstructive form
0% Myocarditis
0% Exudative pericarditis
0% Atrial membrane defect

504

Downloaded from the site - online testing step

2190. a 48-year-old patient complains of weakness, sweating, intense itching of the skin,
wave-like fever, enlarged cervical and supraclavicular lymph nodes. Objectively: pallor
of the skin and mucous membranes, cervical lymph nodes-mobile, tightly elastic, up to 3 cm in size, not painful,
not soldered to the skin. In the blood: red blood cells - 3,0-1012/l, Hb-100 g / l, white blood cells -14*109 / l,
EOSIS.6%, baz.- 3%, pal. - 11%, segm.- 69%, lymph - 7%, mon-4%, platelets - 280*109/l, erythrocyte
sedimentation rate-37 mm / hour. What morphological signs can be found during
a lymph node biopsy?
100% Berezovsky-Sternberg cells
0% Plasma cells
0% Heinz Taurus
0% Botkin-Gumprecht cells
0% Mallory's Taurus

2191. a 72-year-old woman suffers from type II diabetes mellitus, concomitant pathology-
hypertension II c., heart failure II B. uses metformin. The day before, I suffered
a hypertensive crisis, after which there was a sharp weakness, myalgia, increased thirst, dry
mouth, polyuria. Blood pressure-140/95 mm Hg, heart rate-98 / min.,
edema and smell of acetone are absent. What measures should be taken for the patient to prevent the development
of a comatose state?
100% Withdrawal of metformin, administration of short-acting insulin
0% Doubling the metformin dose
0% Use of hypotonic sodium chloride solution
0% Additional administration of prolonged insulin
0% Purpose of glibenclamide

2192. A 59-year-old man


with hypertension of the second grade is registered with a general practitioner of a polyclinic. The patient regularly takes ACE inhibitors

https://translate.yandex.com/en/doc 485/540
22:20 ,27.6.2023 �� C : ; 5 B
and 100% 1 time in 3How
calcium antagonists. months
often should the
therapist
0% examine the patient
1 time (except for periods of exacerbation)?
in 6 months
0% 1 time in 4 months
0% 1 time per year
0% 1 time in 9 months

2193. a 6 - year-old child with anemic and hemorrhagic syndromes. In the blood Hb-80 g / l,
CP-0.9, reticulocytes -20 / 00, white blood cells - 1,0-109/l, platelets -10 * 109 / L. what
is the most likely diagnosis?
100% Aplastic anemia
0% Thrombocytopenic purpura
0% Thrombocytopathy
0% Deficiency anemia
0% Lymphoblastic leukemia

2194. during the forensic examination of the corpse, the medical examiner described a linear wound in the parietal-temporal
region on the right, measuring 6.4 cm in size, with the edges drawn together, the edges uneven, covered with

505

Downloaded from the site - online testing step

abrasions; tissue membranes are visible in the depth of the wound. Give a name to the described wound:
100% Clogged
0% Dissected
0% Chopped
0% Kolotaya Street
0% Sliced

2195. the body of a 24-year-old woman with probable poisoning was found on the street. After examining
the scene and the body, the investigator ordered a forensic medical examination. In accordance with the current
Criminal Procedure Code of Ukraine, the appointment of such a study in this
case is mandatory for:
100% Determining the cause of death
0% Determining the type of death
0% Determining the antiquity of death
0% Determining the type of death
0% Determining the mechanism of death

2196. a 60-year-old patient complains of an almost constant feeling of heaviness and


fullness in the epigastrium, which increases after eating, belching with a rotten smell, sometimes
vomiting of food eaten 1-2 days ago, and weight loss. 12 years ago, he was first diagnosed
with a pyloric ulcer. Noted periodic "hungry" pains, for which he took
omeprazole. Deterioration within 3 months. Objectively: the "splashing noise" in
the epigastrium is determined. What complication are we talking about?
100% Pyloric stenosis
0% Penetration of a stomach ulcer
0% Functional spasm of the goalkeeper
0% Foreign body of the stomach (bezoar)
0% Malignancy of a stomach ulcer

https://translate.yandex.com/en/doc 486/540
22:20 ,27.6.2023 �� C : ; 5 B
2197. A 40-year-old female patient has uterine fibromyoma. During the last 10 years, she has been experiencing
long, heavy menstruation. Complaints of weakness, palpitations, shortness of breath during
exercise, decreased performance. Objectively: the skin is pale, dry,
brittle nails, reduced muscle strength, atrophy of the papillae of the tongue are observed. Heart rate -96
/ min. Systolic murmur at the apex of the heart. Red blood cells -2,6-1012/l, HB-70 g / l, CP-0.7,
reticulocytes-1.2, leukocytes - 4,6-109/l, platelets - 170*109/L. what disease can
the patient have?
100% Chronic post-hemorrhagic anemia
0% Hypoplastic anemia
0% Thalassemia
0% Iron deficiency anemia
0% Acute anemia

2198. a 39-year-old patient complains of dizziness and rapid fatigue. Three days ago
, he underwent surgery for an active duodenal ulcer complicated by bleeding.
Operation-suturing a bleeding ulcer. Objectively: the skin is pale. Arterial

506

Downloaded from the site - online testing step

blood pressure-100/60 mm Hg, pulse-98 / min. In the blood: red blood cells - 2,8-1012/l, Hb-76 g / l, CP-0.8,
reticulocytes-8%, platelets - 320*109/l, white blood cells-9.0 G / l, erythrocyte sedimentation
rate-20 mm / h. What is the most likely diagnosis?
100% Acute post-hemorrhagic anemia
0% Hypoplastic anemia
0% Neurocirculatory dystonia
0% Hemolytic anemia
0% B12 - deficiency anemia

2199. a 65-year-old man complains of respiratory dyspnea. COP has been ill for 15
years. Blood pressure-150/90 mm Hg, heart rate-52 / min. The heart
sounds are muffled, the accent is tone II over the pulmonary artery. In the lungs: fine-grained wheezing in the lower
parts. ECG: pathological Q in III, AVF visits without dynamics. Echocardiography: left ventricular dilation
, left ventricular ejection fraction -35%. What is the basis of hemodynamic disorders in the
patient?
100% Systolic dysfunction
0% Diastolic dysfunction
0% Systolic-diastolic dysfunction
0% Metabolic dysfunction
0% Violation of peripheral resistance

2200. A 62-year-old patient accidentally felt a dense,


slightly larger than a pea, mobile nodule in his left subclavian fossa, which was not soldered to the skin. When questioned
, it turned out that over the past 6 months, he had lost 12 kg of weight. Notes weakness, decreased
performance, and decreased appetite. What research is of primary importance for
making a diagnosis?
100% EFGDS
0% Lymph node puncture
0% Sternal puncture
0% Chest radiography
0% Ultrasound of the abdominal cavity

https://translate.yandex.com/en/doc 487/540
22:20 ,27.6.2023 �� C : ; 5 B

2201. A 25-year-old patient is being treated by a hematologist for hemophilia A. After falling from
a horizontal bar, acute hemarthrosis of the knee joint has developed. Objectively: the right knee joint is sharply
enlarged in volume, the skin over it is hyperemic. The bleeding time for Duke is 3 minutes,
the blood settling time for Lee-White is 20 minutes. Which drug is most effective for treating
this patient?
100% Recombinant factor VIII
0% Recombinant X Factor
0% Tromboconcentrate
0% Amben
0% Aminocislaproic acid

2202. A patient became acutely ill 2 days after returning from a trip to Nigeria. Complaints of
frequent liquid bowel movements in the form of "rice broth", unrestrained vomiting, muscle cramps.

507

Downloaded from the site - online testing step

Objectively: body temperature-35.0 oC, pulse-120 / min., blood pressure-50/30 mm Hg.


facial features are pointed, dark circles under the eyes. The stomach is painless. What is the most
likely diagnosis?
100% Cholera
0% Food toxicoinfection
0% Malaria
0% Hemorrhagic fever
0% Salmonellosis

2203. after a complicated delivery, the baby developed paralysis of the right arm, muscle tone
is reduced, and reflexes are not triggered. What complication did the baby have?
100% Plexitis of the right brachial plexus
0% Cervical radiculopathy
0% Congenital malformation
0% Infantile cerebral palsy
0% Syringomyelia

2204. A 48-year-old female patient complains of pain and weakness in the muscles of the neck, upper and lower
extremities, and fever. Ill for about 2 years. Objectively: body temperature -37.8°C.
Periorbital lilac-pink skin color. Palpation of the muscles of the upper and lower extremities
causes soreness and tightness. Reduced limb muscle strength. In the blood
, the erythrocyte sedimentation rate is 45 mm/h. To verify the diagnosis, it is most appropriate to perform:
100% Muscle biopsy study
0% Determination of blood creatine phosphokinase level
0% Determination of antibodies to native DNA
0% Microcirculation research
0% Study of the level of immunoglobulins

2205. the boy is 12 years old. During the year, the child periodically has bouts of pain in
the parotid region and epigastrium, nausea, loss of appetite, flatulence, loose
stools. Palpation: pain in the epigastrium, Desjardins ' pancreatic point
,Shoffar's choledochopancreatic zone, positive Mayo-Robson symptom. The level

https://translate.yandex.com/en/doc 488/540
22:20 ,27.6.2023 �� C : ; 5 B
of blood
100%and urine amylase
Chronic is elevated, sweat chlorides - 17 mmol / l. Ultrasound of the pancreas: increased
pancreatitis
echogenicity, expansion of the pancreatic duct. What will be the diagnosis?
0% Duodenal ulcer disease
0% Functional diarrhea
0% Chronic cholecystitis
0% Cystic fibrosis

2206. A 68-year-old patient went to a urology clinic complaining of urination calls


and lack of urine during the last 10 hours. Objectively:
a painful formation is palpated above the pubis, the upper edge of which reaches the navel, during rectal examination, the prostate
gland is enlarged, painless, and of a dense consistency. An additional study revealed the level
of prostate-specific antigen in the blood - 3.2 ng / ml. What is the most likely diagnosis?

508

Downloaded from the site - online testing step

100% Benign prostatic hyperplasia, acute urinary retention


0% Prostate cancer, acute urinary retention
0% Chronic paraproctitis, acute urinary retention
0% Benign prostatic hyperplasia, chronic urinary retention
0% Prostate sclerosis, acute urinary retention

2207. A 64-year-old diabetic patient was treated with metformin. Due to high
blood pressure, I took diuretics. Gradually, the effectiveness of the diuretic decreased,
nausea and vomiting appeared. Contact with the patient is difficult. The skin is dry.
There is no smell of acetone. Blood pressure-180/100 mm Hg. heart sounds are deaf. Pulse-98 / min. Respiration
is vesicular. The abdomen is painful in the epigastrium. Liver + 4 cm. Blood glucose - 48 mmol/ l, Na-156
mmol/l, K - 5.2 mmol/l, urea - 15 mmol/l. what pharmacological approach should
I start with?
100% Emergency hydration of the patient
0% Correction of Na + content in the blood
0% Purpose of sodium bicarbonate
0% Correction of blood pressure with sodium sulfate
0% Transfer the patient to protamine-zinc insulin treatment

2208. A 14-year-old girl lost consciousness during blood collection. The day before complained
of a headache. The skin is pale. Respiratory rate-20 / min., heart rate-80 / min.,
blood pressure-90/60 mm Hg. the stomach is soft. There are no meningeal symptoms. What
is the preliminary diagnosis?
100% Fainting
0% Collapse
0% Acute left ventricular failure
0% Epilepsy
0% Respiratory failure

2209. a 39-year-old patient complains of an increase in body temperature up to 41oc,headache,


and weakness. He became acutely ill, and on the 5th day of the disease, a roseolous-petechial rash appeared
on the lateral surface of the chest and back. RZK With Provacek rickettsias - 1: 640, lgM-89%. What
is the most likely diagnosis?
100% Typhus fever
0% Enterovirus infection

https://translate.yandex.com/en/doc 489/540
22:20 ,27.6.2023 �� C : ; 5 B
0% Brill's disease
0% Typhoid fever
0% Flu

2210. a 26-year-old patient


experienced mixed dyspnea, dry cough, and itchy skin half an hour ago after taking aminopenicillin for acute bronchitis. Objectively:
edema and redness of the skin in the orbital region, wheezing breath, pulse-114 / min., blood
pressure-90/60 mm Hg.:
100% Prednisone
0% Euphyllinum

509

Downloaded from the site - online testing step

0% Diphenhydramine
0% Activated Carbon
0% Heparin

2211. in an 18-year-old patient, blood pressure -120/70 mm Hg in the III intercostal space to the left of
the sternum edge systolic tremor. The left border of the heart is shifted 1 cm outwards. In the II-III intercostal
spaces on the left, there is a systolic-diastolic murmur, which increases during systole, and an accent of the second tone. What
is the most likely diagnosis?
100% Open Ductus arteriosus
0% Aortic coarctation
0% Ventricular septal defect
0% Atrial septal defect
0% -

2212. a woman in labor 3 weeks after giving birth had a body temperature of up to 38 ° C, developed a
cold, weakness, and pain in the left mammary gland. The mammary gland is enlarged, rough, painful
on palpation; there is no softening or fluctuation in the infiltrate area. In the blood: moderate
leukocytosis. What is the most likely diagnosis?
100% Serous mastitis
0% Lactostasis
0% Abscessed mastitis
0% Mastopathy
0% Gangrenous mastitis

2213. on the 7th day after delivery, a 28-year-old woman complained of pain in the right
breast, fever up to 39oc, coldness, and deterioration of her general condition.
During the examination, an infiltrate is determined in the upper-outer quadrant of the breast with
a softening area. What will be the doctor's tactics?
100% Surgical treatment
0% Antibacterial therapy
0% Discontinuation of lactation
0% Dynamic monitoring
0% Physical therapy treatment

2214. In city N, the incidence of myocardial infarction was studied in previous years (from
2009 to 2013). What type of epidemiological study was used?
100% Retrospective

https://translate.yandex.com/en/doc 490/540
22:20 ,27.6.2023 �� C : ; 5 B
0% Experimental
0% Prospective
0% Descriptive
0% Analytical report

2215. A 52-year-old female patient has been experiencing enlargement of the right breast for 2 years.
During the last 3 months, redness of the skin of this breast has appeared. On the right side

510

Downloaded from the site - online testing step

in the axillary region, a tight-elastic lymph node of up to 1.5 cm in size is palpated.


The right mammary gland is enlarged, its skin is hyperemic, a symptom of lemon peel,
the nipple is retracted. What disease is most likely in this case?
100% Breast cancer
0% Mastitis
0% Breast fibroadenoma
0% Axillary lymphadenitis
0% Mastopathy

2216. a 49-year-old patient complains of compression in the heart, weakness, acrocyanosis,


dilated subcutaneous veins, ascites, leg edema, and neck vein pulsation. TSVT-200 mm of water. st., pulse
of weak filling -100 / min., heart tones are muffled, there is no noise. Radiologically: small
heart with clear contours, calcareous deposits in the pericardial cavity. What is the most
likely diagnosis?
100% Constrictive pericarditis
0% Effusion pericarditis
0% Myocarditis
0% Mitral stenosis
0% Left-sided pneumonia

2217. a 52-year-old patient complains of periodic paroxysmal pain in the right lumbar
region, which radiates to the groin area and the inner surface of the right thigh,
urinary disorders, which are accompanied by cuts and blood impurities in the urine. In the urine: specific gravity -1014,
protein-0.078 g / l, red blood cells-1/2 of the visual field, white blood cells-5-7 in the visual field, a large number
of uric acid crystals. Specify the most likely preliminary diagnosis:
100% Urolithiasis
0% Chronic pyelonephritis
0% Acute glomerulonephritis
0% Kidney neoplasms
0% Acute cystitis

2218. A 20-year-old female patient went to the antenatal clinic complaining about the
absence of menstruation for 7 months. From the anamnesis: at an early age, she was ill with childhood
infections and sore throats, menarche from the age of 13, regular monthly periods, menstrual cycle of 28 days,
menstruation lasts 5-6 days, painless. 7 months ago I suffered stress. During the gynecological
examination, no changes were found on the part of the uterus and appendices. What is the most likely diagnosis?
100% Secondary amenorrhea
0% Primary amenorrhea
0% Algodismenorrhea
0% Oligomenorrhea
https://translate.yandex.com/en/doc 491/540
22:20 ,27.6.2023 �� C : ; 5 B
0% Fake amenorrhea

2219. the child is 8 years old, no complaints, active. The general condition is not disturbed. Moderate
systolic murmur with maximum sound in the III intercostal space to the left of the sternum,
which does not change during breathing, bifurcation of the second tone. What is the most likely diagnosis?

511

Downloaded from the site - online testing step

100% Atrial septal defect


0% Ventricular septal defect
0% Tricuspid valve insufficiency
0% Open Ductus arteriosus
0% Mitral valve prolapse

2220. during the professional examination, it was established that the 23-year-old woman had no pregnancies.
Bimanual examination revealed: the body of the uterus is of normal size, on the anterior wall - a dense
rounded formation on the pedicle associated with the uterus,up to 6 cm in diameter,non-painful, but
without features. Ultrasound confirmed the diagnosis of subserous uterine fibroids.
What treatment method should I suggest?
100% Conservative myomectomy
0% Uterine amputation
0% Defundation of the uterus
0% Uterine extirpation
0% High ultrahigh uterine amputation

2221. over the past three years, a family doctor has observed an increase in the incidence
of diphtheria among the population he serves. What medical and preventive measures should
be taken to reduce the incidence of diphtheria at the site?
100% Get the population vaccinated against diphtheria
0% Perform hospitalization of diphtheria patients
0% Perform dysenfection in the cell and record contact persons
0% Identify carriers and conduct a course of treatment for them
0% Conduct bicillin prophylaxis at the site

2222. a 1-year-old child was hospitalized on the 4th day of illness with complaints of fever
up to 39oc, shortness of breath. Objectively: cyanosis of the nasolabial triangle, with auscultation of the lungs
, the respiratory function is sharply weakened, wet crepitating wheezes. Standard antibiotic
therapy during the first three days of treatment is ineffective. Staphylococcal
etiology of pneumonia is suggested. Which antibacterial therapy is most appropriate in this case?
100% Vancomycin
0% Pennicillin
0% Ampicillin
0% Erythromycin
0% Azithromycin

2223. A 10-year-old boy was admitted to the polytrauma department after a blunt chest injury
caused by a fall from a bicycle. On admission, blood pressure is 110/80 mm Hg,
pulse is 96 / min. Chest X-ray is unencumbered. An echo cardiogram shows
up to 100 milliliters of free fluid in the pericardial cavity. An hour after admission

https://translate.yandex.com/en/doc 492/540
22:20 ,27.6.2023 �� C : ; 5 B
, signs of heart failure began to increase: swelling of the cervical veins, a decrease
in blood pressure to 90/70 mm Hg, an acceleration of the pulse to 120 / min. Auscultatively, the heart
tones are barely audible. What should be the primary medical tactics?
100% Pericardial puncture

512

Downloaded from the site - online testing step

0% IV Cardiac glycosides
0% Permanent oxygen therapy
0% Intravenous diuretics
0% Intravenous antibiotics

2224. A 38-year-old woman complains of an itchy skin rash that appeared a day
after taking an analgesic. Objective: the general condition of the patient is moderate, on the skin
of the face, trunk and limbs there is a profuse urticarial rash, which sometimes merges. What
medication should be prescribed to the patient first?
100% Prednisone
0% Enterosgel
0% Reosorbylact
0% Calcium chloride solution
0% Tavegil

2225. A 35-year-old female patient complains of severe general weakness, yellowing of the sclera,
heaviness in the right hypochondrium, periodic fever, joint pain, and bleeding gums. In
the blood: ALT-1.98 mmol/hl, AsAT-2.5 mmol/hl, total bilirubin - 105 mmol/l, direct-65
mmol/l, ANA in a titer of 1: 160, smooth muscle antibodies in a titer of 1: 40, an increase in IgG by 2
times. HLW-DNA ( - ), HSU-RNA ( - ). What is the preliminary diagnosis?
100% Autoimmune hepatitis
0% Systemic lupus erythematosus
0% Gilbert's disease
0% Viral hepatitis B
0% Wilson's disease-Konovalov

2226. A 45-year-old woman complains of knee-like pain in the right hypochondrium, which occurs
after eating fatty foods, physical exertion, radiating to the right shoulder blade and right shoulder,
and decreases when taking antispasmodics. He is ill for a year, periodically notes
yellowing of the sclera during bouts of pain. What examination method should
be prescribed first to establish the diagnosis?
100% Ultrasonography
0% Computer tomography
0% Radiography of the abdominal organs
0% Duodenal probing
0% Fibrogastroduodenoscopy

2227. A 40-year-old victim was stabbed under the right shoulder blade. Objectively:
normal skin color, pulse-96 / min., rhythmic, vesicular respiration on both sides,
respiratory rate-20 / min., blood pressure -130 / 90 mm Hg. back wound-4. 0x0.5 cm, blood flow.
No pathology was found on the X-ray survey. What will be the tactics?
100% Primary surgical treatment of the wound
0% Puncture of the pleural cavity

https://translate.yandex.com/en/doc 493/540
22:20 ,27.6.2023 �� C : ; 5 B
0% Thoracocentesis
0% Thoracoscopy

513

Downloaded from the site - online testing step

0% Thoracotomy

2228. A 35-year-old female patient complains of malaise, fever, sore throat and neck
radiating to the lower jaw. All this within 7 days was accompanied by increased
nervousness, sweating, weight loss. The thyroid gland is moderately enlarged and has limited
pain during palpation. The survey revealed data corresponding to thyrotoxicosis. In
the blood, the erythrocyte sedimentation rate is 64 mm / hour. Subacute thyroiditis was diagnosed.
The cause of thyrotoxicosis in subacute thyroiditis is:
100% Destruction of thyrocytes and the release of follicle contents into the bloodstream
0% Production of antibodies to TSH receptors
0% TSH overproduction in response to inflammatory changes in the thyroid gland
0% Compensatory hyperfunction of the thyroid gland in response to inflammatory changes
0% Hyperproduction of thyroid hormones by the thyroid gland

2229. the woman in labor on the 6th day had a body temperature of up to 38oC, fever
and weakness appeared. On examination, the right breast is enlarged due to an infiltrate without
clear contours in the upper quadrant, painful during palpation. What is the likely diagnosis?
100% Infiltrative mastitis
0% Purulent mastitis
0% Phlegmonous mastitis
0% Gangrenous mastitis
0% Lactostasis

2230. a 42-year-old patient complains of severe weakness, significant weight loss, increased
hair loss, bleeding gums, menstrual irregularities, pain in the bones and
muscles, bloating, rumbling, emptying 5-10 times a day. Feces are thin and smelly.
He has been ill since childhood. Objectively: the tongue is moist, the papillae are smoothed, the abdomen is moderately swollen, more
around the navel. Coprogram data: found a lot of fatty acids, connective tissue fibers,
fiber. What is the patient's diagnosis?
100% Chronic enteritis
0% Chronic colitis
0% Cohn's disease
0% Whipple's disease
0% Non-specific ulcerative colitis

2231. A 32-year-old birthing child in the first stage of labor with moderate contractions. This pregnancy
is the fourth, the previous two ended in a medical abortion, the third-a caesarean section due
to placenta previa. Suddenly, Rodilli developed severe abdominal pain, weakness,
and blood pressure dropped to 80/50 mm Hg. Moderate spotting appeared from the vagina.
The fetal heartbeat is not listened to, small parts are determined to the left of the midline
of the abdomen. Labor activity stopped. What is the most likely diagnosis?
100% Uterine rupture that occurred
0% Threat of uterine rupture. Fetal distress
0% Detachment of the normally located placenta
0% Clinically narrow pelvis

https://translate.yandex.com/en/doc 494/540
22:20 ,27.6.2023 �� C : ; 5 B
514

Downloaded from the site - online testing step

0% Discoordinated labor activity

2232. a 57-year-old patient complains of a feeling of dryness, pain during swallowing, frequent
annoying cough, hoarse voice. The disease developed suddenly. During laryngoscopy
,the laryngeal mucosa is hyperemic, the vocal folds are swollen, and there is a viscous secretion in the laryngeal lumen
. What is the most likely diagnosis?
100% Acute laryngitis
0% Acute stenosing laryngotracheitis
0% Bronchial asthma
0% Phlegmonous laryngitis
0% Laryngeal diphtheria

2233. A 42-year-old patient complaining of acute pain in the epigastric region, which began
suddenly against the background of complete health, and an increase in body temperature to 38.5 oC, with a positive
Shchetkin-Blumberg symptom, needs surgical medical care. What kind
of time-based surgical care can be provided to the patient?
100% Emergency (urgent)
0% Emergency (urgent) or postponement
0% Planned
0% Deferred or planned
0% Emergency (urgent) or planned

2234. a patient operated on for acute paraproctitis, during 5 days on the background
of complex therapy, with positive dynamics of the local course of the disease,
there are colds,hyperthermia, tachycardia, euphoria. The doctor suggested that the patient
developed sepsis. What research can confirm the diagnosis?
100% Blood culture for the presence of the pathogen
0% Lung radiography
0% Ultrasound examination of the liver
0% Determination of the degree of microbial contamination of the wound
0% Determination of the level of medium-weight molecules

2235. a 25-year-old woman in labor had a second, urgent, normal delivery. Day 3
of the postpartum period. Body temperature-36.8 oC, pulse-72 / min., blood pressure-120/80
mm Hg. mammary glands are moderately rough, nipples are clean. The abdomen is soft, painless, the bottom of the uterus
is 3 transverse fingers below the navel. Lochia are bloody, moderate. What is the likely diagnosis?
100% Physiological course of the postpartum period
0% Subinvolution of the uterus
0% Postpartum metroendometritis
0% Remnants of placental tissue after delivery
0% Lactostasis

2236. during the examination of a 5-year-old child complaining of a persistent cough with
purulent sputum and wet wheezing on the right side in the posterior-lower parts, it was revealed: Purulent

515

https://translate.yandex.com/en/doc 495/540
22:20 ,27.6.2023 �� C : ; 5 B

Downloaded from the site - online testing step

endobronchitis of II-III degree, mainly on the right, sweat chlorides-36 mmol / l, on CT - cylindrical
bronchiectasis in S9 and S10. Suggest the best treatment method for this child:
100% Removing affected segments
0% Kinesotherapy
0% Genetic engineering therapy
0% Bronchial tree lavage
0% Long-term antibiotic therapy

2237. the mother of a 1-year-old child complains of a persistent, frequent, unproductive


cough, sometimes to the point of vomiting. During an objective examination, the patient has
rapid breathing, moderate retraction of the lower intercostal muscles, and an increase
in the anteroposterior size of the chest. During respiratory infections
, bronchial obstruction occurs. In the period of new birth, he suffered meconial ileus. Choose
a priority survey:
100% Research on sweat chlorides
0% Chlamydia and mycoplasma research
0% Chest radiography
0% Computed tomography of the lungs
0% Genetic testing

2238. a 55-year-old patient complains of constant pain in the lower back, pelvic bones, general
weakness, sweating, and weight loss. Has an appetite, gets sick for 6 months. Ro-listening
to pelvic bones: pronounced osteoporosis, multiple rounded destructions up to 1 cm in diameter with clear
contours. Blood test: red blood cells - 2,7*1012/l, erythrocyte sedimentation rate-70 mm / h,
platelets - 120*109/l, total protein -110 g / l. in the urine: protein-7 g / l. what is the preliminary diagnosis?
100% Myeloma disease
0% Ankylosing spondylitis
0% Chronic lumbosacral sciatica
0% Metastatic stomach cancer
0% Chronic glomerulonephritis, Nephrotic syndrome

2239. A 25-year-old female patient complains of several tight, painful nodes in the right
armpit area, an increase in body temperature up to 38°C, and general weakness. He has been ill for 3 days.
Locally: in the right axillary region there are three nodular formations that rise above
the surface of the skin, ranging in size from 0.5 cm to 1.5 cm in diameter, sharply painful on palpation, with
clear contours, the skin above them is purple-cyanotic, the elevation of the upper limb
is limited due to pain. Make a diagnosis:
100% Purulent hydradenitis
0% Abscess
0% Carbuncle
0% A boil
0% Purulent lymphadenitis

2240. A 54-year-old patient complained to a neurologist of hand tremor, headache,


metallic taste in the mouth, insomnia, and increased salivation. From the medical history, it is known that he

516

https://translate.yandex.com/en/doc 496/540
22:20 ,27.6.2023 �� C : ; 5 B
Downloaded from the site - online testing step
He has been working at a fluorescent lamp factory for more than 10 years. What will be your diagnosis?
100% Chronic mercury intoxication
0% Chronic lead intoxication
0% Chronic nitrocrass intoxication
0% !manganese intoxication
0% Benzene intoxication

2241. the boy is 2 years old, body weight-9.0 kg, Height-80 cm, head circumference - 45 cm. The values of body weight,
height, and head circumference are between "-2” and" -3 " standard deviations. Determine
the child's level of physical development:
100% Low
0% Very low
0% Average
0% High
0% Very high

2242. a 26-year-old female patient complains of irritability, tearfulness, headache,


engorgement and tenderness of the mammary glands. Symptoms occur 5-6 days before menstruation and
disappear on the first day. Vaginally: uterus and appendages unchanged. What is the most likely
diagnosis?
100% Premenstrual syndrome
0% Algodismenorrhea
0% Neurosis
0% Genital endometriosis
0% Mastopathy

2243. An emergency medical team delivered a patient with acute myocardial infarction
to the regional cardiology center, where cardiac surgery was performed. Under
what type of medical care were these medical services provided?
100% Highly specialized medical care
0% Emergency medical care
0% Qualified medical care
0% Specialized medical care
0% Palliative care

2244. A 67-year-old patient has been suffering from COP, stable tension angina for 6 years.
Objectively: the general condition is satisfactory. Respiratory rate-16 / min. In the lungs in the lower parts
of the stagnant wheezing. ECG: atrial fibrillation, LV hypertrophy and overload, no focal
changes. Heart rate-96 / min, blood pressure-156/92 mm Hg. liver +2.0
cm. Moderate swelling on the lower legs. Indicators: troponin T-0.08 ng / ml, fasting glucose-7.2
mmol / l. What is the recommended level of EOM (international normalized ratio) in
the blood for the prevention of thromboembolic complications?
100% 2,0-3,0
0% 1,0-1,5
0% 1,0-2,0

517

Downloaded from the site - online testing step

0% 0,8-1,2

https://translate.yandex.com/en/doc 497/540
22:20 ,27.6.2023 �� C : ; 5 B
0% 1,2-1,5

2245. during the examination for lumbar pain and periodic


increase in blood pressure, the patient was found to have the right kidney - 7. 3x3. 2 cm,
parenchyma thickness - 0.6 cm. The structure of the kidney is not changed. On excretory urograms, the right kidney
is reduced in size, CHMS is not changed, and kidney function is not reduced. Pathologies from the left
kidney were not found. What is the most likely diagnosis?
100% Hypoplasia of the right kidney
0% Doubling of the right kidney
0% Cancer of the right kidney
0% Solitary cyst of the right kidney
0% Aplasia of the right kidney

2246. a district pediatrician examined a 1-month-old baby born from the first physiological
pregnancy, during normal labor, weighing 3,400 g. breastfed,
who gained 750 g in the first month, the doctor prescribed specific prevention of rickets. What drug
did he choose?
100% 0.125% cholecalciferol solution
0% 0.125% ergocalciferol solution
0% 0.0625% ergocalciferol solution
0% 3.44% retinol acetate solution
0% 5% tocopherol acetate solution

2247. Rodilya is accepted for the third delivery in the II period. Height-172 cm, weight - 67 kg. She has a history of two
normal births and three articular abortions. She gave birth to a boy weighing 3600.0 g. 15
minutes after the birth of the fetus, bleeding began, reached 380 ml and does not stop.
There are no signs of placental abruption. What will be the diagnosis?
100% Third delivery, urgent, bleeding in the third stage of labor
0% Third delivery, urgent, bleeding in the second stage of labor
0% Third delivery, bleeding in the early postpartum period
0% Third delivery, bleeding in the late postpartum period
0% Third birth, physiological course of the third period

2248. A 36-year-old patient went to the family doctor with complaints of shortness of breath, rapid
heartbeat, rapid fatigue, and hemoptysis. Objectively: the cheeks have a blush with a cyanotic
tinge. During auscultation: the first tone was increased above the apex of the heart, the second tone was accentuated above
the pulmonary artery, presystolic murmur above the apex; the heart rate was 80 / min,
the rhythm was regular, blood pressure was 100/60 mm Hg. the doctor referred the patient for
echocardiographic examination. What will be the preliminary diagnosis?
100% Mitral stenosis
0% Mitral defect with predominance of stenosis
0% Congenital heart disease
0% Aortic insufficiency
0% Tricuspid insufficiency

518

Downloaded from the site - online testing step

2249. a 53-year-old patient complains of acute pain in the right hypochondrium, which occurred 2
hours after dinner, nausea, vomiting of bile. Ill for 18 hours. The patient is restless, fidgeting
in bed. Pulse -98 / min., tongue covered with white layers, moist. Body temperature -38.2

https://translate.yandex.com/en/doc 498/540
22:20 ,27.6.2023 �� C : ; 5 B
oC. During the examination: the right half lags behind in the act of breathing, palpation in the right
hypochondrium muscle tension, soreness. Positive frenicus-a symptom and
a Grekov symptom. Symptoms of peritoneal irritation are negative. Make a diagnosis:
100% Acute cholecystitis
0% Acute pancreatitis
0% Acute peritonitis
0% Acute appendicitis
0% Intestinal obstruction

2250. a 38-year-old patient, a bus driver, received a severe spinal cord injury during working hours as a result of a car accident
(there is an industrial accident report). As a result, he
was recognized as a disabled person of the first group. Determine the cause of disability:
100% Occupational injury
0% Common disease
0% Occupational disease
0% Severity of injury
0% Traffic accident

2251. a family doctor who works in a family outpatient clinic has a family with
a cancer patient to serve. During the flu epidemic, all family members became ill, and the patient's flu
was complicated by pneumonia; there was a need for constant systematic therapy and anesthesia.
What decision should a family doctor make?
100% Refer the patient to the palliative care unit of the hospital or to a hospice
0% Refer the patient to the infectious diseases department of the hospital
0% Organize a home hospital
0% Call an infectious disease specialist for consultation
0% Prescribe vaccination to the patient

2252. Ultrasound examination of a 56-year-old patient revealed the presence of vegetations


on the artificial mitral valve, which was implanted 3 years ago, and sub-valve
structures with signs of vegetation destruction and the development of artificial valve insufficiency. What
treatment is indicated for the patient?
100% Operative (reprosthetics)
0% Appointment of vasodilators
0% Follow-up
0% Artificial pacemaker implantation
0% Limiting physical activity

2253. a patient complains of weight loss, pain in the lower abdomen, bowel movements with
admixtures of mucus and blood up to 15 times a day. During fibrocolonoscopy
, local pseudopolypous growths, flat surface ulcerative areas
of irregular shape, not merging, covered with mucus and fibrin, and contact bleeding were found in the sigmoid colon.

519

Downloaded from the site - online testing step

What is the most likely diagnosis?


100% Non-specific ulcerative colitis
0% Pseudomembranous colitis
0% Irritable bowel syndrome
0% Crohn's disease

https://translate.yandex.com/en/doc 499/540
22:20 ,27.6.2023 �� C : ; 5 B
0% Intestinal polyposis

2254. a 45-year-old patient complains of nocturnal attacks of suffocation up to 2-4 times a month, a feeling
of expiratory shortness of breath during physical exertion. In the anamnesis-seasonal pollinosis
for 20 years. During spirometry, the following data were obtained: FEV1-62%, FVC -70
% of the prescribed values. Reversibility of bronchial obstruction is 17.9%. What drugs for basic therapy
should be prescribed to this patient?
100% Inhaled glucocorticoids
0% Systemic glucocorticoids
0% Kromons
0% Leukotriene modifiers
0% Short-acting bronchodilators

2255. a 32-year-old female patient developed lower abdominal pain,


frequent and painful urination, and low-grade body temperature after hypothermia. In the urine analysis: protein-0.3 g / l,
white blood cells in the field of vision, red blood cells-12-15 in the field of vision. What disease can be assumed?
100% Acute cystitis
0% Acute adnexitis
0% Ectopic pregnancy
0% Acute appendicitis
0% Acute pyelonephritis

2256. a 38-year-old patient complains of sore throat and suffocation. Got sick after drinking
cold milk. Breathing is noisy, respiratory rate-28 / min., body temperature-39oc.
Indirect laryngoscopy: epiglottis edema and hyperemia, no glottis visible. What
urgent surgical measure should be taken?
100% Tracheostomy
0% Conicotomy
0% Intubation
0% Artificial respiration
0% Tracheal puncture

2257. A 70-year-old patient complained of vomiting after eating, losing weight, and lack
of appetite. On examination, the turgor of the skin is reduced, in the left supraclavicular region there is a dense lymph
node with a diameter of 1 cm. During palpation of the abdomen, an overgrown stomach is determined
, and a tumor-like formation is palpated in the epigastrium. What pathology determines such a picture?
100% Stomach cancer
0% Peptic ulcer disease of the stomach
0% Gastritis

520

Downloaded from the site - online testing step

0% Pancreatitis
0% Duodenal ulcer

2258. during the plane crash at the air show, adults and children with
mechanical, thermal and combined injuries were among the victims. Everyone was in a state of mental stress.
Only two ambulance crews were on the field in the first minutes after the crash,while

https://translate.yandex.com/en/doc 500/540
22:20 ,27.6.2023 �� C : ; 5 B
the rest of the teams were driving up to the parking lot. Which category of victims
100%
should Children
have received firstregardless
aid in the of theplace?
first severity of the condition
0% Elderly people in serious condition
0% Children in serious condition
0% Persons in serious condition regardless of age
0% Elderly people regardless of their condition

2259. A 5-year-old child was admitted to the hospital complaining of a temperature of 38 ° C and pain in his right
side. It is known that he is ill for a week. Objectively: the condition is severe, due to intoxication and
mixed respiratory failure. Percussion: on the right below the angle of the scapula
there is a dull sound, auscultation: hard breathing, under the scapula on the right there is no respiratory
noise. Leukocytosis with neutrophil shift to the left, increased erythrocyte sedimentation
rate (ESR). Your diagnosis:
100% Pleurisy
0% Pneumonia
0% Obstructive bronchitis
0% Appendicitis
0% Staphylococcal lung destruction

2260. a 65-year-old patient complains of a rash, severe pain in the right scapular region.
Objectively: linear pink-red swollen
foci, somewhat infiltrated, with clear borders, are located on the skin in the subscapular region on the right. Vesicles with
transparent contents are grouped on the surface of the foci. Make a diagnosis:
100% Shingles (Herpes Zoster)
0% Duhring's dermatitis
0% Erysipelas
0% Allergic dermatitis
0% Impetigo

2261. a patient operated on for acute paraproctitis, during 5 days on the background
of antibacterial and detoxifying therapy, with positive dynamics of the local course
of the disease, there are colds, hyperthermia, tachycardia, euphoria. The doctor assumes that the
patient has developed sepsis. What research can confirm the diagnosis?
100% Blood seeding for the presence of the pathogen
0% Lung radiography
0% Ultrasound examination of the liver
0% Determination of the degree of microbial contamination of the wound
0% Determination of the level of medium-weight molecules

521

Downloaded from the site - online testing step

2262. a 6 - year-old girl with acute respiratory viral infection (ARVI) showed changes
in the urine analysis: traces of protein, white blood cells-30-40 v / s, red blood cells (fresh) - 10-12 v / s. blood
pressure-100/60 mm Hg. which of the above diagnoses is most likely?
100% Urinary system infection
0% Acute glomerulonephritis
0% Hemorrhagic vasculitis
0% Vulvovaginitis
0% Urolithiasis

https://translate.yandex.com/en/doc 501/540
22:20 ,27.6.2023 �� C : ; 5 B
2263. among residents of homestead plots of p. Ivanovka, adjacent to the field of
the agrofirm Druzhba, 1 week after irrigation with pesticides (hexachlorane)
, symptoms of pesticide poisoning appeared. As a result of laboratory studies, the content of hexachlorane in
the soil of private estates adjacent to the field exceeded the maximum permissible concentration by 2 times, in the air - by 10 times, in the
water
of wells (the water supply of the settlement is decentralized) - by 2 times. What is the type
of impact
100% of pesticides on the human body that occurs?:
Comprehensive
0% Isolated
0% Combined
0% Combined
0% -

2264. after nervous stress, a 17-year-old girl developed shortness of breath, a” foreign
body " sensation in the throat, and a stabbing and aching pain in the upper part of the heart. Objectively: red spots
on the face and neck, vesicular respiration, respiratory rate - 26/min. The borders of the heart are normal,
the tones are clear, rhythmic, the heart rate is 120 / min., the blood pressure is 130/80 mm Hg.
the ambulance doctor believes that this is a somatoform disorder. Which drug will be most useful
in this situation?
100% Corvalolum
0% Aspirin
0% Nitroglycerin
0% Paracetamol
0% Drotaverine

2265.The city has a centralized drinking water supply system. Its


source is a surface reservoir-a river that belongs to the second
class of water sources according to water quality indicators. During the current laboratory monitoring of water quality at the point
before water enters the external distribution network, "
two consecutive deviations of water quality in terms of epidemic safety indicators were recorded by the local SES. What is the most
likely
reason for the deterioration
100% of water
Unsatisfactory quality
operation in terms
of water of epidemic
treatment (main)indicators?
facilities
0% Deterioration of the sanitary condition of a water body-river
0% Violation of sampling rules
0% Unsatisfactory transport of the sample to the laboratory
0% Water stagnation in the distribution network

522

Downloaded from the site - online testing step

2266. 37-year-old female patient, chemical engineer. Worries about bruising on the abdomen, thighs, shins after
minor injuries or for no reason. On examination, numerous small and somewhat larger
bruises were found, which sometimes merge (from 1x1 to 4x5 cm). !there are no significant changes. Menstruation lasts up to 7
days. The spleen is not palpable. In the blood: HB-92 g / l, white blood cells-7, 2 * 109 / l, platelets - 6 •
109/l, erythrocyte sedimentation rate-33 mm / hour. What is the pathogenesis of this disease?
100% Reducing platelet life expectancy
0% Reducing the level of anti-hemophilic globulin
0% Reduced blood prothrombin levels
0% Reduced fibrinogen levels
0% Increased fibrinolytic activity

https://translate.yandex.com/en/doc 502/540
22:20 ,27.6.2023 �� C : ; 5 B
2267. the child is 8 years old. Complaints of a body temperature of 39.3 oC, headache, vomiting. Day 1
of the disease. The skin is clean. The pharynx is hyperemic. The language is overlaid. Conjunctivitis. Scleritis.
Weakly expressed rigidity of the occipital muscles. Positive Kernig symptom on the right. CSF:
cytosis-340 / µl, lymphocytes-87%, neutrophils-13%, protein-140 mg/l. what is the most likely
diagnosis?
100% Serous meningitis
0% Arachnoiditis
0% Neurotoxicosis
0% Brain tumor
0% Purulent meningitis

2268. a four-year-old boy spent the whole day at the beach. In the evening, the child
developed a headache, weakness, and vomiting. During an objective examination: the face is hyperemic, body temperature
-38.8 oC, respiratory rate -28 / min., heart rate-130 / min. The most likely
cause of this condition is:
100% Sunstroke
0% Anaphylactic shock
0% Syncopal state
0% Dizziness
0% Collapse

2269. a 13-year-old girl has been complaining for 5 years of pain in the right hypochondrium
radiating to the right scapula, pain attacks are associated with a violation of the diet, they are short-lived, easily
relieved by antispasmodic agents. During an attack of pain, palpation of the abdomen is painful,
as much as possible at the point of projection of the gallbladder. The patient is most likely to have:
100% Biliary dyskinesia
0% Chronic cholecystitis
0% Chronic gastroduodenitis
0% Chronic pancreatitis
0% Duodenal ulcer disease 12

2270. in an 18-year-old student, the skin of the extensor surfaces of the extremities and scalp
shows a diffuse pulpy rash of bright red color, measuring from
1.0 to 5.0 cm in diameter, the surface of which is covered with silvery scales in the center. To apply

523

Downloaded from the site - online testing step

method of scraping, it is possible to detect the symptom of "stearin stain ""terminal film" and
"blood dew" what preliminary diagnosis does it seem most likely to you?
100% Psoriasis
0% Lichen planus erythematosus
0% Secondary syphilis
0% Papular necrotic tuberculosis of the skin
0% Atopic dermatitis

2271. A 65-year-old patient was hospitalized in the intensive care unit in a comatose state. He has been suffering
from type II diabetes for 10 years. The last 2 weeks were marked polyuria,
polydipsia. Serum glucose -30 mmol / l, arterial blood pH-7.3. Osmolarity
of blood plasma-350 mOsm/l. the patient was diagnosed with diabetic hyperosmolar coma. What
is the main pathogenetic mechanism of coma?
100% Dehydration
https://translate.yandex.com/en/doc 503/540
22:20 ,27.6.2023 �� C : ; 5 B

0% Hypocoagulation
0% Hyponatremia
0% Increased tubular reabsorption
0% Increased glomerular filtration rate

2272. A 10-year-old boy with hemophilia has symptoms of acute


respiratory viral infection with fever. Which of the above drugs, administered
for antipyretic purposes, is contraindicated in this patient?
100% Acetylsalicylic Acid
0% Drotaverine
0% Pipolfen
0% Paracetamol
0% Ibuprofen

2273. a 14-year-old girl complains of sleep disorders, weight loss, palpitations,


cardialgia, and fatigue. There is hyperplasia of the thyroid gland of the II century, exophthalmos.
What disorders of hormonal levels are most characteristic of this disease?
100% Increased thyroxine and triiodothyronine levels
0% Increased thyroid-stimulating hormone
0% Increased protein-bound iodine levels
0% Thyroxine reduction
0% Reduced Triiodothyronine

2274. a patient was hospitalized in a medical and preventive institution with the following diagnosis:
urolithiasis. In the course of chemical analysis, it was found that the stones consist of
uric acid salts (urates). Nutrition for this pathology should be adjusted in
the direction of reducing the content of food in the diet.:
100% Meat products
0% Dairy products
0% Grain products

524

Downloaded from the site - online testing step

0% Egg products
0% Vegetables and fruits

2275. during the clinical examination of the child, there is a sharp pallor of the skin with
a grayish tinge, anemia, the appearance of basophilic granularity in the cytoplasm of polychromatophilic
erythroblasts and reticulocytes. In the urine, the content of delta-aminolevulinic acid and
coproporphyrin is increased. Excessive intake of what xenobiotic in the child's body led to
poisoning?
100% Lead
0% Cadmium
0% Mercury
0% Nitrates
0% Arsenic

2276. a 10-year-old boy went to the doctor with complaints of general weakness, rapid

https://translate.yandex.com/en/doc 504/540
22:20 ,27.6.2023 �� C : ; 5 B
fatigue, irritability, decreased performance, bleeding gums, petechiae on the legs.
What vitamin deficiency can occur in this case?
100% Ascorbic acid
0% Thiamine
0% Riboflavin
0% Vitamin A
0% Vitamin D

2277. in a full-term baby,on the 3rd day of life, erythema,


erosive surfaces, cracks, and peeling of the epidermis are found on various skin areas. The baby looks like it's been scalded with boiling
water.
A positive symptom of Nikola was detected. The child's general condition is serious. Marked
restlessness,
100% hyperesthesia, and febrile fever. The most likely diagnosis in this case is:
Exfoliative dermatitis
0% Phlegmon of the newborn
0% Figner's pseudofurunculosis
0% Newborn's pemphigus
0% Mycotic erythema

2278. A 7 - year-old boy has been undergoing medical treatment for a month. At admission
, there were pronounced edema, proteinuria-7.1 g / l, Protein in the daily urine-4.2 g.in
the biochemical analysis of blood, hypoproteinemia (43.2 g/l), hypercholesterolemia (9.2
mmol / l) were retained. Which of the listed variants of glomerulonephritis is most likely to occur in the
patient?
100% Nephrotic
0% Nephritic
0% Isolated urinary tract
0% Iematuric
0% Mixed

525

Downloaded from the site - online testing step

2279. A 38 - year-old female patient went to a antenatal clinic complaining of moderate bloody
discharge from the genital tract, which occurred after a 1.5-month delay in the next menstrual period. During
the vaginal examination: the cervix is not eroded, the "pupil" symptom ( + + + ); the uterus is not
enlarged, dense, mobile, not painful; the appendages on both sides are not enlarged, not painful; the arches
are deep. What is the most likely diagnosis?
100% Dysfunctional uterine bleeding
0% Internal endometriosis of the uterine body
0% Ectopic pregnancy
0% Uterine pregnancy
0% Cancer of the uterine body

2280. The heat generated by the body of a hot shop worker in conditions of high
temperatures and low humidity is lost mainly by evaporation, which can lead
to convulsive illness. What kind of exchange is decisive in this case?
100% Water-salt
0% Carbohydrate
0% Adipose tissue
0% Protein powder
https://translate.yandex.com/en/doc 505/540
22:20 ,27.6.2023 �� C : ; 5 B

0% Vitamin Complex

2281. a 58-year-old patient complains of bloating, constipation, severe weakness,


and weight loss. During the last month of bowel movements only after taking laxatives.
During palpation of the abdomen, a dense formation is observed in the left iliac region. What
is the most likely diagnosis?
100% Sigmoid colon cancer
0% Colorectal cancer
0% Tumor of the mesentery of the colon
0% Tumor of the retroperitoneal space on the left
0% Cancer of the left ureter

2282. A 16-year-old teenager complains of periodic weakness, dizziness,


and heaviness in the left hypochondrium. the skin and visible mucous membranes are jaundiced.
Tower skull. Liver + 2 cm, lower pole of the spleen at the level of the navel. In the blood: erythrocytes -2.7
× 1012/l, Hb - 88 g/l, leukocytes -5.6 × 109 g/l, erythrocyte sedimentation rate -15 mm/h.
Indicate the most likely change in the patient's bilirubin level:
100% Increase in free bilirubin
0% Increased bound bilirubin
0% Increase in both bilirubin fractions
0% Reduced bound bilirubin
0% Reduced free bilirubin

2283. a 28-year-old woman went to the doctor with complaints of puffiness of the face, swelling of the legs,
and sometimes sees urine the color of "meat slop". As a teenager, she often had sore throats.
Objectively: pale skin, temperature-36.8 oC, pulse-68 / min., rhythmic. Blood
pressure - 170/110 mm Hg. what changes in urine are most likely?

526

Downloaded from the site - online testing step

100% Proteinuria, hematuria, and cylindruria


0% Increased relative density, hematuria, bacteriuria
0% Reduced relative density, proteinuria, poor urinary sediment
0% Erythrocyturia and urinosuria
0% Reduced relative density, proteinuria, myoglobinuria

2284. a 64-year-old patient complains of general weakness, noise in the head, hoarseness of voice.
Objectively: pallor with an icteric tinge, red tongue with smooth papillae,
asymmetry of tactile and pain sensitivity. Pulse-120 / min, blood pressure-80/50 mm Hg.
palpable spleen. In the blood: HB-58 g / l, red blood cells - 1,2-1012/l, white blood cells -2,8
-109/platelets-140-109/l, erythrocyte sedimentation rate-17 mm / h, anisocytosis, poikilocytosis-
pronounced (++). What research will be crucial to elucidate the genesis of anemia?
100% Sternal puncture
0% Indirect Coombs test
0% Direct Coombs test
0% Fibrogastroscopy
0% Lumbar puncture

2285. a 43-year-old woman complains of pain in the wrist,ankle,

https://translate.yandex.com/en/doc 506/540
22:20 ,27.6.2023 �� C : ; 5 B
and interphalangeal joints of the hands and feet, and stiffness of movement in the morning. He has been ill for 7 years.
The disease began with a symmetrical lesion of the small joints of the hands and feet.
There is a deformity of the feet, their movements are limited and sharply painful. Internal organs without
special features. What is the most formative indicator for making a diagnosis?
100% Rheumatoid factor
0% Leukocytosis
0% Increased erythrocyte sedimentation rate (ESR)
0% Dysproteinemia
0% C-reactive protein

2286. you had to come to the aid of a 58-year-old woman who was bleeding profusely from
a ruptured varicose vein in her left lower leg. What is first
aid?
100% Elevated position of the limb. Compressive sterile dressing
0% Applying a tourniquet distally to the source of bleeding
0% Applying a tourniquet proximal to the source of bleeding
0% Troyanov-Trendelenburg Operation
0% Z-shaped suture on a torn varicose node

2287. a 38-year-old man fell ill 2 weeks ago, developed a cough, weakness,
and fever of up to 38.0°C. The condition worsened sharply by the end of the 1st week, when there were cold spells,
torrential sweats, and in the evening the temperature rose to 39.0°C. 2 days before hospitalization
, a large amount of smelly sputum with blood was released during coughing, after
which the patient's condition improved. Pulse -80 / min., respiratory rate-20 / min., body temperature -37.6
°C. What changes are possible on the chest X-ray?
100% Presence of a cavity with a horizontal liquid level

527

Downloaded from the site - online testing step

0% Uniform rounded shadow in the pulmonary field


0% Displacement of the mediastinum towards a homogeneous shadow
0% Shadow in the lower section with an oblique upper border
0% Darkening of the lung lobe

2288. A 39-year-old man who was treated in the surgical department for
acute cholecystitis was transferred to the therapeutic department for increased
blood pressure up to 180/120 mm Hg, protein in the urine, and persistent fever. A stable
subfebrile patient was resistant to antibiotic therapy. 10 days after treatment, the
patient experienced an attack of suffocation with a heavy exhalation, subsequently arthralgia and
erythematous skin damage appeared. In the blood test-18% of eosinophils. What disease did the patient have?
100% Nodular periarteritis
0% Non-specific aortoarteritis
0% Acute glomerulonephritis
0% Systemic lupus erythematosus
0% Infection with worms

2289. a 37-year-old patient complains of constant dull pain in the hypochondria radiating to the
back, which increases after eating. Worries about bloating, frequent bowel movements with impurities
of undigested food. She has been ill for more than 5 years, lost 15 kg of weight. Objectively: moderate bloating,
pain in the Shoff-ra zone, Desjardins points, Mayo-Robson points. Which of the research methods

https://translate.yandex.com/en/doc 507/540
22:20 ,27.6.2023 �� C : ; 5 B
will be the most informative for confirming
external
100% endocrine pancreatic insufficiency?
Coprogram
0% Oral cholecystography
0% Retrograde pancreatography
0% EFGDS
0% Ultrasound examination of the abdominal organs

2290. the patient injects insulin into the lateral surfaces of the shoulders, abdomen, and thighs. It
feels most comfortable when the drug is injected into the stomach. How can this be predetermined?
100% The rate of insulin absorption
0% The convenience of injecting insulin into the stomach
0% Fewer nerve receptors
0% Autosuggestion
0% Availability of injections

2291. A 20-year-old patient underwent polychromyotherapy under the VAMP scheme for acute
lymphoblastic leukemia. What morphological picture of the bone marrow can indicate the
onset of remission?
100% Blast cell count up to 5%
0% Blast cell count up to 15 %
0% Blast cell count up to 10%
0% Blast cell count up to 1%

528

Downloaded from the site - online testing step

0% Absence of blast cells

2292. A 50-year-old patient complains of an attack of pain in the right hypochondrium, vomiting with
bile admixtures, and has been bothered by epigastric pain, nausea, and stool disorders for 5 years.
Objectively: the heart rate is 92 / min. Increased fullness, overlaid tongue,
icteric sclera. The abdomen is soft, painful in the projection of the gallbladder, local muscle
tension in the right hypochondrium, a positive Murphy's symptom. In a general blood test:
leukocytes - 9.6109/l, erythrocyte sedimentation rate-14 mm / h. What kind of research should
be prescribed to confirm the diagnosis in this case?
100% Ultrasound examination of the gallbladder
0% Liver scintigraphy
0% Bacteriological examination of bile
0% Cholecystography
0% Retrograde cholangiopancreatography

2293. A 36-year-old woman complains of pain, restricted movement in the small joints of her hands,
difficulty swallowing solid food, weakness, and a dry cough. Objectively: the skin of the hands and
forearms is dense and smooth. The proximal joints of the fingers II-IV of the hands are swollen and painful
on palpation. Above the lungs there are dry scattered wheezes, the borders of the heart are shifted to the left by 2 cm, the tones
are muted. In the blood: erythrocyte sedimentation rate - 36 mm / h, y-globulins -24%. In the urine:
no changes. What is the most likely diagnosis?
100% Systemic scleroderma
0% Systemic lupus erythematosus
0% Dermatomyositis

https://translate.yandex.com/en/doc 508/540
22:20 ,27.6.2023 �� C : ; 5 B
0% Rheumatoid arthritis
0% Sarcoidosis

2294. A 49-year-old woman complains of pain in the shoulder joints that increases during movement,
limited mobility, and short-term morning stiffness. He's been ill for several years. In the past
, she was engaged in gymnastics. The X-ray of the shoulder joints shows narrowing of the joint
gap, subchondral osteosclerosis, and osteophytes in the lower eastern part
of the humerus head. What is the basis of the pathogenesis of joint damage in a patient?
100% Violation of cartilage metabolism
0% Deposits in the synovial membrane of immune complexes
0% Violation of uric acid metabolism
0% Calcium pyrophosphate deposits in the joints
0% Synovial membrane damage caused by an infectious agent

2295. A 56-year-old woman was admitted to the clinic with complaints of pain and swelling of the right
knee joint, which appeared a week ago. In the anamnesis: frequent colds,
chronic cholecystopancreatitis. Objectively: on the tilno-lateral surfaces of the distal
interphalangeal joints there are dense nodules of small size, slightly painful on palpation,
deformity of the first metatarsophalangeal joints of both feet is observed, the right knee
joint is deformed, movements in it are limited. What is the most likely diagnosis?
100% Osteoarthritis

529

Downloaded from the site - online testing step

0% Rheumatic polyarthritis
0% Rheumatoid arthritis
0% Gout
0% Reactive arthritis

2296. a 38-year-old man was admitted to the hospital in an unconscious state. I got sick yesterday:
headache, nausea, vomiting, body temperature - 38.5°C, dizziness, delirium.
During the last 4 days, he complained of pain and hearing loss in his left ear. Objectively: the condition
is soporotic, rigidity of the occipital muscles, Kernig's symptom on both sides, general hypersthesia,
suppuration from the left ear. Which of these diagnoses is most likely?
100% Secondary purulent meningitis
0% Primary purulent meningitis
0% Tuberculosis meningitis
0% Subarachnoid hemorrhage
0% Parenchymal-subarachnoid hemorrhage

2297. a 74-year-old man complains of periodic dizziness, speech disorders, loss


of memory for current events, slow gait. He has been ill for about 5 years. Objectively: dysarthric speech,
poor facial expressions, stooped posture. Tendon reflexes S > D, Babinsky's symptom bilaterally,
symptoms of oral automatism, muscle tone increased by plastic type. In
the Romberg position, the nystagmus is unstable and horizontal. What is the most likely diagnosis?
100% Dyscirculatory encephalopathy
0% Parkinson's disease
0% Meningoencephalitis
0% Encephalomyelitis
0% Brain edema
https://translate.yandex.com/en/doc 509/540
22:20 ,27.6.2023 �� C : ; 5 B

2298. a 70-year-old man has coronary heart disease. The mood is noticeably low,
anxious. Against the background of prolonged insomnia, there were fears, unwillingness to live, thoughts
of suicide. He sits in the same position for a long time, does not answer immediately, quietly, in a monotonous voice.
The expression on his face is one of suffering, pain, and fear. What is the leading psychopathological syndrome?
100% Depressive syndrome
0% Paranoid syndrome
0% Asthenic syndrome
0% Phobic syndrome
0% Obsessive-compulsive disorder

2299. A 28-year-old woman has been suffering from chronic glomerulonephritis for 12 years, which
has been latent all this time. Over the past six months, there has been a general weakness, decreased
appetite, working capacity, nausea. The patient complains of the main 6il, 6il in the joints. On
examination: anemia, increased blood urea level to 34.5 mmol / l, blood creatinine-0.766
mmol/l, hyperkalemia. What has complicated the course of the disease?
100% Chronic renal failure
0% Acute renal failure
0% Nephrotic syndrome

530

Downloaded from the site - online testing step

0% Amyloidosis of the kidneys


0% Pyelonephritis

2300. A patient with duodenal ulcer disease complained of weakness,


shortness of breath with a slight load, and a desire to eat chalk. Objectively: pale, trophic
skin changes. In the blood: Hb-82 g / l; red blood cells-3, 3 * 1012/l; CP-0.75, reticulocytes-0.7%,
blood iron-5.6 mmol / l. what complication did the patient develop?
100% Iron deficiency anemia
0% Hemolytic anemia
0% Erythremia
0% Aplastic anemia
0% B12-folate deficiency anemia

2301. a 1.5-year-old child was born with a body weight of 3100 g and a length of 520 cm. I was breastfed.
After the introduction of complementary foods (oatmeal porridge), I stopped gaining weight, and bowel movements with
an unpleasant smell appeared in large quantities. Objectively: signs of hypotrophy of the second century, pallor of the skin,
large belly. What is the most likely diagnosis?
100% Celiac disease
0% Simple dyspepsia
0% Cystic fibrosis
0% Intestinal infection
0% Infection with worms

2302. a 36-year-old female patient became acutely ill. The disease began with a high body temperature,
cold, profuse sweat. There was a dull pain in the lumbar region, unpleasant sensations when
urinating. Objectively: tension of the lumbar muscles, a positive symptom
of tapping on both sides. In the blood: white blood cells up to 12 * 109/l, neutrophilosis. In the urine: protein-0.6
g / l, white blood cells for all p/S, bacteria-more than 100,000 in 1 ml of urine. What will be the preliminary diagnosis?
https://translate.yandex.com/en/doc 510/540
22:20 ,27.6.2023 �� C : ; 5 B

100% Acute pyelonephritis


0% Tuberculosis of the kidney
0% Acute glomerulonephritis
0% Acute cystitis
0% Urolithiasis

2303. A 46-year-old patient first noticed swelling in the legs, weakness, a feeling of "fullness” and
heaviness in the right hypochondrium. He has been suffering from rheumatoid arthritis for 20 years. Enlarged liver and
spleen, dense consistency. Blood creatinine - 0.23 mmol / l, proteinemia-53 g / l,
cholesterol-4.2 mmol/l, urine specific gravity-1012, proteinuria-3.3 g/l, single waxy
cylinders, red blood cells are leached in p / s, white blood cells-5-6 in p / s. what is the most
likely complication?
100% Amyloidosis of the kidneys
0% Chronic glomerulonephritis
0% Acute glomerulonephritis
0% Heart failure
0% Chronic pyelonephritis

531

Downloaded from the site - online testing step

2304. The 26-year-old athlete fell with an emphasis on the right shoulder joint. - Sharp pain,
restriction of passive movements, active movements in the shoulder joint are impossible. The injured arm that is withdrawn and bent at
the shoulder joint is supported by a healthy one. Objectively: there is a depression in the deltoid
muscle, the acromial process of the scapula protrudes sharply under the skin, the head
of the humerus is palpated under the coracoid process. What is the most likely
clinical diagnosis?
100% Dislocated shoulder
0% Fracture of the anatomical neck of the shoulder
0% Surgical shoulder neck fracture
0% Fracture of the head of the humerus
0% Large humerus hill Detachmentedit

2305. A 30-year-old patient has been working as a riveter for 6 years. Complaints of sharp pain in
the shoulder girdle, especially at night. Fingers go numb and turn white when cooled. Became
irritable. Objectively: the hands are swollen, cold, cyanotic. All types of sensitivity
(pain, temperature, vibration) are sharply reduced. When tested for cold-a symptom of "dead fingers".'
There is weakness in the adductor muscles of the V finger; altered electro-excitation.
Tendon and periosteal reflexes are alive. What is the preliminary diagnosis?
100% Vibration disease of local action
0% Autonomic-sensory polyneuropathy
0% Raynaud's syndrome
0% Nodular periarteritis
0% General vibration disease

2306. a 31-year-old man has been treated by a psychiatrist for many years. Against the background of prolonged insomnia
, there were fears, thoughts of suicide, tried to hang himself. The mood is noticeably reduced,
refuses treatment. What are the most recommended actions for suicide prevention
?
100% Hospitalization in a mental hospital
0% Hospitalization in the neurological department

https://translate.yandex.com/en/doc 511/540
22:20 ,27.6.2023 �� C : ; 5 B
0% Outpatient treatment
0% Psychotherapy conversation
0% Strict supervision at home

2307. A 7-month-old child was taken to the surgical department 8 hours after
the disease with complaints of anxiety attacks, abdominal pain, and a single vomiting.
Objectively: a tumor-like formation is palpated in the right half of the abdomen. During
rectal examination-blood in the form of "raspberry jelly" what disease can
be assumed?
100% Invagination
0% Doubling the bowel
0% Abdominal tumor
0% Infection with worms
0% Enterocystoma

532

Downloaded from the site - online testing step

2308. a 25-year-old patient has no menstruation for 6 months. Menstruation from the age of 16 for
3-5 days with delays per month, moderate, painful. The woman is overweight,hirsutism,
infertility. During bimanual examination, the uterine body is of normal size. On both sides of
the uterus, enlarged ovaries up to 4 cm in diameter,dense consistency,
painless, mobile when shifted are palpated. What will be the diagnosis?
100% Polycystic ovary disease
0% Chronic bilateral adnexitis
0% Galactorrhea and amenorrhea syndrome
0% Sheehan's syndrome
0% Dermoid ovarian cysts

2309. a 3-month-old girl has diarrhea and hypotrophy. Born with a weight of 3300 g. From 2 months
she was transferred to artificial feeding with cow's milk. The skin is dry, the muscle tone
is reduced, the bowel movements are liquid, greenish, smelly. There is a lot of neutral fat in the coprogram.
Sweat chlorides - 70 meq / l. what is the most likely disease?
100% Cystic fibrosis
0% Cow's milk protein intolerance
0% Intestinal infection
0% Disaccharide deficiency
0% Celiac disease

2310. The following indicators were used to assess the population health of children
: disease prevalence, primary morbidity, morbidity structure,morbidity
and disability. Which of these indicators reflects the level
of morbidity of children only with chronic diseases and morphofunctional disorders?
100% Pathological lesion
0% General morbidity rate
0% Prevalence rate
0% Primary morbidity
0% Disability status

2311. A 13-year-old child has a combined mitral valve defect with a predominance
https://translate.yandex.com/en/doc 512/540
22:20 ,27.6.2023 �� C : ; 5 B
of insufficiency. Against the background of acute respiratory viral infection (ARVI), shortness of breath and
general weakness increased, a feeling of tightness in the chest, and a dry cough appeared. In the lungs
, various wet wheezes are heard. Liver at the edge of the costal arch. What
emergency condition did the child develop?
100% Acute left ventricular heart failure
0% Acute right ventricular heart failure
0% Acute complete heart failure
0% Chronic left ventricular heart failure
0% Acute vascular insufficiency

2312. 50 patients were treated in the therapeutic department of the hospital, 40 of them
are working. Before being discharged from the hospital, employees were given ID cards.
Who has the right to sign disability certificates?

533

Downloaded from the site - online testing step

100% Curator doctor, Head of the department


0% Head of the Department, Deputy Chief Physician for Medical Work
0% Chief Medical Officer, Curator
0% Chief Physician, Head of the Department
0% -

2313.accidents, poisoning and


injuries, malignant neoplasms, diseases of the circulatory system, endocrine, respiratory organs and the like predominate among the
causes of death of the population of Ukraine
. Name
100% the first-place disease
Diseases of theclass:
circulatory system
0% Accidents, poisonings and injuries
0% Respiratory diseases
0% Endocrine diseases
0% Malignant neoplasms

2314. a 3-year-old child has been ill for 3 days. Fever, cough,runny nose,
conjunctivitis, photophobia, whitish mottled rash on the mucous membrane of the gums and
the inner surface of the cheeks, and mottled enanthema of the hard and soft palate are observed. What
disease can be diagnosed?
100% Measles
0% Acute respiratory viral infection
0% Herpes infection
0% Enterovirus infection
0% Infectious mononucleosis

2315. a 10-year-old child complains of pain and swelling of the knee and ankle joints and
pain in the cervical spine. Morning stiffness. The prescribed anti-inflammatory therapy gave
a positive therapeutic effect. What disease can be assumed?
100% Juvenile rheumatoid arthritis
0% Osteochondrosis
0% Rheumatism
0% Reiter's Syndrome
0% Infectious arthritis

https://translate.yandex.com/en/doc 513/540
22:20 ,27.6.2023 �� C : ; 5 B

2316. the child is 5 years old. 2 weeks ago I had a sore throat. Mother's complaints of poor appetite,
disturbed sleep. An objective study revealed a heart rate of -100 / min.
How should this indicator be evaluated?
100% Age limit
0% Tachycardia
0% Bradycardia
0% Respiratory arrhythmia
0% Paroxysmal tachycardia

2317. A 36-year-old female patient came to the antenatal clinic complaining of high blood pressure.

534

Downloaded from the site - online testing step

irritability, tearfulness, headache, palpitations, swelling of the hands and feet, decreased
urination, engorgement of the mammary glands. These symptoms appear and gradually increase a
few days before menstruation and disappear with its onset. Menstrual cycle without violations.
These complaints have appeared over the past year. Make a diagnosis:
100% Premenstrual syndrome
0% Sheehan's syndrome
0% Stein-Leventhal syndrome
0% Adrenogenital syndrome
0% Menopausal syndrome

2318. A 67 - year-old patient complains of shortness of breath, chest pain, and general weakness. He's been ill for 5
months. Objectively: body temperature-37.3 ° C, pulse - 96/min. Above the right lung, the vocal
tremor is not detected, the percussion sound is dull, and breathing is not listened to. In
sputum-an admixture of blood, diffusely mixed with mucus. What is the most likely diagnosis?
100% Lung cancer
0% Bolshegnishchevaya pneumonia
0% Bronchiectasis
0% Focal tuberculosis of the lungs
0% Exudative pleurisy

2319. a 48-year-old patient was hospitalized with seizures. He has been suffering
from duodenal ulcer (duodenal ulcer) for many years. During the last month, he has been observing daily vomiting, lost
20 kg of weight. Objectively: the patient is exhausted. In the epigastric region, "splashing noise " is detected'
The lower border of the stomach is at the level of the Scallop line. Laboratory: zagalny biloxin-47 g / l; K-2.1
mmol/l, Na-118 mmol/l, Ca-1.6 mmol/l, chloride-82 mmol/l, hematocrit-64%. What
is the preliminary diagnosis?
100% Decompensated pyloric stenosis
0% Bleeding duodenal ulcer of the 12th duodenum
0% Subcompensated pyloric stenosis
0% Malignated stomach ulcer
0% Compensated pyloric stenosis

2320. a 34-week-old pregnant woman was taken to the maternity hospital due to bleeding from
the genital tract that appeared during sleep. The pain doesn't bother me. The bleeding is about 300.0 ml.
Make a diagnosis:
100% Placenta previa

https://translate.yandex.com/en/doc 514/540
22:20 ,27.6.2023 �� C : ; 5 B
0% Premature detachment of the normally located placenta
0% Cervical cancer
0% Bleeding erosion
0% Rupture of the umbilical cord vessels

2321. A 60-year-old patient complains of frequent painful difficulty urinating,


intermittent urine flow. At night, urinate up to 5 times. After physical exertion
, it notes blood impurities in the urine. Rectally, the prostate is enlarged, tightly elastic, painless,
without nodes. In the urine: specific gravity -1020, protein-0.1 g / l, white blood cells-20-30 v / s, red blood cells-10-20 v

535

Downloaded from the site - online testing step

P / S. What is the priority survey method?


100% Transrectal ultrasound examination
0% Cystoscopy
0% Overview of urography
0% Excretory urography
0% Bacteriological examination of urine

2322. the labor activity of a worker requires concentration of attention and emotional tension.
There are zoro - and auditory-motor reactions, a high density of working hours. This work should
be evaluated in terms of:
100% Nervous tension
0% Physical severity
0% Amount of energy consumed
0% Oxygen consumption
0% Static load

2323. A 48-year-old patient was admitted to the surgical department with hip wounds. On examination
, it was found that the surface of the wound is covered with a dirty gray coating with an unpleasant sweet
smell. The wound's contents resemble raspberry jelly. The skin around the wound is tense
and shiny. Moderate crepitation is observed during palpation of the tissues. What microflora
is most likely to cause inflammation?
100% Anaerobic clostridial
0% Anaerobic non-clostridial
0% Streptococci
0% Staphylococci
0% Pseudomonas aeruginosa

2324. To serve the population, the city polyclinic has 30 positions of district doctors. In addition
, 3 outpatient clinics of family doctors
(general practitioners) are organized on the territory of the polyclinic service. What type of medical and preventive
care is provided by these doctors?
100% Primary
0% The first one
0% Secondary
0% Tertiary
0% -

https://translate.yandex.com/en/doc 515/540
22:20 ,27.6.2023 �� C : ; 5 B
2325. To serve the population, the district polyclinic of the Central District Hospital has 25 positions of specialist doctors.
What type of medical and preventive care is provided by these doctors?
100% Secondary
0% Primary
0% The first one
0% Tertiary
0% -

536

Downloaded from the site - online testing step

2326. a 53-year-old patient complains of aching pain in the lower abdomen, a significant increase in it over
the past 5 months, weight loss, weakness. Gynecological examination revealed that
the cervix is clean, the uterus is not enlarged, not painful, and sedentary. On both sides, tumors
with a size of 10x13 cm, with an uneven surface, dense consistency, and immobile are determined. When percussion of the abdomen
is observed fluctuation. What will be the preliminary diagnosis?
100% Ovarian cancer
0% Uterine fibromyoma
0% Tubo-ovarian tumor
0% The wandering kidney
0% Endometriosis

2327. an ambulance delivered a woman with


cramping pain in the right iliac region, radiating to the rectum,
and bloody discharge from the genital tract that occurred after a delay in menstruation. Objectively: heart rate -100
/ min, blood pressure-90/60 mm Hg. the skin is pale. The abdomen is painful on palpation,
a positive Shchetkin-Blumberg symptom. Gynecological examination
shows painful cervical dislocations, enlarged right appendages, painful posterior arch overhanging, bloody discharge.
Make a preliminary diagnosis:
100% Ectopic aborted pregnancy
0% Acute right-sided adnexitis
0% Apoplexy of the right ovary
0% Appendicitis
0% Abortion is in progress

2328. after the birth of a child weighing 4500 g, the litter peeled off and separated on its own.
The birth canal is not damaged. The condition of the woman in labor is satisfactory, pulse-92 / min., rhythmic.
Blood pressure - 100/60 mm Hg. The uterus is contracting poorly, bleeding has begun.
External uterine massage was performed, uterotonic drugs were administered. Blood loss has reached 350 ml
and is being followed. What will be the doctor's tactics?
100% Manual examination of the uterus and massage of the uterus on the fist
0% Uterine extirpation
0% Clamps for Schenkel-Tikanadze parameters
0% Uterine tamponade
0% Superpichival uterine amputation

2329. A 38-year-old patient complains of purulent discharge from the left side of the nose, difficulty
in nasal breathing, headache, heaviness in the left cheek area, and an increase in body temperature up
to 37.5 ° C. Gets sick for 6 days. The disease is associated with hypothermia. Objectively:
soreness on palpation in the area of the left cheek. The mucous membrane of the left half of the nasal
cavity is hyperemic, edematous, and there is a purulent streak in the middle nasal passage. What
is the most likely diagnosis?

https://translate.yandex.com/en/doc 516/540
22:20 ,27.6.2023 �� C : ; 5 B
100% Left-sided acute sinusitis
0% Left-sided acute frontitis
0% Left-sided acute etmoiditis
0% Acute rhinitis (runny nose)
0% Curvature of the nasal septum to the left

537

Downloaded from the site - online testing step

2330. a 22-year-old patient complains of fever up to 39oc, pain in the lower abdomen,
bloody-purulent discharge from the genital tract. From the medical history: 5 days ago, an artificial
abortion was performed at 8 weeks of pregnancy. During bi-manual examination: the uterus is enlarged, soft, painful.
The apps on both sides are unchanged. What will be the diagnosis?
100% Acute metroendometritis
0% Spilled peritonitis
0% Pelvioperitonitis
0% Acute adnexitis
0% Uterine perforation

2331. A 46-year-old patient called an ambulance in the middle of the night due to sudden, sharp
pain, redness and swelling of the first toe of the right foot, and an increase in body temperature.
The day before, I consumed dry grape wine and fatty meat. I've never had anything like this before.
Make a preliminary diagnosis:
100% Gouty arthritis
0% Rheumatoid arthritis
0% Rheumatic arthritis
0% Reactive arthritis
0% Deforming osteoarthritis

2332. A 50-year-old woman complains of high blood pressure, headache, and nausea.
From the anamnesis, it is known that he has been ill with COPD for 15 years. Objectively: redness of the face,
blood pressure -170 / 120 mm Hg. heart tones are increased. Pulse-76 / min., rhythmic. In the lungs
, against the background of a hard shade of vesicular respiration, a large number of dry, scattered wheezes. Which
blood pressure monitor will be most indicated?
100% Amlodipine
0% Propranolol
0% Furosemide
0% Clonidine
0% Captopril

2333. a 50-year-old patient has been abusing alcohol for 15 years. After the last binge
, he does not drink alcohol for 5 days. He hides under the bed, tries to resist
others, sees them as devils and aliens, believes that he is in hell, correctly calls his
last name and first name, patronymic. What will be the diagnosis in this case?
100% Alcoholic delirium
0% Alcoholic hallucinosis
0% Oneiroid catatonia
0% Twilight confusion of consciousness
0% Pathological intoxication

2334. a 16-year-old boy with increased greasiness of the skin of the face, upper back,
https://translate.yandex.com/en/doc 517/540
22:20 ,27.6.2023 �� C : ; 5 B
and anterior surface of the chest has multiple comedones,follicular nodules,
and pustules periodically appear. There are no subjective feelings. What is the most likely diagnosis?
100% Common acne

538

Downloaded from the site - online testing step

0% Secondary syphilis
0% Vulgar sycosis
0% Furunculosis
0% Rosacea

2335. a postoperative patient who underwent removal of the lower lobe


of the left lung 6 days ago due to bacterial destruction and the development of pneumothorax developed
shortness of breath up to 48 / min, PaO2 decreased to 50 mm Hg, PaCO2 increased to 65 mm Hg,
cough, sputum production, and general cyanosis hypotension-60/20 mm
Hg during auscultation of the right and left lungs - a lot of different-sized wheezing, sharply weakened
breathing. How much emergency care should be provided to the patient in the first place to
eliminate respiratory failure?
100% Perform tracheal intubation with mechanical ventilation
0% Perform lavage of the tracheobronchial tree
0% Oxygen therapy
0% Orthopic position
0% Puncture of the pleural cavity

2336. a 32-year-old woman was taken to the emergency department by ambulance. On a


hot day, she worked for a long time in the field, collecting vegetables. I felt a headache,
dizziness, general weakness, dry mouth, but continued to work. Soon
"darkened in the eyes", vomiting, sharp pain in the calf muscles. Then there was motor
arousal, loss of consciousness. Blood pressure-170/95 mm Hg. Temperature-37.8°C.
Formulate the most likely diagnosis:
100% Heat stroke
0% Hyperthermic syndrome
0% Hypertensive crisis
0% Fainting
0% Neurotoxicosis

2337. A 39-year-old patient complains of shortness of breath and constrictive pain behind the sternum at
rest. I had the flu 10 days ago. Objectively: forced pose-sitting, torso tilted
forward, face swollen, cyanotic, swollen neck veins. The boundaries of the heart are significantly expanded in both
directions, the tones are muffled, rhythmic, the heart rate is 104 / min., the respiratory rate is 28
/ min. On the ECG: reduced voltage of the teeth, concordant changes in the ST segment. On the radiograph:
bullet-shaped shadow of the heart, signs of congestion in the lungs. In the blood: the erythrocyte sedimentation rate is 38
mm / hour. Make a diagnosis:
100% Exudative pericarditis
0% Viral myocarditis
0% Rheumatic fever
0% Dilated cardiomyopathy
0% Unstable angina pectoris

2338. a 50-year-old patient, as a result of a sharp flexion of the cervical spine during

https://translate.yandex.com/en/doc 518/540
22:20 ,27.6.2023 �� C : ; 5 B
a traffic accident, felt pain in the neck radiating to the lateral surface of the right ear.

539

Downloaded from the site - online testing step

shoulder area. During the examination, a zone of hypesthesia was established on the radial surface of the right
forearm, in the area of the index and middle fingers, loss of the extensor-ulnar
reflex. Specify the most likely diagnosis:
100% Traumatic sciatica
0% Spinal cord contusions
0% Hematomyelia
0% Compression of the spinal cord by an epidural hematoma
0% Compression of the spinal cord by vertebral fragments

2339. the patient is 54 years old. During the year, he is treated for an undifferentiated form
of lung cancer. Now there is a feeling of heaviness and soreness in the right hypochondrium. which
diagnostic method will be most informative in this case?
100% Computed tomography of the abdominal organs
0% Radioisotope hepatography
0% Liver vasography
0% Diagnostic laparoscopy
0% Ultrasound biolocation with targeted puncture biopsy

2340. a 22-year-old patient complains of frequent and painful urination, calls for urine-starting
at night, urinary incontinence, pain in the suprapubic region and in the lower back. Often the urine is the color of beer.
I got married a month ago. Objectively: the general condition is satisfactory. In the lungs-vesicular
respiration. Heart sounds are rhythmic, heart rate-78 / min, blood pressure-128/68
mm Hg. the abdomen is soft, painful in the suprapubic region. In the urine: red blood cells-12-18 in the field of vision,
white blood cells-12-15, bacteria +++ in the field of vision. What is the most likely diagnosis?
100% Lower urinary tract infection-cystitis
0% Urolithiasis
0% Upper urinary tract infection-pyelonephritis
0% Gonorrhea
0% Primary syphilis

2341. on the seashore you saw a young man lying on the ground. Gooseflesh skin, sharply
cyanotic, white foam on the lips. The pulse is frequent and arrhythmic, and breathing is not audible in the lungs.
What will be the first actions at the pre-hospital stage?
100% Remove mucus and foam from the mouth and oropharynx, perform artificial respiration
0% Leave it alone, call an ambulance
0% Rub and warm the patient
0% Put your feet in a raised position
0% Make a sharp blow to the sternum

2342. A 34-year-old patient is being treated in a psychiatric hospital for acute


schizophrenia. Objectively: he is in bed, mobile inhibited, there is no contact.
Doesn't answer the question. The pose is monotonous, the patient is hypomimic, there is a symptom of "proboscis "'
waxy flexibility of the muscles, a symptom of "air cushion "' in this state remains for about
a week. Nutrition is parenteral. Identify an existing motor-volitional disorder syndrome:
100% Catatonic stupor

540
https://translate.yandex.com/en/doc 519/540
22:20 ,27.6.2023 �� C : ; 5 B

Downloaded from the site - online testing step

0% Depressive stupor
0% Psychogenic stupor
0% Apathetic stupor
0% Exogenous stupor

2343. a 32-year-old female patient complains of a rise in body temperature up to 39 ° C with cooling, pain in
the lumbar region on the right, dysuria during the day. On the isotope renogram
, the obstructive curve type is shown on the right. What should be the primary actions of the doctor?
100% Restoring the outflow of urine from the right kidney
0% Antibacterial therapy
0% Detoxification therapy
0% Physical therapy for the lumbar region
0% Introduction of diuretics

2344. a 38-year-old coal miner with 15 years of experience complains of coughing with
dark-colored mucosal sputum, suffocation during physical exertion, chest pain during
breathing. Auscultation: hard breathing, dry and wet wheezing. On the radiograph:
the broncho-vascular pattern is reinforced, deformed, single focal shadows up to
1-3 mm in diameter in the middle and lower parts of the lungs, the roots are expanded, compacted. The function of external
respiration is impaired by the obstructive type. Make a preliminary diagnosis:
100% Anthracosis
0% Bronchopneumonia
0% Chronic obstructive pulmonary disease
0% Tuberculosis
0% Talcosis

2345. a 35-year-old patient complains of pain in the upper third of the shoulder, which increases at night.
On examination: moderate swelling in the upper third of the shoulder, skin over it with elevated temperature,
pain on palpation, limited movement in the shoulder joint. On radiographs: the site
of destruction of the humerus in the metadiaphyseal region with the phenomena of tibial periostitis
(spicula) and detachment of the periosteum in the form of "dasha’' establish a preliminary diagnosis:
100% Osteogenic sarcoma
0% Osteoma
0% Hemangioma
0% Chondroblastoma
0% Chondroma

2346. A 53 - year-old woman complains for 2-3 months of constant aching pain in the stomach,
feels the urge to urinate after physical overload. There are no visual changes to the Sich
. So far, there have been no kidney diseases. When performing ultrasound
examination of the abdominal organs, the following data were obtained: at the level of IV-V
lumbar vertebrae, an S-shaped formation of an inhomogeneous structure,measuring 14x12. 5
cm, was found. Preliminary diagnosis: congenital malformation (S-shaped kidney). Which diagnostic method is
the most informative to confirm the diagnosis?
100% Excretory urography

541

https://translate.yandex.com/en/doc 520/540
22:20 ,27.6.2023 �� C : ; 5 B

Downloaded from the site - online testing step

0% Overview radiograph of the kidneys


0% Radionuclide renography
0% Angiography
0% Thermography

2347. A 60-year-old woman complains of periodic pain in the knee joints, which occurs
more often during long walks, descending stairs, in the evening, worries in the first half of the night and
subsides in the morning after a long rest. The examination revealed an overweight body.
The joints are not externally changed, the amount of movement in them is not limited. An X-ray of the right
knee joint shows osteophytes. To prevent further progression
of the disease, you should recommend:
100% Avoid excessive loads on the knee joints
0% Limit your diet to foods rich in purines
0% Daily jogging
0% Regular intake of allopurinol
0% Short-term immobilization of the long-leg joint

2348. A 24-year-old repeat patient with Rh-negative blood type is under the supervision
of a perinatologist. In the anamnesis: in previous births, manual placental separation was performed for
bleeding in the third period. At 36 weeks of pregnancy, the antibody titer increased from 1: 16 to
1: 64. Ultrasound revealed thickening of the placenta and slowing of fetal movements. How often
should blood tests for Rh antibodies be performed in the future?
100% Daily until delivery
0% 1 time per week
0% 1 every two weeks
0% 1 every three weeks
0% Before giving birth

2349. the patient has an alienation of mental functions, his ”I”: "I have become somehow
different, I feel it, but I can not describe it”" this is both me and not me”’ my thoughts are in a fog, they are not mine "
"I hear my language somewhere to the side’" everything my senses are gone." Identify the psychopathological syndrome:
100% Depersonalization
0% Derealizations
0% Depressive
0% Paranoid
0% Hypochiondrial

2350. A 25-year-old woman was prescribed a 10-day


complex of antibiotic therapy to treat acute pyelonephritis. After the end of taking antibiotics, thick white
vaginal discharge of a curd-like nature appeared, which was accompanied by hyperemia
of the external genitalia, pronounced itching of the vulva. For the described type of vaginitis, choose a rational
therapy:
100% Antifungal medications
0% Douching with boric acid solution
0% Estrogen-containing creams

542

Downloaded from the site - online testing step

https://translate.yandex.com/en/doc 521/540
22:20 ,27.6.2023 �� C : ; 5 B

0% Vaginal suppositories with sulfonamides


0% Ceftriaxone

2351. a patient complains of purulent discharge from the penis in the morning before urination,
resis,and pollakiuria. Pyuria was determined during a three-cup test in the first portion of urine.
Make a diagnosis:
100% Urethritis
0% Cystitis
0% Pyelonephritis
0% Prostatitis
0% Balanoposthitis

2352. A 13-year-old girl


was diagnosed with primary idiopathic dermatomyositis, primary chronic course, minimal activity, FN 1 based on clinical, biochemical,
and EMG changes and the result of a muscle biopsy
.: 100% Hormone Therapy
0% Cytostatic therapy
0% Nonsteroidal anti-inflammatory drugs
0% Massage, physical therapy
0% Balneotherapy

2353. a 54-year-old patient complains of chilliness, drowsiness, and weakness. In the past
, she was treated for an autoimmune nodular goiter. During the year, her condition worsened: she became
weak, gained weight, and became slow. The thyroid gland is -1 B. The skin is cold, dry, pale,
and the tongue is thickened. His voice is hoarse. Heart sounds are rhythmic, muffled, heart rate -56
/ min., stomach is swollen. Bowel movements-constipation. What medication should be prescribed to
prevent significant thyroid enlargement?
100% Thyroxine
0% Mercazolil
0% Prednisone
0% Radioactive iodine
0% Methylthiouracil

2354. a 6 - year-old child with anemic and hemorrhagic syndromes. In the blood: HB-80 g / l,
CP-0.9, reticulocytes-2o/oo, leukocytes-1.0109/l, platelets-10109 / L. what
is the most likely diagnosis?
100% Aplastic anemia
0% Thrombocytopenic purpura
0% Thrombocytopathy
0% Deficiency anemia
0% Lymphoblastic leukemia

2355. a 60-year-old patient complains of an almost constant feeling of heaviness and


fullness in the epigastrium, which increases after eating, belching with a rotten smell, sometimes

543

Downloaded from the site - online testing step

vomiting of food eaten 1-2 days ago, weight loss. 12 years ago
, a pyloric canal ulcer was first discovered. I observed periodic "hungry" pain, for which I took
https://translate.yandex.com/en/doc 522/540
22:20 ,27.6.2023 �� C : ; 5 B
Ranitidine. Deterioration within 3 months. Objectively: the "splashing noise" in
the epigastrium is determined. What complication are we talking about?
100% Pyloric stenosis
0% Penetration of a stomach ulcer
0% Functional spasm of the goalkeeper
0% Foreign body of the stomach (bezoar)
0% Malignancy of a stomach ulcer

2356. A 25-year-old patient was admitted 2 hours after a traffic accident with
complaints of persistent intense pain in the right groin and pelvic symphysis,
and inability to move independently. On examination: the patient's position is forced-
the "frog" pose (a positive symptom of Volkovich). Abrasions and bruises on the skin in the projection of the symphysis and
horizontal branch of the pubic bone on the right, pronounced soft tissue edema. Postural
symptom of "stuck heel" on the right. Make a preliminary diagnosis:
100% Closed fracture of the pubic bone on the right side
0% Pelvic bone contusion
0% Traumatic inguinal hernia
0% Traumatic femoral artery aneurysm
0% Dislocation of the right hip

2357. a patient after undergoing Q-infarction of the myocardium is concerned about shortness of breath with minor
loads, night attacks of dry cough, a feeling of wheezing in the chest and orthopnea. During
the inspection: acrocyanosis, heart rate-96 / min, tachycardia, weakening of the igo tone above
the crown, the third tone is heard. In the lungs in the basal parts of silent wheezing. During
echocardioscopy, left ventricular dilatation, thinning and dyskinesia of the inter-ventricular
septum, ejection fraction -39%. Identify a variant of myocardial dysfunction:
100% Left ventricular systolic dysfunction
0% Left ventricular diastolic dysfunction
0% Right ventricular diastolic dysfunction
0% Right ventricular systolic dysfunction
0% Mixed dysfunction of both ventricles

2358. a 7-year-old girl complains of pain in the vaginal area, significant purulent
discharge, which bothers for 5 days and gradually increases. On examination, the doctor found
significant swelling of the external genitalia, redness, purulent discharge from the vagina with
an unpleasant smell. An ultrasound examination (ultrasound) shows an
echo-positive shadow in the vaginal area. What reason can lead to this condition in children?
100% Presence of a foreign body in the vagina
0% Vaginal tumor
0% Cervical tumor
0% Purulent colpitis
0% Vulvovaginitis

544

Downloaded from the site - online testing step

2359. a 1.5-year-old child in the midst of an acute respiratory viral infection (ARV!)
I was eating walnuts. Suddenly she coughed and turned blue. Periodic dry cough with whistling on exhalation
continues. The child's condition is mild, respiratory failure. Shortening

https://translate.yandex.com/en/doc 523/540
22:20 ,27.6.2023 �� C : ; 5 B
of percussion sound, hard breathing, dry and wet wheezing, bronchospasm on exhalation over the right
lung.100%
What will Bronchoscopy
be the next tactic?:
0% Overview radiography of the abdomen
0% Chest X-ray survey
0% Ultrasound of the abdominal cavity and chest
0% Computed tomography of the chest

2360.A 43-year-old patient complained of heartburn,


acid belching, pain when swallowing food, and the inability to eat solid food. The patient
reported that he had been experiencing heartburn and acid belching for 8 years.
Deterioration of his condition has been observed for the last 3 months. What examination should
the patient undergo first?
100% Fibroesophagogastroduodenoscopy
0% Proton Pump Inhibitor Test
0% Daily esophago-pH monitoring
0% Multi-hour esophago-pH monitoring
0% Computed tomography of the chest organs

2361. A 5-year-old boy developed pain in his right


hip joint on day 5 after suffering from angina. Body temperature is up to 39oc. Leukocytosis-18.0 • 109/l, shift of the formula to the left.
Movement in the hip joint is painful and limited. The stomach is soft, not painful. X-rays
of the pelvic bones revealed a zone of destruction. A thorough examination revealed an infiltration in
the area of the right ilium, painful, tense. Make a preliminary diagnosis:
100% Acute hematogenous osteomyelitis of the right ilium
0% Malignant tumor of the right tumor bone
0% Acute appendicitis
0% Benign tumor of the right tumor bone
0% Toxic-allergic coxitis

2362. On the second day after arriving from India, a 29-year-old patient was admitted to the clinic with
complaints of abdominal pain, loose stools with vitreous mucus and blood in the form
of"raspberry jelly’."The general condition is satisfactory, the skin is pale, there is no rash. The abdomen is soft,
sensitive in the area of the cecum and ascending intestine. Choose an etiotropic drug for the treatment of this
pathology:
100% Metronidazole
0% Nifuroxazide
0% Erythromycin
0% Dexazone
0% Ceftriaxone

2363. A 43-year-old man without a permanent place of residence was hospitalized on day 5

545

Downloaded from the site - online testing step

diseases with complaints of fever up to 39.6 oC, general weakness,


headache. Objectively: excited, euphoric. There is a large number of lice on the patient's clothes.
The face is hyperemic, swollen. Isolated petechiae on the transitional fold of the conjunctiva, on the skin
of the rose-painful petechiae rash. The tongue trembles when protruding and deviates to the left.
Enlarged liver and spleen. What is the most likely diagnosis?
100% Typhus fever

https://translate.yandex.com/en/doc 524/540
22:20 ,27.6.2023 �� C : ; 5 B
0% Typhoid fever
0% Adenovirus infection
0% Infectious mononucleosis
0% Flu

2364. a 30-year-old patient is unconscious after a road traffic accident,


the skin is pale, and the pulse is threadlike. In the S / 3 of the right thigh there is a large laceration with profuse
long-term external arterial bleeding. A tourniquet was applied. The bleeding is stopped. What is the
maximum time that a tourniquet can be applied to a limb in the summer?
100% No more than 2 hours
0% No more than 5 hours
0% No more than 10 hours
0% No more than 2 minutes
0% No more than 10 seconds

2365. a 40-year-old woman underwent bilateral adnexectomy 5 years ago. Complains of


weakness, fatigue, lack of menstruation, lower back pain, memory loss, does not
remember recent events. During the examination, the presence of obesity, osteoporosis and
hypercholesterolemia. What syndrome does the patient have?
100% Post-castration syndrome
0% Andrenogenital syndrome
0% Adiposogenital dystrophy
0% Psychoneurotic syndrome
0% Menopausal syndrome

2366. an 18-year-old patient was admitted to the clinic complaining of severe shortness of breath, cough,
and itchy skin. I was acutely ill, about an hour ago after taking analgin. Since childhood-
urticaria,” aspirin asthma " objectively: paraorbital edema, redness and swelling
of the face. My breath is wheezing. Pulse rate-96 / min., blood pressure-80/50 mm Hg.
The emergency medicine will be:
100% Prednisone
0% Euphyllinum
0% Suprastin
0% Strophanthin
0% Dopamine

2367. A 25-year-old female patient complains of infertility and secondary amenorrhea. Objectively:
excessive nutrition, hirsutism phenomena. Bimanually: the uterus is of normal size, the ovaries on
both sides are slightly enlarged, not painful. LH and testosterone levels are elevated, ACTH test -

546

Downloaded from the site - online testing step

negative. Make a diagnosis:


100% Polycystic ovary syndrome
0% Adrenogenital syndrome
0% Bilateral chronic salpingoophoritis
0% Virilizing ovarian tumors
0% Resistant ovarian syndrome

https://translate.yandex.com/en/doc 525/540
22:20 ,27.6.2023 �� C : ; 5 B
2368.A 3-year-old child is prescribed Biseptol, paracetamol, and nasoferon due to acute respiratory viral infection
(ARVI). On the third day, the child's condition worsened:
there was a sore throat, stomatitis, conjunctivitis, hypersalivation, painful spots of dark red
color on the neck, face, chest and limbs, then blisters appeared in place of the spots.
There was a lesion of the mucous membranes around the mouth and anus. What is the preliminary diagnosis?
100% Stevens-Johnson syndrome
0% Atopic dermatitis
0% Chickenpox
0% Serum sickness
0% Bullous dermatitis

2369. an emotional 22-year-old medical university student was present during


the surgical operation, felt weak, dizzy, slightly nauseous, lost consciousness and
fell. This state lasted 2 minutes. Objectively: eyes are closed, skin is pale, diffuse
hyperhidrosis, pulse is weak, blood pressure is 90/60 mm Hg, breathing is shallow, reaction to
light is slow, tendon reflexes are preserved, convulsions are not present. What is the most likely
diagnosis?
100% Syncopal state
0% Hysterical neurosis
0% Vegetative-vascular paroxysm
0% Transient ischemic attack
0% An epileptic seizure

2370. a 19 - year-old patient has a penetrating wound to the left half of the chest cavity. The condition
is severe, cyanosis, shortness of breath. Objectively: on the left-breathing is sharply weakened. Heart - sounds are deaf,
pulse-102 / min., blood pressure -90 / 50 mm Hg. Echocardiography - detected echonegative
space surrounding the chamber of the left ventricle. A small amount of fluid in the left pleural
cavity and a large amount of air. Which pathological condition is most likely
to correspond to this picture?
100% Left-sided hydropneumothorax and hydro-rickard
0% Left-sided pneumothorax and hemopericardium
0% Right-sided pneumothorax and hemopericardium
0% Left-sided pneumothorax and exudative pericarditis
0% Dilated cardiomyopathy

2371. A 35-year-old female patient complains of pain and enlargement of the right breast. Suffers
from infertility for 15 years. The right breast is enlarged in size, its skin is pasty,
hyperemic, palpable infiltrate of a dough-like consistency without clear contours, a symptom

547

Downloaded from the site - online testing step

"lemon peel’.'Make a diagnosis:


100% Breast cancer
0% Mastitis
0% Nodular mastopathy
0% Breast abscess
0% Breast fibroadenoma

2372. A 15-year-old boy has been suffering from duodenal ulcer disease for the last 5 years
. The last exacerbation of the disease was in the spring. Suddenly there was " dagger pain in

https://translate.yandex.com/en/doc 526/540
22:20 ,27.6.2023 �� C : ; 5 B
the epigastrium, which was somewhat characterized in the knee-elbow position. The child's condition is severe, severe
pallor,
100%tachycardia, and symptoms
Perforation of peritoneal
of duodenum 12 irritation. What is the most likely diagnosis?

0% Acute appendicitis
0% Calculous cholecystitis
0% Acute intestinal obstruction
0% Acute pancreatitis

2373.A 27-year-old patient with bilateral hydrothorax underwent multiple pleural


punctures on both sides. After the next puncture-deterioration of the condition, fever, chest pain.
The next day, during a pleural puncture on the right, pus was obtained. The following diagnosis was made:
right-sided acute pleural empyema. What is the mechanism of occurrence of this complication?
100% Contact-aspiration pump
0% Lymphogenic
0% Hematogenic
0% Implantation
0% Air

2374. The boy is 14 years old. Within 3 years, there is an increase in blood pressure
to 170/100 mm Hg. treatment with antihypertensive drugs is ineffective. Holter hell:
stable arterial hypertension. Renal vascular doppler: partial visualization of the renal
arteries on the left. What will be the preliminary diagnosis?
100% Renovascular hypertension
0% Primary arterial hypertension
0% Pheochromocytoma
0% Adrenogenital syndrome
0% Hypothalamic syndrome

2375. in order to create safe working conditions for personnel in the treatment department of remote
radiation therapy, where a gamma therapy unit is used for therapeutic purposes
,it is necessary to apply the principle of shielding protection. What material should protective
screens be made of?
100% Lead, baritone concrete
0% Plastics, rubbers
0% Wood, brick

548

Downloaded from the site - online testing step

0% Aluminum, paraffin
0% Glass, organic glass

2376. a 16-year-old girl constantly vomits after eating in order to lose weight
. Mental status: asthenic, irritable, prone to hysterical reactions,
aggressive towards the mother. Objectively: thin, pale skin, increased body hair,
stomach ulcer, amenorrhea. Identify a mental disorder:
100% Mental anorexia
0% Depressive disorder
0% Phobic disorder
0% Dysphoric disorder
0% Hysterical disorder
https://translate.yandex.com/en/doc 527/540
22:20 ,27.6.2023 �� C : ; 5 B

2377. a 35-year-old patient complains about the presence of enlarged


peripheral lymph nodes for a long time, which do not bother him. From the anamnesis of the disease: first
, the lymph nodes of the neck, supraclavicular, and axillary regions increased, and new groups
of lymph nodes appeared. Objectively: the lymph nodes on palpation have a soft-elastic consistency, enlarged,
painless, not soldered to the surrounding tissues. What research method is the most
informative for early diagnosis of the disease?
100% Puncture biopsy
0% Magnetic resonance imaging
0% Radioisotope scanning of the skeleton
0% Ultrasound examination
0% X-ray examination

2378. A 23-year-old patient went to the clinic complaining of palpitations, sweating, weakness,
decreased performance, and trembling fingers. I lost 10 kg of weight in a month. Upon inspection:
heart rate-120 / min, blood pressure-130/70 mm Hg. during
ultrasound examination (ultrasound), a 2x2 cm node was found in the thyroid gland.
Scanning revealed an area of increased accumulation of radiopharmaceuticals at the site of the node.
RFP does not accumulate other thyroid tissues. What is the most likely diagnosis?
100% Toxic thyroid adenoma
0% Mixed toxic goiter
0% Autoimmune thyroiditis
0% Subacute thyroiditis
0% Thyrotropinoma

2379. a 22-year-old patient went to the doctor with complaints about the appearance of white spots on the background of tanned
skin. On the skin of the trunk there are multiple hypopigmented spots, when scratching
, a cystic peeling is observed. Balzer's test is positive. What disease can be assumed?
100% Pityriasis versicolor
0% Vitiligo
0% Scleroderma
0% Leucoderma
0% Pink lichen

549

Downloaded from the site - online testing step

2380. A 14-year-old boy complains of changes in the color of urine - " meat slop’.'From the medical history
it is known that 2 weeks ago I had tonsillopharyngitis. Objectively: pasty face, eyelids.
Blood pressure-135/90 mm Hg. the guy is active, does not lose consciousness. In the urine test:
specific gravity - 1025, red blood cells-cover the entire field of view, protein-0.165 g / l,
red blood cell cylinders. Make a preliminary diagnosis:
100% Acute post-streptococcal glomerulonephritis
0% Urolithiasis
0% Acute tubulointerstitial nephritis
0% Nephrotic syndrome
0% Acute pyelonephritis

2381. a 22-year-old woman in labor on the 4th day of the postpartum period complains of headache,
weakness, pain in the right breast, a sharp increase in body temperature to 39°C,
fever. In the area of the Upper-outer quadrant of the right breast

https://translate.yandex.com/en/doc 528/540
22:20 ,27.6.2023 �� C : ; 5 B
, a dense painful infiltrate is palpated, the skin over which is hyperemic. Blood pressure-120/70 mm
Hg, pulse-110 / min. The belly is soft, painless. What is the likely diagnosis?
100% Lactation mastitis, infiltrative form
0% Lactostasis
0% Phlegmonous mastitis
0% Gangrenous mastitis
0% Retromammary abscess

2382. a 26-year-old woman complains of infertility for 3 years. The man is healthy.
Examination revealed that the body of the uterus is of a dense consistency, of normal size. Applications are not
palpated. Metrosalpingography was performed to provide contrast fluid in the abdominal cavity on
both sides. The uterine cavity is deformed. It is assumed that the cause of female infertility is
an anomaly of uterine development. What examination should be performed to confirm the diagnosis?
100% Hysteroscopy
0% Advanced colposcopy
0% Probing the uterus
0% Ultrasound examination (US)
0% Scraping the walls of the uterine cavity

2383. among the general practitioner's assigned population, 30% are


elderly people. What features of the morbidity of this population should
a doctor take into account in order to organize medical care for them?
100% A large number of chronic diseases
0% Increase in the share of acute infectious diseases
0% Significant proportion of uncomplicated atypical diseases
0% Seasonal nature of morbidity and requests for help
0% A large number of diseases with a mild typical course

2384. in a 45-year-old patient with megaloblastic anemia, endoscopic examination with


a biopsy of the gastric mucosa revealed diffuse atrophic areas in
the fundal part of the stomach. What is the leading mechanism of development of this disease?

550

Downloaded from the site - online testing step

100% Autoimmune (antibodies to lining cells)


0% N. pylori infection
0% Radiation damage
0% Chemical damage
0% Use of NSAIDs

2385. a 63-year-old man was hospitalized 5 days ago for a myocardial infarction. Suddenly
, he complained of severe chest pain and lost consciousness. Pulse is not detected,
heart sounds are absent. ECG: sinus rhythm, QS and ST segment elevation in V1-Y4. Resuscitation measures
are ineffective. The puncture revealed blood in the pericardial cavity. Specify the most likely
complication:
100% Violation of the integrity of the left ventricular wall
0% Pulmonary embolism
0% Acute mitral insufficiency
0% Acute pericarditis
0% Violation of the integrity of the interventricular septum
https://translate.yandex.com/en/doc 529/540
22:20 ,27.6.2023 �� C : ; 5 B

2386. A 55-year-old patient complains of bowel movements 3-4 times during 1-2 hours, mainly
in the morning, after breakfast. Increased diarrhea is associated with increased demands from the environment,
excitement, and the expectation of danger. Objectively: fussy, multilingual, overweight.
The skin is unchanged, the turgor is normal, the stomach is soft and painless. There are no changes in the blood test;
fecal analysis: unformed, insignificant amount of starch grains and muscle fibers,
white blood cells-3-4 v / s; colonoscopy without pathology. For the purpose of treatment, you should prescribe:
100% Loperamide
0% Drotaverine
0% Anaprilin
0% Ampicillin
0% Bisacodyl

2387. A 25-year-old female patient complains of several tight, painful nodes in the right
armpit area, an increase in body temperature up to 38°C, and general weakness. He has been ill for 3 days.
Locally: in the right axillary region there are three nodular formations that rise above
the surface of the skin, ranging in size from 0.5 cm to 1.5 cm in diameter, sharply painful on palpation, with
clear contours, the skin above them is purple-cyanotic, the elevation of the upper limb
is limited due to pain. Make a diagnosis:
100% Purulent hydradenitis
0% Abscess
0% Carbuncle
0% A boil
0% Purulent lymphadenitis

2388. in a 48-year-old patient who complains of heavy and prolonged menstruation, the doctor
assumes endometrial hyperplasia based on ultrasound data. What
research methods can be most efficiently applied to clarify the diagnosis?

551

Downloaded from the site - online testing step

Hysteroscopy, fractional curettage of the uterus followed by pathohistological


100%
examination
0% Colposcopy
0% Hysterosalpingography
0% Culdoscopy
0% Colpocytology

2389. during the medical examination of children of the children's home No. 5, they were found to have reduced
working capacity, rapid fatigue, apathy, and drowsiness. 10 % of the examined children
have bleeding gums, subcutaneous spot hemorrhages. What vitamin deficiency
is found in the examined children?
100% Vitamin C
0% Vitamin E
0% Vitamin D
0% Vitamin H
0% Vitamin B1

https://translate.yandex.com/en/doc 530/540
22:20 ,27.6.2023 �� C : ; 5 B
2390. a child of 11 years of age has periodic constipation from birth, bloating,
decreased appetite, sometimes nausea, and muscle hypotrophy. On the irrigogram, there is a narrowing in
the rectum with Supra-stenotic expansion of the colon above this place.
In anomanometry, the anorectal reflex is negative. Which diagnostic method will be
most informative to confirm the diagnosis?
100% Full-layer biopsy of the rectal and colon wall
0% Ultrasound examination (US)
0% Overview radiography of the abdominal cavity
0% Biochemical blood testing
0% Thermography of the anterior abdominal wall

2391. A 54-year-old patient complained to a neurologist of hand tremor, headache,


metallic taste in the mouth, insomnia, and increased salivation. From the medical history, it is known that he
has been working at a fluorescent lamp factory for more than 10 years. What will be your diagnosis?
100% Chronic mercury intoxication
0% Chronic lead intoxication
0% Chronic nitrocrass intoxication
0% Manganese intoxication
0% Benzene intoxication

2392. a 52-year-old patient who went to a gastroenterologist for a stomach ulcer


was found to have dysplastic obesity, striae on the abdomen and thighs. Blood pressure -170 / 100 mm
Hg during a laboratory examination, an increased level of cortisol was detected
, the level of ACTH was moderately reduced.:
100% Itsenko-Cushing Syndrome
0% Itsenko-Cushing's disease
0% Alimentary obesity
0% Hypothalamic syndrome with endocrine-metabolic disorders

552

Downloaded from the site - online testing step

0% Solinger-Ellison syndrome

2393. a 23-year-old woman in labor, the delivery is urgent and normal. 2 days after delivery
, the general condition was found to be satisfactory. Pulse-80 / min., blood pressure-120/70 mm Hg.
The uterus is dense, located 3 fingers below the navel, painless. Discharge from the genital tract
is moderate, bloody. What is the most likely diagnosis?
100% Physiological postpartum period
0% Subinvolution of the uterus
0% Hematometer
0% Postpartum metroendometritis
0% Lochiometer

2394. A 23-year-old patient was delivered urgently with complaints of pain in the lower abdomen, intense pain on the
right side, radiating into the rectum, and dizziness. The aforementioned complaints appeared suddenly
at night. Last menstruation 2 weeks ago. Objectively: the skin is pale, pulse-92 / min.,
body temperature-36.6°C, blood pressure-100/60 mm Hg. the abdomen is somewhat tense,
slightly painful in the lower parts, the symptoms of irritation of the peritoneum are slightly positive.
Hemoglobin-98 g / l. what will be the preliminary diagnosis?
100% Ovarian apoplexy
0% Disturbed ectopic pregnancy

https://translate.yandex.com/en/doc 531/540
22:20 ,27.6.2023 �� C : ; 5 B
0% Acute appendicitis
0% Intestinal obstruction
0% Renal colic

2395. a child born at 39 weeks of gestation


developed respiratory disorders after resuscitation, the severity of which increased over time. The amniotic fluid
was contaminated with meconium. The respiratory rate is -90 / min,significant retractions,
moaning on exhalation, movements of the wings of the nose, paradoxical breathing attract attention. Please indicate the most likely cause
of respiratory diseases in this child:
100% Meconium aspiration syndrome
0% Congenital pneumonia
0% Transient tachypnea of the newborn
0% Respiratory distress syndrome
0% Congenital heart disease

2396. a woman in labor went to the doctor on the third day of the postpartum period with complaints of
pain in the right breast, difficulty expressing milk, and fever up
to 37.2°C. Examination revealed: local, moderately painful, hard formation in the area of the right
breast, the skin above it is not changed. What is the most likely diagnosis?
100% Lactostasis
0% Phlegmonous mastitis
0% Purulent mastitis
0% Serous mastitis
0% Gangrenous mastitis
2397. a 52-year-old patient has unstable (first-time) coronary heart disease (CHD)

553

Downloaded from the site - online testing step

angina pectoris. Stenosing coronary arteriosclerosis, HF and art. hypertension II art., art. 3, risk
4. Stenting of the right coronary artery was performed. The cardiologist recommended therapy
that includes a beta-blocker, a double antiplatelet combination (pla-Vix and cardiomagnil),
and lipid-lowering agents (statins). Please indicate the laboratory parameters that you will monitor
for safe long-term statin use.:
100% Transaminases
0% Daily proteinuria
0% Triglycerides, beta-lipoproteins in blood serum
0% Blood glucose level
0% Serum Alpha-amylase

2398. during resuscitation, a 77-year-old patient


cannot perform chest compressions to a depth of 5-6 cm due to chest rigidity. What
will be your actions?
100% Increase the force of pressure despite the risk of broken ribs
0% Continue pressing with preliminary force
0% Reduce the force of pressing to prevent fractures of the ribs and sternum
0% Shift the pressure point towards the xiphoid process
0% Increase the number of clicks to 150 times per minute

2399. a 36-year-old patient suffered a severe craniocerebral injury as a result of a traffic accident
and is in a comatose state. For diagnostic purposes

https://translate.yandex.com/en/doc 532/540
22:20 ,27.6.2023 �� C : ; 5 B
, echoencephalography was performed, and a shift of the median brain structures to the right by 12 mm was found.
Computed tomography revealed a subdural hematoma
measuring 6x7x7 cm in the frontoparietal region on the left. What is the most correct treatment strategy?:
100% Surgical treatment
0% Application of oxybar therapy
0% Monitoring of the patient in dynamics
0% Conducting detoxification therapy
0% Performing craniocerebral hypothermia

2400. During the study of the corpse of a 50-year-old citizen, it was revealed: high on the neck, above the level
of the thyroid cartilage, there is a dark brown strangulation furrow, which
goes from bottom to top and from right to left. The furrow is absent on the posterior surface of the neck, but is best
expressed on the anterior surface, where the depth is 0.2 cm and the width is 1.5 cm. Cadaveric spots
are intense, purple in color, located on the lower extremities. What type of mechanical asphyxia
occurred?
100% Hanging
0% Noose choking
0% Hand strangulation
0% Closing the mouth and nose openings
0% Compression of the chest and abdomen

2401. administration of glucocorticosteroid hormones for the purpose of preoperative preparation


for diffuse toxic goiter is necessary for:

554

Downloaded from the site - online testing step

100% Prevention of thyrotoxic crisis


0% Prevention of anaphylactic shock
0% Prevention of acute kidney failure
0% Prevention of hypothyroidism
0% Prevention of hypoparathyroidism

2402. the human body receives several chemicals from the atmospheric air. What
is the name of the type of combined action in which its compatible effect exceeds the sum of the effects of each
substance included in the combination, with their isolated effect on the body?
100% Potentiation
0% Antagonism
0% Isolated action
0% Combined action
0% Complex action

2403. a 67-year-old patient suddenly developed acute respiratory failure on the 6th day after elective surgery
. Heart rate-130 / min. Upper body, neck and
face cyanotic, cough, sharp pain behind the sternum. Blood pressure -90/60 mm Hg,
central venous pressure -330 mm Hg on the ECG inversion of the T wave what complication
did the patient develop?
100% Pulmonary embolism
0% Repeated bleeding
0% Acute coronary artery insufficiency
0% Acute myocardial infarction
https://translate.yandex.com/en/doc 533/540
22:20 ,27.6.2023 �� C : ; 5 B
0% Hypovolemic shock

2404. A 20-year-old patient experienced a sudden


increase in headache, nausea, vomiting, and a feeling of tightness and weakness in the left arm on the day after the closed head injury
. The neurological status showed Kernig's symptom, dilation of the right pupil and
central paresis of the left hand. Make a preliminary clinical diagnosis:
100% Subdural hematoma
0% Concussion of the brain
0% Brain contusion
0% Secondary meningitis
0% Brain abscess

2405. A 35-year-old female patient went to the doctor complaining of a neoplasm on the skin
of her right shin. Objectively: the nevus is irregular in shape with a diameter of 1 cm, without clear borders, with a dry
surface, dark brown in color. The diagnosis was established: melanoma of the skin of the right
shin is likely. What is the method for verifying the diagnosis of skin melanoma?
100% Excisional biopsy
0% Smear impression
0% Precision biopsy
0% Positron emission tomography-computed tomography (PET-CT)

555

Downloaded from the site - online testing step

0% Ultrasound examination (US)

2406. a 38-year-old patient, a bus driver, suffered a severe spinal cord injury during working hours as a result of an accident
(there is a certificate of an industrial accident). As a result, he was recognized
as a disabled person of the first group. Determine the cause of disability:
100% Occupational injury
0% Common disease
0% Occupational disease
0% Severity of injury
0% Traffic accident

2407.the first finger of the right hand was wounded by a circular saw. Medical
assistance was provided in the emergency room. Data of the outpatient patient: "the condition is satisfactory, blood
pressure-90/60 mm Hg, pulse-100 / min. The first finger of the right hand is separated from the main
phalanx and is held by a skin flap’.'At a forensic examiner's appointment:
" the first finger of the right hand is missing at the level of the main phalanx. Cult formed "'
determine the severity of the injury:
This bodily injury is considered to be of moderate severity according to the criterion
100%
of percentage disability up to 30%
This bodily injury is considered to be of moderate severity according to the criterion
0%
of the duration of the health disorder
This injury is classified as a minor injury
0%
resulting in a short-term health disorder
This personal injury is classified as serious bodily injury according to the criterion
0%
of danger to life
This bodily injury is classified as a serious bodily injury based on the criterion
0%
of a percentage disability of more than 30%

https://translate.yandex.com/en/doc 534/540
22:20 ,27.6.2023 �� C : ; 5 B

2408. A 57-year-old patient is undergoing treatment for polytrauma. After 8 days, there
were complaints of severe cough with purulent sputum, severe shortness of breath,
fever up to 38.8°C. History: chronic bronchitis, smokes for 20 years.
Objectively: hard breathing is heard in the lungs, moist fine - bubbled wheezes are heard in the lower parts of the right lung
. The HIV test is negative. What is the diagnosis of this patient?
100% Nosocomial pneumonia
0% Extrahospital pneumonia
0% Aspiration pneumonia
0% Pneumonia of people with immunodeficiency
0% Infectious exacerbation of bronchitis

2409. at night, the patient developed: headache, pain in the right eye radiating to the right temple,
nausea, vomiting, decreased vision in the right eye, fog, iridescent circles around the light source.
Objectively: the right eye is a congestive injection of scleral vessels, the cornea is swollen,its sensitivity is reduced,
the anterior chamber is shallow, the pupil is wide. The reflex from the fundus is dull pink, the details are not
ophthalmic-they are triggered due to corneal edema. Intraocular pressure - 58 mmHg.
make a diagnosis:
100% Acute glaucoma attack

556

Downloaded from the site - online testing step

0% Open-angle glaucoma
0% Iridocyclite
0% Bacterial conjunctivitis
0% Swollen cataracts

2410. a mother with a 4-month-old child went to the pediatrician's office with complaints of
fever up to 37.5°C. The child was born full-term, 8-9 points on the Apgar scale.
When analyzing medical records, it was found that the child does not have a vaccination against tuberculosis.
When should the TB vaccination have been carried out?
100% Full-term baby, 3-5 days of life
0% Full-term baby, no later than 48 hours from birth
0% During the first week of life
0% During the first 24 hours of life
0% During your stay in the hospital after a preliminary Mantoux test

2411. The chief physician of the district hospital issued an order on the procedure for conducting mandatory
routine preventive vaccinations. Which population groups are subject to mandatory
routine vaccinations?
100% Children, adolescents, and adults who have reached the appropriate age
0% Individuals from certain occupational risk groups (gamekeepers, hunters, etc.)
0% Persons who have had contact with the source of infection in the epidemic center
0% Children who receive documents for kindergarten or school
0% Persons who may have been infected

2412. A 65-year-old woman has chronic obstructive pulmonary disease with exacerbations
of the disease at least twice a year. The last exacerbation with fever and purulent
sputum was treated 3 weeks ago. What is the patient's management strategy regarding secondary
immunoprophylaxis of exacerbations and progression of respiratory failure?
https://translate.yandex.com/en/doc 535/540
22:20 ,27.6.2023 �� C : ; 5 B
100% Vaccination against pneumococcus and influenza
0% Vaccination against type B hemophilic infection
0% Administration of homologous immunoglobulin
0% BCG vaccination after a negative Mantoux test result
0% All vaccinations are contraindicated

2413. Pervovagitnaya 25-year-old woman went to the department of pathology of pregnant women in January at the age of 27
weeks with signs of severe acute respiratory infection.
The diagnosis of "flu" was established and laboratory confirmed. The pregnant woman's condition gradually worsened, the patient was
transferred to the
intensive care unit, and artificial ventilation was started. Despite
treatment, antenatal fetal death occurred. Which of the following approaches is
most100%
effective inFlu
preventing thisfor
vaccination pregnancy complication?
all pregnant women and women planning pregnancy
0% Taking antiviral medications during a flu epidemic
0% Use of immunomodulators
Use of de-escalation antibiotic therapy immediately after the first
0%
signs of acute respiratory viral infection (ARV!)appear

557

Downloaded from the site - online testing step

0% Refuse to visit any gatherings of people during the epidemic

2414. A 4-year-old child is healthy and unvaccinated against measles. In the region where it lives
, an outbreak of measles has been recorded. The doctor prescribed the first dose of the KPC vaccine (against measles, mumps and
rubella). When should this child receive a second dose of the vaccine?
100% After 1 month
0% After 3 months
0% At 6 years of life
0% At the age of 14
0% After 6 months

2415. a 14-year-old student complained to the family doctor about scratches on


her face caused by a neighbor's cat 2 days ago. On examination, the scratches are superficial, covered with scabs.
Primary surgical treatment of wounds was performed. It is known that the child received all planned vaccinations
according to the calendar. What will be the next steps?
100% Animal observation
0% Introduction of rabies vaccine
0% Introduction of rabies immunoglobulin
0% Introduction of rabies vaccine + rabies immunoglobulin
0% Introduction of rabies vaccine + tetanus toxoid

2416. A 33-year-old pregnant woman at 1617 weeks gestation was bitten on the neck by a dog.
The patient was taken by an ambulance team to the emergency room. General condition of moderate
severity. Blood pressure -100 / 60 mm Hg, pulse-92 / min. On examination, a wound in the neck with
ragged edges and moderate bleeding. What will be the doctor's tactics?
Surgical treatment of the wound, administration of tetanus toxoid and
100%
rabies vaccine
Surgical treatment of the wound, observation of the dog, decision on
0%
rabies prevention in 10 days
0% Stopping bleeding, deciding on vaccination for signs of rabies in an animal

https://translate.yandex.com/en/doc 536/540
22:20 ,27.6.2023 �� C : ; 5 B
0% Termination of pregnancy for medical reasons after wound treatment
0% Stopping bleeding, surgical treatment of the wound

2417. a girl was born from the first pregnancy, the first vaginal delivery at the gestation period of 39 weeks
(weight 3350 g, body length 53 cm). The 17-year-old mother of the child is generally positive. The child's condition after
birth is satisfactory. When can this child be vaccinated with BCG?
100% After clarifying the child's HSV status
0% Do not vaccinate in any case
0% On the 1st day
0% On the 3rd day
0% After the Mantoux test

2418. a newborn premature baby was born with a mass of 1900 g. he was transferred to the Department
of neonatal Pathology for further treatment with a diagnosis of "perinatal hypoxic
encephalopathy" BCG vaccination was not carried out in the maternity hospital. When should I get vaccinated

558

Downloaded from the site - online testing step

this child against tuberculosis?


100% Upon reaching a body weight of 2500 g
0% After elimination of signs of neurological pathology
0% At the age of 6 months
0% In 1 year of life
0% Do not vaccinate your child at all

2419. A 17-year-old boy came to the surgery complaining of a cut


wound in his right foot. From the medical history, it is known that he was injured with an axe while working on
the farm. I received all vaccinations according to my age. Objectively: body temperature-36.8 oC,
pulse-68 / min., blood pressure-120/75 mm Hg. what are the doctor's actions?
100% Dynamic monitoring
0% Enter tetanus toxoid
0% Enter tetanus serum
0% Enter tetanus toxoid and tetanus immunoglobulin
0% Enter tetanus toxoid with diphtheria toxoid

2420. A 30-year-old female patient has been suffering from systemic lupus erythematosus (SLE) for 5 years and
takes 16 mg of methylprednisolone daily. Which statement about flu vaccination is correct in
this case?
100% Recommended annual routine vaccination
0% Vaccination after reducing the dose of prednisone to the minimum
0% Vaccination is contraindicated in patients receiving glucocorticoids
0% Vaccination is carried out under the cover of anti-allergic drugs
0% Any vaccination is contraindicated

2421. A 67-year-old patient with chronic lymphocytic leukemia receives a course of chemotherapy. What
are the recommendations for flu vaccination with a sufficient number of granulocytes and
lymphocytes in the peripheral blood?
100% Not earlier than 4 weeks after chemotherapy
0% You can do it during chemotherapy
0% Not earlier than 1 week after chemotherapy

https://translate.yandex.com/en/doc 537/540
22:20 ,27.6.2023 �� C : ; 5 B
0% 1 year after chemotherapy
0% Generally contraindicated

2422. A 23-year-old woman, 12 weeks pregnant, who had a history of surgery with
tetrad Fallot at the age of 1 year, received an annual flu vaccination, the last one was 1 year
ago. What recommendations should your family doctor make for flu vaccination this year?
100% Carry out vaccination with an inactivated vaccine
0% Perform recombinant vaccination
0% Don't get vaccinated this year
0% Postpone vaccination until the third trimester
0% Postpartum vaccination

559

Downloaded from the site - online testing step

2423. A 26-year-old dentist was diagnosed with moderate hyperenzymemia


and tested positive for HBsAg. His 24-year-old wife is not vaccinated against hepatitis B
virus. The 2-year-old child is properly vaccinated according to the age schedule. What prevention of viral
hepatitis B should be carried out in the family?
100% Immediate vaccination of the wife with a negative HBsAg test
Prescribing specific chemoprophylaxis with non-nucleoside
0%
analogues to the wife and child
0% Prescribing specific immunoglobulin to your wife and child
Prescribing specific chemo-prophylaxis with non-nucleoside
0%
analogues to women
0% Prescribing specific immunoglobulin to a child

2424. a 56-year-old man complains of rapid fatigue, loss of appetite during


the last month, which is associated with overloading at work. Has no permanent family,
reports questionable sexual relations. In his youth, he was a donor for several years.
Examined by a family doctor. RNA for hepatitis C virus was detected by PCR, the degree
of fibrosis according to the METAVIR scale according to fibroscan - F3. What vaccinations should the doctor
suggest to the patient?
100% Vaccinations against pneumococcus, viral hepatitis A, B and annually against influenza
0% Vaccination against pneumococcus, hepatitis B virus and annually against influenza
0% Flu shot every year
0% Vaccinations against viral hepatitis A and B
0% All vaccinations are contraindicated

2425. A 6-month-old child was born to an HIV-infected mother. The child's HIV status has not
been determined. On examination: the condition is satisfactory. Physical and neuropsychic development is
age-appropriate. Vaccination was carried out at 2 months and 4 months against diphtheria, tetanus, pertussis,
polio. According to the schedule of preventive vaccinations in Ukraine, the doctor ordered
the continuation of vaccination against these infections. What vaccine is contraindicated for this child?
100% Oral (live) polio vaccine
0% Inactivated polio vaccine
0% AKDP-vaccine
0% Vaccination is not carried out until the HIV status is determined
0% -

https://translate.yandex.com/en/doc 538/540
22:20 ,27.6.2023 �� C : ; 5 B
2426. a 15-year-old girl contracted measles. It is known that she suffers from bronchial asthma with
a high level of total IgE and iron deficiency anemia: the HB level is 110 g / l,
the serum iron content is 6.6 mmol/l (the normal indicator is 10.6-33.6 mmol/l). Vaccinated
with the measles, rubella, and mumps vaccine once at the age of 1 year. What factor caused
the child's illness?
100% Incomplete vaccination
0% Atopy
0% Anemia
0% Puberty age
0% Sideropenia
2427. a 26-year-old man who "lives with HIV" TMJ " for which he receives anti-retroviral therapy

560

Downloaded from the site - online testing step

I went to the doctor for a routine vaccination. The patient is vaccinated according
to the vaccination calendar. The doctor ordered a vaccination against diphtheria and tetanus. What class
do these vaccines belong to?
100% Anatoxins
0% Corpuscular living cells
0% Chemical properties
0% Corpuscular kills (inactivated)
0% Recombinant

2428. A 58-year-old woman with sensorineural deafness due to complications after


purulent meningitis is planning to install a cochlear implant.
What infection should be vaccinated against to prevent infectious diseases of the central
nervous system and hearing organs?
100% Pneumococcal infection
0% Tick-borne encephalitis virus
0% Tuberculosis after a negative Mantoux test result
0% Hemophilic infection
0% Poliomyelitis

https://translate.yandex.com/en/doc 539/540
22:20 ,27.6.2023 �� C : ; 5 B

561

https://translate.yandex.com/en/doc 540/540

You might also like